Sie sind auf Seite 1von 338

ll«ns to

Dlfliltt"
qM/; IMIIi!
1000 Questions to Help
You Pass the Emergency
Medicine Boards
1000 Questions to Help
You Pass the Emergency
Medicine Boards

Amer Z. Aldeen, MD
Clinical Instructor
Department of Emergency Medicine
Northwestern University Feinberg School of Medicine
Chicago, illinois

David H. Rosenbaum, MD
Attending Physician
Department of Emergency Medicine
Ingalls Memorial Hospital
Harvey, Illinois

I
@Wolters Kluwer Lippincott Williams & Wilkins
Health
Philadelphia • Baltimore • New York • London
Buenos Aires • Hong Kong • Sydney • Tokyo
Acquisitions Editor: Frances R. DeStefano
Developmental Editor. Louise Bierig
Managing Editor. Nicole Dernoski
Project Manager: Nicole Walz
Manufacturing Manager: Kathy Brown
Marketing Manager. Angela Panetta
Design Coordinator. Holly Reid McLaughlin
Cover Designer: Carol Tippit Woolworth
Production Services: Laserwords Private Limited, Chennai, India
C 2008 byUPPINCOITWILUAMS & WILKINS, a Wolters Kluwer business

530 Walnut Street


Philadelplrla, PA 19106USA
LWW.com

All rights reserved. This book is protected by copyright. No part of this book may be
reproduced in any form or by any means, including photocopying. or utilized by any
information storage and retrieval system without written permission from the copyright
owner, except for brief quotations embodied in critical articles and reviews. Materials
appearing in this book prepared by individuals as part of their official duties as U.S.
government employees are not covered by the above-mentioned copyright.

Printed in the USA

library of Congress Cataloging-in-Publication Data


Aldeen, Amer Z.
1000 questions to help you pass the emergency medicine boards I Am.er Z. Aldeen,
David H. Rosenbaum.
p. ;em.
Includes bibliographical references and index.
ISBN-13: 978-0-7817-7718-6
1. Emergency medicine-Examinations, questions, etc. I. Rosenbaum, David H. II. Title.
ITI. Title: One thousand questions to help you pass the emergency medicine boards.
[DNLM: 1. Emergency Trea'bnent-methods-Examination Questions. 2. Emergency
Medicine-methods-Examination Questions. WB 18.2 A357z 2008]
RC86.9.A43 2008
616.0215----dc22
2007024885

Care has been taken to confirm the accuracy of the information presented and to
describe generally accepted practices. However, the authors, editors, and publisher are not
responsible for errors or omissions or for any consequences from application of the
information in this book and make no warranty, expressed or implied, with respect to the
currency, completeness, or accuracy of the contents of the publication. Application of this
information in a particular situation remains the professional responsibility of the
practitioner.
The authors, editors, and publisher have exerted every effort to ensure that drug
selection and dosage set forth in this text are in accordance with current recommendations
and practice at the time of publication. However, in view of ongoing research, changes in
government regulations, and the constant flow of information relating to drug therapy and
drug reactions, the reader is urged to check the package insert for each drug for any change
in indications and dosage and for added warnings and precautions. This is particularly
important when the recommended agent is a new or infrequently employed drug.
Some drugs and medical devices presented in this publication have Food and Drug
Administration (FDA) clearance for limited use in restricted research settings. It is the
responsibility of the health care provider to ascertain the FDA status of each drug or device
planned for use in their clinical practice.
To purchase additional copies of this book, call our customer service deparbnent at
(800) 638-3030 or &x orders to (301) 223-2320. International customers should call
(301) 223-2300.
VISit Lippincott Williams & Wilkins on the Internet: at LWW.com. Lippincott
Williams & Wilkins customer service representatives are available from 8:30 am to 6 pm,
EST.
10 9 8 7 6 5 4 3 2 1
I dedicate this book to my parents. To my father, Osman Ziauddin, for always showing me that true
excellence lies in simplicity. To my mother, Nadira Ziauddin, for being the greatest teacher I have ever
known and for passing on her love of teaching to me. I love you both far more than I can ever show. If
I am even a fraction of the parent you both have been to me, I will consider my life a true success.
Amer Z. Aldeen, MD

For my beautifUl wife, Mindy and our daughter Sophie, who give purpose and meaning to everything
I do. To my mother and father, Rachel and Peter Rosenbaum, for a lifetime of sacrifice, love, and
encouragement. You have been my greatest advocates in all of my endeavors.
David H. Rosenbaum, MD
Contents
preface 0 0 0 0 0 0 0 0 0 0 0 0 0 0 0 0 0 0 0 0 0 0 0 0 0 0 0 0 0 0 0 0 0 0 0 0 0 0 0 0 0 0 0 0 0 0 0 0 0 0 0 0 0 0 0 0 0 0 0 0 0 0 0 0 0 0 0 0 0 0 0 0 0 0 0 0 0 0 0 0 0 0 0 0 0 0 0 0 0 0 ix

Acknowledgm.ents . . . . . . . . . . . . . . . . . . . . . . . . . . . . . . . . . . . . . . . . . . . . . . . . . . . . . . . . . . . . . . . . . . . . . . . . . . . . . . . xi

Test 1. . ..................... . .................................. . . . . . . .. . .. . .. . . . .. . .. . .. . . . .. .. 1

Test 2.................. . ..................... . ................ . ....... . ....... . .. . .......... . .. 31

Test 3 ........................................ . ................ . ....... . ..................... . .. 61

Test 4 ......................................................... . ............................. . .. 94

Test 5 .................. . .... . ................ . ................ . ................................ 126

Test 6 ....................... . ..................... . ............ . . .. . . .. .. . . .. . . . .. . .. . .. . . . .. . . 158

Test 7 ............................................................. . ....... . .. . ....... . .. . ...... 188

Test 8 ............................................................. . .......... . ....... . .. . ...... 220

Test 9 ....................... . .................................. . . . . . . .. . .. . .. . . . .. . .. . . . . . . .. . . 249

Test tO .................. . .... . ................ . .... . ......... ...... . . . . .. . .... . .. . .... . ... . . . ... 280

Index .......................................... . ......... . ...... . .... . .. . . . ........ . ....... . .. . .. 311

vii
Preface

We created 1000 Questions to Help You Pass the Emergency are largely "case-based," and involve clinically relevant
Medicine Boards because we felt there was a dearth of patient scenarios. Unfortunately, medical students and
adequate preparatory material for the American Board residents have become accustomed to the inefficient
of Emergency Medicine's In-training and Qualifying practice of digesting endless lists of facts as a means
examinations. Excellence in emergency medicine (EM) of study. In contrast, we believe that this patient-focused
requires both the development of clinical skills and the approach enhances knowledge acquisition and will serve
acquisition ofa broad foundation ofknowledge. Although as an invaluable resource for examination preparation.
residency training prepares one very well for the practical Teachers can never stop being learners. In that spirit,
aspects of the profession, only a spirit of intellectual we hope that you will bring to our attention any errors
curiosity combined with a personal commitment to in medical information that you encounter while going
continuous study can improve one's knowledge base. through the tests. We apologize in advance for any such
This book is intended for medical students, EM errors and will endeavor to correct them in future editions
residents, and EM physicians in practice. The questions ofthebook.
that follow are designed to mimic the style of questions on
Amer Z. Aldeen
the American Board ofEmergency Medicine's In-training
and Qualifying Examinations. Therefore, the questions David H. RDsenbaum

ix
Acknowledgments
I would like to thank the many great teaching physicians not for you, this book would have remained just another
in the Department of Emergency Medicine at the good idea. You have always inspired me to take on new
Northwestern University Feinberg School of Medicine challenges, and my life has been the better for it.
who trained me in this wonderful specialty. I especially
thank my chainnan, James Adams MD, for identifying
Amer Z. Aldeen, MD
the challenges inherent in a project ofthis magnitude and I would like to acknowledge the fantastic attending physi-
providing suggestions for overcoming them. clans in the Department of Emergency Medicine at Cook
I would also like to thank the following resident County Hospital who have provided me with an essential
physicians in the Northwestern University Emergency framework upon which to build my knowledge of the
Medicine Residency Program who helped me with specialty. Special thanks are also due to Dr. Jeff Schaider,
edits: Jeremy Branzetti MD, Patrick Coleman MD, who was instrumental in turning this idea into a reality.
Ambrose Insua MD, Patrick Lank MD, Matt Lazio MD, Finally, I am indebted to my colleagues, Jon BankoffMD,
Andrew Louie MD, Danielle McCarthy MD, and Daniel and Andrew Osugi MD, for their thorough and thoughtful
ReavenMD. commentary that helped to improve this book.
Lastly, I would like to thank my wife, Dr. Kulsoom
Ghias, for encouraging me to start this project. If it were David H. Rosenbaum, MD

xi
Test 1
Questions
[II A 34-year-old man presents with chest pain of (A) No acute therapy
4 hours' duration, resolved at presentation. He has (B) Atropine l mg IV
been smoking marijuana, which he obtained from (C) Amiodarone 150mgiV
an unknown source. His vital signs are 98.6°F, 115, (D) Transcutaneous pacing
20, 167/95,98% RA. His physical examination shows (E) Synchronized cardioversion at 50 J
pupillary dilation, tachycardia. and diaphoretic skin.
His electrocardiogram (EKG) shows ST depressions [i] Which of the following opioids may predispose to
in V3-VS. Initial troponin level is 0.20 ng per mL. QRS prolongation?
Which of the following is the most appropriate (A) Fentanyl
therapy at this time? (B) Meperidine
(A) Percutaneous transluminal coronary (C) Propoxyphene
angioplasty (PTCA) (D) Hydrocodone
(B) Tissue plasminogen actiw.tor(tPA) (E) Morphine
(C) Abciximab
(D) Metoprolol IJ] A 25-year-old woman presents to the emergency
(E) Aspirin department (ED) with worsening left-sided pelvic
pain and vaginal disc:barge. She states that she was
[i] A 29-year-old man is sent by the primary medical seen in the office by her primary care doctor 3 days
doctor's office for evaluation of his abnormal EKG. ago for similar problems and received an injedion
He is asymptomatic and his physical examination is and was given a prescription that she has not had
normal The EKG is shown in Figure 1-1. Which of a chance to fill. On the phone, the office manager
the following is the most appropriate next step in tells you the patient received an 1M injection of
management? ceftriaxone. In the ED, she has a fever, purulent

1 ;-rv-r~r~.r- r~~~_,,..--.._.._r-v- 1 ;r---r


:1 II
1.~~~~--L---,..J.;'-.-.JA.~-.J~--~__jr A-J.~_Jr
11t_J_. _J~'- ~·----k-l-A\..--4-l~J~~-~~~~
Vi
Plgure 1-1. (See color insert.)

1
2 1000 Qut:stions fD Help You Ptm the Emerfenc.y MiltliciM Bolris

vaginal discharge, cervical motion tenderness, and a (!] A 65-year-old man presents with complaint ofdouble
palpable, tender left-sided adnaal. mass. Which of vision. On examination. covering either eye causes
the foll.owins is true? the diplopia to raolve. Which of the following is the
(A) She has Fitz-Hugh-Curtis syndrome. most likely cause?
(B) Aspiration and culture of the uws is likely to (A) Multiple sclerosis
reveal Neis:serUI g<motriweae. (B) Thyroid disease
(C) If untreated, rupture of the mass and secondary (C) Isolated cranial nerve palsy
peritonitis may occur. (D) Myasthenia gravis
(D) The patient should be kept NPO for emergent (E) Orbital absceas
surgery.
(E) The patient most likely has a hemorrhagic (I) Which of the following is a negative symptom of
ovarian cyst. schizophrenia?
(A) Delusions
(!] A 65-year-old woman presents with 2 hours ofacute (B) Hallucinations
abdominal pain. The pain is severe, diffuse, and
(C) Flat affi:ct
progressively wonening. Blood pressure is 150/90, (D) Disorganized speech
and abdominal e.umination demonstrates a palpable
(E) Disorganized behavior
pulaatile mass. Abdominal computed tomography
(Cf) scan is shown in Figure 1-2. Which of the
following is true regarding management of this
(!) Which of the following i& the moat common cause of
dilated cardiomyopathy?
patient?
(A) Idiopathic
(B) Systemic lupus erythematosus (SI.E)
(C) Tuberculosis (TB)
(D) Cocaine
(E) Familial

(!] A 42-year-old man presents with pain, warmth, and


swdling over his posterior elbow (see Fig. 1-3). The
patient repom frequen.tfy having to lean on his elbow
while performing electrical work aa part of his job.
Although he is able to flex and enend the joint,
flexion results in increased pain. After aspiration
and cell count of the fluid obtained, which of the
following is the minimum WBC count suggestive of
inD:ction?

figure 1-2.

(A) mood pressure should be reduced to systolic


100 mm Hg or bdow.
(B) The patient should be crossmatched for 10
units of packed red blood c:ells.
(C) Wtrasound may help better characterize the
anatomy.
Figure 1-3.
(D) Angiogram may reduce the need for urgent
surgery.
(E) Hemodynamically stable patients with
ruptured abdominal aortic aneurysms (AAAs) (A) White blood cell (WBC) >500 per mm~
may be observed for signs of deterioration. (B) WBC>l,OOOpermm3
Testl 3

(C) WBC > 10,000 per mm3 (B) Antibiotics should be given to cover typical
(D) WBC >50,000 permm3 polymicrobial vaginal flora.
(E) WBC > 100,000 per mm3 (C) When not infected, this gland is palpable at the
5 o'dock and 7 o'dock positions around the
~ Which ofthe following is the most effective treatment vaginal introitus.
for lithium poisoning? (D) A Word catheter should be placed after incision
(A) Activated charcoal and drainage and left in plac:e for 6 to 8 weeks.
(B) Sodium bicarbonate (E) lfleft untreated, ascending cystitis,
(C) Glucagon pyelonephritis and sepsis occurs in 35% of
(D) Potassium chloride patients.
(E) Hemodialysis
(!j) Which of the following findings should cast
(ll) Which ofthe following is the most appropriate initial doubt on a diagnosis of Guillain-Barre syndrome
antihypertensive medication for use in patients with (GBS)?
severely elevated blood presaure and acute cardiac (A) A patient who presents with symmetrical
ischemia? sensory loss but who has no motor wealmess.
(A) Nifedipine (B) A patient with bilateral lower extremity
(B) Labetalol wealmess and paresthesias who has no reflexes
(C) aonidine in his lower or upper extremities.
(D) Enalaprilat (C) A patient with bilateral lower extremity
(E) Hydrocblorothiazide wealmess, urinary retention, and a normal
spinal magnetic resonance imaging
(!j) A 22-year-oldwoman presents with a chiefcomplaint (MRI).
of a painful vaginal lump and vulvar pain while (D) A patient with a 2-day history ofbilateral
walking (see Fig. 1-4). She denies any vaginal lftakness and paresthesias of the feet who has a
discharge, fevers, or abdominal pain. Her urine normal cerebrospinal fluid (CSF) examination.
hwn.an chronic gonadotropin (/1-hCG) is negative. (E) None of the above.
Which of the following is true?
(M) Traumatic hyphema •..
(A) Is usually associated with an afferent pupillary
defect.
(B) Is treated with supine position and oculomotor
exercises.
(C) May require analgesic treatment with aspirin.
(D) Can be complicated by re-bleeding.
(E) Rarely requires specific management.

I!ID A35-year-old marathon runnerwho is asymptomatic


presents from an outside hospital with an "abnormal
EKG.'' The EKG is shown in Figure 1-S. Your advice
to the patient is
(A) Telemetry admission for pacemaker.
(B) Atropine 1 mg IV.
Figure 1-4. (C) Amiodarone 300 mg IV.
(D) Adenosine 6 mg IV.
(E) No acute therapy, routine follow-up.
(A) N. gonmrhoetle and Chldmydill trachottU~tis
cause most infections.
4 1000 Quatimu fD Help You Ptw the~ MetlicW&iris

Figlln :1-5. (See color insert.)

[!il A 6-year-ol.d child presents with seizure. She has had ~ Which of the following is true regarding ischemic
acute gastroenteritis for the past 2 days. Electrolytes heart diseasef
are normal. Which of the following is the most likely (A) It is the second leading cause of death in the
causer United States.
(A) Salmonella (B) lt is the second leading cause of death of
(B) ShigelJa women (af"W breast cancer).
(C) Campylobacm (C) Recent .reduction in mortality is partly related
(D) Yersinia to decline in death from acute myocardial
(E) Vibrio in&rction (Ml).
(D) Less than one fourth of all Mls occur in
t Which of the following is a major Jones criterion for patients older than 65 years.
diagnosis of rheumatic fever? (E) Only 5% of sudden deaths from acute MI
(A) Fever occur outside the hospital
(B) Sore throat
(C) Increaaed C-reactive protein (CRP) [fi) A 22-yea.r-old diabetic man presents l day after
(D) Sinus tachycardia obtaining a puncture wound to his left arm during
(E) Chorea a hiking accident. Since the accident, he has noted
increuingly intense pain in his arm along with mild
(jj Which ofthe following is the most frequently affected swelling and .redness. On aami.nation, his arm is
mucture in thoracic outlet syndrome? noted to be mildlyswollen and erythematous with an
(A) Subclavian artery
innocuous-appearing puncture wound on the volar
(B) Subclavian vein
aspect of his right forearm. His arm is extremely
tender, although there is no aqntus. A plain film
(C) Ulnar nerve
(D ) Radial nerve
is obtained which reveals subcutaneous emphysema.
Which of the following iJ the next best step in
(E) Median nerve
management?
(!i Which ofthe following iJ the most common cause of (A) Irrigation of the wound with sterile saline
multiple rib fractures in children? (B) Surgical consult
(C) Incision and drainage in the ED
(A) Motor vehicle collision
(B) Childabwe
(D) cr scan of the arm
(E) MRI of the arm
(C) Pall
(D) Sports injury
(E) Gunshot wound
Testl 5

~ A 15-year-old man is brought to the ED after being (D) 50 g


submerged in a lake for "a minute or two." He (E) 100 g
had been water skiing when he lost control and
"wiped out" and then was lying face down in ~ Which of the following is a risk factor for pelvic
the water without moving. He was not breathing inflammatory disease (PID)?
when his friends pulled him out of the water (A) Age >35 years
but he regained spontaneous respirations after they (B) Smoking
performed cardiopulmonary resuscitation (CPR). In (C) Cocaine use
the ED, he is somnolent, but breathing spontaneously (D) White race
with a pulse oximetry of 95% on room air and making (E) Tampon use
purposeful movements. The next most important
step in management is ~ Which of the following is the best modality to
(A) IV antibiotics. diagnose posterior sternoclavicular dislocation?
(B) Rapid sequence intubation. (A) Anteroposterior (AP) chest x-ray
(C) IV dexamethasone. (B) Lateral chest x-ray
(D) Cervical spine films. (C) Anteroposterior clavicle x-ray
(E) ED thoracotomy. (D) CT chest
(E) Thoracic ultrasound
~ Which of the following treatments has been shown
to be effective in the prevention of acute mountain ~ A 62-year-old woman with ahistoryof02-dependent
sickness (AMS)? chronic obstructive pulmonary disease (COPD)
(A) Propranolol presents to the ED with a chief complaint of dyspnea
(B) Acetazolamide and increased cough productive of yellow phlegm.
(C) Furosemide The patient uses 2 L of 0 2 at home. When you
(D) Caffeine enter the room to assess the patient, you find an
(E) Nifedipine ill-appearing, dyspneic woman speaking in sentence
fragments. Her Pox reads 85% and you note she is on
~ Which of the following helps to differentiate a 4 L of 02 by nasal cannula. As you start to increase
sympathomimetic crisis from an anticholinergic her 02, you wonder ifyou are going to eliminate her
crisis? respiratory drive. The next best step is to:
(A) Tachycardia (A) Immediately intubate the patient using rapid
(B) Mydriasis sequence intubation.
(C) Diaphoresis (B) Increase the 0 2 to 6 L, because minute
(D) Seizures ventilation changes little in COPD patients
(E) Altered mental status exposed to higher levels of oxygen.
(C) Decrease the Oz to 2 L (her baseline) in order
~ A 28-year-old man presents with acute, progressively to increase the patient's respiratory drive.
worsening headache and fever for 1 day. Physical (D) Perform an arterial blood gas (ABG) to assess
examination reveals a toxic-appearing, slightly con- the patient's exact ventilatory and oxygenation
fused patient with fever and meningismus. Which of status because you do not know what effect
the following is the most appropriate next step in changing the oxygen will have.
management? (E) Leave the 0 2 at 4 L because the patient is likely
hypoxic at baseline and continue treatment
(A) CT brain without contrast
hoping the patient will improve.
(B) CT brain with contrast
(C) Lumbar puncture
~ Which of the following is true regarding Lyme
(D) Ceftriaxone
disease?
(E) Ceftriaxone and dexamethasone
(A) Incidence is highest in the San Joaquin Valley.
~ Which of the following is the lowest acute toxic (B) The causative agent is a gram-negative bacillus.
ingestion ofacetaminophen requiring treatment with (C) The most common neurologic sign is facial
N -acetylcysteine (NAC) in the adult patient? nerve palsy.
(D) Lyme meningitis is clinically indistinguishable
(A) 5 g
from bacterial meningitis.
(B) 7.5 g
(E) First-line therapy is with clindarnycin.
(C) 15 g
6 1000 Questicms to Help You Pass the Emergency Medicine Boards

~ In a patient with benzodiazepine poisoning, which extension. Which of the following is the most likely
of the following is an indication for flumazenil use? structure to be injured?
(A) Accidental pediatric ingestion (A) Anterior cruciate ligament (ACL)
(B) Co-ingestion of tricyclic antidepressant (B) Posterior cruciate ligament (PCL)
(C) Chronic benzodiazepine user (C) Medical meniscus
(D) Alcoholic patient (D) Lateral meniscus
(E) Seizure activity (E) Medial collateral ligament (MCL)

~ A 26-year-old man develops headache, dizziness, and ~ Which of the following is true regarding metacarpal
nausea while hiking at 10,000 ft. His destination is a neck fractures?
cabin that is 500ft higher. The cabin has a medical (A) Fractures of the ring finger require more
kit with acetazolamide and dexamethasone. Which accurate alignment than fractures of the index
of the following is the next best step? finger.
(A) Set up a tent and sleep at their current altitude (B) Rotational deformities are poorly tolerated and
for the night. must be corrected.
(B) Continue to the cabin for the medications. (C) Fractures should be reduced with the finger in
(C) Descend 1,500 to 2,000 ft or until symptoms extension.
resolve. (D) Fractured digits can be buddy taped to their
(D) Stimulate the hypoxic ventilatory response healthy neighbors.
with an energy drink. (E) All of the above.
(E) All of the options are reasonable.
[llJ Which of the following statements about exercise-
~ A 27-year-old woman, G2 Ph at 5 weeks by dates, induced asthma (EIA) is correct?
presents with abdominal pain. She has had minor (A) Attacks during active exercise are common and
"spotting" but no frank vaginal bleeding, her internal prevent most of these patients from engaging
cervical os is closed, and her serum ,6-hCG is 750 miU in strenuous exertion.
per mL. Which of the following is true? (B) Recovery is usually slow and requires
(A) She should be diagnosed with a threatened prolonged use of inhaled ,82-agonists.
abortion, and asked to return in 2 days for a (C) Peak symptoms actually occur typically 5 to
repeat quantitative ,6-hCG level. 10 minutes after exertion.
(B) She has a missed abortion. (D) Attacks occur more commonly in asthmatics
(C) She should receive an ultrasound to assess who participate in summer sports than in
uterine contents and the adnexa. winter sports.
(D) She should be diagnosed with an early (E) None of the above.
pregnancy versus an ectopic pregnancy and
told to return in 2 days for a repeat quantitative ~ A 65-year-old woman presents to the emergency
,6-hCG level and an ultrasound. room (ER) with signs and symptoms of digitalis
(E) She has an inevitable abortion. toxicity, ventricular tachycardia, and digoxin level of
8.5 ng per mL. She is treated with digitalis antibody
~ Which ofthe following is the most commonly injured fragment therapy and the cardiac rhythm is now
abdominal organ in pediatric blunt trauma? sinus. A repeat digoxin level after the fragments are
(A) Liver given is 12 ng per mL. Which of the following is the
(B) Spleen most appropriate next step in management?
(C) Kidney (A) No acute therapy
(D) Small intestine (B) Cardioversion at 50 J
(E) Large intestine (C) Procainarnide 1 g IV
(D) Calcium chloride 1 g IV
~ A 45-year-old man presents with locking and clicking (E) Potassium chloride 40 rnEq IV
of his knee for several days. He twisted it several
weeks ago and did not seek medical care at the ~ According to the current guidelines for tPA ad-
time. Physical examination demonstrates no knee ministration, which of the following represents the
instability or patellar tenderness. His knee clicks with
flexion and occasionally gets locked just before full
Testl 7

maximum blood pressure threshold for a patient to ~ A 33-year-old man presents with severe agitation,
be eligible for tPA? psychosis, and violent behavior. Physical examina-
(A) Systolic blood pressure (SBP) < 185 and tion demonstrates vertical nystagmus. Which of the
diastolic blood pressure (DBP) <110 following is the most likely drug ingested?
(B) DBP <120 (A) Cocaine
(C) Mean arterial pressure (MAP) <130 (B) Phencyclidine (PCP)
(D) SBP <220 and DBP < 120 (C) Lysergic acid diethylamine (LSD)
(E) SBP < 140 and DBP <90 (D) Heroin
(E) Methylenedioxyrnethamphetamine (MDMA)
~ A 35-year-old man presents with severe head
trauma. Funduscopic examination demonstrates pa- ~ A 65-year-old woman with history of diabetes
pilledema and increased intracranial pressure (ICP) and hypertension presents with chest pain for 5
and impending herniation is suspected. Mannitol hours. Physical examination demonstrates blood
is given and the decision is made to hyperventilate pressure of 100/55, jugular venous distension (JVD),
the patient to induce cerebral vasoconstriction and and hepatomegaly. No crackles are noted. EKG
decrease blood volume. Which of the following is demonstrates inferior ST depressions. You suspect
an appropriate target level of PC02 for therapeutic the possibility of right ventricular infarction. You
hyperventilation? instruct the ED technician to place the rV4 lead at:
(A) 22mmHg (A) Right fourth intercostal space, mid-clavicular
(B) 27mmHg line.
(C) 32mmHg (B) Right fifth intercostal space, mid-clavicular
(D) 37mmHg line.
(E) 42mmHg (C) Right sixth intercostal space, mid-clavicular
line.
~ Which of the following is true about systemic lupus (D) Right fourth intercostal space, right sternal
erythematosus (SLE)? border.
(A) Arthralgias and arthritis are typically (E) Right fifth intercostal space, right sternal
asymmetrical. border.
(B) The classic malar rash is seen in only 40% to
50% of patients. ~ Which of the following is the most common form of
(C) Stroke is the most common neurologic botulism?
manifestation. (A) Food-borne
(D) Libman-Sachs endocarditis is the most (B) Infant
common cardiac manifestation. (C) Wound
(E) Drug-induced lupus is typically irreversible. (D) Respiratory
(E) Cardiac
~ A 22-year-old woman presents in anticholinergic
crisis. She is delirious, agitated, and requires sedation. ~ Which of the following is true regarding falls from
Which of the following medications would be most buildings?
appropriate to sedate this patient?
(A) Mortality is unrelated to height of the fall.
(A) Thorazine (B) Feet-first falls cause retroperitoneal bleeding
(B) Fluphenazine more often than intraperitoneal bleeding.
(C) Lorazepam (C) Renal injury is uncommon in falls onto supine
(D) Etomidate position.
(E) Ketamine (D) Calcaneal fracture is the most common cause
of death from all falls.
~ Which of the following is most characteristic of (E) Falls onto prone position almost never result in
complex regional pain syndrome (CRPS) (formerly death.
known as reflex sympathetic dystrophy)?
(A) Diminished pulse [!!) A 15-year-old boy comes to the ED complaining
(B) Prolonged capillary refill time of shortness of breath and nausea. He admits to
(C) Paralysis "huffing'' glue before presentation. Which of the
(D) Diaphoresis
(E) Male predominance
8 1000 Qumimss fD Help Yow Ptw the Emerfmc.y Metlicine Bolris

following is the most appropriate therapy at this strength is 1/5 and bilateral lower ememity strength
time? is 4/5. Which of the following is the most likely
(A) Supportive care only pathophysiologic process?
(B) Antibiotics (A) Anterior cord syndrome
(C) Corticosteroids (B) Central cord syndrome
(D) Diuretics (C) Brown-~uard syndrome
(E) Activated charcoal (D) Cauda equina syndrome
(E) Complete cord injury
~ A 72-year-oldman presents totheEDwith a bump on
his nose (see Fig. 1-6). He denies any pain associated ~ A 30-year-old. 80-.kg male patient who presented to
with the lesion but has noticed that it has been your ED with an acute asthma exacerbation is not
growing over the last several months. Which of the responding to optimal therapy. You decide to treat
following is the most likely diaposis! him with epinephrine. The appropriate dose is
(A) 0.3 mL 1:10,000 SQ.
(B) 0.5 mL 1:10,000 SQ.
(C) 0.3 mL 1:1,000 SQ.
(D) 0.5 mL 1:1,000 IV.
(E) 0.3 mL 1:10,000 IM.

~ A 33-year-old woman presents with a chieC com-


plaint of dysphagia. She feels a sensation of foods,
particularly solids, getting ••stuck" in her cheat and
she sometimes needs to raise her arms above her
head or straighten her back after eating to help things
pass. She also complains of intermittent substernal
buming chest pain. Her doctor has been treating her
for gastroesophageal reJlux disease (GERD) for the
last 9 months but she seems to be getting worse.
What is the likely cause of her symptoms?
(A) Nutcra.c:ker esophagus
(B) Diffuse esophageal spasm
(C) Schatzki's ring
(D) Achalasia
(E) Zenker diverticulum
Figure 1-&.
~ A 45-year-oldman presents with sips and symptoms
ofbacterial meningitis. Which of the following is the
(A) Basal cell carcinoma (BCC) single most likd.y cause?
(B) Melanoma (A) Group B streptococcus
(C) Kaposi's sarcoma (B) Listeria monocytogenes
(D) Mycosis fungoides (C) Streptococcus pneumon~
(E) Squamous cell carcinoma (SCC) (D) Neisseria mmingitidis
(E) Hemophilus injluenzu
~ Which ofthe following is the most common cause of
seizures in a term newbomf @!) Which of the following is true regarding status
(A) Hypom epilepticus?
(B) Hydrocephalus (A) Children younger than 16 years have the
(C) Intracranial hemorrhage highest mortality due to status epilepticus.
(D) Idiopathic (B) Treatment with diazepam has better outcomes
(E) Hypoglycemia than with lorazepam.
(C) The most common side effect of
~ A 44-year-old woman presents after a motor vehicle benzodiazepines given for status epileptic:ua is
collision with a complaint of neck pain. Neurologic hypoventilation.
aamination reveals that bilateral upper extremity
Test I 9

(D) Among adults, the most common etiology of (A) Glucuronidation


status epilepticus is subtherapeutic (B) Sulfation
antiepileptic drug levels. (C) Cytochrome P-450 oxidation
(E) None of the above is true. (D) Direct renal excretion
(E) Plasma breakdown
~ Which of the following is true regarding aortic
dissection? [!!) Of the following, which is the most common cause
(A) Dyspnea is the most common symptom. of true pediatric hemoptysis?
(B) Syncope often indicates pericardia! effusion (A) Mycobacterium TB infection
and tamponade. (B) Lung cancer
(C) Aortic regurgitation occurs in most cases. (C) Goodpasture's syndrome
(D) Dissection into coronary arteries occurs most (D) Congenital heart disease
often in the left coronary artery. (E) Cystic fibrosis
(E) Interarm differences in SBP of> 20 mm Hg
occur in most patients. ~ Which of the following is true about acute diarrheal
illnesses due to Shigella spp.?
~ A 53-year-old man with hypertension presents with a (A) Antimotility agents such as diphenoxylate are
2-month history ofback pain and difficulty urinating. useful as monotherapy in patients with
His prostate is normal on examination, but you Shigellosis.
appreciate mildly decreased strength in his lower (B) Infection with Shigella spp. requires an
extremities bilaterally. While waiting for an MRI to unusually large inoculum.
investigate the cause of his suspected epidural spinal (C) Colicky abdominal pain with high fever and
cord compression, you order a postvoid residual to diarrhea is the most common presentation.
be checked in the interim. Which of the following is (D) The treatment of choice is
the upper limit for a normal postvoid residual in this trimethoprim -sulfamethoxazole.
patient? (E) Fecal leukocytes are only rarely detected in
(A) 15mL. patients with Shigellosis.
(B) 50mL.
(C) 100 mL. ~ A 22-year-old man presents with cough, fever, and
(D) 150 mL. shortness ofbreath for 3 days. He has no past medical
(E) 200 mL. history. His vital signs are: 100.5°F, 92, 22, 122/72,
98% RA. Examination reveals left lower lung field
~ Which of the following is the most common side crackles that do not clear on coughing. Which of the
effect of dihydroergotamine (DHE), used in the following is the most appropriate therapy?
treatment of acute migraine headache? (A) Doxycycline PO
(A) Diarrhea (B) Linezolid PO
(B) Flushing (C) Cephalexin PO
(C) Shortness of breath (D) Clindamycin PO
(D) Nausea (E) Piperacillin-tazobactam N
(E) Chest pain
~ In patients with measles, where are Koplik's spots
~ A 43-year-old woman with a history of a renal trans- most likely to be seen?
plant presents with fever, headache, and purulent (A) Hard palate
rhinorrhea. On examination she is found to have (B) Soft palate
blackened nasal mucosa. Which of the following is (C) Tonsils
the likely cause of her illness? (D) Tongue
(A) Rhizopus spp. (Mucormycosis) (E) Buccal mucosa
(B) Coccidioides imitis
(C) Histoplasma capsulatum ~ A 26-year-old previously healthy man is brought to
(D) Blastomyces dermatiditis the ED after being found unconscious outside a bar
(E) Cryptococcus neoformans after having vomited on himself. He is drunk and
afebrile, and his pulse oximetry reveals a saturation
~ Which ofthe following represents the major route of of96%. His chest x-ray is shown in Figure 1-7. Which
metabolism for acetaminophen? of the following is true?
10 1000 Qvaticms ftl Hdp You Ptw the Emerfmcy Metlit:W BoMds

(C) Hypophosphatemia
(D) Hyponatremia
(E) Hypomagnesemia

(!!] Which of the full.owins are common find.inss


in patients diagnosed with pneumocystis carinii
pneumoniaf
(A) FJevated. transaminases (aspartate
aminotransfe:rase [AST], alanine
transaminase [ALT])
(B) Reapiratory acidosis due to C02 retention
(C) Pleural effusions
Figure 1-7.
(D) Elevated arterial lactate levels
(E) Elevated lactate dehydrogenase (LDH) levels
(A) The patient likely has a devdoping bacterial
pneumonia and requires broad spectrum ~ Which of the following rashes is chara.cterized by a
ant.J.biotics. positive NikobkYs sign?
(B) The patient likely has a chemical pneumonitis (A) Roseola infantum
caused by the aspiration of acidic gastric (B) Bullow impetigo
contenta, which may later develop into (C) Pempb.i.gua vulgaris
pneumonia. (D) Tinea corporis
(C) Antibiotics should be started early in chemical (E) Erysipelas
pneumonitia to prevent the subsequent
development of bacterial superinfection. ~ A mother hrin8s in her 4-year-ol.d daughter com-
(D) Corticosteroids have been proved to be plaining of persistent perianal pruritw. The symp-
bene6cial in patients such as these. toms are wone at nisht and the mother has had to cut
(E) This patient is probably suffering from a her daughter's nails short because she was scratching
combination of chemical pneumonitis and and irritating her slcin. Which of the following is
bacterial infection. true?
(A) The most sensitive test is a stool sample for ova
(ij) A 24-year-old man presented to the ED with an acute and parasites.
asthma exacerbation that was refractory to optimal (B) The organism respoDSlble for these symptoms
treatment. A few minutes after intubating him, the can occasionally cause urinary tract infections
resuscitation nune tells you that the patient is waking and even vulvovaginitis.
up and "fighting the vent" You realize that the drugs (C) The inkction is moat commonly acquired by
you used for rapid sequence intubation are wearing ingestion ofcontaminated food or water.
off. Your best initial course of action is to: (D) Metronidazole is the antibiotic ofchoice.
(A) Paralyze the patient with cis-atracurium to (E) Eosinophilia is commonly associated with her
eruure that he "synchronizes" with the vent. condition.
(B) Give the patient a bolus of IV midazolam and
place the patient on a midazolam drip. (fi) Physical examination of a patient with Reiter's
(C) Re-induce the patient with k.etamine, at a dose syndrome may be expected to reveal:
of 1.5 mg per kg. (A) Waxy plaques on the palms and soles
(D) Paralyze the patient with succinylcholine to (B) Sawl8e-J.ih swelling of the fingers
eruure that he "synchronizea" with the vent. (C) Painful, shallow ulcers in the mouth
(E) Check a stat metabolic pand to ensure the (D) Iritis
patient is not hyperkalem.ic:. (E) All of the above

li!] A25-year-old man with history ofacute myelogenow [!j] The most specific finding for carpal tunnel syndrome
leukemia (AMI.) presents with acute onset of gene- (CTS) is
ralized weakne.s. H e received his first dose ofchemo-
(A) Normal sensation on the medial side but
therapy 3 days before. Which of the following is the
abnormal sensation on the lateral side of the
most likely abnormality on laboratory analysis?
ring finger.
(A) Hyperkalemia (B) Weakness of thumb opposition.
(B) Hypercalcemia
Test 1 11

(C) Abnormal sensation of the clistal palmar tip of leg swelling, orthopnea, and cough with frothy
the index finger. sputum. She denies chest pain. Physical examination
(D) The presence of a positive Tinel's sign. demonstrates presence of an S3, bilateral pulmonary
(E) Lumbrical weakness. crackles, and pitting edema of her lower extremities.
Which of the following is true regarding this
~ A 20-year-old man presents to the ED with several condition?
days of progressive chest pain, fatigue, myalgias, (A) Cardiac catheterization is the next step in
and exertional dyspnea. He states that he had the management.
"flu" 1 week before. He denies any illicit drug (B) It occurs in approximately 10% of all
use or family history of heart clisease. Physical pregnancies.
examination reveals temperature of 100.5°F, heart (C) Patients who survive do not develop the
rate of 125, no murmurs on cardiac examination, and condition in subsequent pregnancies.
scattered bilateral crackles on lung examination. EKG (D) Mortality is as high as 30%.
demonstrates sinus tachycardia, chest x-ray reveals (E) Aspirin is the mainstay of therapy.
cardiomegaly and mild pulmonary edema, and
laboratory reports are normal except for troponin I, [!!) A 15-year-old girl is brought by her parents because
which is elevated at 10 ng per mL. Which of the her parents suspect her of drug use. In the absence of
following is the most likely etiology? her parents, the patient states that she was at a rave
(A) MI party the previous night and took a pill to help her
(B) Aortic dissection "feel closer" to her friends. Which of the following is
(C) Viral infection the most likely drug used?
(D) Stroke (A) Lysergic acid diethylamine (LSD)
(E) Diabetic ketoacidosis (B) PCP
(C) 3, 4-methylenedioxymethamphetamine
lMJ Which ofthe following is true regarding interpersonal (D) Sertraline
violence and intimate partner abuse (IPA)? (E) Phenelzine
(A) Most violence against women is perpetrated by
strangers. ~ A 55-year-old man presents with pelvic pain after a
(B) Most of the violence against men is perpetrated motor vehicle crash. His primary survey is intact
by intimate partners. and vital signs are normal, but careful pelvic
(C) Less than 1% of all men report an incident of examination demonstrates a fractured pelvis with
IPA. a fragment jutting through the rectal mucosa. Which
(D) Victim substance abuse contributes to IPA. of the following is the most appropriate definitive
(E) Most patients will not report IPA unless management?
directly asked. (A) Operative repair
(B) Angiography with embolization
~ Which of the following patients has the highest (C) Bedside irrigation
statistical chance of completed suicide? (D) Bedside rectal suturing
(A) 75-year-old black man (E) Foley catheter placement
(B) 75-year-old white man
(C) 18-year-old black woman [ig) Which of the following is the most common risk
(D) 35-year-old pregnant woman factor in the development of a gastric ulcer?
(E) 50-year-old married white man (A) Aspirin or nonsteroidal anti-inflammatory
drug (NSAID) use
(fi) Which of the following tissues has the greatest (B) Cigarette smoking
resistance to electrical flow? (C) Alcohol abuse
(A) Nerves (D) Shock states
(B) Blood (E) Helicobacter pylori infection
(C) Fat
(D) Skin [i!) Patients with spinal stenosis:
(E) Muscle (A) Most commonly present with a radiculopathy
that is worse with walking.
[Ill A 23-year-old woman presents with shortness of (B) Typically feel relief when leaning forward.
breath. She is postpartum day 3 after a term, normal
spontaneous vaginal delivery. She describes bilateral
12 1000 Qvaticms ftl Hdp You Ptw the Emerfmcy Metlit:W BoMds

(C) Often have a lumbar radiculopathy at several (jiJ Which of the following neurologic functions is
levels. commonly impaired in patients with anterior cord
(D) May be able to walk uphill more easily than syndromes?
downhill.
(A) Vibration sensation
(E) All of the above.
(B) Fine touch sensation
(C) Temperature seruation
[ij] A 54-year-old diabetic woman presents with perineal
(D) Position sensation
pain, fever, and ldhargy. Her perineal examinaOOn
(E) Extraocular motor function
is shown in Figure 1-8. Which of the following is the
most appropriate next step in management?
lij] A 23-year-old woman presents with fever, myalgias,
and headache for 3 days. She then developed a rash,
which started on her wrists and ankles and has now
spread all over her body. The nonblan.ching rash is
shown in Figure 1-9. Which of the following is the
most likely etiology?

Figure 1-8.

(A) Corticosteroids
(B) Oral antibiotics
(C) Intravenous antibiotia
(D) Intravenoua ant1biotics and surgical
debridement
(E) MRI pelvis

~ Which ofthe following is true regarding the afterdrop


phenomenon in hypothermic patients?
(A) Afte:rdrop prcdiaposes patients to the
development ofventricular fibrillation.
(B) Afterdrop is exacerbated by actM: external
rewarming.
(C) Afte:rdrop is caused by the return of cold
peripheral blood to the core upon rewarming.
(D) Afterdrop results from a paradoxical slowing of
metabolism during rewarmirJs.
(E) Afte:rdrop hu not been shown to be of any
clinical importance. Ffgun 1-9.

~ A 34-year-old man overdoses on bupropion. Which


of the following serious sequelae is most likel~ (A) Ridcettsia ridcett5ii
(A) Coma (B) BoTTe1i4 burgdqrfori
(B) Seizure (C) Coxsadcieviru.s
(C) Torsade de pointe& (D) N.grmo"hoeae
(D) Hypotension (E) N. meningitidis
(E) Hypertension
Test 1 13

~ A 35-year-old woman presents after a fall from the (D) Norepinephrine


fourth-story window. She was initially awake and (E) Amrinone
moaning, but then becomes unresponsive. Her pupils
are both initially pinpoint and progress to midpoint ~ A 34-year-old woman with acquired immune de-
with loss of light reflex. She exhibits decorticate ficiency syndrome (AIDS) presents with first-time
posturing in response to pain and hyperventilation. seizure. Which of the following is the most likely
You suspect cerebral herniation. Which of the cause of her seizure?
following is the most likely mechanism? (A) Mass lesion
(A) Uncal (B) Meningitis
(B) Central transtentorial (C) Stroke
(C) Upward transtentorial (D) Electrolyte abnormality
(D) Cerebellotonsillar (E) Syphilis
(E) Medullary
~ Which of the following represents the major phar-
~ An 85-year-old man without any past medical history macologic action of low molecular weight heparins?
presents to the ED with a fall from standing height (A) Direct platelet inhibitor
with accompanying head trauma. A CT scan of the (B) Factor II (thrombin) inhibitor
brain is ordered. Which of the following is the most (C) Factor Xa inhibitor
likely abnormal finding on CT scan? (D) von Willebrand factor inhibitor
(A) Epidural hematoma (E) Factor XIII inhibitor
(B) Subdural hematoma
(C) Subarachnoid hemorrhage ~ Which of the following is true regarding Guillain-
(D) Cerebral contusion Barre syndrome (GBS)?
(E) Cerebellar hematoma (A) The clinical course is more severe in the elderly.
(B) Patients with a rapid onset are more likely to
[!!) Which of the following is an indication for replanta- have a benign recovery.
tion after amputation? (C) Patients with preceding gastroenteritis caused
by Campylobacter jejuni have a more benign
(A) Amputation of the thumb in the nondominant
course.
hand.
(D) Autonomic involvement, such as urine
(B) Ring finger amputation distal to the distal
retention, ileus, sinus tachycardia, and postural
interphalangeal joint (DIP) joint.
hypotension is uncommon in GBS.
(C) A 58-year-old diabetic factory worker with an
(E) Respiratory failure requiring mechanical
amputation of his index finger at the
ventilation eventually occurs in 75% of
metacarpophalangeal (MCP) joint.
patients.
(D) A 23-year-old laboratory worker with a middle
finger amputation at the level of his middle
phalanx who stored his finger packed in dry ice
~ Which of the following is a unique aspect of
tarantulas?
for the last 12 hours.
(E) All of the above. (A) Their abdominal hairs can become embedded
in the skin resulting in subsequent allergic
~ Which of the following is the most common mode reactions.
of completed suicide? (B) In addition to venom, they serve as a vector for
Borrelia spp. and may transmit Lyme disease.
(A) Drug ingestion
(C) Their venom triggers widespread mast cell
(B) Firearms
degranulation resulting in diffuse flushing and
(C) Carbon monoxide
pruritus.
(D) Hanging
(D) Tarantula envenomation may result in
(E) Wrist cutting
anaphylaxis and rapid respiratory failure.
(E) Envenomation of an extremity can cause
[!!) Which of the following is the appropriate initial transient paralysis distal to the wound.
treatment for a patient who becomes hypotensive
after receiving nitroglycerin? ~ A 21-year-old woman presents to the ED for a
(A) IV fluids "recheck" of her f:l-hCG level. She was seen by a
(B) Dopamine colleague in your ED 2 days ago after presenting with
(C) Dobutamine crampy low abdominal pain and vaginal spotting.
14 1(}()() Questions to Help You Pass the Emergency Medicine Boards

Her f3 -hCG level at that time was 1,350 miD per mL (C) Scarlet fever occurs acutely with GAS
but her ultrasound was not diagnostic. She states that pharyngitis but acute rheumatic fever does not
her pain has resolved and she is no longer having any typically occur until2 to 4 weeks after GAS
spotting. Her f3 -hCG level is currently 2,400 miU per pharyngitis.
mL. What is the best next step in management? (D) Scarlet fever is now a rare occurrence due to
(A) Discharge her home with a diagnosis of the Hemophilus influenzae type B (HiB) vaccine
pregnancy. (E) Both scarlet fever and acute renal failure ( ARF)
(B) Obtain an ultrasound to evaluate uterine can be prevented by the use of antibiotics for
contents and the adnexa. GAS pharyngitis.
(C) Consult an obstetrician (OB) for a probable
missed abortion. ~ The use of "triptans" for the treatment of migraine-
(D) Discharge her home with a diagnosis of related headache should be limited to 2 days per week
threatened abortion. for which of the following reasons?
(E) Discharge her with instructions to follow up (A) Prolonged use may result in pulmonary
with OB in 2 days for serial {3-hCG levels. fibrosis.
(B) Increased risk of rebound headache.
~ In an otherwise normal x-ray, the finding of a (C) Increased risk of cardiac ischemia.
"posterior fat pad" on a lateral x-ray of the elbow in (D) Development of permanent lower extremity
adult and pediatric patients is suggestive of which of paresthesias and numbness.
the following? (E) They increase the risk of ischemic stroke.
(A) Normal finding in adults, supracondylar
fracture in children [100] A 3-year-old previously healthy girl is brought to
(B) Supracondylar fracture in adults and children the ED by her parents with a complaint of fever and
(C) Radial head fracture in adults, supracondylar cough. Her physical examination findings and chest
fracture in children x-rayare consistent with pneumonia. She is tolerating
(D) Olecranon fracture in adults, supracondylar PO without difficulty and appears nontoxic so you
fracture in children plan to discharge her home. Which of the following
(E) Radioulnar dislocation in adults, is the most appropriate antibiotic regimen?
supracondylar fracture in children (A) Doxycycline
(B) Erythromycin
[!!] Which of the following statements is true regarding (C) Levofloxacin
scarlet fever and acute rheumatic fever? (D) Trimethoprim-sulfamethoxazole
(A) Both occur concomitantly with group A (E) High-dose amoxicillin
f3- hemolytic streptococcal (GAS) pharyngitis.
(B) Acute rheumatic fever is a complication of
acute GAS pharyngitis that occurs more
commonly in adults.
Answers and Explanations
[I) Answer E. The patient is exhibiting a sympath- abscesses continue to expand and may spontaneously
omimetic picture after smoking marijuana, which rupture, resulting in secondary generalized peritoni-
commonly occurs because of crack cocaine addi- tis. Roughly 5% to 10% of women with acute PID
tives. His myocardial ischemia is likely because of will develop perihepatitis known as Fitz-Hugh-Curtis
a combination of vasospasm and hyperaggregatory syndrome. This is primarily because of hematoge-
platelets. Aspirin is indicated for treatment. His chest nous or transperitoneal spread of C. trachomatis,
pain is resolved and he is not having a segment eleva- and is characterized by right upper quadrant pain
tion myocardial infarction (STEMI), so PTCA, tPA, and tenderness, pleuritic chest pain, and occasionally
and G2b3a inhibitors are not warranted. Metopro- elevated liver enzymes.
lol is relatively contraindicated in cocaine-induced
chest pain as there is a theoretical risk of reducing [!) Answer B. The patient has a ruptured abdominal
cardiac output by inhibiting ,8-receptors in the face aortic aneurysm (AAA), as indicated by the adjacent
of increased peripheral vascular resistance. retroperitoneal hematoma.
Ruptured AAAs should be operated on in all patients
[!) Answer A. The patient has asymptomatic first- who do not have absolute contraindications to
degree atrioventricular (AV) block with a prolonged surgery, as loss of complete blood volume can occur
PR interval but no dropped beats. This is commonly within minutes. There is no evidence that aggressive
seen in healthy individuals, has no prognostic reduction of blood pressure is helpful in AAA, and
significance, and requires no further evaluation or it may predispose patients to unnecessary risk of
management. hypotension. Once a ruptured AAA is definitively
diagnosed by CT scan, there is little role for
[!) Answer C. Propoxyphene has sodium channel further imaging in the ED with angiography or
blocking properties that can cause QRS prolongation, ultrasound. Observation of patients with ruptured
especially when used in conjunction with other med- AAAs is contraindicated. (Figure courtesy of Robert
ications that block sodium channels (e.g., tricyclic Hendrickson, MD. Reprinted with permission from
antidepressants). Hendrickson R. Greenberg's text-atlas of emergency
medicine. Lippincott Williams & Wilkins; 2004:194.)
~ Answer C. This patient most likely has a tuba-
ovarian abscess (TOA) complicating pelvic inflam- 00 Answer C. The patient has binocular diplopia,
matory disease (PID). Her history suggests that which resolves when either eye is covered. Causes
she presented to her primary care doctor with include all the answer choices, of which choice C
symptoms of PID, but because she only received (cranial nerve palsy) is the most common. Monocular
an antibiotic effective against N. gonorrhoeae and diplopia occurs in a specific eye and is usually caused
has not filled her prescription, she may still have by localized eye pathology.
untreated C. trachomatis infection. Interestingly, al-
though N. gonorrhoeae and C. trachomatis are known [1] Answer C. Schizophrenia is a chronic, progressive
to be instrumental in the development of a TOA, thought disorder present in 1% of the population.
they are very rarely obtained at culture. Instead, It is characterized by the presence of both positive
the abscesses tend to be polymicrobial, and include symptoms (choices A, B, D, and E) and negative
gram-negative enteric organisms as well as anaer- symptoms-flat affect, lack of speech, and inability
obes, such as E. coli and B. fragilis. It is thought to perform goal-directed activities. Although the
that N. gonorrhoeae and C. trachomatis initiate the positive symptoms are often managed successfully by
infection, whereas other organisms invade and take pharmacotherapy, the negative symptoms are often
over once the initial damage allows them to gain refractory.
entry and proliferate. All patients with a TOA should
be admitted for broad-spectrum antibiotic therapy. 00 Answer A. Dilated cardiomyopathy is most com-
Ampicillin, clindamycin, and gentamicin have been monly idiopathic. Viral myocarditis, chronic alcohol
the antibiotic combination of choice. Abscesses be- use, Chagas disease, and certain chemotherapeu-
tween 4 and 6 em in diameter respond to antibiotics tic agents (doxorubicin, daunorubicin) may also
alone 85% of the time. Abscesses > 10 em how- predispose to dilated cardiomyopathy. Tuberculous
ever, often require surgical intervention. If untreated, myocarditis, connective tissue disease, and familial

15
16 1(}()() Questions to Help You Pass the Emergency Medicine Boards

disease are less common etiologies. Cocaine usually are located at 5 and 7 o'clock around the vagi-
causes hypertrophic cardiomyopathy. nal introitus and are not palpable. Nonspecific in-
flammation or trauma may obstruct the glandular
[!] Answer C. This patient has olecranon bursitis. duct, however, resulting in accumulation of glandu-
Olecranon bursitis most commonly occurs because of lar fluid inside the gland and a Bartholin's gland cyst.
repetitive microtrauma caused by leaning or rubbing Asymptomatic cysts in young women do not require
of the elbow. Although most cases of olecranon treatment. Older women should have the cyst ex-
bursitis are sterile inf1ammatory reactions, septic cised by a specialist in order to search for glandular
bursitis may account for as many as 33% ofolecranon adenocarcinoma. All patients with an abscess should
bursitis cases. Furthermore, because ofthe superficial have incision and drainage followed by the place-
location, septic bursitis is most common in the ment of a Word catheter. The catheter is left in place
olecranon and prepatellar bursae, and rarely occurs for 6 to 8 weeks to allow formation of a fistulous
elsewhere. Predisposing factors to septic bursitis tract, which enables ongoing drainage and prevents
include patients who are immunocompromised recurrence. Bartholin's gland abscesses are almost
because of diabetes, renal insufficiency, and cancer always caused by polymicrobial vaginal flora. An-
as well as patients with anatomic abnormalities of tibiotics are unnecessary unless there is an associated
the joint spaces and surrounding structures such as cellulitis or unless sexually transmitted organisms are
patients with gouty or rheumatoid arthritis (both suspected. Cultures of the abscess can be obtained if
of which may involve the bursa). Aspiration of the there is concern about the etiology. (Figure courtesy
bursa is the only reliable means available to help of Mark Silverberg, MD. Reprinted with permission
differentiate between septic and sterile bursitis. In from Silverberg M. Greenberg's text-atlas ofemergency
contrast to the higher number of leukocytes in septic medicine. Lippincott Williams & Wilkins; 2004:350.)
arthritis, septic bursitis is suggested when the WBC
count exceeds only 10,000 per mm3 • Gram stain [H) Answer A. Patients with Guillain-Barre syndrome
is positive in only 50% of cases but Staphylococcus (GBS) always have some element of motor involve-
aureus is the most common responsible organism. ment. Purely sensory syndromes do not occur and
(Figure from Roberts JR. Olecranon bursitis: The rule out the diagnosis. Furthermore, motor involve-
technique of bursal aspiration. Emerg Med News. ment resulting in weakness is usually predominant
2002;24:20--21, with permission.) over sensory findings such as numbness or paresthe-
sias. Universal areflexia is the rule in patients with
~ Answer E. Lithium toxicity generally causes gas- GBS, although some patients have only distal are-
trointestinal (nausea, vomiting, abdominal pain), flexia with definite hyporeflexia of the biceps and
renal (diabetes insipidus), and neurologic (tremor, knee jerks. Patients with GBS commonly suffer au-
ataxia, coma) dysfunction. Management involves tonomic dysfunction, including the development of
whole bowel irrigation with polyethylene glycoL in- urinary retention during the evolution of symptoms.
travenous saline rehydration, and dial}'liis in severe Urinary retention is rarely persistent and the anal
cases. Activated charcoal does not bind lithium. sphincter muscles are also rarely affected. Although
Bicarbonate is used to treat tricyclic antidepres- the classic cerebrospinal fluid ( CSF) finding is an
sant overdoses. Glucagon is used to treat .f:l-blocker elevated protein level with a normal WBC count,
overdoses. Potassium chloride is not useful for man- 50% of patients will not experience an increase in
agement of lithium toxicity except in cases of severe protein level in the first week ofillness. By the second
hypokalemia. week of illness, 80% of patients have an elevated CSF
protein level.
[!!J Answer B. .f:I-Blockers are the best initial antihy-
pertensive agents to use in acute cardiac ischemia, ~ Answer D. Hyphema refers to the presence of
due to demonstrated reduction in mortality. Intra- blood in the anterior chamber of the eye. The eye
venous medications are most appropriate for rapid may re-bleed after the initial traumatic hyphema, es-
control of blood pressure--therefore, choices A, C, pecially in patients with severe myopia or with large
and E are not indicated. Choice D is an intravenous hyphemas. General treatment of traumatic hyphema
angiotensin-convertingenzyme (ACE) inhibitor, but involves analgesia and antiemesis, head elevation,
does not precisely reduce blood pressure and is diffi- restriction of eye movement, and avoidance of ther-
cult to titrate. apies that may cause bleeding. Ophthalmologic con-
sultation should be sought emergently, especially for
~ Answer D. This patient has a Bartholin's gland large hyphemas, as specific management with topi-
abscess, which is an infection of fluid that has ac- cal steroids or operative drainage may be instituted.
cumulated in the gland. Healthy Bartholin's glands There is usually no afferent pupillary defect.
Test 1 17

~ Answer E. The EKG demonstrates Wenckebach children. Gunshot wounds are much more likely to
phenomenon, or Mobitz type I AV block. The cause thoracic organ damage than serious chest wall
PR interval progressively increases in length and injuries.
predictably drops a QRS beat after the second pro-
longation. In the absence of symptoms of other ~ Answer C. Therapies such as angioplasty with
serious cardiac disease, management involves out- stenting have significantly reduced the mortality of
patient follow-up. Pacemaker placement would be acute Ml, causing a reduction in the overall mortality
required for patients with Mobitz type II second- due to heart disease. Ischemic heart disease is the
degree or third-degree AV block. Atropine is used most common cause of death in this country among
to treat hemodynamically unstable bradydysrhyth- all patients, including all women. Most Mls occur in
mias. Amiodarone may be used to treat a variety patients older than 65. Most deaths due to MI occur
of tachydysrhythmias, but is not indicated for pa- outside the hospital setting.
tients with AV blocks. Adenosine is used to abort
supraventricular tachycardias. ~ Answer B. This patient has necrotizing fasciitis
caused by Clostridium spp., which gained entry when
~ Answer B. Acute gastroenteritis is most commonly his arm was punctured while hiking. Clostridium
viral, but bacteria account for an important subset perfringens is the most common species and is
of cases. In the absence of severe electrolyte abnor- prevalent in soil. Pain is the most common early
malities, seizures associated with diarrhea are often finding and is generally intense and unremitting.
because of Shigella species, which are the most com- Swelling, pallor, and tenderness rapidly develop.
mon bacterial causes of acute gastroenteritis overall. Hemorrhagic bullae and brownish, serosanguineous
Salmonella is not typically associated with seizures. discharge may develop as the wound progresses.
Campylobacter and Yersinia species may cause an ill- Crepitus may also be present, but it is neither a
ness that mimics appendicitis. Vibrio species usually sensitive nor specific finding. Treatment is prompt,
cause a typical, nonspecific gastroenteritis. aggressive surgical debridement and intravenous
antibiotics. Despite this, amputation is frequently
[!lJ Answer E. Rheumatic fever occurs several weeks necessary. Use of advanced imaging to delineate
after untreated streptococcal pharyngitis. The di- spread along fascial planes delays definitive therapy
agnosis is made by the Jones criteria: Either two and may result in a worse outcome.
major (polyarthritis, erythema marginatum, chorea,
carditis, subcutaneous nodules) or one major and ~ Answer D. Trauma in the setting of submersion
two minor (arthralgias, fever, increased erythrocyte injuries is usually because of motor vehicle accidents
sedimentation rate [ESR] or CRP, prolonged PR in- (in which the vehicle crashes into water), or accidents
terval). Migratory arthritis ofmajor joints is the most involving diving, boating, or falls from a height into
common symptom, followed by carditis. Chorea and water. Although antibiotics are of use in patients
erythema marginatum are uncommon, but fairly who were submerged in grossly contaminated fluid
specific given a hlstory of antecedent pharyngitis. (e.g., sewage), they have no role in routine fresh
Despite its name, fever is not common in patients or saltwater submersion. Corticosteroids also have
with rheumatic fever. no role in drowning patients unless patients have a
history ofreactive airway disease and have evidence of
[!jl Answer C. The thoracic outlet syndrome com- bronchospasm on physical examination. Because this
prises a group of pathologic conditions associated patient was involved in a high-speed crash, trauma-
with compression ofthe structures at the junction of related injury should be the next most important
the upper extremity and trunk. The findings are neu- issue after ensuring an adequate airway, breathing
rologic (95%), venous (4%), and arterial (1%). The and circulation.
most commonly affected structure is the ulnar nerve.
~ Answer B. Only acetazolamide and dexametha-
~ Answer B. Nonintentional blunt thoracic injury sone have been shown to be effective for the
in children rarely causes multiple rib fractures, due prevention of acute mountain sickness (AMS). Ac-
to the compliance of the pediatric chest wall. For etazolamide is a carbonic anhydrase inhibitor, which
this reason, external injury is often absent. although prevents the reabsorption ofbicarbonate in the proxi-
pulmonary and cardiac injuries may be prominent. mal tubule. The resulting bicarbonate diuresis causes
Multiple rib fractures are usually caused by child a metabolic acidosis within 1 hour of ingestion,
abuse, especially rib fractures which are observed which stimulates ventilation and speeds ventilatory
radiographically to be in various stages of healing. acclimatization. In addition, acetazolamide reduces
Falls and sports injuries rarely cause rib fractures in periodic breathing during sleep, thereby eliminating
18 1(}()() Questions to Help You Pass the Emergency Medicine Boards

the associated apnea and hypoxia. Dexamethasone multiple sexual partners, smoking, and bacterial vagi-
has also been shown to reduce the symptoms of nosis. It is caused by Chlamydia, gonococcus, and
AMS, although its mechanism ofaction is not known. organisms, which cause bacterial vaginosis. The peak
Furthermore, patients can experience a rebound phe- time of onset is within 1 week of menses, as men-
nomenon upon stopping the drug so it is generally strual flow is thought to provide an optimal culture
reserved for the treatment of AMS rather than pro- medium for bacterial ascension. Symptoms include
phylaxis. Nifedipine is useful for prophylaxis of high diffuse pelvic pain, fever, nausea, vomiting, vaginal
altitude pulmonary edema (HAPE) but it has no discharge, and dyspareunia. Patients usually exhibit
role in AMS. Caffeine stimulates the hypoxic venti- bilateral adnexal tenderness with significant cervical
latory response but has not specifically been shown motion tenderness and cervical discharge. Labora-
to reduce the incidence of AMS. tory studies are more useful to rule out other causes
of symptoms as PID is a clinical diagnosis. Treat-
~ Answer C. Sympathomimetic and anticholinergic ment involves antibiotics to cover Chlamydia and
crises may be very difficult to distinguish on clin- gonococcU&--the most common regimen is IM cef-
ical grounds. Both toxidromes exhibit tachycardia, triaxone plus doxycycline/azithromycin. Sequelae of
delirium, mydriasis, and hyperthermia. The presence untreated PID are extremely serious, including tubal
of diaphoresis indicates that a sympathomimetic scarring causing infertility and ectopic pregnancy,
cause is far more likely, as patients with anticholin- chronic pelvic pain, and tubo-ovarian abscess {TOA).
ergic crises have inhibition of their sweat glands.
However, the absence of diaphoresis is not nec- ~ Answer D. Sternoclavicular dislocations may oc-
essarily diagnostic of an anticholinergic syndrome, cur in the anterior or posterior direction. Anterior
as patients with sympathomimetic crises may have dislocations are more common and occur because of
profound dehydration. Seizures may occur in either a medially directed force to the shoulder. These in-
toxidrome, although it is probably more common in juries are unlikely to cause serious injury beyond the
sympathomimetic states. dislocation itself. Posterior dislocations result from a
posteriorly directed force to the sternoclavicular joint
~ Answer E. The patient has dear evidence of and may cause great vessel, mediastinal, or airway
meningitis. The acute onset, progressive nature, and injury. CT is more accurate than either plain films
severe symptoms all indicate a likely bacterial cause. or ultrasound for the diagnosis of both anterior and
With evidence ofalteration in mental status, CT scan posterior sternoclavicular dislocations.
before lumbar puncture may be advisable. However,
in cases where bacterial meningitis is clinically sus- ~ Answer B. Hypoxemia is the most immediate life
pected, antibiotics should be administered before the threat to patients with COPD exacerbations. This
patient is sent to CT scan or lumbar puncture is per- patient's pulse oximetry of 85% indicates severe hy-
formed. Additionally, adult patients with suspected poxemia. Although an arterial blood gas (ABG) could
bacterial meningitis should receive dexamethasone be performed to verify this patient's hypoxemia and
with or before the first dose of antibiotics. to elucidate the degree ofC02 retention, it is clear that
what this patient needs is oxygen. A reasonable goal of
~ Answer B. Acetaminophen is metabolized by a va- oxygen therapy should be to titrate it to a saturation
riety of pathways, the most important of which of 90%. Early observational studies demonstrated
is through the cytochrome P-450 system, which that some degree of worsening hypercapnia usually
produces N -acetyl-p-benzoquinone imine (NAPQI) occurs following increased oxygen delivery to pa-
which is the toxic metabolite causing hepatocyte tients with COPD. This should not be confused with
necrosis. The drug N-acetylcysteine (NAC) reduces a decrease in the patient's respiratory drive. Instead,
the amount of acetaminophen metabolized by this increasing hypercapnia results primarily from wors-
route by replenishing glutathione, the reducing agent ened ventilation/perfusion (V/Q) mismatching and
which induces sulfation of acetaminophen to a non- because of the Haldane effect in which oxygenated
toxic compound. A single, acute dose of 7.5 g of erythrocytes have a decreased affinity for C02 , caus-
acetaminophen may be enough to overwhelm en- ing C02 offloading and an increase in blood C02
dogenous glutathione activity in an adult patient. The concentrations. Although this patient may eventually
lowest acute ingestion of acetaminophen requiring require intubation, it is prudent to try noninvasive
treatment with NAC in a child is 150 mg per kg. measures, including noninvasive ventilation before
endotracheal intubation.
~ Answer B. Pelvicinfiammatorydisease (PID) is an
infection of the fallopian tubes. Risk factors include ~ Answer C. Lyme disease is a multisystem illness
young age (15 to 25 years is the highest risk group), caused by the spirochete, B. burgdorferi, which is
Test 1 19

acquired by tick bites. The major endemic areas for obtaining an ultrasonograph is to search for an
are the northeastern and midwestern United States. ectopic pregnancy. Approximately 50% of women
The initial manifestation is the characteristic rash with an ectopic pregnancy have a IJ-hCG level <3,000
called erythema migrans, which starts near the tick miU per mL and symptomatic patients with a IJ-hCG
bite and spreads outward with central clearing. A level <1,000 miU per mL are four times more likely
few weeks after the onset of erythema migrans, to have an ectopic pregnancy than those patients
patients can develop nonspecific symptoms such with higher IJ-hCG levels. If the ultrasonograph is
as malaise, fatigue, and headaches. Cranial nerve nondiagnostic and the patient is otherwise stable, she
palsies, meningitis, and carditis may follow. Several can be discharged with instructions to follow-up with
months to years later, patients can exhibit chronic her OB in 2 days for a serial tJ-hCG level. In that case,
arthritis, persistent neurologic deficits, and chronic her diagnosis remains unclear, so she could be given
futigue. Seventh nerve palsy, which may be bilateral, a diagnosis of "possible ectopic pregnancy versus
is the most common neurologic physical exami- threatened abortion." Once her IJ- hCG level has risen
nation finding in Lyme disease. Lyme meningitis beyond the discriminatory zone, her ultrasonogram
is similar to aseptic meningitis, lacking the clinical should be repeated to evaluate for the presence of a
or laboratory findings of bacterial meningitis. First- normal pregnancy.
line therapy is with doxycycline (or amoxicillin for
pregnant/nursing women and children younger than ~ Answer B. The spleen and the liver (in that order)
8years). are the most commonly injured abdominal organs in
children with blunt trauma. Liver lacerations tend to
~ Answer A. Flumazenil is a benzodiazepine antag- have higher mortality than spleniclacerations. Unlike
onist that is used only in selected cases to reverse the past, splenic lacerations are currently nonopera-
benzodiazepine overdose. The only real indications tively managed as much as possible, because of the
for flumazenil are to speed recovery in accidental deleterious immunologic effects of splenectomy. Re-
pediatric ingestions and during procedural seda- nal injury is also common, given its proportionally
tion. Flurnazenil can precipitate seizures in patients larger size in children relative to adults. Bowel injury
who are chronic benzodiazepine users, alcoholics, in blunt trauma is rare.
and those who have co-ingested medicines which
lower the seizure threshold. The morbidity and mor- ~ Answer C. Locking or clicking of the knee is often
tality of benzodiazepine overdose is mostly from due to a meniscal injury, usually medial. Patients
respiratory depression. Therefore, standard airway may not recall a specific traumatic event and chronic,
management, oxygenation, and ventilation preclude repeated trauma may predispose to the injury. The
the use of flumazenil in almost all cases. medial meniscus is less mobile and therefore more
predisposed to injury than the lateral. Knee locking
~ Answer C. This patient has acute mountain sick- from medial meniscal tears may be relieved by
ness (AMS), which is characterized by the presence applying valgus stress and gentle extension.
of a headache (which is typically bitemporal or oc-
cipital) and at least one other symptom including ~ Answer B. Metacarpal neck fractures are common
gastrointestinal upset (nausea, anorexia, or vomit- fractures that are managed differently depending
ing), fatigue, dizziness, lightheadedness, or difficulty on the digit involved. Although most metacarpal
sleeping. The key principle in the management neck fractures have volar angulation, the hand can
of AMS is that further ascent is absolutely con- tolerate angulation to a certain extent due to mobility
traindicated. This is especially important because the at the carpometacarpal (CMC) joint. Up to 45
severity of the symptoms at onset cannot predict degrees of volar angulation is acceptable in the ring
the clinical course. If the symptoms are mild and and little finger metacarpals due to their increased
shelter is available, remaining at the current altitude CMC joint mobility. In contrast, the relatively fixed
or treatment with acetazolamide or dexamethasone index and long finger metacarpals permit only 15
are options. However, descent is the most effective degrees of angulation. Therefore, metacarpal neck
means of treatment. fractures of the index and long finger require
more accurate alignment. Rotational deformities
~ Answer C. Although this patient may have an early are poorly tolerated in all digits and almost always
normal pregnancy and a threatened abortion; she is require operative fixation. Reduction of metacarpal
at risk for an ectopic pregnancy. Because the patient's neck fractures is most easily accomplished in the
serum IJ-hCG level is below the discriminatory "90-90" method with the MCP joint and proximal
threshold (the level at which a normal pregnancy can interphalangeal (PIP) joint flexed to 90 degrees.
be detected by ultrasonography), the main reason Pressure in a volar direction is then applied to the
20 1(}()() Questions to Help You Pass the Emergency Medicine Boards

metacarpal shaft while simultaneous pressure in a ischemic stroke. Patients with blood pressure below
dorsal direction is applied to the flexed PIP joint this threshold do not require treatment.
Once reduced, metacarpal neck fractures should be
placed in a gutter splint. ~ Answer C. In the past, patients with signs of
intracranial hypertension after head trauma were
@11 Answer C. Exercise-induced asthma (EIA) is com- hyperventilated to PC02 <25 mm Hg in order to
mon, although estimates for the prevalence of this cause reflex cerebral vasoconstriction and reduced
entity vary widely, from 50% to 90% ofpatients with cerebral blood volume. However, more recent
asthma, as well as 40% of patients with allergic rhini- research suggests that reducing the PC02 levels
tis, and 11% of the general population. Although to below 30 mm Hg may cause cerebral hypoxia
patients with EIA frequently have symptoms during in many areas of the brain, potentially worsening
exercise, the peak of their symptoms generally occurs chances for neurologic recovery. The appropriate
5 to 10 minutes after exertion, and typically dissi- PC02 range for hyperventilation appears to be
pates after 30 minutes. If treatment is required, it between 30 and 35 mm Hg-this will result in
is typically required only to abort the acute symp- modest cerebral vasoconstriction without hypoxia.
toms after which the patient remains symptom-free. An arterial catheter is useful for measuring rapid,
Because both heat and water loss apparently induce serial blood gases to maintain the PC02 in this range.
bronchospasm, cold, dry air is more likely to cause
EIA Therefore, outdoor winter athletes (e.g., cross- ~ Answer B. As in patients with rheumatoid arthri-
country skiers, runners) are more likely to be affected tis, arthralgias and arthritis affect primarily the
than summer athletes (e.g., swimmers). Interestingly, hands and wrists and are symmetrical. Joint in-
through a poorly understood mechanism, patients volvement tends to be less severe in patients with
who have had an episode ofEIA frequently experience systemic lupus erythematosus (SLE), however, and
a 1-hour partial refractory period to further exacerba- joint deformities are less common. The classic
tions. Finally, although EIA can be exercise limiting, malar rash is only present in 40% of patients at
most patients can exercise fully without any ill ef- onset. but 50% of patients at any time throughout
fects. In fact, the prevalence ofEIA in US Olympians the disease course. Seizures are the most common
competing in the 1996 and 1998 Summer and Win- neurologic manifestation, and may be present in
ter Olympics has been reported as 16.7% and 22.4% up to 70% of patients. Stroke may occur, partic-
respectively. Prophylactic treatment with /h. -agonists ularly in patients with SLE and antiphospholipid
may help prevent symptoms from occurring during syndrome. Pericarditis is the most common cardiac
exercise. manifestation, present in 30% of patients. Libman-
Sachs endocarditis (sterile vegetations) is present
~ Answer A. The standard serum digoxin assay in 10% of patients. Drug-induced lupus normally
measures levels of all digoxin in the body, including resolves within days to weeks of stopping the of-
drug bound to Fab fragments. It is not useful to fending agent. Hydralazine and procainamide are
measure digoxin levels once Fab has been given. the most common offenders, although isoniazid can
Precedence should be given to dialysis to remove also result in drug-induced lupus.
the drug-Fab complexes. Cardioversion may be
performed in unstable patients, but is unlikely ~ Answer C. Benzodiazepines are recommended for
to be curative in patients with digitalis toxicity. sedation of patients in anticholinergic crises due to
Procainamide should be avoided in patients with their antiepileptic activity and absence of anticholin-
digitalis toxicity is it may exacerbate dysrhythmias. ergic activity. Neuroleptic agents may exacerbate
Calcium chloride should be avoided in patients seizures and anticholinergic symptoms. Etomidate is
with digitalis toxicity to prevent theoretical risk of too short acting for sedation due to agitation and
"stone heart," which occurs from massive calcium may cause rapid respiratory insufficiency. Ketamine
influx into cardiac myocytes causing sustained increases blood pressure and will exacerbate delir-
contraction. Potassium chloride should be given with ium, especially with its potential for the emergence
extreme caution in patients with digitalis toxicity, as phenomenon.
hyperkalemia is life threatening in this setting.
~ Answer D. CRPS (formerly known as reflex sym-
~ Answer A. Patients may be treated with labetalol pathetic dystrophy) refers to a chronic pain condi-
or nitroprusside in order to reach this blood pressure tion that occurs in an extremity following trauma
goal ifthey are initially more hypertensive. Answer D or surgery. Formerly a vaguely characterized pro-
represents the blood pressure limits that should cess, criteria have been established to make the
necessitate antihypertensive in any patient with diagnosis-allodynia (pain due to nonpainful
Test 1 21

stimuli), burning quality, bone scan and radio- ~ Answer B. Falls from buildings are an important
graphic abnormalities, and changes in sweating/ mechanism of multisystem blunt trauma. Feet-
temperature/hair/color. When an actual peripheral first falls tend to cause lower extremity, spinous,
nerve is demonstrated to be damaged, the condi- and pdvic injuries, the last of which results in
tion is known as CRPS-II. Treatment is controversial retroperitoneal bleeding. Mortality is rdated to the
and has traditionally been handled like most chronic height of the fall-half the number of patients
pain conditions. who fall from four stories die. Falls onto the
back commonly cause spinous and retroperitoneal
~ Answer B. Phencyclidine, or PCP, causes ex- injuries, including serious trauma to the kidneys.
treme dissociation, agitation, psychosis, and violent Calcaneal fracture often occurs in patients with feet-
behavior. Superhuman strength often occurs in first falls, but rardy by itselfcauses mortality. Instead,
patients with PCP intoxication, sometimes requir- it is a sign of other potential injury, including spinous
ing a dozen people to adequatdy restrain them. or pelvic fracture. Falls onto prone position can cause
Vertical or rotary nystagmus is a physical exam- serious abdominal and thoracic injuries and may
ination finding characteristic of PCP intoxication. easily result in death.
Cocaine intoxication may cause agitation, psychosis,
and mydriasis, but not nystagmus. Lysergic acid ~ Answer A. Management ofhydrocarbon toxicity is
diethylamine (LSD) is a typical hallucinogen, and generally supportive. Hydrocarbons most commonly
methylenedioxymethamphetamine (MDMA) or ec- cause pulmonary toxicity and cardiac dysrhythmias.
stasy is similar to a combination of a hallucinogen The most common scenarios are inhalation through
and amphetamine. Heroin causes a typical opioid "huffing" paint or glue cans and oral ingestion of
toxidrome, with constricted pupils, sedation, and hydrocarbons, followed by nausea, vomiting, and
respiratory depression. pulmonary aspiration. There is no evidence that
antibiotics, steroids, or diuretics improve outcomes
~ Answer B. Right ventricular infarction can be in hydrocarbon aspiration. Activated charcoal does
better assessed with a right-sided EKG, which is not bind hydrocarbons and should not be used.
the mirror image of a standard 12-lead EKG. The
right-sided V4 lead (rV4) is placed at the right fifth ~ Answer A. The lesion represents basal cdl carci-
intercostal space, mid-clavicular line. Elevation ofthe noma (BCC), which classically appears as a pearly
ST segment in this lead is the most specific for right white papule with raised borders and telangiectasias
ventricular infarction. Nitrates should be avoided in over the surface of the lesion. BCC is the most com-
patients with right ventricular infarction, as these mon form ofskin cancer, and most commonly occurs
patients are very dependent upon passive filling of on the face, with approximately one third of lesions
the right heart by the great veins (prdoad) due to the appearing on the nose alone. However, it also fre-
decreased active contraction of the right ventricle. quently occurs in relatively sun-protected areas such
Reducing the preload with nitroglycerin will reduce as behind the ears. In contrast, Squamous cell carci-
the passive filling and cause hypotension. noma (SCC), which also most commonly appears on
the head and neck, occurs in areas of maximal sun
~ Answer B. Botulism is a neurologic syndrome exposure.
caused by Clostridium botulinum, an anaerobic,
gram-positive bacillus that produces botulinum ~ Answer A. Intrapartum or antepartum asphyxia
toxin. Botulinum toxin is the strongest known bio- resulting in either global or focal brain ischemia
logic toxin, but is heat-labile, and can be inactivated is the most common cause of seizures in the term
by adequate preparation. Infant botulism, usually neonate. Intracranial hemorrhage accounts for ap-
spread by honey, is the most common form of bo- proximatdy 15% ofcases (most commonly intraven-
tulism, followed by food-borne botulism. Botulinum tricular or intraparenchymal). Sepsis, inborn errors
toxin blocks presynaptic acetylcholine release, caus- of metabolism, metabolic abnormalities (primarily
ing cranial nerve palsies, parasympathetic inhibition, hypoglycemia, hypocalcemia and hypomagnesemia)
and descending paralysis. The diagnosis is generally and toxins account for an additional 10%. Neona-
made clinically, with specific toxin assays to aid in tal seizures are rarely idiopathic, so an extensive
confirmation. Management involves aggressive air- diagnostic workup must be performed.
way evaluation and protection (due to pharyngeal
muscle weakness), monitoring of vital capacity and ~ Answer B. The classic neurologic deficit seen in
respiratory strength, and equine antitoxin. There is central cord syndrome is upper extremity weak-
little data regarding the efficacy of antibiotic therapy, ness greater than lower extremity weakness. This is
and currently antibiotics are not indicated.
22 1(}()() Questions to Help You Pass the Emergency Medicine Boards

because of the cervical motor axons being closer ~ Answer C. The most common cause of bacterial
to the midline than the lumbar motor axons. Large meningitis in middle-aged and older adults is
centralcordinjuriescaninitiallybeclinicallyindistin- pneumococcus. Group B streptococcus is common
guishable from complete cord syndromes. Anterior among neonates, Listeritl in extremes of age, and
cord syndrome results in deficits of bilateral motor meningococcus in young adults. H. influenzae used
function and pain/temperature sensation with spar- to be a very common cause of bacterial meningitis
ing of vibrationlposition sensation. Brown-Sequard among children, but the introduction of the HiB
syndrome, or cord herrrisection, results in deficits vaccine has drastically reduced the frequency of this
in ipsilateral motor function and vibrationJposition pathogen.
sensation and contralateral pain/temperature sen-
sation. Cauda equina syndrome, usually due to disk ~ Answer D. Children have the lowest mortality
herniation, preferentially affects the lower extremities from status epilepticus, whereas elderly adults have
and boweVbladder function. Complete cord injury the highest mortality. In the most widely cited study
affects all neurologic functions below the level of of status epilepticus to date {the "Richmond" study),
injury. overall mortality was 3% in children younger than
16 years, and 38% in elderly adults older than 60.
~ Answer C. Epinephrine dosing remains a confus- Although lorazepam is favored by many physicians,
ing topic, largely because guidelines vary in their there is no clear evidenced-based drug of choice. In
nomenclature. To make things simple, it is easy the veterans affairs (VA) cooperative study, there was
to remember that 1:1,000 doses are always given no statistically significant difference when lorazepam
subcutaneously, whereas 1:10,000 doses are given in- was compared with phenobarbital, diazepam plus
travenously. The concentration of a 1:1,000 solution phenytoin, or phenytoin alone as frontline agents.
of epinephrine is 1 mg per mL, whereas the concen- There was only a statistically insignificant trend in
tration of a 1:10,000 solution is 0.1 mg per mL {i.e., favor of lorazepam. The most common side effect
ten times less concentrated). Therefore, 0.3 mL of a of benzodiazepines given for status epilepticus is
1:1,000 solution is 0.3 mg. Delivery of 0.2 to 0.5 mg hypotension. Hypoventilation is the second most
of epinephrine subcutaneously is an adequate dose common side effect. Subtherapeutic drug levels are
in the setting of refractory asthma. This dose may be the most common trigger of status epilepticus and
repeated every 20 to 30 minutes up to three doses as status epilepticus due to subtherapeutic drug levels
with terbutaline. has a low mortality. The next most common cause
of status epilepticus among adults is cerebrovascular
~ Answer D. Achalasia is an esophageal motility accidents (CVA).
disorder due to failure of the lower esophageal
sphincter to relax and a complete absence of ~ Answer B. Syncope with aortic dissection man-
esophageal peristalsis. It affects men and women dates evaluation of the pericardia! space to check
equally and presents between the third and fifth for hemopericardium and tamponade. Dissections
decade. Dysphagia is the most common symptom. extending back to the pericardium are ascending and
All patients have difficulty with solid foods but two should be taken emergently to surgery. The most
thirds of patients describe dysphagia with liquids common symptom of aortic dissection is chest pain,
as well. Patients may stand after eating, raise their although dyspnea is often seen as well. Aortic re-
arms above their head or straighten their back to gurgitation is not usually seen in aortic dissection,
increase esophageal pressure and help esophageal but indicates a valvular disruption when present.
emptying. Patients with achalasia have symptoms Dissection into the coronary arteries occurs rarely,
for an average of 2 years before diagnosis and they but when it does, the right coronary artery is most
are often treated for GERD due to the presence of frequently affected. Interarm differences in blood
burning chest pain. Diffuse esophageal spasm and pressure due to unilateral extension ofthe dissection
nutcracker esophagus are hypermotility disorders into the subclavian artery occur in the minority of
resulting in exceedingly strong esophageal peristaltic patients with aortic dissection. However, in patients
contractions. The most common complaint is chest with undifferentiated chest pain, interarm differences
pain, although the presence of dysphagia is less of> 20 mm Hg indicate a higher likelihood of aortic
common and more variable. Schatzki's ring is a dissection.
fibrous band-like stricture in the distal esophagus that
is present in up to 15% of the normal population. ~ Answer B. In normal adults, the postvoid residual
Patients who develop symptoms typically present volume should normally be <50 mL. Values above
with acute esophageal obstruction. 200 mL are considered definitely abnormal Values
Test 1 23

between 50 and 200 mL require clinical correlation. be predisposed to develop mucormycosis. Interest-
The normal postvoid residual volume increases with ingly, affected patients without diabetes but with
age, butdoesnotriseto >50to lOOmL. Thepostvoid another predisposing risk factor have much higher
residual volume should be checked in all patients with mortality rates from 60% to 80%.
back pain and any urinary complaints. Although
the most common finding in patients with epidural ~ Answer A. Acetaminophen is metabolized by
spinal cord compression is motor weakness in the choices A through D. The major route is glucuronida-
lower extremities, occasionally patients may present tion catalyzed by UDP-glucuronyl transferase in the
with isolated urinary complaints. Typically, however, liver. Sulfation is the next most common route and
urinary retention or incontinence is a late finding of is the target mechanism for N-acetylcysteine (NAC)
spinal cord compression. therapy. Oxidation by cytochrome P-450 results in
the formation of NAPQI, which is responsible for
~ Answer D. Dihydroergotamine (DHE) is a nore- the hepatic necrosis caused by acetaminophen over-
pinephrine reuptake inhibitor at sympathetic nerve dose. Direct renal excretion represents approximately
endings, which results in potent venoconstriction, 5% of the metabolism of acetaminophen. Plasma
and it is an agonist at multiple serotonin receptors metabolism of acetaminophen does not generally
(5-HTlA-F). The half-life of DHE and its metabo- occur.
lites is roughly 72 hours compared with the shorter
duration of the triptans, which have a half-life of [i!J Answer E. As with many problems, the incidence
3 to 6 hours. Therefore, DHE is particularly use- of the varying underlying causes of hemoptysis in
ful for the treatment of patients with chronic daily children is dependent on the specific population
migraine as well as status migraine, which is de- which is studied. In tertiary care hospitals which
fined as any migraine headache lasting longer than serve as major referral centers for congenital heart
72 hours. The major side effect associated with DHE disease, this will be a much more frequent cause of
is nausea, so pretreatment with an antiemetic such hemoptysis encountered by physicians. Still, cystic
as prochlorperazine or metoclopramide 30 minutes fibrosis is, by far, the most common cause of
before administration is recommended when given hemoptysis in the pediatric population, responsible
in the hospital. for approximately 65% of cases in tertiary centers.
Other common causes include upper airway bleeding
~ Answer A. Mucormycosis (also known as zy- from infected tonsils or adenoids, which was thought
gomycosis) is a rapidly invasive opportunistic infec- to represent hemoptysis, as well as foreign body
tion that may affect numerous organ systems in the aspiration. Infectious causes such as TB are much
host. However, the most common entity it causes is less common in the United States, although it may
rhinocerebral mucormycosis (RCM). It is acquired be prevalent in the developing world. Congenital
by inhalation of the spores (ubiquitous in soil, de- cardiac disease is an uncommon cause, as is trauma,
caying fruit and vegetables, old bread) followed by whereas tumors and pulmonary-renal syndrome are
rapid invasion of the vasculature. Because of its quite rare.
propensity for invading vascular structures (i.e., an-
gioinvasive), infarction of tissues dependent on the ~ Answer C. Abdominal pain and diarrhea occur
affected blood supply frequently ensues. The result- in practically all cases of Shigellosis, and is of-
ing necrotic mucosa appears black to the examiner, ten accompanied by fever. However, only 35% to
and it may be friable and insensate as well, but 40% of patients have evidence of blood in their
no bleeding is present because of vascular infarc- stools. Resistance to trimethoprim-sulfamethoxazole
tion. Although a black eschar is the classic finding is widespread, making fluoroquinolones the drug of
of RCM, it is present in less than half the num- choice. Antimotility drugs may be safely given if
ber of patients. Furthermore, its presence portends antibiotics are also administered but are contraindi-
a rapidly progressive course with a poor progno- cated when used alone as they may actually worsen
sis. Urgent surgical debridement and intravenous the clinical course. Most infections are caused by
antifungal therapy are required to prevent intracra- Shigella sonnei, with only a minority of infections
nial extension and further tissue destruction. With caused by Shigella dysenteriae. Although Salmonella
such early intervention, mortality rates are now spp. requires a very large inoculum to cause disease,
<20%. RCM most commonly occurs in diabetic infection with Shigella spp. requires a very small
patients with very poor glycemic control. How- inoculum, making Shigella the most efficient en-
ever, transplant recipients, as well as patients un- teric human pathogen known. Fecal leukocytes are
dergoing deferoxamine treatment, or patients with almost universally detected in patients with Shigel-
renal failure, extensive burns or trauma, may all losis, as Shigella invades the mucosa resulting in local
24 1(}()() Questions to Help You Pass the Emergency Medicine Boards

destruction and inflammation. Seizures may occur Harris JH. The radiology of emergency medicine, 4th
in children infected with ShigeUa. ed. Lippincott Williams &Wilkins; 1999:547.)

~ Answer A. The patient is a young adult with clin- ~ Answer B. Inadequate sedation of recently intu-
ical evidence of community-acquired pneumonia. bated patients is a common problem in the ED.
In a young patient without serious comorbidity Although the common initial response to the wak-
or severe distress, outpatient therapy is appropri- ing patient is to infuse paralytics, such action is
ate. Major pathogens in this age-group include typically not necessary. Usually, all that is required
pneumococcus, Mycoplasma pneumoniae, Chlamy- is adequate sedation with benzodiazepines with or
dia pneumoniae, and others. Appropriate therapy without narcotic agents such as morphine or fen-
could include a macrolide, a second- or third- tanyl. Paralytics should be considered in the patient
generation cephalosporin, a fluoroquinolone, or with persistently high plateau pressures despite op-
doxycycline. Doxycycline is inexpensive, covers most timal ventilatory management and sedation. In such
organisms implicated in community-acquired pneu- patients, paralysis ensures that the patient's intrinsic
monia, and has convenient twice-a-day dosing. Ilne- respiratory effort will not interfere with the ven-
zolid is used to treat vancomycin-resistant organisms, tilator's attempts at delivering breaths. However,
most often seen in the nosocomial setting. Cephalexin because paralysis increases the risk of aspiration
is a first-generation cephalosporin with poor and subsequent pneumonia, paralytics should not
coverage of atypical organisms and gram negatives. be routinely used. Hyperkalemia is not likely to cause
Clindamycin provides excellent coverage of gram the patient's agitated mental status.
positives and anaerobes, but does not cover atypicals
or gram negatives. Piperacillin-tazobactam is a po- [!Zl Answer A. The most likely diagnosis is tumor
tent, broad-spectrum, anti-pseudomonal antibiotic lysis syndrome, a constellation of events that re-
used only for nosocomial pathogens causing severe sults from rapid cellular death due to chemother-
illness. apy. Rapidly growing and hematologic malignancies
highly responsive to chemotherapy are at highest
~ Answer E. Koplik's spots are irregularly shaped risk for development of tumor lysis syndrome. Hy-
bright red macules that contain a central, punctate, perkalemia, hyperuricemia, and hyperphosphatemia
bluish-white spot. Their presence is pathognomonic are the most common laboratory abnormalities.
for measles. They are most commonly found on the Hypocalcemia is more common than hypercalcemia.
buccal mucosa and are classically located opposite Severe fluctuations in magnesium levels are rare.
the second molars. Renal insufficiency and dysrhythmias are the most
serious complications. Management involves nor-
~ Answer B. This patient has aspiration pneumoni- malization of electrolyte abnormalities (especially
tis, which is a chemical pneumonitis caused by the hyperkalemia), intravenous fluids, and treatment
aspiration of acidic gastric contents. The severity of of renal insufficiency. Alkalinization therapy to
lung injury increases as the pH drops, and most counteract hyperuricemia is not universally recom-
studies agree that the pH must be <2.5 to cause sig- mended, as it may exacerbate electrolyte abnor-
nificant injury. Chemical aspiration most commonly malities. Renal failure requiring dialysis is a poor
occurs in patients who have depressed levels of con- prognostic indicator.
sciousness, as in this patient, and usually occurs in
young persons. Aspiration is otherwise a common ~ Answer E. Because Pneumocystis cannn cannot
problem among elderly persons in nursing homes af- be cultured, and the gold standard in making
ter strokes. Though they are widely used, antibiotics the diagnosis remains invasive (bronchoscopy and
and corticosteroids are not routinely recommended subsequent staining), there has been much interest in
in patients with aspiration pneumonitis. Gastric acid trying to find surrogate serum markers to indicate the
inhibits bacterial growth so gastric contents are ster- presence of Pneumocystis carinii. Most ofthis interest
ile under normal conditions. Unless the patient has has focused on LDH. Unfortunately, although it is
a coexisting condition promoting gastric bacterial true that LDH levels are elevated in the setting ofPCP,
growth, antibiotics should not be used. Such con- this finding is not specific for PCP. A recent study
ditions include patients on antacid therapy, patients found that the average LDH level was 1,217 ± 88 U/L
receiving enteral feeding, and patients with known in patients with PCP compared with 776 ± 55 U/L
gastroparesis or small bowel obstruction. Corticos- in patients with non-PCP (p <0.001). However, in
teroids are also widely used in patients with chemical this study, the LDH level was not clinically useful
aspiration, although they have not been proved to be in differentiating between PCP and non-PCP at
beneficial. (Figure reprinted with permission from any particular level. Instead, the investigators found
Test 1 25

that the level of LDH correlated with the degree Enterobius may cause urinary tract infections and
of radiographic severity. This supports the idea vulvovaginitis through retrograde migration into the
that elevated LDH levels are more a reflection of urethra or vagina. Interestingly, girls with urinary
lung inflammation than for any particular organism. tract infections are twice as likely to have a concomi-
Lactate levels and liver transaminases have no role tant pinworm infection.
in the diagnosis of PCP. Most patients with PCP
and hypoxia have a respiratory alkalosis, as their [!!) Answer E. Patients with Reiter's syndrome fre-
respiratory rate (and depth) increases with the quently have conjunctivitis earlyin the disease course.
severity of their hypoxia. Finally, patients with CD4 Uveitis (or iritis) may also occur but is less common
counts < 200 are most susceptible to PCP and it is this and unrelated. Roughly 10% of patients will de-
population who should be on chemical prophylaxis. velop keratoderma blenorrhagica, waxy plaques most
Pleural effusions are not common in PCP. commonly present on the palms and soles. Sausage-
shaped inflammation of the digits (dactylitis) is
~ Answer C. A positive Nikolsky's sign occurs another common occurrence in Reiter's syndrome.
when pressure applied to the margin of a blis- Finally, painless ulcers may develop in the mouth
tered or ulcerated lesion expands the lesion into or on the penis (where they are called balanitis
the adjacent apparently normal skin. This is also circinata), where they more frequently occur in un-
known as marginal modification. In addition, di- circumcised men.
rect pressure applied to normal-appearing skin that
is distant from any blistered lesions may also re- ~ Answer A. This is known as splitting the fourth
sult in an erosion or ulceration. This is known as digit and it represents the dividing line between
direct modification. These findings occur because median and ulnar innervation to the ring finger.
of intraepidermal acantholysis (separation of ker- The median nerve also serves the "LOAF" muscles,
atinocytes from their neighbors within the epider- which include the lumbricals, as well as the muscles,
mis). Nikolsky's sign is most commonly associated which allow thumb opposition, abduction and
with pemphigus vulgaris, but may also be found in flexion. However, the hallmark of CTS is sensory
staphylococcus scalded skin syndrome (SSSS) and involvement, with motor abnormalities developing
toxic epidermal necrolysis (TEN). There are also later. The most sensitive finding for CTS is abnormal
multiple reports of other disease associations but sensation of the distal palmar tip of the index finger,
the finding (particularly of direct modification) is as this represents the autonomous zone ofthe median
very specific for pemphigus vulgaris. nerve (the area where there is no overlap with other
cutaneous nerves). Tinel's sign is the presence of
[!g) Answer B. This patient is infected with the com- distal paresthesias in the setting of median nerve
mon pinworm, or Enterobius vermicularis. It is percussion at the wrist. In the absence of sensory
probably the most common parasitic infection in or motor symptoms, Tinel's sign has inadequate
the United States. The most common clinical man- sensitivity and specificity to guide referral for further
ifestation is pruritus ani. However, most infections specialized testing.
are asymptomatic. Adult worms are white colored,
are approximately 1 em in length and live in the ~ Answer C. The patient most likely has myocardi-
cecum. At night, pregnant female worms containing tis, which in the United States is usually because
an average of 10,000 ova migrate to the perianal skin, of viruses, most commonly Coxsackie B. A viral
deposit their eggs, and die. The resulting pruritic prodrome usually precedes overt signs of cardiac
sensation induces the patient to scratch and enables involvement, such as chest pain or signs of heart
further autoinoculation or spread to other persons fuilure. No common ED laboratory or imaging
unless the patient engages in proper hand washing study is helpful in making the diagnosis of my-
before touching others. The most sensitive test is the ocarditi&-antimyosin scintigraphy or more invasive
"scotch tape" test in which tape attached to a tongue endomyocardial biopsy is indicated. Coronary artery
blade is pressed against the perianal skin in an at- disease is uncommon in patients ofthis age in the ab-
tempt to affix some of the Enterobius ova to the tape. sence of risk factors. Aortic dissection is uncommon
The contents ofthe tape are then spread on a slide and in the absence of trauma or history of hyperten-
viewed under a microscope in toluene. Stool sam- sion or Marfan's disease. Stroke is not suggested by
ples for ova and parasites are not effective because the symptoms, and diabetic ketoacidosis (DKA) is
the organism is not shed in the stool. Metronidazole unlikely with normal glucose and bicarbonate.
is not effective for treatment. Instead, treatment is
with albendazole, mebendazole, or pyrantel pamoate. [B) Answer E. Intimate partner abuse {IPA) encom-
Eosinophilia generally does not occur. On occasion, passes the following old terms-domestic violence,
26 1000 Questions to Help You Pew the Emergency Medicim Boards

domestic abuse, and spousal abuse. It is likely to be assessing structural cardiac abnormalities. Patients
vastly underreported by both men and women. ED are at high risk of developing peripartum cardiomy-
patients are at high risk for being victims of IPA, but opathy in subsequent pregnancies. Aspirin plays no
that large majority of patients will not report IPA role in management, although patients may benefit
unless specifically asked to by a care provider in the from heparin prophylaxis due to the thromboem-
absence of their intimate partners. Screening of all bolic risk.
patients is the only way to accurately evaluate IPA
among ED patients. Most violence against women is [!iJ Answer C. Choice C is also known as MDMA,
perpetrated by intimate partners, but most violence or ecstasy. It is in a newly assigned class of drugs
against men is perpetrated by strangers. Almost one known as entactogens. These drugs have properties of
fourth of women and 10% of men ofall sexual orien- both hallucinogens and amphetamines, causing mild
tations report an incident of IPA. Victim substance hallucinations, increased interpersonal emotions,
abuse is a marker for IPA, but is not thought to and stimulatory neurotransmitter release. Ecstasy
contribute to IPA-perpetrator substance is known is commonly ingested at raves and other dance
to contribute to IPA. parties. Pathophysiology includes sympathomimetic
effects, hyperthermia, and hyponatremia (both from
~ Answer B. Elderly white men have the highest a central antidiuretic hormone (ADH)-like effect
rate of completed suicide, representing more than and increased thirst, causing water consumption).
three fourths of all suicide deaths, and women have Choice A is lysergic acid diethylamine (LSD), which
the highest rate of suicide attempts. White patients is not as commonly used currently as it was in
are more likely to commit suicide than blacks or the 1960s to 1980s. Choice B is PCP, causing an
Hispanics, and nonpregnant women ofchild-bearing unpleasant psychosis and violence. Choice D is a
age are more likely than pregnant women to do so. selective serotonin reuptake inhibitor (SSRI) and
Divorced patients have higher rates than unmarried choice Eisa monoamine oxidase (MAO) inhibitor,
patients, who, in tum, have higher rates than married neither of which is used recreationally.
patients. Most successful suicide attempts involve
firearms and most unsuccessful attempts involve ~ Answer A. Open pelvic fractures, whether through
drug ingestions. The presence of a firearm in the the skin, vagina, or rectum, always require definitive
house is an independent risk factor for completed operative management. Rectal violation of pelvic
suicide and the patient should be directly asked about fragments requires laparotomy with colostomy. Bed-
this on history. Substance abuse, especially alcohol side packing and antibiotics is indicated emergently
and cocaine, is extremely common in patients who but does not constitute definitive care. Angiography
complete suicide. Patients who present to the ED with with embolization is useful in cases oflife-threatening
attempted suicide must be evaluated for medical hemorrhage from unstable pelvic fractures, but has
illness that may masquerade as mood disorder or no role in the management of stable, open fractures.
thought disorder leading to the suicide attempt. Bedside irrigation and suturing are not indicated.
Roughly 20% of patients with major depression and Foley catheter placement should be withhdd in cases
10% of patients with schizophrenia commit suicide. ofpelvic fracture until physical examination demon-
strates a low risk of urologic injury (absence of
(!!] Answer C. Fat, tendons, and bones have the blood at the urethral meatus, perineal hematoma, or
greatest resistance to electrical flow (bones have high-riding prostate).
the highest resistance of all) whereas nerves, blood,
mucous membranes, and muscle have the least [jQJ Answer E. H. pylori is found in roughly 90% of
resistance. The resistance of dry skin is intermediate, patients with duodenal ulcers and 70% to 90% of
although it varies greatly depending on the skin patients with gastric ulcers. NSAID use, cigarette
surface involved. Tissues with high resistance tend to smoking and shock states are also risk factors for
heat up and coagulate in response to electrical flow. peptic ulcer disease. Although 200 proof alcohol has
been shown to cause damage to the gastric mucosal
(llJ Answer D. The patient has peripartum cardiomy- epithelium in animals, there are no clinical studies
opathy, which occurs in approximately 0.03% of all demonstrating a link between alcohol use in humans
pregnancies and almost one third of patients die. and peptic ulcer disease. In filet, one study suggests
Risk factors include greater maternal age, multipar- a protective effect from modest alcohol ingestion.
ity, and twin pregnancies. Clinical characteristics and H. pylori infection is the most ubiquitous risk factor
acute management are the same as that of congestive for peptic ulcer disease, regardless of the location
heart failure due to a dilated cardiomyopathy. ECG, of the ulcer. Eradication of H. pylori reduces ulcer
not cardiac catheterization, is usually the first step in recurrence from 75% iftreated with acid-suppression
Test 1 27

therapy alone to <20% in patients who undergo a importance and active external warming has not
course of anti-Helicobacter therapy. It is estimated been shown to worsen the degree of afterdrop. Al-
that 50% of the U.S. population is colonized by though the mechanism of afterdrop is still unclear,
H. pywri by the age of 50. However, only 15% to the leading current theory is based on conductive
20% of patients colonized with H. pylori will develop heat transfer from the warm core to the cool pe-
peptic ulcer disease over their lifetime. Therefore, riphery. Despite the initiation of rewarming, a heat
other factors must be at work. In addition, details gradient remains from the relatively warm core to
regarding different strains of H. pylori and their the cool periphery. Until this gradient is eliminated,
relative virulence are still being worked out. heat transfer continues and the core temperature
drops.
[j!) Answer E. Patients with spinal stenosis are often
elderly and constitute a small minority of patients ~ Answer B. Bupropion is an atypical antidepres-
with low back pain. Most patients present with sant with dopamine reuptake inhibitory properties.
subacute or chronic pain that frequently mimics Seizure is the most likely severe effect in overdose and
symptoms of vascular claudication (often termed can occur up to 24 hours after ingestion. Seizures can
neurogenic intermittent claudication). This is prob- even occur in patients who are taking therapeutic
lematic because vascular claudication strikes the same doses of bupropion. They cannot be predicted by
population. Symptoms of neurogenic intermittent clinical or laboratory data and patients with bupro-
claudication include buttock pain, which frequently pion overdose should be monitored for 24 hours
radiates to the thighs and lower legs as well as cramp- postingestion. Coma, dysrhythmia, hypotension, and
ing, paresthesias, back pain, and difficulty walking. In hypertension do not occur with bupropion and sug-
contrast to patients with vascular claudication, how- gest coingestion of another drug.
ever, patients with spinal stenosis typically maintain
a forward-leaning posture to reduce symptoms. In ~ Answer C. Anterior cord syndromes cause bilat-
addition, patients are better able to walk uphill than eral deficits in motor function and pain/temperature
downhill. Sitting also improves symptoms, which sensation below the level of injury. The lateral
helps to differentiate patients with spinal stenosis spinothalamic and the corticospinal tracts carry
from patients with disc herniation. Although pa- pain/temperature and motor fibers, respectively. The
tients may have involvement at several lumbar levels, dorsal columns, carrying vibration, position, and
15 is most commonly involved (75%) followed by fine touch sensation, are commonly spared. Cranial
L4, L3, and U. nerves arise directly out of the brainstem and are not
commonly injured in cord syndromes except in rare
~ Answer D. The patient has evidence of necrotiz- circumstances. Causes of anterior cord syndrome
ing fasciitis of the perineal area, commonly referred include anterior spinal artery injury or infarction,
to as Fournier's gangrene. Fournier's gangrene is a spinal cord trauma, and intervertebral disk herni-
systemic, life-threatening, polymicrobial infection, ation. Prognosis is not as good as other causes of
which requires intravenous antibiotics and surgical incomplete cord injuries.
debridement. Intravenous immunoglobulin and hy-
perbaric oxygen therapy may be helpful in certain [!!) Answer A. The rash is petechial in nature and
cases. Corticosteroids are not indicated except in combined with the history of starting on the
cases of concomitant adrenal insufficiency. Antibi- wrists and ankles along with the nonspecific viral
otics without surgical debridement are not sufficient symptoms indicate the diagnosis of Rocky Moun-
for management. The diagnosis is usually made tain spotted fever (RMSF). This tick-borne illness
clinically-in unclear cases, CT, or MRI may aid the is caused by R. rickettsii. Treatment of RMSF is
diagnosis but neither is 100% sensitive or specific. with doxycycline and admission to the hospital.
(Figure from Isaacs L. Necrotizing fasciitis: Diagnosis Borrelia burgdorferi is the causative bacterium of
and treatment. Emerg Med News. 2002;24(8):4, with Lyme disease. Coxsackievirus causes herpangina and
permission.) myocarditis. Gonococcus may cause a vesicular rash
in association with septic arthritis, cervicitis, or pelvic
~ Answer E. Core temperature afterdrop refers to inflammatory disease (PID). Meningococcus may
the phenomenon of a further decline in core tem- cause a petechial rash with signs and symptoms of
perature after the initiation of warming. In the past, meningitis, but the onset is much more acute and
it has been blamed for the development of cardiac severe than RMSF. (Figure from Habif TP. Clinical
dysrhythmias and active external rewarming meth- dermatology: A color guide to diagnosis and therapy.
ods were thought to amplify the drop. However, Philadelphia: Mosby; 2004, with permission.)
afterdrop has not been shown to be of any clinical
28 1(}()() Questions to Help You Pass the Emergency Medicine Boards

~ Answer B. Cerebral herniation syndromes are a of the amputation are encouraging for replantation.
result of severely increased ICP, usually because of Finally, all patients with an amputation should wrap
trauma. Herniation occurs when the ICP causes the amputated digit in saline soaked gauze, place the
downward movement of various parts of the brain gauze in a ziplocked bag and place the bag on ice
through the foramen magnum. Mortality is virtually or in an ice-water bath. Freezing the digit should be
100% in untreated patients. Uncal herniation is the avoided as it results in irreversible damage to cellular
most common type, and results from a lateral injury, structures due to ice crystal formation.
causing the ipsilateral temporal lobe to be com-
pressed against the tentorium cerebelli. The initial ~ Answer B. The majority of completed suicides
clinical manifestation is oculomotor nerve injury, among both men and women involve firearms. The
causing ipsilateral ptosis, oculomotor dysfunction, presence of a firearm in the house is an independent
and mydriasis. As the uncal herniation progresses, risk factor for completed suicide and the patient
contralateral hemiparesis occurs. Eventually, the con- should be directly asked about this on history. Drug
tralateral uncus is compressed, leading to bilateral ingestion, usually with antidepressants, is the most
decerebrate posturing, and the brainstem herniates, common method ofsuicide attempts, and the second
causing respiratory failure. In central transtentorial most common method of completed suicide by
herniation, an expanding midline lesion causes bilat- women. Carbon monoxide poisoning is employed
eral pinpoint pupils, positive Babinski reflexes, and less often. Hanging is the second most common
decorticate posturing. Cerebellotonsillar herniation method of completed suicide by men. Wrist cutting
results from a cerebellar hematoma, which causes the almost never results in completed suicide.
sudden onset ofpinpoint pupils, and respiratory and
cardiovascular compromise with complete, bilateral [!!) Answer A. Nitroglycerin dilates the great veins and
paralysis. Brainstem and medullary herniation is the reduces preload. An acute reduction in preload may
common endpoint for all herniation syndromes and cause hypotension from decreased cardiac filling.
is manifested by respiratory arrest. Appropriate therapy involves fluid resuscitation
and avoidance of further preload-reducing agents.
I!!] Answer D. The most common abnormal finding Choices B, C, D, and E are all vasoactive agents that
on CT scan in elderly patients with head trauma is may be indicated if fluid therapy alone does not
cerebral contusion. Most of these patients have mild improve hypotension caused by nitrates.
intraparenchymal capillary damage and are usually
managed nonoperatively, with serial CT scans and ~ Answer A. The single most common identifiable
neurologic exams to assess for deterioration. Epidu- cause of seizures in human immunodeficiency virus
ral hematomas are less common in the elderly than (HIV)/AIDS patients is toxoplasmosis, which causes
in younger adults, as the dura is tightly adherent seizures through mass effect. Other mass lesions
to the skull and does not usually expose arteries such as malignancy and abscess are additional
to shear forces. Subdural hematomas become much causes. In cases that are not due to any identifiable
more common as patients get older because of brain cause, HIV-encephalopathy is postulated to be the
atrophy, which stretches cranial bridging veins. Sub- etiology. Meningitis, usually cryptococcal, is the
arachnoid hemorrhage and cerebellar hematomas are second most common identifiable cause. Electrolyte
less commonly seen in this population. abnormalities, stroke, and neurosyphilis are less
commonly implicated.
!!!] Answer A. In general, indications for replantation
include multiple digit amputations, thumb ampu- ~ Answer C. Low molecular weight heparins
tations, wrist and forearm amputations, single digit (LMWHs) such as enoxaparin have been developed
amputations between the PIP and DIP joints, and as a less-frequent dosing alternative to unfraction-
any pediatric amputation. Amputations distal to the ated heparin, which requires a constant IV drip and
DIP joint are typically debrided and closed along with dosage monitoring. A major advantage ofLMWHs is
amputations proximal to the PIP. Amputations that their ability to be used in outpatient anticoagulation
are between the PIP (distal to the flexor digitorum regimens. The major mechanism ofLMWHs is to in-
superficialis insertion) and DIP joints tend to do well. hibit factor Xa, causing a more upstream inhibition
However, single digit amputations are often consid- of the clotting cascade than unfractionated heparin,
ered unnecessary to replant. In addition, patients which binds to antithrombin III to inactivate throm-
with underlying vascular disease, diabetes, conges- bin (factor II). Inactivation of thrombin represents
tive heart failure, or other medical problems may not an additional, minor mechanism of LMWHs. Hep-
have a good outcome even when the characteristics arins do not function as direct platelet inhibitors,
Test 1 29

although they can cause thrombocytopenia as an un- present, an ectopic pregnancy is presumed to be
intended side effect. Heparins do not directly affect present and treated.
von Willebrand factor or factor XIII.
~ Answer C. In the setting of trauma, >90% of
~ Answer A. The clinical course of Guillain-Barre patients with a "posterior fat pad" sign have an intra-
syndrome (GBS) is more severe in the elderly. In articular elbow injury. The most probable injury in
children with the disease, death is the exception, and children is a supracondylar fracture, whereas radial
rapid recovery is the rule. Overall, the mortality rate head fractures are the most common entity in adults.
is between 4% and 15%, although 20% of survivors Although a small anterior fat pad is a common finding
have some residual disability. Approximately 25% in healthy patients, a posterior fat pad is always an
of patients have a preceding C. jejuni infection, abnormal finding.
and these patients typically experience a more
severe course and delayed recovery. Autonomic ~ Answer C. ARF is a nonsuppurative complication
involvement, resulting in urinary retention, ileus, of GAS pharyngitis that typically occurs 2 to 4 weeks
postural hypotension, sinus tachycardia, and cardiac after the initial pharyngeal infection. Its onset is
dysrhythmia is common and respiratory failure is characterized by the presence of fever plus one or
more common in this group of patients. Ultimately, more of the five "major" manifestations, including
25% of patients with GBS experience respiratory pancarditis, migratory arthritis, Sydenham's chorea,
failure that requires mechanical ventilation. erythema marginatum, and subcutaneous nodules.
A diagnosis of ARF is made in the setting of two
~ Answer A. Although tarantula bites may inflict a of these five major Jones criteria and evidence of
significant amount of pain, there is usually minimal recent GAS infection, or one of the major criteria
erythema and swelling at the bite site. Severe enven- and two of the minor criteria (fever, arthralgias, and
omation is extremely uncommon and fatalities have a history ofprior rheumatic fever or rheumatic heart
not been described in the United States. However, disease). Rheumatic fever is rare in patients older than
tarantulas are covered with "urticating hairs" that 20 years. Treatment ofacute GAS pharyngitis reduces
it can cast out toward a victim. The hairs become the subsequent incidence of ARF. Scarlet fever is a
embedded in the skin and may cause an intense consequence of acute GAS infection. Patients usually
inflammatory response, resulting in pruritus and oc- present with symptoms of GAS pharyngitis plus the
casionally erythematous papules. After rubbing the presence of a diffuse maculopapular rash that has a
area, patients may also unintentionally transfer the fine, sandpaper-like feel (caused by an erythrogenic
hairs to their eyes resulting in a severe keratocon- toxin). The rash typically begins in the inguinal
junctivitis that requires ophthalmologic referral for creases before becoming quickly generalized. Shortly
treatment and hair removal. after generalization, the rash may become more
intense along skin folds (e.g., in the antecubital
~ Answer B. The minimal rise in the ,8-hCG level fossa), producing lines ofconfluent petechiae known
of a viable intrauterine pregnancy is 53% over a as Pastia's lines. The rash begins to fade 3 to 4 days
2-day period, although 85% of women will have an after its onset, and enters a desquamative phase with
increase of >66%. Unfortunately, 21% of patients flakes first peeling from the face and subsequently
with ectopic pregnancy will exhibit a rise in their from the palms and fingers. The tongue is also
,8-hCG level that mimics a normal intrauterine involved, developing a white coat through which
pregnancy. Furthermore, 10% of patients with ec- the tongue's erythematous papillae protrude giving
topic pregnancy do not have abdominal pain. This the appearance of a "white strawberry" tongue.
patient's ,8-hCG level rose 78% in the 2 days since Treatment of scarlet fever is the same as for acute
her previous visit (appropriate) and now exceeds GAS pharyngitis. Severe scarlet fever with systemic
the discriminatory threshold. Therefore, transvagi- toxic effects is not common.
nal ultrasonograph should reveal an intrauterine
gestation, if one is present. If an intrauterine ges- [!!) Answer B. The triptans are serotonin 5-HTlB/ID
tation is not seen, an OB consultation should be receptor agonists. Triptans are well tolerated though
obtained. Because the patient is asymptomatic and they have a number ofirritating side effects, which in-
her ,8-hCG level is rising appropriately, some prac- clude tingling, paresthesias, and sensations ofwarmth
titioners would continue to check serial ,8-hCG in the head, neck, chest, and limbs. Flushing, dizzi-
levels until the level is more definitively above the ness, and neck pain or stiffness occur less frequently.
discriminatory zone. However, many practitioners However, their use is limited because more frequent
would evacuate the uterine contents and exam- use results in rebound headache. In fact, this may oc-
ine for the presence of chorionic villi. If none are cur with other abortive drugs, particularly butalbital.
30 1(}()() Questions to Help You Pass the Emergency Medicine Boards

Triptans can cause coronary artery constriction and of 4 months and 4 years. However, S. pneumoniae
may cause chest symptoms, which mimics angina is the most common bacterial cause of pneumo-
pectoris. Although these symptoms may be frighten- nia in this age-group. Although chest x-rays are not
ing, they are rarely life threatening. However, there reliable in distinguishing between viral and bacte-
have been a few case reports of significant myocar- rial pneumonias, this patient has an infiltrate, so
dial ischemia or infarction in patients using triptans. it seems prudent to treat the patient with an an-
Therefore, the use of triptans should be consid- tibiotic with antipneumococcal activity. Doxycycline
ered contraindicated in the setting of ischemic heart and levofloxacin are contraindicated in children. Ery-
disease, poorly controlled hypertension, and cere- thromycin, a macrolide antibiotic, would be a better
brovascular disease. choice for children older than 5 years or younger than
4 months since M. pneumoniae and C. tTachomatis
ltoo] Answer E. Viruses are the predominant etiol- are the most common bacterial causes of pneumonia
ogy of pneumonia in children between the ages in these respective groups.
Test 2
Questions
[!] Which of the following represents the characteristic (B) Urinary tract infection (UTI)
acid-base disturbance seen in aspirin toxicity? (C) Pneumonia
(D) Stroke
(A) Respiratory alkalosis alone
(E) Pulmonary embolism
(B) Metabolic acidosis alone
(C) Mixed respiratory alkalosis with metabolic
acidosis
[I) Which of the following is true regarding electrical
injury?
(D) Mixed respiratory acidosis with metabolic
alkalosis (A) Direct current (DC) is more dangerous than
(E) No acid-base abnormality alternating current (AC).
(B) In high-voltage injuries, the extent of
W The most common cause of acute mesenteric cutaneous burns is a good predictor of internal
ischemia is tissue damage.
(C) Asystole is the most common dysrhythmia
(A) Thrombosis in the superior mesenteric
resulting from low-voltage electrical injury.
artery (SMA).
(D) In contrast to other multi-victim traumatic
(B) Mesenteric vein thrombosis.
events, patients without signs of life should be
(C) Nonocclusive mesenteric vascular disease.
resuscitated first.
(D) SMA embolism.
(E) All of the above.
(E) Abdominal aortic aneurysm involving the
SMA.
[I) A 55-year-old woman presents with left knee pain
after being struck at 5 miles per hour by a car turning
[I) A 23-year-old woman presents with fever and sore
into her pedestrian lane. Her knee is significantly
throat for 3 days. She thinks she has strep throat.
tender inferior and lateral to the patella. She is able to
Which of the following findings is more char-
fully extend her leg with pain and her neurovascular
acteristic of streptococcal pharyngitis than viral
and ligamentous exams are intact. Knee radiographs
pharyngitis?
are normal. Which of the following is the most
(A) Runny nose appropriate next step in management?
(B) Odynophagia
(A) Discharge with full weight bearing
(C) Absence of cough
(B) Discharge with crutches and weight bearing as
(D) Conjunctivitis
tolerated
(E) Bilateral tonsillar enlargement
(C) Computed tomography ( CT) scan of the knee
(D) Admit for knee observation
[!) Which of the following is true regarding posterior
(E) Urgent arthroscopy
shoulder dislocations?
(A) External rotation is usually intact. [!) Which of the following is true regarding treatment
(B) Neurovascular injury is more common than in of otitis media?
anterior dislocations.
(A) Few cases will resolve spontaneously without
(C) The absence of pain excludes the diagnosis.
antibiotics.
(D) Seizures are a common mechanism of injury.
(B) Oral antibiotics are superior to intramuscular
(E) Recurrent injury is more common than in
(IM) antibiotics in efficacy of treatment.
anterior dislocations.
(C) High-dose amoxicillin (80 mg!kg/day) should
W Which of the following is the number one cause of be reserved for those patients who are older
than 2 years of age.
death in patients with congestive heart failure (CHF)?
(A) Progressive hemodynamic deterioration

31
32 1(}()() Questions to Help You Pass the Emergency Medicine Boards

(D) Otitis media represents the number one reason (A) Ask the nurses to place a second large bore
for outpatient antimicrobial prescriptions in peripheral IV and immediately bolus the
the United States. patient with 2 L of normal saline.
(E) Auralgan may be beneficial in patients with (B) Extubate the patient and deliver breaths using a
tympanic membrane perforations. bag-valve mask.
(C) Disconnect the ventilator but keep the
[!] A 68-year-old man with a history of chronic ob- endotracheal tube (ETT) in place and allow the
structive pulmonary disease (COPD) presents to the patient to exhale.
emergency department (ED) with worsening dysp- (D) Ask the nurses to start a dopamine drip at
nea, cough, and subjective intermittent fevers. He 5 p.g/kglminute.
tells you that he spent 1 week in the intensive care (E) Perform an immediate needle thoracostomy.
unit (ICU) 6 months ago after being intubated for a
similar episode and states "I don't ever want to be ~ A 54-year-old man with a history of chronic alcohol
intubated again." You discuss the use of noninvasive abuse is brought in by his wife because of a "big
positive pressure ventilation with him (bilevel pos- stomach" and confusion. She tells you his stomach
itive airway pressure [BiPAP]) and he is agreeable. has been big for 1 year but over the last month he
With which of the following comorbidities is BiPAP seems to be forgetting things, has difficulty sleeping,
safe to use? and is not tending to his own appearance. Which of
(A) Excessive secretions the following is true?
(B) Decreased sensorium (A) Ammonia levels correlate with the severity of
(C) Hypertension his illness.
(D) Midfacial trauma (B) The most common finding on Cf of the brain
(E) Uncooperative patient is hydrocephalus.
(C) Spontaneous bacterial peritonitis is the most
~ A 44-year-old woman presents with right leg pain. common precipitant.
She has noticed increasing pain in the middle of her (D) Gastrointestinal ( GI) bleeding may exacerbate
tibia over the last few weeks since she started training or trigger this condition.
for a marathon. Physical examination of the entire (E) The treatment of choice is ceftriaxone.
lower extremity is normal except for mild tenderness
to palpation in the area of pain. Radiographs of ~ A patient presents with new-onset psychosis. Which
the tibia are normal. Which of the following is true of the following suggests a medical, rather than
regarding this patient's condition? psychiatric, cause for the symptoms?
(A) Most cases require surgery for definitive (A) Auditory hallucinations
management. (B) Age <35
(B) Bone scan has higher sensitivity than plain (C) Gradual onset
radiographs. (D) Aphasia
(C) Magnetic resonance imaging (MRI) is (E) Flat affect
indicated to evaluate for complications.
(D) Men are at higher risk than women. ~ Which of the following is more characteristic of
(E) The femur is the most common bone involved. migraine headaches than cluster headaches?
(A) The pain is unilateral.
(!!) Thirty minutes after a 35-year-old woman presents to (B) Pain is of moderate to severe intensity.
the ED with a severe asthma exacerbation, you intu- (C) The pain is throbbing in quality.
bate her because she is showing signs of fatigue and (D) The pain responds to sumatriptan.
ventilatory failure. You use ketamine and succinyl- (E) The patient has a concomitant upper
choline and pass the endotracheal through the vocal respiratory infection.
cords without difficulty. A colleague who is assisting
you aggressively "bags" the patient until the respi- ~ A 38-year-old pregnant woman presents to the ED
ratory therapist connects the mechanical ventilator. with abdominal pain radiating to the back and vaginal
Thirty seconds after intubation, the nurse reports bleeding. She is 25 weeks by dates and has had an
that the patient's blood pressure (BP) has dropped uncomplicated pregnancy with routine prenatal care.
to 93/46. Her BP before intubation was 138/80. The She admits to smoking one pack of cigarettes per day
patient has an 18-gauge peripheral intravenous (IV) and otherwise has no significant history. Her uterus
line in her left antecubital fossa and her trachea ap- is firm and tender on examination and there is bright
pears midline. What is the best course of action?
Tsst2 33

red blood oozingfrom the cervical os. The most li1rely


diagnosia i&
(A) Appendicitis.
(B) Placenta previa.
(C) Vasa previa.
(D) Fibroid degeneration.
(E) Abruptio placentae.

~ What i& the duration of action of n.al.omne?


(A) 30 minutes
(B) 1 hour
(C) Shoun
(D) lOhours Figure 2-1.
(E) 20houra
(A) 2.5% selenium sulfu:le shampoo
(!!1 Which of the foUowing is true regarding hy- (B) Clotrimazole cream
pokalemia? (C) High-dose amo:xicillin (80 to 90 mg per kg
(A) Patients with a recent myocardial infarction divided b.i.d.)
(MI) are at increased risk for ventricular (D) Terbinafine lotion
arrhythmias if the [K+] <4.0 mBqperL. (E) Oral griaeofulvin
(B) The presence ofU waves and ST-segment
depression on electro<:ardiograms (E.KGs) ~ Which of the following is true about pilonidal
correlates with the severity of hypoka- abacesses?
lemia. (A) Tb.ey are more common in women.
(C) Neurologic problems are the most common
(B) They are more common in hairless patient!.
manifestations of hypokalemia. (C) Recurrence after incision and drainage is
(D) Vomiting or nasogastric suctioning may lead to
uncommon.
profound hypokalemia. (D) Longitudinal incisions should be made off of
(E) Potassium i& the most prominent extracellular
the sacral midline.
cation.
(E) All of the above.
(ji] Which of the following is true regarding orbital
~ Which of the following i& true regarding acute
cdlulitiaf
salicylate toxicltf.
(A) Pain with extraocular movements ia a
(A) Treatment should be instituted in strict
characteristic finding. accordance with the Done nomogram.
(B) Hematogenous spread of bacteria commonly
(B) Initial symptoms of toxicity are tinnitus and
occurs.
altered hearing.
(C) Sinusitis is an uncommon predisposing &.ctor.
(C) Metabolic alkalosis i& cha.racteristic.
(D) AspergiUus is a common cause of acute disease.
(D) Significant bleeding tbroush the Gl tract is
(E) Periorbital cellulitis often spontaneously
present in most patients.
progresses to involve the orbit.
(E) Life-threatening hyperkalemia may occur.
(!!) Which of the following is the most conunonly
ingested alcohol after ethanoU
[ij) The manifestations ofhyperphosphatemia are related
to its effects on:
(A) Methanol
(B) Ethylene glycol
(A) Sodium
(B) Potassium
(C) bopropanol
(D) Propylene glycol (C) Magnesium
(D) Calcium
(E) Acetone
(E) Chloride
~ A mother brings in her 12-year-old son with a
~ Which of the following is true about primary
chief complaint of alopecia. The patient denies any
complaints and hu been well recendy. The lesion is spontaneous pneumothorax?
shown in Figure 2-1. Which of the following ia the (A) Most patients develop primary pneumothorax
treatment ofchoice? during vigorous exercise.
34 1(}()() Questions to Help You Pass the Emergency Medicine Boards

(B) Expiratory chest x-rays are critical to make (C) Most patients will present with isolated melena.
a diagnosis. (D) The mortality rate ofbleeding due to a peptic
(C) It is typically a less dangerous finding than ulcer is 25%.
primary spontaneous pneumomediastinum. (E) Perforation may be accompanied by
(D) Hamman's crunch is pathognomonic for the hemorrhage in 50% of cases.
diagnosis.
(E) Smoking is the most significant risk factor. ~ A 28-year-old man presents to the ED with an
extensive vesicular, weeping, and crusted eruption
~ Among trauma patients, which ofthe following is the arranged in a linear pattern on his lower legs. He tells
most common incomplete spinal cord syndrome? you that he cleaned out the brush from the woods
(A) Brown-Sequard syndrome behind his house a couple ofdays ago. You suspect he
(B) Central cord syndrome has a contact dermatitis due to poison ivy. In addition
(C) Anterior cord syndrome to cool compresses and antihistamines to help control
(D) Posterior cord syndrome pruritus, the ideal course ofcorticosteroids should be
(E) Conus medullaris syndrome (A) Topical triamcinolone until resolution.
(B) Three days of oral prednisone.
~ Radiation enteritis ... (C) A commercially available steroid dose pack
(A) Has a mortality rate of 75% in its acute form. (e.g., Medrol Dosepak).
(B) Presents most commonly in its chronic form. (D) At least 14 days of oral prednisone.
(C) Occurs more than 2 years after completion of (E) Corticosteroids are unnecessary in most cases
radiotherapy in 10% of patients. of poison ivy induced contact dermatitis.
(D) Is best diagnosed with CT of the abdomen and
pelvis with oral and N contrast. [llJ A 23-year-old man with a past medical history
(E) Is all of the above. of acquired immunodeficiency syndrome (AIDS)
presents with fever and headache. Brain CT scan
~ Which of the following toxins is suggested by the is unremarkable and cerebrospinal fluid results from
smell of garlic? lumbar puncture results are shown in the subsequent
text:
(A) Cyanide
(B) Zinc White blood cell (WBC): 35 per 11-L, lymphocytic
(C) Toluene predominance
(D) Organophosphate Red blood cell (RBC): 2 per 11-L
(E) Hydrogen sulfide Glucose: Normal
Protein: Normal
~ A 25-year-old woman presents to the ED with thumb Gram stain: Negative
pain after a skiing accident in which she fell with her India ink: Positive
thumb caught in the straps ofher pole. Which of the Which of the following is the most appropriate
following is most likely true? medication at this time?
(A) She should be placed in a thumb spica splint. (A) Ceftriaxone
(B) She has tom the ulnar collateral ligament (B) Vancomycin
(UCL) at the thumb metacarpophalangeal (C) Acyclovir
(MCP) joint. (D) Itraconazole
(C) She will have difficulty with pinching and (E) Amphotericin B + flucytosine
grasping.
(D) The presence of a Stener lesion mandates ~ A 34-year-old woman has a hemoglobin level of
operative repair. 10.0 g per dL with a low mean corpuscular volume.
(E) All of the above. Which of the following is the most likely cause?
(A) a-Thalassemia
~ Which of the following is true about upper gas- (B) ,8-Thalassemia
trointestinal bleeding (UGIB) due to peptic ulcer (C) Iron -deficiency anemia
disease (PUD)? (D) Sideroblastic anemia
(A) In patients with normal GI anatomy, (E) Folate deficiency
hematochezia will not occur.
(B) Bleeding will stop spontaneously in 70% to ~ A 47 -year-old woman presents with a severe occipital
80% of patients. headache and a general feeling of malaise. Her
Test2 35

neurologic examination is normal A stat head Cf (E) Fetal abortion occurs in 50% of pregnant
is shown in the image (see Fig. 2-2). Which of the women with perforated appendicitis.
following is true about this patient1
~ Which ofthe following is true regarding spontaneous
abortion?
(A) Patients diagnosed with threatened abortion
should not receive anti-D immunoglobulin
(RhoGAM) because the antibody may
provoke an immune response to the live
fetus.
(B) When a fetal heartbeat is seen on
ultrasonograph, patients diagnosed with
threatened abortion will experience a
spontaneous :miscarriage 50% of the
time.
(C) Patients diagnosed with threatened abortion
should be placed on bedrest restrictions until
the bleeding resolves.
(D) All patients who present to the ED with fetal or
placental tissue and resolution of vaginal
bleeding can be diagnosed with a complete
abortion and discharged.
(E) Up to 80% of women with first trimester
spontaneous abortion complete the abortion
without intervention.

I!!) Which of the following radiographic views of the


Figure 2-2.
knee best identifies longitudinal patellar fractures?
(A) Anteroposterior (AP)
(B) Lateral
(A) The development ofhydrocephalus requires
(C) Medial oblique
(D) Lateral oblique
urgent neurosurgical intervention.
(B) The patient should be gmn stat IV (E) Sunrise
corticosteroids and loaded with an
anticonvulsant. ~ In patients olderthan85years, which ofthe following
(C) Such patients often succumb to uncal is the most common symptom during Ml1
herniation. (A) Chest pain
(D) A systolic blood pressure (SBP) of90 mm Hg is (B) Dyspnea
probably due to the Cushing response. (C) Syncope
(E) All of the above. (D) Altered mental status
(E) Fever
~ Which of the following is true about appendicitis in
adult women? @jJ Which of the following is the best adjunct to the
(A) Pregnant women are twice as likdy as physical examination in assessing the severity of an
nonpregnant women to develop appendicitis. asthma attsck?
(B) Cervical motion tenderness (CMT) rules out (A) Arterial blood gas (ABG)
appendicitis. (B) Peak Expiratory Flow Rate (PEFR)
(C) Even in the third trimester, most pregnant (C) Chestx-ray
women 8till have pain in the right lower (D) Continuous cardiac monitoring
quadrant. (E) FEVt (forced expiratory volume in one second.
(D) Owing to anatomic changes, appendicitis in expressed as liters per second)
pregnant women occurs most often in the third
trimester.
36 1(}()() Questions to Help You Pass the Emergency Medicine Boards

~ How long after a toxic acetaminophen ingestion ~ The most common cause of bacterial arthritis in
does N-acetylcysteine (NAC) still result in 100% adults is
prevention of hepatic injury? (A) Staphylococcus aureus.
(A) 2hours (B) Neisseria gonorrhoeae.
(B) 4hours (C) Streptococcus pyogenes.
(C) 8hours (D) Hemophilus influenzae.
(D) 12 hours (E) Polymicrobial.
(E) 24 hours
~ A 23-year-old man presents after being assaulted by
~ Which of the following is true regarding control of several men. Per eyewitness report, the patient was
hypertension and stroke? kicked and struck with a baseball bat several times.
(A) Patients with both ischemic and hemorrhagic The patient is brought in on backboard and cervical
strokes should have their BP reduced to a collar. He is extremely agitated and combative,
SBP of 140 to 160 mm Hg or prestroke punching and kicking staff and climbing off the
levels. bed. He is yelling, "I want to get out of here!"
(B) BP should not be controlled in any stroke when any question is asked of him. He appears
patient unless the patient is a candidate for to be moving every extremity except his left arm.
tissue plasminogen activator (tPA). You complete the primary survey, which is intact
(C) Only patients with ischemic infarcts should except for the paralyzed left arm. His vital signs
have their BP reduced to a target SBP of 140 to are 98.0, 95, 22, 166/94, and 98% RA. Which of
160 mm Hg or prestroke levels. the following is the most appropriate next step in
(D) Only patients with intracerebral hemorrhage management?
should have their BP reduced to a target SBP (A) CTbrain
of 140 to 160 mm Hg or prestroke levels. (B) MRI brain
(E) All stroke patients who experience clinical (C) CT cervical spine
deterioration in the ED should have emergent (D) MRI cervical spine
BP control to a target SBP of 140 to 160 mm (E) Rapid sequence intubation
Hg or prestroke levels.
~ A 25-year-old woman presents with white vagi-
~ A 17-year-old basketball player collapses and dies nal discharge. Physical examination demonstrates
while playing a game. Which of the following is the grayish-white discharge with no cervical lesions.
most likely cause? A wet mount is positive for the presence of clue
(A) Coronary artery disease cells. Which of the following is best therapy for
(B) Pulmonary embolism this patient?
(C) Hypertrophic cardiomyopathy (A) Fluconazole 150 mg PO once
(D) Subarachnoid hemorrhage (SAH) (B) Metronidazole 2 gPO once
(E) Spontaneouspneurnothorax (C) Clindamycin 300 mg b.i.d. for 7 days
(D) Ceftriaxone 125 mg IM once
~ A 37-year-old previously healthy woman is brought (E) Azithromycin 1 gPO once
to the ED by her family with a complaint of
fever, headache, confusion, and lethargy. Empiric ~ Which of the following is true regarding the
antibiotics for community-acquired meningitis were peripheral WBC count?
started and a CT scan was performed which was (A) Elevation in the WBC count is specific for an
negative. Before getting a bed, her lumbar puncture infectious process.
revealed the following: WBC 412 per mm3 with a (B) The presence of95% neutrophils is also known
differential of98% lymphocytes, protein 120 mg per as a left shift.
dL, glucose normal. Which of the following should (C) Exercise can elevate the WBC count to more
be added to her regimen? than 15,000 cells per mm3 •
(A) Acyclovir (D) The degree ofleukocytosis distinguishes
(B) Ampicillin between viral and bacterial infection.
(C) Dexamethasone (E) A WBC count >50,000 cells per mm3 is specific
(D) Amphotericin B for chronic myeloid leukemia.
(E) None of the above.
Test2 37

~ Which of the following is true regarding physostig- (A) Fever


• ?
mme. (B) Periorbital edema
(C) Eye tenderness
(A) It is the drug of choice to treat most
(D) S. aureus as the etiologic agent
anticholinergic aises.
(E) Restricted eye movement
(B) It affects muscarinic, but not nicotinic
receptors.
(C) It affects nicotinic, but not muscarinic
(H) A 5-year-old boy presents with bloody diarrhea for
several days, followed by fatigue, pallor, and malaise.
receptors.
(D) It is able to cross the blood-brain barrier.
Several kids at school have similar complaints. Blood
(E) It should be rapidly pushed to achieve clinical
tests at his pediatrician's office demonstrate severe
anemia, thrombocytopenia, and renal insufficiency.
effect.
Which of the following is the most likely cause?
~ An 8-year-old boy is brought in by his parents after (A) ShigeUa
apparently ingesting a pin. He looks well and has (B) Salnume'lla
a normal physical examination. A flat plate of the (C) Escherichia coli
abdomen is shown in Figure 2-3. Which of the (D) Rotavirus
following is true? (E) Child abuse

~ Which of the following is true regarding transient


ischemic attack (TIA)?
(A) Neurologic findings in patients with TIAs are
more commonly "positive" (tingling or
involuntary movements) than "negative"
(aphasia, weakness, numbness).
(B) A "march" ofsymptoms affecting various body
parts in succession is common in TIAs.
(C) TIA was recently redefined as transient
neurologic dysfunction that resolves within
1 hour.
(D) Confusion and generalized weakness are
common in patients with a TIA.
(E) The moat common mimic ofsymptoms caused
by a TIA is a complicated migraine.

~ A23-year-old womanisstruckin the eye with a soccer


ball Penlight eye examination is shown in Figure 2-4.
Figure 2-3. Which of the following is the most appropriate nat
step in management?

(A) The most common site of perforation is the


ileocecal valve.
(B) Perforation occurs in 50% ofcases.
(C) Emergent consultation to a pediatric
gastroenterologist is required for endoscopic
removal
(D) The patient am be safely discb.arged home with
follow-up with his pediatrician.
(E) The patient's parents should give consent for
an emergent laparotomy for su.rgical removal.

~ A 9-year-old man presents with right eyelid swelling,


pain, and redness, progressively worsening over
3 days. He does not report blurry vision. Which
of the following is more characteristic of orbital
cellulitis than periorbital cellulitis? FigllN 2-4,
38 1(}()() Questions to Help You Pass the Emergency Medicine Boards

(A) Trendelenburg positioning (B) Troponin T


(B) Ibuprofen for pain (C) Myoglobin
(C) Activated factor VII (D) CK-MB
(D) Eye shielding (E) Total creatine phosphokinase (CPK).
(E) IV antibiotics
~ The best route and location of epinephrine adminis-
~ The agent of choice to inactivate coelenterate tration for anaphylaxis is
nematocyst toxin (such as jellyfish) is (A) IM in the deltoid.
(A) 5% acetic acid (vinegar). (B) Subcutaneous (SQ) in the deltoid.
(B) Ambient temperature fresh water. (C) IM in the lateral thigh.
(C) Ambient temperature seawater. (D) SQ in the lateral thigh.
(D) Ammonia. (E) There is no preferable route.
(E) Alcohol.
~ A 15-year-old boy presents with progressively wors-
~ Which of the following is the most common ening groin and scrotal pain an swelling over the
presenting symptom of multiple sclerosis (MS)? last 8 hours. He noticed a bulge in his scrotum
the day before when he lifted a heavy object. Phys-
(A) Urinary retention
ical examination demonstrates an afebrile patient
(B) Hemiparesis of the upper extremities
with moderate tenderness and fullness, with bowel
(C) Ataxia
sounds present in his right hemiscrotum. The testes
(D) Aphasia
are not tender or enlarged. Which of the following is
(E) Eye pain and monocular visual loss
the most appropriate next step in management?
~ Which ofthe following is true regarding Mycoplasma (A) Ice pack to the groin
pneumoniae infections? (B) Immediate operative reduction
(C) Outpatient urology referral
(A) The presence of bullous myringitis is central to (D) Oral hydration
making a diagnosis of M. pneumonia. (E) Urinalysis
(B) Outbreaks are common in institutional settings
such as camps and military bases. ~ Which of the following is true regarding adrenal
(C) Neurologic complications occur in up to 30%
insufficiency?
of patients.
(D) Cold agglutinin titers rise within 1 to 2 days of (A) Patients with secondary adrenal insufficiency
infection with M. pneumoniae. frequently have hyperpigmentation.
(E) Mycoplasma is the most frequent cause of (B) Hypernatremia is the most common electrolyte
community-acquired pneumonia in elderly abnormality.
patients older than 65. (C) Hyperkalemia is a common side effect of
prednisone or hydrocortisone therapy.
(D) Nausea and vomiting are present in >50% of
~ A 65-year-old man presents with fever, chills, and
patients.
dysuria for 2 days. He denies vomiting or back
(E) All of the above.
pain. Physical examination reveals a patient in mild
discomfort, with normal cardiac, pulmonary, and
~ A 26-year-old woman is brought to the ED after
abdominal examinations. He lacks costovertebral
a motor vehicle accident in which she was thrown
angle tenderness, but rectal examination reveals a
from the vehicle. Initial evaluation reveals a con-
boggy, tender prostate. Which of the following is the
fused patient with multiple scalp wounds and vital
most appropriate therapy?
signs of P 130, BP 85/55. After intubation and fluid
(A) Ceftriaxone 125 mg IM and doxycycline resuscitation, initial plain films reveal clear lungs
100 mg PO b.i.d. for 10 days but an obvious pelvic fracture. Diagnostic peritoneal
(B) Azithromycin 1 gPO lavage (DPL) reveals a grossly positive tap (aspira-
(C) Ciprofloxacin 500 mg PO b.i.d. for 3 days tion). Which of the following is the next step in
(D) Ciprofloxacin 500 mg PO b.i.d. for 7 days management?
(E) Ciprofloxacin 500 mg PO b.i.d. for 30 days
(A) Abdominal CT to better determine the need for
laparotomy.
~ Which of the following has the highest sensitivity for (B) Pelvic angiography.
acute MI in the first 2 hours? (C) Thoracotomy with cross clamping of the aorta.
(A) Troponin I (D) Exploratory laparotomy.
Test2 39

(E) Use DPL cell counts to determine the need for (B) Result in central pontine myelinolysis.
laparotomy. (C) Worsen her hyponatremia.
(D) Suppress further antidiuretic hormone
~ Which ofthe following is the most common cause of (ADH) secretion.
pleural effusion in the United States? (E) Make no difference in her sodium level and
(A) Cancer cause water retention and edema.
(B) Tuberculosis
(C) Pneumonia [ill A 40-year-old man presents with testicular pain for
(D) CHF 2 days and fever of 101°F. He also complains of
(E) Pancreatitis dysuria, but denies scrotal edema, flank pain, nausea,
or vomiting. Physical examination demonstrates
~ Which of the following is the most common moderate tenderness with mild edema and erythema
complication of acute sinusitis? in the scrotal area. Cremasteric reflexes are present
bilaterally. A testicular ultrasound is performed and
(A) Maxillary cellulitis
is negative for torsion. Which of the following is the
(B) Cavernous sinus thrombosis
most likely etiology of the patient's symptoms?
(C) Meningitis
(D) Preseptal cellulitis (A) Viral
(E) Orbital cellulitis (B) Chlamydia
(C) N. gonorrhoeae
~ Which of the following is true regarding labetalol? (D) E. coli
(E) Pseudomonas aeruginosa
(A) Effect on a-receptors is greater than that on
~-receptors.
(B) Reflex tachycardia is a common complication.
[!!) A 23-year-old woman presents with pain in her right
lateral chest after a low-speed motor vehicle collision.
(C) Orthostatic changes with N use are rare.
She is most tender in the fifth rib at the posterior
(D) BP is more easily controlled than with
axillary line. Which of the following is the most
nitroprusside.
appropriate next step in evaluation?
(E) It is superior to nitroprusside as monotherapy
for aortic dissection. (A) Chest x-ray
(B) Rib x-rays
~ A 55-year-old woman presents with a laceration on (C) CT abdomen/pelvis
her arm after falling while riding her bicycle. She does (D) CTbrain
not know the last time she had a tetanus booster, but (E) Cervical spine radiographs
wants to know why she should have one. Which of
the following is true regarding tetanus? ~ Peritonsillar abscess (PTA) refers to a collection of
pus adjacent to which of the following structures?
(A) Mortality for clinically evident tetanus is
almost 50%. (A) Palatine tonsil
(B) All patients with tetanus have a history of (B) Pharyngeal tonsil
preceding injury. (C) Lingual tonsil
(C) Wound cultures are helpful for diagnostic (D) All of the above
screening. (E) None of the above
(D) Cardiac dysrhythmia is the most common
cause of death. [!Q} The daughter of an 82-year-old woman brings her
(E) Tetanus boosters should be updated every year mother in with a chief complaint of a "foreign
with clean wounds. body'' in her vagina (see Fig. 2-5). The patient had
reported the uncomfortable sensation of sitting on
[!!) A 64-year-old woman with small cell lung cancer a ball and of something "falling out" of her vagina.
presents with a chief complaint of fatigue, dizziness, Which of the following is the next best step in
and imbalance. Her blood work is significant for management?
a sodium level of 112 mmol per L. You suspect (A) Consultation with obstetrics and gynecology
she has a syndrome of inappropriate secretion of ( OB-GYN) for immediate hysterectomy.
antidiuretic hormone (SIADH) due to her lung (B) Discharge the patient with a prescription for
cancer. Administering normal saline to this patient metronidazole and an appointment with
will likely: OB-GYN in 2 days.
(A) Slowly correct her sodium level. (C) Manually reduce the mass.
40 1000 Qvaticms ftl Hdp You Ptw the Emerfmcy Medit:ine BoMds

(B) Owing to significant airway obstruction, such


patients require very low or no PEEP similar to
asthma patients to avoid air trapping.
(C) Owing to higb compliance, such patients
require lower tidal volumes and lower PEEP to
improve oxygenation.
(D) Owing to low compliance, such patients
require lower tidal volumes and higher PEEP to
avoid barotrauma.
(E) Owing to high compliance, patients with ARDS
do not require PEEP.

(MJ Which of the following is true regarding allergic


Figure 2-5. reactions?
(A) I..ib infanta, many adults with food allergies
will outgrow their allergies over time.
(D) Incision and drainage of the mass. (B) Patients allergic to honeybee stings will also
(E) None of the above. have similar reactions to wasp stings.
(C) Most adverse drug reactions are allergic.
(fi) Which of the following opioids may predispose to (D) Oral allergen exposure provokes a stronger
QRS prolongation? anaphylactic response than topical
(A) Fentanyl expoaure..
(B) Meperidine (E) Atopy predisposes patients to develop
(C) Propoxyphene anaphyJaxis.
(D ) Hydrocodone
(B) Morphine ~ Which of the following is the most common initial
dysrhythmia in symptomatic patients with Wol1f-
l!il Which of the following cervical vertebral levels Padrinson-White (WPW) syndrome?
exhibit pseudosubluxation in almost half of all (A) Multifoc:al atrial tachycardia (MAT)
pediatric patients? (B) Atrioventricular (AV) nodal reentrant
(A) Cl -2 tachycardia
(B) C2-3 (C) Mobit:z type I second-degree AV block
(C) C3-4 (D) Mobitz type II second-degree AV block
(D) C4-5 (E) Torsade de pointes
(E) CS-6
(!j] Which of the following is true regarding laboratory
~ A 63-year-old woman is broqht to the ED by her testing in patients with abdominal trauma?
children because she is lethargic and has labored (A) Liver enzymes are used to help distinguish
breathing. They last saw her 4 days ago when she minor contusions &om high-grade lacerations.
seemed weD. Her vital signs are T 101.8°F, HR. (B) Elevated serum amylase and lipase are always
120 per minute, RR 32 per minute and a POx value indicative of pancreatic injury.
of 89% on 100% oxygen by face mask. She is in- (C) Microscopic hematuria usuaUy indicates a need
tubated and placed on assist-control ventilation. A for abdominal cr scanning in pediatric blunt
subsequent chest x-ray reveals diffuse bilateral in- trawna patients.
filtrates, and normal heart size. You suspect she (D) The hematocrit is only useful when serial
has severe pnewnonia and acute respiratory dis- measurements are conducted.
tress syndrome (ARDS). Which of the following (E) None of the above.
summari7.es the best ventilation strategy in patients
with ARDS? (ll] Which of the following is a positive prognostic sign
(A) Owing to low compliance. patients with ARDS in patients with frostbita
need higher tidal volumes and higher positive (A) Violaceous color after rewarming
end-expiratory p.ressure (PEEP) to ensure (B) Lack of edema formation
adequate ventilation.
Test2 41

(C) Woody firmness of the SQ tissue with a fully dilated cervix. Because your hospital has
(D) Early formation of clear blebs in the affected no obstetrics services, you prepare for delivery. One
tissue minute after a successful and apparently unevmtful
(E) None of the above delivery, the patient becomes abruptly hypoxic,
severely hypotensive with a BP of 76/palp and
(!!] Which of the following is the most useful historic obtunded. The most likely diagnosis is
detail to distinguish chest pain caused by coronary (A) Sepsis.
ischemia versus that caused by gastroesophageal (B) Pulmonary embolism.
reflux (GERD)? (C) Peripartum cardiomyopathy.
(A) Radiation of pain to the jaw (D) Amniotic fluid embolism (AFE).
(B) Emotional precipitation of pain (E) Eclampsia.
(C) Precipitation of pain by exercise and relief by
rest lii) Which of the following cervical spine vertebrae is
(D) Radiation of pain to the back most commonly fractured from falls in the elderly!
(E) None of the above (A) Cl
(B) C2
(!!J A 32-year-old man presents with eye pain and red- (C) C3
ness. Slit lamp examination is shown in Figure 2-6. (D) C4
Which of the following is the most appropriate next (E) C7
step in management?
~ Which of the following cardiac findings is expected
in hypothermic patients?
(A) QT interval prolongation
(B) Sinus bradycardia
(C) Atrial fibrillation
(D) Jwaves
(E) All ofthe above

~ The combination ofceftriaxoneandazithromycinisa


common dual-anu"biotic regimen used in the empiric
treatment of community-acquired pneumonia. For
which of the following organisms is Azithromycin
included?
(A) H. inftuenzae
(B) S. m4reus
Figure 2-f. (C) Legionella pneumophilR
(D) Streptococcus pneunwniRe
(E) MomxeUa camrrhalis
(A) Topical prednisolone
(B) Valacyclovir PO (ii} A 27-year-old man is brought to the ED after being
(C) Acetazolamide IV stabbed in the neck with a knife and robbed. Upon
(D) Erythromycin PO examination, you note a 3-cm. wound to zone n
(E) Ceftriaxone IV of the neck with an intact platysma. Which of the
following is the next best step in management?
(jg) Which of the following animals confus the highest (A) Local wound care, reassurance, and discharge
risk of transmitting rabies? (B) Admission for 23 hours ofobservation
(A) Squirrd (C) Soft tissue x-ray of the neck
(B) Skunk (D) Carotid angiography
(C) Hamster (E) CT of the neck
(D) Chipmunk
(E) Rabbit Iii} The most common cause ofmassive lower gastroin-
testinal bleeding (LGIB) is
lijJ A 32-year-old woman G,P2 at 39 weeks' gestation (A) Angi.odysplasia.
presented to your community ED in active labor (B) Diverticulosis.
(C) Ischemic colitis. ~ A 14-year-old boy presents with rash all over his
(D) Colon cancer. chest and bade. u shown in Figure 2-8. His mother
(E) In11ammatory bowel disease. reports that he had "strep throat" a rew days ago and
she gave the patient some leftover anb.oiotics that she
I!!J Which of the fOllowing is used to prevent cerebral had been prescribed. Which of the following is the
vasospasm after traumaticsubarachnoid hemorrhage most likely antibiotic given?
(SAH)?
(A) Labetalol
(B) Lorazepam
(C) Isoproterenol
(D ) V e.rapamil
(E) Nicardipine

~ A 23-~ar-old man praents with shoulder pain after


falling on his left shoulder. Physical examination
demonstrates tenderness in his lateral clavicle. He is
able to touch his opposite shoulder with his left hand.
There are no neurovascular deficits. His shoulder is
shown in Figure 2-7. Which of the following is the
most likely diagnoaia?

Figure 2-8.

(A) Ciprofl.oxacin
(B) Doxycycline
(C) Amoricillin
Figure 2-7. (D) Azithromycin
(E) Clindamycin

(A) Anterior shoulder dislocation ~ A 22-year-old woman presents with a 3-day h.iatory
(B) Inferior shoul.de:r dislocation ofa severe sore throat and painful swallowing. Whlle
(C) Posterior shoulder dislocation obtaining her history, you note that she has a muffled
(D) Acromioclavicular separation voice, but no respiratory distress. On cnmination
(E) Sternoclavicular dislocation you find "two-finger" trismus, an erythematous
pharynx, swellingofthe left:anterior pillar, and uvular
(@!) The most common auociated finding in pharyn- deviation to her right. You diagnose a peritonsillar
gim caused by adenavirw is which of the fOllow- abscess (PTA). Which of the following is true?
ing? (A) Fusobaamum specie. are the most commonly
(A) Pneumonia isolated organi.uns.
(B) Encephalitis (B) This patient requires urgent tonsillectomy.
(C) Peritonsillar abscess (PTA) (C) Incision and drainage is best accomplished
(D) Scarlatiniform rub with the patient supine.
(E) Conjunctivitis
Test2 43

(D) Most of these infections are polymicrobial. (C) Serious head injury is more common in adults
(E) Absence of pus upon aspiration rules out the than in children.
presence of an abscess. (D) Renal injury is more common in adults than in
children.
~ A 27-year-old man is brought to the ED after an (E) Liver injury is more common in adults than in
accident he had while riding an all-terrain vehicle children.
(ATV). He was riding in an open field with a helmet
and bodyannorwhen he was struck in the neck with a ~ Which of the following is the most common
wire fence that knocked him offthe bike and knocked symptom of mitral valve prolapse (MVP)?
him out. He now complains of a mild headache and (A) Dyspnea
neck and back soreness but is otherwise without (B) Palpitations
complaints. His examination reveals left-sided ptosis (C) Peripheral edema
and anisocoria with a smaller left pupil. Which ofthe (D) Lightheadedness
following is most likely to reveal significant injury? (E) Orthopnea
(A) CT head without contrast
(B) Carotid angiography [!1) A 36-year-old man was bitten on the hand by a
(C) Laryngoscopy reddish-brown snake while he was cleaning debris
(D) Cervical spine series from his backyard. He did not catch a good glimpse
(E) Chest x-ray ofits head but thinks it had a greenish-yellow tail and
he knows copperheads have been seen in the area.
~ A 55-year-old woman with a history of rheumatoid In the ED, he has two fang marks on the dorsum of
arthritis presents with progressive swelling and pain his hand associated with local pain and mild swelling
in her knee for 6 days. She denies trauma to the area of the area extending to his wrist. The grade of his
or fever. She is on prednisone for her rheumatoid envenomation is
arthritis and states that her standard flares involve (A) Grade 0 (no envenomation).
her ankles and fingers. Vital signs are 99.0°F, 100, (B) Grade 1 (mild).
20, 132/65, 98% RA. Physical examination reveals (C) Grade 2 (moderate).
a moderate-sized knee effusion with warmth and (D) Grade 3 or 4 (severe).
tenderness and extreme pain on range of motion (E) Copperheads are not known to be venomous in
of the joint. Which of the following is the most the United States.
appropriate next step in management?
(A) Joint aspiration [!!) A 23-year-old man is bitten on his forearm by
(B) MRI of the knee a raccoon. Which of the following is the most
(C) Colchicine PO appropriate anatomical region to administer human
(D) Stress-dose steroids rabies immune globulin (HRIG)?
(E) Indomethacin PO (A) Deltoid
(B) Gluteus maximus
~ Which of the following is the most characteristic (C) At the wound site
finding on funduscopic examination for central (D) Contralateral forearm
retinal artery occlusion? (E) Corpora cavernosum
(A) Pale gray retina with cherry red fovea
(B) Cloudy cornea with mid-dilated pupil ~ A 52-year-old man with a long-standing history of
(C) Disc edema with tortuous veins alcohol abuse presents with a chief complaint of
(D) Reddish haze with black reflex vomiting blood. He was last seen by a doctor 8 months
(E) Grayish-green subretinal membrane ago because of abdominal swelling and he was told
he had "a liver problem." Which of the following is
~ Regarding victims of motor vehicle crashes, which of most likely to be useful in this patient?
the following is true? (A) Octreotide
(A) Hypotension due to hemorrhagic shock occurs (B) Famotidine
earlier in adults than in children. (C) Pantoprazole
(B) Hypothermia during resuscitation occurs (D) Vasopressin
earlier in adults than in children. (E) Propranolol
[tOO] An 8-year-old girl presents with rash on her trunk.
as shown in Figure 2-9. Which ofthe following is the
most likely etiologic agentl
(A) Herpes simplex virus (HSV)
(B) Vari.cdla•zoste:r virus (VZV)
(C) Human immunoddidencyv:irus (HIV)
(D) Epstein-Barr virus (EBV)
(E) Parvoviroa

Figure 2-t.
Answers and Explanations
[I) Answer C. Aspirin (acetyl-salicylic acid} causes [!) Answer A. Patients with CHF die most commonly
a mixed acid-base disorder, with a metabolic due to hemodynamic decline, followed by dysrhyth-
acidosis (because of its uncoupling of oxidative mia. Mortality from fluid overload has been reduced
phosphorylation} and respiratory alkalosis (because by .fJ-blockers and ACE inhibitors and short-term
of its stimulation of medullary respiratory centers mortality from dysrhythmias has been reduced by
causing hyperventilation). The net effect on pH automated internal cardiac defibrillators (AICDs).
varies, but generally results in acidosis in children Choices B, C, D, and E are all important, but are less
and alkalosis in adults. common causes of death in patients with CHF.

W Answer D. Emboli in the superior mesenteric 00 Answer D. For a given voltage, AC is thought to
artery (SMA} account for 50% of the cases of acute be three times more dangerous than DC. This is due
mesenteric ischemia. Emboli usually originate in the to the fact that AC current causes repetitive muscle
left atrium or ventricle. Most emboli lodge just distal contraction or tetany once the "let-go current" is
to a major branch point, and >50% of SMA emboli exceeded. This results in prolonged exposure and
are located just distal to the origin of the middle colic more severe injury. High-voltage electrical injuries
artery. Thrombosis of the SMA and mesenteric vein should be treated like crush injuries, because there
thrombosis each account for approximately 15% of is often a large amount of tissue damage underneath
cases. normal appearing skin. It is impossible to predict
the degree of underlying damage from the extent
[1] Answer C. Both viral and group A streptococcal of cutaneous bums. Fewer than 10% of patients
pharyngitis usually cause odynophagia and tonsillar experiencing low-voltage electrical injury develop a
enlargement. The absence of cough and presence cardiac dysrhythmia. In those patients who do suffer
of tender anterior cervical lymphadenopathy are cardiac arrest due to an arrhythmia, ventricular
much more common with bacterial pharyngitis. fibrillation is most common. Triage priorities are
Upper respiratory symptoms and conjunctivitis in different in cases of high-voltage electrical injury or
conjunction with pharyngitis and fever are almost lightning strikes. Patients with obvious signs of life
always viral, usually due to adenovirus. The sensitivity tend to do well and can afford a small delay in
of older rapid antigen testing for streptococcal definitive care. Furthermore, due to the possibility of
pharyngitis is reported to be approximately 80%, a good outcome with cardiopulmonary resuscitation
compared with 90% to 95% for throat culture. Newer (CPR), patients without signs of life should receive
rapid antigen tests are reported to have comparable immediate care.
sensitivities to throat culture, potentially eliminating
delays in care. [I) Answer C. Patients with inferolateral knee pain
and tenderness after trauma are at significant risk for
[!] Answer D. Posterior shoulder dislocations occur tibial plateau fractures, which are often not visible on
much less commonly than anterior dislocations. plain knee radiographs. Unlike simple knee sprains,
Mechanisms include seizure (due to stronger in- management of tibial plateau fractures involves
ternal rotator muscles compared with external no weight bearing for several weeks. Diagnosis
rotator muscles) and fall on outstretched hand. should therefore be pursued with advanced imaging
Patients with posterior shoulder dislocation are al- techniques, such as CT or MRI. Discharging a patient
most never able to abduct or externally rotate their with potential for tibial plateau fracture with any
affected arms. Neurovascular injury is much less weight bearing will result in improper healing and
common than with anterior dislocations due to the increased morbidity. Observation of knee injuries
anterior position of the neurovascular structures. is not required except in cases of suspected knee
Posterior shoulder dislocation is often confused dislocation. Urgent arthroscopy is almost never
with adhesive capsulitis and may simply present as indicated acutely in cases of knee trauma.
stiffness and limited range of motion rather than
frank pain. Recurrent injury does occur, but less [!) Answer D. Otitis media is the number one rea-
commonly than in anterior shoulder dislocations. son for antibiotic prescriptions in the United States,
Management is with early reduction and orthope- despite the fact that more than 75% of cases will
dic consultation.
45
46 1(}()() Questions to Help You Pass the Emergency Medicine Boards

resolve spontaneously without treatment. The 2004 reason, some experts recommend bolusing asthmat-
American Academy of Pediatrics (AAP) and Amer- ics with one liter of crystalloid before intubation.
ican Academy of Family Physicians (AAFP) joint In the setting of the "crashing" patient, excessive or
guidelines recommend treatment with antibiotics for overzealous ''bagging" is often mistakenly applied
patients younger than 2 years old, and consider an in an attempt to resuscitate the patient. This creates
observation period for patients older than 2 years old the perfect setup for postintubation hypotension by
an appropriate management strategy, if the patient insuffiating the chest with a large volume of air. Be-
does not have a high fever and is not systemically cause connecting the ventilator was the trigger for
ill. A 3-day course of intramuscular ceftriaxone is as this patient's hypotension, the most prudent step is
effective as a 10-day course of amoxicillin. High-dose to immediately disconnect the ventilator and allow
amoxicillin is recommended for patients younger the patient to exhale for 30 seconds (the apnea test).
than 2 years of age, in day care, or with recent expo- Immediate recovery suggests auto-PEEP and lung
sure to antibiotics. Auralgan, a mixture ofbenzocaine hyperinflation ± hypovolemia and it responds to
and antipyrine, is a local anesthetic that may provide aggressive N fluids (frequently> 1 Lis required). Per-
some direct analgesia, but should not be used in sistent hypotension suggests tension pneumothorax
patients with tympanic membrane perforation. and should prompt an immediate needle thoracos-
tomy once the pneumothorax is located. Of course,
[!) Answer C. Noninvasive positive pressure venti- pneumothorax should be suspected anytime there is
lation (NIPPY) has revolutionized the treatment a sudden clinical deterioration. Needle thoracostomy
of COPD, cardiogenic pulmonary edema, as well may be required given severe hypotension± tracheal
as neuromuscular disease (e.g., myasthenia). Con- deviation.
traindications for NIPPY are noncompliant patients
(which is the most common for treatment fail- [llJ Answer D. This patient is manifesting signs of
ure), midfacial trauma (preventing an appropriate hepatic encephalopathy (HE). HE is graded on
fit for the mask), excessive secretions or retention a 4-stage scale, and its symptoms range from
of secretions, as well as a decreased sensorium with subtle personality changes and sleep disturbances
absent cough and pharyngeal reflexes. Recent gastric to confusion, disorientation, stupor, and coma.
surgery (because of possible gastric distension) and Although elevated ammonia levels may support the
vasopressor-dependent hypotension are relative con- diagnosis, they do not always correlate with the
traindications as well. Hypertension does not affect, severity of illness and normal levels do not exclude
nor is it affected by NIPPY. the diagnosis. Roughly 25% ofpatients will have non-
nitrogenous causes of encephalopathy. However,
~ Answer B. The patient has evidence of stress azotemia is the most common precipitant and GI
fracture of the tibia, which is almost always due to bleeding is a very common cause ofazotemia and may
overuse and occurs more commonly in women than frequently trigger or exacerbate HE. The treatment of
men. The tibia is the most common bone involved. choice is ammonia-lowering therapy with lactulose
Radiographs are usually normal acutely and only or neomycin in the case of azotemia-induced causes.
positive 50% of the time subacutely. Bone scan has Otherwise, treatment should be directed at the
much higher sensitivity than radiographs, but a much underlying precipitant.
lower specificity. Advanced imaging with CT scan and
MRI is not indicated in the vast majority of cases, as [!ID Answer D. Patients with new-onset psychosis
management is rarely affected. Standard treatment must be evaluated in the ED for treatable medical
with rest, ice, and nonsteroidal anti-inflammatory illness before being assigned a psychiatric diagnosis.
drugs (NSAIDs) results in resolution of almost all Signs ofa medical cause ofthe psychosis include acute
cases. Cast immobilization or surgery is used only for onset, older patient, visual hallucinations, disorienta-
refractory cases and severe symptoms. tion, and impaired consciousness. Abnormal physical
examination findings, such as abnormal vital signs,
Ill] Answer C. Postintubation hypotension is a com- aphasia, ataxia, and cranial nerve abnormalities usu-
mon problem in asthmatics, occurring in as many ally indicate a medical cause. Many pharmacologic
as 20% of intubated patients. The presence of severe agents can also cause psychosis, including corticos-
airflow obstruction can cause air trapping even in teroids, antihistamines, antidepressants, and sedative
the setting of normal minute ventilation. Air trap- hypnotic. True psychiatric disease is suggested by a
ping causes elevated intrathoracic pressures, which young adult patient, auditory hallucinations, gradual
decreases venous return and subsequently cardiac progression, flat affect, and intact orientation and
output and BP. These problems are exacerbated in consciousness.
patients who are relatively hypovolemic, and for this
Test2 47

[H) Answer C. The modified diagnostic criteria for 1 to 2 hours, and the ability to be delivered by a variety
migraine defines migraine headache as a headache of routes (IM, IV, subcutaneous [SC], endotracheal
lasting 4 to 72 hours, which includes at least two of [ET]). The duration is of prime importance, as
the following four symptoms: patients with opiate overdose who are given a one-
time naloxone dose in the ambulance often become
Unilateral pain
acutely intoxicated again once the naloxone wears
Throbbing
off. All opiates have longer durations of action than
Moderate to severe intensity
naloxone, even heroin, which can last as long as 2
Pain aggravated by movement
to 3 hours if used by the SC route. For this reason,
And one of the following two symptoms: all patients with severe opiate overdose should be
Nausea or vomiting monitored carefully in the ED at least as long as the
Photophobia or phonophobia expected peak effect of the particular opiate.
Unfortunately, many of these symptoms are
similar to those experienced by patients with cluster [!1) Answer A. Hypokalemia is more common and
headaches. However, the pain in cluster headaches generally better tolerated than hyperkalemia. Di-
is typically described as "boring" or "tearing," uretic therapy is the most common cause. Hypo-
despite being of severe intensity. Furthermore, pain kalemia primarily affects the cardiac (arrhythmias),
associated with cluster headaches is almost always musculoskeletal (weakness, rhabdomyolysis), GI
retro-orbital and in the temporal region (due to (ileus), and renal (nephrogenic diabetes insipidus,
involvement of the VI branch of the trigeminal metabolic alkalosis) systems. Neurologic manifesta-
nerve). Cluster headaches are not typically associated tions are not common. Cardiac dysrhythmias are the
with nausea or vomiting, nor photophobia or most serious complication although patients with-
phonophobia. The key difference between these out heart disease rarely have any complications. In
two headache syndromes is the cyclic nature of contrast, patients with acute or recent MI may de-
cluster headache exacerbations and the stereotypical velop ventricular fibrillation in the setting of even
presentation of patients with duster headaches. mild hypokalemia (five times increased risk if [K+]
Sumatriptan is useful for the acute treatment of <3.9 mEq per L. Therefore, recent recommenda-
patients with either headache syndrome. tions are to maintain serum [K+] >4.5 mEq per L
in such patients. However, the number or degree of
~ Answer E. Abruptio placentae is the most com- EKG changes does not correlate with the severity of
mon cause of third trimester bleeding and has a hypokalemia. Neither vomiting nor nasogastric suc-
correlation with increasing maternal age and cigarette tioning causes significant potassium loss. Potassium
smoking. However, preeclampsia is most commonly is the most prominent intracellular cation.
associated with placental abruption, especially in
severe cases. Interestingly, the incidence of abrup- [!j} Answer A. Pain with extraocular movements, de-
tion peaks between 24 and 26 weeks and then crease in visual acuity, proptosis, and ophthalmople-
steadily declines as patients progress to term. Pa- gia more commonly occur in orbital cellulitis than
tients most commonly experience vaginal bleeding in periorbital cellulitis. The large majority of cases of
and abdominal pain although patients may present orbital cellulitis result from direct spread of adjacent
with only one or neither of these symptoms. Abrup- infections such as ethmoid sinusitis, whereas peri-
tion often results in frequent, low-amplitude uterine orbital cellulitis typically results from hematogenous
contractions resulting in a uterus that is firm and spread. Aspergillus species cause a chronic orbital
frequently tender upon palpation. Disseminated in- cellulitis lasting from weeks to months. Orbital cel-
travascular coagulation is a dreaded complication. lulitis may cause blindness and extension inside the
Ultrasonography must be performed in all patients cranium to involve the dural sinuses and meninges.
with second- or third-trimester vaginal bleeding. Periorbital cellulitis cannot progress to orbital celluli-
However, ultrasonography is poorly sensitive for tis in the absence oftrauma due to a thick fibrous layer
placental abruption, missing approximately 50% of separating the orbit from more superficial tissues.
cases. In contrast, ultrasonography is a very good
modality for detecting placenta previa, the other ~ Answer C. Isopropanol is the second most com-
major cause of second- and third-trimester vaginal monly ingested alcohol after ethanol. Choices A, B,
bleeding. If placenta previa is present, abruption is and Dare less commonly ingested. Choice E, acetone,
less likely. is not an alcohol because it lacks a hydroxyl group.

~ Answer B. Naloxone is a pure opioid antagonist ~ Answer E. This patient has tinea capitis, which
with an extremely rapid onset of action, duration of is a fungal infection of the scalp usually caused
48 1(}()() Questions to Help You Pass the Emergency Medicine Boards

by Trichophyton tonsurans. Tinea capitis should be hypocalcemia, which may present as tetany. When
suspected in all prepubescent children with hair loss, the calcium-phosphate product is >70, precipita-
particularly if the hair loss is focal and incomplete or tion of calcium phosphate can occur in a variety of
associated with scale and lymphadenopathy. There tissues (e.g., renal stones). Apart from renal failure,
may be scattered hairs that are broken near the base any process which results in rapid, extensive cell
within a generalized area of alopecia, resulting in damage may cause phosphate to be released into
the appearance of black dots on the scalp surface the extracellular space in large amounts. Examples
("black dot" ringworm). Although topical agents include rhabdomyolysis, tumor lysis syndrome, and
such as selenium sulfide and topical ketoconazole hemolysis.
may reduce infectivity, they are insufficient to provide
effective monotherapy. In all cases, oral therapy with ~ Answer E. Although it may be counterintuitive,
griseofulvin (either micronized or ultramicronized) most patients develop primary spontaneous pneu-
is required. Transmission from person to person is mothorax while at rest. Traditionally, expiratory
common, and Trichophyton remains infectious in chest x-rays were thought to aid in the diagnosis
combs and hairbrushes for long periods. (Figure of pneumothorax. Because the relative size of the
reprinted with permission from Fleisher GR, Baskin chest cavity is thought to decrease during expiration,
MN. Atlas ofpediatric emergency medicine. Lippincott and since the size of the pneumothorax is theoret-
Williams & Wilkins; 2003:95.) ically constant, the pneumothorax should occupy
a greater fraction of the chest cavity and there-
~ Answer D. Pilonidal cyst and subsequent abscess fore be easier to detect upon expiration. Clinically,
formation is thought to occur as a result of however, expiratory films have not demonstrated
obstruction of a pilonidal sinus. Pilonidal sinuses much utility. Pneumomediastinum is a less com-
are thought to be present from birth although they mon but generally benign finding, and is frequently
are not clinically evident until late adolescence or self-limited. It is usually due to persistent eleva-
the early adult years. It is not clear why obstruction tions in intrathoracic pressure such as that caused
occurs, but it is much more common in hirsute by repetitive severe coughing, asthma exacerbations,
men than any other group of patients. Poor hygiene or seizures. In contrast, secondary pneumomedi-
and repetitive low-level trauma (such as in jeep astinum is a morbid diagnosis as a result ofsignificant
drivers in World War II) have also been implicated. underlying disease such as Boerhaave's syndrome.
Treatment in the ED a longitudinal incision offofthe Hamman's sign or crunch is a physical examination
sacral midline to prevent reaccumulation of hair and finding in the setting of pneumomediastinum. It de-
other debris. Despite this, recurrence is common. All scribes the crunch-like sound heard upon cardiac
patients with pilonidal abscesses should be referred auscultation as the heart expands against the medi-
to a surgeon for further management. astinal air. The most frequent physical examination
finding in pneumomediastinum is SQ emphysema,
~ Answer B. Otic symptoms are the earliest observed frequently in the neck. Male smokers have more
in salicylate toxicity. The Done nomogram is not than a 20-fold increased risk of developing a sponta-
predictive of serious pathology in salicylate toxicity neous pneumothorax, whereas woman smokers have
and is not used clinically (unlike the Rumack- a more than 10-fold increased risk. Other risk fac-
Matthew nomogram for acetaminophen toxicity). tors include tall height and cold weather (there is an
Metabolic acidosis due to uncoupling of oxidative increased incidence in the fall and winter).
phosphorylation is much more likely to be observed
than metabolic alkalosis. Despite the clear role of ~ Answer B. Central cord syndrome is the most
chronic aspirin and NSAID use in GI bleeding, acute common incomplete spinal cord syndrome. Unlike
toxicity causes far less common serious bleeding anterior cord syndrome, central cord syndrome
through the GI tract. Hypokalemia is far more typically results from forced hyperextension injuries
common than hyperkalemia with salicylate toxicity, in arthritic middle-aged and older adults. The
due to a variety of renal and extrarenal mechanisms. posterior ligamentum flavum is thought to buckle
Therapy with bicarbonate may further exacerbate and compress the cord against anterior osteophytes,
this potassium loss. resulting in a contusion to the central aspects of
the cord. Owing to the topography of the spinal
~ Answer D. Like hypermagnesemia, hyperphos- cord, extension injuries occurring at the level of the
phatemia is rare in patients without renal insuffi- cervical spine result primarily in flaccid paralysis of
ciency. Its manifestations are related to its effects on the upper limbs; to a lesser extent, spastic paralysis or
calcium and its rate of rise. A rapid rise in phosphate paresis of the lower extremities occurs in the setting
levels results in calcium chelation and subsequent of large cord lesions. Furthermore, distal muscles
Test2 49

are affected more than proximal muscles. There laxity of the thumb MCP joint, although MRI may
is variable sensory dysfunction, but because most be required for diagnosis. Owing to inadequate UCL
sensory neurons are located peripherally, sensory healing in the setting of a Stener lesion, all such
findings tend to be less prevalent. Finally, there patients require surgery.
should be preserved perianal sensation, voluntary
rectal motor function, big toe flexor activity, and ~ Answer B. Hemorrhage occurs in approximately
preservation ofthe bulbocavernosus refl.ex. Together, 15% of patients with PUD, although it is twice as
the presence ofthese functions is referred to as sacral likely in patients with duodenal ulcers as in patients
sparing. with gastric ulcers. Roughly half the number of pa-
tients with UGIB due to PUD present with melena
~ Answer C. Acute radiation entent:Js occurs in and hematemesis. Isolated melena occurs in only
20% to 70% of patients (varies widely depend- 20% of patients and isolated hematemesis occurs
ing on technique, dose, and patient factors) and in only 30% of patients. The mortality rate has re-
is more common than chronic radiation enteritis, mained relatively unchanged over the last 30 years,
which occurs in as many as 15% of patients. m- and is between 6% and 10%. Perforation is accom-
timately, many patients die from their malignant panied by hemorrhage in roughly 10% of cases.
process before the development of chronic radia- Therefore, these entities are not usually confused
tion enteritis, so the true incidence is unknown. upon evaluation.
The acute form typically occurs 3 weeks after the
onset of radiation and presents with crampy ab- ~ Answer D. Severe poison ivy requires systemic
dominal pain, tenesmus, and bleeding. It is rarely corticosteroids. Commercially available "dose
life threatening. The chronic form ordinarily oc- packs" should be avoided because they provide an
curs within 2 years of therapy but it can occur at inadequate amount of medication. In addition, a
any time after therapy and occurs later than 2 years prolonged course of prednisone treatment is gen-
in 10% of patients. In acute radiation enteritis, no erally required due to the high chance of rebound
diagnostic testing is typically required and manage- dermatitis if prednisone is discontinued abruptly.
ment is supportive. In chronic radiation enteritis, Therefore, a 14- to 21-day taper generally works best.
CT may only be useful to exclude the recurrence of
malignancy or other problems, but has no findings ~ Answer E. With a positive India ink stain of the
specific for radiation enteritis. cerebrospinal fluid (CSF), the patient has fungal
meningitis, most likely due to Cryptococcus neofor-
[!Z) Answer D. Patient or toxin odor may provide mans, an opportunistic infection common patients
important clues to the toxic agent. The smell of with AIDS. Patients usually present with typical
garlic is suggested by organophosphates, arsenic, or symptoms of aseptic meningitis. The CSF WBC
selenium. Cyanide smells like almonds, zinc has a and protein are usually only slightly elevated. In-
fishy odor, and toluene smells like glue. Hydrogen dia ink staining has approximately 80% sensitivity,
sulfide has the odor of rotten eggs. so CSF should also be sent for cryptococcal antigen,
which has close to 100% sensitivity and specificity.
~ Answer E. Gamekeeper's thumb is an avulsion Treatment for cryptococcal meningitis is with am-
injury of the ulnar collateral ligament (UCL) at the photericin B plus flucytosine, which is superior to
thumb-MCP joint. It most commonly occurs during itraconazole monotherapy. Fluconazole monother-
a skiing accident in which a patient's thumb is trapped apy may be used in very mild cases. Ceftriaxone and
in the loop of the pole, resulting in forced abduction vancomycin are used for therapy ofbacterial menin-
and extension of the thumb. An avulsion fracture gitis. Acyclovir is used for herpes simplex encephalitis
may also occur at the site of the UCL insertion. (HSE), which is suggested by altered mental status
Examination will reveal difficulty with pinching and and elevated CSF RBC count.
grasping, tenderness and swelling along the ulnar
aspect of the base of the thumb, as well as laxity of [!ID Answer C. Iron-deficiency anemia is the most
the MCP joint upon stress testing. Patients should common cause of all anemias in women of child-
be placed in a thumb spica splint and referred to a bearing age, likely due to menstrual blood loss.
hand surgeon for further evaluation. A Stener lesion Iron-deficiency anemia is the number one cause
occurs when the adductor pollicis tendon inserts itself of microcytic anemia and is characterized by a
between the two ends ofthe ruptured UCL. They may rapid response to oral iron therapy. Both a- and
occur in up to 29% of patients with UCL rupture P-thalassemia cause microcytic anemia, but are far
and will prevent healing of the UCL. A Stener lesion less common than iron-deficiency anemia. Microcy-
should be suspected in any patient with extreme tosis is more severe in patients with thalassemia than
50 1(}()() Questions to Help You Pass the Emergency Medicine Boards

with iron-deficiency anemia. Definitive diagnosis is ~ Answer E. All Rh-negative pregnant patients with
made by hemoglobin electrophoresis. Sideroblastic first-trimester vaginal bleeding should be given
anemia is usually found in elderlypatients, alcoholics, RhoGAM. The dose in the first trimester is 50 J.Lg,
and in those with lead poisoning. Folate deficiency, whereas the dose after the first trimester is 300 J.Lg.
although common, causes macrocytic anemia. RhoGAM has not been shown to cause fetal harm.
The classic teaching is that 50% of patients diagnosed
~ Answer A. The patient's head CT demonstrates an with a threatened abortion progress to spontaneous
acute cerebellar hemorrhage. Surgical intervention miscarriage. However, once a fetal heartbeat is
has been the mainstay of management of cerebel- identified on ultrasonograph, only 15% of such
lar hemorrhage. However, in awake patients with women will progress to spontaneous miscarriage
relatively small infarcts ( < 3 em), patients may be can- whereas the remainder will carry the pregnancy
didates for observation in an intensive care setting. All normally to term. Patients with threatened abortion
patients with cerebellar hemorrhage, however, may should be advised to carry out their normal activities
deteriorate rapidly due to hydrocephalus or pro- although patients are often advised to avoid tampons,
gressive brainstem compression. Owing to the local intercourse, and douching to prevent infection.
mass effect, the fourth ventricle may become com- Patients frequently confuse blood clots with tissue
pressed resulting in an obstructive hydrocephalus. and even in the presence oflaboratory confirmation
This requires emergent ventricular drainage. Owing of products of conception, a diagnosis of complete
to the possibility of rapid deterioration, all patients abortion is ill-advised. Only if a complete gestational
with cerebellar infarction should be admitted to an sac or fetus is present should a diagnosis of
intensive care setting. Corticosteroids and anticon- complete abortion be considered. Otherwise, an
vulsant& have no role in the management ofcerebellar ultrasound should be performed to determine if
hemorrhage. Corticosteroids help to reduce the va- retained products of conception are present. Finally,
sogenic edema associated with tumors, but not the approximately 80% of patients with first trimester
cytotoxic edema associated with infarction. Prophy- spontaneous abortion will complete the abortion
lactic anticonvulsant& have not proved to be useful, without intervention. Classic teaching is that all
and because seizures are initiated in the cortex, cere- patients diagnosed with an embryonic pregnancy
bellar lesions should not trigger epileptic events. (or blighted ovum), intrauterine fetal demise, missed
Herniation may occur in these patients, but results abortion, or incomplete abortion require surgical
in upward transtentorial herniation, not uncal her- evacuation. However, current research demonstrates
niation. Finally, the Cushing response may occur, that the volume of intrauterine contents is the best
but results in very high BPs, with systolic pressures predictor of the need for surgical evacuation and
in the neighborhood of 200 mm Hg. This degree most women do not require intervention.
of hypertension usually portends a bad outcome.
(Figure courtesy of Mark Silverberg, MD. Reprinted ~ Answer E. Patellar fractures usually occur due to
in Silverberg M. Greenberg's text-atlas of emergency direct blows to the patella from falls or motor vehicle
medicine. Lippincott Williams & Wilkins; 2004:46, collisions. They may be invisible on standard AP
with permission.) and lateral views of the knee, and patellar sunrise
views may be necessary for diagnosis. Transverse
~ Answer C. Pregnant women develop appendicitis patellar fractures are the most common type, but
at the same rate as nonpregnant women. Owing longitudinal fractures also occur and are easily seen
to nearby gynecologic structures, women with on sunrise view.
appendicitis are more frequently misdiagnosed. As
many as 33% ofwomen with appendicitis are initially [llJ Answer B. Elderly, diabetic and female patients
misdiagnosed, and as many as 45% of women with have atypical anginal symptoms. The extreme el-
symptoms ofappendicitis are found to have a normal derly (>85 years) experience shortness of breath
appendix during surgery. Although cervical motion as the most common symptom during an MI.
tenderness (CMT) is more common in gynecologic Nausea, typical chest pain, syncope, and fatigue
diseases, as many as one fourth of women with may also be present. The emergency physician (EP)
appendicitis have CMT upon physical examination. can never clinically rule out the diagnosis of acute
Even in the third trimester, most pregnant women coronary syndrome in elderly patients just because
still have pain in the rightlower quadrant. In pregnant they lack frank chest pain. Choices D and E occur
women, appendicitis occurs slightly more often in the more frequently in dehydration and infection than
second trimester, although the reasons for this are in MI.
not known. Fetal abortion complicates perforated
maternal appendicitis in 20% of cases.
Test2 51

~ Answer B. The peak expiratory flow rate (PEFR, hematoma volume, strict BP control may have a
in liters per second) and the FEV1 (in liters) are dramatic positive effect in such patients.
both valuable adjuncts in assessing the severity
of airflow obstruction. Because of its portability ~ Answer C. Hypertrophic cardiomyopathy is the
and ease of application, however, PEFR is much most common cause of atraumatic sudden death
more easily measured and is more useful for EPs. in young athletes. Choices A, B, and D are far less
Although the absolute value of the PEFR may be common in this age-group. Choice E can occur in
useful, it is most useful by comparing any value tall, thin athletes and although death may result
to a patient's typical best (as a percentage). A from tension physiology, it is almost never sudden
patient who generates a PEFR <50% of his or or without preceding symptoms.
her typical best has severe airflow obstruction. The
utilization of arterial blood gas (ABG) determination ~ Answer A. This patient's pleocytosis with a lym-
widely varies in clinical practice. However, because phocytic predominance in concert with a normal
of the accuracy of pulse oximetry monitoring, glucose and elevated protein level supports the diag-
only patients whose POx value is <90% despite nosis of a viral etiology. Herpes simplex encephalitis
oxygenation requires an ABG. Furthermore, repeat (HSE) is the most common cause of severe en-
ABGs are generally not needed to determine whether cephalitis in the United States despite the fact that
a patient is improving or deteriorating. Chest x-rays arboviruses such as West Nile virus continue to
are not generally useful in patients with asthma get significant media coverage. The most common
exacerbations. They do not reflect the severity of the symptoms of HSE are fever, altered mental status,
exacerbation. They are generally useful only for those decreased consciousness, and focal neurologic find-
patients in whom pneumonia, or a complication (e.g., ings. Focal seizures may also occur (temporal lobe
pneumothorax) is suspected, or in those patients who seizures). Because acyclovir is extremely well toler-
remain refractory despite optimal therapy. Neither ated and is very effective for treating HSE, it should be
continuous cardiac monitoring, nor routine EKGs started as soon as the diagnosis of HSE is suspected.
are useful in routine asthma exacerbations. Both of Dexamethasone is used as an adjunctive treatment in
these may be useful, however, in older patients with bacterial meningitis and should be given just before
coexisting cardiac disease. Most patients have pulse or with the first dose of antibiotics. Amphotericin B
rates between 90 and 120, and only 15% exceed is an antifungal agent.
this value.
~ Answer A. S. aureus remains the most common
~ Answer C. NAC promotes the metabolism of cause of septic arthritis, accounting for 37% to
acetaminophen into a nontoxic compound by 56% of infections. It causes 80% of the infections
sulfation through replenishment of glutathione. in patients with underlying diabetes or rheumatoid
Prevention of hepatic injury is complete when the arthritis. Group A ,8-hemolytic streptococci are the
first dose of NAC is given within 8 hours of acute second most common cause of septic arthritis. The
ingestion. Beneficial effects still occur as far out as remainder are caused by other streptococci, gram-
48 hours after ingestion, but efficacy in preventing negative organisms, and gonococcus. N. gonorrhoeae
hepatic injury decreases progressively starting at the accounts for only 20% ofmonoarticular septic arthri-
8-hour mark. tis, although it accounts for a larger proportion of
polyarticular septic arthritis, which is its usual pre-
~ Answer D. Patients with ischemic stroke should sentation. Gram-negative organisms are the most
not have their BP reduced unless they are either common cause of septic arthritis in newborns and in
candidates for tPA, have concomitant aortic dissec- children younger than 5 years old, although vaccina-
tion, MI or renal failure, or their pressure is above tion programs have nearly eliminated H. injluenzae
220/120 mm Hg. In fact, hypertension is probably as a cause.
neuroprotective in patients with ischemic stroke by
ensuring that cerebral perfusion pressure is main- ~ Answer E. Although the patient has clear evidence
tained. In contrast, patients with hemorrhagic stroke of brain or spinal injury from blunt trauma, no
should have their BP reduced to a target SBP of radiographic evaluation can take place until the
140 to 160 mm Hg (or the patient's prestroke level patient is cooperative. In cases where patients put
if it is higher). Lowering the BP in such patients themselves or caregivers at risk of physical harm, the
may reduce the stimulus for bleeding and prevent trauma team leader's first responsibility is to control
hematoma expansion. Because prognosis in patients the patient's behavior to prevent this occurrence.
with intracerebral hemorrhage is tightly linked to Sedating the patient with lorazepam or haloperidol
52 1(}()() Questions to Help You Pass the Emergency Medicine Boards

is the ideal first -line management, but if this is not an contraindicated in patients with tricyclic overdose
option, sedation and paralysis with rapid-sequence due to its potential for causing seizures and asystole.
intubation should be performed. This will allow Supportive care is more beneficial than physostig-
the trauma evaluation to take place expediently and mine therapy in most anticholinergic crises.
reduce the risk ofhann to the patient and caregivers.
Once the patient's combative behavior is controlled, ~ Answer A. Eighty to 90% of objects that have
CT of the brain and spinal cord, along with the rest made it into the stomach will pass through the
of the secondary survey may be initiated. remainder ofthe GI tract without difficulty. However,
as many as 15% to 35% of sharp or pointed objects
~ Answer C. The patient has evidence of bacte- may cause perforation if untreated. The ileocecal
rial vaginosis, a bacterial overgrowth process due valve is the most common site of perforation.
to polymicrobial infection with Gardnerella, My- Objects that are in the stomach are amenable to
coplasma, and anaerobes. Clue cells refer to vaginal endoscopic removal, but objects distal to this point
squamous epithelial cells lined with bacteria. The generally cannot be retrieved. Because this child is
sniff test (fishy odor of the discharge with addition asymptomatic, there is no indication for surgery
of potassium hydroxide) may also be positive in bac- at this point. Surgery is only indicated in cases
terial vaginosis. Treatment is with an antibiotic to of perforation, hemorrhage, or in cases of fistula
cover anaerobic bacteria-therapy must be instituted formation or obstruction. Because this child has
for at least 1 week, unlike in cases of TrichomotulS a relatively high risk of perforation, he cannot be
vaginitis, where one-time therapy with a 2 g dose of discharged home. Appropriate management includes
metronidazole is adequate. First-line therapy for bac- daily abdominal radiographs to follow the passage
terial vaginosis is with metronidazole 500 mg b.i.d. of the object. Surgical management should be
for 7 days, but clindamycin is an acceptable alterna- individualized but may be considered if the object
tive. Fluconazole is used to treat candidal vaginitis. fails to pass for a number of days. (Figure reprinted
Ceftriaxone is used to treat gonorrheal infections. with permission from Fleisher GR, Baskin MN. Atlas
Azithromycin is used to treat chlamydial infections ofpediatric emergency medicine. Lippincott Williams
and a number of other sexually transmitted diseases. & Wilkins; 2003.)

~ Answer C. The WBC count is a relatively non- ~ Answer E. Patients with periorbital (preseptal) cel-
specific test that is routinely ordered in the ED. A lulitis may have fever, periorbital edema, and eye
multitude of physiologic and pathologic states can tenderness, but ophthalmoplegia is characteristic of
cause leukocytosis, rendering the test useless in most orbital cellulitis. Gram-positive cocci are common
clinical situations. Any body stress, such as pain, causes ofboth conditions, but S. aureus is more com-
exercise, or pregnancy, can increase the WBC count monly implicated in periorbital cellulitis. Orbital cel-
to 20,000 cells per mm3• An elevated WBC has little lulitis is usually caused by Streptococcus pneumoniae,
specificity for the presence of infection. The pres- H. influenzae, M. catarrhalis, S. pyogenes and polymi-
ence of 95% neutrophils indicates a neutrophilic crobial infection. Diagnosis can be definitively made
predominance, not a ''left shift." A "left shift" is said by CT scan of the brain and orbits. Orbital celluli-
to be present when immature WBC forms such as tis mandates hospital admission with IV antibiotics,
bands, metamyelocytes, or myelocytes are detected whereas uncomplicated cases of periorbital cellulitis
in the circulation. The degree of leukocytosis (and may be managed on an outpatient basis.
even the differential) cannot accurately distinguish
between viral and bacterial infection. Few clinical ~ Answer C. The triad of anemia, thrombocytope-
situations cause an extremely elevated WBC count. nia, and renal insufficiency should prompt evaluation
Although chronic myeloid leukemia is one of the for either hemolytic uremic syndrome (HUS) or
common causes, other leukemias, the leukemoid re- thrombotic thrombocytopenic purpura (TTP). Fever
action, C. difficile colitis, and disseminated candidia- and neurologic signs and symptoms are more com-
sis can all cause a WBC count >50,000 cells per mm3 • mon in the latter, but the two are thought to be
on the same spectrum of disease. The toxin-forming
~ Answer D. Physostigmine is an acetylcholines- bacterium E. coli 0157:H7 is responsible for most
terase inhibitor that serves to antagonize the affect of epidemic cases of HUS. Treatment is primarily sup-
anticholinergic agents. It affects both nicotinic and portive, aimed at preventing complications of severe
muscarinic receptors and crosses the blood-brain anemia and thrombocytopenia. Plasmapheresis is
barrier. Despite its comprehensive cholinergic effects, used for cases of idiopathic HUS or TI'P. Shigella is
potential toxicity may occur during rapid administra- a less common precipitant of HUS than E. coli, but
tion, severely limiting its clinical use. It is absolutely Salmonelltl species and rotavirus are not implicated.
Test2 53

Child abuse would be unlikely to cause these symp- and variable visual loss primarily affecting central
toms in an epidemic manner. vision. However, sensory disturbances, diplopia (in-
ternuclear ophthalmoplegia), Lhermitte's sign (trunk
~ Answer C. In 2002, the Transient Ischemic Attack and limb paresthesias evoked by neck flexion), limb
Working Group redefined transient ischemic attack weakness, clumsiness, gait ataxia, and neurogenic
(TIA) as "a brief episode of neurologic dysfunction bladder and bowel symptoms may also be presenting
caused by focal brain or retinal ischemia, with signs and symptoms. Approximately 20% of patients
clinical symptoms typically lasting < 1 hour, and with MS will present with optic neuritis and 50%
without evidence of acute infarction." Positive of patients with MS will experience optic neuritis at
findings such as tingling or involuntary movements some point in the disease course.
are the exception in patients with a TIA. In
addition, symptoms affecting multiple different body ~ Answer B. Myringitis, which is inflammation of
parts usually occur simultaneously. "Marching'' the tympanic membrane, is a rare occurrence in the
symptoms are more common in patients with setting of Mycoplasma infections and is not required
migraines or seizures. Nonfocal symptoms such as to establish a diagnosis. Furthermore, the presence of
generalized weakness, dizziness, lightheadedness, and myringitis, bullous or not, is not pathognomonic for
confusion are not commonly due to a TIA. Finally, Mycoplasma infection as a host of other etiologies are
the most common mimic of symptoms attributable possible. Mycoplasma commonly occurs in outbreaks
to a TIA is hypoglycemia. in closed communities such as camps, military bases,
hospitals, religious groups, and facilities for the men-
~ Answer D. The patient has a total (or "eight-ball") tally ill. Neurologic complications occur in 6% to 7%
hyphema. The pupil is almost completely obscured of children hospitalized with Mycoplasma infections,
by blood in the anterior chamber due to the ocular and may include aseptic meningitis, encephalitis, and
trauma. ED management is with ophthalmologic Guillain-Barre paralysis, among other complications.
consultation, topical steroids (under the direction Cold agglutinins are autoantibodies directed toward
of the ophthalmologist only), head elevation, eye RBCs which have been antigenically altered by My-
shielding, and tetanus booster. The eye is shielded coplasma. Their titers do not rise to detectable levels
to prevent secondary injury and further bleeding. until approximately 2 weeks after infection and they
Trendelenburg positioning and NSAIDs will likely may not be present in all patients. Mycoplasma is not
worsen the bleed and are contraindicated. Although a common cause of pneumonia in the elderly.
factor VII has important procoagulant activities, it
has currently not been approved for use in patients ~ Answer E. The patient has acute bacterial pro-
with hyphemas. IV antibiotics are not indicated statitis. Fever, low back pain, and UTI symptoms
except in patients with evidence of globe rupture. are common. A warm, tender prostate is the char-
(Figure from Tasman W, Jaeger EA, eds. The Wills acteristic physical examination finding. The prostate
Eye Hospital atkls of clinical ophthalmology, 2nd ed.. should never be massaged because of the possibil-
Philadelphia: Lippincott Williams & Wilkins; 2001, ity of bacteremic spread. Etiology is almost always
with permission.) due to gram-negative enteric bacilli, most commonly
E. coli. Diagnosis is made by physical examination
~ Answer A. Many agents have been advocated, but combined with urinalysis, as there is often a concomi-
vinegar is the agent of choice. However, some tant cystitis. Treatment is with either trimethoprirn-
coelenterates have species-specific treatment and sulfamethoxazole or a fluoroquinolone. Treatment
most jellyfish stings require no treatment at all. must be continued for 1 month to assure clinical
However, in cases in which tentacles and nematocysts cure. Ceftriaxone and doxycycline are the drugs of
remain adherent to skin, vinegar should be liberally choice for young men with urethritis. Azithromycin
poured over the nematocysts before their removal. is an acceptable alternative for treating chlamydial
Tentacles and nernatocysts should then be removed infections. Three-day and 7-day regimens with a flu-
by a gloved hand. Fresh water and alcohol may oroquinolone are used for treating uncomplicated
induce further nematocyst discharge and should not UTI and complicated UTI, respectively, in women.
be used. Gently rinsing the wound with seawater is
advocated as part of general wound care but it does ~ Answer C. Myoglobin has excellent sensitivity and
not inactivate the toxin like vinegar. negative predictive value for acute MI early in the
course ofthe disease, beginning to rise within 2 hours'
~ Answer E. Optic neuritis is the most common cra- postinfarct, and peaking at 5 hours. The specificity
nial manifestation of MS, and describes a syndrome of myoglobin is lower than that of troponin I,
of monocular eye pain, decreased color perception, troponin T, or CK-MB, due to its release by skeletal
54 1(}()() Questions to Help You Pass the Emergency Medicine Boards

muscle during trauma and exercise. Myoglobin is far fracture. In an unstable patient with a pelvic fracture,
less useful as a test in patients who present > 12 hours a positive aspirate (i.e., the aspiration of gross blood
after onset of symptoms. upon entering the peritoneal cavity with the lavage
catheter) is suggestive of ongoing intraperitoneal
~ Answer C. Recent studies have demonstrated that hemorrhage and organ injury and is an indication
serum epinephrine levels are higher and rise faster for emergent laparotomy. If the aspirate is negative
in patients given epinephrine through IM injection but the lavage is positive by cell counts, or if the aspi-
in the lateral thigh instead of through SQ injection. rate and lavage are negative, pelvic angiography and
SQ administration results in variable absorption and stabilization is indicated. A positive diagnostic peri-
may be delayed by the vasoconstrictive effect of toneallavage (DPL) by cell count in the setting of a
epinephrine. Patients with epinephrine autoinjectors pelvic fracture but a negative aspirate is usually due to
are taught to inject epinephrine into the lateral retroperitoneal hemorrhage from the pelvic fracture.
thigh musculature (vastus lateralis). Although no Therefore, measures to stabilize pelvic bleeding (an-
trials assessing outcome and comparing SQ and IM giography, external fixators) should be undertaken
administration have been performed, the current as soon as possible.
recommendation is that epinephrine be delivered
through the IM route in the lateral thigh. ~ Answer D. In Western countries, CHF is the most
common cause of a pleural effusion. In the setting
~ Answer A. The patient has evidence of an incar- ofCHF, pleural effusions are most commonly bilat-
cerated hernia. The presence of fullness and bowel eral although they may present unilaterally. Isolated
sounds in the scrotum indicates a hernia, and tender- right-sided pleural effusions are nearly twice as com-
ness with inability to easily reduce the hernia indicates mon as isolated left-sided pleural effusions in the
incarceration. Ice packs should be applied to the area setting of CHF. When the effusion is bilateral, the
to reduce the attendant bowel edema and an attempt volume of fluid in each pleural space is roughly equal.
should be made to reduce the hernia, under proce- All of the other etiologies listed may cause a pleural
dural sedation if necessary. Operative reduction and effusion, although each of the other etiologies clas-
IV antibiotics would likely be necessary if the patient sically causes an exudative effusion. Differentiating
presented with signs of strangulation such as fever, between an exudative and a transudative effusion is
acidosis, and/or severe tenderness. Outpatient refer- best accomplished by performing a thoracentesis and
ral for an incarcerated hernia is not appropriate. Oral applying Light's criteria. Exudative effusions meet at
hydration may maintain euvolemia, but the possibil- least one of the three Light's criteria, whereas tran-
ity of procedural sedation or surgery should preclude sudative effusions meet none of the Light's criteria
any oral intake. Incarcerated hernia is a clinical diag- (ratio of pleural fluid protein to serum protein >0.5;
nosis and urinalysis will not aid diagnostic accuracy. ratio of pleural fluid lactate dehydrogenase (LDH)
to serum LDH >0.6; pleural fluid LDH greater than
~ Answer D. Patients with primary adrenal insuf- two thirds the upper limit ofnormal for serum LDH).
ficiency develop hyperpigmentation due to in- Emergent thoracentesis is rarely indicated, and only
creased corticotropin (adrenocorticotropic hormone necessary to stabilize a patient's respiratory or cir-
[ACTH]) secretion and its effects on melanocyte culatory status (e.g., in the rare circumstance of a
receptors. Adrenal insufficiency secondary to pitu- tension hydrothorax, or in the case of a very large
itary insufficiency precludes elevated ACTH levels, effusion causing severe atelectasis and resulting in
so hyperpigmentation is not seen. Hyponatremia, inadequate ventilation).
hyperkalemia, and hypoglycemia are common elec-
trolyte abnormalities as a result of adrenal insuffi- ~ Answer D. Preseptal cellulitis is the most com-
ciency (specifically mineralocorticoid insufficiency). mon complication of acute sinusitis. Orbital cellulitis
Therefore, hypokalemia may occur as a result of may also occur and is often difficult to distinguish
replacement therapy with prednisone or hydrocor- from preseptal sinusitis based on clinical examina-
tisone (but not with dexamethasone, which does tion alone. cr scans are generally able to differentiate
not have mineralocorticoid activity). Nausea and between preseptal and orbital cellulitis. They also
vomiting are common GI manifestations of adrenal provide additional information about neighboring
insufficiency. structures, including the sinuses. Cavernous sinus
thrombosis is a rare, but life-threatening compli-
~ Answer D. Hemodynamically unstable patients cation that results from extension of the infection
with evidence of intra-abdominal hemorrhage through valve-free veins to the cavernous sinus. Pa-
should be taken to the operating room (OR) for tients typically present with severe headache as well as
emergent laparotomy, even in the setting of a pelvic CN III and VI palsies along with retinal engorgement,
Test2 55

chemosis, proptosis, and a high fever. Like cavernous Therefore, in symptomatic patients with critical hy-
sinus thrombosis, meningitis is another uncom- ponatremia (sodium <120 mmol per L), hypertonic
mon intracranial complication. Other complications saline must be given. Neoplasms are the most com-
include brain abscesses, subdural empyema, or- mon cause of SIADH and small cell lung carcinoma
bital abscesses, maxillary cellulitis, and localized is the most common neoplasm associated with the
osteomyelitis. disorder.

~ Answer E. Labetalolantagonizesa-1, tJ-1, andtJ-2 [ill Answer D. The patient has evidence of acute
adrenergic receptors. The tJ-blockade is much greater epididymitis. In men older than 35 years, the
than the a-blockade, although both function to pro- most common cause is E. coli. In men younger
vide rapid, predictable BP control without causing than 35 years, Chlamydia is the number one
reflex tachycardia. This blunting of the tachycardic cause, followed by gonococcus. It is crucial for
response along with concomitant arteriolar vasodi- the EP to distinguish between testicular torsion
lation makes labetalol ideal as the initial single agent and epididymitis. Epididymitis is characterized by
for management of aortic dissection, where reduc- the gradual progression of symptoms, dysuria,
tion of heart rate is as important as reduction ofBP. and the presence of cremasteric reflexes. Focal
Orthostatic hypotension with oral labetalol is rare, epididymal swelling is followed by generalized edema
but IV formulations can cause major changes. Ni- and erythema of the scrotum. Low-grade fever is
troprusside and labetalol are roughly equal in their present in most patients. Only half the number
ability to control BP rapidly and predictably. of patients with epididymitis have leukocytes in
the urine. Treatment involves antibiotics to cover
~ Answer A. Tetanus is a serious, toxin-mediated suspected organisms based on age of the patient:
disease with a mortality of up to 50%. Clostridium Ceftriaxone plus doxycycline for patients younger
tetani, a ubiquitous, gram-positive anaerobic bac- than 35 years, and trimethoprim-sulfamethoxazole
terium, is the causative organism, producing or ciprofloxacin for patients older than 35 years.
tetanospasmin toxin which is responsible for the
pathologic effects. Though clinical tetanus is rare ~ Answer A. The patient likely has a rib fracture or
in the United States, the underimmunized popula- contusion given the focal pain and tenderness in her
tion is steadily increasing with immigration. Patients fifth rib. However, the true danger of a rib fracture
are inoculated with the bacteria during any break is not the bony injury itself, but potential injury
in the skin, such as simple wounds. Approximately to the underlying structures, such as pleura, lung,
one third of patients with clinically evident tetanus liver, spleen, or kidney. The fifth rib is cephalad
do not recall a preceding injury. Skeletal mus- enough that an intra-abdominal injury would be
cle spasm affecting the upper respiratory tract is less likely than thoracic injury. A chest x-ray is
the most common cause of death. Diffuse muscle indicated to evaluate the lung parenchyma and
rigidity, autonomic instability, hyperthermia, and pleural lines. Rib x-rays are not routinely indicated
rhabdomyolysis all accompany the pharyngeal and in patients with thoracic trauma, except in cases
facial muscle spasm. Tetanus is a clinical diagno- where multiple fractures are suspected or elderly
sis and cultures have no role in acute diagnosis or patients are involved, as significant intra-thoracic or
screening. Tetanus prophylaxis is initiated in child- intra-abdominal injuries occur at higher frequency
hood with a series of three vaccines, followed by in these instances. Abdominal CT scan may be
boosters every 10 years for clean, uncontaminated indicated with corroborative physical examination
wounds. Tetanus immunoglobulin is given to pa- findings, but is never undertaken before routine
tients with dirty wounds who have never undergone x-rays of the chest or pelvis in the trauma patient.
primary vaccination. Treatment of clinically evident Brain CT scan and cervical spine radiographs are
tetanus involves antibacterial therapy with penicillin not indicated without loss of consciousness, altered
and/or metronidazole, tetanus immunoglobulin, and mental status, focal neurologic deficit, headache, or
aggressive supportive care with early intubation and neck pain/tenderness.
tracheostomy placement.
~ Answer A. The palatine tonsils are structures of
~ Answer C. In the setting of SIADH, hypertonic lymphoid tissue in the posterolateral aspect of the
saline (3%) must be given in order to correct hy- oropharynx situated between the palatine arches
ponatremia. When isotonic normal saline is given (palatoglossal arch anteriorly and palatopharyngeal
(0.9%), the body "desalinates" it, delivering the salt arch posteriorly). These are the tonsils to which
to the kidneys to make concentrated urine, retain- most people are referring when they use the word,
ing the free water, and worsening the hyponatremia. "tonsils." The single pharyngeal tonsil (also known as
56 1(}()() Questions to Help You Pass the Emergency Medicine Boards

the adenoid) is actually located at the posterior aspect fluid-filled alveoli are difficult to distend). Owing
of the nasopharynx, superior to the palatine tonsils. to the presence of poorly compliant alveoli, both
The lingual tonsils are collections oflymphoid tissue peak and plateau airway pressures in ARDS are
along the dorsal base of the tongue. Peritonsillar higher than in healthy subjects. Ventilating such
abscesses (also known as quinsy) occur near the patients with "normal" or high tidal volumes pushes
superior pole of the palatine tonsils, thought to arise airway pressure higher and may result in barotrauma
from a suppurative infection ofthe adjacent tonsil or and further injury to the lung. The ARDS Network
from obstruction of Weber's glands at the superior group of investigators published a study in 2000
pole of the palatine tonsil. describing a "lung protective" strategy in which
such patients were ventilated with tidal volumes
[!g) Answer C. This image reveals uterine prolapse. that were much lower than normal. The idea of this
There are no contraindications to the manual strategy is to reduce ventilator-associated lung injury
reduction of uterine prolapse, although attempts as a result of alveolar over-distension from high
at reduction may not be successful. The primary tidal volumes and airway pressures. Owing to the
indication for reduction is symptomatic improve- low tidal volumes, however, such patients require
ment. After reduction, the patient should be fitted higher PEEP levels to recruit alveoli in order to
with a pessary to prevent recurrence. The pessary ensure adequate oxygenation. The goal is to use the
is a short-term solution that may serve as a bridge lowest PEEP required to achieve an Fio2 :::=: 0.50. An
to surgical repair. Pessaries should not be used in inadvertent result of this strategy is hypoventilation
the setting of concomitant genital tract infection. and a resultant rise in Paco2 levels (hypercapnia).
(Figure courtesy of Mark Silverberg, MD. Reprinted Some studies suggest that the resulting acidosis
in Silverberg M. Greenberg's text-atlas of emergency (respiratory acidosis from high Paco2 levels) may
medicine. Lippincott Williams & Wilkins; 2004:353, actually be protective, although the ARDS Network
with permission.) investigators treated the acidosis with bicarbonate
infusions. An ideal strategy regarding this has not yet
!!!] Answer C. Propoxyphene has sodium channel- been developed.
blockingproperties that can cause QRS prolongation,
especially when used in conjunction with other med- [MJ Answer E. Although many infants outgrow their
ications that block sodium channels (e.g., tricyclic allergies to food (e.g., eggs, milk, and soy products),
antidepressants). Fentanyl may cause chest wall rigid- adults do not. Honeybees and wasps are both part
ity. Meperidine can cause seizures due to a toxic of the order Hymenoptera, which is a large order
metabolite (normeperidine) and may lead to sero- of insects that is characterized by locked pairs of
tonin syndrome when used with other serotonergic membranous wings. Honeybees and bumblebees
agents. Hydrocodone has few extragastrointestinal are part of the Apidae family, whereas wasps,
side effects. Morphine can cause pruritus and hy- yellow jackets, and hornets are part of the Vespidae
potension due to histamine release. family. Although cross-reactivity is common among
members of the vespid family, little cross-reactivity
~ Answer B. Pseudosubluxation of C2 on C3 is a occurs between different families of hymenoptera.
common variant seen in almost half the number of Only 5% to 25% of adverse drug reactions are
pediatric patients younger than 8 years. Pseudosub- allergic. Topical or parenteral allergen exposure
luxation of C3 on C4 occurs in almost 15% of cases. each provokes a more significant anaphylactic
Increased ligamentous laxity and more horizontally response than oral exposure. Atopy refers to the
positioned facet joints in young children are the main genetic predisposition to develop hypersensitivity
causes. Unfortunately, this makes interpretation of to environmental allergens as is characterized by
cervical spine films more difficult than in the adult, asthma, allergic rhinitis (hay fever), and atopic
especially because most cervical spine fractures in dermatitis. It also predisposes patients to develop
children occur in the Cl-3 region. Pseudosubluxa- anaphylactic reactions.
tion is present when the spinolaminalline connecting
the anterior portions of the spinous processes of Cl ~Answer B. Wolff-Parkinson-White (WPW) syn-
and C3 is within 2 mm of the C2 spinous process. drome is the most frequently occurring accessory
Any greater distance indicates true subluxation. pathway syndrome. Patients have an accessory con-
ductive pathway from the atria to the ventricles
~ Answer D. In acute respiratory distress syndrome which pre-excites the ventricular myocytes before
(ARDS), the alveoli are flooded with protein-rich the AV node releases the normal sinoatrial depolar-
fluid due to leaky pulmonary capillaries. The result is ization. As a result, patients with WPW often have a
poorly ventilated and poorly compliant alveoli {i.e., shortened PR interval and a delayed QRS upstroke,
Test2 57

called the delta wave. The accessory pathway can ischemia. Radiation of pain is a variable feature
cause reentrant dysrhythmias, of which atrioventric- of both entities and should not be relied upon to
ular nodal reentry (AVNRT) is the most common. In differentiate between the two. Both types of pain
reentrant dysrhythmias, the accessory pathway can may be precipitated by emotional or physical stress
conduct retrograde (where the AV node conducts in and may abate once that stress is relieved (either
the normal direction, producing a narrow QRS com- by rest or relaxation). Nevertheless, the pain due to
plex and an "orthodromic" pattern) or anterograde GERD is three times more likely to radiate to the
(where the AV node conducts backwards, producing abdomen than in cardiac ischemia. In addition, pain
a wide QRS complex and an "antidromic" pattern). due to GERD is more common after meals, and
Multifocal atrial tachycardia (MAT) is seen in the set- patients may complain of a feeling of "fullness." Of
ting of chronic pulmonary disease and is not usually course, the most common manifestation of GERD is
seen with WPW syndrome. AV blocks and torsade de heartburn, although patients with cardiac ischemia
pointes are not commonly seen in WPW syndrome. may again feel similar symptoms.

[!!) Answer C. Laboratory testing has a limited, but [!!) Answer B. The slit lamp examination demon-
important role in the evaluation of patients with strates dendritic lesions with fluorescein uptake char-
abdominal trauma. The hematocrit is primarily im- acteristic of herpes simplex keratitis. The dendritic
portant because it establishes a baseline value before lesions may scar, and ocular HSV is a common
resuscitation and redistribution. However, although cause of corneal blindness in the United States. Man-
the hematocrit is primarily useful for following se- agement involves oral and/or topical antivirals and
rial levels in the setting ofsolid organ injury, it is also ophthalmologic consultation to assess for surgical
valuable in any patient who presents with a very low management. Topical steroids are absolutely con-
hematocrit in the setting of trauma because it most traindicated as they may cause worsening of the
likely indicates that significant blood loss has already corneal epithelial defect. Acetazolamide is used to
occurred. Liver enzymes are not helpful and are not increase aqueous humor excretion as part of non-
used to distinguish between minor and major liver invasive temporizing therapies for acute glaucoma
contusions or lacerations. Although often elevated in attacks. Erythromycin is used to treat corneal ulcers
the setting of pancreatic injury, serum amylase and or prevent infections from occurring in patients with
lipase are nonspecific and poorly sensitive. There- corneal abrasions. Ceftriaxone is used in patients with
fore, normal levels do not exclude pancreatic injury hyperacute bacterial conjunctivitis, which is usually
and high levels may be due to a host of other causes. due to gonococcal infection. (Figure from Tasman
Although conflicting data exist regarding the utility W, Jaeger EA, eds. The Wills Eye Hospital atlas ofclin-
of microscopic hematuria in pediatric patients, most ical ophthalmology, 2nd ed. Philadelphia: Lippincott
authors agree that any degree of microscopic hema- Williams & Wilkins; 2001, with permission.)
turia in pediatric patients is an indication for cr
scanning. Various thresholds have been established, ~ Answer B. Rabies is a virus that carries virtu-
but at least a few studies have correlated any amount ally 100% mortality. Humans contract the virus
of microscopic hematuria with intra-abdominal or- from being bitten by infected animals. Bats are
gan injury. CT scanning has supplanted N pyelo- the number one vector, followed by skunks, rac-
gram in this regard. Furthermore, the presence of coons, and foxes. Dogs and cats may also carry
microscopic hematuria may signify injury to the liver rabies, but the vast majority of domesticated ani-
or spleen as well as the kidneys. mal bites do not cause rabies, due to vaccination
programs. The virus is transmitted from the an-
[D) Answer D. Hyperemia and erythema are expected imal's saliva through open skin into the victim's
findings upon rewarming frostbitten tissue, but bloodstream. It then jumps onto peripheral nerves
a residual violaceous color is an ominous sign. and travels up the spinal cord to the brain. The
Positive prognostic signs include a return to normal incubation period in humans lasts 1 to 3 months.
pliability, early return of normal sensation, and early A nonspecific viral prodrome affects most patients.
formation of large clear blebs in the affected area. Specific symptoms of rabies include altered mental
Persistent firmness of the SQ tissue, lack of edema, status and hydrophobia (inability to swallow water
or the delayed development ofhemorrhagic blebs all or saliva due to hyperactive airway reflexes). Coma
portend a worse outcome. and death are inevitable, and no effective treatment
exists once rabies is clinically evident. Postexposure
~ Answer E. The pain due to gastroesophageal reflux prophylaxis, consisting of local wound care, hu-
(GERD) may radiate to all of the same places man rabies immune globulin (HRIG), and rabies
that commonly occur in the setting of cardiac vaccine, is essential to prevent rabies infection.
58 1(}()() Questions to Help You Pass the Emergency Medicine Boards

~ Answer D. Amniotic fluid embolism (AFE) is a between the superficial and deep fascia of the
rare complication of pregnancy also known as the neck. There are no vital structures superficial to
anaphylactoid syndrome of pregnancy. Through a the platysma. Therefore, patients with penetrating
still unclear mechanism, amniotic fluid gains entry trauma who have an intact platysma can be given
into the maternal circulation and triggers as im- local wound care and discharged for routine follow-
mense inflammatory cascade resulting in pulmonary up. However, although neck wounds may be carefully
vasoconstriction, pulmonary capillary leak, and my- examined to determine whether the platysma is
ocardial depression. Clinically, patients develop acute intact, wounds should never be probed outside of
hypoxia, hypotension, and altered mental status. the operating room. If, after shallow examination,
Disseminated intravascular coagulation and seizures there is any uncertainty regarding platysma integrity,
may also occur. In patients with eclamptic seizures, a surgeon should be consulted for further evaluation
however, hypertension will be present instead of the and treatment.
profound shock ofAFE. AFE most commonly occurs
during labor and delivery or within 30 minutes of ~ Answer B. Diverticulosis is the most common
delivery although it may occur during the second cause of lower gastrointestinal bleeding (LGIB),
and third trimesters and it has been reported as a which may be massive in 5% of cases. However,
rare complication ofamniocentesis. Treatment is en- most cases ofLGIB due to diverticulosis are mild and
tirely supportive and the mortality rate exceeds 60%. resolves without intervention. Interestingly, although
Neurologic deficits are common among survivors. most diverticula are located in the left (descending) or
sigmoid colon, most bleeding diverticula are located
~ Answer B. Odontoid fractures are the most com- in the right colon. This is fortuitous, because most
mon cervical spine fractures in the elderly. They cases of angiodysplasia, a common cause of severe
usually result from falls from standing height with LGIB in the elderly, are also located in the right
accompanying head trawna. For this reason, the colon. Angiographic studies performed to localize
importance of an adequate open-mouth odontoid bleeding, therefore, canalize the superior mesenteric
radiograph cannot be overemphasized. More than artery (SMA) first.
one third of cervical spine fractures in the elderly
involve multiple vertebrae, which are usually (but (@l) Answer E. Traumatic subarachnoid hemorrhage
not always) contiguous. Most cervical spine fractures (SAH) results from bleeding in the subarachnoid
in the elderly (as in children) involve Cl-3. vessels, causing blood to accumulate in around
the brain parenchyma and in the sulci. It is an
~ Answer E. Though a transient tachycardia may oc- extremely common cause of traumatic intracranial
cur, sinus bradycardia develops and the heart pro- hemorrhage. Symptoms include headache, vomiting,
gressively slows as the body cools. In fact, the finding and photophobia, but focal neurologic deficits are
of a relative tachycardia in a hypothermic patient rare due to the diffuse nature of the bleeding and
should trigger a search for a cause. The PR, QRS, and generally low incidence of accompanying increased
QT intervals may all become prolonged although this intracranial pressure. Cerebral vasospasm may occur
finding is most common in the QT segment and also as a result of the SAH and can be attenuated by the
correlates with the degree ofhypothermia. J waves, or use of peripheral calcium channel blockers such as
Osborn waves, are the most discussed EKG feature in nicardipine.
hypothermia. They are additional waves at the junc-
tion of the QRS complex and the ST segment and [iiJ Answer D. The patient has a prominent lateral
tend to be upright in aVL, aVF, and the lateral pre- clavicular border with the deltoid and humeral
cordial leads. Any atrial or ventricular dysrhythmia margin displaced inferiorly. The most likely cause
may occur, but atrial fibrillation is the most com- for this is an acromioclavicular separation due
mon and it usually spontaneously converts to sinus to a torn acromioclavicular ligament. Radiographs
rhythm upon rewarming. will demonstrate this injury more definitively.
Management involves sling immobilization acutely
~ Answer C. Legionella spp., along with Chlamydia with orthopedic follow-up. If there were a shoulder
spp. and Mycoplasma spp. all respond to macrolide dislocation, it would be impossible for the patient to
therapy. The remaining organisms are usually treated touch his opposite shoulder with the affected hand.
with cephalosporins or fluoroquinolones. A sternoclavicular dislocation is not apparent due to
the lack of abnormalities noted on the medial sternal
~ Answer A. The platysma is the most superficial border. (Figure courtesy of Mark Silverberg, MD.
muscular layer of the neck, as it is sandwiched Reprinted in Silverberg M. Greenberg's text-atlas of
Test2 59

emergency medicine. Lippincott Williams & Wilkins; pus may be highly viscous, a needle placed into the
2004:501, with pennission.) center of an abscess may not drain any material,
resulting in a falsely negative aspirate. Patients with
~Answer E. Approximately 30% to 50% of pa- suspected PTAs but negative aspirates can undergo
tients with adenovirus pharyngitis have an associated cr to determine ifan abscess is present.
conjunctivitis, which is typically a unilateral, fol-
licular conjunctivitis. Patients presenting with con- ~ Answer B. This patient has Horner's syndrome
junctivitis, pharyngitis, and fever are said to have caused by disruption to the sympathetic fibers
pharyngoconjunctival fever, which is diagnostic for that encircle the carotid artery. Division of the
adenovirus infection. No further testing is required. sympathetics results in ptosis, miosis, and anhidrosis
Outbreaks of community-acquired pharyngocon- on the side of the injury and can occur after either
junctival fever due to adenovirus have been attributed blunt or penetrating trauma. Though the presence
to exposure to water from contaminated swimming of Homer's syndrome is not a life-threatening
pools as well as fomites from shared pool towels. emergency, it may represent a life-threatening
The EP should keep this in mind when treating vascular emergency due to the proximity of the
young children with pharyngitis and conjunctivitis sympathetic chain to the carotid artery. Therefore,
in the summer. In the absence of conjunctivitis, it all patients with Horner's syndrome should have a
is not possible to differentiate adenovirus-induced definitive evaluation of the carotid artery to exclude
pharyngitis from group A ,8-hemolytic streptococcal intimal injury (e.g., carotid angiography or helical CT
pharyngitis as both organisms may cause an intense angiography). Interestingly, delayed presentation of
exudative pharyngitis. neurologic deficits is typical of vascular injuries to
the neck due to blunt trauma. In the absence of
~ Answer C. The patient presents with a truncal, Homer's syndrome, most patients experience stroke
morbilliform rash with history of pharyngitis and symptoms between 1 and 24 hours after injury due to
concomitant antibiotic use. His rash is likely due to carotid or vertebral artery dissection or thrombosis.
an aminopenicillin in association with viral infectious Vascular injury should be suspected in all patients
mononucleosis. Classically caused by ampicillin, with neurologic findings that are incongruent with
this rash may also be caused amoxicillin. Any of head cr findings.
the antibiotics listed may cause a drug rash, but
amoxicillin is most likely to do so in the presence of ~ Answer A. Although the patient has a history
infectious mononucleosis. (Figure from Ostler HB, of rheumatoid arthritis, which may be responsible
Maibach HI, Hoke AW, et al. Diseases of the eye and for the symptoms, it is crucial to exclude septic
skin: A color atlas. Philadelphia: Lippincott Williams arthritis by aspiration of the joint and synovial
& Wilkins; 2004, with permission.) fluid analysis. Rheumatoid arthritis, like any process
that causes joint destruction, predisposes patients to
!!!) Answer D. Polymicrobial infections are the rule developing septic arthritis. MRI is not useful in the
in peritonsillar abscess (PTAs). Although Fusobac- acute setting and will unnecessarily delay appropriate
terium sp. are isolated, so are numerous other management. Colchicine and indomethacin are used
anaerobic and aerobic bacteria, including Peptostrep- for acute gouty flares and without a clear diagnosis
tococcus sp., Porphyromonas sp., and Prevotella sp., of crystals in the synovial fluid, neither is indicated.
group A .fJ-hemolytic Streptococcus, H. infiuenzae, Stress-dose steroids may be reasonable in this patient,
and S. aureus. Historically, many patients with PTAs but should not precede evaluation for a septic joint.
were subjected to immediate tonsillectomy. How-
ever, either needle aspiration or incision and drainage ~ Answer A. Choice B is seen in glaucoma, choice C
are effective therapy in concert with antibiotics. Most in central retinal vein occlusion, choice D in vitreous
of the debate in the management of PTAs focuses hemorrhage, and choice E in macular degeneration.
on the best drainage strategy. To date, the evidence
does not clearly favor one method, and both meth- ~ Answer A. Owing to anatomic and physiologic
ods appear to be effective. Regardless of the method differences, pediatric blunt trauma patients tend
used, drainage should be performed with the patient to exhibit more injuries to specific organs than
sitting upright to reduce the chance of aspiration. adults. Hypotension is an extremely late sign
Suction equipment should also be assembled at the of hemorrhagic shock in pediatric patients and
bedside to remove purulent debris. It is not possi- often signals imminent cardiac arrest. Extreme
ble to accurately differentiate between the presence tachycardia in the pediatric patient may compensate
of a PTA and peritonsillar cellulitis upon physical for severe acute blood loss and maintain an adequate
examination. Furthermore, because collections of cardiac output for a brief period of time. Most
60 1(}()() Questions to Help You Pass the Emergency Medicine Boards

pediatric patients lack sufficient amounts of adipose from the site. Choices A, B, and D are all reasonable
tissue to withstand the cold environment of the choices. The distal extremities, especially the penis,
trauma resuscitation room and unwarmed IV fluid should not be used as HRIG sites. Rabies vaccine
administration, putting them at much higher risk should be administered at a site distant from both
than adults for serious hypothermia. Solid organs, the bite and HRIG. Along with passive and active
including liver, spleen, and kidney, are more often immunization, local wound care is extremely impor-
injured in pediatric patients due to the lower amount tant as soap and povidone-iodine cleansing solutions
of surrounding extra-abdominal soft tissue. Because are 100% virucidal when used early enough.
children's heads are proportionally larger than those
of adults, serious head injury is more common in ~ Answer A. Octreotide is a longer-lasting, synthetic
children. analog of somatostatin. It inhibits the release of
several vasodilatory hormones such as glucagon and
~ Answer B. The prevalence of mitral valve prolapse has direct effects on vascular smooth muscle. The
(MVP) is 10%. MVP most commonly causes chest end result of its effects is selective vasoconstriction
pain and palpitations, but lightheadedness and of the splanchnic vasculature, thereby decreasing
dyspnea may also occur. Overt signs of heart pressure and bleeding in existing esophageal varices.
failure such as peripheral edema and orthopnea are Propranolol is useful as a prophylactic adjunct to
rare. Patients with MVP are at increased risk for prevent rebleeding. Vasopressin has been used in
dysrhythmias, endocarditis, and sudden death. the past to control variceal hemorrhage but it
has prohibitive side effects, including myocardial
~ Answer B. Copperheads are pit vipers that are part ischemia. Therefore, it is currently not recommended
of the Crotalidae family. Copperheads are typically for routine use.
found in the eastern United States and are generally
considered less toxic than other pit vipers, rarely [tool Answer B. The patient has a dermatomal, vesic-
requiring treatment. Mild envenomations are dis- ular rash consistent with shingles, caused by
tinguished by the presence of only local symptoms VZV. Shingles is due to reactivation of dormant
and minimal swelling. Patients with swelling that VZV-prirnary VZV is also known as chickenpox.
extends beyond 15 em from the bite site or patients Immunocompromised patients may have dissemi-
who develops systemic symptoms or abnormal co- nated zoster infection, which spreads beyond the
agulation tests have experienced moderate to severe initial dermatome and systemic involvement. Pa-
envenomation. tients with shingles may require antiviral therapy
with acyclovir (or equivalent), pain medication, and
[!!] Answer C. Human rabies immune globulin steroid therapy. Recurrent infection and posther-
(HRIG) should be administered as close to the bite petic neuralgia are the most common complications.
site as possible. When this is not suitable (such as in (Figure reprinted with permission from Fleisher GR,
the digits), give as much HRIG as possible at the bite Baskin MN. Atlas of pediatric emergency medicine.
site and the remainder at any skeletal muscle distant Lippincott Williams & Wilkins; 2003:214.)
Test 3
Questions
mWhich of the following is the most common
complication of otitia media?
rapidly progresse.. to sepsis and respiratory
failure.
(D) The most common causes are Mympklsma
(A) Tympanic membrane perforation
(B) Hearing lo&s pneumoniae and C. pneumonit.le, and
(C) Labyrinthitis
macrolides are the antibiotics of choice.
(D) Meningitis (E) If his chest x-ray is normal, then a diagnoais of
(E) Brain abscess waJkins pneumonia is less likdy.

(!] A 13-year-old boy is brought to the emergency [!) A 22-~-old man presents with palpitations. He
reports no chest pain. shortness of breath, or lower
department (ED) byhismother with a3-weekhisto ry
extremity edema. He states that he "got really drunk"
of a nonproductive cough that "won'tgo away." She
asks you if he might have 'Walking pneumonia."
the night before. He denies any past medical hi.rtory,
family history, or illidt drug uae. His examination
Which of the following is true?
is unremarkable en:ept for irregular tachycardia. His
(A) Walking pneumonia may frequently be caUBed electrocardiogram (EKG) is shown in Figure 3-1.
by Chlamydia pneumoniae, a sexually Which of the following is the most likdy etiologr.
tranmritted organism.
(A) Myocardial infitrction (MI)
(B) This is wually cawed by Epstein-Barr virus,
. that. causes " mono,, and it may
the same 'Vll"W (B) Pulmonary embolism
{C) Alcohol use
tab several weeks for him to improve.
(D) Hypertension
(C) The patient should be admitted to the ICU for
(E) Diabetes
IV antibiotics as walking pneumonia

Figure 3-1. (See color insert.)


62 1000 Questions to Help You PMS the Emerfmq Metlicins &Iris

(!) Which of the fOllowing is the most effective therapy (I) Which of the fOllowing is true regarding temporo-
fOr acute arsenic poisoning? mandibular joint (TMJ) dislocations?
(A) Activated charcoal (A) Most of them are posterior.
(B) Ipecac (B) The patient is unable to open the mouth in
(C) Dimercaprol bilateral dislocations.
(D) Penicillamine (C) The jaw is rotated toward the aifected side in a
(E) Deferoxamine unilateral dislocation.
(D) The patient will present with a protruding
(!) A 23-year-old man presents to the ED after being mouth.
kicked in the head. with lou of consciousness. He (E) All of the above.
is awake and alert and complains of a headache.
Brain computed tomography (CT) is performed and (!) WhkhofthefOllowingistrueregardingnecktrauma?
a slice is shown. Right after the cr scan, he becomes (A) Delayed neurologic deficits after blunt neck
umesponsive. Which of the fOllowing is the most trauma suggest carotid artery clissection.
appropriate next step in management (see Fig. 3-2)1 (B) All patients with suspected esophageal injury
should receive barium contrast esophagram.
(C) Zone Til injuries are most amenable to surgical
exploration.
(D) All neck wounds should be probed to
determine the depth of the wound and
integrity of vital structures.
(E) Impaled objects should always be removed in
patients with penetrating neck trauma.

(!] You are working in the ED when a 74-year-old


woman with chronic obstructive pulmonary dis-
ease (COPD) presents with an acute COPD exac-
erbation. You start her on bilevel positive airway
pressure (BiPAP) at a rate of 10, an inspiratory
positive airway pressure (IPAP) of 10 mm. Hg and
an expiratory positive airway pressure (EPAP) of
4 mm Hg. Twenty minutes later, the patient's oxy-
genation has not improved_ Which of the following
changes would most likdy increase this patient's
oxygenationf
(A) Inaease the patient's IPAP from 10 to 15.
(B) Inaeasethepatient'sratefrom tOto 12.
(C) Decrease the patient's EPAP from 4 to 2.
Figure 3-2. (D) Increasethepatient•sEPAPfrom4to 7, and
the IPAP from lOto 15.
(E) Decrease thepatient'sEPAP from 4 to 2, and
(A) Burr hole placement decrease the IPAP from 10 to 5.
(B) Endotracheal intubation
(C) Diagnostic peritoneal lavage (DPL) ~ A 75-year-old man presents with a fall on out-
(D) Emergent thoracotomy stretched hand. Radiographs demonstrate a distal
(E) Nasogastric tube placement radius fracture with dorsal displacement of the distal
segment. Which of the fOllowing is the most likely
(!) Which of the followins is true regarding duster nerve inju.l)i
headaches?
(A) Median
(A) The pain is typically bilateral. (B) Radial
(B) They occur more commonly in men. (C) U1nar
(C) The average age of onset is 45 years. (D) Axillary
(D) The average cluster period laats for 1 week. (E) Vagus
(E) Pain typically occurs in the V2 distribution of
the trigeminal nerve.
Test3 63

(!j) A 22-year-old man presents with left-sided facial and ~ A 20-year-old previously healthy female college
periorbital pain after a fisht. A cr scan reveals the student presents to the ED with diarrhea. She went
following image shown in Figure 3-3. Which of the on a camping trip 2 'M!eks ago but denies any
following examination findings may be present? other travel. She notes seven to eight watery, foul-
smelling stools per day and generalized abdominal
c::ramping. A test for fecal leukocytes, ordered in
triage after the patient had a large diarrheal stool,
is negative. Which of the following is the best
management?
(A) Azithromycin 500 mg daily for 3 days
(B) Metronidazole 250 m.g t.i.d. for 7 days
(C) Ciprofioncin 500 m.g b.id. for 3 days
(D) Supportift care with IV hydration and
antimotility agents
(E) Vancomycin 125 m.g q.i.d. for 10 days

~ A 65-year-old woman with a history of chronic


alcoholism presents with confusion, ataxia, and
nystagmus. Which of the following is the most
appropriate therapy?
(A) Lorazepam
Figure 3-3. (B) Haloperidol
(C) Thiamine
(D) Pyridoxine
(A) Hypoesthesia over the maxilla (E) Potassium
(B) Inability of upward gaze
(C) Enophthahnos (!!} Which ofthe following is the most common cause of
(D) Periorbital emphysema viral otitis media?
(E) All of the above
(A) Respiratory syncytial virus (RSV)
(B) In1luen2a
(!j} Which of the following is true regarding Rocky
Mountain spotted fever (RMSF)? (C) Rhinovirus
(D) Adenovirus
(A) It ia most commonly transmitted by ticks living (E) Human immunode1iciencyvirus (HIV)
in the Rocky Mountain range of North
America. 1!1) A 7-year-old girl is brought in by her father after
(B) Owing to treatment toxicity, antibiotics should choking on a plastic toy. She was coughing violently
not be started until a positive Wcil-Pelix and gasping in the car, so the father tried the
reaction is obtained. Heimlich maneuver and a blind finger sweep but
(C) The peripheral white blood cell (WBC) count is she seemed to get worse. His daughter is now
usually normal unconscious and cyanotic. After performing a jaw-
(D) Despite antibiotic therapy, the case fatality rate
thrust maneuver, you fail to locate a foreign body.
is 50%. Attempts to place an endotracheal tube fail, as the
(E) Oose contacts of the patient should be treated tube seems to be striking an object. What is the
with a 1-'M!ek course of prophylactic best nat step~
doxycycline.
(A) Laryngeal mask airway
~ Which ofthe following is true regarding patients with (B) Surgical cricothyroidotomy
temporal (or giant cell) arteritis? (C) Baek blows to dislodge the foreign body
(D) Blind finger sweeps to remove the foreign body
(A) The most specific finding is jaw claudication. (E) Needle cricothyroidotomy
(B) Permanent visual loss occurs in 50% of
patients. (!i) Which ofthe following is true regarding patients with
(C) Treatment with corticosteroids should be a primary spontaneous pneumothorax?
withheld until there is biopsy-proved disease.
(D) Vertigo ia the most sensitive clinical finding. (A) Pleuritic chest pain and dyspnea are the most
(E) The peak age of onset ia 40 years. common symptoms.
64 1000 Qvaticms mHdp You Ptw the Emerfmcy Metlit:W BoMds

(B) Hemoptysis is present in most patients. ~ Which of the following ia the most important factor
(C) It occurs most frequently in women aged 20 to in determining the risk of rupture of an abdominal
40yean. aortic aneur}"m (AAA)?
(D) Without treatment, syrnptoma tend to intensify
(A) Age of the patient
within 24 to 48 hoUI"'.
(B) Hyperte.osion
(E) Atrial fibrillation is the most common cardiac
(C) Size of the aneurysm
rhythm in the acute setting of a spontaneous
(D) Location of the an.euryam
pneumothorax.
(E) Male gender

(ji) A 4-month-old male infant pre&entll with mild


~ A 26-year-old woman presents with a rash on her
abdominal distension, pencil-thin stools, failure
legs (see Fig. 3-4). She states that she has been feelins
to thrive, and bilicnu vomiting. Digital rectal
somewhat tired lately and notes generalized body
eumination reveals an empty rectal vault Which and joint aches, u well as a sore throat_ Over the last
of the following ia the most likely cause of these
day or so, she has noted a rash developing over her
symptoms?
anterior shins. The rash is very tender to the touch
(A) Intussusception and is non-pruritic. Which of the following is true?
(B) Pyloric stenosis
(C) Gastroesophageal re:8ux cllieaae (GERD)
(D) Hirschsprung's d.isea.se
(B) Incarcerated hernia

~ A 65-year-old man presents with sudden, painful


loss of vision in his right eye. His visual acuity
ia markedly decreued in the ~cted eye. Which
of the following ia the most likely cause of his
symptomsf
(A) Acute angle closure glaucoma
(B) Central retinal artery occlusion
(C) Central retinal vein occlusion
(D) Retinal detachment
(B) Vitreous hemorrhage

(ij) Which of the following antidysrhythmics does


phenytoin most closely resemble?
(A) Procainamide
(B) Amiodarone
(C) Metoprolol
(D) Lidocaine
(B) Verapamll
Figura 3-4.
(ii} A 63-year-old man with a history of multiple
myeloma presents with acute onset of generalized
wealcness, blurry vision, diffuse mucosal bleeding, (A) She should be refm'ed to a dermatologist for
and headache. A complete blood count and compre- a biopsy.
hensive chemimy panel ia sent, but the laboratory is (B) First-line treatment is with aspirin or
unable to perform the analysis, citing "inappropriate nonsteroidal anti-inflammatory drugs NSAIDs.
blood sample." Which of the following is the most (C) These lesions do not occur in children.
appropriate definitive management? (D) The ruh tends to be nontender and pruritic.
(A) H~is (E) These lesions tend to be recurrent over a
(B) Plasmapheresis patient'a lifetime.
(C) Platelet transfusion
(D) Packed red blood cell (RBC) tranafuaion ~ A 26-year-ol.d man with a history of bipolar disorder
(B) Erythropoietin is brought in by hia mother with a chiefcomplaint of
Test3 65

agitation. She states that her son has been very anx- (C) Helicobacter pylori is the major risk factor
ious and restless, has not been sleeping, and abruptly linked to the development of duodenal ulcers
stopped takins his lithium medication 2 days ago. but has almost no role in the development of
In the ED, his vital sips include a temperature of gastric ulcers.
104.1°F, pulse of 120, with sinus tachycardia visible (D) Barium contrast is the diagnostic study of
on the monitor, and a blood pressure (BP) of 140/70. choice to diagnose peptic ulcer disease.
On examination he is mildly agitated, warm and di- (E) Only 50% of those people infected with
aphoretic, tremulous, and has a slight lid lag. Which H. pylori will develop a peptic ulcer in their
of the following is the next best step? lifetime.
(A) Oral lithium adminiatration
(B) Intravenous propanolol administration (ii) Which of the following is true regarding orbital wall
(C) Oral administration oflugol's solution fractures?
(D) Oral aspirin administration (A) The orbital floor is the \ftakest: part of the orbit.
(E) Intravenous dantrolene administration (B) Globe injuries occur in almost all orbital floor
fractures.
(!!] Parents of an 11-month-old male infant bring him (C) The teardrop sign refers to mt extending from
to your ED with a rash (see Fig. 3-5). They state the globe into the optic nerve.
that he appeared to have some discharge from his (D) Anb'biotic:s are indicated in all orbital wall
eyes and then developed a diffuse, tender "redness" fractures.
to his skin, which had a "rough" feel to it. Their (E) Patients with orbital wall fractures should be
pediatrician diagnosed him with a viral syndrome instructed to blow their noses every 6 hours to
and prescribed oral antipyretics as needed. Since reduce nasal congestion.
then, his skin appears to have wrinkled, formed
blisters, and is now peeling in large sheets. Which of li!) A 22-year-old man presents with a painful, red
the following is the treatment ofchoice? area on his right leg for the last 3 days. It is
spreading up his leg and he noted fevers, chills,
and lightheadedness today. He also describes a
sunburn-like rash on his chest, back, and arms but
• denies recent sun exposure. Vital signs are 102.5°F,
112, 22, 82/45, 94% RA Which of the following is
true regarding this patient?
(A) Pseudomonas is the most likely cause.
(B) A bacterial toxin is responsible for the
pathologic effects.
(C) Blood cultures are almost always positive.
(D) Mortality is <1%.
(E) Intravenous fluids usually worsen the clinical
outcome.

[jg} Which of the following is characteristic of rotavirus


Figure 3-5.
diarrhea?
(A) Fecalleukocytes
(A) Amoodcillin (B) Fecal erythrocytes
(B) Nafcillin (C) Summer predominance
(C) Valacydovir (D) Duration of 1 month
(D) Corticosteroids (E) Peak age between 6 months and 2 years
(E) Continue with supportive care only
!!!) A mother brings in her 7-year-old daughter to the ED
~ Which of the following is true about peptic ulcer with a chief complaint of paiDful vaginal. lesions and
disease? '"bumps," burning dysuria, and generalized malaise
(A) Pain that wakes patients in the middle of night (see Fig. 3-6). On examination, you note tender
is typical of duodenal ulcers. inguinal lymphadenopathy and the lesions shown.
(B) The inclclence of bleeding from gastric ulcers is Which of the following is the most likely cause of her
approximately two times that ofduodenal symptoms?
ulcers.
66 1000 Quatimu to Hdp Yow Ptm tlte ~ Metlici1re Botris

(C) The initial ventilator settings are no different


than for a patient intubated fur airway
protection due to altered mental status.
(D) Inspiratory flow rates are set very low to avoid
causing very high peak airway pressures due to
bronchoconstriction.
(E) The inspiratory flow curve should be a
ramp-style wave instead of a square-style wave
to maximize expiratory time.

~ Which of the following ia the most commonly


identified cause of bacteremia in children?
(A) Neisserill meningitidis
Figure 3-t. (B) Hemophilus inftuenzae
(C) Streptococcus pneumonilu
(D) 8mphylococcus aureus
(A) Herpes simplex virus type 2 (HSV-2) (E) Pseudomoruu aerugino.sa
(B) Syphilis (Treponema JH11lidum)
(C) Chancroid (Hmwphilus ducreyi) ~ Which ofthe following is true regarding avulsed and
(D) Lymphogranuloma venereum {LGV, mbluxed teeth?
Chlamydia trachomatis) (A) Avulaed teeth can almost always be successfully
(E) Granuloma inguinale (Calymmatobacterium reimplanted ifreturned to their sockets within
granulcmuztU) 3hours.
(B) Avulsed primary teeth are never reimplanted.
~ Aaide from the eyes, which of the following is the (C) The best known transport medium fur avulsed
moo common site of pathologic effi:ct of methanol teeth is milk.
overdosel (D) Teeth can be temporarily secured for up to 1
(A) Basal ganglia week with a periodontal pad made from resin
(B) Facial nerve and catalyst paste.
(C) Vagus nerve (E) Avulsed teeth should be scrubbed with a
(D) Olfactory nerve povidone-iodine sponge to kill microbes
(E) Glossopharyngeal nerve before reimplantation.

~ Which of the followins is true regarding idiopathic ~ Which of the following is true regarding diaphrag-
thrombocytopenic purpura (ITP)? matic injuries?
(A) Five-year mortality is 50%. (A) Left-sided injuries are three times as common
(B) Aspirin is the treabuent of choice in adults. u right-sided.
(C) Treatment in children is usually supportive. (B) The majority heal spontaneously.
(D) There is a male predominance in adults. (C) Delayed herniation of abdominal contents into
(E) Platelet therapy is indicated in patients with the thorax is extremely rare.
<50,000 cells per mm3 • (D) Almost all cases are caused by penetrating
trauma to the abdomen.
~ The most widely used critical care ventilatory strategy (E) Ultrasonography is the diagnostic modality of
in acute uthmat:ic patients aims to accomplish which choice.
of the fullowins objectives?
(A) Patients are purposefully hypoventilated, ~ Diagnosis of which of the followirJ8 findings is
maintaining elevated Pac~ values, to keep the primary utility of the focused asseument of
their airway pressures at sab: levels to avoid sonographyin trauma (FAST) scan?
barotrauma. (A) Pelvic fracture
(B) Patients are purposefully hyperventilated to (B) Renal injury
bring their Pac~ levels back to normal because (C) Diaphragmatic rupture
ventilatory failure is the primary reason fur (D) Hemoperitoneum
intubating patients in status asthmaticus. (E) Aortic injury
Test3 67

~ A 27-year-old woman without past medical history of dizziness, nausea, and severe "stomach cramps."
presents with 2 days of pain in her right ear. Which of the following is the likely culprit?
The patient notes that the symptoms started with (A) Tarantula
an itchy ear which progressed to pain, discharge, (B) Hobo spider
and hearing loss. Examination reveals an afebrile (C) Brown recluse spider
patient who is nontoxic, moderate tenderness on (D) Wolf spider
manipulation of the auricle, erythema and edema of (E) Black widow spider
the tympanic canal, and no external rash. Cranial
nerve examination is normal. Which of the following ~ Which ofthe following is the most common cause of
will be most helpful in treating this condition? gastritis?
(A) Antihistamines (A) Alcohol
(B) Tympanostomytubes (B) Caffeine
(C) Systemic antivirals (C) H. pylori
(D) Adenoidectomy (D) Aspirin or NSAID use
(E) Acetic acid otic washes (E) Crohn's disease

~ The most common sexually transmitted organism in ~ The most common cause of hypomagnesemia in the
the United States is ED is likely:
(A) T. pallidum. (A) Alcoholism
(B) C. trachomatis. (B) Diuretic therapy
(C) Neisseria gonorrhoeae. (C) Acute tubular necrosis
(D) Herpes simplex virus. (D) Chronic diarrhea
(E) H. ducreyi. (E) Diabetic ketoacidosis

~ Which of the following is true regarding scorpion ~ Which of the following is the most common risk
envenomation? fuctor associated with aortic dissection?
(A) Scorpion envenomation is typically more (A) Smoking
severe in adults than in children. (B) Atherosclerosis
(B) Narcotic analgesics should be the first-line (C) Marfan's syndrome
agents for pain control. (D) Hypertension
(C) Envenomation by United States scorpions (E) Bicuspid aortic valve
generally results in significant
cardiopulmonary collapse. [!!) Which of the following is one of the Kanavel's
(D) Extreme hypertension may be managed with cardinal signs of flexor tenosynovitis?
nitroprusside. (A) Symmetrical swelling of the digit
(E) All of the above. (B) Tenderness to palpation of the volar aspect of
the digit
~ Among the elderly, which ofthe following is the most (C) Pain upon passive extension of the digit
common cause of deliriwn? (D) Maintenance of the digit in a flexed posture
(A) Stroke (E) All of the above
(B) Electrolyte abnormalities
(C) Infection ~ Which of the following is true regarding sick sinus
(D) Medication interactions or side effects syndrome?
(E) Trauma (A) Long-term treatment generally involves
pacemaker and antidysrhythmics.
~ An otherwise healthy 22-year-old man presents to (B) The condition is more common in patients
the ED after a "spider bite." He states that he felt younger than 40 years.
a pinprick sensation on his hand when he lifted the (C) Lidocaine is the mainstay of management of
hood ofan old car he has been storing in his driveway. acute tachycardias.
He continued working, but soon developed crampy (D) Atropine is ineffective at improving
muscle aches that spread up his arm and now seem bradycardia.
most severe in his chest and abdomen. In the ED, (E) Atrial fibrillation is almost never present.
his abdomen is rigid and the patient is complaining
~ A 78-year-old man presents with marked left foot the left flank lesion. Tbe remainder of his physical
and mild left arm weakness and hypoesthesia. In examination is normal Which ofthe following iJ the
addition, his 6unily states that he is not acting himself most likely cause (see Fi3- 3-7A and B)?
and seems to be having difficulty making decisions. (A) Henoch-Schonlein purpura
Which ofthe following arteries is most likelyaffectecU (B) Idiopathic thrombocytopenic purpura (ITP)
(A) Vertebrobasflar artery {C) Lyme disease
(B) Posterior oerebral artery {D) Child abuse
(C) Mid.dle urebral artery {E) Roseola
(D) Anterior cerebral artery
(E) None of the above ~ A 38-year-old woman presents to the ED com-
plaining of throat irritation and a 3-week. history
~ Which of the followiJJs is true regarding nitroprus- of episodic spastic cough. The cough is worse at
5ide?• night and is occasionally so forceful that she vomits
after she coughs. She is accompanied by her teenage
(A) Cyanide toxicity iJ common.
son who has had a similar illness for 6 weeb without
(B) Extravuati.on causes severe local skin neaosis.
(C) It decreases intrac.ranial pressure (ICP).
abating. She is a nonsmo~r. What is the most likely
cause of her illne.ssf
(D) It is safe for we during pregnancy.
(E) It has a delayed onset of action compared with (A) Corynebactmum diphtheritu
other IV antihypertenaive agents. (B) Legionella pneumophila
(C) Bcm:letella pertussis
~ A 5-year-old boy is brought by his mom for a "raah, {D) M. pneunumw
on his back. His back is shown with close-up of (E) S. Pneumonitu

A B
Figur. 3-7.
Tcst3 69

~ After postoperative adhesions, which ofthe following 98.6, 60, 22, 97/(IJ, 92% on RA. Physical examination
ia the most common cause of this x-ray finding (see demonstrates an S, and bilateral crackles to the
Fig. 3-8)? midlung fields. The EKG is shown in Figure 3-9.
Which of the followins ia the most appropriate
definitive management?
(A) Admit-immediate percutaneous transluminal
coronary angioplasty.
(B) Admit-immediate fibrinolytic therapy.
(C) Admit-immediate upper endoscopy.
(D) Admit-immediate heparin, G2b3a inhlbitor,
and cardiac care unit (CCU) admission.
(E) Discharge home for outpatient stress test.

~ Which of the following ia true regarding pemphigw


vulgaris?
(A) It is most common in children.
(B) NikoJskYs sign is infrequently present.
(C) Antibiotics are the most important part of
management.
(D) Oral bliaten that spontaneously rupture to
form painful erosions are the earliest
Figure 3-L
m.anifatation.
(E) Outbrealu are commonly triggered by
(A) Incarcerated hernias sulfonamide& or tJ-lactam drugs.
(B) Intussusception
(C) NeoplUill8 ~ A term kma.le neonate develops purulent ocular
(D) Volvulus discharge on day 3 of life. Which of the following
(E) GalJatone ileus
etiologies carries the greatest morbidity in this
patient?
~ A 54-year-old man presents with chest pain of (A) C. tNchoffUitiJ
4 hours' duration. He also complains of diaphoresis, (B) N. grmorrhoeu
shortness of breath, and orthopnea. Vital signs are (C) S. aumu

~~/L-{L-,/vs'-----..JV
\, ~~~~vo'--..lv
n

\~

Figure 3-t.
70 1000 Qvaticms ftl Hdp You Ptw the Emerfmo/ Metlit:W BoMds

(D) No organiam--illlergic conjunctivitis ~ Which of the following medications is safe to use on


(E) No organism-chemical conjunctivitis facial rashes?
(A) Triamcinolone
!!!) A 44-year-old alcoholic man presents with he-
(B) Fluocinolone
matcmesis. Which of the following is the most likely (C) Betametlwone
causel (D) Hyd.rocort:isone
(A) Peptic ulcer disease (E) AU of the above
(B) Gastric varices
(C) Eaophageal varicea ~ Which of the following ia true regarding the role of
(D) Boerhaave's syndrome ipratropium in asthma management?
(E) Arteriovenoua malformation
(A) The main benefit of ipratropium, instead of
atropine or other anticholinergic drugs, is
~ A 65-year-old man presents with a syncopal event
ipratropium has a more rapid onset ofaction.
without prodromal symptoms. His vital signs are
(B) Ipratropium is useful as a sole bronchodilator
98.6, 60, 18, 142/75, 99% RA. The BKG is shown
in the treatment of acute asthma exacerbations.
in Figure 3-10. Which of the following is the most (C) Ipratropium is most useful as an adjunct for
appropriate next step in management?
patients with severe asthma exacerbations.
(A) Discharge home with cardiology foUow-up (D) Ipratropium bas never been proved to be of
(B) Cardiovenion at 50 J benefit in patients with acute asthma
(C) Defibrillation at 200 J exacerbations.
(D) Amiodarone 150 mg IV (E) None of the above.
(E) Admission and pacemaker placement

Figure 3-10.
Test3 71

~ The most common cause of distal symmetric ~ A 57-year-old woman is found to have bilateral
polyneuropathy is popliteal artery aneurysms. Which of the following
(A) Amyotrophic lateral sclerosis. conditions is she at most risk of developing?
(B) Guillain-Barre syndrome. (A) Alzheimer's disease
(C) Alcoholism. (B) Cystic fibrosis
(D) Diabetes mellitus. (C) Hepatic failure
(E) Paraneoplastic syndrome. (D) Endocarditis
(E) Abdominal aortic aneurysm (AAA)
~ Which of the following is true of vitreous hemor-
rhage? [!Z) A 25-year-old man with schizophrenia presents with
(A) Symptoms progress from floaters to visual loss. acute agitation. According to a family member who
(B) The red reflex is enhanced. is present, he was seen by a psychiatrist during the
(C) Sudden onset of unilateral pain is typical. previous week and diagnosed with schizophrenia.
(D) Valsalva can be helpful in management. He is extremely agitated, tachycardic, diaphoretic,
(E) Treatment involves recumbent position. febrile, and exhibits muscle rigidity. Which of the
following is the most appropriate next step in
~ Which of the following is the serum half-life of management?
adenosine? (A) Acetaminophen
(A) 1 second (B) Lorazepam
(B) 10 seconds (C) Amantadine
(C) 1 minute (D) Bromocriptine
(D) 5 minutes (E) Haloperidol
(E) IOminutes
~ Which of the following is a risk factor for completed
~ A 44-year-old man presents with painful, swollen suicide?
gums that bleed easily, malaise, and intermittent (A) Women younger than 50 years
fevers over the last week. He smokes, takes no (B) Antisocial personality disorder
medicines, and denies weight loss, night sweats, or (C) Family history of suicide
family history of hematologic malignancy. Examina- (D) Generalized anxiety disorder
tion reveals poor dental hygiene, inflamed gingival (E) Arachnophobia
tissue, and mild regional lymphadenopathy. Which
of the following is true regarding this patient? ~ A 3-year-old girl presents with right elbow pain after
(A) Analgesics alone are sufficient for therapy. being pulled by the hand by her mother. There is no
(B) Spirochetes and fusobacteria are commonly history of trauma. She has minimal tenderness of the
present in gingival creases. elbow, but refuses to use her right upper extremity.
(C) Follow-up with a dentist is generally not Which of the following is the most appropriate next
necessary. step in management?
(D) Vincent's angina, a common complication, (A) Elbow x-rays
involves extension to the buccal mucosa. (B) Wrist x-rays
(E) The disease is communicable by direct contact (C) Flexion and hyperpronation of the forearm
with secretions. (D) Extension and hyperpronation of the forearm
(E) Extension and supination of the forearm
~ Which of the following is true regarding perforated
peptic ulcers? [!g) A 55-year-old man presents with a ..bump" on his
(A) Cocaine use is a risk factor for perforation. upper eyelid. The eyelid is shown in Figure 3-11.
(B) Patients may develop paradoxical Which of the following is the most appropriate next
improvement of their pain without treatment. step in management?
(C) Evidence of intraperitoneal free air on an (A) Intravenous antibiotics
upright abdominal film or chest x-rayis only (B) Eyelid culture
present 70% of the time. (C) Warm compresses
(D) Regardless of patient symptoms, a rigid (D) Needle aspiration of the raised area
abdominal wall on physical examination is a (E) Topical antivirals
nearly constant finding.
(E) All of the above.
72 1000 Quatimu to Hdp Yow Ptm tlte ~ Metlici1re Botris

(A) Nitroprusaide
(B) Nitroglycerin
(C) Clon.idine
(D) Nicardipine
(E) Hydrochlorothiazide

~ Which of the following is true reguding reciprocal


ST depressions seen in acute .MI?
(A) They have little prognostic significance.
(B) They are more commonly seen in inferior Mis
than anterior Mls.
(C) They have poor specificity for acute MI.
(D) They are more commonly seen in acute
Plgu,. 3- u. (See color insert.) pericarditis than acute MI.
(E) The are usually upsloping when seen in the
setting of acute MI.

[!j) A 28-yea.r-old pregnant woman at 15 weeks presents (lj) Which of the following is true regarding hyper-
to the ED in July with generalized mala.Ue along with osmolar hyperglycemic syndrome or state (HHS),
a high fever, headache, myalgias, particularly in her also known as hyperglycemic hyperosmolm nonketotic
calves, and a rash. She returned from visiting relatives COmG (HHNC)?
who live in North Carolina apprmimately 4 days ago. (A) The presence of ketonuria rules out the
She had no difficnlries on her trip but she became diagnosis.
acutely ill on the morning of prac:ntation. The rash (B) The mortality rate is lower than in patients
is petechial. currently somewhat difrwe and includes with diabetic ketoacidosis (DKA).
her pahns, but she states that it started on her wrists (C) Infection is the most common precipitant.
and ankles. Which of the followins is the next best (D) The degree of mental status change correlates
step in management? with the plasma pH.
(A) Doxycycline 100 mg IV (E) Insulin infusion& are frequently necessary to
(B) Ciprofloxacin 500 mg IV regain eug1ycemia.
(C) Chloramphenicol (CAM) 500 mg IV
(D) Penicillin V 500 mg IV (llJ A patient with a history of depreuion on parox-
(E) Gentamicin 1.5 mg per kg IV etine presents with acute onset of altered mental
status, muscle rigidity, tremor, hyperrefl.aia. and
~ A 55-year-old man presents with an anterior shoulder hyperthermia. Which of the following additional
dislocatio11- Skin overlying which of the following medications is the patient most l.ikcly usint.
areas is most l.ikcly to have diminished sensation? (A) Dextromethorphan
(A) Base of the neck (B) Pseudoephedrine
(B) Sternum (C) Diphenhydramine
(C) Deltoid (D) Acetaminophen
(D) Triceps (E) Valproate
(E) Dorsolateral foot
~ Clinically, condyloma acuminatum may be differen-
~ A 20-yea.r-old man who has recently recovered from tiated from condyloma lata by
gastroenteritis due to ShigelliJ ftexneri is most at risk (A) Extensive wart-like lesions.
for developing which of the following? (B) Location in the perineum.
(A) Ankylosing spondylitis (C) Drier, keratinized appearance.
(B) Systemic lupus erythematosus (D) Foulodor.
(C) Rheumatoid arthritis (E) Bluish superficial telangiectasias.
(D) Psoriatic arthritis
(E) Reiter's syndrome ~ Acalculous cholecystitis •..
(A) May occur in patients with acquired immune
(MJ Which ofthe following is the most appropriate initial deficiency syndrome (AIDS) and
antihypertensive medication for hypertensive crises Cryptosporidium infection.
in pregnant patients?
Test3 73

(B) Represents halfofall cases ofacute (C) Rosette test


cholecysti.tis. (D) Apttest
(C) Is most common in patients with hemolytic (E) All of the above
anemia.
(D) Responds to oral ursodeoxycholic: acid therapy. ~ A 22-year-old man presents with severe left elbow
(E) Tends to have a more benign course than pain and swelling after being tackled in a football
calculous cholecystitis. game (see Fig. 3-12). His elbow appears to be
locked in partial flexion and he guards against any
{ig} The Kleihauer-Betke test (KBT): motion. X-rays reveal the image shown. Which of
(A) Should be performed in all pregnant patients in the following is the most common complication of
cues of trauma. this injury?
(B) Detennines whether pregnant patients should
receive Rh immune globulin (RhoGAM).
(C) Detects as little as 0.5 mL of fetal blood in the
maternal circ:u1ation.
(D) Is most useful in c:ases of significant
maternal-fetal hemonhage (MFH).
(E) All of the above.

{ij} Which of the following patients should be placed in


respiratory isolation?
(A) A 72-year-old female nursing home resident
being treated for a COPD exacerbation.
(B) A 46-~ar-old man with community acquired
pneumonia.
Figure 3-12.
(C) A 23-year-old man who thinks he was exposed
to anthrax.
(D) A 3-month-old female ex-premature in&.nt
being admitted with bronchiolitis. (A) Median nerve injury
(E) A 28-year-old asymptomatic medical student (B) Uhw nerve injury
who recently had a positive pressure (C) Brachial artery disruption
differential (PPD) test. (D) Compartment syndrome
(E) Radial nerve injury
(jiJ A 67-year-old female diabetic patient. hypertension,
and a history of an aortoiliac: bypass graft presents (iiJ Which of the following is an expected finding in
with abdominal pain and dark. ''funny-smelling.. pregnancyf
stools. She states the symptoms started 2 days ago and (A) Increased tidal volume
seem to have gone away as she had a "near normal" (B) Decreased RBC mass
bowel movement this morning and no longer has (C) Decreased WBC count
pain. On examination, she has guaiac positive, dark (D) Increased respiratory rate
brown, but notmelenicstool. Which of the following (E) Decreased glomerular filtration rate
is the presumed diagnosis until proved otherwise?
(A) Acute mesenteric .ischemia l!!} Which of the following is true regarding placenta
(B) Abdominal aortic aneurysm (AAA) previa~
(C) Aortoenteric fistu1a (A) Most women are diagnosed after an episode of
(D) Duodenal ulcer second or third trimester vaginal bleeding.
(E) Ischemic colitis (B) Digital vaginal examination is recommended
before ultrasonography for more rapid
~ A 3-day-old female neonate is brought to the ED diagnosis and obstetrician (OB) consultation.
with a chief complaint of rectal bleeding. A guaiac (C) With progression of pregnancy, 90% of
test of the stool is positive. Which of the following low-lying placentas will migrate toward the
tests can help to determine ifthe blood in the stool is cervical OS.
of fetal origin? (D) Disseminated intravascular coagu1ation is a
(A) Kleihauer-Betke test (KBT) common complication.
(B) Meckel's scan (E) None of the above.
74 1(}()() Questions to Help You Pass the Emergency Medicine Boards

~ The best means of rewarming a frozen or partially (A) Mannitol


thawed extremity is (B) Saline
(C) Bicarbonate
(A) Direct tissue massage.
(D) Furosemide
(B) Direct exposure to dry heat sources such as
(E) Deferoxamine
heat lamps or open fires.
(C) Immersion in a water bath maintained between
40°C and 42°C.
[!!) A 45-year-old woman is referred to the ED from an
optometrist with a diagnosis ofbilateral papilledema
(D) Slow thaw through immersion in an ice water
and "rule out pseudotumor cerebri." Which of the
bath.
following is true about pseudotumor cerebri (also
(E) Forced-air warming device (convection
known as idiopathic intTacranial hypertension)?
method, e.g., Bair Hugger).
(A) Blurry vision is the most common presenting
[!!] A 25-year-old man ingests an unknown quantity complaint.
of iron tablets 2 hours before presentation in an (B) CT scanning will demonstrate hydrocephalus
overdose attempt. Which of the following is the most in 80% of cases.
appropriate next step in management? (C) Oculomotor nerve palsy is the most common
(A) Activated charcoal associated cranial nerve palsy.
(B) Gastric lavage (D) Men outnumber women 4:1.
(C) Ipecac (E) Headaches associated with the disorder are
(D) Polyethylene glycol usually worse in the recumbent position.
(E) Hemodialysis
~ Which of the following is the treatment of choice for
~ A 6-year-old boy is brought by a distraught mother acute cluster headache?
who thinks her son had a stroke. She notes that (A) 100% oxygen
he had a fever and was complaining of right ear (B) Sumatriptan
pain 1 day before. The fever was controlled with (C) Morphine
acetaminophen and the patient's pain improved, but (D) Lorazepam
recurred the day of presentation. The patient denies (E) Dexamethasone
any headache or neck stiffness. On examination, he
is febrile to 101 °F, well appearing, has a bulging right ~ Which of the following is best able to diagnose
tympanic membrane (TM) with poor mobility, and penetrating esophageal injuries?
the entire right side of his face is paralyzed. There (A) Bronchoscopy
is no rash present. What is the most appropriate (B) Esophagoscopy
management plan? (C) Esophagograrn
(A) Oral antibiotics and follow-up with PCP (D) Esophagoscopy and esophagograrn
(B) N antibiotics and admission to hospital for (E) CT chest
myringotomy
(C) N steroids and acyclovir ~ A 10-year-old boy presents with elbow pain after
(D) CT brain and neurosurgery consultation falling off the bed. Radiographic visualization of
(E) Lumbar punctures, N steroids, and N which of the following usually indicates an occult
antibiotics elbow fracture?
(A) Anterior fat pad
~ A 35-year-old patient is sent to the emergency room (B) Posterior fat pad
(ER) for evaluation of a heart murmur. The murmur (C) Baumann's angle of75 degrees
is diastolic, rumbling, and loudest at the apex. Which (D) Bilaterally equal Baumann's angles
of the following is the most common etiology of the (E) Anterior humeral line bisecting the capitellum
murmur?
(A) Pulmonary embolism [!!] Diverticular disease ...
(B) MI (A) Is more common in the right colon among
(C) Rheumatic heart disease Japanese-Americans.
(D) Atrial myxoma (B) Most commonly affects the sigmoid colon in
(E) Aortic dissection the Western world.
(C) Is treated with a high-fiber diet.
I!!] Which of the following may worsen acute renal
failure due to rhabdomyolysis?
Test3 75

(D) Is best diagnosed with a barium enema. (A) The recurrence rate of diverticulitis after a
(E) Is all of the above. single, uncomplicated episode is 75%.
(B) Diverticulitis is the most common cause of a
l!l] Which of the following symptoms, associated with colovesical fistula.
chest pain, has the highest predictive value for (C) All patients younger than 40 years with
coronary ischemia? diverticulitis should have resection of the
(A) Dyspnea disease segment of colon.
(B) Nausea (D) Barium-soluble contrast enema is the
(C) Back pain radiographic study of choice in the ED.
(D) Diaphoresis (E) The mortality rate of hospitalized patients with
(E) Fever acute diverticulitis is 35%.

~ A 61-year-old male smoker recently performed some [100] Which of the following statements is true regarding
repairs on several air conditioning units during the general management of poisoned patients?
late summer. He is now brought in by his family (A) Addressing circulatory insufficiency should
confused, with high fevers, chills, a dry cough, and precede airway management in most cases.
diarrhea. The test that will best determine the likely (B) Specific antidotes are indicated for most
specific cause of his illness is which of the following? poisoning.
(A) Urine antigen testing (C) Recognition of common toxidromes is helpful
(B) Blood cultures to direct therapy.
(C) Sputum cultures (D) Ipecac is now the mainstay of gastrointestinal
(D) Chest x-ray decontamination therapy.
(E) Serology testing (E) Standard hospital urine toxicologic screens
detect the most common toxins.
~ Which of the following is true regarding diverti-
culitis?
Answers and Explanations
[!] Answer B. All of the above choices are complica- rn Answer c. Acute arsenic poisoning affects multi-
tions of otitis media, but hearing loss is the most ple organs, including the liver, kidneys, lungs, and
common. The primary reason for antibiotic treat- heart. It replaces phosphate in high-energy adeno-
ment of otitis media is the prevention of these sine triphosphate (ATP) bonds and decreases energy
complications. Current treatment guidelines by the production. Management is supportive plus chela-
American Academy of Pediatrics involve both im- tion therapy. The first-line therapy for chelation
mediate antibiotic therapy as well as deferment of is dimercaprol (or British antilewisite [BALl). Ac-
therapy with watchful waiting for 48 hours to observe tivated charcoal does not absorb arsenic or heavy
for development of complications. metals. Ipecac is almost never indicated for any poi-
soning. Penicillamine is a less effective alternative
[I) Answer D. "Walking pneumonia" most fre- to dimercaprol, and is only used when the latter's
quently refers to "atypical" pneumonia, a term gastrointestinal side effects are prohibitive. Deferox-
which has been used to distinguish infections caused arnine is used for chelation of iron.
by M. pneumonif:le, C. pneumonif:le and Legronelkl,
from infections caused by "classic" bacteria such as [!) Answer B. TheCTscanshowsaright-sidedepidu-
S. pneumoniae. The term walking pneumonia came ral hematoma. While the patient is initially awake and
about because patients with this entity are usually well alert, this may be indicative of the lucid interval that
enough to be out of bed and able to perform their is often seen in patients with epidural hematoma
normal activities without difficulty. The course tends (although it is neither sensitive nor specific for the
to be mild although it may be prolonged. Several re- diagnosis). Acute worsening of the clinical status
cent studies have demonstrated that it is not possible mandates returning to the ABCs of trauma eval-
to differentiate between typical and atypical causes uation. Although burr hole placement is the most
of pneumonia based on symptoms or chest radiogra- important definitive management, control of the air-
phy. Furthermore, chest x-rays may appear normal way is the most important initial action. Diagnostic
in either typical or atypical cases. However, because peritoneal lavage (DPL) may be performed if the
Mycoplasma and Chlamydia are the most common patient is hypotensive and intraperitoneal source is
causes of community-acquired pneumonia in this expected, but in solitary head trauma, this is unlikely
patient's age-group (5 to 15 years), and because he to be the case. Thoracotomy is indicated in patients
lacks signs and symptoms suggesting a critical illness, with penetrating trauma to the chest who arrest in the
it is reasonable to treat him empirically for "atyp- ED or shortly beforehand. Nasogastric tube place-
ical" community-acquired pneumonia. Macrolides ment is contraindicated in patients with severe head
are the antibiotics of choice for Mycoplasma and and facial trauma as damage to the inferior portion
ChlamydUI. of the skull may allow the tube to pass from the
nose into the cranium. (Figure couresy of Robert
[!] Answer C. The EKG demonstrates atrial fibrilla- Hendrickson, MD. Reprinted from Hendrickson R
tion with rapid ventricular response. The most likely Greenberg's text-atlas of emergency medicine. Lippin-
cause in this young adult with no medical history ex- cott Williams & Wilkins; 2004:51, with permission.)
cept for alcohol abuse is "holiday heart syndrome,"
which can occur within 2 days of an alcohol binge. [!] Answer B. Of the primary headache syndromes,
It often resolves spontaneously, but may require cluster headaches have the most consistent presenta-
rate control therapy and possibly anticoagulation. In tion. The average age of onset is 28 to 30 years. They
the absence of cocaine use, MI would be extremely are strictly unilateral headaches that occur in the
unlikely in an otherwise healthy 22-year-old. Pul- ophthalmic division (Vl) of the trigeminal nerve.
monary embolism is an important cause of atrial Therefore, the pain is most commonly maximal
fibrillation, but is not likely in the absence of chest in the retro-orbital and temporal region, although
pain, dyspnea, or risk fu.ctors. The physical exami- patients may experience radiating pain to the fore-
nation is unremarkable, so hypertension is unlikely. head, temple, cheek, and jaw. The pain is severe
Diabetes does not confer an increased risk of atrial and is often described as ''boring" or "tearing" in
fibrillation. nature. Patients are often very restless or agitated

76
Test3 77

during cluster headaches and characteristically rock and visual changes. Esophageal injuries are relatively
or pace back and forth. The headaches last on aver- uncommon, frequently subtle, and often missed in
age from 45 to 90 minutes and occur one to three the setting of neck trauma. Because barium may
times daily. The typical cluster period (during which provoke an inflammatory mediastinitis, patients with
headaches occur) lasts 6 to 12 weeks with typical re- suspected esophageal injury should first receive an
missions lasting 12 months. Traditionally, they have examination with water-soluble contrast such as
been much more common in men, although recent gastrograffin. If the initial study is negative, a follow-
epidemiologic studies have revealed a declining pre- up examination with barium can be performed
dominance of men. In addition, patients affected because of its superior sensitivity for smaller defects.
by cluster headaches more frequently indulge in Zone II injuries are most amenable to surgical repair
cigarette smoking and alcohol abuse. Interestingly, due to relatively uncomplicated surgical exposure
once a cluster period begins, alcohol usually triggers and vascular control. Neck wounds should never
an attack within minutes. There is also a seasonal be probed outside the operating room. If careful
pattern to the clusters, with exacerbations occurring examination fails to determine platysma integrity, a
more often in the spring and fall. Interestingly, how- surgeon should be consulted for presumed violation
ever, attacks occur most commonly in the 2 weeks of the platysma. Impaled objects should always be
after the summer and winter solstices. left in place because they may tamponade vascular
injuries. They should only be removed in the
W Answer D. TMJ dislocations are anterior disloca- operating room under direct visualization.
tions that may result after trauma or after widely
opening the mouth as in yawning, laughing or [!) Answer D. Bilevel positive airway pressure
singing. The mandibular condyle slides anteriorly (BiPAP) ventilation emulates pressure support me-
during normal mouth opening. Occasionally the chanical ventilation with positive end-expiratory
mandibular condyle may slide anterior to the artic- pressure (PEEP) in which the machine cycles be-
ular eminence of the temporal bone resulting in an tween two different pressure levels during inspiration
anterior dislocation. Subsequent muscle spasm pre- and expiration. This is in contrast to continuous pos-
vents spontaneous reduction and the patient presents itive airway pressure (CPAP) in which the pressure is
to the ED with a fixed, protruding and widely open the same throughout inspiration and expiration (i.e.,
mouth. The patient may not be able to handle the se- IPAP = EPAP). When a patient's oxygenation status
cretions and may therefore be drooling. Posterior and is not improving, the two main adjustments that can
lateral dislocations can occur but they almost always be made are to increase the inspired Fio2 and to
occur in the setting oftrauma. Most spontaneous dis- increase the PEEP. With BiPAP, there is a limit to the
locations are bilateral. When unilateral dislocations amount of oxygen that can be applied because the
occur, the jaw is rotated toward the unaffected side. oxygen is not being delivered to the lower airways,
Reduction is achieved by applying downward pres- as is the case with mechanical ventilation. However,
sure on the most posterior molars followed by slight the EPAP (i.e., PEEP) can be increased in order
posterior pressure. Gauze should be thickly wrapped to recruit additional alveoli (by stenting them open
around the physician's thumbs to prevent injury with increased pressure at the end of expiration),
from the typical bite that occurs upon reduction. which increases the surface area for gas exchange and
hopefully results in improved oxygenation. In order
00 Answer A. Delayed presentation of neurologic to maintain the positive pressure differential (PPD)
deficits is characteristic of vascular injuries to the during inspiration, the IPAP may be concomitantly
neck due to blunt trauma. Only 10% of patients increased (to ensure ongoing adequate ventilation).
who ultimately develop neurologic deficits due to Ifthese measures fail, the patient requires intubation
vascular injury exhibit signs and symptoms of injury and mechanical ventilation.
within the first hour. Most patients experience
stroke symptoms such as hemiparesis, hemiplegia, ~ Answer A. The patient has a Calles's fracture,
or aphasia between 1 and 24 hours after injury which puts him at risk for median nerve injury. Injury
due to carotid artery dissection or thrombosis. to this nerve can result from either the fracture itself
Furthermore, a significant percentage of patients will or during therapeutic reduction. Fall on outstretched
not develop symptoms until after 24 hours has passed. hand is the most common mechanism for a Calles's
Vertebral artery injury can also occur, although it is fracture. A volarly displaced distal radius fracture is
less common. Because the vertebral arteries combine referred to as a reverse Calles's or Smith's fracture,
to form a single basilar artery, injuries to the vertebral which usually results from trauma to the dorsum of
artery do not produce lateralizing symptoms. Patients the hand. Smith's fracture also involves the median
may present with nausea, vomiting, central vertigo, nerve more often than other peripheral nerves of the
78 1(}()() Questions to Help You Pass the Emergency Medicine Boards

upper extremity. An intra-articular fracture of the poor sensitivity and specificity and has therefore
distal radius with displacement of the carpal bones is been abandoned. There is no known human to
referred to as a Barton's fracture. human spread so prophylactic treatment of contacts
is unnecessary.
[!!] Answer E. The CT reveals the classic "teardrop"
sign associated with an orbital floor fracture. Direct [!ID Answer A. Temporal arteritis (or giant cell arteri-
trauma to the orbit causes an immediate rise in tis) is a large vessel vasculitis that primarily affects
intraorbital pressure fracturing the weakest borders branches of the carotid artery. The disease is rare
of the orbit. This most commonly results fractures before the age of 50, and incidence peaks in the
of the orbital floor fracture and medial orbital wall. seventh decade. The most sensitive finding is a new
Owing to swelling and stretching of the infraorbital headache, whereas the most specific finding is jaw
nerve, there is often transient hypoesthesia in its claudication. However, the disease often presents
distribution lasting for 1 to 2 weeks. In addition, vaguely with systemic symptoms such as a fever of
the inferior rectus muscle may become entrapped unknown origin, fatigue, malaise, and anorexia. The
in the orbital floor defect; effectively tethering the diagnosis should be considered in any older person
globe such that upward gaze is limited or not with fever of unknown origin. Scalp pain is a more
possible. Patients may complain of diplopia ifpartial specific finding than headache, particularly when lo-
upward gaze is possible. Enophthalmos results from calized over the temporal artery. Vertigo is not a
loss of orbital volume and supporting structures. It manifestation of the disease. Permanent visual loss
may not be visible acutely because of the swelling occurs in only 15% of patients and is usually pre-
associated with the injury. However, delayed repair ceded by the development of blurring, diplopia, or
is required if it is present 2 weeks after the injury. amaurosis fugax. Corticosteroids are the treatment
Periorbital emphysema results from communication of choice and should never be withheld if the diag-
of the orbit with the sinus, an air-filled structure. It is nosis is seriously considered. Multiple studies have
important to note that exophthalmos resulting from demonstrated that treatment with steroids does not
a retro-orbital hematoma can also occur. This is a affect the accuracy of biopsy if performed within a
true emergency that may require immediate lateral few weeks.
canthotomy. (Figure reprinted with permission from
Harris JH. The radiology of emergency medicine, 4th ~ Answer B. This patient is infected with Giardia
ed. lippincott Williams & Wilkins; 1999:70.) Iamblia, the most common enteric parasite infection
worldwide. It is most commonly acquired through
~ Answer C. Patients with Rocky Mountain spotted drinking contaminated water. However, foodborne
fever (RMSF) typically develop a maculopapular rash and person-to-person transmission also occur. Back-
on the wrists and ankles 4 days after being bitten by packers frequently contract the illness after drinking
an American dog tick (Dermacentor varnwilis) or inadequately treated or untreated stream water that is
Rocky Mountain wood tick (Dermacentor andersoni) contaminated by animal or human fecal matter. The
infected with Rickettsia rickettsii. Despite its name, diarrheal illness is therefore often known as back-
North Carolina and Oklahoma account for more packer's diarrhea. Giardiasis is a noninflammatory,
than one third of RMSF cases. Wyoming and noninvasive diarrhea, so leukocytes are not seen in
Montana are the only Rocky Mountain states that stool samples. Instead, diagnosis relies on the detec-
have an appreciable number of RMSF cases. Once tion of trophozoites or cysts ("ova and parasites") in
formed, the rash rapidly spreads centrally to the stool specimens. Detection ofova and parasites varies
trunk and extremities and becomes petechial and from 60% to 80% after a single stool to greater than
purpuric although up to 15% of patients do not 90% after three stools. However, when the clinical
develop a rash. Other common findings include a history is consistent with Giardiasis, a diagnosis can
high fever (typically> 102°F), myalgias (particularly be made presumptively. To prevent chronic infec-
of the gastrocnemius), headache, vomiting, and tion as well as person-to-person transmission, both
malaise. Doxycycline is the treatment of choice in symptomatic and asymptomatic patients should be
all patients except for pregnant women, who should treated. The treatment of choice is metronidazole
receive chloramphenicol (CAM). Antibiotics should 250 mg t.i.d. for 7 days and is generally well tolerated.
never be withheld while awaiting confirmation of
the disease, as the case fatality rate in untreated ~ Answer C. Altered mental status, oculomotor dys-
cases is very high (vs. 3% in treated cases). Diagnosis function, and ataxia comprise the clinical triad of
is primarily clinical, as the peripheral WBC count Wernicke's encephalopathy. Alcoholics develop this
is usually normal, and serologic studies are not emergent condition due partly to thiamine defi-
useful until convalescence. The Well-Felix test has ciency and supplementation remains the mainstay of
Test3 79

management. Wernicke's encephalopathy may de- the trachea. The catheter can then be attached to
teriorate into Wernicke-Korsakoff syndrome, which an adapter from a size No. 3.0 endotracheal tube
adds the elements of memory disturbance and con- to allow for bag ventilation, or to high flow oxygen
fubulation. Magnesium therapy, glucose, and intra- tubing for percutaneous transtracheal jet ventilation.
venous fluids are important adjunctive therapies for In either case, these measures are only temporary
these disorders. Lorazepam is used to treat alcohol until a more definitive airway can be established.
withdrawal seizures but has no role in the man-
agement ofWernicke's encephalopathy. Haloperidol [!i) Answer A. Ipsilateral pleuritic chest pain and dys-
may be used to treat agitation and psychosis in al- pnea are the most common symptoms of a primary
cohol withdrawal. Pyridoxine therapy may be used spontaneous pneumothorax. Hemoptysis is uncom-
as part of a multivitamin that contains thiamine, but mon in spontaneous pneumothorax and would
is not essential to treat Wernicke's encephalopathy. signify a specific etiology for the pneumothorax such
Potassium repletion may be indicated if hypokalemia as tumor. Patients may occasionally be asymptomatic
is present or expected during the course of therapy. or have nonspecific complaints. Many patients de-
lay treatment for up to 1 week, and symptoms tend
~ Answer A. RSV is the most commonly isolated to resolve without treatment in 24 to 48 hours.
virus in patients with signs and symptoms of acute Primary spontaneous pneumothorax is three times
otitis media. Influenza is a distant second, and the more common in men than in women, and typically
other choices are uncommon. It is unclear whether occurs in tall, healthy young men. Other fuctors asso-
viruses directly cause pathology or if they simply ciated with spontaneous pneumothorax are smoking,
predispose to bacterial invasion ofthe middle ear. The changes in atmospheric pressure, mitral valve pro-
most common bacterial causes of acute otitis media lapse and Marfun's syndrome. The most common
are S. pneumoniae, M. catarrhalis, H. influenzae, and arrhythmia is a mild sinus tachycardia.
S. pyogenes.
[!!) Answer D. Hirschsprung's disease accounts for
[!1) Answer E. This patient has complete airway ob- roughly 20% ofcases of partial intestinal obstruction
struction due to foreign body aspiration. The classic in earlyinfuncy. Although intussusception is the most
triad of foreign body aspiration is coughing, wheez- common cause of intestinal obstruction in children
ing, and decreased or absent breath sounds. However, younger than 2 years old, the typical presentation
approximately 40% of patients may have no signif- is one of acute-onset, severe abdominal pain which
icant physical examination findings. Although this may be associated with vomiting and bloody stools.
patient initially had partial foreign body obstruc- Patients with pyloric stenosis present with progres-
tion, it progressed to complete obstruction and sive nonbilious projectile emesis. Patients with GERD
she now requires a definitive airway. Back blows do not develop signs of obstruction and most com-
and chest thrusts would be reasonable initim ap- monly have nonbilious emesis. Hirschsprung's dis-
proaches in infunts with foreign body aspiration. ease is usually diagnosed in the newborn nursery due
Abdominal thrusts can be used in children older to fuilure of newborns to pass meconium. Ninety-
than 12 months, although the Heimlich maneuver nine percent of full-term infunts pass meconium
is the initial procedure of choice for older chil- within 48 hours of birth. However, because there is
dren and adults. Blind finger sweeps, which were a spectrum of disease, some infunts may present in
advocated in the past, should be discouraged. As a delayed manner and may have a subtle presenta-
in this case, blind finger sweeps have the potential tion. The absence ofstool in the rectal vault in concert
of converting a partial airway obstruction to com- with signs and symptoms of obstruction should bring
plete airway obstruction. In the setting of complete about consideration ofHirschsprung's disease. Diag-
airway obstruction, a definitive airway must be es- nosis is first suspected through an abnormal "string
tablished. The fustest way to accomplish this in this sign" on barium enema. This is followed by rectal
case is by performing a cricothyroidotomy. A needle biopsy revealing the aganglionic segment of bowel
cricothyroidotomy instead of a surgical cricothy- responsible for the disease.
roidotomy should be performed in children younger
than 8 years. Surgical cricothyroidotomy is difficult ~ Answer A. Of all the choices, acute angle closure
to perform in a small child because of the small size glaucoma is most likely to cause painful loss ofvision.
of the cricothyroid membrane, and it places children All the other choices are much more likely to cause
at risk for subsequent subglottic stenosis. A nee- painless (rather than painful) loss of vision.
dle cricothyroidotomy is performed using a 12- to
16-gauge angiocatheter and inserting it through the [llJ Answer D. Phenytoin and lidocaine are class IB
inferior portion of the cricothyroid membrane into antidysrhythmic agents. Class I agents have their
80 1(}()() Questions to Help You Pass the Emergency Medicine Boards

primary effects on fast sodium channels and slow approximately 2 weeks, although new lesions may
down action potential depolarization and conduc- continue to appear such that the entire illness lasts up
tion. These effects are greatest in class IC agents to 6 weeks. The most common cause is streptococcal
(flecainide, propafenone), moderate in class lA infection in children and streptococcal infection and
agents (procainamide, quinidine), and least in class sarcoidosis in adults. Other causes include tuberculo-
m agents. Unlike classes lA and IC, class IB agents sis (TB), coccidioidomycosis, Yersinia or Chlllmydia
shorten repolarization time and action potential infection, inflammatory bowel disease, Hodgkin's
duration. Phenytoin is not used in the ED as an lymphoma, pregnancy, and drugs including oral
antidysrhythmic, but may cause important cardiac contraceptives and sulfonamides. The lesions usu-
conduction abnormalities in patients who take the ally respond to high-dose aspirin (650 mg every
drug for its antiepileptic effects. Loading phenytoin 4 hours) or NSAIDs (e.g., naproxen or indomethacin)
intravenously can occur at a rate no greater than and bed rest. Occasionally, patients are treated with
50 mg per minute, as its diluent, propylene glycol, supersaturated potassium iodide (mechanism is un-
can cause dysrhythmias or hypotension when given certain). Corticosteroids are effective but are rarely
too quickly. Fosphenytoin lacks this diluent and may used and may worsen the underlying infection if one
be given more quickly. is present. (Figure© David Efffron, MD, 2004. Used
with permission.)
~ Answer B. Patients with multiple myeloma are at
risk for hyperviscosity syndrome, which is char- ~ Answer B. This patient is suffering from thyroid
acterized by extremely high levels of pathologic storm. There are no specific criteria for establish-
proteins in the blood, causing increased viscosity and ing the diagnosis of thyroid storm, although scoring
vascular sludging. Micro-infarctions are common, systems have been developed to aid in its diagnosis.
especially in cerebral and ocular vessels. The triad However, the diagnosis of thyroid storm remains a
of vision problems, neurologic symptoms, and mu- clinical one, as laboratory abnormalities in thyroid
cosal bleeding in a patient with multiple myeloma or storm are no different than in patients with hyperthy-
W aldenstrom's macroglobulinemia strongly suggests roidism. The clinical manifestations of thyroid storm
the presence ofhyperviscosity syndrome. Laboratory include fever, tachycardia, and systolic hypertension
data, although helpful, often returns with errors as with a widened pulse pressure, tremor (especially
most standard equipment may be unable to ana- in the hands) as well as dysfunction of the central
lyze the blood because of the elevated protein levels. nervous system (CNS) and gastrointestinal (GI) dis-
Definitive treatment involves plasmapheresis. In pa- turbances. CNS disturbances range from agitation,
tients with severely altered mental status, simple restlessness and psychosis, to confusion and coma. GI
phlebotomy and saline replacement may rapidly im- manifestations include vomiting and diarrhea (e.g.,
prove the clinical condition. Hemodialysis is not hyperdefecation). Most cases of thyroid storm are
used in dysproteinemias. Colloids such as platelets associated with Grave's disease and occur after a pre-
and packed RBC transfusions will likely exacerbate cipitating event such as lithium withdrawal. Lithium
the viscosity. Erythropoietin has no role in the man- inhibits thyroid hormone release from the thyroid
agement of most acute conditions. gland so abrupt withdrawal may lead to a rapid rise
in free thyroid hormone levels. Although lithium
~ Answer C. Aneurysms> 5 em in size are at greatest can be used for the treatment of thyroid storm,
risk of rupture, although smaller aneurysms may thioamides such as propylthiouracil (PTU) and me-
also rupture. Debate exists as to the optimal time thimazole (MMI) are first -line agents as they prevent
of elective repair of asymptomatic aneurysms, but the production, secretion and peripheral conversion
patients presenting to the ED with symptomatic (in the case of PTU) of thyroid hormone. Lugol's
aneurysms should always be aggressively evaluated. solution or other iodine preparations should not be
used until at least 1 hour after thioamides are admin-
~ Answer B. The patient has erythema nodosum istered. When iodide preparations are given before
(EN), which is thought to be a hypersensitivity re- PTU or MMI, the intrathyroidal increase in iodine
action to a number of different antigens. It most results in increased thyroid hormone synthesis and
commonly occurs in women (female:male ratio of release. Aspirin should never be given to patients in
5:1) during the third decade but it frequently occurs thyroid storm because it prevents thyroid hormone
in children as well. Patients often experience a vague from binding to carrier proteins, resulting in an in-
prodrome offever, malaise, and arthralgias, followed crease in free thyroid hormone levels. Dantrolene is
by the development of painful oval erythematous a muscle relaxant that may be useful in neuroleptic
nodules typically over the shins. Individual lesions malignant syndrome, serotonin syndrome, or ma-
are not pruritic and are usually self-limited, lasting lignant hyperthermia. Propranolol is the first-line
Test3 81

agent in thyroid storm. It effectively combats the ~ Answer B. The patient likely has toxic shock
peripheral adrenergic effects in thyroid storm and syndrome (TSS), given the preceding skin infection,
rapidly improves the clinical scenario. sunburn-like diffuse rash, hypotension, and fever.
TSS is a toxin-mediated disease, either due to
~ Answer B. The patient has staphylococcal scalded staphylococcal TSS toxin-1 or Streptococcus pyogenes
skin syndrome (SSSS), also known as Ritter's exotoxins A and B. Staphylococcal TSS used to
disease. The disease is caused by an epidermolytic occur secondary to superabsorbent tampons in
toxin expressed by S. aureus, phage group II, menstruating women, but now, like streptococcal
and typically occurs in otherwise healthy children. TSS, is largely because of systemic and postsurgical
Infection typically begins as an innocuous infection infections. M ultiorgan failure is characteristic ofboth
of the pharynx or conjunctiva until a diffuse processes. Blood cultures are usually negative in
erythroderma develops that has a sandpaper-like feel, staphylococcal TSS and positive in approximately
resembling scarlet fever. The skin ultimately wrinkles, half of the cases of streptococcal TSS. Mortality for
develops transient blisters and then peels in large staphylococcal and streptococcal TSS is 5% and 30%,
sheets revealing glossy, moist red skin underneath. respectively. Management involves removal of any
Treatment is directed at S. aureus, and nafcillin is the foreign bodies, antibiotics directed at streptococci
best choice. (Figure courtesy of Gary Marshall, MD. and staphylococci, vasopressor&, intravenous fluids,
Reprinted in Chung EK. Visual diagnosis in pediatrics. and intensive care monitoring.
Lippincott Williams & Wilkins; 2006.)
[!g) Answer E. Rotavirus is an RNA virus which
~ Answer A. The pain of duodenal ulcers is usually causes a secretory diarrhea in young children, most
described as a burning or gnawing epigastric sensa- often in winter months. The peak age range is
tion that is decreased with food or antacids. The pain between 6 months and 2 years. Adults may get
typically occurs 2 to 3 hours after a meal. Classically, infected, but are generally asymptomatic. Symptoms
two thirds of patients with duodenal ulcers describe include nausea, vomiting, fever, and severe watery
pain that wakes them from sleep in the middle of the diarrhea. The duration of symptoms is generally <2
night, although few patients have pain on waking in weeks. Rotavirus does not cause an inflammatory
the morning. The pain of gastric ulcers tends to occur gastroenteritis, so fecal leukocytes and erythrocytes
more quickly after meals and may even be precipi- are usually absent. Treatment involves intravenous
tated by food in some patients. Therefore, anorexia hydration and supportive care. Antidiarrheals and
and weight loss occur in approximately 50% of pa- antibiotics are not indicated. Patients may require
tients with gastric ulcers, but rarely occur in patients admission for rehydration-stool studies should be
with duodenal ulcers. Duodenal ulcers are twice as sent for rotavirus culture and the patient should be
likely to be complicated by bleeding as gastric ulcers. contact isolated to help reduce spread of infection to
H. pylori, although more commonly found in the other patients.
setting of duodenal ulcers, is the major risk factor
for the development of either type of ulcer. Flexible [!j) Answer A. HSV-2 is the most common cause of
endoscopy is the diagnostic study of choice for peptic ulcerative vulvar and vaginal lesions, as approxi-
ulcer disease. Finally, only 15% to 20% of patients mately one in five sexually active adults is infected
colonized with H. pylori will develop a peptic ulcer with the virus. Because it is sexually transmitted,
in their lifetime. the presence ofHSV-2 in a pediatric patient should
trigger a meticulous search for other signs of abuse.
[gjJ Answer A. The thin orbital floor is the most Patients typically develop multiple scattered lesions
easily damaged part of the orbit in trauma. Globe in varying types and stages, including vesicles, pus-
injuries occur in one fourth of patients with orbital tules, and ulcers. The lesions tend to be shallow and
floor fractures. The teardrop sign seen on plain painful and frequently coalesce into larger lesions,
radiographs or CT scan is soft tissue that extends particularly in women. Primary infections tend to be
inferiorly from the orbital floor into the maxillary more severe than recurrent infections and are fre-
sinus, indicating a floor fracture. Antibiotics are often quently associated with systemic symptoms, includ-
given in patients with orbital wall fractures, but are ing fever, generalized malaise, headache and fatigue.
recommended only if the fracture extends through Syphilis is initially characterized by a painless chan-
an infected sinus. Patients with orbital wall fractures cre, which disappears without treatment. Chancroid
should be instructed not to blow their nose, as it may is an uncommon infection in the United States and
worsen the degree of herniation of globe contents is characterized by multiple genital ulcerations as-
into the sinuses. Nasal congestion should be treated sociated with a tender inguinal lymphadenitis called
with a 3-day course of nasal decongestants.
82 1(}()() Questions to Help You Pass the Emergency Medicine Boards

a bubo. The lymphadenopathy is typically unilateral maximized by limiting the time spent during inspi-
and occurs in 50% of patients. Lymphogranuloma ration (by setting high inspiratory flow rates between
venereum (LGV) is also a rare disease in the United 70 and 100 L per second and by using a square-wave
States and is characterized by a painless, and often form), as well as by decreasing minute ventilation
overlooked primary genital lesion. Patients typically (by decreasing either Vr or RR). The byproduct of
present during the second stage of illness, with a ten- these changes is hypercapnia. Previous approaches
der unilateral lymphadenopathy that may involve the aimed to normalize alveolar C02 by using higher
inguinal lymph nodes both above and below the in- respiratory rates and tidal volumes but resulted in
guinalligament resulting in a noticeable groove in increased morbidity and mortality from airway baro-
between ("groove sign"). Granuloma inguinale is an- trauma. Peak pressure reflects the pressure applied
other rare disease in the United States characterized to the large- and medium-sized airways as air is
by chronic, painless genital ulcerations. (Figure cour- pushed into the lungs by the ventilator. It strongly
tesy ofAllan R. DeJong, MD. Reprinted in Chung EK. reflects airway resistance and tends to be very high
Visual diagnosis in pediatrics. Lippincott Williams & in asthmatics due to their significant airway obstruc-
Wilkins; 2006.) tion. Plateau pressure reflects the pressure applied to
the small airways and alveoli after the air settles in the
~ Answer A. Methanol is metabolized to formalde- lungs. It is extremely important to monitor plateau
hyde by alcohol dehydrogenase, and formaldehyde is pressure and ensure that it remains <35 em H 20
converted to formic acid by aldehyde dehydrogenase. to avoid alveolar overdistension and barotrauma.
Formic acid accumulates preferentially in the ocular When plateau pressure is normal and intrinsic PEEP
tissues and the brain, most commonly the basal gan- is < 15 em H 2 0, peak pressure elevations are imma-
glia. Long-term morbidity of methanol overdose in- terial. Therefore, while high inspiratory flow rates
cludes blindness and a parkinsonian syndrome, with elevate the peak pressure, it is unnecessary to reduce
bradykinesia and rigidity. Treatment of methanol flow rates to decrease peak pressure. In fact, it is
and ethylene glycol overdoses involves correction of most often necessary to increase the inspiratory flow
the metabolic acidosis with sodium bicarbonate, in- rate in order to decrease inspiratory time, which also
hibition of alcohol dehydrogenase with fomepizole results in increased peak pressure. Peak pressure is
or ethanol, and dialysis of the toxic alcohol. determined by the rate of airflow, not the absolute
volume of air nor the respiratory rate.
~ Answer C. Idiopathic thrombocytopenic purpura
(ITP) causes immune-mediated destruction of [llJ Answer C. Pneumococcus is by far the most com-
platelets. Acute ITP is more often seen in chil- mon cause of pediatric bacteremia. Less common
dren and chronic ITP is seen in adults. Bleeding causes include the other answer choices. Patients
is the most common clinical finding and intracra- with higher peripheral WBC counts are at higher risk
nial bleeding is the most common cause of death. for bacteremia, but patients with H. influenzae and
Treatment in children is primarily supportive with N. meningitidis bacteremia often have only mildly el-
a high rate of spontaneous resolution. Mortality in evated WBC counts. Meningococcal bacteremia has
young adults and children is below 5%; in older the worst prognosis among all the answer choices.
adults, it jumps to almost 50% because of complica-
tions surrounding intracranial hemorrhage. Aspirin ~ Answer B. Avulsed primary teeth should never be
is contraindicated because of its irreversible platelet- reimplanted, as they may fuse with underlying sec-
killing effects. There is a female predominance in ondary teeth and cause considerable cosmetic defor-
adult patients. Platelets are not indicated until levels mity. Avulsed secondary teeth should be reimplanted
reach < 10,000 to 20,000 cells per mm3 • Treatment in as soon as possible. If teeth are reimplanted within
adults is with high-dose corticosteroids and/or IVIG. 30 minutes, approximately 100% will be viable, but
if 2 hours have elapsed since avulsion, the chance
~ Answer A. Owing to their significant airflow ob- of successful reimplantation is essentially zero. The
struction, mechanically ventilated asthmatics are at best-known medium for transporting an avulsed
risk for lung hyperinflation and concomitantly ele- tooth is its own socket, followed by Hank's solution;
vated airway pressures. To avoid these problems, the cold milk is the best alternative if either of these is not
most commonly used ventilatory strategy is "per- available. Teeth may be secured for up to 48 hours
missive hypercapnia." In this strategy, the patient by reimplanting the avulsed tooth and securing it to
is ventilated at settings that ensure adequate time two neighboring teeth on either side with a periodon-
for exhalation, which limits air trapping, and sub- tal pack. Before reimplantation, avulsed teeth should
sequent auto-PEEP and lung hyperinflation (with be gently rinsed with saline. Teeth should never be
elevated plateau pressures). Expiratory time can be scrubbed or treated with any cleaning solution as this
Test3 83

will destroy the periodontal ligament fibers which are Antihistamines can be used for symptomatic relief
essential for successful reimplantation. but do not affect the duration of illness. Tympanos-
tomy tubes and adenoidectomy may be indicated
@1) Answer A. Traumatic diaphragmatic injuries are for prevention of chronic/recurrent otitis media, but
almost equally caused by blunt and penetrating have no effect on acute management ofotitis externa.
mechanisms. They much more commonly occur Herpes zoster can occur in the ear, and is referred to
on the left side, because the liver serves as excellent as Ramsay Hunt syndrome. This condition requires
protection against both injury and post-traumatic antivirals and is associated with a vesicular rash and
herniation. Once the diaphragm is injured, the sometimes cranial nerve palsies.
defect remains without spontaneous healing due to
the constant pressure subjected to the diaphragm ~ Answer B. C. trachomatis causes ocular trachoma
by normal respiratory forces. The main problem in many third world countries (considered the most
with missed diagnosis is the high rate of delayed common cause of preventable blindness worldwide)
herniation of abdominal contents into the thorax, and is an important sexually transmitted disease in
which can occur months or even years after the the United States In the United States, C. trachomatis
initial injury. None of the standard diagnostic tests is a major cause of pelvic inflammatory disease
for trauma (Focused assessment of sonography in ([PID], or cervicitis) as well as urethritis in men. A
trauma [FAST], CT scan, diagnostic peritoneal lavage different serotype of the organism is also responsible
[DPL], radiography) is sensitive or specific enough to for lymphogranuloma venereum, a more invasive
accurately evaluate for diaphragmatic injury. Direct sexually transmitted disease.
visualization with thoracoscopy or laparoscopy in
suspected cases is required. [!!) Answer D . As with other venomous bites, scor-
pion envenomation is more severe in children than
@!) Answer D. The Focused assessment ofsonography in adults due to the smaller body size and subsequent
in trauma (FAST) scan consists of a series of volume of distribution. Though it has not been
ultrasonographlc images designed to assess for the scientifically studied, morphine has been reported to
presence of hemoperitoneum. Ultrasonography of trigger arrhythmias in the setting of scorpion enven-
the right upper quadrant, left upper quadrant, and omation so narcotics are avoided. Scorpion venom
suprapubic regions aids in this determination. A is primarily neurotoxic rather than cardiotoxic. Ini-
fourth view in the subxiphoid region focuses on the tially, the venom produces a cholinergic syndrome
pericardium to assess for effusion or tamponade. resulting in hypersalivation, lacrimation, urination
The FAST scan is ideally performed in the patient and urinary incontinence, defecation, gastrointesti-
with blunt trauma just after the primary survey is nal upset and emesis ("SLUDGE" syndrome). Some
complete and in conjunction with plain radiographs authors recommend atropine use in cases of hyper-
of the chest and pelvis. Pelvic fracture is assessed salivation and bronchorrhea as a means of averting
by the initial radiograph of the pelvis. The kidneys intubation. However, scorpion envenomation also
and abdominal aorta are retroperitoneal structures triggers norepinephrine release resulting in tachy-
which cannot be diagnosed with the use of FAST cardia, hypertension, myocardial depression, and
scan. Diaphragmatic injury is notoriously difficult pulmonary edema. Severe hypertension may occur
to diagnose with noninvasive studies (including and should be treated with nitroprusside. In most
CT, FAST, and diagnostic peritoneal lavage [DPL]) cases, patients only experience local pain and pares-
and may require thoracoscopy or laparoscopy for thesias that may last for several hours. Supportive
definitive diagnosis. care is all that is required, with monitoring of the
airway, EKG forST -segment changes, and vital signs.
~ Answer E. The patient has otitis externa, inflam-
mation ofthe external ear and tympanic canal almost ~ Answer D. Twenty-twopercentto39%ofcasesare
always because of infection. Trauma and excessive caused by medications. Delirium that begins while
moisture are commonly implicated in the develop- patients are taking a drug usually ceases once the
ment of the condition. Patients generally present drug is discontinued. All of the other findings listed
with otalgia and otorrhea. In nontoxic patients, it can cause delirium in elderly patients.
is treated with topical acetic acid washes with or
without topical antibiotics and steroids. A cotton ~ Answer E. Black widow spider bites are character-
or methylcellulose wick for drainage of the tympanic ized by an initial pinprick sensation followed by a
canal may be placed 1 em deep in the ear and stays for mild local inflammatory response. However, within
2 days. Systemically ill patients or diabetic patients re- 1 hour, crampy myalgias develop at the bite site
quire systemic antibiotics and sometimes admission. and spread up the extremity, eventually involving
84 1(}()() Questions to Help You Pass the Emergency Medicine Boards

the entire body. Classically, myalgias are most in- ~ Answer E. Flexor tenosynovitis is a purulent infec-
tense in the chest and abdomen and patients may tion of the flexor tendon sheath that usually results
present with a rigid abdomen that is impossible to from a penetrating wound of the digital flexor sur-
differentiate clinically from peritonitis. Patients also face. The infection spreads unimpeded sheath and
frequently have associated hypertension, diaphoresis, the patient presents with acute pain and signs of
nausea, vomiting, headache, dizziness, and weak- inflammation in the involved digits. Kanavel's four
ness. Symptoms typically begin to abate within a cardinal signs of flexor tenosynovitis include all of
few hours with only supportive care. Owing to their the signs listed, although tenderness along the flexor
small size, however, children may suffer from com- tendon sheath is frequently cited as the most sig-
plete cardiovascular collapse with the same degree of nificant of these findings. Although early infections
envenomation. may be treated with intravenous antibiotics directed
at Staphylococcus, incision and drainage is often re-
~ Answer C. H. pylori is the most common cause of quired. Owing to the limited space in the sheath,
gastritis. In acute H. pylori gastritis, patients com- purulent infections may cause a rapid increase in
monly present with epigastric pain, nausea, and vom- compartmental pressure resulting in ischemia to
iting. Gastritis is a histologic diagnosis. Furthermore, tendons and nerves. A hand specialist should be con-
although endoscopy reveals an inflamed, edematous, suited as soon as the diagnosis is seriously considered.
and friable gastric mucosa, there may be a lack of
neutrophilic infiltrates in which case many authors ~ Answer A. Sick sinus syndrome represents a range
prefer the less specific term gastropathy. In H. pylori of dysrhythmias due to sinus node dysfunction,
gastritis, there is an intense infiltrate, but several of causing bradycardias, narrow-complex tachycardias,
the other agents listed cause generalized inflamma- and combined states. It is treated by limiting the
tion of the gastric mucosa without such an infiltrate. extent of the bradycardia (with a pacemaker) and
Alcohol, aspirin or other NSAIDs, and Crohn's dis- the tachycardia (with an antidysrhythmic agent). It is
ease may all cause gastritis. Caffeine is not a cause usually seen in older patients or in patients taking AV
of gastritis but may predispose patients to GERD by nodal active agents. Lidocaine is used for ventricular
lowering the tone of the lower esophageal sphincter dysrhythmias and has no role in the management of
muscle. sick sinus syndrome. Atropine is the first-line agent
for sick sinus patients with severe bradycardia. Atrial
~ Answer A. Between 30% and 80% of alcoholics fibrillation is a common tachydysrhythmia seen in
have magnesium deficiency. Patients with hypomag- sick sinus syndrome.
nesemia are frequently asymptomatic or manifest
only nonspecific symptoms. The most prominent ~ Answer D. Owing to the anterior communicating
symptoms in the ED are neuromuscular and car- artery, lesions ofthe anterior cerebral artery proximal
diovascular, and magnesium deficiency tends to to the communicating artery are generally well
mimic calcium deficiency. The mechanism of hy- tolerated. Lesions distal to this anastomosis result
pomagnesemia in alcoholism is thought to be a predominantly in leg weakness and sensory loss
combination of malnutrition, increased renal excre- as well as a variety of personality and behavioral
tion, and gastrointestinal losses from vomiting and changes. These changes include abulia, which is the
diarrhea. Diuretic therapy is also a very prevalent inability of patients to make decisions. The upper
cause of hypomagnesemia, although the subsequent extremities may be involved but are typically only
volume loss increases magnesium reabsorption in the mildly affected. Furthermore, the deficits are usually
proximal tubule. Therefore, magnesium depletion in most marked distally. The tongue and the face are
the setting of diuretic therapy tends to be modest. generally spared.
Hypomagnesemia is the most common electrolyte
abnormality in ambulatory diabetic patients and is ~ Answer B. Nitroprusside is a strong arterial and
also common in diabetic ketoacidosis. In these pa- venous dilator only available as an IV drip. It is easily
tients, magnesium is lost through the urine due to titrated and provides effective and predictable BP
glycosuria. control. Extravasation of nitroprusside causes severe
skin necrosis. Nitroprusside is metabolized to thio-
~ Answer D. Although all the choices are risk factors cyanate and excreted renally. Cyanide toxicity is not
for aortic dissection, the most common is hyper- common, but thiocyanate toxicity can occur in renal
tension. Though approximately half the number of failure, causing systemic symptoms. Nitroprusside
patients with Marfan's syndrome develop aortic dis- may increase cerebral blood flow and intracranial
sections, < 10% of all dissections are in Marfan's pressure (ICP). Thiocyanate may cause damage to
patients.
Test3 85

the fetal thyroid and nitroprusside should be avoided isolated from affected individuals who are 3 weeks
during pregnancy. Nitroprusside is extremely fast into their illness, treatment is recommended for all
acting, with a rapid onset and offset, making it ideal individuals who have had symptoms for 4 weeks
for rapid, predictable BP control. or less. Such treatment is thought to decrease the
carriage rate and shedding of the bacterium, thereby
~ Answer D. The patient has scattered human bite reducing transmission. High-risk personnel, includ-
marks, which are circular in nature and have a per- ing health care workers, or persons who may come
forated, erythematous border. Human bites are a into contact with infants of pregnant women in the
common sign of child abuse. Management involves third trimester, may receive treatment up to 6 to 8
tetanus prophylaxis, treatment of associated celluli- weeks after developing symptoms.
tis, and notification of appropriate social services.
The rash of Henoch-Schonlein purpura presents as ~ Answer C. The image demonstrates a small bowel
palpable purpura on the buttocks and lower extrem- obstruction (SBO). Postoperative adhesions account
ities. Idiopathic thrombocytopenic purpura (ITP) for more than 60% of cases, malignant neoplasms
presents as nonpalpable petechiae and purpura with account for approximately 20%, and incarcerated
laboratory evidence oflow platelets. The rash ofLyme hernias account for roughly 10% of cases. The
disease, erythema rnigrans, is an erythematous rash classic mnemonic for the top three causes of small
with central clearing, classically on the trunk in a pa- bowel obstruction is "ABC," representing adhesions,
tient with travel to an endemic area. Roseola presents bulges (hernias) and cancer. However, neoplasms
as sudden onset of fever in young children, fol- are actually the second most common cause. Most
lowed by a generalized macular rash after the patient tumors are metastatic lesions that cause extrinsic
has completely defervesced. (Figure from Reece RM, compression of the intestine secondary to peritoneal
Ludwig S. Child abuse: medical diagnosis and man- implants that have spread from an intra-abdominal
agement, 2nd ed. Philadelphia: Lippincott Williams primary tumor such as from the colon, ovary, or
& Wilkins, 2001;150, with permission.) pancreas. The most common hernias to result in
obstruction are ventral and inguinal hernias. Flat
~ Answer C. In symptomatic individuals, the most (or supine) films classically demonstrate multiple
common manifestation of infection with B. pertus- dilated loops of small intestine in a "stepladder''
sis is a prolonged, spasmodic cough that may last pattern without evidence of distal small bowel
for several weeks. In adolescents and adults, the or colonic distention. Upright films demonstrate
mean duration of illness is between 36 and 48 days. multiple air-fluid levels. As a general rule, the larger
Despite widespread belief that childhood vaccina- the number of dilated bowel loops, the more distal
tion confers lifelong immunity, the last booster is the obstruction. The overall sensitivity of plain films
given at age 4 to 6 in the United States, and it for an SBO is roughly 60%. (Figure reprinted with
generally provides immunity until the age of 10 to permission from Fleisher GR, Baskin MN. Atlas of
12 years. Because no further boosters are currently pediatric emergency medicine. Lippincott Williams &
recommended in the United States, patients remain Wilkins; 2003.)
susceptible to infection in their early teenage years
into adulthood. In children, the classical pertussis in- ~ Answer A. The patient has an acute anterior ST-
fection occurs in three discrete phases--the catarrhal segment elevation myocardial infarction (STEMI)
phase, paroxysmal phase, and convalescent phase. In with evidence of congestive heart failure (CHF) due
adults, however, prolonged cough may be the only to cardiogenic shock. In these patients, immediate
manifestation of infection. Between 70% and 99% angioplasty has been shown to reduce mortality more
of adolescents and adults have paroxysmal cough, than fibrinolytic therapy, and is the preferred defini-
associated with an inspiratory whoop in 8% to 82% tive therapy. Choices C and E are not appropriate
of patients (i.e., highly variable), coughing during for patients with acute STEMI. Choice D represents
the night in 65% to 87% of patients, and post-tussive treatment ofa non-ST-segment elevation myocardial
vomiting in 17% to 50%. Patients may also experi- infarction (NSTEMI).
ence episodes of choking, sweating attacks, syncope,
or encephalopathy. Erythromycin is the treatment of ~ Answer D. Pemphigus vulgaris is the most com-
choice for symptomatic patients and for prophylaxis mon form ofpemphigus, although it is a rare disease.
of asymptomatic patients who have come into con- It most commonly affects older individuals in the
tact with infected individuals. Antibiotics may reduce sixth decade. Nikolsky's sign, which occurs when
both the severity and duration ofsymptoms ifthey are pressure applied to the margin of a blistered or ul-
given to affected patients within 1 week of the onset cerated lesion expands the lesion in to the adjacent
of symptoms. However, because B. pertussis has been apparently normal skin, is the clinical hallmark of
86 1(}()() Questions to Help You Pass the Emergency Medicine Boards

the disease. Although antibiotics are important in Fowler NO. Clinical electrocardiographic diagnosis:
cases of secondary infection, corticosteroids are the A problem-based approach. Philadelphia: Lippincott
mainstay of therapy. Most patients have oral lesions Williams & Wilkins; 2000, with permission.)
that start as blisters, which then spontaneously rup-
ture to leave painful oral ulcers that are slow to heal. ~ Answer D. All of the drugs listed except for
Oral ulcers may precede cutaneous disease by sev- hydrocortisone are fluorinated corticosteroids that
eral months. The cause of the disease is unknown, should not be used on the face. Corticosteroids
although there is a genetic predisposition and an are fluorinated to increase potency but fluorination
autoimmune mechanism involving circulating IgG also increases side effects. Although fluorinated
antibodies to k.eratinocytes. compounds may have a role in the treatment of
facial rashes, drugs used for that purpose should be
~ Answer B. The most dangerous cause of neonatal prescribed by dermatologists.
conjunctivitis is N. gonorrhoeae. It is less often seen in
Western countries due to chemoprophylaxis, but is ~ Answer C. Ipratropium is primarily useful as a
still an important cause of ophthalmia neonatorum second-line, adjunctive agent in the treatment of
due to its potential for invading the intact corneal patients with acute asthma exacerbations. Multiple
epithelium and causing blindness. Treatment should trials in the late 1990s as well as a few meta-analyses
be prompt with topical and parenteral antibiotics. have demonstrated benefit in patients treated with
Choices A and C occur more often in slightly older in- a combination of albuterol and ipratropium versus
fants and cause less severe sequelae. Choices D and E albuterol alone. In pediatric populations, combina-
may be more common given the practice of prophy- tion therapy decreases treatment time in the ED, al-
laxis of all neonates with erythromycin ointment, but buterol dose requirements before discharge, as well as
both processes are easily halted by removing the ery- hospitalization rates. In adults, combination therapy
thromycin and cause no long-lasting sequelae. has been shown to increasepeak expiratory flow rate
(PEFR) more than albuterol alone. However, there
~ Answer A. The most common cause of upper is conflicting data regarding ipratropium and some
GI bleeding in both the general population and studies have failed to demonstrate benefit. Owing to
alcoholics is peptic ulcer disease. When combined this conflicting data, ipratropium has been relegated
with NSAIDs and other ulcerogenic drugs, alcohol to use as a second-line agent. As it appears to be safe
can contribute to the loss of protective gastric and well tolerated, most experts advise using it as an
mucosal lining. Treatment in alcoholics is the same as adjunctive agent in severe asthma exacerbations. It is
for nonalcoholics, with special attention to electrolyte not recommended for use as a single agent primarily
management, prevention of alcohol withdrawal, because of its slower onset of action and because it
and attention to seizure precautions. Gastric and only alleviates cholinergicallymediated bronchocon-
esophageal varices occur at a higher rate in those striction. The main benefit of ipratropium over at-
alcoholics who have a history of portal hypertension ropine is its superior side effect profile.
from cirrhosis. Boerhaave's syndrome refers to
rupture of the esophagus from forceful vomiting. ~ Answer D. Initial symptoms are typically "pos-
It usually occurs in alcoholics, and presents more itive," such as tingling or burning, rather than
often as sepsis and hypovolemia due to mediastinitis "negative,'' such as numbness. The symptoms be-
rather than GI bleed. Arteriovenous malformation is gin distally and progress more proximally and affect
a more common cause of lower GI bleed than upper the lower extremities more than the upper extremi-
Gibleed. ties. Over time, weakness develops usually involving
dorsiflexion of the big toe followed by weakness of
~ Answer E. The patient has Mobitz type II second- foot dorsiflexion, foot drop, loss of ankle jerks, and
degree atrioventricular (AV) block, which is an finally a "steppage gait."
indication for admission to a telemetry unit and
possible pacemaker placement. The PR interval [g) Answer A. Vitreous hemorrhage is usually caused
is prolonged and does not show any observable by diabetic retinopathy or retinal detachments.
pattern of increasing prolongation as in Mobitz Symptoms are similar to retinal detachment, with
type I second-degree AV block (Wenckebach phe- floaters early in the course and painless visual
nomenon). Dropped QRS complexes do not occur loss later. The red reflex is usually diminished
in any predictable pattern like they do in Mobitz or darkens into a black reflex. As in most acute
type I. Cardioversion and amiodarone are not indi- ophthalmologic disorders, Valsalva maneuvers and
cated in stable bradydysrhythmias, and defibrillation recumbent positioning are to be avoided. The
of an awake patient is never indicated. (Figure from presence of an afferent pupillary defect indicates
Test3 87

a concomitant retinal detachment. Ophthalmologic arterial aneurysm locations are femoral, splenic, and
consultation should be sought in the ED in patients hepatic.
with suspected retinal detachment or traumatic
vitreous hemorrhage. ~ Answer B. With the history of recent diagnosis of
schizophrenia (and likely newly started antipsychotic
~ Answer B. Adenosine is an extremely fast-acting medication) the patient has evidence of neuroleptic
AV nodal blocker used for the treatment of narrow- malignant syndrome. Hyperthermia, muscle rigidity,
complex tachydysrhythmias, and has little effect altered mental status, and elevated creatine phos-
on ventricular cells. Onset of action is 5 seconds phokinase (CPK) levels are characteristic. Treatment
and duration is 30 seconds, with a half-life of 10 involves aggressive sedation with benzodiazepines,
seconds. For this reason, adenosine must be pushed cooling, and paralysis with neuromuscular block-
rapidly with a saline flush immediately following the ade in severe cases. Acetaminophen is unlikely to
intravenous push. be of benefit in patients with hyperthermia because
hyperthermia does not exhibit an elevation of the hy-
~ Answer B. The patient has acute necrotizing ul- pothalamic set point as is seen in fever. Amantadine
cerative gingivitis (ANUG), commonly known as and bromocriptine are dopamine agonists which
trench mouth. The gingiva is painful and friable, have not been proved to be beneficial in patients
unlike ordinary gingivitis. Treatment involves oral with neuroleptic malignant syndrome. Haloperidol
antibiotics and good oral hygiene. Spirochetes and is an antipsychotic which might further exacerbate
fusobacteria predominate in what is likely a bacterial the pathophysiologic process in this case. Dantro-
overgrowth process. Vincent's angina refers to exten- lene, which blocks calcium release in muscle cells,
sion of ANUG to the tonsils, and cancrum oris refers may afford some benefit, but is unlikely to be more
to extension to the lips and buccal mucosa. Direct effective than benzodiazepines and paralytics.
contact with secretions does not confer increased risk.
[!!) Answer C. Family history of suicide is an inde-
~ Answer E. Most patients suffering peptic ulcer pendent risk factor for completed suicide. Other risk
perforation are elderly and NSAID use is involved factors include depression, schizophrenia, substance
in roughly 50% of cases. Smoking is the leading abuse, prior attempts, presence of a firearm in the
risk factor for perforation among young patients. home, and feelings of hopelessness and long-term
Cocaine use may lead to perforation of ulcers loneliness. Young women have the lowest rate of
of the juxtapyloric region, possibly because of completed suicide, but have the highest rate of sui-
vasoconstriction or vascular thrombosis. The pain cide attempts. Patients with antisocial personality
of peptic ulcer perforation is classically described in disorder do not have higher rates of suicide com-
three stages. Initially, patients present with severe, pletion unless there is comorbid substance abuse.
diffuse abdominal pain and may present with signs Generalized anxiety disorders and simple phobias do
of shock. After minutes to hours, this phase tends not confer a higher risk of completed suicide in the
to resolve and patients begin to look and feel better absence of depression or substance abuse.
and may have normal vital signs. However, they will
nearly always have signs of peritonitis upon physical [!!) Answer C. The patient likely has a radial head
examination. Most notably, patients will have a subluxation, commonly referred to as a nursemaid's
rigid, tender abdomen, and signs of pelvic peritoneal elbow. With forced extension of the arm, the radial
inflammation (assessed by a rectal examination). The head slips out of the annular ligament. Without a
final stage is characterized by worsening abdominal history of trauma or severe tenderness on exam-
pain, abdominal distension, and signs of sepsis. ination, radiographs are not necessary. Reduction
Interestingly, only 70% ofpatients will have evidence is best performed by flexion-supination or flexion-
of intraperitoneal free air on upright chest or hyperpronation of the forearm with pressure on the
abdominal radiography. In vague cases, an upper GI radial head. An audible or palpable click indicates
series may be performed with water-soluble contrast successful reduction. Flexion-hyperpronation may
media. Endoscopy is contraindicated because the be more successful than flexion-supination, but both
insufflation of air may open a spontaneously closed strategies can be employed in the same reduction
perforation. attempt for optimal results.

~ Answer E. Popliteal artery aneurysms are the ~ Answer C. The superior lid contains a hordeolum
most common nonaortic aneurysms, and the large (or stye) with surrounding blepharitis. A horde-
majority occurs in conjunction with concomitant olum refers to localized infection of a meibomian
abdominal aortic aneurysm (AAA). Other peripheral gland in the lid, usually because of S. aureus.
88 1(}()() Questions to Help You Pass the Emergency Medicine Boards

Treatment involves warm compresses, with topi- genitourinary tract or Shigella, Salmonella, Campy-
cal erythromycin ointment applied to the lid if there lobacter, or Yersinia infection of the gastrointestinal
is a surrounding blepharitis. Systemic antibiotics are tract. It is part of a group of arthritides known as
rarely indicated in patients with hordeolum. Eyelid the seronegative spondyloarthropathies. This group
culture is unlikely to be helpful or easy to obtain. As- includes ankylosing spondylitis, psoriatic arthritis,
piration of the area is strictly contraindicated given reactive arthritis and the arthropathy of inflamma-
the proximity to the globe. Topical antivirals are tory bowel disease. They are grouped because of their
indicated only in patients with herpes keratitis. The common involvement of the sacroiliac joint, lack of
lower lid has a chalazion, which is a subacute, granu- rheumatoid factor, and presence of the HLA-B27
lomatous lesion also treated with warm compresses. genetic marker. Reiter's syndrome is most common
(Figure from Tasman W, Jaeger EA, eds. The Wills in young men aged 15 to 35 and occurs 2 to 6 weeks
Eye Hospital atlas of clinical ophthalmology, 2nd ed. after an episode of urethritis or dysentery. The classic
Philadelphia: Lippincott Williams & Wilkins; 2001, triad is arthritis, urethritis, and conjunctivitis. The
with permission.) arthropathy in Reiter's syndrome is an enthesopa-
thy, which refers to pathology at the site of ligament
II!] Answer C. This patient has Rocky Mountain spot- or tendon insertion to bone. It most commonly in-
ted fever (RMSF). Classically, patients with RMSF volves the lower extremities, particularly the Achilles
develop a maculopapular rash on the wrists and tendon ("lover's heel").
ankles 4 days after being bitten by an American
dog tick (D. variabilis) or Rocky Mountain wood ~ Answer D. Intravenous medications are most ap-
tick (D. andersoni) infected with R. rickettsii. Despite propriate for rapid control ofBP. Nicardipine is safe
its name, North Carolina and Oklahoma account and effective during pregnancy and produces pre-
for more than one third of RMSF cases. The rash dictable control of BP during hypertensive crises.
rapidly spreads centrally to the trunk and extremities Nitroprusside also provides excellent BP control, but
and becomes petechial and purpuric. However, up has not been proved safe during pregnancy. Nitro-
to 15% of patients do not develop a rash ("spot- glycerin causes preload reduction, but has little effect
less fever''). Other common findings include a high on afterload reduction, which is the more important
fever (typically> 102°F), myalgias (particularly ofthe intended mechanism of action for antihypertensive
gastrocnemius), headache, vomiting, and malaise. agents during crises. Clonidine and hydrochloroth-
Doxycycline is the treatment of choice in all patients iazide are oral medications that have a limited role in
except for pregnant women, who should receive acute management of BP during hypertensive crises
chloramphenicol (CAM). CAM is the drug of choice due to unreliable absorption and efficacy.
during pregnancy due to the effects oftetracyclines on
fetal bones and teeth. However, CAM should be used ~ Answer B. Reciprocal ST depressions associated
with caution and should be avoided in near-term with ST elevations have high specificity for acute
pregnancies or during the third trimester in order MI (reported to be >90%) and are not usually
to avoid fetal bone marrow suppression, which re- present in nonischemic conditions. Their presence
sults in the "gray baby syndrome." Previously, CAM confers a higher risk ofpoor outcomes. Reciprocal ST
was the recommended agent in children as well, but depressions are usually downsloping. Most inferior
because doxycycline is more effective and the risk wall Mls exhibit reciprocal ST depressions, whereas
of teeth staining is low given the short duration of most anterior wall Mls do not.
treatment, doxycycline is now the drug of choice for
all patients except pregnant women. ~ Answer C. Most authors have abandoned the
old terminology of hyperglycemic hyperosmolar
~ Answer C. Patients with anterior glenohumeral nonketotic coma (HHNC) in favor of HHS. HHS
(shoulder) dislocations may have associated axillary reflects the fact that <20% of these patients present
nerve injury. Findings include diminished sensation in a coma. HHS is the result of an unremitting
in the region of the deltoid and impaired deltoid osmotic diuresis due to hyperglycemia, and is
function exhibited by loss of muscle contraction with characterized by extreme hyperglycemia (mean
abduction against resistance. Other less common glucose is > 1,000), hyperosmolarity (at least >320
findings may include more distal upper extremity mOsm per L), and dehydration (average fluid deficit
nerve compression. Lower extremity or truncal area is roughly 10 L). Although a minor ketosis may
hypoesthesia would be extremely atypical. be present, it is never significant. Infections such
as pneumonia or urosepsis are the most common
[liD Answer E. Reiter's syndrome is a reactive arthritis precipitants, accounting for 60% of cases. The
that occurs following C. trachomatis infection of the mortality rate, which hovers at approximately 15%,
Test3 89

is significantly higher than in patients with DKA. and noxious bile. Such patients also often experience
The degree of mental status change correlates with hemodynamic instability as a result of their primary
the serum osmolarity rather than the degree of disease process, which places the biliary epithelium
hyperglycemia. Insulin is a second-line agent in HHS. at risk for ischemia. The presentation of acalculous
It should never be given without prior intravenous disease in this population is often subtle, and may
fluid administration because insulin delivery will declare itself as an isolated and unexplained fever.
encourage glucose utilization and entry into cells This leads to a delay in diagnosis and contributes to
resulting in an acute drop in intravascular fluid its far more fulminant course. By the time it is di-
volume and possible circulatory collapse. agnosed, approximately half the number of patients
have already experienced a major complication such
[!lJ Answer A. The patient has evidence of serotonin as gangrene or perforation. The mortality rate ranges
syndrome, a constellation of neurologic, GI, and car- from 10% to 50%. Patients with AIDS are known
diac findings caused by excessive serotonin activity to be at increased risk for the disease, especially in
due to medications. The diagnostic criteria include the setting of cytomegalovirus or Cryptosporidium
specific symptoms, the presence of two or more infection, though the reasons for this are not known.
serotonergic drugs, and the absence of neuroleptic
agents or other cause for the symptoms. Hyper- ~ Answer D. The Kleihauer-Betke test (KBT) de-
reflexia, hyperthermia, altered mental status, and tects the presence and quantifies the volume of
diarrhea are characteristic. Medications associated fetal RBCs in the maternal circulation. Unfortu-
with serotonin syndrome include combinations of nately, it is an insensitive test, requiring a mini-
the following: Selective serotonin reuptake inhibitor mum of 5 mL of fetal hemorrhage for detection.
(SSRI)s, monamine oxidase inhibitors (MAOis), cat- Because as little as O.Ql to 0.03 mL of fetal blood
echolamine releasers (cocaine, amphetamines, and may result in maternal Rh sensitization, the KBT is
dextromethorphan), nonselective serotonin reup- not useful in most pregnant patients. Therefore, all
take inhibitors (tricyclic and atypical antidepressants, Rh-negative pregnant patients should be given Rh
carbarn.azepine, meperidine, methadone), and sero- immune globulin. The dose is 50 li-S in the first
tonin agonists (buspirone, lithium, LSD, sumatrip- 12 weeks, and 300 ~g after 12 weeks' gestation.
tan). Treatment ofserotonin syndrome is supportive, However, in cases in which extensive maternal-
but cyproheptadine, a serotonin antagonist, may be fetal hemorrhage (MFH) is suspected, the KBT
helpful in some cases. Choices B to E do not con- may be useful because it can identify patients in
tribute to serotonin syndrome. whom >30 mL of MFH has occurred. Such pa-
tients should received a second 300 li-S dose of Rh
[!!) Answer C. Condyloma acuminatum is a sexually immune globulin because each 300-~-tg dose is only
transmitted disease caused by human papillomavirus enough to prevent sensitization from 30 mL of fe-
(HPV). It is usually found in the genital area, but tal blood. The only patients in whom this occurs
may be found elsewhere. It most commonly occurs in are patients who have suffered catastrophic trauma
young men. It typically starts as small, pedunculated ( < 1% of pregnant trauma victims require a second
papules that are 2 to 3 mm in diameter and 10 to dose). Therefore, the KBT should not be performed
20 mm long. They may occur as single papule or in most pregnant ED patients who have suffered
in clusters may develop into large, cauliflower-like trauma.
masses. The surface is dry and highly keratinized, and
they are typically asymptomatic unless they become [i!) Answer D. RSV is the most common cause of
superinfected. In contrast, condyloma lata, which bronchiolitis, followed by parainfluenza, influenza,
is caused by T. pallidum (syphilis), is a weeping, adenovirus, and rhinovirus. To avoid infecting other
wart-like lesion that emits a foul odor. The two are uninfected patients and health care workers (who
easily distinguishable clinically and neither lesion has then transmit infections to yet more patients),
bluish telangiectasias. patients with RSV bronchiolitis should be placed
in respiratory isolation. Ifmultiple patients with RSV
[!!) Answer A. Acalculous cholecystitis comprises bronchiolitis are admitted, then the group could be
roughly 15% of acute cholecystitis cases. It most isolated as a cohort until discharge. Bacillus anthracis
commonly occurs in elderly men who are recovering is not transmitted from person to person. Patients
from nonbiliary tract surgery in an intensive care with COPD exacerbations and community-acquired
unit (ICU) setting. Patients in ICU settings are of- pneumonia also do not require isolation. Finally, the
ten subjected to prolonged fasting and immobility, medical student with a positive pressure differential
which prevents gallbladder emptying and prolongs (PPD) but no symptoms of active TB has latent TB
exposure of the biliary epithelium to the highly acidic infection and is not contagious to other people.
90 1(}()() Questions to Help You Pass the Emergency Medicine Boards

~ Answer C. This patient has an aortoenteric fistula should have a low threshold to perform angiography
until proved otherwise. The classic presentation is on individuals at risk for brachial artery injury.
abdominal pain and gastrointestinal bleeding that The presence of a distal pulse deficit mandates
resolves spontaneously (a so-called "herald bleed") exploration and repair. A compartment syndrome
in a patient with a prior history of an aortic graft. The at the elbow may result in Volkmann's ischemic
initial bleeding is thought to stop when splanchnic contracture due to ischemia, injury, and fibrosis
pressure drops and allows an adequate clot to form. of forearm structures. In its most severe form,
However, the initial bleed is often followed by a Volkmann's ischemic contracture results in elbow
massive and often fatal hemorrhage days or possibly flexion, forearm pronation, wrist flexion, thumb
weeks later. The fistula is caused by S. aureus or adduction, metacarpophalangeal joint extension,
Escherichia coli infection of a the prior graft and and finger flexion. (Figure courtesy of Robert
typically forms between the aorta and the distal Hendrickson, MD. Reprinted with permission from
duodenum. Therefore, all patients with a history Hendrickson R. Greenberg's text-atkis of emergency
of aortic graft and gastrointestinal bleeding should medicine. Lippincott Williams & Wilkins; 2004:492.)
undergo esophagogastroduodenoscopy to search for
a fistula in the distal duodenum. Most of these ~ Answer A. The diaphragm elevates during preg-
patients, however, have an alternative, more common nancy, resulting in decreased total lung capacity
cause of gastrointestinal bleeding. and functional residual capacity. However, diaphrag-
matic excursion actually increases resulting in an
~ Answer D. Swallowed maternal blood is a com- increased tidal volume and mild alveolar hyperventi-
mon cause of factitious gastrointestinal bleeding in lation despite the fact that the respiratory rate remains
young neonates. The Apt test involves the application unchanged. The RBC mass and hemoglobin increase
of alkali to a small sample of bloody stool. Owing throughout pregnancy. RBC mass rises steadily to
to its different composition from adult hemoglobin, term, as it increases 18% without iron supplemen-
fetal hemoglobin (which is composed of two a and tation. In contrast, the plasma volume, which also
two y subunits instead of two a and two f3 subunits increases during pregnancy, plateaus at 30 to 34
as in adult hemoglobin) is resistant to denaturation weeks' gestation. The peripheral WBC count steadily
by this application. After the application of alkali, rises during pregnancy and may be as high as 20,000
fetal hemoglobin will remain pinkish red upon mi- to 30,000 mm3 during labor after which it re-
croscopic examination whereas adult hemoglobin turns to normal in approximately 1 week. Finally,
will appear brownish. Both the Rosette test and the the glomerular filtration rate increases throughout
Kleihauer-Betke test (KBT) are used to detect the pregnancy.
presence of fetal maternal hemorrhage.
[iiJ Answer E. Placenta previa refers to a placenta that
~ Answer B. The image reveals a posterior elbow overlies or lies in close proximity to the internal
dislocation. Brachial artery disruption or injury cervical os. The classic presentation of women
is the most serious complication of posterior with placenta previa is painless second or third
elbow dislocations, but ulnar nerve injuries are trimester vaginal bleeding. However, most women
the most common complication. Median nerve are asymptomatic and are diagnosed on routine
injuries are the second most common associated ultrasonography. Digital vaginal examination should
injuries and may occur in concert with ulnar nerve never be performed before ultrasonography in the
injuries. Decreased function in the distribution of second or third trimester because it can provoke
either the ulnar, median, or radial nerves after disastrous bleeding in patients with asymptomatic
reduction is an indication for surgical exploration placenta previa. With progression ofpregnancy, 90%
and decompression. Postreduction functional loss of low-lying placentas will migrate away from the
most commonly occurs because of entrapment of cervical os. Disseminated intravascular coagulation
the median nerve. Functional loss that exists before is a common complication of abruptio placentae but
reduction is most commonly a neurapraxia and not placenta previa.
spontaneous recovery is the rule. Therefore, such
injuries should be well documented and followed [j!) Answer C. There is some disagreement regarding
by dose outpatient observation. The brachial artery the optimal temperature for rewarming or thawing
is the most commonly injured vascular structure in frostbitten tissue. However, all authors agree that
posterior elbow dislocations. Although the presence optimal rewarming occurs through immersion in
of a radial pulse is reassuring, it does not ensure a water bath with a closely regulated temperature.
an intact brachial artery, particularly in the setting Most sources cite 40°C to 42°C as the optimal
of a compartment syndrome. Therefore, physicians range, although temperatures as low as 35°C have
Test3 91

been recommended as they tend to cause less pain is atrial fibrillation, which puts the patient at high
during rewarming. Upon arrival, the affected area risk for thrombus formation and embolism.
should be rapidly rewarmed for 15 to 30 minutes or
until thawing is complete. Indicators of successful ~ Answer D. The foundation of treatment for rhab-
thawing include increased flexibility, erythema, and domyolysis is the administration of large volumes
hyperemia. Rewarming can be intensely painful, and of normal saline early in the course of the disease.
parenteral analgesics may be required, especially in Urine output should be maintained between 200 and
cases of deep frostbite. Direct tissue massage should 300 mL per hour and patients may require as much
never be performed as it may cause increased tissue as 20 L of fluid in the first 24 hours to achieve such a
loss. In addition, field rewarming and rewarming flow rate. Mannitol is an osmotic diuretic that may
with direct heat sources should never be performed help maintain urine output, especially in cases of
because of the high risk of incomplete thawing and oliguric renal failure. Bicarbonate causes urine alka-
refreezing which results in increased tissue loss. linization, which helps to keep myoglobin soluble,
and may therefore enhance its clearance. The goal
[!!] Answer D. Iron toxicity can be life threatening. is to keep the urine pH above 6.5. Deferoxarnine is
Ingestion of large quantities of iron overwhelms the an iron chelator that may have a protective role as
body's iron-binding capacity and causes gastroin- it inhibits lipid peroxidation, which may shield my-
testinal, cardiac, CNS, hepatic, and renal damage. ocyte membranes. Furosemide has also been used in
Nausea, vomiting, diarrhea, and GI bleeding are the cases of oliguric renal failure. However, furosemide
most common symptoms. Diagnosis involves serial causes urine acidification and may therefore enhance
serum iron levels and plain abdominal radiographs myoglobin precipitation into casts resulting in wors-
to demonstrate passage of the radiopaque iron pills. ening renal function.
Treatment involves whole bowel irrigation with
polyethylene glycol, deferoxamine chelation in pa- ~ Answer E. Pseudotumor cerebri, also known as
tients with severe overdoses (defined as rising iron idiopathic intTacranial hypertension, is a syndrome
levels, absolute level >500 JLg per d.L, or wors- of increased intracranial pressure (ICP) without an
ening clinical course), and dialysis of the chelated identifiable cause. To satisfy the diagnostic criteria
iron when renal failure is present. Activated char- for the disease, there can be no evidence of a mass,
coal does not adequately bind heavy metals. Gastric or a structural or vascular lesion on neuroimaging.
lavage is rarely indicated for any overdose, except in Furthermore, the composition of the cerebrospinal
certain cases when patients present within 30 min- fluid ( CSF) must be normal, and any symptoms
utes of overdosing. Ipecac is almost never indicated resulting from the disease must be completely
for any overdose. Hemodialysis is only necessary attributable to papilledema or generalized elevated
when renal failure limits the body's ability to clear ICP. The most common presenting symptom is
chelated iron. headache, which tends to be worse in the recumbent
position and in the morning (after prolonged
~ Answer B. The patient has facial nerve paralysis recumbency overnight). CT scan never reveals
from ipsilateral otitis media. The facial nerve is hydrocephalus. Abducens nerve palsy is the only
the most commonly affected cranial nerve in otic cranial nerve palsy that commonly occurs and
pathology due to its physical proximity to the typically presents as intermittent or constant lateral
middle ear. Paralysis of the whole face (upper and binocular diplopia. Women are more commonly
lower) indicates a peripheral rather than a central affected, with a 19 times increased incidence in obese
lesion. Treatment of facial nerve paralysis due to women of childbearing age.
acute otitis media involves intravenous antibiotics,
myringotomy to allow drainage, and tympanostomy ~ Answer A. Oxygen is the standard of care for clus-
tube placement. ter headache. Seventy-five percent of patients with
cluster headache given 100% oxygen through face
~ Answer C. The patient has mitral stenosis (MS)- mask will experience complete or near-complete re-
he has a diastolic murmur (either aortic regurgitation lief within 15 minutes. Because attacks of cluster
or MS) which is loudest at the apex (mitral). The headaches are self-limited and typically last no longer
number one cause of mitral stenosis by far is than 90 minutes, patients may not have pain by the
rheumatic heart disease. Symptoms include those time they reach theED. Therefore, oxygen is an in-
associated with heart failure (dyspnea on exertion convenienttherapy.Sumatriptanisthernosteffective
and orthopnea), due to left atrial hypertrophy and self-administered medication. It is administered as
left heart failure, which eventually proceeds to right a nasal spray, and is effective in more than 50% of
heart failure. The most common complication ofMS patients within 15 minutes of use. However, its use is
92 1(}()() Questions to Help You Pass the Emergency Medicine Boards

not recommended in patients who are having more of diverticulitis) is a high-fiber diet. Barium enema is
than two attacks per day because this would result in the study of choice for making a diagnosis, although
an overdose of the medication. Dihydroergotamine it should be avoided in the setting of diverticulitis,
and zolmitriptan are also effective treatments. Nar- because perforation may occur, allowing leakage of
cotics, benzodiazepines, and corticosteroids have no barium into the peritoneum.
role in acute cluster headache management.
[!Z) Answer D. Diaphoresis with chest pain is an omi-
~ Answer D. Patients with midline penetrating in- nous clinical finding that should increase suspicion
juries to the neck and chest are at risk for a variety for acute coronary syndrome. Dyspnea and nausea
of injuries, including esophageal perforation. Symp- are other, less specific symptoms associated with my-
toms of esophageal injury include pleuritic chest ocardial ischemia. Fever is uncommon, as is back
pain and odynophagia. Subcutaneous emphysema pain, and should point to other causes of chest pain.
may be present, but is neither sensitive nor specific Radiation to one or both arms increases the likeli-
for the diagnosis. Esophagoscopy and esophagogram hood of cardiac cause-radiation to the right upper
by themselves are not sensitive enough, but together extremity may be even more specific than to the
improve accuracy for the diagnosis considerably. left side.
Bronchoscopy is useful for the significant proportion
ofpatients who have concomitant tracheal injury, but ~ Answer A. L. pneumophila is an important cause of
has no role in diagnosing esophageal injury. Chest severe community acquired pneumonia. In patients
CT may offer indirect signs of esophageal injury with more severe symptoms due to community-
such as pneumothorax or pneumomediastinum but acquired pneumonia (CAP), the percentage of
cannot quantify the location or extent of esophageal Legionella spp. isolates increases. Epidemiologic stud-
perforation. ies have exposed links oflegionellosis to exposure to
contaminated water sources, such as air-conditioning
~ Answer B. Initial radiographs ofpediatric patients units and cooling towers. Furthermore, older patients
with elbow trauma may not clearly demonstrate with a history ofalcoholism, tobacco use, and COPD
a fracture. The posterior fat pad, which is closely as well as patients on immunosuppressive therapy
applied to the posterior portion of the humerus, appear to be at higher risk. Although the role of
is normally invisible in the intact elbow. Blood in Legionella spp. is still being defined in the setting
the elbow joint due to fracture will cause this fat of more benign illness, legionellosis is classically de-
pad to show up as a dark line just posterior to the scribed as a severe infection, associated with high
supracondylar region of the humerus. The anterior fevers, a dry cough which may turn productive late in
fat pad may be present in the intact elbow as a the course, pleuritic chest pain, and prominent gas-
dark line, but enlargement of this into a sail shape trointestinal symptoms including abdominal cramps
indicates likely injury. On an anteroposterior (AP) and diarrhea. However, several studies have demon-
view, the Baumann's angle is formed by a line parallel strated that the clinical and radiographic features
to the capitellar growth plate and a line parallel to of the disease are nonspecific. Therefore, to make
the long axis of the humerus. A normal Baumann's the diagnosis, laboratory testing is required. Urinary
angle is 75 degrees, and bilaterally equal Baumann's antigen testing is the test of choice as it is both highly
angles reduce the likelihood of unilateral fracture. sensitive (>90%) and specific (>99%), and is also
On a lateral view of the normal elbow, the anterior quite rapid. Although antibiotic therapy should be
humeral line should bisect the capitellum. With a started empirically before such results are obtained,
supracondylar fracture, the anterior humeral line lies urine testing can be initiated in the ED to guide
anterior to the midpoint of the capitellum. the patient's further therapy and to provide useful
information about prognosis.
~ Answer E. Diverticula are most common in the
left colon, particularly the sigmoid colon, amongst [!!) Answer B. The recurrence rate after a single
patients from the Western world. Japanese patients episode of uncomplicated diverticulitis is 20% to
most commonly have diverticula in the right colon. 30%. Furthermore, a high-fiber diet may help prevent
Even among Japanese who have immigrated to the further episodes. Young patients with diverticulitis
United States and who consume a low-fiber, high- are a special population because they tend to have
fat Western diet, diverticula are typically limited more aggressive disease and the recurrence rate
to the right colon. However, such patients have of diverticulitis is higher. Although resection of
a significantly higher incidence of disease than the disease segment remains elective, some authors
their counterparts in Japan. The most common recommend resection in all young patients due to
treatment for diverticular disease (in the absence their higher risk of recurrence. CT is the study of
Test3 93

choice in the ED because it reliably visualizes the considerably. As in all patients, airway should be
site of inflammation and is very useful for detecting assessed first, then breathing, then circulation. The
various complications (e.g., abscesses, perforation, only exception to this rule is when patients exposed
fistulas). The mortality rate of hospitalized patients an environmental toxin such as organophosphates
with diverticulitis is 1% to 6% for those requiring must be decontaminated even before airway assess-
only medical management and 12% to 18% for ment to prevent exposure to health care workers.
those requiring surgery. Diverticulitis is the most Most poisoned patients need only good supportive
common cause of a colovesical fistula. Furthermore, care for management-a specific antidote is helpful
colovesical fistulas are the most common ofall fistulas in only a minority of cases. Activated charcoal and
complicating diverticulitis. Such patients frequently whole bowel irrigation, not ipecac, are the mainstays
present with findings of a urinary tract infection in of gastrointestinal decontamination, although inhi-
addition to findings of diverticulitis. bition of absorption with these agents has not been
conclusively shown to improve clinical outcomes.
[100] Answer C. Diagnosing the poisoned patient can Hospital urine drug screens detect several drugs of
be extremely difficult, but identifying a specific tox- abuse, but these represent only a fraction of common
idrome can speed up evaluation and management toxins.
Test 4
Questions
(I) Which of the following is the most common cawe of (C) She should receive total parenteral nutrition for
death in children in the United State&? 72 hours.
(D) She will be allowed to eat once abdominal
(A) Infection
computed tomography (cr) reveals resolution
(B) Malignancy
of the signs of pancreatitis.
(C) Trauma
(E) She may eat a low calorie, carbohydrate-rich
(D) Congenital abnormality
diet.
(B) Stroke

(!) Common features ofatopic dermatitis include which


[!) In setting of a normal peripheral white blood c:ell
(WBC) count and a suspected "traumatic'' lumbar
of the following?
puncture, the cerebrospinal fluid (CSF) should
(A) Pruritw contain appromnately 1 WBC per every - - red
(B) Flexor surface involvement in adulta blood cells (RBCs)?
(C) Facial involvement amongst infanta
(A) 1
(D) Frequent involvement of the bands in
(B) 100
adulthood (hand dermatitis)
(C) 300
(E) All of the above
(D) 700
(E) 1,000
[!) Which of the following is true regarding pediatric
community-acquired pneumonia (CAP)?
(!] A 6-year-old girl is brought to the emergency room
(A) The most common cause of pneumonia in the 4 hours after developing a brief choking episode
neonate is Mycoplasttua pneumoniae. while playing with her toys. Her chest x-ray is
(B) The incidence of CAP in children younger than shown in Figure 4-1. Where is the foreign body
five years old is higher than in middle-aged locattdl
adults.
(A) Eaophagw
(C) lt is ea&ier to clliferentiate between typical and
(B) Hypopharynx
atypical pneumonia in pediatric patienta.
(C) Trachea
(D) Streptococcus pneumonilu is the most
commODly isolated organism in children aged 5
to IS yean.
(B) The presence of rhinorrhea. myalgias, or a
concomitant illness in a family member is more
common in viral pneumonJa.

[!) A 42-year-old woman presented to the emugency


department (ED) with acute-onset epigastric ab-
dominal pain and n.a.usea without vomiting 15 hours
earlier. Her workup revealed acute pancreatitis and
she wu admitted. After .receiving appropriate anal-
gesia and antiemetics, she is now hungry and wants
something to eat. Which of the following is true?
(A) She should undergo a period of bowel rest for
48 hours regardless of her laboratory results.
(B) She requires ongoing nasogastric (NG)
auctioning until pancreatic enzyme
abnormalities resolve.
94
Test4 95

(D) Anterior mediastinum (A) Hypocalcemia


(E) Not possible to determine from the (B) Hypokalemia
information provided (C) Microcytic anemia
(D) Thrombocytopenia
(!) A 4-year-old boy is brought to the ED with a severe (E) Hypermagnesernia
sore throat and a history of "refusing to eat." He
has a severe pharyngitis on examination, but a lateral (!j) A 42-year-old previously healthy woman presents
neck x-ray is taken that you feel is consistent with a with a "bad" sore throat and painful swallowing. She
retropharyngeal abscess (RPA). You are surprised to is febrile, but nontoxic and in no respiratory dist:reas.
find. however, that the patient's subsequent CT was A lateral soft tissue neck film is ordered and is shown
normal. The radiologist tells you thia was probably in Figure 4-2. Which of the following is the cause of
due to poor technique. What technique should be this patient's illness?
used to most accurately assess the prevertebral space
onx-rayf
(A) X-ray should be taken in 11mon durlns
expiration.
(B) The patient should be sitting upright when the
x-ray is taken.
(C) The x-ray should be taken in 11exion during
inspiration.
(D) The x-ray should be taken in extension during
expiration.
(E) The x-ray should be taken in atension and
inspiration.

(!] Which of the following is the most important &ctor


in determining the chance ofspontaneous passage of
a kidney stoner
(A) Composition of the stone
(B) Size of the stone
(C) Degree of pain
(D) Degree of nausea
(E) Age of the patient

(!] Which of the following is a manifestation of


hypocaicemiar
(A) QTc shortening Figure 4-2.
(B) Polyuria
(C) Perioral paresthesias (A) Retropharyngeal abscess (RPA)
(D) Nephrolithiasis (B) Epiglottitis
(E) None of the above (C) Peritonsillar abscess (PTA)
(D) Bacterial tracheitis
~ A nuogastric tube (NGT) should be (E) Ludwig angina
(A) Placed in all patients with a complaint of
gastrointestinal (GI) bleeding. {!jJ A 47-year-old man without significant past medical
(B) Avoided in all patients with a complaint ofGI history presents with a chief complaint of burning
bleeding. epigastric abdominal pain. It does not radiate and
(C) Placed only in patients who have ongoing occasionally wakes him up in the middle ofthe night.
hematemesis. He is on no medicines and denies drinking alcohol.
(D) Placed in patients who have rectal bleeding of You suspect he may have peptic ulcer disease and
uncertain origin. want to test him for the presence of Helicobacter
(E) Avoided in patients with esophageal varices. pylori. Which of the following is the best initial
screening method?
(!j) Which of the following is most characteristic of (A) Serologic testing (IgG antibody testing)
ethylene glycol overdoses? (B) 13c or 14c urea breath testing
96 1000 Qvaticms ftl Hdp You Ptw the Emerfmcy Metlit:W BoMds

(C) Referral for endoscopy and biopsy Which of the following is the most appropriate next
(D) Stool antigen test step in management?
(E) Rapid ureaae test (A) Admit to the intensive care unit (ICU)
(B) Discharge home
~ Which of the following is true regarding malarial (C) Di.sc.harge home with pain medication
(A) The causative organism is a parasite. (D) Discharge home with pain medication and
(B) The vector is the male Anopheles .tn05q1lito. ina::ntive spirometer
(C) Human-to-human transmission may occur (E) Di.sc.harge home with pain medication,
through saliva. ina::ntive spirometer, and anu'biotics
(D) Viwa malaria is the most severe variety.
(E) Blackwater fever is wually cawed by (!!] A 22-year-old woman presents to the ED with
O'Vtlh malaria. symptoms of enreme panic. She tells you that she
jwt used lysergic add diethylamine (LSD) fur the
~ A 42-year-old man with a history of alcohol abuse first time. Which of the foll.owing is true regarding
presents with acute epigast::ric abdominal pain. His this patient?
workup ~ acute pancreatitis and he denies any (A) Lorazepam may be wed to reduce agitation.
prior history of pancreatitis. A Cf scan that was (B) The patient is likely to get addicted to I.SD.
ordered aa part of his workup is shown in Figure 4-3. (C) LSD is structurally and functionally similar to
Which of the foRowing is true? y-aminobutyric add (GABA).
(D) The lethal dose of lSD is very close to the
typical dose taken to induce a "trip."
(E) The patient ia unlikely to develop tolerance
with repeated wes of lSD.

(jjJ "Hard" findings of vascular injury mandating im-


mediate angiography after trauma to m atremity
include
(A) Nonpulsatile hematoma.
(B) Palpable thrill.
(C) &sodated peripheral nerve deficit.
(D) Diminiahed distal pulae.
(E) All of the above.

Figure 4-S.
(!?] Which ofthe following effects is directly responsible
fur the QRS prolongation seen in tricyclic antide-
pressant poisoning?
(A) Past sodium channel blockade
(A) The CT is indicative of chronic pancreatitis.
(B) Potassium efl:lux blockade
(B) An urgent surgical COtUult is required for
(C) "-1 antagoo..ism
drainage.
(D) Anticholinergic activity
(C) The patient should be given broad-spectrum
(E) Magnesium channel blockade
anu'biotic therapy.
(D) This finding increases his mortality 10-fold.
(E) This CT finding is present in up to half the
(igJ A 65-year-old man with multiple myeloma presents
with generalized weakness and fatigue. His physical
number of patients with acute pancreatitis and
examination ia unremarkable except for severe
normally resolves without intervention.
lassitude. Serum calcium level ia 14 mg per dL.
Which of the following ia the most appropriate next
~ A 76-year-old woman presents after a fall from
step in managemenrl
!tanding he.ight onto a countertop. She lands on the
right side of her ribcage and complains of pain in that (A) Magnesium sulfate
area and diBiculty taking a deep breath. Vital signs (B) Potassium phosphate
are normal and physical examination is remarkable (C) Normalsaline
only for point tendemeu in the right lateral fourth (D) Vitamin D
and fifth rib&. A chest x-ray done in the ED is normal. (E) Hydrochlorothiazide
Test4 97

~ Cancer of which of the following organs is the and chest tightness typical of her asthma. She had
most common cause of superior vena cava (SVC) recently run out of her medicines and complains of
syndrome? upper respiratory symptoms for the last one week.
Accompanying her is her 6-year-old son and 72-year-
(A) Breast
old mother, both of whom have asthma. Which of
(B) Lung
the following summarizes the treatment differences
(C) Testicle
of acute asthma exacerbations between these groups?
(D) Colon
(E) Thyroid (A) Corticosteroids are avoided in pediatric
populationsbecauseofconcernsabouttherr
~ The absence of which of the following has almost effects on growth.
100% sensitivity.... (B) The treatment of children with acute asthma
exacerbations is similar to the treatment of
(A) Scrotal hematoma
adults and includes 13-agonists,
(B) Penile hematoma
anticholinergics, and corticosteroids.
(C) Pelvic fracture
(C) Cromolyn sodium has a prominent role in the
(D) Gross hematuria
treatment of acute exacerbations of pediatric
(E) Rectal blood
but not adult asthma.
(D) The first-line agent in treating elderly patients
~ A 44-year-old man presents after a motor vehicle
with acute asthma is ipratropium due to the
crash (MVC) with scrotal pain. Blood was noticed
high likelihood of underlying coronary artery
initially at the urethral meatus, but a 16-Fr Foley
disease and subsequent risk with J:l-agonist
catheter was mistakenly placed with return of yellow
induced tachycardia.
urine. Which ofthe following is the most appropriate
(E) Leukotriene modifiers have recently been
next step in management?
shown to be useful in acute asthma
(A) Remove the 16-Fr catheter and place a 12 Fr exacerbations in elderly but not pediatric or
Foley catheter. young adult patients.
(B) Remove the 16-Fr catheter and place a 12 Fr
Coude catheter. ~ The most common electrocardiographic (EKG)
(C) Remove the 16-Fr catheter and perform abnormality in patients with heatstroke is
retrograde urethrogram.
(A) Sinus bradycardia.
(D) Remove the 16-Fr catheter and perform
(B) Atrial fibrillation.
retrograde cystogram.
(C) QT interval prolongation.
(E) Leave the catheter in place and obtain urologic
(D) Ventricular fibrillation.
consultation.
(E) Supraventricular tachycardia (SVT).
~ Which of the following therapies~ the fast~t onset
~ A 55-year-old man with diabetes presents with
of action in reducing serum potassaum levels m cases
painful vision loss in his left eye, which occurred
of hyperkalemia?
when he sat down to watch a movie in the theater.
(A) Calcium gluconate His acuity is markedly reduced in the left eye and
(B) Calcium chloride his left pupil is poorly reactive to light and fixed at
(C) Insulin and glucose 4 mm. Which of the following is true regarding this
(D) Sodium polystyrene sulfonate (Kayexalate) patient's condition?
(E) Sodium bicarbonate
(A) A unilateral shallow anterior chamber is
diagnostic.
~ Rectal prolapse in children ...
(B) Retinal venules demonstrate a characteristic
(A) Is more common in girls. boxcar appearance.
(B) Usually occurs between the ages of 10 and 15. (C) Pilocarpine is typically administered to both
(C) Usually involves all the layers of the bowel. eyes.
(D) May be a sign of underlying cystic fibrosis (CF). (D) Intravenous therapies are withheld until
(E) Should never be reduced in the ED. ophthalmologic evaluation is obtained.
(E) Ocular massage is a helpful temporizing
~ A 38-year-old woman with a history of asthma measure.
presents to the ED with a chief complaint ofwheezing
98 1(}()() Questions to Help You Pass the Emergency Medicine Boards

~ Which ofthe following is true regarding psychogenic ~ The most reliable early indicator of shock in a
seizures? pregnant patient after blunt abdominal trauma is
(A) Self-injurious behaviors such as urinary (A) Hypotension.
incontinence and tongue biting may occur. (B) Elevated lactate.
(B) More than 50% of patients with psychogenic (C) Tachycardia.
seizures may have a concomitant real seizure (D) Peritoneal signs on examination.
disorder. (E) Cool, clammy skin.
(C) When turning the head of a seizing patient
back and forth, patients with psychogenic ~ A 76-year-old woman presents to your ED in the
seizures will avoid looking at the examiner. middle of summer complaining of swelling of her
(D) Patients with psychogenic seizures may ankles and feet. The daily high temperature has
demonstrate pelvic thrusting motions during exceeded 100°F for the last 10 days. Which of the
their "seizures." following is true?
(E) All of the above. (A) She should be treated with furosemide.
(B) Her condition may resolve with acclimation.
~ Which ofthe following is the most common cause of (C) She should be treated with
death among nursing home residents? hydrochlorothiazide.
(A) Congestive heart failure (D) An echocardiogram should be performed to
(B) Pneumonia exclude heat-induced congestive heart failure.
(C) Urosepsis (E) Her condition is actually most common in the
(D) Massive stroke pediatric population.
(E) Myocardial infarction
~ A 57-year-old woman with a history of hyperten-
~ A 34-year-old woman presents with weakness, fa- sion presents with headache. She describes acutely
tigue, rash, and fever for several days. She was worsening global headache over the last 4 hours with
prescribed an antibiotic by her primary care physi- nausea and vomiting. She admits to not taking any of
cian for an upper respiratory infection just before the her blood pressure medications for the last week. Vi-
symptoms started. The rash is diffuse, maculopapu- tal signs are: T 98.4, HR. 92, BP 220/130, RR 20, Sp02
lar, and confluent. Laboratory work demonstrates 97%. Examination reveals a patient in moderate dis-
normal electrolytes, but an elevated creatinine at comfort, papilledema, hypertensive retinopathy, and
3.6 mg per dL. Peripheral blood and urine contain a nonfocal neurologic examination. Laboratory stud-
eosinophils. Which ofthe following is the most likely ies, EKG, and noncontrast CT brain are all normal.
etiologic agent? Which of the following is the most appropriate next
(A) Amoxicillin step in management?
(B) Doxycycline (A) Neurosurgical consultation
(C) Clindamycin (B) Reduction of blood pressure by 25%
(D) Azithromycin (C) Lumbar puncture
(E) Erythromycin (D) Corticosteroids
(E) Noncontrast magnetic resonance imaging
~ A 47-year-old man is brought to the ED by ambulance (MRI) of the brain
after a motor vehicle accident. He was a restrained
driver of a vehicle traveling 40 mph when he collided @!) Which of the following is true regarding the
with a vehicle that turned in front of him. He management of epistaxis?
vigorously "slammed" the brakes and locked his (A) Silver nitrate sticks work best when activated
knee in an effort to stop the car. He is now unable to by fresh bleeding.
ambulate and he has an obviously deformed ankle. (B) Silver nitrate sticks should not be used on
Initial x-rays reveal a pilon fracture. Which of the bilateral surfaces of the septum.
following structures might also be injured? (C) Avoid blowing the nose before placement of
(A) Calcaneus packing.
(B) Femoral neck (D) Antibiotics are indicated in patients with
(C) Lumbar spine posterior packs, but not anterior packs.
(D) Tibial plateau (E) Admission is indicated in patients with anterior
(E) All of the above packs, but not posterior packs.
Test4 99

~ Which of the following is true regarding travder's di- (B) Homocystinuria


arrhea? (C) Marfan syndrome
(D) Trauma
(A) Prophylactic antibiotics are effective in
(E) Ehler-Danlos syndrome
reducing the incidence of traveler's diarrhea.
(B) Campywbacter has begun to develop
~ Which of the following is true regarding treatment
widespread resistance to fluoroquinolone
of acute lead toxicity?
therapy.
(C) Routine antibiotic prophylaxis is not (A) Acute lead encephalopathy is generally
recommended for all travders. self-limited and requires no specific therapy.
(D) Bismuth subsalicylate (Pepto-Bismol) taken (B) Dimercaprol should be given before calcium
four times daily reduces the incidence of disodium ethylenediamine tetraacetic
travder's diarrhea. acid (EDTA).
(E) All of the above. (C) Activated charcoal is the mainstay of GI
decontamination.
~ Which of the following is true about Pneumocystis (D) Succimer should be the first chelator given in
c:arinii pneumonia (PCP)? patients with severe lead poisoning.
(E) Penicillamine is more effective than succimer
(A) Typical radiographic findings include bilateral
in chdation oflead.
lobar infiltrates, more commonly in the lower
lobes.
~ Which of the following is the most reliable physical
(B) Among patients with acquired immune
examination test for evaluating thoracic outlet
deficiency syndrome (AIDS), PCP, and
syndrome?
respiratory failure requiring ventilation,
patients who have been compliant with (A) Arms abducted to 90 degrees, elbows flexed to
prophylactic trimethoprim-sulfamethoxazole 90 degrees, open and close fists for 3 minutes
(TMP-SMX) therapy before being diagnosed (B) Arms abducted to 90 degrees, thumbs pointed
with PCP have a better outcome. down, resist adduction
(C) Patients who have PCP without AIDS usually (C) Dorsa of hands held against each other for
present with a more abrupt course of 1 minute
respiratory difficulty. (D) Repeated tapping of the volar wrist
(D) Corticosteroids are ofbenefit in treating all (E) Pressure on the infraclavicular region for
patients with PCP, regardless of the severity of 1 minute
the illness.
(E) All patients with PCP require respiratory ~ A 24-year-old woman presents with dysuria and
isolation. increased frequency of urination for 2 days. She
denies fevers, vomiting, or back pain. She is allergic
~ Which of the following is the most common to sulfa drugs and fluoroquinolones. Urinalysis
complication of diverticulosis? demonstrates 25 WBCs per high-powered fidd,
leukocyte esterase, and nitrites. Which of the
(A) Perforation
following is the most appropriate antibiotic regimen?
(B) Bleeding
(C) Obstruction (A) Ciprofloxacin
(D) Diverticulitis (B) TMP-SMX
(E) None of the above (C) Doxycycline
(D) Azithromycin
~ Injury to which of the following is the most common (E) Amoxicillin-davulinic acid
cause of traumatic death in children?
~ Which of the following is true regarding prosthetic
(A) Head
heart valves?
(B) Chest
(C) Abdomen (A) Bioprosthetic (porcine) valves require
(D) Pdvis anticoagulation therapy with Coumadin.
(E) Femur (B) Prophylactic anticoagulation should be
maintained at international normalized ratios
~ Which of the following conditions is the most (INRs) between 1 and 2.
common cause of lens dislocation? (C) Chronic hemolysis occurs in most patients.
(A) Tertiary syphilis
100 1000 Questions to Help You Pass the Emergency Medicine Boards

(D) In patients with mechanical valves, clindamycin, which was prescribed by his primary
auscultation of a metallic closure sound care doctor after she performed an incision and
indicates serious valvular dysfunction. drainage of a small cutaneous abscess on his flank.
(E) Endocarditis develops in most patients. The patient's symptoms started toward the end
of his antibiotic therapy and he has been taking
~ Which of the following is true regarding pericardia! diphenoxylate "around the clock" since then without
effusion? much benefit. Which of the following is true?
(A) As little as 50 mL of pericardia! fluid can cause (A) Stool culture is the gold standard.
abnormalities on the cardiac shadow on chest (B) The patient's diarrhea is an expected, minor
x-ray. side effect of his recent antibiotic therapy.
(B) Beck's triad is seen in less than half the number (C) Diphenoxylate is probably contributing to this
of patients with signs of tamponade. patient's current illness.
(C) MRI is the diagnostic test of choice. (D) Colonoscopy is usually required for effective
(D) Blind pericardiocentesis is the treatment of treatment.
choice for stable pericardia! effusions. (E) Children affected by this illness tend to have
(E) Electrical alternans is the most common EKG milder disease.
abnormality.
~ Cholelithiasis is an uncommon disease entity in
~ Which of the following is true regarding Ludwig's children. Which of the following is most commonly
angina? associated with biliary colic in children?
(A) Endotracheal intubation is the preferred (A) Cystic fibrosis (CF)
method of airway controL (B) Hemolytic anemia
(B) The mortality rate ofLudwig angina is (C) Obesity
approximately 75%. (D) Diabetes
(C) Ludwig angina may occur in children without (E) Cerebral palsy
any preceding cause.
(D) Extension to the retropharyngeal space is the ~ A 50-year-old man with a history of end-stage renal
most common cause of death. disease on peritoneal dialysis presents with abdom-
(E) In patients with an associated oral malignancy, inal pain. You suspect peritonitis as a cause of his
radiation is the therapy of choice. pain. Which of the following is the minimum dial-
ysis fluid WBC which would make the diagnosis of
~ Which of the following is a risk factor for death in continuous ambulatory peritoneal dialysis (CAPD)-
patients with asthma? associated peritonitis?
(A) Usage of more than two canisters of albuterol (A) 20 per rnm3
per month. (B) 50 per rnrn3
(B) History of prior intubation secondary to (C) 100 per mm3
asthma. (D) 200 per mm3
(C) History of prior ICU admission secondary to (E) 250 per mm3
asthma.
(D) Underappreciation of the severity of an ~ Urine containing crystals suggests ingestion of which
exacerbation by the ED physician. of the following substances?
(E) All of the above are risk factors for death from (A) Ethylene glycol
asthma. (B) Methanol
(C) Isopropanol
~ Which of the following is indicated as supplemental (D) Salicylates
treatment for patients with methanol poisoning? (E) Acetaminophen
(A) Cobalamin
(B) Folate ~ The clinical factor that best differentiates heat stroke
(C) Niacin from heat exhaustion is
(D) Vitamin D (A) Core temperature >102°F.
(E) Vitamin K (B) The presence of anhidrosis.
(C) Elevation ofhepatic transaminases.
~ A 52-year-old diabetic man presents to the ED with (D) Central nervous system (CNS) dysfunction.
fever, crampy abdominal pain, and watery brown (E) History of exertion in a hot environment.
diarrhea. He recently completed a 14-day course of
Tm4 101

~ A 22-year-old woman presents to the ED after a


domestic dispute with a boyfriend in which she
was stabbed in the neck just lateral to her thyroid
cartilage. Which of the following is an indication for
mandatory operative explorationf
(A) Palpable thrill
(B) Subcutaneous emphysema
(C) Violation ofthe platysma
(D) Bruit upon auscultation
(E) All of the above

~ A 1-week-old infant is broughtto the ED with central


cyanosis. Pulse omnetry is 85% on maximal oxygen
therapy, and chest x-ray is shown in Figure 4-4.
Which ofthe following medications may be indicated
in the treatment of this patient?

~ Which ofthe following indicates a likely globe injury


in a patient with an eyelid lacerationt
(A) Verticallaceration through lid lll.II1'JPn
(B) Horizontal laceration extending the length of
the eyelid
(C) Fat protruding from laceration
(D) Lacerations with a large degree of tissue loss
(E) Stellate lacerations

~ A 64-year-old man with a history of hypertension


presents to the ED with a painful rash on the right
side ofhis back spreadins to his trunk (see Fig. 4-6).
Which of the following underlying diseases should
be suspected?
Figure 4-4.

(A) Prostaglandin Et
(B) Albuterol
(C) Indomethacin
(D) Aspirin
(E) R.ibavirin

~ A 45-year-old woman presents with a red eye on


waking. She is completely asymptomatic other than
this. She denies any past medical history and takes
no medications. The eye is shown in the Figure 4-5.
Physical examination is otherwise normal. Which of
the following is the most appropriate next step in
management?
(A) Emergent ophthalmologic consultation
(B) Platelet function assay
(C) Topical antihi&tamines
(D) Topical antibiotics
(E) No specific therapy
Figunt 4-6. (See color insert.)
(A) Chronic lymphocytic leukemia (Cll) Platelets: 15,000 cells per mm'
(B) Human immunodeficiency virus (HIV) Na: 135 mEq per L
(C) A.spl.enia K: 4.4 mEq per L
(D) Rheumatoid arthritis Cl: 100 mEq per L
(E) He u most likely to be healthy. HC03 : 24 mBq per L
Blood urea nitrogen (BUN): 24 mEq per L
~ A 24-year-old man presents with abdominal fullness. Cr: 2.2 mEqper L
He is very n<:rvous, but in no acute distress. Vital signs Glucose: 109 mEq per L
and physical eumination are normal. An obstructive Which of the following is the most appropriate next
radiography series ia ordered. Which of the following step in management?
ia the most appropriate next step in management (see (A) Platelet transfusion
Fig. 4-7)? (B) Hemodialysis
(C) Splenectomy
(D) Plasmaph.eresu
(E) Acyclovir

~ Which patients with a transient ischemic attack (TIA)


and the following associated signs and symptoms are
at lowest .riak for reCWTent TIA or future stroke?
(A) Isolated monocular blindness (amaurosis
fugu:)
(B) Aphasia and new-onset atrial flutter
(C) "Crescendo" TIA (more than three ischemic
events in 72 houn)
(D) Left arm weakness in a patient who is already
taking aspirin therapy
(E) Left arm weakness and &.cial droop in a
diabetic patient that persisted fur 45 minutes

rnJ A 36-year-old pregnant female at 27 weeks' gestation


presents to your coiiUDunity BD after a motor
vehicle accident. The patient was a restrained
passenger (three point restraints) in a vehicle
traveling approximately 30 mph when it collided
figure 4-7. with a stationary vehicle. Air baga deployed, but the
patient self-extricated and was found sitting next to
the car at the scene. She has not had any vaginal
bleeding or leakage, but she is complaining ofpainful
(A) MRI abdomen
contractions. Which of the following is true?
(B) Surgery
(C) Polyethylene glycol (A) Uterine rupture hu occurred.
(D) Nasogastric (NG) aspiration (B) The patient most likely has placental abruption.
(E) E.ndotracheal intubation (C) Uterine contractions usually resolve
spontaneously.
!!!] A 45-year-ol.d woman presents with headache and (D) The patient should be started on a terbutaline
fatigue for 1 week. She reports inttnnittent diffuse infusion and immediately transferred to a
headaches, which occur in different locations in her tertiary care facility.
head and do not follow any predictable pattern. She (E) Digital enmination should be performed to
has also noticed that she feels tired easily on her daily determine ifthere is cervical dilation and the
activities. Physical examination is normal except for patient is in active labor.
temperature of 100.5°F and pale conjunctiYae. She
exhibits no meni:ngismua. Laboratory work is as (HJ A pregnant woman at 37 weeks' gestation is
follows: undergoing tocographic and fetal monitoring in your
BD after a motor vehicle accident (see Fig. 4-8).
WBC: 8,000 cells per mm3
The obstetric nurses bring you a fetal strip for
Hemoglobin: 8.5 g per dL
Test4 103

240 240

210 210

.... 110
.......
1111

12111
- 110

1
.....

10 10

30
Fetal Heart Rate

Ill

Uterine Contractions

Figure 4-8.

interpretation. Which of the following is represented demonstrates tenderness in the epididymis, normal
in the strips? descended testes, and normal bilateral cremasteric
(A) Head compression reflexes. Which of the following is the most likely
(B) Uterine rupture cause of the symptoms?
(C) Umbilical cord compression (A) Chlamydia
(D) Placenta previa (B) Neisseria gonorrhoeae
(E) Uteroplacental insufficiency (C) Escherichia coli
(D) Chemical epididymitis
~ Which ofthe following is true regarding concussions? (E) Testicular torsion
(A) Loss of consciousness is required to meet the
technical definition. [!!) Which of the following is the peak time for rotavirus
(B) CT brain is acutely abnormal in half of all cases. infections?
(C) MRI brain is almost always abnormal acutely. (A) Spring
(D) Postconcussive anosmia may be permanent. (B) Summer
(E) Athletes should return to play as soon as (C) Spring and summer
possible after concussion to prevent brain (D) Spring and winter
atrophy. (E) Summer and winter

~ The most common bacterial cause of diarrhea in [!!) Which of the following correctly matches the clinical
patients seeking medical attention is entity and its effects on phosphate metabolism?
(A) Campylobacter spp. (A) Rh.abdomyolysis causes hypophosphatemia.
(B) Salmonella spp. (B) Respiratory alkalosis causes
(C) ShigeUa spp. hyperphosphatemia.
(D) Escherichia spp. (C) Hyperparathyroidism causes
(E) Yersinia spp. hyperphosphatemia.
(D) Chronic renal insufficiency causes
~ A 45-year-old man presents with progressive scrotal hypophosphatemia.
pain for 3 days. He denies any swelling in the (E) Treatment of diabetic ketoacidosis (DKA)
area, but reports mild dysuria. Physical examination causes hypophosphatemia.
104 1000 Quatioru to Htlp You Ptw tile~ MetliciM BoGrr1s

~ What is the approodmate sensitivity and specificity of (A) Spleen


themonospottestfor EBV inkctiousmononucl.eosis? (B) Kidneys
(C) Liver
(A) 50%,50%
(D) Diaphragm
(B) 80%,80%
(E) Pancreas
(C) 95%,80%
(D) 80%,95%
(B) 95%,95% (!i) A 25-ycar-old GtPo presents at 18 weeks' gestation
with a chief complaint of painless vaginal discharge
(see Fig. 4-9). Her pregnancy has progressed normally
(nJ A 56-year-old man with a history of a prosthetic
to date. Speculum examination revea.la an adherent
aortic valve preaents for evaluation offever and chills
whitish discharge with a pH of 6.0. There is no
for 1 -mek. Echocardiogram reveals vegetations on
one available to interpret the microscopy so the
the prosthetic valve. Which of the following is the
laboratory asks you to look at it. Which of the
moat likely etiologic organism? 4'..11......:~n • ?
~......, ... ~IS true.
(A) Coagu]ase-negative Staphylococcus
(B) Streptoax:cw viridtms
(C) Ha4mophilus influenzae
(D) Pseudomonas tU!TUginosR
(B) Klebsiel14 pneumoniae

~ Among children, which of the following is the most


rommon cause of pancreatitis?
(A) Biliary tract disease
(B) Trauma
(C) Alcohol
(D) Coxsackievirus .in.fcction
(B) Erythromycin toxicity

~ Which of the following is the most oo.mmon


symptom in patients with spinal oord oompression? Figure 4-9.
(A) Urinary retention
(B) Saddle aneathesia
(C) Motor wealc:ness (A) Metronidazole is the treatment of choice.
(D) Bowel incontinence (B) Amoxicimn should be used due to the
(E) Aacendingparesthesias patient's pregnancy.
(C) No treatment is required until the third
~ Which of the following neurologic findings is trimester, after organogenesis has occurred.
characteristic of tick paralysis? (D) Treabnent is dective, as the infection poses no
(A) Cranial nerve palsy health risks to the mother or fetus.
(B) Descending flaccid paralysis (E) None of the above.
(C) Ascending flaccid paralysis
(D) Decreased pain and temperature sensation (!!] A 65-year-old woman presents with lightheaded.nes.s.
(E) Decreased vibratory and position sensation She denies chest pain or shortness of breath. Vital
signs are 99.0°F, 160, 20, 144/75, 96% RA. The EKG
~A -42-year-old man is brought into the ED after be- is ahown in Figure -4-10. Which of the following is
the most appropriate next step in management?
ins thrown from a train during a derailment. He is
hypotensive but has no hemothorax. a normal medi- (A) Adenosine
astinum and no pelvic fracture. Focused assessment (B) Diltiazem
of sonography in trauma (FAST) scanning reveals (C) Amiodarone
hemoperitoneum. Which of the following is moat (D) Cardioversion at 50 J
likely injured? (E) Cardioversion at 200 J
TII!St4 105

Figure 4-10. (See coklr insert.)

(1j) Which ofthe following sleeping positions is the best (D) Tbe water seal chamber does not have enough
method to reduce the risk of sudden infant death water in it.
syndrome (SIDS)? (E) There is still an unhealed defect in the patient's
(A} Prone bronchial tree.
(B) Supine
(C) Side
r!!) Which of the following is true regarding irritable
bowel syndrome (IBS)?
(D) Standing
(E) Head down (A) It is more couunon in men than in women.
(B) Pain associated with ms is usually reliewd with
(!!) Which of the fullowing may be a cause of central defecation.
vertigof (C) IBS is a psychiatric diagnosis.
(D) IBS is most commonly caused by unrecognized
(A) Basilar artery migraine
food allergies.
(B) Vertebrobasilar artery iluuffi.ciency
(E) All of the above.
(C) Multiple sclero&is
(D) Alcoholic cerebellar degeneration
(E) All of the above
~ Which of the following is the gold standard for
diagnosing choledocholithiasis?
~ You are consulting the thoracic surgery service after (A) CT scan of the abdomen with intravenous and
having recently placed a chest tube in a 22-year-old oral contrast
man for a 25% spontaneous pneumothorax. Tbe (B) Ultrasonography (US)
consultant uks you if there is still an "air leak" (C) Chol.eacintigraphy (e.g., HIDA scan)
present. Assuming the patient's lung re-expanded (D) Endoscopic retrograde
appropriately, what is the most likely significance of cholangiopancreatography (ERCP)
an air leakl (E) CT scan of the abdomen with intravenous
cont:rut only
(A} All patients will have an air leak after chest tube
insertion.
(B) The chest tube was inserted into a branch of
~ The primary risk fActor for uterine rupture amongst
pregnant women is
the patient's tracheobronchlal tree.
(C) The suction holes on the chest tube are not (A) Vaginal delivery after a prior cesarian-section.
completely inserted into the chest cavity. (B) Diabetes.
106 1000 Questions to Help You Pass the Emergency Medicine Boards

(C) Large for gestational age fetus. ~ A 32-year-old man presents to the ED with headache
(D) Prolonged use of oxytocin. and fever for 2 days. He also reports a stiff neck and
(E) Maternal collagen vascular disease. photophobia. Past medical history is unremarkable.
Physical examination reveals a febrile patient with
~ A 23-year-old woman presents with fever, chills, and nuchal rigidity, no papilledema, and no focal
right flank pain. She just completed treatment for neurologic deficits. Which of the following is the
pyelonephritis with a 2-week course of ciprofloxacin. most appropriate next step in management?
The patient states that the symptoms are very similar
(A) Antibiotic therapy
to when she had pyelonephritis, and she cannot
(B) Antibiotic therapy with corticosteroids
understand why she did not get better with the
(C) CT brain with N contrast
antibiotics. She admits to having waited "longer than
(D) MRI brain with gadolinium contrast
usual" before seeking care for the pyelonephritis
(E) Discharge home with diagnosis of viral
during the first visit, but swears that she took all the
meningitis
antibiotics as directed. Which of the following is the
most appropriate next step in management?
~ A 4-year-old boy presents with generalized abdom-
(A) Continue ciprofloxacin for 3 more days inal pain and multiple episodes of bloody diarrhea.
(B) Continue ciprofloxacin for 7 more days His mother says he had a hamburger at a local fast
(C) Start TMP-SMX for 3 days food restaurant 5 days ago but that "they eat there all
(D) Start metronidazole for 3 days the time." The patient's illness started 1 day before
(E) CT scan of the abdomen/pelvis seeing you. It began with watery diarrhea and mild
abdominal pain but his symptoms have progressed
~ Which ofthe following oral medications is most likely to more severe pain with bloody diarrhea. He has no
to cause serious damage to the esophageal mucosa fever. Which of the following statements is true?
when swallowed?
(A) TMP-SMX is the treatment of choice
(A) Amoxicillin (B) Up to 25% of children may develop
(B) Potassium chloride hemolytic-uremic syndrome (HUS)
(C) Metoprolol (C) Most affected children have a very high fever
(D) Lovastatin (D) Thrombotic thrombocytopenic purpura is the
(E) Hydrochlorothiazide most common systemic complication
(E) The hamburger is likely unrelated to this
~ A 6-month-old male infant is brought in by his patient's diarrheal illness as the incubation
mother after 4 days of continuous diarrhea. He period is more commonly 12 to 24 hours
appears mildly dehydrated on examination. Which
of the following acid-base disturbances is likely to be ~ A 42-year-old healthy male experiences crampy
present? abdominal pain and persistent, violent recurrent
(A) Elevated anion gap metabolic acidosis retching and vomiting 3 hours after eating a
(B) Normal anion gap metabolic acidosis hamburger, corn and potato salad at a backyard
(C) Metabolic alkalosis picnic. Which of the following is true about his
(D) Respiratory acidosis illness?
(E) Respiratory alkalosis (A) The disease is caused by rapid invasion of the
intestinal mucosa
li!J Which ofthe following is the most specific finding in (B) Antibiotics are instrumental in management
diagnosing necrotizing enterocolitis (NEC)? (C) Most cases are caused by Bacillus cereus
(A) Abdominal radiograph demonstrating multiple (D) A heat-stable toxin is responsible for his
dilated loops of small bowel symptoms
(B) Abdominal radiograph with a "double (E) This syndrome is the most common cause of
bubble" sign acute food poisoning in the United States.
(C) Abdominal radiograph with pneumatosis
intestinalis ~ Which ofthefollowingis true regarding the treatment
(D) The presence of abdominal distension, bilious of peritonsillar abscess (PTAs)?
emesis and guaiac positive stools (A) Emergency tonsillectomy totally prevents
(E) The presence of portal vein thrombosis upon recurrence ofPTAs.
US examination
Test4 107

(B) Recurrent PTAs typically occur > 1 year after His tympanic membranes appear normal bilaterally.
the initial episode. Which of the following is the most common single
(C) Antibiotic therapy is as effective as surgical pathogen implicated in this condition?
incision and drainage. (A) Staphywcoccus aureus
(D) Penicillin is the antibiotic of choice. (B) Staphywcoccus epidermidis
(E) Recurrence ofPTAs is more common in (C) Streptococcus pneumoniae
children than in adults. (D) Streptococcus pyogenes
(E) P. aeruginosa
~ Which of the following is a criterion for liver
transplantation in acetaminophen overdose? ~ Which of the following medications is the most
(A) AST greater than twice normal common cause of pill esophagitis?
(B) pH <7.30 (A) Alendronate
(C) PT prolongation >5 seconds (B) Potassium chloride
(D) Ammonia >40 mg per dL (C) Aspirin
(E) GGT >300 mg per dL (D) Doxycycline
(E) Captopril
~ A 22-year-old primigravida presents to the ED with
crampy low abdominal pain. She is 11 weeks by ~ A 45-year-old man presents after a motor vehicle
dates and denies any vaginal bleeding. Her physical collision with hypotension. His physical examination
examination reveals a normal vaginal vault and a demonstrates clear lungs but an unstable pelvis.
closed internal cervical os. US examination reveals The pelvis is secured tightly with a bed sheet, but
a single intrauterine gestation with a crown-rump the patient continues to be hypotensive despite
length > 20 mm corresponding to 9 weeks, with no crystalloid and blood replacement. No other obvious
fetal heart tones. Persistent failure to expel the fetal source of hemorrhage is identified. Which of the
and maternal uterine contents would result in a following is the most appropriate next step in
(A) Threatened abortion. management?
(B) Incomplete abortion. (A) CTchest
(C) Complete abortion. (B) CT abdomen/pelvis
(D) Missed abortion. (C) CTbrain
(E) Inevitable abortion. (D) Angiography with embolization
(E) Laparotomy
~ A first-time mother brings her 9-day-old infant in
with a chief complaint of seizures. The infant had ~ An 11-year-old boy hit the curb while riding his
an uncomplicated term delivery and has been well. bicycle and was thrown forward into his handlebars.
Her infant has been and remains afebrile. Which The classic injury associated with this accident is
of the following is the most likely cause of this
(A) Myocardial contusion.
infant's seizures?
(B) Pancreatic injury.
(A) Hypokalemia (C) Liver contusion.
(B) Hyponatremia (D) Splenic contusion.
(C) Hypocalcemia (E) Diaphragmatic rupture.
(D) Hypomagnesemia
(E) Maple syrup urine disease ~ Which of the following is characteristic of facial pain
due to trigeminal neuralgia?
~ A 24-year-old man presents with pain behind his
(A) It is most commonly bilateral.
right ear. He thought he had an ear infection
(B) Attacks last for an average of30 minutes.
1 week before, and took two of his girlfriend's
(C) Patients demonstrate partial facial nerve palsy
azithromycin pills without seeking medical attention.
on examination.
His ear pain improved transiently for a few days,
(D) It most commonly involves the V2 and V3
but he began developing otalgia again with fevers.
branches of the trigeminal nerve.
On examination, his temperature is 100.5°F, and
(E) Most patients have concomitant dental disease.
he has significant right postauricular tenderness.
108 1000 Questions to Help You Pass tlu EmeTgfltlcy Medicine Boards

1100] A 78-year-old male nursing home resident is oximetry of92% on room air. What is the most likely
brought to the ED with fever and hypoxia. He has diagnosis and appropriate initial treatment?
a history of dysphagia and dysarthria secondary to (A) Community-acquired pneumonia (CAP),
stroke and receives tube feedings and small spoon levofloxacin, and corticosteroids
feeds. Approximately 1 week before admission, he (B) Aspiration pneumonia, metronidazole
had an episode of vomiting and appeared to choke (C) Aspiration pneumonitis, no antibiotics
on some ofthe regurgitated contents. His chest x-ray (D) Aspiration pneumonia, aztreonam
now shows bilateral lower lobe infiltrates. He has (E) Aspiration pneumonia, piperacillin-
a fever of 101.8°F, a WBC of 16,000 and a pulse tazobactam
Answers and Explanations
[I) Answer C. Trauma accounts for over half of all virus, influenza, adenovirus, rhinovirus) as well
pediatric deaths in this country, with motor vehicle as S. pneumoniae are the most common causes.
collisions as the most common mechanism and head Increasingly, C. pneumoniae is thought to be a
trauma as the most common specific etiology. Child common cause of pneumonia in children aged 5
abuse accounts for up to one third of all traumatic to 15 years, although Mycoplasma remains the chief
deaths in some areas of the country. Malignancy and cause of pneumonia in this group. It is no easier to
congenital abnormalities are the next most common differentiate between typical and atypical pneumonia
causes of death in the United States. Respiratory in a pediatric population than in adults. Studies have
and GI infections are the most common causes of demonstrated that the presence of symptoms that
pediatric death worldwide. Stroke is an extremely may suggest a viral etiology to pneumonia such as
uncommon cause of death in children, but is a rhinorrhea, myalgias, or an illness in a family member
leading cause in adults. do not help to determine the cause of pneumonia.
The use of the pneumococcal conjugate vaccine is
[!] Answer E. Atopic dermatitis is a chronic inflam- likely to drastically reduce the role of S. pneumoniae
matory skin disease that is characterized by intense as a cause of disease.
pruritus and an eczematous rash. There is no classic
lesion in atopic dennatitis. The disease may ini- [!) Answer E. The decision about when to resume
tially manifest itself with erythematous papules or feeding and what patients should be allowed to eat
vesicles, with diffuse erythema and frequently con- is a matter of ongoing controversy. In the past,
tains a weeping exudate. Over time, dryness, scaling, all patients underwent continuous nasogastric (NG)
and lichenification predominate. Although there is suctioning or were kept NPO. Currently, the only
no typical lesion, there are many classic features of indication for NG suctioning is intractable vomiting
atopic dermatitis. Pruritus is the hallmark of the dis- or ileus. Some studies suggest that early enteral nu-
ease and is typically intense. In adults, the disease trition may actually improve outcomes. Laboratory
has a predilection for the flexural creases such as and radiographic evidence of pancreatitis is likely to
the anterior elbow, ankle, and neck as well as the persist until patient discharge so these are not use-
posterior knee. In contrast, in infants the disease is ful guides for resuming feeding. Many authors now
characterized by dry, red, scaly involvement of the recommend that enteral feeds should be started as
cheeks. The chin is also commonly involved and chin soon as a patient is able to tolerate them. Although
inflammation may be more severe due to the added there is a dearth of evidence regarding the composi-
irritation from drooling. Involvement of the hands tion of feeds, it is known that pancreatic secretions
is extremely common in adults and is frequently decrease as carbohydrate composition exceeds 50%
exacerbated by occupational exposures. of the caloric content of the diet. Therefore, it makes
sense to start with a low calorie, carbohydrate-rich
[!) Answer B. With the possible exception of elderly diet, and steadily increase both the caloric and fat
patients older than 75 to 80 years old, the annual content of the diet over a period of days.
incidence of pneumonia in children under five
years old is higher than at any other time of [ID Answer D. To get a more accurate estimate of the
life. M. pneumoniae is the most common cause of actual number of WBCs in the CSF, the following
community-acquired pneumonia (CAP) in "school- formula should be used: ([CSF RBC] x [blood
aged children," 5 to 15 years old as well as in young WBC]/[blood RBC]).
adults. The most common causes of pneumonia
in the neonate, from birth to 3 weeks, are group [!) Answer A. Diagnosis of aspirated foreign bod-
B Streptococcus, gram-negative enterobacteria (e.g., ies relies on plain radiographs. posteroanterior
E. coli), and Listeria monocytogenes. Such infections (PA)/lateral chest x-rays and anteroposterior (AP)
are uncommon, but can be severe when present. and lateral soft tissue neck films are diagnostic in the
Between the ages of 3 weeks and 3 months, case of radiopaque esophageal and tracheal foreign
Chlamydia trachomatis is most common (i.e., not bodies. Esophageal foreign bodies are seen "en face"
Chlamydia pneumoniae) followed by S. pneumoniae. in AP views and on edge in lateral views, whereas
Between 4 months and 4 years, viral pneumonias the opposite is true for tracheal foreign bodies. Fre-
(e.g., respiratory syncytial virus (RSV), parainfluenza quently, both the AP and lateral views will be needed

109
110 1000 Questions to Help You Pass the Emergency Medicine Boards

to determine the exact location of the foreign body. [!9] Answer D. Nasogastric (NG) tubes are useful in
Additionally, the patient may present with ongoing determining the origin of bleeding. In patients who
symptoms that provide further clues to the location of give a history of hematemesis, coffee-ground emesis,
the foreign body. Such symptoms include dysphagia, or melena, placement of an nasogastric tube (NGT)
odynophagia, or regurgitation of food in the set- is generally not necessary and rarely yields useful
ting of esophageal obstruction, or stridor, wheezing, information. Likewise, in patients who give a history
or generalized respiratory distress in the case of of bright red blood-streaked stools, bleeding can be
tracheal foreign bodies. (Figure reprinted with per- presumed to be of rectal origin. However, roughly
mission from Fleisher GR. Atlas ofpediatric emergency 11% of patients with reddish stools or frank rectal
medicine. Lippincott Williams & Wilkins; 2003.) blood have an upper gastrointestinal (UGI) source
of bleeding. Because UGI bleeds tend to be more
[I) Answer E. To avoid obtainingx-rays demonstrat- deadly (especially in cases in which patients present
ing a fulsely widened retropharyngeal space, lateral with reddish stools indicating rapid transit and a
neckx-rays should be taken with full extension (cervi- large volume of blood loss), an NGT is indicated
callordosis should be visible), and during inspiration. in patients with rectal bleeding of uncertain cause
Although there are several criteria or rules, the most to explore the UGI tract for diagnostic purposes.
common rule is that the prevertebral space should The presence of clear bile in the aspirate either
be no wider than the width of the vertebral body indicates that the bleeding has stopped or that it did
behind it. Specifically, the anteroposterior width of not originate from the UGI tract (the mere absence
the prevertebral soft tissue should be :::: 7 mm at of blood is less definitive). There is no evidence
the level ofC2 (from the anteroinferior border) and that placement of an NGT in the setting of known
<14 mm at the level of C6 in kids, and 22 mm at esophageal varices or Mallory-Weiss tears worsens
C6 in adults. Because retropharyngeal abscesses are or triggers hemorrhage.
most common in children younger than 4 years old,
compliance with extension films during inspiration [!!) Answer A. Ethylene glycol is metabolized to gly-
is difficult to achieve. CT scanning can resolve this coaldehyde by alcohol dehydrogenase, and glycoalde-
difficulty although sedation (which carries its own hyde is converted to glycolic acid by aldehyde
risks) may be required. dehydrogenase. Glycolic acid is then converted to
glyoxylic acid, which is converted to oxalic acid. Ox-
[!] Answer B. The large majority of kidney stones alic acid binds calcium and forms calcium oxalate
<5 mm will pass spontaneously without the need for crystals, which can precipitate in the renal tubules,
lithotripsy or surgical extraction. The large majority brain, and lungs, causing necrosis. Approximately
of stones >5 mm will not pass spontaneously. Stones one third of patients with ethylene glycol poisoning
may become stuck in several areas along the urinary have hypocalcemia, which can lead to QT prolon-
tract-renal calyx, ureteropelvic junction, midureter gation and seizures. Hyperkalemia is more common
at the iliac vessels, ureterovesicularjunction, and vesi- than hypokalemia, probably due to the metabolic
cal opening. Urologic management of kidney stones acidosis caused by ethylene glycol poisoning. Ane-
is through extracorporeal shock wave lithotripsy, mia, thrombocytopenia, and hypermagnesemia do
percutaneous nephrolithotomy, or surgical extrac- not usually occur in patients with ethylene glycol
tion. Pain and nausea are potential indications for poisoning.
admission of patients with kidney stones, but are not
independently associated with low passage rate. Age [!ID Answer B. This patient's x-ray demonstrates the
of the patient and composition of the stone are not classic "thumb" or "thumbprint'' sign signifying
directly associated with passage of the stone. a swollen epiglottis. Although nasopharyngoscopy
(NP) is an alternative to diagnose epiglottitis through
[!] Answer C. The most common symptoms of direct visualization, patients with respiratory distress
hypocalcemia are neurologic and generalized irri- are not candidates for NP until a secure airway is
tability such as twitching and paresthesias which may established. Therefore, the lateral soft tissue neck
progress to frank tetany, perioral paresthesia&, as well x-ray will continue to have a role in supporting
as Chvostek's and Trousseau's signs. Chvostek's sign a diagnosis of epiglottitis. H. influenzae is the
is a twitch of the upper lip when the area around the most common organism causing adult epiglottitis.
facial nerve is tapped. Trousseau's sign is carpopedal However, only a minority of throat cultures are
spasm when a blood pressure cuff is inflated over the positive, suggesting a possibly significant role for
arm at greater than systolic blood pressure for longer viruses as an etiology as well. Patients with epiglottitis
than 3 minutes. The other signs and symptoms listed may subsequently develop an epiglottis abscess, in
are all manifestations of hypercalcemia. which case, Streptococcus and Staphylococcus are the
Test4 111

most common isolated species. Although there is H. pywri. Urea breath testing involves ingestion
no age or seasonal prevalence, men and smokers of radiolabeled urea that is then cleaved by the
are most commonly affected. The clinical course urease enzyme present in H. pylori yielding carbon
is variable, as epiglottitis is frequently preceded by dioxide and urea. These products are absorbed into
a prodrome resembling a mild upper respiratory the blood and the radiolabeled carbon dioxide is
infection. In patients who have a brief prodrome excreted by the lungs and detected in expired air. It
with rapid progression of their symptoms, airway is the test of choice for confirmation of eradication
complications are more common. Symptoms of but has the small downside of prolonged, low-level
epiglottitis include fever, dysphagia, odynophagia, radiation exposure in the setting of 14 C, or the need
sore throat, and a muffled voice although fever for additional specialized equipment in the case of
13
may be absent in up to 50% of patients. Patients C. Stool antigen testing is a newer, promising
frequently have throat pain out of proportion to noninvasive test but has yet to establish consistent
their physical findings on examination. This can be results. Obtaining biopsy specimens and performing
an important clue in diagnosis, as epiglottitis should the rapid urease test both require invasive endoscopy
be suspected in any patient in whom you expect to to obtain tissue samples.
find severe tonsillopharyngitis on examination, and
yet find a relatively benign-appearing oropharynx. [M) Answer A. Malaria is caused by the Plasmodium
Up to one third of patients with adult epiglottitis parasite, which is transmitted to humans through
were seen within 48 hours before admission with bites from the female Anopheles mosquito. Plas-
symptoms that were mistaken for another entity, modium falciparum has the capacity to cause severe
usually pharyngitis. On x-ray, an retropharyngeal systemic malaria, cerebral malaria, and blackwater
abscess (RPA) is diagnosed by prominent swelling of fever, a syndrome ofhemoglobinuria associated with
the prevertebral tissues. To avoid artifactual effects as chronic infection. Human-to-human transmission is
a result of redundant tissue, lateral neck films should rare and generally occurs only with blood transfu-
be taken at full extension with deep inspiration. sion or organ transplantation. Malaria is suspected in
The retropharyngeal space will appear erroneously patients who travel to an endemic area and develop
enlarged ifx-rays are taken in expiration and flexion. symptoms offever, chills, headache, abdominal pain,
RPA should be suspected if the prevertebral soft nausea, and myalgias. Episodic fevers at a regular
tissue from the anteroinferior aspect of C2 to the frequency is classic for malaria. Diagnosis involves
border of the tracheal air column is > 7 mm in thick blood smears to identify Plasmodium and thin
children and adults or the same space at the level smears to identify the specific organism. Treatment
of C6 is > 14 mm in children and 22 mm in is with oral quinine plus doxycycline or intravenous
adults. Peritonsillar abscess (PTA) is a diagnosis quinidine.
that is typically made on physical examination. A
CT may be necessary in cases that are unclear but ~ Answer E. The CT scan reveals a hypodense
a lateral neck x-ray generally contributes little to fluid collection surrounding an inflamed pancreas.
the diagnosis. Bacterial tracheitis is uncommon in An acute fluid collection is associated with acute
adults. Lateral neck films may reveal narrowing pancreatitis in 30% to 50% of patients. These fluid
of the tracheal air column below the level of the collections resolve spontaneously in most patients.
glottis, or a ragged posterior tracheal margin. Finally, A pseudocyst is a fluid collection that persists for
patients with Ludwig's angina are usually diagnosed 4 to 6 weeks and becomes encapsulated by a wall
clinically, although lateral neck films will support the of fibrous or granulation tissue. Unless pancreatitis
diagnosis and show prominent submandibular soft is due to an infectious agent, acute infection of
tissue swelling. fluid collections or of necrotic pancreatic tissue is
uncommon. Infection usually occurs within the first
~ Answer A. Serologic tests are cheap, fast, reliable, 2 weeks although abscess formation may not occur
and yield nearly the same sensitivity and specificity until1 month after the acute infection. If an abscess
as biopsy specimens. Serologic testing is useful in does occur, urgent drainage is required. (Figure
those patients who have not had a documented prior reprinted with permission from Harris JH. The
history of H. pylori infection. Therefore, serologic radiowgy of emergency medicine, 4th ed. Lippincott
testing seems reasonable to use in ED patients Williams & Wilkins; 1999:618.)
presenting with symptoms consistent with peptic
ulcer disease. Serologic tests are not useful for ~ Answer D. Elderly patients with even mild blunt
monitoring the success of eradication therapy. Urea chest trauma are at high risk for pulmonary com-
breath testing and stool antigen testing are other plications, including atelectasis, delayed contusion,
noninvasive tests are useful for the diagnosis of infection, and ARDS. Aggressive pain management
112 1000 Questions to Help You Pass tM Emergency Medicine Boards

and pulmonary toilet is mandatory for these pa- ~ Answer C. Hypercalcemia is extremely common
tients to maintain proper lung expansion in order to in cancer patients and is probably the most com-
prevent the complications mentioned earlier. Most mon type of severe metabolic abnormality in this
patients are successfully managed on an outpatient population. Mechanisms include direct bone de-
basis-admission is indicated if the patient already struction with release of calcium into the serum as
has indications of pulmonary injury, such as abnor- well as PTH-like hormones secreted by some tu-
mality on chest x-ray, hypoxia, or severe symptoms. mors. Symptoms of hypercalcemia include fatigue,
Prophylactic antibiotics are not recommended in nausea, vomiting, altered mental status, and abdom-
the absence of clinical and radiographic findings of inal pain. Treatment involves urgently lowering the
infection. serum calcium level with either oral hydration in mild
cases or intravenous saline hydration with optional
[!lJ Answer A. LSD is part of a class of substances furosemide therapy. Bisphosphonates may also be
known as psychedelics, which produces hallucina- used when the calcium level is extremely elevated.
tions, generally without confusion or disorientation. Magnesium sulfate may be given in patients with
Users are usually aware that they are experiencing concomitant hypomagnesemia, but is not indicated
drug-induced hallucinations. Panic and agitation are for solitary hypercalcemia. Although hypokalemia
the most common unintended effects of these sub- commonly accompanies hypercalcemia (and is of-
stances-benzodiazepines are indicated as first-line ten exacerbated by therapies which lower calcium
therapy, along with placement in a low-stimulation levels), phosphate salts should be avoided for hyper-
atmosphere (dark, quiet room). Addiction to LSD calcemia, as they may cause precipitation of calcium
is rare, but tolerance occurs with repeated use. LSD phosphate. If potassium levels are borderline or low,
is structurally and functionally similar to serotonin. potassium chloride should be used for replacement
The lethal dose of LSD far exceeds the typical hal- while calcium levels are being lowered. Vitamin D
lucinogenic dose and lethal overdoses are rare and and hydrochlorothiazide both increase serum cal-
usually due to contaminants, such as phencyclidine cium levels and are contraindicated in patients with
orcoaune. hypercalcemia.

(!j) Answer B. "Hard" signs of vascular injury are [!!) Answer B. SVC syndrome refers to obstruction of
signs that strongly suggest the presence of vascu- the SVC, usually by a neoplastic cause, of which lung
lar injury. They include an expanding hematoma, cancer is the most common. Breast and testicular
pulsatile blood loss, a palpable thrill, or audible cancer are less common causes of SVC syndrome.
bruit and any combination of the "six P's" indica- Colon and thyroid neoplasms are uncommon causes.
tive of distal limb ischemia (pain, pallor, pulse- Symptoms include facial and upper extremity edema,
lessness, paresthesias, paralysis and poikilothermia shortness of breath, cough, and chest pain. Diagnosis
[cool to touch]). Clinical "soft" signs provide less is made by physical examination, chest x-ray, and
definitive evidence of vascular injury but mandate advanced imaging such as echocardiography, cr.
nonurgent evaluation. They include a nonpulsatile and/or MRL Management has traditionally involved
hematoma, an associated peripheral nerve deficit radiation therapy, but chemotherapy and surgical
(due to the proximity of nerves to vascular struc- evaluation have potential roles in the acute treatment
tures in the neurovascular bundle), and a dimin- as well.
ished distal pulse. A decreased ankle-brachial index
(ABI) is also sometimes cited as a sign of lower ex- ~ Answer D. Patients without gross hematuria on
tremity vascular injury, but its diagnostic utility is initial urine output after trauma almost never
controversial. have bladder rupture. Only certain patients with
pelvic fractures and microscopic hematuria can
(!?) Answer A. Tricyclics block fast sodium channels, have bladder ruptures without the presence of
slowing phase zero myocardial depolarization and gross hematuria. Bladder rupture can occur with
causing QRS prolongation. Negative inotropy occurs either blunt or penetrating trauma and does not
due to reduced numbers ofopened calcium channels. require the presence of a concomitant pelvic fracture.
Potassium efD.ux blockade causes QT prolongation Retrograde cystography is used to evaluate suspected
from impaired repolarization, a -1 antagonism causes bladder injury, but must be performed only after
hypotension, and anticholinergic effects cause tachy- urethral injury has been adequately ruled out.
cardia, hyperthermia, urinary retention, and agita- Extraperitoneal bladder rupture is usually managed
tion. Tricyclics have no known effect on magnesium conservatively with Foley catheter urine drainage
channels.
Test4 113

alone. Intraperitoneal bladder rupture almost always exacerbations is the concomitant administration of
requires nonemergent surgical repair. systemic corticosteroids, IJ-agonists (albuterol} and
frequently ipratropium as an adjuvant therapy. Cro-
~ Answer E. Urethral injury occurs most commonly molyn sodium has no role in the management of
in men due to the length of the male urethra. The acute asthma exacerbations. It works by inhibiting
injury is suspected on physical examination by a the release of inflammatory mediators from mast
variety of findings-blood at the urethral meatus, cells through chlorine channel blockade (also known
scrota1/penile hematoma, severe pelvic fracture, or as a mast ceU stabilizing agent) and is used to prevent
high-riding or mobile prostate. Evaluation of sus- inflammation in long-term management. There is
pected urethral injury generally involves retrograde limited data that suggests that leukotriene receptor
urethrogram to assess for extravasation of contrast. antagonists (e.g., montelukast, zafirlukast) are bene-
However, once a Foley catheter is successfully placed ficial in the treatment of patients with acute asthma
and drained the bladder of urine, it should not exacerbations who fail initial J3 -agonist therapy. Until
be removed until urologic consultation is obtained. further studies are done, however, leukotriene modi-
The key to managing urethral injuries is to pre- fiers are not currently recommended for acute asthma
vent urine from leaking out of the bladder into the management.
retroperitoneum, pelvis, or abdomen. Ifthe Foley has
already successfully decompressed the bladder, the ~ Answer C. QT interval prolongation is the most
urethral injury may be evaluated later in the oper- common EKG abnormality in patients with heat-
ating room or with a modified bedside urethrogram related illness. Interestingly, QT interval prolon-
around the Foley, ifneeded. Removal of the Foley can gation is also very common in patients with
convert a partial urethral disruption into a complete hypothermia. Other common EKG findings include
one and should be avoided. sinus tachycardia, atrial fibrillation, supraventricular
tachycardia (SVT}, right bundle branch block, and
~ Answer E. Hyperkalemia predisposes patients to occasional ST segment changes. Sinus bradycardia
serious dysrhythmias. The management of hyper- and ventricular fibrillation do not commonly occur.
kalemia involves all of the answer choices given
in the preceding text. Sodium bicarbonate begins ~ Answer C. The patient has acute angle closure
to function in approximately 10 minutes to move glaucoma. The history of ambient light suddenly
potassium ions from the blood into the intracel- decreasing and forcing rapid pupillary dilatation is
lular space, thereby reducing measurable (but not classic. Patients with glaucoma may often complain
total body} potassium. Calcium salts, although ef- of headache, nausea, vomiting, and abdominal pain
fective within 5 minutes, do not reduce potassium without any specific ocular symptoms. Bilateral shal-
levels-they function to stabilize the cardiac myocyte low anterior chambers are the specific anatomic
membrane to prevent the deleterious effects of hy- abnormality predisposing to closure of aqueous out-
perkalemia. Insulin and glucose begin to work in 30 flow-pilocarpine is therefore administered to both
minutes, and Kayexalate starts to exchange sodium eyes (either eye has an equal chance of being af-
for potassium in 1 hour. Albuterol, another therapy fected by the acute angle closure). Boxcar retinal
to reduce extracellular potassium, works in approxi- venules are characteristically seen in central retinal
mately 30 minutes, but is less effective than the other artery occlusion. Although emergent ophthalmo-
therapies. Hemodialysis is the management of choice logic consultation is required, all modalities for
in cases of hyperkalemia that are refractory to the treatment of a severe glaucoma attack should be
medical management given in the preceding text. initiated early-pilocarpine, timolol, apraclonidine,
prednisolone, acetazolamide, and mannitol. Intra-
~ Answer D. Rectal prolapse in children is typically venous medications may be withheld if intraocular
incomplete, involving only the mucosal layer of pressures are not severely elevated. Sedatives and
bowel. It is more common in boys and usually occurs antiemetics are used as needed. Ocular massage is
by the age of 2. It may be a sign of malnutrition absolutely contraindicated, as this will increase in-
or underlying cystic fibrosis (CF}. Reduction should traocular pressure.
be attempted in the ED, although sedation may be
required. ~ Answer E. It may be very difficult and occasionally
impossible to differentiate patients with psychogenic
~ Answer B. Fortunately for emergency medicine seizures from patients with a true seizure disorder.
physicians, management of acute asthma exacer- However, patients with psychogenic seizures tend to
bations is consistent across age-groups. The foun- have several distinctive clinical features. Interestingly,
dation of effective management of acute asthma self-injurious behaviors such as urinary incontinence
114 1000 Questions to Help You Pass the Emergency Medicine Boards

and tongue biting may occur in patients with drugs, phenytoin, and diuretics are most commonly
psychogenic seizures and are therefore not helpful implicated. Macrolides and tetracyclines are less of-
in differentiating these patients from patients with ten involved. Interstitial nephritis may also be due
a true disorder. Fecal incontinence is uncommon. to infectious and immunologic causes. Urinalysis
Most problematic, perhaps, is that up to 58% of generally demonstrates sterile pyuria with possible
patients with psychogenic seizures may also have hematuria and mild proteinuria. Eosinophilia oc-
a concomitant real seizure disorder. Patients with curs in approximately half the number of patients.
psychogenic seizures also demonstrate avoidance Definitive diagnosis is by kidney biopsy. Renal insuf-
behaviors (e.g., eyes forced shut, would not allow ficiency due to allergic interstitial nephritis is usually
hand to be dropped on their face), have "out- reversible, and treatment always involves removal of
of-phase" movements of their extremities and the offending drug.
frequently demonstrate either forward or backward
pelvic thrusting. Patients with true seizures typically ~ Answer E. Pilon fractures are frequently commin-
have in-phase extremity movement (synchronous uted fractures ofthe tibial plafond (the distal articular
contraction of their extremities) rather than the surface of the tibia) associated with high-energy
thrashing movements of patients with psychogenic trauma. They most commonly occur due to axial
seizures. Finally, patients with psychogenic seizures loading after a fall in which the force of the fall is
demonstrate "geotropic" eye movements. This refers transmitted through the calcaneus and talus to the
to the tendency of patients with psychogenic seizures tibial plafond (sometimes referred to as a hammer
to persistently tum their gaze away from the examiner fracture). Owing to the axial nature of these in-
whenever the patient's head is turned. juries and the high energy involved, other structures
of the axial skeleton are at risk for injury as well,
~ Answer B. Pneumonia is the most common cause including all of the listed structures. Therefore, a
of death among residents oflong-term care fucilities. thorough physical examination should be performed
It is also the most common reason for transfer to evaluate for tenderness is these areas.
to such a facility. As with many other diseases,
the clinical presentation of pneumonia in the ~ Answer B. Signs and symptoms of shock may be
elderly may be very vague or atypical. Frequently, misleading in the pregnant trauma patient. Acid-base
elderly patients may lack fever, cough, chest pain, abnormalities such as decreased bicarbonate, or ele-
headache, and myalgias. They may also not be vated lactate or base deficit are the earliest indicators
strong enough or oriented enough to vocalize of maternal shock. Due to an increased intravascu-
complaints about dyspnea. Studies have revealed lar volume, pregnant patients will often not develop
that in general, elderly persons manifest fewer hypotension or tachycardia until significantly more
overall symptoms than do their younger cohort in blood volume is lost than their non-pregnant peers.
the setting of pneumonia S. pneumoniae remains Thus, early crystalloid administration is indicated
the most common pathogen in both community- to sustain the patient's increased blood volume and
acquired pneumonia (CAP) and in pneumonia avoid fetal hypoperfusion. If blood loss occurs more
acquired in a nursing home setting. Although the gradually, a pregnant woman can lose as much as 35%
etiology of nursing home-acquired pneumonia is of her blood volume before the development of bla-
often undetermined, the microbiology more closely tantly abnormal vital signs. Additionally, because of
resembles community-acquired pneumonia than the physiologic decrease in peripheral vascular tone
nosocomial pneumonia. Interestingly, the risk of during pregnancy, pregnant patients in shock may
invasive pneumococcal disease is fourfold higher remain warm and dry even in the setting of shock.
in the nursing home population than in elderly Finally, abdominal examination is less sensitive in
persons living in the community. Although its pregnant patients. As many as 50% of pregnant pa-
efficacy has not been 100% validated, most authors tients with hemoperitoneum will not have peritoneal
agree that all patients of long-term care facilities signs on physical examination. Therefore, ED physi-
should be vaccinated against both influenza and cians need to maintain a high suspicion for injury
S. pneumococcus. and a low threshold for further diagnostic testing.

~ Answer A. The onset of renal insufficiency, serum ~ Answer B. Acclimation or acclimatization is a col-
sickness-like signs and symptoms, and eosinophilia lection of physiologic changes that occur in response
points to allergic interstitial nephritis as the most to repeated heat stress. Physiologic changes typically
likely cause. A hypersensitivity reaction to vari- occur over a 7- to 14-day period presuming consis-
ous drugs is the usual cause-penicillins, nons- tent, daily exposure. Physiologic changes include
teroidal anti-inflammatory drugs (NSAID)s, sulfa increased plasma volume, a lower threshold for
Test4 115

sweating (earlier onset), increased rate and volume of Disease Control (CDC) currently recommends their
sweating with decreased electrolyte content of sweat, use only in immunocompromised patients or in
increased aldosterone secretion, decreased heart rate patients with a concomitant serious illness. The
and increased capacity for peripheral vasodilation. chief concern of routine prophylaxis is the develop-
This patient has heat edema, which is a benign con- ment of antibiotic resistance and patient neglect of
dition most commonly seen in the elderly and in common-sense precautions. Widespread resistance
nonacclimatized individuals. It is thought to be due to fluoroquinolones has already occurred among
to a combination of orthostatic pressure and vascular many Campylobacter isolates. Bismuth subsalicylate
leak. Diuretic therapy is not beneficial and may cause has been shown to reduce the incidence of traveler's
dehydration. Although a minimal workup for other diarrhea. However, patients should be made aware
common conditions resulting in lower-extremity of its high aspirin content and its tendency to cause
edema may be necessary, an echocardiogram is not patient stools to turn black.
required. Generally, heat edema resolves with accli-
mation or upon return to a patient's baseline climate, ~ Answer C. Typical radiographic findings in PCP
and may be treated with simple measures such as leg include bilateral perihilar interstitial infiltrates that
elevation and support stockings. become progressively diffuse as the disease pro-
gresses. Other less common findings include the
~ Answer B. The patient has hypertensive en- presence of solitary or multiple nodules, pneumato-
cephalopathy. The history of acute headache with celes, and the presence of a pneumothorax. Patients
vomiting and severely elevated blood pressure is with HIV/AIDS who have been on prophylactic
characteristic. Neuroimaging with CT scan is often TMP-SMX therapy have a worse outcome in the set-
normal. The treatment is immediate reduction of ting PCP requiring mechanical ventilation. A study
blood pressure by as much as 30%. By definition, hy- reported in 2000 revealed that only 28% of mechan-
pertensive encephalopathy is reversible when blood ically ventilated patients with PCP who had been
pressure is reduced. Neurosurgical consultation may taking TMP-SMX before their diagnosis survived to
be indicated later in the course, but blood pres- hospital discharge. This compares with 47% of pa-
sure management should be instituted early. Lumbar tients who had not been on prior prophylaxis. There
puncture would be contraindicated in this circum- is some suggestion that this effect may be due to
stance, due to the papilledema indicating increased emerging resistance among P. carinii isolates. Pa-
intracranial pressure. Corticosteroids are indicated in tients with AIDS and PCP usually present with a
cases oftemporal arteritis, which is on the differential subacute history of dyspnea, nonproductive cough,
diagnosis, but is far less common than hypertensive low-grade fevers, and occasionally weight loss. They
encephalopathy and requires the presence of an ele- are frequently found to have tachycardia and tachyp-
vated erythrocyte sedimentation rate (ESR). MRI of nea on physical examination. Patients without AIDS
the brain can add important structural information, who are diagnosed with PCP usually present with an
but is not indicated emergently. abrupt onset ofdyspnea and respiratory insufficiency.
A similar course in patients with AIDS may suggest
~ Answer B. Silver nitrate sticks should not be the presence of a pneumothorax. Corticosteroids are
applied to both sides of the septum to avoid beneficial in addition to antibiotic therapy in reduc-
possible septal perforation. Silver nitrate sticks will ing mortality and clinical deterioration in patients
not work if active bleeding is present. They are best with HIV and PCP who have hypoxemia (defined
reserved for raw areas of nasal mucosa to which as a Pao2 <70 rnm Hg or an alveolar-arterial gradi-
topical vasoconstrictor agents (such as cocaine or ent >35). They are not used indiscriminately for all
phenylephrine) have already been applied. Blowing patients with PCP. Although recent studies suggest
the nose is essential before placement of any packing airborne transmission as the mechanism of infec-
material to remove any clot that would prevent tion in PCP, respiratory isolation is not currently
effective packing of a site of continuous bleeding. recommended.
Cephalexin or amoxicillin-clavulanate is generally
used prophylactically in patients with any nasal [!!) Answer D. Diverticulitis occurs in 10% to 30% of
packing; TMP-SMX or azithromycin may be used patients with diverticulosis. Severe bleeding occurs
in the penicillin-allergic patient Admission is always in only 3% to 5% of patients with diverticula. Most
indicated in patients with a posterior pack. bleeding is minor and resolves spontaneouslywithout
intervention.
@1) Answer E. Although prophylactic antibiotics have
been shown to be effective in reducing the inci- ~ Answer A. Head trauma accounts for the large
dence of traveler's diarrhea, the U.S. Centers for majority of all pediatric traumatic deaths. Falls
116 1000 Questions to Help You Pass the Emergency Medicine Boards

and motor vehicle crashes (MVCs) are the most Tinel signs, respectively, to evaluate for carpal tunnel
common mechanisms. When compared to adults, syndrome and median nerve compression.
children's heads are proportionally larger and heavier
relative to the rest of their body resulting in a ~ Answer E. The patient has evidence of uncom-
higher likelihood of serious injury. The possibility plicated urinary tract infection (UTI). The most
of intentional injury should be sought in all cases common cause is E. coli, followed by other gram-
of pediatric head trauma. Unlike adults, in whom negative bacilli, then streptococci. Treatment in
the extent of intracranial bleeding is limited by cases of uncomplicated UTI is for 3 days with ei-
the fixed bony skull, infants may develop severe ther TMP-SMX, fluoroquinolone, or a penicillin.
hemorrhagic shock from intracranial bleeding into The patient has allergies to both sulfa drugs and
a more flexible skull Furthermore, intracranial fluoroquinolones. Many bacteria which cause UTis
catastrophes such as epidural hematomas may occur are resistant to amoxicillin alone, so amoxicillin-
due to venous bleeding (rather than arterial bleeding clavulanic acid is an alternative. Doxycycline and
as is more common in the adult) resulting in delayed azithromycin have better coverage against gram-
presentations. positive organisms and atypicals and often lack
effectiveness against gram-negative bacilli.
~ Answer D. Although A, B, C, and E all confer an
increased risk of lenticular dislocation, trauma as a ~ Answer C. A small degree of chronic hemoly-
whole is the most common cause. Marfan's syndrome sis occurs in all patients, which in most cases is
is the most common cause of inherited lenticular clinically insignificant if appropriate iron supple-
dislocation. Glaucoma and retinal detachment may mentation is instituted. Mechanical, not porcine,
complicate lens dislocation and must be managed valves require anticoagulation with Cournadin, and
in concert with an ophthalmologist. Iridectomy or the International normalized ratio (INR) is generally
lensectomy may be indicated in some cases. kept between 2.5 and 3.5. Mechanical valves always
have a metallic closure sound--absence of this sound
~ Answer B. Chelation therapy for acute lead tox- indicates valve dysfunction. The risk of endocarditis
icity is indicated in patients with worsening clin- depends on the length of time the valve is func-
ical course or severe CNS or GI symptoms. Sev- tioning, but occurs in less than half the number of
eral chelation therapies exist for lead. Dimercaprol patients.
(or British antilewisite [BALl) should be the first
chelator given in patients with severe poisoning. It ~ Answer B. Pericardial effusions are seen as car-
should be given before calcium disodium EDTA, diomegaly on chest x-ray when approximately
as the latter, if given first, will cause chelated lead 250 mL of fluid has accumulated around the heart.
to cross the blood-brain barrier. Acute lead en- Beck's triad of hypotension, muffled heart sounds,
cephalopathy should be treated aggressively with and jugular venous distention (JVD) occurs in <25%
chelation and management of attendant cerebral of patients with tamponade and should not be re-
edema (hyperventilation and mannitol). Activated lied upon to make the diagnosis. Echocardiography,
charcoal does not bind lead or other heavy met- not MRI, is the diagnostic test of choice in most
als. Patients deemed stable enough for outpatient cases--CT scan may add helpful information, but
chelation therapy should be given oral succimer. is not required. Blind pericardiocentesis should only
Penicillamine is a less effective alternative to suc- be performed in unstable cases of tamponade when
cimer and should be given only if succimer is not echocardiography is not available as the complica-
tolerated due to GI side effects. tion rate is high. Tachycardia is the most common
abnormal finding seen on EKG, and decreased volt-
~ Answer A. The thoracic outlet syndrome com- ages may also be seen more commonly than electrical
prises a group of pathologic conditions associated alternans.
with compression of the structures at the junction
of the upper extremity and trunk. The findings are ~ Answer C. In children, Ludwig's angina may occur
neurologic (95%), venous (4%), and arterial (1%). without an antecedent cause, although the disease is
The elevated arm stress test is the best physical ex- less common in children than in adults. Asphyxiation
amination tool to determine the presence of thoracic is the most common cause of death, and it
outlet syndrome. The test is accurately described by results due to upper airway obstruction due to the
choice A, and a positive result is indicated by ann fa- extensive swelling and edema of the floor of the
tigue and pain, and the inability to keep it abducted. mouth and neck. In patients with impending airway
Choice B is the supraspinatus test to evaluate for compromise, fiberoptic nasotracheal intubation is
rotator cuff tear. Choices C and D are Phalen and the preferred method ofairway control. Owing to the
Test4 117

significant edema, trismus, secretions, and anatomic and increased time for toxin action. Colonoscopy is
distortion of the airway, endotracheal intubation not required for treatment but may help to rule out
may be extremely difficult. Furthermore, although a other causes or confirm the diagnosis. In mild cases,
surgical tray should always be present while fiberoptic withdrawal of the offending antibiotic may be all
intubation is undertaken, cricothyroidotomy is also that is necessary. If symptoms do not rapidly resolve,
more difficult to perform in the setting of Ludwig's metronidazole is the treatment of choice with van-
angina. With early antibiotic therapy, the mortality comycin reserved for refractory cases. Children with
rate of Ludwig's angina is <10%. Patients who C. difficile colitis typically have a more severe course,
have an underlying oral malignancy are not treated especially in children undergoing chemotherapy.
any differently than other patients with Ludwig's
angina. All such patients are treated with immediate ~ Answer B. Patients with hemolytic disorders, such
intravenous antibiotics, with rigorous observation of as sickle cell anemia or hereditary spherocytosis,
the airway. represent the largest group of pediatric patients with
symptomatic cholelithiasis. Hemolysis puts these
~ Answer E. Not surprisingly, one of the best pre- patients at risk for the formation of pigmented
dictors for treatment failure and death is a history of gallstones. Cystic fibrosis (CF) and obesity also put
prior intubation and ICU admission. In addition, ex- children at risk for cholelithiasis. In CF, patients
tensive use of albuterol MDI canisters ~2 per month, experience inspissated biliary secretions leading to
asthma hospitalizations ~2 per year, ED visits for stone formation. Diabetes and cerebral palsy are
asthma ~3 per year, hospitalization or ED visit for unrelated diseases. Neonates often have multiple
asthma in the last month, current use or recent dis- factors predisposing to gallstones but these patients
continuation of systemic corticosteroids, comorbid are usually diagnosed in the neonatal ICU, before
cardiac disease, concomitant illicit drug use as well discharge and presentation in an ED. Examples of
as serious psychosocial or psychiatric problems all neonatal risk factors include prematurity, parenteral
predict death from an asthma exacerbation. nutrition, surgery, blood transfusion, sepsis, and
diuretic administration.
~ Answer B. Folate is a cofactor for the conver-
sion of methanol's toxic metabolite, formic acid, ~ Answer C. Diagnosis of CAPO-associated peri-
to carbon dioxide and water. Once formic acid is tonitis is through analysis of the peritoneal
produced, significant toxicity is probably inevitable, fluid-100 or more WBC per mm3 with a predomi-
but the addition offolate to the standard treatment of nance of neutrophils or a positive Gram stain makes
methanol overdose (bicarbonate, alcohol dehydro- the diagnosis. Unlike cases of spontaneous bacterial
genase inhibitors, and dialysis) may attenuate further peritonitis, where gram- negative enteric organ-
injury. isms predominate, patients with peritoneal dialysis
catheters tend to develop peritonitis from gram-
~ Answer C. Clostridium difficile is a gram-positive positive organisms, most commonly Staphylococcus
rod that is present in approximately 3% of healthy species. Clinical signs and symptoms of infection
adults. However, antibiotic therapy used to treat can be extremely mild and asymptomatic infection is
unrelated infections increases the carriage rate of common. Most cases can be treated on an outpatient
C. diffieile tremendously. It has been implicated in basis with intraperitoneally administered antibiotics.
10% to 25% of patients with antibiotic-associated di- The antibiotics of choice are vancomycin plus any
arrhea, but in 50% to 75% of patients with antibiotic- antibiotic with good gram-negative coverage, such as
associated colitis and nearly all patients with evidence a fluoroquinolone, third-generation cephalosporin,
of pseudomembranous colitis. Clindamycin is clas- aminoglycoside, or aztreonam.
sically associated with the development of C. difficile
colitis. However, the use of other antibiotics such ~ Answer A. The metabolism of ethylene glycol is:
as cephalosporins and fluoroquinolones may also Ethylene glycol ""*Glycoaldehyde ""*
Glycolic acid
result in colitis. The gold standard for diagnosis is ""*""*""*
Oxalic acid. Oxalic acid forms calcium ox-
considered to be a cell cytotoxicity assay but due to alate crystals which can deposit in the renal tubules
the cost and labor involved, most laboratories use a and cause renal insufficiency, and the other metabo-
toxin detection assay. Positive stool cultures are not lites ofethylene glycol are directly nephrotoxic as well.
diagnostic because the bacteria are often present Approximately one fourth of ethylene glycol is di-
in healthy subjects and are increasingly present rectly excreted in the kidneys, but hepatic metabolism
in patients who have been on antibiotics. Diphe- with alcohol dehydrogenase catalyzes the formation
noxylate (Lomotil) or other antimotility agents may of the toxic metabolites. The goals of therapy in
worsen the disease by allowing further overgrowth patients with ethylene glycol toxicity are to block
118 1000 Questions to Help You Pass the Emergency Medicine Boards

the availability of alcohol dehydrogenase with either of prostaglandin E1 are apnea and hypotension.
fomepizole or ethanol, and to hemodialyze the unme- Albuterol is useful in cases of reactive airways disease
tabolized ethylene glycol. Methanol toxicity results but does not benefit with congenital heart disease.
in the formation of formic acid, which accumulates Indomethacin promotes closure of the ductus and
in the brain and causes blindness and death. Iso- would be detrimental. Aspirin is never indicated in
propanol causes generalized CNS depression similar pediatric patients except in the setting of Kawasaki
to ethanol intoxication. Salicylate overdose results in disease. Ribavirin is indicated in select patients
direct nephrotoxicity, metabolic acidosis, electrolyte with respiratory syncytial virus (RSV) bronchiolitis.
abnormalities and pulmonary and cerebral edema. (Figure courtesy of Mark Silverberg, MD. From
Acetaminophen overdose causes fuhninant hepatic Silverberg M. Hurst's the heart, 11th ed. McGraw-
failure. Hill; 2004:602.)

~ Answer D. Neurologic dysfunction is the hallmark ~ Answer E. The patient has a large, spontaneous
of heatstroke. Patients with heat exhaustion present subconjunctival hemorrhage. There is no apparent
with intact mental status, although they may present chemosis or hyphema. Management is purely sup-
with generalized malaise, fatigue, headache, impaired portive with avoidance of NSAIDs. Patients with
judgment, vertigo, as well as nausea and vomiting. history offrequent subconjunctival hemorrhage may
Patients with heat exhaustion also typically present require a coagulopathy workup, but most patients
with persistent and profuse sweating, with a core with coagulopathies will have other manifestations of
temperature that may be only mildly elevated and bleeding as well. Emergent ophthalmologic consulta-
which is always < 104°F. Patients with heat stroke tion is not indicated in patients with subconjunctival
usually have a core temperature >105°F resulting hemorrhage, but bloody chemosis or hyphema would
in multiorgan failure. Patients may be anhidrotic necessitate this. Topical antihistamines or antibiotics
and have elevated hepatic transaminases, although are not indicated in patients with subconjunctival
these findings are not required for the diagnosis. hemorrhage. (Figure courtesy of Anthony Morocco,
In addition, patients with heat exhaustion may also MD. Reprinted with permission from Morocco A.
develop elevated hepatic transaminases, although the Greenberg's text-atlas of emergency medicine. Lippin-
increase is less severe. Most significantly, patients cott Williams & Wilkins; 2004:85.)
with heat stroke have an altered sensorium (delirium)
and may develop coma or seizures. ~ Answer C. Eyelids do not contain subcutaneous
fat-the presence of fat indicates likely globe injury.
~ Answer C. This patient has an injury to zone Patients with globe injuries should be seen emer-
II of the neck defined as the region of the neck gently by the ophthalmologist. ED management
superior to the cricoid cartilage and inferior to involves broad-spectrum antibiotics, eye shielding,
the angle of the mandible. Trauma to zone II is avoidance of recumbent position or Valsalva maneu-
most common but also most amenable to repair vers, tetanus immunization, and treatment of nausea
due to relatively uncomplicated surgical exposure and vomiting. Choices A, B, D, and E are poten-
and vascular control. Traditionally, all patients tial indications for ophthalmologic or plastic surgical
with violation of the platysma have been taken to repair to prevent significant cosmetic defects.
orthopaedic research (OR) for exploration. In the
modern era of rapidly improving radiologic testing, ~ Answer E. This patient has herpes zoster (shin-
operative repair may increasingly be deferred in favor gles). Although shingles may be more likely to occur
of diagnostic testing such as helical CT angiography in patients with leukemia, Hodgkin's lymphoma
(e.g., to evaluate vascular injury indicated by a bruit as well as other malignancies, most cases occur in
or thrill), laryngoscopy, or esophagoscopy (e.g., to otherwise healthy patients. (Figure reprinted with
evaluate subcutaneous emphysema). permission from Weber J. Health assessment in nurs-
ing, 2nd ed. Lippincott Williams & Wilkins; 2002.)
~ Answer A. The chest x-ray shows an enlarged,
boot-shaped heart, consistent with tetralogy ofFallot. ~Answer C. The abdominal radiograph demon-
Patients with suspected tetralogy of Fallot should strates multiple, radiopaque packets consistent with
be managed in a step-wise manner-oxygen, fluid body packing of recreational drugs. Because cocaine
resuscitation, morphine, and a-agonist therapy. and other sympathomimetic agents are often packed,
Prostaglandin E1 may be indicated in neonatal cardiac monitoring is indicated for these patients.
patients to prevent closure of the ductus arteriosus Asymptomatic patients may not require any spe-
and ensure enough shunting of blood to the cific therapy, but symptomatic patients should be
pulmonary vasculature. The major adverse effects evaluated carefully for signs of systemic toxicity and
Test4 119

treated with supportive care and antidotes as in- patient population who had ischemic symptoms
dicated. Patients may be given polyethylene glycol for an average of 3.5 hours, which is longer than
to induce whole bowel irrigation to promote more the typical TIA and longer than the newly revised
rapid transit of the packets through the GI tract. definition of a TIA. However, he found five risk
MRI is not indicated in patients with body packets, factors (age older than 60 years, diabetes mellitus,
as it will not change management and adds little to duration > 10 minutes, weakness with episode,
the diagnosis. Surgical removal is indicated for signs speech impairment with episode) that correlated to
of severe toxicity or bowel obstruction. Nasogastric the risk of stroke within 90 days of ED discharge. The
(NG) aspiration is unlikely to provide any benefit risk ranged from 0% without any of the risk factors
and may instead cause retching and vomiting. Endo- to 34% with all five risk factors. These criteria have
tracheal intubation is not indicated in the absence of yet to be prospectively validated.
severe systemic toxicity. (Figure courtesy of Robert
Hendrickson, MD. Reprinted with permission from ~ Answer C. The development of uterine contrac-
Hendrickson R. Greenberg's text-atlas of emergency tions is the most common consequence of maternal
medicine. Lippincott Williams & Wilkins; 2004:805.) trauma. This is due to stimulatory prostaglandins that
are released upon contusion to the maternal uterus.
~ Answer D. The patient meets clinical criteria for Ninety percent of contractions stop spontaneously
thrombotic thrombocytopenic purpura (TTP). The and tocolytics are generally not used. However, all
classic clinical pentad includes fever, microangio- pregnant patients with a viable fetus should undergo
pathic anemia, thrombocytopenia, renal dysfunc- continuous cardiotocographic monitoring. In pa-
tion, and neurologic symptoms. Pathophysiology tients with abdominal pain and overt contractions,
involves formation of microthrombi in the systemic most authors recommend admission for 24 hours
vasculature, consuming platelets and causing mi- of monitoring. Though placental abruption is the
croinfarctions, usually manifested in the kidneys and most common cause of fetal loss related to trauma,
brain. Diagnosis is suspected by thrombocytopenia the presence of uterine contractions is not enough to
with anemia in a patient with suggestive clinical make this diagnosis. Fetal distress and persistent uter-
findings. Confirmation is made by the presence of ine irritability upon cardiotocographic monitoring is
schistocytes on peripheral blood smear. Notably, the most sensitive indicator of placental abruption.
hematologic laboratory studies such as PT, PTT, and Uterine rupture is rare and most commonly occurs in
DIC panel are almost always normal. Treatment is ur- women who have a history of prior cesarean delivery.
gent plasmapheresis with or without corticosteroids. It may be difficult to diagnose and is associated with a
Platelet transfusions are contraindicated, as they ex- very high fetal mortality. Digital examination should
acerbate microthrombi formation. Hemodialysis is never be performed after the first trimester because
not usually required as renal dysfunction is rarely ED physicians could trigger catastrophic bleeding in
severe. Splenectomy is used as a second-line ther- patients with undiagnosed placenta previa. Such ex-
apy if plasmapheresis is not successful. Acyclovir aminations are best left for the obstetrician after US
may be used for patients with herpes simplex has verified placental position and fetal viability.
virus (HSV) encephalitis-although the patient has
fever and headache, she lacks altered mental sta- ~ Answer E. The strips demonstrate "late deceler-
tus or meningeal findings as would be seen in ations" of the fetal heart rate relative to uterine
encephalomeningitis, and the laboratory findings are contractile activity. Late decelerations typically be-
more suggestive ofTIP. gin roughly 30 seconds after the onset of a uterine
contraction and their nadir occurs after the peak of
~ Answer A. Interestingly, patients with amaurosis the contraction. This most often represents utero-
fugax are considered low risk because treatment placental insufficiency, which is an interruption in
with antiplatelet agents (e.g., aspirin) is twice as uteroplacental blood flow. Early decelerations have
effective in preventing strokes versus patients with the same gradual slope and shape as late decelera-
hemispheric ischemia. High-risk patients include tions but they occur with different timing relative to
any patient with new-onset atrial fibrillation or uterine contractions. The nadir ofearly decelerations
flutter (potential cardioembolic source), patients and the peak of uterine contractions occur simulta-
with crescendo transient ischemic attack (TIA) neously and the fetal heart rate returns to baseline
(more than three discrete ischemic episodes within by the end of the contraction. Early decelerations are
a 72-hour period), patients who develop a TIA thought to result from compression ofthe fetal head,
while already on aspirin therapy (considered aspirin which causes a vagal reflex. Variable decelerations
failure) and any patient meeting several of the are the most common type of pattern seen during
"Johnston" criteria. Johnston et al. studied an ED fetal cardiac monitoring. These decelerations have an
120 1000 Questions to Help You Pass tM Emergency Medicine Boards

inconsistent appearance with respect to shape, width, for isolation precautions during hospitalization.
depth, and timing relative to uterine contractions. Most normal hosts recover in < 1 week but immune-
They represent umbilical cord compression and are compromised patients are at risk of significant
typically benign. However, frequent or particularly dehydration.
deep decelerations (i.e., representing fetal bradycar-
dia) may be an indicator of fetal distress. (Figure ~ Answer E. The most common causes of hypo-
reprinted with permission from Pilliterri A. Mater- phosphatemia in the ED are probably respiratory
nal and child health nursing. Lippincott Williams & alkalosis, treatment of diabetic ketoacidosis (DKA),
Wilkins; 2006.) and alcoholism. The most common mechanism is an
intracellular shift of phosphate (respiratory alkalosis,
~ Answer D. Concussion is defined as a clinical treatment of DKA). Renal insufficiency leads to
syndrome following mild traumatic brain injury with phosphate retention, whereas hyperparathyroidism
or without loss of consciousness. The symptoms are causes increased renal excretion.
most commonly confusion, amnesia surrounding
the traumatic event, headaches, and nausea. Other ~ Answer D. The heterophile antibody test, or
neurologic signs such as slurred speech, attention monospot, is a rapid assay for the diagnosis of in-
deficits, incoordination, mild personality changes, fectious mononucleosis due to EBV. It has lower
and disorientation may occur in conjunction with sensitivity early in the illness ( <70% in the first
the amnesia and confusion. Neuroimaging is usually week), but steadily improves in the next few weeks.
normal acutely, as concussion is characterized by Specificity is excellent and reported to be near 100%.
functional, rather than structural, findings. MRI may It is important to tell patients suspected of having
detect subtle signs of cerebral contusion or axonal infectious mononucleosis with a negative monospot
injury a few days after the injury in a minority of test that either the illness is early in its course or a
cases. Generally accepted guidelines indicate that non-EBV cause is possible. Infectious mononucleo-
athletes who have had a concussion should not sis is a subacute to chronic multisystem syndrome of
return to sports until approximately 1 week after pharyngitis, lymphadenopathy, splenomegaly, hep-
symptoms have resolved. This is in part due to fears atitis, and fatigue. Splenomegaly is perhaps the most
about the brain being particularly vulnerable to a important clinical feature for the emergency physi-
second impact in the week immediately following cian (EP) as demonstration of this finding requires
a concussion. Anosmia, the absence of the sense strict avoidance of contact sports and aggressive
of smell, may occur in concussions and may be physical activity to prevent splenic rupture. Treat-
permanent, also causing alterations in taste. ment is supportive. Almost one third of all patients
with infectious mononucleosis carry group A strep-
~ Answer A. Campylobacter spp. is found in the tococcus, which confounds the diagnosis of a patient
stools of 5% to 14% of patients presenting with with febrile pharyngitis and leads to inappropriate
a chief complaint of diarrhea. The exact incidence in antibiotic use.
the United States is not known due to underreporting
and sporadic testing. [!!) Answer A. The patient has endocarditis of a
prosthetic valve, which is most commonly due to
1!1] Answer C. The patient has evidence of acute coagulase-negative staphylococci. S. viridans is the
epididymitis, an infection of the epididymis causing most common cause of native valve endocarditis.
local tenderness and lacking findings suggestive of Choices C, D, and E are uncommon causes of native
testicular torsion (unilateral testicular tenderness or valve endocarditis.
edema or absent cremasteric reflex). The etiologies
in sexually active men younger than 35 years of ~ Answer B. All of the items listed may result
age are Chlamydia or gonococcus. In men older than in pancreatitis. In adults, gallstones are the most
35 years, E. coli is the most common cause. Antibiotic common cause in most populations (45% of cases)
therapy is directed to the causative organism. whereas alcohol abuse causes the bulk of the
remainder (35% of cases). In children, however,
~ Answer D. Rotavirus is the most common viral trauma is the most common cause. The most
and overall cause of acute gastroenteritis among common infectious cause is the mumps virus.
pediatric patients. The peak seasons for rotavirus Overall, trauma, infection, and idiopathic causes
infection are spring and winter with significant account for 70% of pediatric pancreatitis cases.
hospital outbreaks. Fever, vomiting, and diarrhea are
the most common clinical findings. Diagnosis may be [liD Answer C. Sixty percent to 85% of patients have
confirmed with a stool assay, but this is useful mostly motor weakness at the time ofpresentation. The most
Test4 121

common pattern of muscle weakness is symmetric be treated. As in nonpregnant women, metronidazole


lower-extremity weakness although any pattern can is the treatment of choice (250 mg orally three times
be seen. Bladder and bowel findings occur late in daily for 7 days) although clindamycin is an accept-
the course of the disease. Although it also occurs able alternative (300 mg orally twice daily for 7 days).
late in the course of compression, any patient with There is no evidence that metronidazole is unsafe
saddle anesthesia should be considered to have cauda in pregnancy. (Figure from Mandell GL, ed. Essen-
equina syndrome until proved otherwise. Back pain tial atlas of infectious diseases, 2nd ed. Philadelphia:
is the most common symptom ofepidural spinal cord Current Medicine Inc; 2001, with permission.)
compression, occurring in 83% to 95% of patients,
and characteristically precedes neurologic symptoms [!11 Answer C. The EKG shows a wide-complex, reg-
by approximately 2 months. ular tachycardia, which is almost always ventricular
tachycardia (VT). Amiodarone, procainamide, or li-
~ Answer C. Tick paralysis is likely due to an docaine may be used to treat stable VT. Although
unidentified toxin, which is transmitted to the human cardioversion may also be performed, it is painful for
host within a week of tick attachment. The typical awake patients and may not be necessary for patients
clinical presentation is that of ascending flaccid without hemodynamic instability. Adenosine and
paralysis with loss of deep tendon reflexes, similar diltiazem are both used in patients with narrow-
to Guillain-Barre syndrome. Respiratory failure can complex tachycardias and have no role in ventricular
occur due to diaphragmatic weakness. Ataxia is a less dysrhythmias. Supraventricular tachycardia (SVT)
common presentation and may occur in conjunction with aberrant conduction can also cause a regular,
with the flaccid paralysis. Cranial nerve and sensory wide-complex tachycardia. VT is fur more common,
findings are rare. Treatment is careful removal of and may be distinguished from SVT with aberrancy
the tick, which results in complete resolution of by the presence offusion beats, atrioventricular (AV)
symptoms within 2 days. dissociation, wider QRS complexes {>0.14 second),
and concordance ofprecordial leads. When in doubt,
~ Answer A. The spleen is the most common organ the EP should always treat regular wide-complex
injured in blunt abdominal trauma. In order of tachycardia as VT.
decreasing frequency, it is followed by the liver,
kidney, small bowel, bladder, colon, diaphragm, ~ Answer B. Sudden infant death syndrome (SIDS)
pancreas, and retroperitoneal duodenum. is defined as the sudden, inexplicable death of any
infant whose cause cannot even be diagnosed by
[!!] Answer A. This patient has bacterial vaginosis autopsy. The peak age for SIDS is 2 to 4 months.
(BV), which is the most common lower genital tract Risk factors include maternal smoking, young
infection among women of reproductive age. Clue maternal age, preterm age, among others. Apnea and
cells, which are squamous vaginal epithelial cells hypoventilation are the most likely explanations, but
coated with bacteria, are evident on the wet mount. dysrhythmias, airway obstruction, and trauma are
Classically, the disease has been diagnosed when three all proposed as possible contributors. Infants should
of the four Amsel criteria are present: be placed on their backs to sleep to help reduce the
• An adherent and homogenous vaginal discharge incidence of SIDS.
• Vaginal pH >4.5
• Detection of clue cells on saline wet mount [!!) Answer E. Owing to the large distribution of
• An amine odor after the addition of potassium vestibular pathways in the brain, there are many pos-
hydroxide (whiff test) sible centrally located lesions that result in vertigo.
The most important of these for emergency room
Though these criteria remain widely used, they
physicians is vertebrobasilar insufficiency (transient
have been criticized for their subjectivity. New assays
ischemic attacks [TIAs] ofthe vertebrobasilar arterial
are in development. Though Gardnerella vagirndis
circulation) or frank cerebellar stroke. V ertebrobasi-
is present in approximately 95% of cases, BV is
lar arterial ischemia is especially common in elderly
a polymicrobial infection with poorly understood
patients with typical risk factors for vascular dis-
origins. Furthermore, although it was originally con-
ease (e.g., diabetes, hypertension, hyperlipidemia,
sidered a relatively benign illness, recent research has
and smoking). It is important to recognize these
shown a clear correlation with preterm labor and
patients because they require admission and pos-
delivery, preterm premature rupture of membranes,
sible monitoring in an ICU setting (depending on
spontaneous abortion, chorioamnionitis, and post-
their syndrome). Cerebellar infarctions are known
partum infections such as endometritis. Therefore, all
for an unpredictable clinical course and sudden
pregnant and nonpregnant women with BV should
12 2 1000 Questions to Help You Pass the Emergency Medicine Boards

deterioration ofpreviously alert, stable patients. N eu- to food allergies. It is at least twice as common in
rosurgical intervention is typically required when women as in men. Although there are several clinical
the cerebellar infarct results in impingement on the criteria for the disease, one well known set of criteria
fourth ventricle with subsequent hydrocephalus. The (the Rome II criteria) includes abdominal pain that is
optimal time of intervention, however, has yet to be relieved after defecation as necessary for a diagnosis
determined. ofiBS.

~ Answer E. Tube thoracostomy is the most com- [iiD Answer D. Although US is the gold standard for
mon procedure performed in the setting of thoracic the diagnosis of cholelithiasis, only 50% of stones in
trauma. It is also the primary means by which a the common bile duct (CBD) can be visualized by US.
primary or secondary spontaneous pneumothorax This is due, in part, to the proximity of the duodenum
is treated (a small pneumothorax may be observed to the CBD and the interference that occurs due to
without intervention). The presence of a pneumoth- luminal bowel gas. However, US may detect a dilated
oraxmust have been caused by a defect in the patient's CBD in excess of 6 mm (the upper limit of normal)
bronchial tree resulting in communication between in up to 75% of cases. Therefore, US may suggest but
the bronchioles and the pleural space. The chest tube not confirm the diagnosis. CT scanning is extremely
is placed within the pleural space such that when useful in diagnosing complications ofgallstones, such
suction is applied to the chest tube the accumulated as perforation of the gallbladder, abscess formation,
pleural air will be removed allowing the lung to ex- pericholecystic fluid, pancreatitis, and gas in the
pand. However, because the original lesion does not gallbladder wall. The sensitivity and specificity of
heal instantaneously, the application of suction to CT has been improving with the use of helical
the pleural space may also remove air directly from scanners, however, and its sensitivity and specificity
the bronchial system through the original defect that is now roughly 85%. Likewise, cholescintigraphy has
caused the pneumothorax. The expectation is that the inadequate sensitivity and specificity to be useful for
rate of air accumulation in the pleural space through the detection of CBD stones.
the defect is slower than the rate of air removal,
thereby allowing the lung to expand. The vacuum is ~ Answer A. Uterine rupture may occur as a result
maintained until there is no longer any evidence of a of substantial maternal trauma. However, among
leak (therefore signifying that the defect has healed). typical pregnant women, the primary risk factor
The easiest way to test for the presence of a leak is to for uterine rupture is a prior cesarean delivery
ask the patient to cough and then examine the water (c-section), particularly with a "classic" vertical
seal chamber for air bubbling up through the column incision. Patients may present with significant
of water. Coughing increases intrathoracic pressure abdominal pain and vaginal bleeding or they may be
which forces air through the defect, if one is still relatively asymptomatic if only minor dehiscence has
present. In the case of a very small defect, there may occurred. A more recent meta-analysis suggests that
not be a detectable leak after correct tube placement only the risk of dehiscence is increased among women
Leaks can also be due to inadequate insertion of the with a prior cesarean section (excluding women with
chest tube such that some of the suction holes lie a classic vertical incision). Oxytocin did not increase
outside the pleural cavity. Alternatively, there may the risk of rupture. Emergent cesarian-section is
simply be a leak in the vacuum tubing or connectors. indicated in all cases of uterine rupture.
In these cases, however, the lungs will frequently fail
to expand as the suction capacity is "wasted" by ~ Answer E. The patient has failed outpatient ther-
continuously removing air from the limitless ambi- apy for pyelonephritis. The possible reasons for this
ent environment instead of the pleural space. This are noncompliance with antibiotics, resistant organ-
will be evident by virtue of a constant bubbling in ism, concomitant kidney stone, or renal abscess. The
the water seal chamber. The amount of water in the patient has had 2 weeks of an appropriate antibi-
water seal chamber has no effect on the amount of otic for pyelonephritis, which should be adequate
air escaping from the system. for clinical cure unless a kidney stone or renal
abscess is present-in this case, a CT scan is rec-
[j!] Answer B. IBS is a common, chronic GI illness ommended to evaluate these possibilities. Starting
characterized by abdominal pain or discomfort, a less broad-spectrum antibiotic such as TMP-SMX
bloating, and either constipation or diarrhea. Al- or metronidazole is not indicated, especially if no
though it is not a psychiatric diagnosis, patients with urine culture is performed beforehand. Extending
ms more commonly have a concomitant psychiatric the course of ciprofloxacin beyond 2 weeks is only
diagnosis (most commonly anxiety or depression). indicated in cases of male prostatitis.
The cause of IBS is not known, but it is not related
Test4 123

~ Answer B. Certain oral medications cause severe undercooked ground beef. However, outbreaks from
esophageal irritation when swallowed-doxycycline, contamination of apple cider, raw milk, and most
tetracycline, aspirin, and potassium chloride. Of recently, spinach, have also been reported. Antibi-
these, potassium chloride is the most caustic, otics are contraindicated in all cases because they
sometimes leading to esophageal perforation and may induce the expression and release of toxins
penetration into the mediastinal great vessels. (Shiga toxins), which may worsen the disease and
increase the risk of developing hemolytic-uremic
~Answer B. Diarrheaisthemostcommoncauseofa syndrome (HUS). HUS is a syndrome characterized
normal anion gap metabolic acidosis. Fluid from the by microangiopathic hemolytic anemia, thrombo-
intestine distal to the stomach is bicarbonate rich, so cytopenia, and renal failure and occurs in as many
diarrhea results in bicarbonate loss and subsequent as 25% of cases (most of which occur in children).
metabolic acidosis. As bicarbonate is lost, chloride HUS is the most common cause of renal failure
is avidly reabsorbed by the kidneys resulting in a in children. Thrombotic thrombocytopenic purpura
hyperchloremic metabolic acidosis. (TIP) is a less frequent complication of EHEC in-
fection, and more commonly occurs in the elderly or
liZ) Answer C. NEC is the most common GI immunocompromised Infection with EHEC results
emergency affecting neonates. However, because in a hemorrhagic colitis after an incubation period,
premature infants are predominantly affected, it is which ranges from 3 to 8 days. Fever is atypical and
a disease that is most commonly diagnosed in the a different pathogen should be considered if fever is
neonatal intensive care unit (NICU) and only rarely present.
seen in the ED. Interestingly, however, the age at
onset of NEC is inversely related to the gestational ~ Answer D . This patient experienced acute food
age and birth weight. Therefore, low birth weight poisoning due to Staphylococcus. The illness is not
infants who initially look well in the NICU may be caused by infection of the bacteria but by a heat-
discharged home before the development of NEC. stable enterotoxin produced by the bacteria before
Initial findings in NEC are nonspecific and include ingestion. Staphylococcus proliferates with ease in
ileus (multiple dilated loops of small bowel) or an foods with a high protein content, such as ham,
asymmetric bowel gas pattern. With progression of eggs, poultry, custard-based pastries as well as potato
the disease, however, air spreads through ulcerated GI or egg salads. The illness occurs 1 to 6 hours after
mucosal epithelium resulting in pneumatosis intesti- ingestion and is typically acute in onset. Abdominal
nalis or air within the biliary tract (portal venous). pain and vomiting are the most prominent symptoms
Pneumatosis intestinalis occurs in 75% of infants although occasionally a mild diarrhea may also
with NEC and is the most specific finding. be present. Symptoms are self-limited, typically
resolving within 8 hours and only rarely lasting
~ Answer B. The patient has clear clinical evidence for a full day. Because the disease is caused by a
of meningitis. Given the time course, it is unclear heat-stable toxin, cooking will not remove the toxin
whether the etiology is viral or bacterial, so a cautious once it is formed, and antlbiotics have no role in
approach should be taken. Ideally, the patient should treatment. Although BaciUus cereus causes a similar
have a lumbar puncture as soon as possible, but illness by virtue of a heat-stable toxin it produces,
antibiotics should not be delayed in such cases as it almost always occurs after ingestion of fried rice.
they do not appear to significantly affect culture Of note, B. cereus may produce a second, different
results in the first 4 hours oftherapy. Corticosteroids syndrome characterized primarily by diarrhea and
are now part of the standard of care for treatment abdominal pain. This latter syndrome results from
of suspected bacterial meningitis as they improve the production of a heat-labile toxin that is released
functional outcomes. The indications for CT scan in vivo after ingestion oflive organisms. It is clinically
before lumbar puncture are altered mental status, similar to food poisoning caused by Clostridium
focal neurologic deficit, suspected brain mass lesion, perfringens, and results from ingestion of meats or
and signs of increased intracranial pressure. MRI of vegetables colonized with the bacterium.
the brain has little role in the emergent evaluation of
meningitis. ~ Answer D. If they occur, 90% of recurrent peri-
tonsillar abscess (PTAs) develop within 1 year of
[ji) Answer B. E. coli 0157:H7 (also known as en- the initial infection with the majority occurring very
terohemorrhagic E. coli or enterohemorrhagic Es- shortly after the initial infection. Therefore, many
cherichia coli (EHEC]) is the most important strain of authors consider such recurrences an inadequately
E. coli that commonly causes diarrhea in the United treated initial infection, or simply a continuation of
States. It is most frequently associated with eating the same infection. Recurrence is more common in
124 1000 Questions to Help You Pass tM Emergency Medicine Boards

adults than in children although the reasons for this vaginal bleeding or they are noted to have a uterus
are not known. Although tonsillectomy, whether it that is too small for their dates. Subsequent US
is performed emergently or after the initial infection identifies intrauterine fetal demise before coagulation
has resolved, drastically reduces the rate of recur- complications occur and the patient is referred to an
rence, PTAs have been known to occur after the obstetrician for definitive management.
tonsils have been removed. The treatment ofPTAs is
surgical with adjunctive antibiotic therapy. Surgical ~ Answer B. In a well newborn without fever,
modalities include aspiration, incision and drainage, hyponatremia is the most common cause of seizures.
or tonsillectomy. Antibiotic therapy in the absence of Water intoxication is the most common cause of
surgical drainage is not effective. However, penicillin hyponatremia during infancy. Infants are unable
is the antibiotic of choice in these patients, whereas to adequately concentrate urine so parents who
erythromycin or other macrolides are used for pa- dilute formula or give their infants tap water put
tients with penicillin allergies. Adding metronidazole their infants at risk. Hypocalcemia is also a very
to the initial regimen may increase the success rate al- common cause of seizures in the neonate, so serum
though limited data is available regarding alternative calcium levels should be checked. In fact, all infants
antibiotic strategies. have a slight decline in serum calcium levels with a
nadir at 24 to 48 hours. Symptomatic hypocalcemia
~ Answer B. There are no universally accepted cri- is more common in infants of diabetic mothers,
teria for liver transplantation in patients with ac- preterm infants, or infants with a history of anoxic
etaminophen overdose. However, the King's College encephalopathy. Hypomagnesemia is not as common
criteria are the most widely used and accepted. The as hypocalcemia, but symptoms ofhypomagnesemia
King's College criteria recommends transplantation mimic those of hypocalcemia and it is difficult to
for: correct hypocalcemia if the serum magnesium is
(1) All patients with a pH <7.30 after adequate also low. Therefore, infants with seizures should
fluid resuscitation; or (2) Patients in whom all have a comprehensive evaluation oftheir electrolytes.
of the following occurs within a 24 hour period: Hypokalemia is uncommon in infancy and does not
(a) Creatinine >3.4 g!dL, (b) PT >100 s (INR typically cause seizures. Maple syrup urine disease is a
>6.5), (c) Development of grade III or N hepatic rare disease resulting from the inability to catabolize
encephalopathy. branched chain amino acids. Infants typically present
Without transplantation, only 15% of patients between 4 to 7 days of life with poor feeding,
who meet these criteria will survive. Since several pa- vomiting, or lethargy. Neurologic manifestations
tients who meet the criteria die before a transplanted rapidly develop, such as alternating hypotonia and
organ becomes available or are too ill for transplant, hypertonia, dystonia, seizures, and encephalopathy.
there is a need for earlier identification of high risk
patients. To this end, a recent study suggested that the ~ Answer C. The patient has acute mastoiditis,
King's College criteria be modified to include an ar- resulting from partially treated otitis media. Up
terial blood lactate level > 3.5 mmoVL within 4 hours to 10% of mastoiditis cases have normal tympanic
of admission or a level of >3.0 mmoVL after ade- membranes on examination. Acute mastoiditis is
quate fluid resuscitation (12 hours after admission). most commonly due to S. pneumonitze; chronic
The authors suggested that the addition ofthe lactate mastoiditis is usually due to Pseudomonas. CT scan
criteria increases the detection of high risk patients of the mastoid area may be useful in making
and improves the chances for early transplantation. the diagnosis, and management generally involves
admission for N antibiotics and surgical consultation
~ Answer D. The absence of fetal heart tones in for possible mastoidectomy.
an intrauterine gestation with a crown-rump length
> 5 mm (correlates roughly to 6.2-weeks' gestation) ~ Answer A. Patients with pill esophagitis present
is convincing evidence of fetal demise. Missed with a history of odynophagia and severe, constant
abortion refers to the continued presence of a retrosternal pain. The pain of esophagitis may be
nonviable fetus aged <20---weeks' gestation for at confused with pain due to myocardial infarction.
least 8 weeks without the passage ofmaternal or fetal However, detailed history taking should reveal that
tissue. The persistent presence of a dead fetus may the pain worsens with swallowing. In addition,
result in coagulation abnormalities and disseminated patients may reveal that they took their medicines
intravascular coagulation. However, this is primarily without liquids or did not remain upright for a
a historical diagnosis due to the prevalence of US. sufficient amount of time after taking their pills. All
Therefore, most women present to a physician with of the medicines listed have been reported to cause
vague complaints of crampy abdominal pain or pill esophagitis. However, alendronate is the most
Test4 125

common cause. Doxycycline is the most common nerve testing reveals completely normal function of
antibiotic resulting in pill esophagitis. the trigeminal nerve and remaining cranial nerves.
Patients typically have normal dentition. In addi-
l!l] Answer D. Patients with a combination of hy- tion, patients with trigeminal neuralgia typically have
potension and pelvic fracture have extremely high "trigger zones" especially in the perioral area and
mortality and should be managed aggressively. Ini- near the nostril. Tapping or palpating these areas
tial stabilization involves securing the pelvis with a may provoke an attack. Other stimuli such as wind
bedsheet or tightening device to reduce the volume on the face, chewing, brushing teeth, and shaving
into which hemorrhage can occur and copious may also provoke attacks of pain.
crystalloid and blood resuscitation. If the patient
stabilizes, further evaluation with CT scan may be [tOOl Answer E. This patient is at risk for aspiration
performed to better delineate the injuries. However, pneumonia by virtue ofhis decreased airway protec-
ifthe patient remains unstable, angiographywith em- tion and tube feeds. Although there is no definitive
bolization should be performed to limit the extent way to diagnose aspiration pneumonia, this patient's
of hemorrhage. As a general rule, hemodynamically history of witnessed vomiting with choking in the
unstable trauma patients should never have CT scans setting of a patient at risk for aspiration is virtually
performed. diagnostic for aspiration. Aspiration pneumonitis is
caused by an inflammatory response in the lungs after
~ Answer B. Although handlebar injuries are simply the aspiration oflow pH gastric contents. Aspiration
a form of blunt abdominal trauma, which therefore pneumonitis develops within hours of aspiration,
put patients at risk for liver and spleen injuries, there and its severity is directly related to the volume
is an increased risk of pancreatic and small bowel and pH of the aspirated material. Treatment is sup-
injuries. Classically, pancreatic or duodenal injuries portive, and antibiotics and steroids are not useful.
are associated with pediatric handlebar injuries. Pa- However, episodes of aspiration pneumonitis may
tients with pancreatic injury often develop delayed become secondarily infected, resulting in aspiration
symptoms and may have a relatively benign presen- pneumonia. More commonly, aspiration pneumonia
tation initially. Eventually, they develop abdominal develops without significant preceding pneumonitis.
pain, nausea, and vomiting and have evidence of Such infections are due to the aspiration of oropha-
pancreatic injury by elevated enzymes on laboratory ryngeal flora and are most commonly polymicrobial.
analysis. Acute dosed loop small bowel obstruction While anaerobes have classically been implicated in
and rupture may also occur with handlebar injuries. cases of aspiration, anaerobic antibiotic coverage
Owing to the lack of significant blood loss, patients may actually only be needed in patients with very
may again be relatively asymptomatic. Because the poor oral hygiene, putrid sputum, or evidence of
small bowel contains relatively little air, up to 85% necrotizing pneumonia or lung abscess chest x-ray.
of cases will have a normal upright abdominal film Metronidazole is a specific therapy for anaerobic
(i.e., no free air) and 50% will have no signs of peri- infections, but its use as monotherapy in aspiration
tonitis on examination. Therefore, physicians must pneumonia is associated with a high failure rate.
maintain a high suspicion for injury in cases of The fluoroquinolones are broad spectrum agents
pediatric handlebar trauma. and they achieve good tissue levels. However, they
do not need to be combined with corticosteroids,
[!!) Answer D. The pain of trigeminal neuralgia is for which there is no role in the setting of aspi-
characteristically explosive in onset, severe in in- ration, despite their wide use. Aztreonam does not
tensity, brief, lasting 2 seconds to 2 minutes, and have any gram positive coverage so it should not be
unilateral. It invariably involves either the V2 or V3 used as monotherapy. First-line antibiotic therapy in
branch of the trigeminal nerve and sometimes in- nursing home residents with aspiration pneumonia
volves both branches. It very rarely affects the VI may include piperacillin-tazobactam, levofloxacin,
(ophthalmic) branch ofthe trigeminal nerve. Cranial or ceftazidime.
Test 5
Questions
[!] A 44-year-old woman presents with right wrist pain. (D) The patient should be given bicarbonate
She fell on her hand the previous day and has replacement therapy.
pain in her radial wrist. The wrist is tender in the (E) A repeat arterial blood gas (ABG) should be
anatomic snuffbox. Plain radiographs of the wrist performed to guide further therapy.
are completely normal. Which of the following is the
most appropriate next step in management? [1J A 6-year-old girl presents with abdominal pain.
She has had moderate, constant periumbilical pain
(A) Orthopaedics consultation
for several hours with associated nausea. Her
(B) Discharge home with thumb range of motion
parents noted a rash on her legs and buttocks
exercises
for several days, which they attributed to poison
(C) Discharge home with Velcro wrist splint
ivy. Physical examination demonstrates an afebrile,
(D) Discharge home with thumb spica splint
uncomfortable patient, diffuse abdominal tenderness
(E) Admit for observation ofwrist
without true rebound or guarding, a maculopapular
rash on the legs and buttocks, and diffuse joint
[!] A 35-year-old woman presents in a coma (Glasgow
tenderness. Which of the following is the most
Coma Scale [GCS] 3) after a motor vehicle crash
appropriate next step in evaluation?
(MVC) and is intubated for airway protection.
Further evaluation reveals no life-threatening chest, (A) Urinalysis
abdomen, or pelvic injuries. Vital signs are normal. (B) CT scan of the abdomen/pelvis
A computed tomography (CT) scan is performed (C) Anti-streptolysin 0 antibodies (ASO) titer
and is normal. Which of the following is the most (D) Blood cultures
likely diagnosis? (E) Meckel scan
(A) Epidural hematoma
(B) Subdural hematoma
W Which of the following is true regarding acute
mesenteric ischemia (AMI)?
(C) Diffuse axonal injury (DAI)
(D) Cerebral contusion (A) Most patients have a lactic acidosis early in the
(E) Intraparenchymal hematoma course of their illness.
(B) The mortality rate of acute mesenteric ischemia
IIJ Upon starting your shift, you receive sign-out on (AMI) is roughly 70%.
a 46-year-old male diabetic patient who is being (C) In the absence of angiography, intravenous
treated with an intravenous insulin drip for dia- heparin infusion is the standard of
betic ketoacidosis (DKA). Two hours later, a repeat management.
chemistry panel reveals the following: Na+141 mEq (D) Tenderness on physical examination is most
per L, CI-112 mEq per L, HC03-17 mEq per L, often worse than a patient's subjective
blood urea nitrogen (BUN) 16, creatinine 0.9, glu- complaint of pain.
cose 278 mg per dL. Which of the following is (E) The most common CT finding is gas in the
true? portal venous system.
(A) The patient has a mixed high anion gap (AG)
ketoacidosis and nonanion gap hyperchloremic
W A 32-year-old pregnant woman in her third trimester
presents with dysuria. She has a penicillin allergy.
metabolic acidosis (HCMA).
Urinalysis demonstrates bacteria, but no white
(B) He should be given subcutaneous insulin and
blood cells, leukocyte esterase, or nitrites. Physical
his insulin infusion can be discontinued after 30
examination is normal. Which of the following is the
to 60 minutes.
most appropriate next step in management?
(C) His diabetic ketoacidosis (DKA) is not yet
resolved and he requires an ongoing (A) dose outpatient observation
intravenous insulin infusion. (B) Trimethoprim-sulfamethoxazole

126
Tm5 127

(C) Ciprofioncin (A) Perform an immediate therapeutic


(D) Nitrofurantoin thoracentesis.
(E) Amoxicillin (B) Perform a diagnostic thoracentesis to rule out
empyema.
(!) Which ofthe following is the most common cause of (C) Administer oxygen, nitrates, and furosemide.
acute respiratory distress syndrome (ARDS)f (D) Perform rapid sequence intubation.
(A) Sepsis (E) Order a B-type natriuretic (BNP) level in order
(B) Near drowning to determine the beat management.
(C) Multiple blood transfusions
(D) Multiple blunt trauma (!g) The most common cause of an intracranial tumor is
(E) Pancreatitis (A) Meningioma.
(B) Astrocytoma.
(!] A 70-year-olcl woman presents with blurry vision (C) Medulloblastoma.
in her right eye and a right-sided headache. The (D) Metastases.
patient also complains of malaise, and is tender in (E) Pituitary adenoma.
her right temporal region. Which of the following is
true regarding this patient's illness? Ill) Which of the following is true regarding heart
(A) Steroids are ineffective in preventing disease in transplant recipients?
the contralateral eye. (A) The resting heart rate is decreased from their
(B) Patients have a higher risk ofarterial aneurysms. pretransplant baseline.
(C) Steroid therapy should be witbhelcl until (B) Tamponade cannot occur in a transplanted
definitive diagnosis is made. heart.
(D) Men are more commonly affected. (C) Acute rejection is usually cfia8nosed by
(E) The disease is very rare in patients older than endom:yocardial biopsy.
50 years ofage. (D) There is no increased risk of endocarditis with
invasive procedures.
(!) A 62-year-old woman presents with hypertension, (E) The heart rate increases only minimally with
diabetes mellitus, and emphysema with a 3-week exercise or stress.
history ofdyspnea on exertion, three-pillow orthop-
nea, and bilateral lower extremity edema. She denies (!j) Whichofthefollowingi.strueregardinghipfractures?
fevers, cough. or cheat pain. A chest x-ray is shown in (A) The highest incidence ofavascular necrosis
Figure 5-lA and B. The patient's electrocardiogram (AVN) occurs in patients with intertrochanteric
(EKG) does not show any ischemic changes. Her fractures.
'Vitals are: Respiratory rate 22 per minute, pulse 108, (B) Isolated fractures of the lesser trochanter are
BP 154/88, pulse oximetry 88% on room air. What is most commonly seen in young adults.
the nat best step in management?

A B
Figure 5-1.
128 1000 Questions to Help You Pass tM Emergency Medicine Boards

(C) Femoral nerve block is contraindicated in the (A) Albuterol


setting of a hip fracture. (B) Indomethacin
(D) The development of acute respiratory distress (C) Ribavirin
in a patient with an acute femoral shaft fracture (D) Cefotaxime
but no other trauma is most likely due to a (E) Propanolol
pulmonary embolism (PE).
(E) All of the above. [1!) Which of the following is true about appendicitis in
pregnancy?
(!j) Which ofthe following is the most common cause of (A) Leukocytosis may be physiologic during
large bowel obstruction? pregnancy.
(A) Malignancy (B) Appendicitis is more common in pregnant than
(B) Sigmoid volvulus in nonpregnant women.
(C) Adhesions (C) Ultrasonography has a sensitivity of
(D) Diverticular disease approximately 40% for appendicitis during
(E) Fecal impaction pregnancy.
(D) Appendiceal rupture is most common during
~ Which ofthe following is the most common cause of the first trimester.
dysuria? (E) All of the above.
(A) Bacterial infection
(B) Viral infection ~ A 77-year-old man with type 2 diabetes presents
(C) Fungal infection with 4-day history of progressively worsening left ear
(D) Parasitic infection pain, hearing loss, and discharge. On examination,
(E) Allergic urethritis he has a temperature of 101°F, he appears fatigued,
and his tympanic canal is markedly edematous with
~ Which of the following is the most sensitive physical foul drainage. His glucose level is 400. Which of the
examination test for an anterior cruciate ligament following is true regarding this condition?
(ACL) tear? (A) IV ciprofloxacin and ENT consultation are
(A) Anterior drawer indicated.
(B) Posterior drawer (B) Streptococcus pneumonitle is the single most
(C) Lachman likely pathogen.
(D) McMurray (C) Cranial nerve involvement almost always begins
(E) Thompson with the abducens nerve.
(D) Sinus x-ray may be warranted to evaluate extent
~ The most common cause of sexually transmitted of disease.
disease in the United States is (E) Antiviral therapy should empirically be started.
(A) Trichomonas vaginalis.
(B) Chlamydia trachomatis. ~ A 32-year-old G3P2 at 39 weeks' gestation presented
(C) Treponema pallidum. to your community ED after spontaneous rupture
(D) Neisseria gonorrhoeae. of membranes and with regular uterine contractions
(E) Candida spp. roughly 3 minutes apart. Although the fetal head
delivers without difficulty, the shoulders appear to
(!!] Which of the following is the most common toxicity be trapped and you suspect shoulder dystocia. The
associated with cyclosporine? next best step in management should be
(A) Hyperuricemia and gout (A) Midline episiotomy.
(B) Hyperlipidemia (B) McRobert maneuver.
(C) Nephrotoxicity (C) Suprapubic pressure.
(D) Hepatotoxicity (D) Rubin maneuver.
(E) Hypertension (E) Wood corkscrew maneuver.

(!jJ A 1-week-old infant is brought to the emergency ~ A 2-year-old boy is brought in by his parents with a
department (ED) with cyanosis. Room air pulse rash on his trunk. On examination, you discover a
oximetry is 88% and the patient appears to have rose-colored maculopapular rash on his chest, neck,
decreased activity. Chest x-ray reveals a slightly en- and arms. The patient is currently afebrile, but the
larged heart with increased pulmonary consolidation. parents tell you that the patient was seen by one of
Which of the following medications is indicated?
Tm5 129

your colleagues yesterday for a febrile seizure. Which (D) Nasopalatine branch ofsphenopalatine artery
of the followins is the most likely diagnosis? (E) Septal branch of superior labial artery
(A) Roseola infantum
(B) Rubeola [ilJ A 5-day-old neonate presents for routine follow-up.
(C) Rubella The child is acting normally. Funduscopic exami-
(D) Erythema infectiosum nation demonstrates bilateral retinal hemorrhages.
(E) Scarlet fever Which of the following is the most likely cause?
(A) Child abuse
~ A 9-year-old boy presents with fever, sore throat, (B) Accidental fall
and refusal to eat or drink because of .severe (C) Normal birth trauma
odynophagia. His oropharyngeal examination is (D) Intracerebral hemorrhage
shown in Figure 5-2. Which of the following is the (E) Congenital finding
most likely etiology!
{ijJ A 26-year-old primigravida is diagnosed with a
spontaneous abortion at 6 weeks' gestation. She
asks you, "why did this happen?.. Which of the
followins is the most common cause offirst-trimester
miscarriage?
(A) Fetal chromosomal trisomy
(B) Uterine structural abnormality
(C) Maternal stress
(D) Minor trauma, e.g., falls
(E) Cigarette smoking

li!) Which of the following is the most specific EKG


Figure 5·2. finding in acute pericarditis~
(A) Concave ST elevations
(B) Convex ST elevations
(A) Aphthous stomatitis
(C) PR depressions
(B) Streptococcus pyopw
(D) Hyperacute T waves
(C) Corynebacterium diphthmae
(E) Primary atrioventricular (AV) bloc:k
(D) CoxsackieYirus
(E) Herpes simplex 'Virus
~ A 34-year-old man with a history of schizophrenia
is brought to the ED by police with acute agitation.
~ In a perilunate dislocation, which bone is dorsaRy He was reported to be threatening passersby on the
dislocated?
street. He now begins to threaten sbdt stating that
(A) Lunate he will kill anyone who comes near him, and starts
(B) Scaphoid to swing punches at people standing near him. The
(C) Capitate patient is physically restrained by security staff and
(D) Hamate secured to a cart in four-point restraints. He is still
(E) Pisiform yelling at the top of his lungs and struggling against
the restraints. Which of the following is the most
~ Exciaion of thrombosed hemorrhoids is not recom- appropriate next step in management?
mended or effective after: (A) Observe for 4 hours and discharge when calm.
(A) 12hours (B) Observe for 1 hour and repeat history and
(B) 24hours physical aamination.
(C) 48hours (C) CT scan of the brain without contrast.
(D) 96hours (D) Administer haloperidol, lorazepam, and
(E) 1 week benztropine.
(E) Perform rapid sequence intubation.
(ij) What is the most common source of bleeding in an
anterior nosebleed? ~ A 44-year-old man is struck on the head with a
(A) Anterior ethmoidal artery baseball bat. A CT scan of the brain is shown in
(B) Posterior ethmoidal artery Figure 5-3. Which of the following is the most likely
(C) Kiesselbach ple:ms diagnOSlS.
. '
130 1000 Quatioru to Htlp You Ptw tile~ MetliciM BoGrr1s

Figure 5-4.

(C) There is a high rate of nonunion of the involved


fracture.
Figure 5-l. (D) The peronew longus tendon insert& at the site
of the fracture.
(E) This patient should be allowed to bear weight as
tolerated.
(A) Epidural hematoma
(B) Subdural hematoma ~ Which of the following is true regarding gonococcal
(C) Subarachnoid hemorrhage aeptic arthritis?
(D) Cerebral contusion (A) Open surgical drainage is usually required.
(E) Diffuse axonal injury (DAI) (B) It is more common in men than in women.
(C) Genitourinary symptoms occur in most
~ Which of the following is true regarding elder abuse? patients.
(A) Victims of elder abuse die earlier than elders (D) Synovial fluid Gram stain is positive more often
who are not abused. than c:ulture.
(B) Men are at hlgher risk than women. (E) The hip is the most common joint afkcted.
(C) Sexual abwe is the most common type of elder
abuse. ~ The jointa most commonly affected by decompres-
(D) Most elder abuse reporting is performed by aion sickness (DCS) are
physic:iana. (A) Ankles and feet.
(E) Most perpetraton are strangers. (B) Knees.
(C) Hips and axial skdeton.
~ Which of the following is the preferred imaging (D) Shoulders and elbows.
modality to diagnose a parapharyngeal abscess in the (E) Wrim and hands.
Ern
(A) Lateral neck x-ray !llJ A 27-year-old man is hammering nails at a construc-
(B) Anterior-posterior neck x-ray tion site without eye protection and feels something
(C) crscan of the neck strike his right eye. After washing out the eye, he still
(D) Magnetic resonance imaging (MRI) of the neck complains of pain and presenta to the ED. Which of
(E) Ultrasonography of the neck the following is the safest and most accurate modality
for locating the potential foreign body!
~ Which of the following is true regarding the image (A) X-ray
shown in Figure 5-4? (B) Ultrasonography
(A) The image depicts a Jones fracture. (C) MRI
(B) Jones fractures and avulsion fractures are (D)Cfscan
typicaiiy cawed by a direct blow to the foot. (E) Nuclear medicine scan
Test5 131

~ Which of the following is a risk factor for develop- ~ Which of the following patients has a contraindi-
ment of kidney stones? cation to the use of iodinated radiographic materi-
(A) Female gender als (RCMs)?
(B) Hypoparathyroidism (A) 24-year-old woman with asthma
(C) Crohn disease (B) 60-year-old man on atenolol
(D) Hyperthyroidism (C) 37-year-old woman with a shellfish allergy
(E) Diabetes mellitus (D) 8-year-old boy with atopic dermatitis
(E) None of the above
~ Which of the following is the most common
dysrhythmia in pediatric patients? ~ A 35-year-old man with history of sickle cell disease
(A) Atrial fibrillation presents with acute onset of fever, malaise, fatigue,
(B) Atrial flutter and lightheadedness. Physical examination demon-
(C) Complete heart block strates a tachycardic patient with pale conjunctivae.
(D) Paroxysmal supraventricular You suspect aplastic crisis and draw a complete
tachycardia (PSVT) blood count with reticulocyte count. You review
(E) Ventricular tachycardia his old records and note that the patient's baseline
hemoglobin level is 8 g per dL. Which of the follow-
~ Which of the following is true regarding reduction ing laboratory abnormalities is most consistent with
of an anterior shoulder dislocation? an aplastic crisis?
(A) The Kocher maneuver is the most reliable (A) Hemoglobin 8 g per dL, reticulocyte count 6%
method. (B) Hemoglobin 8 g per dL, reticulocyte count 1%
(B) Adequate muscle relaxation is the most (C) Hemoglobin 6 g per dL, reticulocyte count 6%
important factor in successful reduction. (D) Hemoglobin 6 g per dL, reticulocyte count 1%
(C) The Hippocratic method should be the first one (E) Hemoglobin 4 g per dL, reticulocyte count 6%
attempted.
(D) Scapular manipulation is the method of choice ~ Which of the following is true regarding inflamma-
in third trimester pregnant patients. tory bowel disease?
(E) Intra-articular anesthetic injection is (A) Toxic megacolon is more common in patients
contraindicated. with Crohn disease than ulcerative colitis.
(B) Perianal complications are most common in
~ The normal anion gap (AG) is primarily due to which patients with Crohn disease.
of the following? (C) Crohn disease always involves the rectum.
(A) Phosphate (D) Erythema nodosum is most common in male
(B) Albumin patients with ulcerative colitis.
(C) Sulfate (E) Anal fissures in patients with Crohn disease
(D) Citrate tend to be located in the posterior midline.
(E) Acetone
~ A 42-year-old woman presents with a chiefcomplaint
~ Which of the following is true regarding lum- of right hand numbness. Decreased sensation in the
bar puncture in patients with Guillain-Barre syn- volar aspect ofthe little finger indicates a problem in
drome (GBS)? which of the following?
(A) Campylobacter jejuni can often be visualized in (A) Radial nerve
the cerebrospinal fluid (CSF) of patients with (B) CS nerve root
GBS. (C) Ulnar nerve
(B) The classic CSF finding is an elevated protein (D) Median nerve
level with a normal number ofwhite blood cells. (E) C6 nerve root
(C) Lumbar puncture is necessary to make a
diagnosis of GBS. ~ Which of the following is the strongest environ-
(D) Lumbar puncture is contraindicated in the mental risk factor for the development of pancreatic
acute phase of GBS due to elevated intracranial cancer?
pressure (ICP). (A) Alcohol use
(E) The finding of oligoclonal bands is (B) High fat diet
pathognomonic for GBS. (C) Excessive coffee drinking
(D) Living in a cold weather environment ~ Which of the fo11.owiq is the most common cause of
(E) C:igarctte smoking nongonococcal urethritis?
(A) Un:t~plasma
~ A 38-year-ol.d mildly obese primigravida at 34 weeks'
(B) Tri,homonGS
gestation presents with a dlief complaint of"swoUen
(C) Hemophilus ducreyi
legs" and abdominal pain. Her blood pressure is
(D) C. trachomatis
170/100 and abe baa 3+ protein on urine dipstick.
(E) Herpes simplex virus
After giving her magnesium for prophyl.u:i.s of her
seizures and hydralazine for blood pressure control.
~ What is the moat common source of bleeding in a
the nurse tells you that her urine output seems
posterior nosebleed?
a bit low, and asb you what you want to do
about her signi1icant edema. The next best step in (A) Anterior ethmoidal artery
management is (B) Posterior ethmoidal artery
(C) Kiesselbach plCI.US
(A) Furosemide 40 mg IV push.
(D) Nasopalatine branch of sphenopalatine artery
(B) Maintenance intravenous fluids.
(E) Septal branch of superior labial artery
(C) HydrochlorO'I:bW.ide 25 mg orally.
(D) Mannitol 0.5 mg per kg IV push.
(E) 25% albumin given intravenously at 1 g per kg.
~ A 9-year-old girl is brought in to the ED with a chief
complaint of a palpable, nonblanching rash, achy
knee and ankle pajn, and colicky abdominal pain
~The "discriminatory zone" is the ~uanti~ve
(see Fig. 5-5). Which of the following is true?
serum fi-hCG level at which a normal mtrauterine
gestation should be seen on ultrasonography. The
discriminatory zone for tran5V88inal ultrasono-
graphy is
(A) <500 miU per mL fi-hCG.
(B) 500 to 1,000 mill per mL fi-hCG.
(C) 1,000 to 2,000 miD per mLP-hCG.
(D) 2,000 to 3,000 miU per mL P-hCG.
(E) >3,000 miU per mL fi-hCG.

~ Which of the following is true regarding diagnosis of


kidney stones?
(A) Normal urinalysis e.:ntiallyrules out the
diagnoais.
(B) KUB radiograph has >90% specificity.
(C) Most kidney stones are radiolucent.
(D) Ultruonography has >90% sensitivity.
(E) cr scan baa roughly 90% sensitivity and
specificity.

~ The most common manifestation of barotrauma


associated with scuba divers during descent is
(A) Nitrogen narcosia.
(B) Barosinusitis.
(C) Temporomandibular joint dysfunction.
(D) Facial barotrauma.
(E) Middle ear barotrauma.

~ Which ofthe foDowing is a potential complication of


Bordettll4 pertussis infection?
(A) Pneumonia Ffgure 5-5.
(B) Subconjunctival hemorrhage
(C) Pneumothorax
(D) Otitis media (A) Renal involvement, with progression to renal
(E) AU of the above failure, is common.
TestS 133

(B) Most patients have thrombocytopenia. ~ A complaint that may help d.ifl"uentiate patients with
(C) Treatment i& with high-dose aspirin. Crohn disease from patients with irritable bowel
(D) Intuaswception complicating thi& .illness i& syndromei&
usually ileoileal. (A) Noc::turnal diarrhea.
(E) Most patients require inpatient
(B) Bloating.
immunosuppressive therapy.
(C) Weight loss.
(D) Colicky abdominal pain.
~ Which of the foUowing is the most useful treatment (E) Bilious vomiting.
modality in patients with heat stroke?
(A) Spraying tepid water on the patient and fanning ~ A 4-year-old boy is brought to your office by par-
the patient ents with a fever to l02°P over the last 2 days,
(B) Intravenous dantrolene malaise, and complaint of right ear pain. Examina-
(C) Applyins ice packs to the uilla, groin, and tion reveals an active, febrile child with a bulging
extremities right tympanic membrane (TM) minimally mobile
(D) Delivery ofcooled intravenous saline on insufllation. Which of the following i& the most
(E) Tylenol or ibuprofen delivered per rectum likdy pathogenl
(A) Mycoplluma pneumonUie
~ Which of the foUowing is true about volvulus?
(B) S. pyogenes
(A) Plain films are more often diagnostic in case& of (C) Moraxell4 aztarrludis
cecal volvulus than sigmoid volvulus. (D) Hemophilus injlumzlle
(B) Reduction of cecal volvulus is beat achieved (E) Adenovirus
endoscopically.
(C) One of the classic radiographic findings in cecal ~ A 27 -~ar-old man is brought to the ED after a motor
volvulus is a massively dilated cecum located in vehicle accident complaining of right arm pain. Hi&
the left upper quadrant.
(D) Cecal volvulus is more rommon than sigmoid
volvulus.
(E) Cecal volvulus is most common in patients
aged 25 to 35 years.

~ Which ofthe following i& true about .iachemic colitis?


(A) The mortality rate is similar to that for acute
ischemia of the small bowel.
(B) "Tbumbprinting'' on plain radiographs is seen
in only 50% ofcases.
(C) The disease is uaua1ly isolated to the cecum.
(D) The diagnosis ofcolon .i.sche:mia is commonly
confused with ulcerative colitis.
(E) Angiography should be performed as soon as
the diagnosis is considered.

~ Which of the following is true regarding viral


hepatitis?
(A) Ten percent of adult patients infected with
acute hepatitis A become chronic carriers.
(B) Pregnant women infected with acute hepatitis E
are more susceptible to fuhninant hepatitis.
(C) Children infected with hepatitis A virus devdop
symptoms more often than adults.
(D) Hepatitis B is the most common cause of viral
hepatitis worldwide.
(E) Although hepatitis A primarily undergoes
fecal-oral tranamisaion, percutaneous
transmission occurs at similar rates to
hepatitis B.
134 1000 ~ tDHdp YouPo~W the~ MetlidMBoGrds

arm is swollen and shortened compared with the chicken pox. There is no reported cough, shortness of
unaffected side. His x-ray reveals the image shown breath, headache, or stiff neck. Physical examination
in Figure 5-6. Which of the following associated demonstrates a nontoxic, playful child with a crop of
findings is most likely to be present upon physical vesicles in one area and dried crusted lesions in other
examination? areas. Which ofthe following is the most appropriate
(A) Inability to extend the wrist next step in management?
(B) Loss of two-point disaimination at the volar (A) Aspirin
tip of the index. :finger (B) Acetaminophen
(C) Absence of a radial pulse (C) IV acyclovir
(D) Inability to oppose the thumb (D) VariceUa-Zostenaccine
(E) Hypoesthesia of the palmar aspect of the little (E) VariceUa-Zoster immune globulin
and ring fingers
(iii A 46-year-old man presents with chest pain 8 hours
~ Which of the following is the narrowest portion of after having upper endoscopy for dysphagia (see
the pediatric airway? Fig. 5-7). Esophageal perforation is suspeaed and
(A) Nasopharynx a screening chest x-ray is ordered. Which of
(B) Oropharynx the following is likely to be found on physical
(C) True cords examination?
(D) False cords
(E) Cricoid cartilage

~ Which of the following is true regarding tinea


versicolor?
(A) Recurrence oflesions after treatment is
common.
(B) Swilight accelerates repigmentation.
(C) Griseofulvin is inactive against the disease.
(D) The upper trunk is the most commonly affected
area.
(E) All of the above.

(ii) Which of the following is true regarding thyroid


storm?
(A) Glucocorticoids may be useful in management.
(B) Apathetic thyrotcmcosis is an uncommon
presentation that typically occurs in children.
(C) Most patients have underlying Hashimoto
thyroiditis. Figure 5-7.
(D) Atrial fibrillation is the most common
dysrhythmia.
(E) All of the above.
(A) Dif£usely diminished heart sounds
(ii) Which of the following is the most common (B) Deaeased breath sounds at lung bases
complication of anterior shoulder dislocation? bilaterally
(C) Shock
(A) Axillary nerve injury (D) A crunching sound upon cardiac auscultation
(B) Brachial artery injury (E) A rigid, tender abdomen
(C) Reamence
(D) Rotator cuff tear
(E) Adhesive capsulitis
(i!) A 23-year-old man presents with right ankle pain. He
twisted it playing basketball and has been unable to
walk. Physical examination demonstrates tenderness
(i!) A 5-year-old girl without past medical history in his medial malleolus and prmimal fibula_ Radio-
presents with fever and pruritic rash on several areas graphs demonstrate a nondisplaced medial malleolar
of her trunk. The parents report a sick contact with fracture with nondisplaced pnmmal fibular fracture.
similarsymptomsatschooland thinktherashmaybe
TII!St5 1.35

Which of the following is the most appropriate next (A) The disease is caused by a gram-negative
step in management? coccobacillus.
(A) MRI of the knee (B) Antibiotic therapy should eliminate the
(B) MRI of the ankle symptoms within a rew days.
(C) Stress x-ra}'l of the ankle (C) Bacterial culture is in.dicated to confirm the
(D) DUdw:ge with knee immobilizer and diagnosis.
nonweight bearing (D) The disease is not contagious
(E) Discharge with Aircast (E) Mortality is close to 30%.

~ Which of the following is true regarding cerebral [8) Which of the following is true regarding viral
venous thrombosis (CVT)? hepatitis?
(A) CT with IV contrast is the current "gold (A) Adult patients infected with acute hepatitis B
standard" for diagnosis of CVT. are more likdy to become chronic carriers than
(B) The moat common presentation is lethargy. patients infected with acute hepatitis C.
(C) In most cases the outcome of CVT is worse (B) Hepatitis Cis most commonly acquired
than arterial stroke. through saual intercourse with an .infected
(D) Men are more commonly affected than women. person.
(E) Most seizurea that occur are focal seizures. (C) Hepatitis C .is the most common viral cause of
fulminant hepatic failure.
(!!) A 19-year-old boy presents with acute onset of right (D) Both direct and indirect bilirubin are typically
testicular pain and nausea for 5 hours. Physical elevated in roughly equal amounts.
examination reveals a markedly swollen and tender (E) Leukocytosis is a harbinger offulminant
testis in a horizontal lie with absent right cremasteric hepatic fiillure.
refl.eL Which ofthe following is the most appropriate
definitive ~Ill~Qgement? ~ A 24-year-old man is brousJlt to the ED for altered
mental statw. A friend states that they were eating
(A) Antibiotics
the seeda of the fruit shown in Figure 5-8. Vital signs
(B) Analgesia
are 99.6°F, 100, 18, 156/94, 98% RA. The patient
(C) Manual detorsion
is agitated, reaponda only to name, but has intact
(D) Operative orchidopexy
airway reflexea. Which of the following is the most
(E) Extracorporeal shockwave lithotripsy
appropriate nat step in management?
~ Which of the following is true about fungal pul-
monary disease?
(A) Most casea of primary pulmonary infection
with Himplluma capsulatum are undetected
and re&Olve without treatment.
(B) Cryptococcw neofomums, Blasromyces
dermutiditis, H. c:apsulahlm, and Cocddioides
imitis all have a specific geographic distribution.
(C) Patients with fungal pneumonia are generally
contagiow to other patients.
(D) Fungal pneumonia& generally cause acute,
self-limiting illness in healthy patients.
(E) C. imitis pulmonary infection most commonly
results in disseminated disease. filure s..a.

~ A 24-year-old woman presents with persistent cough


for 4 weeks. She had upper respiratory infection (A) Atropine
(URI)-like symptoms 2 weeks before and then (B) Pyridostigmine
developed a persistent cough for the nat month. (C) Edrophonium
She states she lw had coaghina fits many times (D) Pralidomne
during the day and "couldn't stop coughing'' for (E) Supportive care only
almost a minute when she started. Which of the
following is true regarding this patient?
13 6 1000 Questions to Help You Pass the Emergency Medicine Boards

~ A 22-year-old woman presents with acute onset of (B) Gonococcal and chlamydia) infection rarely
right eye discharge 3 hours before presentation. The coexist.
discharge reaccumulates almost immediately after (C) Gram-negative intracellular diplococci on urine
wiping it away. She does not wear contact lenses. Gram stain indicate Escherichia coli infection.
Physical examination demonstrates normal visual (D) Chlamydial infection is more common in men
acuity and copious greenish-yellow discharge in the older than 35 years.
right eye. Conjunctival injection and chemosis are (E) First-line therapy is with amoxicillin.
prominent. Which of the following is the most
likely cause? ~ Which of the following is the most appropriate test
in a patient with an intermediate probability of a PE
(A) S. pneumoniae
by Wells criteria?
(B) H. inftuenzae
(C) M. catarrhalis (A) Lung scintigraphy
(D) Klebsiella pneumoniae (ventilation-perfusion[V/Q] scan)
(E) N. gonorrhoeae (B) o-dimer
(C) CT pulmonary angiography
[!lJ The most common cause of acute liver failure is (D) Lower extremity Doppler ultrasonography
(E) Arterial blood gas
(A) Acetaminophen toxicity.
(B) Idiopathic.
(C) Hepatitis B virus infection.
~ Which of the following is true regarding cerebral
edema after head trauma?
(D) Amanita phalloides ingestion.
(E) Reye syndrome. (A) Vasogenic edema usually occurs in the gray
matter.
~ A 56-year-old man with a history of uncontrolled (B) Vasogenic edema occurs because of membrane
hypertension is brought in to the ED unresponsive pump failure.
after being found slumped over the steering wheel (C) Cytotoxic edema develops when cerebral blood
of his car. His BP is 245/130, his HR is 62 and he flow is reduced.
has irregular breathing. On examination, you note (D) Cytotoxic edema rarely occurs in the absence of
that his left pupil is fixed and dilated. What does this vasogenic edema.
physical examination finding likely indicate? (E) The presence of cerebral edema mandates
operative management.
(A) Tonsillar herniation
(B) Uncal herniation
~ A 47-year-old noncompliant man with hypertension
(C) Subfalcine herniation
and hyperlipidemia presents with a history of 10
(D) Sphenoid herniation
minutes of weakness on the right side of his body
(E) None of the above
that has since resolved. His workup in the ED
is normal but you decide to admit him with a
~ Which of the following is the most appropriate
diagnosis of transient ischemic attack (TIA)? Which
crystalloid of choice during trauma resuscitations?
of the following should be included in his treatment
(A) DlOW in the ED?
(B) DlO 0.9 N NaCI
(A) Clopidogrel
(C) D5 0.9 N NaCI
(B) Ticlopidine
(D) 0.45 N NaCl
(C) Heparin
(E) Lactated Ringers
(D) Aspirin
(E) Warfarin
[ig] The most common cause ofesophageal perforation is
(A) Caustic ingestion. ~ Which of the following is the most common cardiac
(B) Boerhaave syndrome. rhythm in patients with PE?
(C) Trauma.
(A) Normal sinus rhythm
(D) Iatrogenic.
(B) Atrial fibrillation
(E) Esophageal cancer.
(C) Atrial flutter
(D) Ventricular tachycardia
[!!] Which of the following is true regarding urethritis in
(E) Sinus tachycardia
men?
(A) Gonococcal infection is almost always
symptomatic.
TII!St5 1.37

~ Which of the following is the strongest risk factor for (C) Selective serotonin reuptake inhibitors (SSRis)
developing an abdominal aortic aneurysm (AAA)? (D) Tricyclic antidepressants
(E) Mood stabilizers
(A) AF younger than 50
(B) Obesity
(C) Female gender
~ Which of the following is true regarding ischemic
(D) Hypertriglyceridemia
stroke and cerebral edemaf
(E) First-degree relative with AAA (A) Herniation in large strokes resulting from
cerebral edema is more common in young
~ Which of the following pediatric heart d.iaeaaes causea patients than in elderly patients.
cyanoais? (B) The effects of hyperventilation on reducing ICP
(A) Mitral stenosis
laat for up to 1 week.
(C) Methylprednisolone should be gM:n to all
(B) Coarctation of the aorta
stroke patients with evidence of edema on CT.
(C) Tranaposition of the great vessels
(D) Cerebral edema due to ischemic stroke peaks
(D) ASD
within 2 to 4 hours.
(E) VSD
(B) None of the above.
~ A 26-year-old woman presents to the ED with ankle
~ A34-year-old man presents with severe left.knee pain
pain. She was playing tag with her ki~ in the ~d
when her foot got caught in an open prpe. snappmg after a motor vehicle collision. He has a large joint ef-
it outwud as she fell to the ground. The squeeze test fusion and his knee is unstable in all directions. Plain
radiographs demonstrate no fracture. Pulsea in the
produ.cea pain at the ankle and her x-ray is shown in
left lower extremity are normal. Which of the follow-
Figure 5-9. Which of the following is true?
ing is the most appropriate next step in management?
(A) Diacharge with no weight bearing
(B) MRI.knee
(C) Angiography
(D) cr~rn=
(B) Arthrosoopy

~ A 22-year-old man is dropped off by frierub at the


ED after being stabbed three timea in the abdomen.
The most likely structure to be injured is the:
(A) Liver.
(B) Small intestine.
(C) Spleen.
(D) Colon.
(E) Diaphragm.

~ The most common cause ofinfectious esophagitis is


(A) Omdid4 species.
(B) Herpes simplex virus 1.
l'lgure 5-9. (C) Cytomegalovirua.
(D) Helicobamr pylori.
(E) Pneumocystis species.

(A) This is a stable injury. ~ Which of the following is a classic symptom of a


(B) The deltoid ligament is probably intact. hydatidiform molel
(C) The syndemwsis is probably intact.
(A) Pregnancy-induced hypertension occurring
(D) This iJ a Maisonneuve fracture.
during the fint trimester
(E) None of the above.
(B) Uterine enlargement greater than expected for
gestational dates
~ Which of the following class of drugs is conaidered
(C) Abnormal vaginal bleeding
first-line therapy for moet anxiety diaorders?
(D) Hyperemesis gravidarum
(A) tJ-Blockers (E) All of theabove
(B) Antipsychotics
~ A 22-year-ol.d Latina-man who recently emigrated the following is the best step in management? The
from Mexico prescntl to the ED with fever, chills, remainder of the patient's examination is normal
abdominal pain, and intermittent nausea and vom- (A) Incision and drainage fullowed by application
iting. A CT scan of his abdomen is shown in of a pressure dressing
Figure 5-10. Which of the following is true? (B) Rderral to otolaryngology with follow-up
arranged in 2 daf'
(C) Reassurance and discharge
(D) Application of a pressure dressing only
(E) Aspiration full~ by application of a pressure
clressi..Qg

~ A 5-year-old boy is brought for evaluation of penile


erythema. He is uncircumcised and has erythema.
edema. and a semisolid discharge around the glans
without accompanying phimosis or paraphimosis.
He has had several similar episodes in the past with
identification of yeast. Which of the following is the
most appropriate diagnostic test?
(A) Liver function tests
(B) Postvoid reaidual
( C} Intravenous pyelogram
(D) Serum glucose
ftgu... .s-to. (E) Rdrograde urethrogram

(!!J A 78-year-ol.d man with history of diabetes presents


with fever and shortness of breath. He has had
(A) This condition requires immediate surgical
progressively worsening fatigue and cough over the
drainage.
(B) Transmission .is typically fecaHml.
last few daY' and developed acute onset of fever,
(C) Infections are usually polymicrobial
shaking chills, and difficolty breathing today. His
vitals are: 101.5"F, 132, 30, 87/44, 88% RA. Physical
(D) Hyperbilirubinemia is the moat commonly
examination reveals right lower lung field crackles,
abnormal laboratory finding.
and a cheat x-ray demonatrates a right lower lobe
(E) All of the above.
infiltrate. After 3 L of isotonic fluids and appropriate
antibiotic therapy, his vital signs are unchanged.
(!!) An 8-year-old boy presents with risht ear pain after
Which of the following is the most appropriate
a fight. He tells you that he was punched and kicked
therapy for this patient?
in the head and ear. On a.amination, you note the
findingvis:ible in the image (see Fig. 5-ll). Which of (A) Mechanical ventilation
(B) Tube thoracostomy on the right
(C) Percutaneous transluminal coronary
angioplasty
(D) Immediate surgical conaultation
(E) Potassium chloride infusion

~ A 25-year-old previously healthy black. woman


presents to the ED complaining of a facial rash
over her nose, mild fever, and achy wrists. She just
returned from an annual weekend trip to the Florida
beaches with a bunch of girlfriends. Which of the
following is the most sensitiw test to aid in her
diagnosis~

(A) Rheumatoid factor (RF)


(B) Anti-Smith antibody (anti-Sm)
(C) Antidouble stranded deoxyribonucleic acid
antibody (anti-d&DNA)
ftgu... .s-u.
Test5 139

(D) Antisingle strand deoxyribonucleic acid (A) Weight loss


antibody (anti-ssDNA) (B) Night sweats
(E) Antinuclear antibody (ANA) (C) Shortness of breath
(D) Chest pain
[100] Whichof the following is the most common (E) Hemoptysis
symptom seen in pulmonary tuberculosis (TB)?
Answers and Explanations
[!] Answer D. Patients with fall on outstretched hand an ongoing insulin infusion. The persistently low
(FOOSH) injuries may have bony or ligamentous bicarbonate signifies that a metabolic acidosis is still
damage in any part of their upper extremity. Most present, but it is an HCMA (nonanion gap). The de-
susceptible to injury are the bones of the wrist. The velopment of an HCMA is a common complication
anatomic snuffbox is demarcated by the extensor of DKA therapy. This is partly due to the infusion
pollicis brevis and longus tendons just proximal to of a large volume of saline, which contains chloride
the thumb and tenderness in this region indicates in concentrations far greater than plasma (154 mEq
possible fracture to the scaphoid. Scaphoid fracture per L). Another major contributor is the loss ofke-
is particularly dangerous because of the high rate of toanions in the urine that would normally serve as
avascular necrosis (AVN) and resulting limitation of precursors for bicarbonate regeneration. The devel-
thumb function. Almost one fifth of all scaphoid opment ofHCMA is benign and requires no therapy.
fractures are invisible on acute radiographs, so In the setting of normal renal function, it will re-
appropriate management in suspected cases includes solve spontaneously over the next 24 hours due to
immobilization in a thumb spica cast and repeat increased renal acid secretion. Subcutaneous insulin
radiographs in 1 to 2 weeks. There is no need should always be overlapped with an insulin infusion
for emergent orthopaedic consultation in patients when discontinuing continuous insulin therapy.
with negative x-rays. Discharging the patient home
without a splint that immobilizes the thumb is [!] Answer A. The patient has evidence of Henoch-
contraindicated. Admission for observation is not Schonlein purpura (HSP), an immune-mediated vas-
an appropriate use of resources. culitis of idiopathic origin. Young children are the
highest risk group. Dermatologic, gastrointestinal,
[!) Answer C. Diffuse axonal injury (DAI) is an renal, and musculoskeletal findings are seen. The rash
important traumatic cause ofcoma that is not due to a ofHSP is characteristic: a maculopapular eruption on
mass lesion or frank intracerebral hemorrhage. Initial the legs and buttocks and almost never involving the
CT scan is almost always normal in patients with DAI, upper extremities or trunk. Abdominal pain is due to
but MRI may show diffuse white matter disruption intestinal vasculitis or intussusception, which occurs
due to axonal fiber injury. Because of the difficulty with higher frequency in patients with HSP than the
in gauging DAI on neuroimaging, the prognosis is normal population. Renal involvement may be due to
based totally on clinical parameters. The duration glomerulonephritis, which is detected as hematuria
of coma obviously correlates with severity of injury. on urinalysis. Current management of HSP involves
Patients with DAI who awaken from coma within potential administration of corticosteroids or intra-
24 hours may have few permanent disabilities. Those venous immunoglobulin (MG) to prevent and treat
in coma for longer than 24 hours tend to have much glomerulonephritis. Abdominopelvic CT scan is not
more grim outcomes, including persistent vegetative indicated, as HSP is a clinical diagnosis. Despite the
state or extreme cognitive dysfunction. Most types possibility of HSP as an immune-mediated response
of acute intracranial hemorrhage severe enough to to streptococcal infection, ASO titers are not rou-
lead to coma would be evident on initial CT scan, tinely indicated, as they do not change management.
including epidural or subdural hematomas, cerebral Blood cultures are not necessary in these patients
contusions, and intraparenchymal hematoma. An except when systemic infection is suspected by fever
important exception is subarachnoid hemorrhage, and focal abdominal tenderness. A Meckel scan is
which may not be visible on CT scan and is a used to detect the presence of Meckel diverticulum,
common hemorrhagic cause of altered mental status which usually presents with painless rectal bleeding
after trauma. rather than the constellation of signs seen in HSP.

(!] Answer B. This patient's anion gap (AG) is 12, W Answer B. Owing to its deceptively innocuous
which is normal. Assuming that the patient has a early course, the diagnosis of acute mesenteric is-
normal albumin concentration (the major contrib- chemia (AMI) remains problematic. Therefore, the
utor to the AG in healthy patients), the normal AG mortality rate has remained essentially unchanged
reflects a resolution of the ketoacidosis. If substan- at roughly 70%. The key to diagnosis is recogniz-
tial ketoacids were still present, the AG would be ing patients at risk, such as any patient older than
persistently elevated and the patient would require 50 years of age who presents with acute abdominal

140
Test5 141

pain and who has known vascular disease, cardiac Pa<>z:Fio2 ratio ~200. Identical findings in patients
arrhythmias, recent myocardial infarction, hypov- with a Pa<>z:Fioz ratio ~300 are diagnosed with acute
olemia, hypotension, or sepsis. The most commonly lung injury (ALI), instead of ARDS, but the etiology
cited clinical finding is pain that is out of proportion of ALI is the same. Although sepsis is the most
to tenderness elicited on physical examination. This common cause of ARDS, severe trauma complicated
is a nonspecific finding that needs to be considered by shock, multiple blood transfusions, and aspiration
carefully in light of the clinical scenario. Unfortu- are all independent risk factors. Pancreatitis and near
nately, there are no laboratory markers or radiologic drowning may result in ARDS but are less common
studies apart from angiography that have sufficient causes. Such causes also illustrate the idea that the
sensitivity and specificity to exclude AMI early in its ARDS may be due to direct lung injury (e.g., severe
course. Lactate levels are elevated in approximately pneumonia with sepsis) or indirect lung injury (e.g.,
100% of patients with bowel infarction, but this is a severe pancreatitis). The latter is presumably due
late finding and mortality rates are high by the time to the widespread release of cytokines and other
infarction has occurred. Plain films are most com- proinflammatory mediators.
monly nonspecific, although findings such as ileus
correspond to more severe disease and a higher mor- 00 Answer B. The patient has temporal arteritis.
tality rate. The sensitivity of CT has been cited to be Steroids are the mainstay of management and should
as high as 82%, but the most common early finding be initiated even before temporal artery biopsy is
is bowel wall thickening, present in 26% to 96% of done. Once significant visual loss has occurred, only
cases. Unfortunately, this is also the least specific find- one out of three patients who receive IV steroids
ing and is often not present in mesenteric ischemia will improve in the affected eye. However, steroids
due to arterial embolism or thrombosis, which is the will nearly eliminate contralateral eye involvement.
most common cause of AMI. Pneumatosis intesti- Patients have a significantly higher risk of thoracic
nalis or gas in the portal venous system is a specific aorta aneurysms, and a slightly higher risk of
finding but is only present after bowel infarction abdominal aorta aneurysms. Women are five times
has occurred. In the absence of angiography, the more likely to be affected than men. Patients younger
treatment is emergent laparotomy. than 50 years are rarely affected.

[!) Answer D. Both bacteriuria and urinary tract in- [!] Answer C. This patient has an exacerbation of
fection (UTI) should be treated aggressively in preg- congestive heart failure (CHF). Although she has
nant women, as they can cause serious complications no prior diagnosis of CHF, her clinical history
with delivery. The prevalence of bacteriuria may is consistent with the diagnosis. Pleural effusions
be higher pregnant women than in nonpregnant are common in CHF and thoracentesis generally
women, but UTI rates are comparable. Symptoms has no role in their management because they
are identical between the two groups. Gram-negative typically resolve with diuretics. This patient also
enteric bacteria cause the vast majority of UTis in lacks infectious symptoms, which would suggest the
pregnant women, but a crucial organism to consider presence ofpneumonia or a parapneumonic effusion.
is Group B Streptococcus, which can cause seri- In patients with severe respiratory compromise as
ous neonatal infection. Nitrofurantoin, penicillins, a result of large pleural effusions in the setting
and cephalosporins are the drugs of choice. Sul- of CHF, a secure airway through rapid sequence
fonamides are not safe during the third trimester intubation would be the first step. Although this
because of the possibility of neonatal hemolysis. patient is hypoxic, she has not yet been given
Fluoroquinolones cause various congenital defects. supplemental oxygen, so it is premature to intubate
Although amoxicillin would be a reasonable choice this patient at this point. Furthermore, noninvasive
in most patients, the allergy to penicillin is an obvious ventilation might be attempted before establishing a
contraindication. secure airway in patients with only mild to moderate
respiratory distress. B-type natriuretic (BNP) levels
[L) Answer A. Sepsis is the most common risk factor may be helpful in determining the etiology of
or condition leading to the development of ARDS. respiratory distress in patients who have mixed
ARDS is defined as the development of acute cardiopulmonary disease without a clear clinical
respiratory failure, with noncardiogenic pulmonary picture. This patient's clinical picture clearly points
edema (established by the presence of bilateral toward a diagnosis of CHF, so a BNP level would not
infiltrates consistent with pulmonary edema on provide any additional information.
chest radiography and a pulmonary capillary wedge
pressure ~18 mm Hg indicating that the edema is ~ Answer D. The American Cancer Society esti-
noncardiogenic), and severe hypoxia such that the mated that approximately 17,000 people were
142 1000 Questions to Help You Pass tM Emergency Medicine Boards

diagnosed with primary brain tumors in 1999 ver- fractures are normally able to ambulate, although
sus > 100,000 people who died with metastatic brain they will complain of pain. Physical examination will
tumors. The most common cause is lung cancer demonstrate a patient's inability to lift the affected
followed by breast carcinoma and colon carcinoma. leg from the floor while in a seated position (iliopsoas
Malignant melanoma and renal carcinomas metas- insufficiency). Patients will also have pain when
tasize to the brain less commonly. The malignant they are asked to flex their hip against resistance.
gliomas, anaplastic astrocytoma, and glioblastoma PE is most common in the postoperative patient.
multiforme are the most common glial tumors, and Acute respiratory distress is most likely because
are typically located in the cerebral hemispheres. of fat embolism, which complicates 2% to 23% of
patients with isolated femoral shaft fractures. Acute
lnJ Answer C. Owing to denervation in the trans- respiratory distress, associated with altered mental
planted heart and the consequent lack of vagal tone, status, tachycardia, and fever are hallmarks of the
the resting heart rate averages between 100 and 110 illness. Although diffusely scattered petechiae are
beats per minute. However, the heart rate can increase nearly pathognomonic for the syndrome, they do
up to 70% of the maximum for age due to circulat- not normally occur until late in the course of the
ing endogenous catecholamines and upregulation of illness and only occur in 50% of patients.
IJ-adrenergic receptors. Although rare, tamponade
can occur in the transplanted heart because of scar ~ Answer A. Malignant neoplasms account for more
tissue formation and its ability to contain pericar- than half of all cases of large bowel obstruction.
dia! fluid or blood under pressure. Before the advent Volvulus and diverticulitis (either through stricture,
of cyclosporine, acute rejection presented as acute- abscess, or phlegmon formation) are the second and
onset CHF or atrial dysrhythmias with a new S3 third most common causes.
and diffusely decreased QRS voltage on EKG. These
features are now only present in cases of severe fail- [H) Answer A. Dysuria in all patients is most com-
ure, and typical acute rejection, which occurs in 75% monly due to bacterial UTI. Gram-negative enteric
to 85% of patients, is diagnosed by endomyocardial rods are the number one causative group, with E. coli
biopsy. There is an increased risk of endocarditis as the single most likely etiologic agent. Urethritis
with invasive procedures, so antibiotic prophylaxis due to Chlamydia and gonococ:cu5 is also extremely
should be used in any procedure expected to produce common, as are candida( vaginitis and bacterial vagi-
bacteremia. nosis. Viral and parasitic infections are uncommon
causes of dysuria. Allergic urethritis may be respon-
~ Answer B. Femoral neck fractures and intertro- sible in patients who have long-term foreign bodies
chanteric fractures account for 90% of all hip (such as Foley catheters) in place.
fractures. They both most commonly occur in
osteoporotic elderly patients after a low-energy fall. ~ Answer C. The anterior cruciate ligament (ACL)
In contrast, young patients develop these fractures is the most commonly injured ligament in the knee
in the setting ofhigh-energytrauma such as in high- which requires surgery. Diagnosis is often made on
speed motor vehicle accidents. Therefore, many of history, with acute knee swelling and audible "pop"
those patients have evidence of multisystem trauma. after twisting or lateral force to the knee. Lachman
The intertrochanteric femur has a better blood supply test is the most sensitive acute physical examination
than the femoral neck, resulting in a much smaller test to evaluate for an ACL tear in the acute setting. It
incidence of avascular necrosis (AVN). As many involves placing the knee in 20 to 30 degrees flexion
as 40% of patients with femoral neck fractures and pulling anteriorly on the leg while holding the
may develop AVN. The use of a femoral nerve distal thigh stable and observing for laxity relative
block in the setting of a femur fracture is an to the contralateral knee. The anterior drawer test
attractive means of delivering adequate pain control is another test for the ACL which is not nearly
to patients. Although it has not been widely used in as sensitive as the Lachman, especially acutely. It
the United States, several recent papers demonstrate is important for the emergency physician (EP) to
that it is an effective, safe means of pain control in remember that the Lachman is not 100% sensitive in
both pediatric and adult patients. Isolated fractures the acute setting due to limited range ofmotion from
of the lesser trochanter almost always occur in joint effusion. The posterior drawer test is used to
young adults. The fracture represents an apophyseal assess the posterior cruciate ligament (PCL), which is
avulsion due to a forceful contraction of the iliopsoas rarely injured. The McMurray test assesses the medial
muscle. A similar injury can occur at the greater meniscus. The Thompson test checks for integrity of
trochanter. Patients with isolated lesser trochanter the Achilles tendon.
Test5 143

~ Answer B. C. trachomatis is the most common was classically described by Baer in a 1932 paper
bacterial cause of sexually transmitted disease in the in which he noted that the appendix moves in
United States. Most women infected with C. tra- a counterclockwise position out of the pelvis and
chomatis are asymptomatic (as many as 85%), toward the right flank. Recent authors have disputed
although up to one third ofwomen will have signs of this, noting that right lower quadrant pain is the most
infection upon physical examination. C. trachomatis common symptom ofacute appendicitis regardless of
takes up residence in the endocervix and causes an gestational age. Because a progressive leukocytosis is
intense inflammatory reaction resulting in mucop- normal throughout pregnancy, the peripheral white
urulent cervicitis. Treatment consists of a single 1 g blood cell count may not be useful in the diagnosis of
dose of azithromycin or 7 days of 100 mg of doxycy- appendicitis during pregnancy, particular during the
cline given twice daily. In the setting of more severe second and third trimesters. Ultrasonography should
infection (pelvic inflammatory disease), doxycycline be the first diagnostic imaging test of choice, and it
should be given for 14 days and combined with has sensitivity >85%. Appendiceal rupture is most
empiric treatment for N. gonorrhoeae and probably common during the third trimester, occurring twice
Trichomonas as well. as often as in the first trimester. This is probably due
to the increased difficulty and of making a diagnosis
[!1] Answer C. Cyclosporine exhibits all the listed and the concomitant delay in treatment.
toxicities, and may also cause tremor, hyperkalemia,
hirsutism, and gingival hyperplasia. However, the ~ Answer A. The patient has necrotizing (malig-
most common toxicity associated with its use is nant) otitis externa. This condition is seen almost
dose-dependent nephrotoxicity, which occurs in one exclusively in the elderly, the immunocompromised,
third of patients. and diabetic patients. It is far more serious than
simple otitis externa due to the risk of spread of
[!!) Answer D. Neonates with any vital sign abnormal- infection to the mastoid bone, the dural sinuses,
ities, including fever or hypoxia, should be suspected and the meninges. Early, mild cases may be treated
of having a serious bacterial infection until proved with outpatient fluoroquinolones active against Pseu-
otherwise. Even patients who exhibit strong signs domonas, but most cases will require admission
of congenital heart disease should receive antibiotics and possible surgical debridement. Pseudomonas and
and an evaluation for septic cause of the clinical Staphylococcus aureus are the most common bacterial
findings. Cefotaxime and ampicillin are indicated for pathogens implicated. The facial nerve is the most
treatment of the most common pathogens in the commonly affected cranial nerve in otitis externa.
neonatal period, Group B streptococci, gram-negative CT scan, rather than x-rays, is indicated for evalua-
bacilli, pneumococcus, and Listeria. Until the exact tion of spread of disease. Antivirals have no role in
congenital heart defect can be determined, no ther- management.
apy that affects ductus arteriosus patency, such as
indomethacin, should be administered. Albuterol is ~ Answer A. Shoulder dystocia occurs when further
indicated for reactive airways disease, and ribavirin is progression of fetal delivery is halted by impaction
used in select patients with respiratory syncytial virus of the fetal shoulders within the maternal pelvis.
(RSV) bronchiolitis. Propanolol may be used in cer- Although it is more common in diabetic mothers
tain patients with tetralogy of Fallot during Tet spells with infants weighing >4,000 g, more than half the
to reduce right ventricular outflow obstruction. cases involve infants <4,000 g and without other
risk factors. Rapid recognition and treatment is
[!ID Answer A. Appendicitis is the most common critical because of an increased risk of fetal hypoxia
surgical emergency during pregnancy, whereas acute and irreversible neurologic damage. Other injuries
cholecystitis is the second most common surgical that complicate shoulder dystocia include brachial
problem. Overall, the incidence of appendicitis plexus injuries and humerus and clavicular fractures.
during pregnancy is widely quoted as equal to The first step in ED management should be a
the nonpregnant population, with an incidence of liberal median episiotomy to create more space to
approximately 1 to 2 of every 1,000 gestations. allow the posterior shoulder to pass. The next step is
However, recent data suggest that appendicitis may the McRobert maneuver, involving hyperflexion of
actually be slightly less common in pregnancy, the maternal hips (placing the maternal knees up
particularly during the third trimester. Regardless, to the chest), which moves the symphysis pubis over
the diagnosis of acute appendicitis is more difficult the fetal anterior shoulder. If this fails to work, the
during pregnancy, due to displacement of the next step is moderate suprapubic pressure to push
appendix from its normal anatomical position. The the fetal anterior shoulder below the symphysis. If
position of the appendix throughout pregnancy this fails, the Rubin maneuver should be used. This
144 1000 Questions to Help You Pass the Emergency Medicine Boards

involves pushing the most accessible fetal shoulder ligamentous injury. On lateral radiographs, the
toward the fetal chest, decreasing the bisacromial lunate is in proper position relative to the radius,
diameter and allowing delivery. The next step is but the capitate falls posterior to the lunate and
Wood corkscrew maneuver, in which the fetus is may even articulate with the distal radius. In lunate
rotated 180 degrees (preferably clockwise) in an dislocation, the lunate dislocates volarly relative to
attempt to free the shoulders. Ifall ofthese efforts fail, the radius and the capitate is in line with radius. The
the posterior arm can be grasped, placed on the fetal median nerve is the most common peripheral nerve
chest, and swept over the face and out of the vagina. injured in cases oflunate or perilunate dislocation.
Tills maneuver may result in fractures or brachial
plexus injury. ~ Answer C. Acutely thrombosed hemorrhoids
should be excised within the first 48 hours. Although
~ Answer A. Roseola infantum, also known as exan- excision provides rapid relief from pain, the natural
them subitum (sudden rash), or sixth disease, is caused history of thrombosed hemorrhoids is spontaneous
by human herpesvirus 6. Patients develop a sudden- resolution after several days. Therefore, patients with
onset high fever from 103°F to 106°F with hardly only mild pain from thrombosis and patients who
any associated symptoms. Owing to the rapid rise in have already dealt with symptoms for several days
temperature and high fever, however, febrile seizures should be managed conservatively. Excision in these
may occur. The fever lasts for 3 to 4 days and then patients will not provide any added relief.
abruptly subsides, at which point the rash begins. The
term exanthem subitum or sudden rash refers to the ~ Answer C. All of the vessels mentioned provide
startling development of a rash just when the patient blood flow to the nose. Kiesselbach plexus is the most
appears to be recovering. The rash, as with the illness anterior and the most easily traumatized. Anterior
in general, is benign and self-limited. No treatment is nosebleeds are usually easily stopped by cautery
necessary. Rubeola (measles) is characterized by the with silver nitrate or packing with Merocel gauze.
finding of Koplik spots on the buccal mucosa. The Nosebleeds that persist despite adequate packing are
hallmark of rubella is generalized lymphadenopathy. posterior in origin until proved otherwise.
Erythema infectiosum is distinguished by its "slapped
cheek" appearance. Scarlet fever is caused by group A ~ Answer C. Although retinal hemorrhage is a well-
streptorocci and is characterized by its sandpaper rash. known marker for child abuse (shaken baby syn-
drome), it is present in almost half of all neonates,
~ Answer D. The vesicular lesions on the soft palate due to normal birth trauma. Resolution tends to
are most characteristic of herpangina, which is usu- occur in 1 month.
ally caused by coxsackie viruses A and B. The clinical
syndrome usually starts with fever, myalgias, dys- ~ Answer A. By far, chromosomal abnormalities
phagia, and sometimes headache and stiff neck. Oral are the most common cause of first trimester
lesions of herpangina usually spare the gingiva and spontaneous miscarriage. At least 50% of first
hard palate, unlike herpes simplex virus gingivos- trimester pregnancy losses are related to fetal
tomatitis. Management of herpangina is completely chromosomal abnormalities. Autosomal trisomy is
supportive and ulcers will recede in 10 days. Atten- the most common abnormality, with trisomy 16
tion to appropriate hydration is integral, as many being the most common specific chromosomal
patients will be unable to even drink liquids because defect. Polyploidy is the next most common defect,
of pain for the first few days of illness. Aphthous with tetraploidy being most common. Uterine
ulcers are classically present on the tongue, buccal structural abnormalities, cigarette smoking, and
mucosa, and soft palate, but they are rarely associated trauma may all contribute to fetal loss. However,
with systemic symptoms of infection. Group A strep- minor trauma such as a full or strike to the abdomen,
tococci may cause pharyngeal and tonsillar ulcers but is very unlikely to cause fetal loss. Other factors
not usually more proximal ulcers. Diphtheria clas- include corpus luteum failure, antiphospholipid
sically causes a grayish tonsillar pseudomembrane antibody syndrome, and maternal endocrine diseases
without ulcerations. (Figure courtesy of Mark Silver- such as diabetes mellitus and hypothyroidism.
berg, MD. Reprinted with permission from Silver-
berg M. Greenberg's text-atlas of emergency medicine. ~ Answer C. The EKG changes over time in patients
Lippincott Williams & Wilkins; 2004:156.) with pericarditis. Acutely, concave ST elevation
and PR depression with tall T waves are seen.
~ Answer C. The capitate articulates directly with Depression of the PR segment is the most specific
the lunate and dorsally dislocates when there EKG finding for acute pericarditis. Concave ST
is enough carpal instability due to fracture or elevation is also common, but can be seen in a
Test5 145

variety of other conditions, including benign early comprise only a small minority of reporters of el-
repolarization and left ventricular hypertrophy. An der abuse for a variety of reasons, including lack
ST-segment to T-wave ratio of >0.25 argues in of awareness or education about the problem. Most
fuvor of acute pericarditis. Convex ST elevations perpetrators are family members or direct caregivers.
are more likely to be due to myocardial infarction Every ED should have specific protocols for screening
than pericarditis. Hyperacute T waves are seen for and reporting elder abuse, along with an action
more often in hyperkalemia and infurction than plan for confirmed abuse.
in pericarditis. AV blocks are rarely seen in acute
pericarditis. Chronic EKG changes associated with ~ Answer C. As with other deep space infections
pericarditis include return ofST segments to baseline, of the neck, cr is the most valuable diagnostic
T -wave flattening, T -wave inversion, and then tool. Although lateral neck x-rays are the preferred
complete normalization after a few weeks to months. initial imaging modality for suspected retropharyn-
geal space infections, they do not typically provide
~ Answer D. Any patient with acute agitation and useful information in patients with suspected para-
the potential for violence either to self or oth- pharyngeal infections. Furthermore, CT is better able
ers must be physically and chemically restrained. to localize the specific site ofinfection in patients with
Rapid tranquilization is ideally accomplished with retropharyngeal abscesses. Anterior-posterior plain
a combination of haloperidol and benzodiazepine. films of the neck are occasionally used to support a
Benztropine is given to reduce the incidence of acute diagnosis of croup (steeple sign), but have no role
dystonic reactions that may accompany administra- in the diagnosis of parapharyngeal space infections.
tion of the antipsychotic medication. Observing the Both MRI and ultrasonography are capable of pro-
patient in an agitated state struggling against re- viding useful information, but MRI is limited by time
straints is contraindicated, as the patient may cause and access constraints and ultrasonography provides
harm to himself while fighting. Trying to perform a much less detail and is less able to accurately localize
CT scan with the patient in an acutely agitated, vio- the site of the problem.
lent state will be impossible and just put the patient
and staff at risk for injury. Rapid sequence intuba- ~ Answer E. The image reflects an avulsion fracture
tion is not indicated unless rapid tranquilization is of the base of the fifth metatarsal. Confusion
unsuccessful and serious morbidity is suspected in regarding the differences between a Jones fracture
the patient, necessitating emergent workup for acute and a simple avulsion of the base of the fifth
medical or traumatic cause for the psychosis. metatarsal (sometimes referred to as a pseudo-Jones
fracture) is partly due to the inconsistency in their
@!) Answer C. The CT scan shows diffuse, bilateral respective definitions in the literature. However, the
bright signal consistent with acute hemorrhage in most consistent definition of a Jones fracture is a
the subarachnoid space. Epidural and subdural fracture of the fifth metatarsal that occurs distal
hematomas are usually focal, unilateral, and often to the junction of the metaphysis and diaphysis.
cause midline shift. Cerebral contusion appears as Radiographically, this is variably identified as a
blood in the parenchyma rather than the cisterns. fracture occurring at least 1.5 em from the base
Diffuse axonal injury (DAI) usually does not appear of the fifth metatarsal to a fracture occurring distal to
on an emergent brain Cf and requires clinical eval- the distal edge of the neighboring cuboid. Both Jones
uation and MRI for diagnosis. (Figure courtesy of fractures and avulsion fractures of the metatarsal
Robert Hendrickson, MD. Reprinted with permis- head are typically due to inversion injuries. In the case
sion from Hendrickson R. Greenberg's text-atlas of of avulsion fractures, the peroneus brevis tendon,
emergency medicine. Lippincott Williams & Wilkins; which inserts at the base of the fifth metatarsal,
2004:45.) avulses a fragment of bone during acute ankle
inversion. Patients with avulsion fractures do well
~ Answer A. Elder abuse is an underreported prob- with conservative treatment such as a compressive
lem that is slowly increasing in incidence with better dressing, a hard soled orthopaedic shoe, and weight
layperson and health provider education and recog- bearing as tolerated (crutches ifneeded). In contrast,
nition. Although much data is still lacking, it is clear there is a high incidence of malunion and nonunion
that elders who undergo abuse are at higher risk in patients with Jones fractures. Such patients are
for morbidity and mortality than nonabused elders. placed in a short leg cast and should be nonweight
Women, the extreme elderly, and patients with severe bearing until orthopaedic follow-up. Most of these
physical and cognitive dysfunction are all at higher patients require surgical fixation with a screw. (Figure
risk. The most common type ofelder abuse is neglect; courtesy of Robert Hendrickson, MD. Reprinted
sexual abuse occurs but is uncommon. Physicians with permission from Hendrickson R. Greenberg's
146 1000 Questions to Help You Pass the Emergency Medicine Boards

text-atlas ofemergency medicine. Lippincott Williams in patients with UTis due to Proteus, KlebsieUa, and
& Wilkins; 2004:542.) Pseudomonas. Uric acid stones occur in patients with
hyperuricemia, often due to gout. They are usu-
~ Answer D. Gonococcus can cause either a mi- ally radiolucent and missed on plain radiographs.
gratory, polyarticular arthritis or monoarticular Cystine stones are the least common and are due
arthritis. Synovial fluid Gram stain is positive more to hypercystinuria, an inborn error of metabolism
often than culture. It is the most common cause of usually diagnosed at birth.
septic arthritis in the young, sexually active adult,
and is much more common in women than men. ~ Answer D. Paroxysmal supraventricular tachycar-
The most common joints affected are the knees, dia (PSVT) is the most common cardiac rhythm
wrists, and ankles. A clinical syndrome of arthritis, disturbance in pediatric patients. Children are more
tenosynovitis, and dermatitis (discrete hemorrhagic likely than adults to have an accessory pathway as a
pustules) may occur. Although urethral and cervical cause, but reentrant AV nodal rhythms are common
cultures are high-yield, patients rarely complain in older children. Treatment involves vagal maneu-
of associated symptoms. Treatment involves intra- vers, adenosine, and AV nodal active agents in a
venous antibiotics (third generation cephalosporin) stepwise manner. As in adults, unstable tachydys-
and inpatient observation. Although orthopaedic rhythmias necessitate electrical cardioversion. Atrial
consultation is warranted, open drainage is rarely fibrillation and flutter are rare in pediatric patients
necessary. without structural heart disease. Complete heart
block is uncommon in children. Ventricular tachy-
~ Answer D. Decompression sickness (DCS) is due cardia is less common than PSVT in children.
to the presence of nitrogen bubbles in the blood and
tissues. It is divided into type I DCS and type II ~ Answer B. Reduction of anterior shoulder dislo-
DCS. Type I DCS affects the musculoskeletal system, cations may be accomplished through a variety of
skin, and lymphatic vessels. Type II DCS involves all methods, none of which is clearly superior to the
other organ systems. The shoulder and elbow are the others. A high rate of complications is associated
most common joints involved in type I DCS. The with use of the Kocher method (leverage, adduction,
arthralgias experienced by patients with type I DCS and internal rotation) and the Hippocratic method
are known as the bends. Joint pain may be reduced by (axillary traction with the physician's foot). The most
inflating a blood pressure cuff over the affected joint important factor in determining success ofrelocation
to 150 to 200 mm Hg. This may also be used to aid is adequate muscle relaxation, which may be ensured
in diagnosis although it has a poor sensitivity. by either procedural sedation or intra-articular anes-
thesia. Scapular manipulation is performed with the
~ Answer D. MRI is the most sensitive test to detect patient in prone position and should not be used in
ocular foreign body, but should never be used when patients who are pregnant in the third trimester due
a metallic ocular foreign body is suspected. Plain to compression of the uterus.
radiographs are useful but are not as sensitive as CT
scan. Wtrasonography is highly operator dependent. ~ Answer B. The anion gap (AG) is used to signify
Nuclear medicine scans have no role in ocular foreign the difference between the concentration of sodium
body detection. ( [Na+ ]), and the sum of the concentrations of chlo-
ride ([Cl-]) and bicarbonate ([HC03D such that:
~ Answer C. Kidney stones most commonly occur AG = [Na+]-[Cl-]-[HC03]. However, because
in middle-aged patients, usually men. Risk fac- of the law of electroneutrality, all aqueous solutions
tors include age, male gender, family history, and must have an equal number of positive and nega-
conditions which increase serum and urinary cal- tive charges such that the entire solution is neutral.
cium levels. Kidney stones are divided into four Therefore, the AG does not reflect a true "positive"
main categorie&---ealcium, magnesium ammonium or "negative" charge in the plasma. Instead, it re-
phosphate, uric acid, and cystine. Calcium stones rep- flects the presence of an anion which the formula
resent approximately two third ofall stones and occur is not measuring. In normal patients, albumin ac-
more often in patients with common precipitants counts for the bulk of these "unmeasured anions."
of hypercalcemia, including hyperparathyroidism, Each 1 g per dL decrease in the concentration of
milk-alkali syndrome, laxative abuse, and sarcoido- albumin will decrease the expected AG by approxi-
sis. Inflammatory bowel disease also leads to the mately 2.5 to 3 (the normal albumin concentration
formation of calcium oxalate stones, due to hyperox- is roughly 4 g per dL x "-'3 Rj expected AG of 12).
aliuria. Magnesium ammonium phosphate (struvite) Therefore, patients with hypoalbuminemia (e.g., cir-
stones account for one fifth of all calculi and occur rhosis, malnutrition) will have a smaller, normal AG.
Test5 147

Sulfate, phosphate, and citrate make up the bulk of postinfectious and are responsible for 5% ofall deaths
the remaining Wlilleasured anions. in sickle cell patients.

~ Answer B. The classic CSF finding in patients ~ Answer B. Although rectal involvement is uncom-
with Guillain-Barre syndrome (GBS) is termed cy- mon in patients with Crohn disease and nearly
toalbumin dissociation, which refers to the fact that universal in patients with ulcerative colitis, peri-
protein levels are elevated without a concomitant anal complications are much more common in
rise in white blood cell counts. Despite this, lumbar patients with Crohn disease. The anal canal is the
puncture and CSF examination is not necessary to most terminal segment of the large intestine. In ap-
make a diagnosis of GBS, although they are usually proximately 25% of patients with Crohn disease,
performed. The only two requirements for the diag- perianal complications may occur before the onset of
nosis are progressive motor weakness of more than overt disease. Toxic megacolon is more common in
one limb and areflexia, which is typically global. patients with ulcerative colitis. Erythema nodosum
Lumbar puncture is not contraindicated in the set- occurs most often in women patients with Crohn
ting of GBS. In contrast, it is commonly performed disease. Anal fissures associated with Crohn disease
to support the diagnosis and to rule out other are typically eccentrically located. In patients without
causes. Oligoclonal bands are most commonly as- Crohn disease, more than 90% of fissures are located
sociated with multiple sclerosis, although they may in the posterior midline.
be present in other diseases as well. They are not
found in GBS. ~ Answer C. One of the guiding principles of pe-
ripheral nerve testing is to evaluate nerves in their
~ Answer E. There is no relationship between "autonomous zone" of innervation (i.e., where there
seafood or shellfish allergies and adverse reactions is no overlap of adjacent nerves or nerve roots). In
to iodinated RCMs. Adverse reactions to RCMs are the hand, the standard autonomous zone for testing
related to their osmolarity and are anaphylactoid the median nerve is the volar aspect of the index
in nature (i.e., not dependent on IgE sensitization). finger, distal to the distal interphalangeal (DIP) joint
They are not related to the presence of iodine in (the tip of the index finger); the zone for testing the
the contrast material. Each of the other patients has ulnar nerve is the volar aspect of the little finger,
a condition that places them at risk for an adverse distal to the DIP joint (the tip of the little finger).
event, although there are no absolute contraindica- The radial nerve, however, has a far less well-defined
tions to the use of RCM. A history of atopy (e.g., autonomous zone as there is much overlap with cu-
almost all patients with asthma, allergic rhinitis, or taneous branches of other nerves. The best area for
atopic dermatitis), preexisting coronary artery dis- testing is the dorsal aspect of the webspace between
ease, the use of ,8-blockers or a prior adverse event the thumb and the index finger. This area overlies the
with RCM administration each places a patient at first dorsal interosseus muscle, which is an ulnar in-
risk for a future event. Various protocols for pre- nervated structure. However, cutaneous innervation
treatment exist and vary by institution, although to this area is primarily served by the radial nerve.
they almost invariably involve the use of corticos- The C5 nerve root does not have contributions to
teroids and antihistamines over a 12-hour period the hand and the C6 nerve root contributes to the
before the study. median and radial nerves but not the ulnar nerve
which receives contributions primarily from C8 and
~ Answer D. Sickle cell disease is a hemoglobinopa- T1 and occasionally C7.
thy causing sickling of red blood cells with any
systemic stress, which results in diffuse microinfarc- ~ Answer E. Therelativeriskforsmokersdeveloping
tions. Sickle cell trait is present in approximately pancreatic cancer is at least 1.5, but this number rises
10% of all African Americans, and sickle cell disease further with the amount of cigarette consumption.
is primarily a disease of this population. Symptoms The second most important factor is a high-fat (or
involve multiple organ systems and result in specific meat-based) diet. Diets rich in fruits and vegetables
acute crises-vaso-occlusive, acute chest syndrome, have been found to be protective. Chronic alcohol
splenic sequestration, and aplastic. Aplastic crises are use also probably increases the risk, although this has
characterized by the acute onset of worsening ane- not been firmly established.
mia combined with bone marrow failure. Laboratory
abnormalities demonstrate a drop of hemoglobin ~ Answer B. Preeclampsia is associated with vaso-
of 2 g per dL from stable levels and an inadequate spasm, reduced renal blood flow and glomerular
reticulocyte response ( <2%) from the bone mar- filtration rate and increased total body water re-
row to this sudden anemia. Aplastic crises are usually sulting in edema. However, hypovolemia results
148 1000 Questions to Help You Pass the Emergency Medicine Boards

in decreased uteroplacental blood flow and possi- problem during descent. Ear pain during descent
ble fetal injury. Diuretics and hyperosmotic agents is the most common symptom, although transient
should therefore never be used in the setting of hearing loss may occur. Additionally, ifthe diver con-
preeclampsia. Although volume expanders such as tinues to deeper water, further increases in pressure
albumin may sound like a good idea, they do not m.ayresultin tympanic membrane (TM) rupture. TM
reverse vasospasm or improve uteroplacental blood rupture may alleviate some of the pain but it also ex-
flow. Instead, maintenance fluids should be given poses the middle ear to cold water, which may result
although recommendations between authors vary. in nystagmus and vertigo. A facial nerve palsy also un-
Owing to the risk of pulmonary edema and the in- commonly occurs. Nitrogen narcosis occurs during
ability of intravenous fluids to reverse vasospasm, descent as increased levels of nitrogen are "forced"
however, aggressive large fluid boluses should also into the tissues. Symptoms typically occur at approx-
be avoided. One author recommends 5% dextrose imately 100 ft and resemble alcohol intoxication.
in lactated Ringers solution with dose monitoring Barosinusitis presents as facial pain that results from
of urine output, which is maintained at approxi- pressure changes in one of the facial sinuses. Facial
mately 30 mL per hour. Excessive urine output may barotrauma occurs when divers fail to equilibrate the
indicate fluid overload, placing patients at risk for airspace created by a dive mask over the eyes and nose.
pulmonary edema. The relative negative pressure causes petechial hem-
orrhages on the face, subconjunctival hemorrhage,
~ Answer C. The discriminatory zone is the quan- and conjunctival edema. Temporomandibular joint
titative serum ,8-hCG level at which a normal dysfunction is caused by teeth clenching and mal-
pregnancy can be detected by either transvaginal occlusion resulting from a poorly fitting mask. The
or transabdominal ultrasonography. As ultrasono- pain is felt near the ear and can be confused with
graphic technology improves, the discriminatory middle ear barotrauma.
zone continues to drop. Furthermore, these levels
may vary somewhat between hospitals due to tech- ~ Answer E. There are numerous potential compli-
nologic differences. However, the accepted range is cations ofB. pertussis infection and complications are
1,000 to 2,000 miU per mL for transvaginal ultra- not uncommon. They can be divided into mechan-
sonography and 2,400 to 3,600 miU per mL for ical complications as a result of severe cough and
transabdominal ultrasonography. Other sources cite infectious complications. Mechanical complications
a range of 1,500 to 2,500 miU per mL as the accepted include subconjunctival hemorrhage, Mallory-Weiss
range in transabdominal ultrasonography. Of course, tears, pneumothorax and pneumomediastinum, at-
the higher the discriminatory threshold, the higher electasis, urinary incontinence (increases with age),
the specificity for an abnormal pregnancy, including syncope, rib fractures, facial and truncal petechiae,
ectopic gestation. abdominal and inguinal hernias, and back pain. In-
fectious complications include pneumonia (up to
~ Answer E. CT has become the test of choice for 20% in children, versus only 2% to 4% in adults),
diagnosis of kidney stones, replacing the intravenous sinusitis, and otitis media (most common infectious
pyelogram in this regard. It has excellent sensitiv- complication). The most serious complication in
ity and specificity and is helpful in evaluating other young infants is apnea leading to hypoxia, cyanosis,
conditions in the differential diagnosis of flank pain. and possibly death. It is because of this complica-
Twenty percent of kidney stones have normal uri- tion, that all infants with B. pertussis infection should
nalyses, without microscopic hematuria, so a normal be admitted to an intensive care unit (ICU) setting
urinalysis in highly suspicious cases by no means for appropriate monitoring. This complication most
rules out the diagnosis of kidney stone. Radiographs commonly occurs in infants just a few weeks old.
have <75% specificity for the diagnosis, and false Pneumonia may be primarily caused by B. pertussis
positives from phleboliths, calcified lymph nodes, in young infants, whereas the pneumonia that occurs
and bone shadowing are common. Most kidney in adolescents and adults is usually caused by a sec-
stones are radiopaque-uric acid stones, represent- ondary infection. Other rare complications include
ing approximately 10% of all stones, are radiolucent. seizures (0.3% to 0.6%), encephalopathy ( 0.1%), and
Ultrasonography, although possessing good sensitiv- even carotid artery dissection.
ity and excellent specificity for hydronephrosis, has
relatively poor sensitivity for ruling out the diagnosis ~ Answer D. Chlamydia is the most common cause
of kidney stone. of nongonococcal urethritis, accounting for over
half of all cases. Symptoms are very similar
~ Answer E. Middle ear barotrauma or "middle ear to a UTI-however, urethral discharge worse in
squeeze" is the most common barotrauma-related the morning is more characteristic of urethritis.
Test5 149

Screening for other sexually transmitted diseases ice water bath. Some authors favor ice water immer-
(STDs) should be pursued. Therapy is with doxy- sion because it is thought to result in more rapid
cycline or azithromycin and sexual partners should cooling. However, ice water immersion presents
also be treated. Choices A, B, and E all cause urethri- challenges as patient monitoring and resuscitation
tis, but are less common than Chlamydia. Choice C is more difficult and immersion containers may not
causes chancroid, a syndrome of a painful, ulcerated be readily available. The application of ice packs to
lesion on the genitalia in association with inguinal the axilla and groin should be considered an adjunct
lymphadenopathy. only as its use is inferior to tepid water and fanning
alone. Antipyretics and dantrolene play no role in the
~ Answer D. Posterior nosebleeds are potentially management ofheat stroke although dantrolene may
life-threatening causes of hemorrhage due to the be considered in refractory cases, thought to be due
difficulty in management. The most common bleed- to malignant hyperthermia. More invasive cooling
ing vessel is the nasopalatine artery, a branch of means, such as gastric, pleural, or bladder irrigation,
the sphenopalatine artery. Posterior nosebleeds are as well as intravascular cooling is reserved for only
assumed to be present if a nosebleed is unable to the most severe, refractory cases and have not been
be controlled with an adequate anterior nasal pack. studied to support their routine use.
Posterior packing devices such as the Epistat may
be placed to provide both posterior and anterior ~ Answer C. Sigmoid volvulus accounts for 75% of
tamponade. Patients with posterior packs should be volvulus cases, whereas cecal volvulus accounts for
admitted to units that provide telemetry and pulse the remaining 25%. Although patients with cecal
oximetry monitoring (usually the ICU), given an- volvulus are younger than patients with sigmoid
tibiotics to cover gram-positive organisms, and ENT volvulus, they are not young, as affected patients are
consultation should be sought. typically in their late fifties. Plain films are diagnostic
in 80% of cases of sigmoid volvulus but <50% of
~ Answer D. This patient has Henoch-Schonlein cases of cecal volvulus. The classic findings of cecal
purpura (HSP), which is a systemic, small vessel volvulus include a massively dilated cecum typically
vasculitis that most commonly affects children. The in the left upper quadrant or epigastrium (i.e.,
classic presentation is a patient with a palpable, pur- not within the right abdomen). Although selected
puric rash in dependent areas such as the buttocks patients with sigmoid volvulus may be candidates for
and lower extremities, abdominal pain, hematuria, endoscopic reduction, patients with cecal volvulus
and joint pain. Renal involvement is common, but almost always require surgical repair due to the
typically manifests as microscopic hematuria and difficulty of accessing this area endoscopically and
resolves without sequelae. However, more severe the need for cecopexy to prevent recurrence.
presentations from nephritis to nephritic syndrome
rarely occur. Long-term prognosis in HSP is deter- ~ Answer D. Colon ischemia or ischemic colitis is
mined primarily by the degree of renal involvement. the most common intestinal ischemic malady. The
Patients with HSP do not have thrombocytopenia. disease can occur at any age but most commonly
High-dose aspirin is a therapy reserved for patients affects elderly patients with known vascular disease.
with Kawasaki disease. Intussusception is commonly Patients typically present with acute-onset, mild,
associated with intussusception. However, in con- crampy abdominal pain associated with an urge
trast to most patients with intussusception in which to defecate and hematochezia. As many as 50%
the obstruction occurs in the ileocolic region, pa- of cases are initially misdiagnosed as inflammatory
tients with HSP experience ileoileal intussusception. bowel disease (typically ulcerative colitis). The disease
In the great majority of cases, HSP is a benign, self- is due to decreased colonic blood flow, which
limited disease that requires only supportive therapy. disproportionately affects the colonic mucosa. It is
Steroids remain controversial and are only used for isolated to the left colon in 75% of cases and most
patients with severe symptoms. (Figure reprinted commonly occurs in the splenic flexure, which is
with permission from Fleisher GR Atlas ofpediatric the watershed area between the superior and inferior
emergency medicine. Lippincott Williams & Wilkins; mesenteric arteries. The classic radiographic finding,
2003:311.) seen in only 20% of cases, is "thumbprinting," which
is the finding ofblurred intraluminal protrusions due
~ Answer A. There are two accepted and commonly to submucosal hemorrhage and swelling. However,
used rapid cooling modalities in heat stroke. One the diagnosis is now made almost entirely through
involves pouring or spraying tepid water over a pa- colonoscopy. Angiography has no role except in
tient and using a fan to enhance evaporative cooling rare cases of isolated right colon involvement, which
through convection. The second is immersion in an raises the possibility of superior mesenteric artery
150 1000 Questions to Help You Pass the Emergency Medicine Boards

thrombosis. Instead, treatment is supportive, with observation. Injuries to the radial nerve in this area
intravenous fluids and bowel rest. Surgery is only result primarily in wrist drop along with weakness of
required in cases of peritonitis. Most patients with finger extension andhypoesthesiaand decreased two-
colon ischemia recover with supportive therapy, point discrimination in the distribution of the radial
although strictures may occur as part of the healing nerve. The radial nerve also innervates the supinators
process. The mortality rate is much lower than of the wrist, resulting in difficulty with supination.
ischemia of the small intestine. Because the radial nerve sends branches to the triceps
before its entrance into the spiral groove, elbow
~ Answer B. Patients with hepatitis A never develop extension is unaffected (although it may be weak due
chronic disease. Approximately 100% of patients to pain resulting from the fracture). (Figure from
recover from acute illness within 6 months, although Reece RM, Ludwig S. Child abuse: Medical diagnosis
fulminant hepatitis leading to death rarely occurs. and management, 2nd ed. Philadelphia: Lippincott
The mortality rate of hepatitis E infection during Williams & Wilkins; 2001:150, with permission.)
pregnancy depends on the trimester during which
a woman is infected. The maternal mortality rate is ~ Answer E. Unlike the adult, the narrowest portion
only 1.5% for infections in the first trimester, 8.5% ofthe pediatric airway is the cricoid cartilage, necessi-
for those in the second trimester, and 21% for those in tating the use of uncuffed tubes in children younger
the third trimester. Roughly 5% of children infected than 8 years. Other important considerations for
with hepatitis A are symptomatic at presentation the pediatric airway are the proportionally larger
versus 70% to 80% of adults. In endemic areas, such tongue, floppier epiglottis, more anterior airway po-
as Southeast Asia, most of the population is infected sition, and shorter tracheal length. These anatomic
as children, and most of the community is immune differences require slightly different techniques from
by age 10. Hepatitis A is the most common cause of those used with adult airway management, including
viral hepatitis worldwide. Hepatitis A is transmitted an adjunctive oral airway, frequent use of a straight
through the fecal-oral route. Although percutaneous blade, and more anteriorly directed laryngoscopic
transmission may occur (e.g., through a needlestick technique. Most children older than 12 years have
injury), it is very rare because the concentration of similar airway characteristics to adults.
the virus in the blood is quite low and the duration
of viremia is brief. ~ Answer E. Tinea versicolor is a fungal infection of
the skin caused by Pityrosporum ovale (oval form) and
~ Answer A. Nocturnal symptoms point against a Pityrosporum obiculare (round form). These organ-
diagnosis of irritable bowel syndrome but are isms were previously called Malassezia furfur. Tinea
common in patients with Crohn disease. Weight versicolor is a benign and common fungal infection
loss also points against irritable bowel syndrome but ofthe skin that most commonly occurs in areas ofthe
is a less specific complaint. Bilious vomiting is not skin with increased sebaceous activity. It is most com-
common in either disease. mon in the upper trunk but it commonly spreads to
the arms, neck, and abdomen. Lesions may be a vari-
~ Answer C. The most common causes of acute ety of colors, but are classically hypopigmented white
otitis media are S. pneumoniae and M. catarrhalis. or tan macules and patches. Griseofulvin is not effec-
M. pneumoniae is an uncommon cause of otitis tive against these fungi, but multiple other agents are
media, especially in this age-group. The incidence of effective, including topical ketoconazole, selenium
infections due to H. influenzae has decreased since sulfide, and terbinafine as well as oral itraconazole,
the introduction of the Hib vaccine. Viruses as a fluconazole, and ketoconazole. Although such ther-
group account for up to one out of every six cases apy is highly effective, recurrence is common after it
of otitis media-respiratory syncytial virus (RSV) is is discontinued.
the single most common viral cause of otitis media.
Group A streptoroccus is the least common of the ~ Answer A. Glucocorticoids are useful in thyroid
bacterial pathogens listed. storm because patients with thyroid storm may de-
velop relative adrenal insufficiency as a result of the
~ Answer A. The radial nerve may be injured in as increased metabolic demands. In addition, high-dose
many as 20% of humeral shaft fractures because corticosteroids inlubit release of thyroid hormone
of its close proximity to the humerus as it travels from the thyroid gland as well as peripheral conver-
posteriorly in the spiral groove. Most of these injuries sion ofT4 toT3 • Apathetic thyrotoxicosis is a rare pre-
are transient neurapraxias and will improve without sentation of hyperthyroidism that primarily occurs
intervention. Therefore, such injuries should be in the elderly. Cardiac manifestations, such as atrial
well documented and followed by close outpatient fibrillation and CHF, typically predominate. Most
Test5 151

patients with thyroid storm have underlying Graves injury is known as Maisonneuve fracture. The force
disease, which is the most common cause of hyper- of the initial ankle injury disrupts the syndesmo-
thyroidism. Hashimoto disease is the most common sis between the tibia and fibula and exits from the
noniatrogenic cause of primary h)pothyroidism (al- pro:ximal fibula. Maisonneuve fracture puts the ankle
though it may cause a transient thyrotoxicosis in its mortise joint at extreme risk for instability and stress
acute phase). Treatment of Graves disease is probably views of the ankle should be performed to evaluate
the most common cause of primary hypothyroidism. for this. Any significant instability seen may require
Sinus tachycardia is the most common dysrhythmia operative management. MRI of the knee may evalu-
in thyroid storm. ate for any associated ligamentous injury of the knee
but is unnecessary in the acute setting. Ankle MRI
~ Answer C. Patients with anterior shoulder dislo- can diagnose an associated deltoid ligament tear, but
cation and subsequent relocation are at extremely it is easier, cheaper, and faster to perform stress ra-
high risk of recurrent dislocation. Young patients diographs acutely to assess for mortise instability.
are the highest risk group, probably because of a Discharging the patient without proper immobiliza-
combination of associated cartilaginous injury and tion of the ankle and evaluation of Maisonneuve
overly aggressive return to previous activity. Surgi- fracture is contraindicated.
cal stabilization is recommended in these patients to
prevent recurrence. The other answer choices are all [!g) Answer E. Cerebral venous thrombosis ( CVf) is
complications of anterior shoulder dislocation but a rare cause of stroke and is extremely variable
occur less commonly than recurrent dislocation. in its presentation. The most common presenting
symptom is headache. In contrast to arterial stroke,
[!lJ Answer B. Chicken pox is an acute illness caused patients with cvr present acutely only 30% of the
byVaricella-Zostervirus (VZV) causing fever, myal- time, subacutely (more than 2 days after symptom
gias, and a maculopapular rash progressing to vesicles onset) in 50% of cases, and chronically (more than
which then rupture and form dry crusted lesions. 1 month after symptom onset) in 20% of cases. The
Children are the most common group affected, and area of the sinus involved determines the symptoms.
serious disease can occur in adults. Immunocom- The most common sinuses to be affected are the
petent children are treated symptomatically with superior sagittal sinus, the cavernous sinus, and the
acetaminophen for fever. Aspirin should be avoided transverse (or lateral) sinus. Thrombosis of other
in children with viral illnesses, as this may pre- sinuses is less common but may be underdiagnosed
dispose to Reye syndrome. Acyclovir is used in because of the difficulty in recognizing the manifes-
adults, immune-compromised patients, and when tations of thrombosis. MRI and magnetic resonance
there are signs of encephalitis or pneumonitis. VZV venography is the gold standard for diagnosis. Al-
vaccine is indicated for prevention and has no role in though changes consistent with cvr may be visible
acute management of evident disease. VZV immune on CT, its sensitivity is inadequate to exclude the
globulin is only indicated in immune-compromised disorder, as it may be normal in up to 30% of cases.
patients in conjunction with acyclovir. Heparin is the mainstay of therapy, even in patients
with evidence ofhemorrhage, although some patients
[!!) Answer D. "Hamman crunch" refers to the crun- may be candidates for catheter-directed thrombol-
ching sound heard during cardiac auscultation in the ysis. In general, CVf has a better outcome than
setting of pneumomediastinum. Radiographic ab- arterial stroke, although patients with deep cerebral
normalities are present in up to 90% of patients with and cerebellar venous thrombosis have poor out-
esophageal perforation, and include pneumomedi- comes. Women with cvr outnumber men by a ratio
astinum, subcutaneous emphysema, pleural effusion of 3:1, in part due to the use of oral contracep-
and pulmonary infiltrate. Pleural effusions may occur tives and because of the increased risk of venous
and are usually right sided if the perforation occurs thrombosis surrounding pregnancy and the imme-
in the midesophagus, but left sided if the perforation diate postpartum period. Although either focal or
occurs in the distal esophagus. Pneumoperitoneum generalized seizures may occur, most seizures are
may also occur if the patient has a rupture of the focal due to the focal irritation of the cortex affected
intra-abdominal esophagus. (Figure from Swi.schuk by the thrombosis.
L. Emergency radiology ofthe acutely ill or injured child,
2nd ed. Philadelphia: Lippincott Williams & Wilkins; tn) Answer D. Testicular torsion is the most likely
1986:63,77,79, with permission.) diagnosis because of the acute testicular pain, nau-
sea, and absence of ipsilateral cremasteric reflex.
~ Answer C. The patient has a medial malleolar frac- Pathophysiology involves twisting of the testis on the
ture with associated proximal fibular fracture-this spermatic cord due to an anatomic abnormality or
15 2 1000 Questions to Help You Pass the Emergency Medicine Boards

trauma. In unclear cases, color Doppler ultrasonog- and usually due to superinfection, most commonly
raphy is the diagnostic test of choice, but in textbook from pneumonia.
cases such as this, emergent urologic consultation is
indicated. Prompt diagnosis is essential, as testicular ~ Answer D. Adult patients infected with hepatitis
survival is directly dependent on duration of symp- B develop chronic hepatitis B <5% of the time.
toms-if surgical management is instituted within In contrast, patients with acute hepatitis C infec-
6 hours of pain, approximately 100% of cases are sal- tion become chronic carriers 80% to 90% of the
vageable. Definitive surgical management involves time. Hepatitis C is most commonly acquired by
bilateral orchidopexy. Antibiotics are used to treat intravenous drug use (IVDU) or blood transfusions.
epididymitis, a common condition in the differen- In the United States, both blood donors and the
tial diagnosis of the acute scrotum. Analgesics and donated blood are screened for hepatitis C, so the
antiemetics should be considered conjunctive ther- risk of infection through this route is quite low and
apy in patients with testicular torsion but do not IVDU predominates. However, worldwide, <40% of
affect the disease process itself. Manual detorsion the blood supply is tested for hepatitis C. Hepatitis C
may be used as a temporizing measure, but should is the least common viral cause of fulminant hepatic
never be considered definitive therapy. Lithotripsy is failure. Coinfection with hepatitis B and D is the most
used to treat kidney stones, and has no role in the common. The white blood cell count has almost no
management of testicular torsion. utility in the setting of hepatitis infection.

~ Answer A. H. capsulatum is the most common ~ Answer E. The fruit shown is of the plant, Datura
pulmonary fungal infection worldwide. It may infect stramonium, commonly referred to as jimson weed
immunocompetent as well as immunocompromised or thorn apple. Seeds inside the fruit contain
individuals, but primary infection is almost always belladonna alkaloids, including atropine and scopo-
asymptomatic and very few cases are ever brought to lamine, which cause anticholinergic symptoms. An-
the attention of a physician. In endemic regions, at ticholinergic crises are treated supportively, with
least 80% to 90% of the population has positive decontamination, IV hydration, benrodiazepines for
skin testing by the age of 20. Histoplasma and agitation or seizures, hyperthermia control, and
Blastomyces are endemic to the Mississippi and cardiac monitoring. Physostigmine is an acetyl-
Ohio River valleys in the United States, whereas cholinesterase inhibitor that may be used in select
Coccidioides is found in the arid southwest. In anticholinergic poisonings. Physostigmine increases
contrast, Cryptococcus is ubiquitous throughout the the amount of acetylcholine in the synaptic cleft,
world, without a specific distribution. It generally allowing it to compete with the anticholinergic agent
causes disease in imrnunocomprornised hosts, and for the acetylcholine receptor. Physostigmine is con-
is the most common cause of life-threatening fungal traindicated in patients with tricyclic antidepressant
infection in patients with human immunodeficiency overdoses, as it may precipitate intractable seizures
virus (HIV). Except in a few rare cases, patients and asystole. Pyridostigmine is a quaternary amine
with fungal pulmonary infection are not capable acetylcholinesterase inhibitor which does not cross
of transmitting disease to others. Of the fungi that the blood-brain barrier, rendering it useless as an
cause systemic mycoses, Blastomyces most commonly agent to reverse agitation in anticholinergic toxicity.
causes disseminated disease. Edrophonium is a short-acting acetylcholinesterase
inhibitor which is used mainly to diagnose myas-
~ Answer A. The patient has evidence of whooping thenia gravis (MG) by improving muscle strength in
cough caused by B. pertussis, a gram-negative coc- patients with MG and worsening muscle strength
cobacillus (like H. inftuenzae). The disease occurs in cholinergic crisis patients. Pralidoxime is used
in three phases-the catarrhal phase, a nonspecific in organophosphate overdoses, where it breaks up
URI-like syndrome lasting 1 to 2 weeks; the parox- the organophosphate-acetylcholinesterase complex
ysmal phase lasting up to 1 month, with paroxysms and frees acetylcholinesterase-this action would be
of coughing fits; and the convalescent phase lasting exacerbate the problem in patients with anticholin-
up to several months, with a chronic, intermittent ergic crises. (Figure courtesy of Robert Hendrickson,
cough. Antibiotic therapy with macrolides is usually MD. Reprinted with permission from Hendrick-
only effective in the catarrhal phase, but should be son R. Greenberg's te.xt-atkls of emergency medicine.
given to patients to reduce the high degree of conta- lippincott Williams & Wilkins; 2004:831.)
giousness. Corticosteroids and ,8-agoni.st nebulizers
may be useful as adjunctive therapy. Cultures are ~ Answer E. Hyperacute bacterial conjunctivitis is
useful only in the catarrhal phase, and have low sen- usually caused by Neisseria species. It is differentiated
sitivity during the paroxysmal phase. Mortality is low from ordinary bacterial conjunctivitis by the rapidity
Test5 153

of onset. Populations at risk include neonates and ~ Answer D. Spontaneous perforation occurs in
sexually active adults. Prompt diagnosis is essential only 15% of all esophageal perforations. It is
because of the rapid course and the ability of most commonly due to Boerhaave syndrome, which
gonococci to invade intact corneal epithelium. typically occurs in patients with a history of heavy
Treatment generally involves systemic and topical drinking or alcoholism and peptic ulcer disease. Most
antibiotics covering both Neisseria and Chlamydia, cases occur after multiple, prolonged episodes of
as 30% of patients are coinfected. Choices A, B, vomiting. Other causes of spontaneous perforation
and Care common causes of bacterial conjunctivitis may be due to the aftereffects of caustic ingestion,
in children. Klebsiella is not a common cause of pill ingestions (e.g., antibiotics, anti-inflammatories,
conjunctivitis. and bisphosphonates), esophageal cancer, aortic
aneurysms, and infections (e.g., herpes simplex,
[!lJ Answer A. Hepatitis B infection is the most com- Candida, TB). However, the most common cause is
mon viral cause of acute liver failure. However, instrumentation. Procedures that have the potential
worldwide, acetaminophen toxicity is the most com- to cause perforation include upper endoscopy, rigid
mon overall cause. Idiopathic causes are probably dilation, pneumatic dilation, sclerotherapy, and even
the third most frequent cause ofliver failure. the placement of nasogastric tubes. Symptoms may
not be present for several hours after perforation.
[!j) Answer B. This patient's symptoms of severe
hypertension in concert with a decreased pulse [!!) Answer A. Urethritis in men is commonly caused
and irregular respirations are consistent with the by N. gonorrhoeae and C. trachomatis. Symptoms
development of elevated intracranial pressure (ICP). of urethritis are dysuria and penile discharge.
Owing to his history of severe hypertension, he likely Gonococcal urethritis is almost always symptomatic,
suffered an acute intracerebral hemorrhage resulting whereas chlamydial infection can be asymptomatic in
in an acute increase in ICP and transtentorial (or one fourth of cases. Both gonococcus and Chlamydia
uncal) herniation. The oculomotor nerve (cranial coexist in up to half of the cases of urethritis.
nerve III) is compressed between the uncus and Diagnosis is made by urine Gram stain and PCR.
the tentorium cerebelli resulting in parasympathetic Gram-negative intracellular diplococci in the urine
paralysis and subsequent mydriasis ("blown pupil"). are diagnostic for gonococcal infection. Chlamydial
Tonsillar herniation results from an increase in infection is much more common in men younger
posterior fossa pressure (e.g., due to a posterior than 20. Treatment of urethritis covers both N.
fossa mass). Symptoms may be subtle or may result gonorrhoeae and Chlamydia---ceftriax.one IM with
in acute cardiorespiratory dysfunction (e.g., when a week of oral doxycycline. Sexual partners should be
a lumbar puncture is performed in the setting of treated as well.
an undiagnosed posterior fossa mass). Subfalcine
herniation occurs in association with a cerebral mass ~ Answer C. Although a high-probability V/Q scan
lesion, causing the medial surface of the affected can "rule in" a PE in the setting of a patient with
hemisphere to be pushed against the rigid falx cerebri an intermediate pretest probability, and a completely
and then herniate underneath it. The cingulate gyrus normal scan can rule out the disease, any other re-
is most commonly affected and it is often clinically sult will require further testing by CT pulmonary
silent. angiography or direct pulmonary angiography. Fur-
thermore, most patients undergoing V/Q scanning
~ Answer E. Lactated Ringers and 0.9 N NaCl are will fall into this intermediate group. Therefore, the
the most appropriate initial crystalloid fluids to best test in a patient with an intermediate probability
administer in trauma patients. Both are isotonic so- for PE is aCT pulmonary angiogram. n-dimer assays
lutions which effectively replete electrolytes. Lactated are only useful in the evaluation ofpatients with very
Ringers may be preferable to normal saline because low probability of having a PE. Elevated n-dimer
it is less acidotic, but no differences in outcomes levels cannot confirm a diagnosis of PE, and normal
have been elicited between the two. DlO water, DlO levels can only rule out disease in very low risk in-
normal saline, and D5 1/2 normal saline are hy- dividuals. Lower extremity Doppler ultrasonography
pertonic solutions containing glucose, which offer has been advocated by some as a means to evalu-
no survival advantage and may predispose to hy- ate the presence of venous thrombosis in the setting
perglycemia. Half-normal saline alone is hypotonic of inconclusive studies for PE. Regardless of their
and would eventually cause electrolyte abnormalities findings, however, lower extremity ultrasonography
and excessive peripheral edema ifgiven continuously cannot prove nor refute the presence of a PE. Arterial
during a trauma resuscitation. blood gas (ABG) has no role in the evaluation ofPE,
154 1000 Questions to Help You Pass tlu EmeTgfltlcy Medicine Boards

as PEs may be present regardless of the findings on ~ Answer E. Family history in older patients with
anABG. abdominal or flank pain is an extremely strong
risk factor-an affected first-degree relative puts an
~ Answer C. Cerebral edema refers to an increase individual at 20 times higher the risk than the general
in brain water content (rather than an increase in population. Other risk factors include age older than
cerebral blood flow) after head trauma. Cf can define 50, peripheral vascular disease, hypertension, and
the extent of cerebral edema. but the appearance on patients with other large artery aneurysms.
imaging may not correlate with clinical severity.
The two types of cerebral edema are vasogenic and ~ Answer C. The cyanotic heart diseases in children
cytotoxic. V asogenic edema is due to dysfunction can be remembered by the fact that each begins with
of the endothelial tight junctions which maintain the letter "T"-truncus arteriosus, transposition of
the blood-brain barrier. It usually occurs in the the great vessels, tricuspid atresia, tetralogy of Fallot,
white matter from a focal injury. Cytotoxic edema and total anomalous pulmonary venous return.
results from cerebral ischemia, which causes cellular Cyanosis generally indicates the presence of right-
membrane pump dysfunction and cellular death. to-left shunting, causing bypass of the pulmonary
Either form of edema may occur with any traumatic circuit and infusion of deoxygenated blood into
injury. Management of cerebral edema is handled the systemic circulation. Cyanosis due to cardiac
on a case-by-case basis and operative treatment is disease is generally central (truncal and facial), unlike
usually based on extent of increased intracranial dehydration or hypothermia, which usually causes
pressure (ICP) and clinical findings rather than the peripheral cyanosis (in the extremities). Evaluation
specific degree of edema observed on cr scan. and management of acute episodes of cyanosis from
congenital heart disease should be performed in close
~ Answer D. Although no trial has definitively eval- conjunction with a pediatric cardiologist.
uated the effect of aspirin when given immediately
after a TIA, aspirin has been shown to reduce the (!!) Answer E. This woman has an oblique fracture of
long-term risk of stroke and cardiovascular events the distal fibula at the level of the mortise as well as
by 22%. No other agent has been studied as thor- rupture of the deltoid ligament. It is clear that the
oughly as aspirin. The optimal dose of aspirin has deltoid ligament is disrupted because of the widened
yet to be determined, as doses ranging from 75 to medial joint space on the mortise view. Such an injury
1,300 mg demonstrate similar reductions in vascular would be classified as a Danis-Weber class B fracture
events (although the risk of intracerebral bleeding or a Lauge-Hansen pronation-abduction (eversion)
increases at high doses). Regular strength aspirin fracture. The need for surgical repair is often based
(325 mg) is probably adequate for most patients. The on whether the distal tibiofibular syndesmosis is
data regarding the immediate initiation of heparin intact. The squeeze test involves the application of
for patients with TIA and atrial fibrillation is mixed. near-circumferential pressure on the tibia and fibula
Finally, patients who are already taking aspirin and approximately 5 em proximal to the intermalleolar
who experience a TIA may be candidates for addi- axis. Pain in the ankle upon squeezing indicates
tional drug therapy such as clopidogrel. Consultation disruption of the syndesmosis, signifies an unstable
with a neurologist is suggested before starting such injury, and is an indication for operative repair in
therapy. this woman's case. The Maisonneuve fracture is also
an eversion injury resulting in a medial malleolar
~ Answer A. Although sinus tachycardia is the most fracture or rupture of the deltoid ligament in concert
common arrhythmia in patients with PE, normal with an oblique fracture of the proximal fibula. Any
sinus rhythm remains the most common cardiac patient with a medial malleolar fracture or deltoid
rhythm. A recent study demonstrated that roughly ligament rupture (medial joint space widening) as
two third of patients with PE have normal sinus well as lateral displacement of the fibula without a
rhythm (there was no significant difference from con- fracture of the distal fibula should be suspected of
trol patients). That does suggest, however, that most having a Maisonneuve fracture. (Figure reprinted
patients with PE have normal EKGs, as rhythm is with permission from Harris JH. The radiology of
only one aspect of EKG analysis. In fact, most pa- emergency medicine, 4th ed. Lippincott Williams &
tients with PEs have been found to have abnormal Wilkins; 1999:856.)
EKGs, but no single abnormality has been shown to
have sufficient sensitivity or specificity to aid in the l@?J Answer C. As for major depression, SSRis are the
diagnosis. Therefore, the role of EKG in the evalua- first-line drugs of choice for treatment of most anxi-
tion of PE is to rule out the presence of alternative ety disorders. They exhibit an excellent safety profile,
diagnoses, such as cardiac ischemia or pericarditis. are effective at managing symptoms, and lack abuse
Test5 155

potential. Benzodiazepines may be used for break- immunocompromised, either by virtue ofHIV infec-
through anxiety, but long-term use can result in tion, or because of diabetes, underlying malignancy,
abuse and withdrawal if stopped suddenly. Mon- chemotherapy, or treatment with corticosteroids. Pa-
amine oxidase inhibitors also work well, but dietary tients who were recently treated with broad-spectrum
restrictions can limit their utility in many patients. antibiotics are also at risk and there has been some
.f:I-Blockers are used only in patients with perfor- suggestion that the proton pump inhibitor, omepra-
mance anxiety. Tricyclic antidepressants used to be zole, may increase the risk of Candida esophagitis as
prescribed in the past for treatment of anxiety and well. Endoscopy is required to make an accurate di-
depression, but their poor safety profile, especially agnosis, as it will reveal the presence ofwhite plaques
in overdose, and frequent side effects have relegated (Candida) or herpetic vesicles. It also affords the en-
them to use as second- or third-line agents. Mood doscopist the opportunity to take biopsy specimens.
stabilizers are not useful in treating anxiety disorders.
~ Answer E. Hydatidiform moles are placental ab-
~ Answer A. Owing to cortical atrophy, elderly pa- normalities characterized by enlarged and edematous
tients have more intracranial space to accommodate placental villi and trophoblastic tissue (into grape-
the edema associated with large strokes. Therefore, like structures) as well as loss of fetal blood vessels.
herniation syndromes are more common in younger The two types of hydatidiform moles are complete
patients with less baseline atrophy. Overall, med- and incomplete. Complete moles almost always have
ical treatment of cerebral edema is poor. Patients a 46, XX karyotype that is paternally derived (due to
frequently receive corticosteroids, osmotherapy in duplication of a paternally derived haploid genome),
the form of mannitol or furosemide, or endotra- whereas incomplete moles have a complete trisomy
cheal intubation followed by hyperventilation as a with a karyotype of 69, XXX or 69, XXY. Incomplete
means of decreasing elevated ICP due to cerebral moles have two sets of paternal chromosomes (again
edema. None of these methods has been conclu- due to duplication of a paternally derived haploid
sively proved to improve outcome. Corticosteroids, genome) and one set of maternally derived chro-
in particular, have only been shown to increase the mosomes. Complete moles are so named because of
rate of infections, gastrointestinal bleeding, and hy- complete absence of fetal parts (no fetus, umbilical
perglycemia. The effects of hyperventilation last 12 cord, or amniotic membrane) and swelling of all pla-
to 36 hours, whereas osmotherapy lasts for 48 to cental villi. In contrast, incomplete moles have partial
72 hours. Cerebral edema following ischemic stroke placental villi swelling, and may have a few fetal parts
occurs within 24 to 36 hours but typically does not present and occasionally a complete fetus. The classic
peak for several days. features associated with hydatidiform moles are more
common in the setting of complete moles. These in-
~ Answer C. Patients with complete knee instability clude pregnancy-induced hypertension occurring in
after significant trauma are suspicious for having the first trimester, uterine enlargement greater than
a knee dislocation. The knee can spontaneously expected for dates, hyperemesis and very high .f:l-hCG
relocate and be mistaken for a severe sprain with levels, as well as theca lutein ovarian cysts. Vagina1
large effusion. The popliteal artery is at high risk for bleeding is the most common presenting symptom
damage, and angiography should be performed in and ultrasonography will reveal the diagnosis in both
every patient with knee dislocation. The presence of cases (molar pregnancy in the case of a complete
pulses in the foot does not rule out popliteal artery mole, and possibly missed abortion or intrauterine
injury, as up to 10% of patients with popliteal injury fetal demise in the case of an incomplete mole). The
will have intact peripheral pulses. The most common classic ultrasonographic appearance of a complete
nerve injury in knee dislocation is the peroneal nerve, molar pregnancy is described as a "snowstorm."
which may be tested with foot dorsiflexion and dorsal
foot sensation. ~ Answer B. Amebiasis occurs in 10% ofthe world's
population and amebic liver abscess is the most
[!ID Answer B. In order of decreasing frequency, the common extraintestinal manifestation of the disease.
small intestine, liver, and colon are the most Patients with amebic liver abscesses more commonly
commonly injured abdominal organs in the setting have an acute presentation than patients with pyo-
of penetrating abdominal trauma. genic liver abscesses. Transmission occurs through
the fecal-oral route and is usually due to contami-
~ Answer A. The Candida species, primarily Can- nated water or food products. Infections are always
dida albicans, are the most common cause of in- caused by Entamoeba histolytica and bacterial super-
fectious esophagitis. Affected patients are generally infections are uncommon. Alkaline phosphatase is
15 6 1000 Questions to Help You Pass the Emergency Medicine Boards

elevated in 75% ofpatients and aminotransferases are infection, and indication of inadequate tissue per-
increased in 50%. Elevated bilirubin levels are un- fusion (hypotension). Lack of improvement with
common and reflect biliary obstruction. In contrast aggressive fluid resuscitation predicts a more se-
to pyogenic liver abscesses, the treatment is non- vere course. Management of septic shock involves
surgical, and involves metronidazole 750 mg t.i.d. invasive monitoring to guide fluid resuscitation,
for 7 days. Following metronidazole, some author- broad-spectrum antibiotics, and vasoactive agent
ities recommend an additional course of a luminal therapy. Intubation with mechanical ventilation is
amebicide such as iodoquinol, diloxanide furoate, helpful in many cases to reduce the work of breath-
or paromomycin. (Figure reprinted with permission ing by the diaphragm and shunt this perfusion to
from Harris JH. The radiology ofemergency medicine, vital organs. Tube thoracostomy is not indicated
4th ed. Lippincott Williams & Wilkins; 1999:613.) if there is no evidence of parapneumonic pleu-
ral effusion. Angioplasty is not indicated in cases
~ Answer A. This patient has an auricular hematoma of septic shock. Surgical consultation is only in-
resulting from bleeding in the potential space be- dicated if there is a surgical cause for the septic
tween the auricular cartilage and the perichondrium shock, such as intra-abdominal abscess or necro-
to which it is normally adherent. It is important tizing fasciitis. Potassium chloride has no role in
to incise and drain these hematomas because fail- the management of septic shock in the absence of
ure to do so may lead to necrosis of the cartilage severe hypokalemia.
and subsequent cosmetic deformation of the ear
such as "cauliflower ear." This is the same princi- ~ Answer E. This patient has features of systemic
ple that guides treatment of nasal septal hematomas lupus erythematosus (SLE). Black women of child-
(cartilage necrosis). Though aspiration is a reason- bearing age are most at risk for developing SLE. This
able strategy, recurrence is common and aspiration is patient is presenting with the classic malar rash after
sometimes incomplete. Incision and drainage is more sun exposure in concert with fever and arthralgias.
definitive and simple to perform. After drainage of As in rheumatoid arthritis, arthralgias in SLE are
the hematoma, the ear is packed in a bulky pressure typically symmetric and most commonly involve in
dressing to preventreaccumulation ofthe hematoma. the fingers, hands, wrists and knees. Fevers are very
Moist cotton is placed within the folds of ear and common in patients with SLE, and nearly all patients
then covered with bulky dry cotton. Several pieces will develop a fever at some point in their course,
of trimmed gauze are placed between the ear and although greater than one third will present with
the scalp. The ear is then bandaged circumferentially a fever. Patients with SLE have numerous autoan-
against the supporting gauze. The patient is dis- tibodies. However, testing for antinuclear antibody
charged with follow-up arranged within 48 hours to (ANA) is most sensitive as 99% of patients with SLE
assess for reaccumulation. (Figure courtesy of Kath- have a positive ANA. The positive predictive value
leen Cronan, MD. Reprinted with permission from of ANA testing suffers because 5% to 7% of healthy
Chung EK. VisUill diagnosis in pediatrics. Lippincott individuals will also test positive for ANA. Anti-Sm
Williams & Wilkins; 2006:96.) is the most specific antibody, with a specificity of
99% and a positive predictive value of 97%. How-
[!1) Answer D. The patient has evidence of fungal ever, its sensitivity is only 25% so it is not a good
balanoposthitis, or infection of the foreskin and screening test.
glans. Recurrent such infections may be due to
an immune-compromised state, most commonly [1001 Answer A. TB is a chronic, progressive, multi-
diabetes. Balanoposthitis most often occurs in pa- system illness caused by Mycobacterium tuberculosis,
tients who are uncircumcised and is usually due to an aerobic, acid-fast bacillus. Initial infection oc-
infection from typical skin flora. Urinalysis may be curs through respiratory particles from infected
indicated to evaluate for sexually transmitted infec- individuals. TB is characterized by primary infec-
tion. Treatment is with penile hygiene and antibiotics tion, during which the positive pressure differential
directed at the most likely organisms. Liver function (PPD) skin test turns positive, followed by reacti-
tests would not be helpful in evaluation of penile vation disease, when the clinical manifestations and
infection. Postvoid residual volume and intravenous pathology become evident. The overall most com-
pyelogram evaluate for urinary obstruction. Retro- mon symptom of pulmonary TB is cough, followed
grade urethrogram is used in patients with pelvic by weight loss, night sweats, chest pain, dyspnea,
trauma and suspected urethral injury. and hemoptysis. Patients rarely present to the ED
until they develop hemoptysis. Initial evaluation in-
r!!l Answer A. The patient is in septic shock, with cludes a chest x-ray, which has excellent negative
evidence of vital sign abnormalities, presence of predictive value except in HIV patients. Findings
TestS 157

on chest x-ray of primary TB can be indistinguish- antimicrobial therapy is given. If the patient is not
able from any lobar pulmonary infiltrate combined producing sputum, inpatient gastric aspiration of
with hilar lymphadenopathy. Reactivation TB char- swallowed sputum or bronchoscopic sampling is
acteristically demonstrates an apical infiltrate which indicated. Given the indolent, chronic nature of pul-
may be cavitary. Sputum samples for acid-fast bacilli monary TB infection, antimycobacterial agents need
analysis should be obtained in the ED before any not be started in the ED.
Test 6
Questions
[!] You are discharging a female patient from the emer- (A) Stop atropine, start epinephrine.
gency department (ED) with a diagnosis of tension- (B) Stop atropine, start vasopressin.
type headache when she asks you if there is anything (C) Stop atropine, start pralidoxime.
she can do to prevent these headaches in the future. (D) Continue atropine therapy alone.
Which of the following drugs would you prescribe? (E) Continue atropine therapy and add
pralidoxime.
(A) Valproate
(B) Neurontin
(C) Propranolol
[!) A 65-year-old man with a past history of hyper-
tension presents with sudden onset of shortness of
(D) Amitriptyline
breath 8 hours ago. He also reports orthopnea and
(E) Verapamil
lower extremity edema, but denies chest pain. He
saw his primary care physician who noted a new
[!] An 87-year-old woman is brought to the ED
diastolic murmur. Vital signs are: T 98.4, HR 110,
by her caretaker with dehydration and excessive
BP 115/76, RR 25, and Spo290% RA. Examination
somnolence. Her initial blood work reveals a sodium
reveals a patient in moderate respiratory distress,
level of 119 mEq/L and aglucose level of 1, 100. Which
diastolic murmur at the right upper sternal border,
of the following represents her actual sodium level?
bilateral crackles, and lower extremity pitting edema.
(A) 109 mEq per L The electrocardiogram (EKG) shows signs ofleftven-
(B) 119 mEqperL tricular hypertrophy (LVH), troponin I is negative,
(C) 125 mEq per L and chest x-ray demonstrates cardiomegaly and pul-
(D) 130 mEqperL monary edema. Which of the following is the most
(E) 135 mEq per L appropriate definitive therapy?
(A) Percutaneous transluminal coronary
[!] Which of the following best summarizes the role of
angioplasty
chest x-rays in the diagnosis of community-acquired
(B) Pacemaker placement
pneumonia (CAP)?
(C) Surgical valve replacement
(A) Validated clinical rules exist to determine which (D) Dobutamine
patients with respiratory symptoms require a (E) Tissue plasminogen activator
chest x-ray.
(B) The chest x-ray is the most important test in the [!) Which of the following is true regarding Bordetella
diagnosis of pneumonia. pertussis infections?
(C) Chestx-rays are useful to differentiate between
(A) Prophylaxis with erythromycin is
"typical" and "atypical" pneumonia.
recommended for adults who come into
(D) Lobar consolidation is the most common
contact with pertussis-infected individuals.
radiographic finding in patients with CAP.
(B) Almost all cases of pertussis in adolescents and
(E) Chest x-ray findings help dictate antibiotic
adults occur in previously unvaccinated
therapy.
patients.
(C) Immunization against B. pertussis confers
[!) A 56-year-old woman presents with acute organo-
lifelong immunity.
phosphate overdose, with severe bronchorrhea,
(D) Older children infected with B. pertussis have
bradycardia, and coma. She is intubated for airway
the most severe disease.
protection, and atropine therapy is initiated. After
(E) The clinical course in symptomatic adults is
10 mg of atropine, her heart rate (HR) is 130,
characterized by a mild cough that resolves
blood pressure (BP) is 160/90, and her secretions
within 3 to 7 days.
are still copious. Which of the following is the most
appropriate next step in management?

158
TII!St6 159

(1] Which of the following is useful as a sensitive (A) Y. enterocolitica gastroenteritis may mimic
screening test for clinically significant complications appendicitis.
of blunt cardiac injuryf (B) Unlike other causes of gastroenteritis, Yminia
(A) Creatine kinase, MB isozyme (CK-MB)
enterocolitis may last fur 2 weeks or longer.
(C) Some adults recovering from Yminia
(B) Troponin I
(C) Troponin T
enterocolitis may develop polyarthritis or
(D) EKG
erythema nodosum.
(E) Eurdse stress test (D) Y. en•ocoUtica infections are most common in
childhood.
(E) AU of the above.
00 A 4-year-old previoualy healthy girl presents to the
ED with a 2-day history ofcough productive ofyeUow
sputum. fevers up to 102.4°F, pleuritic chest pain,
[ll) Which of the following EKG changes may be seen in
the setting of hypercalcemia?
and dyspnea. On aamination, she is tachypneic and
mildly toxic appearing. Her chest x-ray reveals a (A) ST-.segment depression
right middle lobe infiltrate. Which of the following (B) QTc-segment shortening
organisms is the most likely cause of this problem~ (C) Widened T wave
(D) Bradycardia
(A) Klebsiell4 pneumoniae
(E) AU of the above
(B) Mycoplamul ptuUmoniae
(C) Chlamydia tradwmatis
(D) Respiratory syncytial virus (RSV) ~ Which of the following is true regarding retropha-
ryngeal abacesses (RPAs)?
(E) Strtptocoa:us pneumonitle
(A) RPAs are uaually preceded by foreign body
(!] An ED thoracotomy is performed on a trauma aspiration in children.
patient. Which of the following is the structure (B) Patients with RPAs generally prefer to lie
labeled in Pig. 6- u supine.
(C) Prevertebral soft tissue swdling in excess of
22 mm at the level of C2 is diagnostic fur an
RPA in children and adults.
(D) Mycobacterium &pp. are the most common
cause of RPAs.
(E) Atlantou:i.al separation is the moat common
fatal complication ofRPAs.

[!ID Which of the following is true about treatment of


multiple sclerosis (MS)?
(A) Intrathecal baclofen may be useful for the
spasticity associated with MS.
(B) Patients with acute exacerbations become less
responsive to corticosteroids over time.
(C) Treabnent ofoptic neuritis with oral
prednisone alone may increase the rate of
recurrence.
(D) High-dose methylprednisolone has been shown
to speed recovuy from acute exacerbations but
Ftgu... l-1. does not ai&ct long-term outcome.
(E) AU of the above.

(A) Vagus nerve [H) A23-year-oldmanwithahistoryofhumanimmuno-


(B) Phrenk nerve deficienq virus (HIV) presents with shortness of
(C) Sympathetic chain breath, fever, and malaise. A chest :x-ray is shown.
(D) Spinal accessory nerve Arterial blood gas shows a Pao2 of 60 mmHg. Which
(E) Inferior vena cava of the following, in addition to antibiotics, is the
moat appropriate therapy in Fig. 6-2?
(!gJ Which of the following statements about Yminia (A) Albuterol
enterocolitica infection is truef (B) Prednisone
(!!] Which ofthe following is true regarding appendicitis?
(A) Nearly all patients with appendicitis younger
than 3 yean have evidence of perforation at the
time of appendectomy.
(B) Most patients younger than 2 years have diffuse
abdominal tendemeSI.
(C) An appendicolith is only seen in approximately
15%ofcues.
(D) Computed tomography (Cf) scan ia the
diagnostic test of Ghoice.
(E) All of the above.

f!!) A 6-year·old boy presents with diffuse arthralps,


fatigue, and fever for several days. He had a "virus"
with fever and sore throat seve:ral weeks before,
which resolved spontaneowly. Physical aamina-
tion demonstrates a febrile child with significant
tenderness and limited range of motion in his left
Figure ~2. knee with milder findings in his right wrist. Which
of the following the mort appropriate next rtep in
management?

(C) Aspirin
(A) Discharge home with azithromycin.
(B) Prednisone 1 mgperkg
(D) Vasopressin
(C) BKG
(E) Hyperbaric oxygen
(D) Urinalysis
(E) Lumbar puncture
~ o-dimer levels may be elevated in which of the
following?
~ Which of the following opioids may predispose to
(A) Elderly patients serotonin syndrome?
(B) Pregnancy
(C) Multiple blunt trauma victims
(A) Fentanyl
(B) Meperidine
(D) Postoperative patienu
(C) Propoxyphene
(E) All of the above
(D) Hydrocodone
(E) Morphine
1!!1 Whicll of the following has been shown to increase
the ri&k of brain tumors?
(HI Which of the following is the most appropriate suture
(A) High-tension power lines to use for gaping intraoral lacerations involving the
(B) Ionizing radiation mucosal surface?
(C) Head trauma
(D) Cellular telephone use
(A) 6-0 nylon
(B) 6-0vicryl
(E) All of the above
(C) 4-0nylon
(D) 4-0 vicryl
I!!) A 26-year-old pregnant woman at 8 weeks by dates (E) 1rl) silk
presents to the ED with a chief complaint of nausea
and vomiting. Her abdomen and pelvic examinations
~ A 34-~ar-old woman who takes phenelzine for de-
are normal and her ultrasonograph reveals a normal
pression presents with 18itation, severe hypertension.
live intrauterine pregnancy at 8 weeks. Her urine
reveals no ketones. Which of the following is the best mydria.m, and hyperthermia. Which ofthe following
recommendation for reducing her symptoms?
foods did she most likely eat before presentation?
(A) Oranges
(A) Diazepam
(B) Apples
(B) Promethazine
(C) Grahamc.rackera
(C) Vitamin B6
(D) Cheese
(D) Ondansetron
(E) Ice cream
(E) Dexamethasone
Test6 161

~ A 6-month-old infant born at term is brought by her (A) Age older than 85 years
parents for evaluation of cough. The patient has been (B) Pulse > 125
coughing for 3 days, with rhinorrhea and congestion (C) Systolic BP > 185
and fevers up to lOl"F. Her past medical history (D) Known coronary artery disease (CAD)
and birth history are unremarkable. Vital signs in the (E) None of the above
ED are lOO.l "F, 133,42, 90/palp, and 98% RA. The
patient is smiling, active, and tachypneic with mild ~ The use of noninvasive positive pressure ventilation
nasal flaring, and lungs exhibit moderate expiratory (NIPPY) in the setting ofan acute chronic obstructive
wheezes. Which of the following is true regarding pulmonary disease (COPD) exacerbation has been
this patient's condition? shown to:
(A) Albuterol therapy has been shown to reduce (A) Decrease the need for endotracheal intubation.
hospitalization rates. (B) Decrease the duration of mechanical
(B) Ribavirin is indicated. ventilation.
(C) Respiratory syncytial virus (RSV) is the most (C) Decrease the length of intensive care unit
common cause. (ICU) stay.
(D) Corticosteroids reduce the duration of illness. (D) Do all of the above.
(E) The patient has an 80% chance of developing (E) Do none of the above
asthma as a child.
~ Which of the following is an expected laboratory
~ The most commonly encountered anorectal problem abnormality in patients with the (HELLP) syndrome?
encountered in infimts is (A) Decreased hemoglobin
(A) Anal fissure. (B) Elevated prothrombin time (PT)
(B) Hemorrhoids. (C) Decreased fibrinogen
(C) Fistula. (D) Elevated partial thromboplastin time (PTI)
(D) Ischiorectal abscess. (E) All of the above
(E) Pilonidal cyst.
~ Which ofthe following findings on urinalysis is found
in pyelonephritis but not cystitis?
~ Which ofthe following drugs may pose problems for
a patient with asthma? (A) White blood cells (WBCs)
(B) WBC casts
(A) Naproxen
(C) Nitrites
(B) Metoprolol
(D) Leukocyte esterase
(C) Enalapril
(D) Aspirin (E) Bacteria
(E) All of the above
@!) Which of the following is true regarding toxic
alcohols?
~ A 42-year-old anxious woman presents to the ED
with low back pain radiating down her left leg. She (A) Alcohol dehydrogenase has greater affinity for
states that she has a history of a herniated disk but methanol than for ethanol.
has never had advanced imaging or surgery. While (B) Alcohol dehydrogenase has greater affinity for
the patient is lying supine, you lift her right leg ethylene glycol than for ethanol.
to approximately 45 degrees, causing the patient to (C) Alcohol dehydrogenase has greater affinity for
complain of pain radiating down her left leg below ethylene glycol than for methanol.
the knee. How do you interpret this result? (D) As little as one tablespoon of 40% methanol
may be lethal in adults.
(A) The patient is malingering.
(E) Gastrointestinal (GI) absorption of both
(B) The patient probably has a vertebral
methanol and ethylene glycol takes 4 to 5 hours
compression fracture.
(C) The patient has cauda equina syndrome.
~ A 26-year-old woman at 31 weeks' gestation is
(D) The patient has a lumbar radiculopathy
brought to the ED after a high-speed motor vehicle
probably caused by a left-sided herniated disk.
accident. She was an unrestrained passenger but
(E) The patient has crossed sensory nerve fibers
was not ejected from the vehicle. She is brought
resulting in paradoxical left-sided pain.
in lying on a backboard with a c-collar and is
complaining of difficulty breathing. Her room air
~ Which of the following is an absolute contraindica- oxygen saturation reads 82%, although it increases
tion to the use of epinephrine in anaphylaxis?
to 93% on a nonrebreather. Just before intubation, a
162 1000 Qwstimu to Hdp You Ptw the Emerpnt;y Maicitw Bo.rmfs

chest x-ray was taken and is shown in Fig. 6-3. Which the nurse is performing his assessment in the room,
of the following is true? the patient begins to seize. The nat best step in
management is
(A) Hydralazine 10 IDS IV push.
(B) Lorazepam 2 mg per minute IV push.
(C) Phenytoin 20 mg per kg IV.
(D) Magnesium 6 g slow IV push.
(E) Labetalol20 mg slow IV push.

(ij) Erysipelas ...


(A) Is almost alwaya caused by S. aureus.
(B) Most frequently involves the face.
(C) Rarely recurs.
(D) Is an infection of the dermis that involves the
lymphatic system.
(E) Is all of the above.

(ill A 23-year-old woman with a history of sickle


cell disease presents with fever, chills, cough, and
(A) Olest tubes should be placed in the fifth dyapnea. A cheat x-ray demonstrates a focal infiltrate
intercostal space at the midaxillary line. in the right lower lobe. Which of the following ia the
(B) Olest tubes should be placed over the anterior most appropriate management at this time?
chest wall. (A) Heparin
(C) Chest tubes are contraindicated during (B) Tissue plasminogen activator
pregnancy. (C) Percutaneous transluminal coronary
(D) Olest tubes should be deferred until fetal angioplasty
evaluation is complete. (D) Albuterol and prednisone
(E) Ole!t tubes should be placed higher in (E) Intravenous fluids
pregnant than in nonpregnant women.
(iii A group of children are playing outside in the rain
~ Pulmonary infections with which of the following when one ofthem auffera a witnessed lightning strike.
may be transmitted from person to person? The strike did not directly hit the child but it hit the
(A) Coxiella burnetii ground very close to where the child was standing.
(B) Yersinia pestis Which of the following is true?
(C) HistuplasmR C"4psulatum (A) Ventricular fibrillation ia the most common
(D) Fnmc:isella tulamuis cause of death.
(E) Bucillus cmthmc:is (B) He is likely to suffer from severe, deep bums.
(C) Lower extremity paralysis accompanied by
~ A 55-year-old man presents with severe chest pain mottled, blue, cool, and pulseless extremities
radiating to the back. A cr scan of the chest reveals typically reaolvea without treatment.
both ascending and descending aortic diaaection. (D) "Flashover" commonly causes diffuae
Which of the following is true regarding this patient? superficial burna to >50% of the total body
(A) Early, aggressive BP control is lilcel.y to be surface area.
detrimental to outcome. (E) Myoglobinuric renal failure is the most
(B) El:ne:rJmt SUfBCIY is warranted. common complication amongst survivors.
(C) Aspirin should be given in case the dissection
lw extended to the coronary arteries. ~ A 26-year-old GzP:z presents to the ED 3 daya after
(D) Heart rate (HR) should be kept >100 to spontaneous vaginal delivery of a healthy male infant
:m.uimize cardiac output. with a chief complaint of crampy low abdominal
(E) Transthorack echocardiogram has the best pain and a foul smelling vaginal discharge. On
specificity to make the diagnosia. enmination, she lw a fever of 102°F, and a tender
uterus on bimanual pelvic examination. Which of
~ A 36-year-old primigravida presents to the ED at the following is true?
32 ~eks' gestation with epigastric pain. Her BP is (A) This condition is more common after vaginal
150/100, but the other vital signs are normal While delivery than cesarean section.
Test6 163

(B) She has postpartum pelvic inflammatory (A) Thought disorder


disease. (B) Recreational drug use
(C) Chlilmydia and Mycoplasma are the most (C) Anxiety disorder
common etiologic agents. (D) Somatoform disorder
(D) Premature rupture of membranes (PROM) is a (E) Factitious disorder
risk factor for her condition.
(E) All of the above. ~ Which ofthe following is true ofPrinzmetal's angina?
(A) It is not relieved by nitroglycerin.
~ Which of the following patients is at highest risk of (B) It may be relieved by exercise.
developing a lung abscess? (C) The pathophysiology involves acute plaque
(A) A 28-year-old HIV+ man with a recent CD4+ rupture with thrombosis.
T -cell count of 301 (D) Characteristic EKG changes usually distinguish
(B) A 47-year-old woman status post lumpectomy Prinzmetal's from acute myocardial
for breast cancer infarction (MI).
(C) A 68-year-old woman drunk alcoholic with (E) J3-Biockers are contraindicated
diffuse caries
(D) A 32-year-old man with a history of ~ Which of the following is the most common
polypharmacy abuse including IV drug use symptom seen in pulmonary embolism (PE)?
(E) A 72-year-old woman with Parkinson disease (A) Dyspnea
(B) Chest pain
[!!) Which of the following diseases produces palpable (C) Unilateral lower extremity edema
purpura? (D) Palpitations
(A) Idiopathic thrombocytopenic purpura (ITP) (E) Hemoptysis
(B) Thrombotic thrombocytopenic purpura (TIP)
(C) Henoch-Schonlein purpura (HSP) ~ Which of the following is most effective in reducing
(D) Rocky Mountain spotted fever (RMSF) mortality from acute MI?
(E) All of the above (A) Metoprolol
(B) Aspirin
~ The drug ofchoice for sedation in the setting of acute (C) Nitroglycerin
delirium is (D) Abdximab
(A) Haloperidol. (E) Morphine
(B) Diazepam.
(C) Diphenhydramine. ~ A third-year medical student presents to the ED with
(D) Morphine. diffuse arthralgias of the hands, wrists, and knees.
(E) Promethazine. She has been taking isoniazid (INH) because she
was exposed to a patient with active tuberculosis
~ Which of the following statements about Pseu- and subsequently had a positive purified protein
domonas aeruginosa is correct? derivative (PPD) test. She is most likely suffering
(A) Most patients with cystic fibrosis (CF) are from a syndrome mimicking:
ultimately colonized with P. aeruginosa and are (A) Systemic sclerosis.
susceptible to infection. (B) Systemic lupus erythematosus (SLE).
(B) Ceftriax:one plus azithromycin, a common (C) Gouty arthritis.
front-line regimen for community-acquired (D) Rheumatoid arthritis.
pneumonia (CAP), provides antipseudomonal (E) Sjogren syndrome.
coverage.
(C) P. aeruginosa is less common among patients [!!) The amount of time after which a limb exposed to
admitted to the ICU with severe pneumonia. ischemia at room temperature ("warm ischemia")
(D) P. aeruginosa is an important cause of begins to develop irreversible damage is
malignant otitis media. (A) 1 hour.
(E) P. aeruginosa has only a small role in causing (B) 3 hours.
nosocomial infections. (C) 6hours.
(D) 12 hours.
~ Which of the following is the most common class of (E) 24 hours.
psychiatric disorder seen by primary care physicians?
~ A 56-year-old man with chronic hepatitis B presents that started 12 hours ago. She looks uncomfortable,
with mild abdominal paiD. wei8ht loss, and weakness. prcfua to sit in a dark room and states this headache
A cr scan of his abdomen r~ed a hypodense is more severe than any headache she has had before.
non in the right lobe of his liver suspicious for Her CT scan ia shown in Pig. 6-4. Which of the
hepatocellular carcinoma. Which of the following following is true?
blood tests is likely to be helpfuU {A) Seizures may occur in up to one third of
(A) Beta human chorionic gonadotropin patients.
(JJ-hCG) level {B) Lumbar puncture should be pe:rfonned for
(B) Serum total estradiol level cerebroapinal fluid (CSF) analysi&.
(C) Alpha fetoprotein (AFP) level (C) Nifedipine 60 mg PO should be given as soon as
(D) Carcinoemb.ryonic antigen (CEA) the CT scan result ia obtained.
(E) Cancer antisen (CA) 19-9 (D) Hypertension should only be treated if her BP
exceeds 220/120.
I!!J A 56-year-old man presents with generalized &tigue, {E) All of the above.
weakness, and vomiting. He tells you that he has
taken an overdose of his doxepin medication. His ~ A 25-year-old man is punched in the fiu:e at a bar and
BP is 155/95, and his EKG demonstrates a regular, presents to you with dental pain. On examination,
wide-complex tachycardia. Which of the following is his right lower :first premolar has a fracture exposing
the moat appropriate next step in ma.nasement? yellowiJh surface. No blood is seen on the tooth.
Which ofthe followi.ns is the correct type offracture
(A) Cardioversion at 50 J
(B) Lidocaine
and what is the proper management?
(C) Procainamide (A) Ellis I; follow up in dental clinic in 1 week.
(D) Sodium bicarbonate {B) Ellis I; follow up in dental clinic next day.
(E) Propafenone (C) Ellis II; follow up in dental clinic in 1 week.
(D) Ellis II; foUow up in dental clinic next day.
!HJ A 47-year-old &:male smoker with a history of (E) Ellis III; immediate dental consult.
hypertension presents to the ED with a headache
~ Which ofthe foUowingis the most commonly broken
carpal boncl
(A) Lunate
{B) Triquetrum
(C) Trapezoid
(D) Trapezium
{E) Scaphoid

@!) Which of the followiq ia the most common cause of


death among African American adolescents?
(A) Infection
{B) Cancer
(C) Motor vehicle collision
(D) Gunshot wound
(E) Drug overdoae

~ Which of the followiq is the most common


milufiagnosis in cases of missed acute appendicitis
in pediatric patients?
(A) Mesenteric adenitis
{B) Intussusception
(C) Gastroenteritis
(D) Inflammatory bowel disease
{E) Pancreatit:is
Tm6 165

~ A 4-year-old girl presents with signs and symptoms in his mid-right ureter. Which of the following is
of cystitis. Which of the following is the most true regarding this patient?
appropriate initial treatment? (A) He is likely to pass the stone without further
(A) Amo:xicillin-clavulinic: acid medical intervention.
(B) Ciprofloxacin (B) Urinalysis is likdyto be completely normal.
(C) Doxycycline (C) The stone has already traversed the narrowest
(D) Trimethoprim-sulfametho.xazole portion of the ureter.
(E) Gentamicin (D) Strict fluid restriction is the management of
choice.
~A 67-year-old man with hypertension presents (E) The stone is most likely composed of cystine.
with acute onset of abdominal pain. The pain
is periumbilical and radiates to the left lower ~ A44-year-old alcoholic man presents with shortness
quadrant. On physical examination, BP is 140/90, of breath, fever, and productive cough. Chest
and an abdominal mass is noted near the umbilicus. x-ray demonstrates a left lower lobe in1iltrate. The
A bedside ultrasonograph is shown in Fig. 6-5. After diagnosis of pneumonia is made. Which of the
the ultrasonography, the BP drops to 70/40 and the following is the most likely cause?
patient becomes ligb.theaded and dizzy. Which of (A) StaphylocomlS aureus
the following is the most appropriate next step in (B) S. pneumoniae
management? (C) K. pneunwnU.e
(D) Mycob4cterium tuberculosis
(E) M. pneumoniae

lii) Which ofthe following is the correct diagnosis ofthe


injury shown in Pig. 6-6?

Figure 6-5.

(A) CT scan. without contrast


(B) CT scan with IV contrast only
(C) CT scan with IV and PO contrast
(D) Emergent surgery
(E) IV crystalloid to normalize BP

~ Which of the following most commonly complicates


normal labor and delivery!
(A) Pace presentation
(B) Breech presentation
(C) Shoulder dystocia Rgure 6-6.
(D) Brow presentation
(E) Abnormal fetal lie
(A) Trigger finger
(!g) A 45-year-old man presents with acute onset of (B) MaD.et finger
left flank pain. He is extremely uncomfortable and (C) Bennet fracture
writhing around in pain. After appropriate pain (D) Jeneyfinger
control, he is sent for a cr scan of the abdomen (E) None of the above
and pelvis, which demonstrates a 2-mmkidneystone
16 6 1000 Questions to Help You Pass the Emergency Medicine Boards

~ Which of the following is true regarding hyper- ~ The treatment of choice for scabies (Sarcoptes
kalemia? scabiei) is
(A) Neither calcium chloride nor calcium gluconate (A) Permethrin 5% cream.
should ever be used in the setting of (B) lindane 1% lotion.
concomitant digoxin use. (C) Malathion 0.5% lotion.
(B) The effects of calcium chloride or gluconate last (D) Fluconazole 150 rng PO.
for 3 to 4 hours. (E) Ivermectin 200 mgper kg PO.
(C) Bicarbonate therapy is more efficacious than
either insulin or albuterol. ~ A 19-year-old woman presents with unilateral
(D) Sodium polystyrene sulfate (Kayexalate) may purulent discharge out of her left eye that started
exacerbate volume overload. the day before presentation and is worse now.
(E) All of the above. She has blurry vision until she is able to wipe
the pus out of her eye. On review of systems, she
~ Which of the following is true regarding the physical notes nausea, intermittent fevers for the past few
examination for patients with an abdominal aortic days, lower abdominal pain, and dysuria. Slit lamp
aneurysm (AAA)? examination demonstrates a purulent discharge, but
(A) Abdominal bruits are audible in half the no corneal or anterior chamber abnormalities. Which
number of cases. ofthe following is the most appropriate management
(B) Aneurysmal rupture often occurs with deep strategy?
palpation of the abdomen. (A) Topical antibiotics, follow up with
(C) Most aneurysms >5 em in size are palpable. ophthalmology in 2 days
(D) Femoral pulses are usually decreased. (B) Topical antivirals, follow up with
(E) Abdominal obesity does not appreciably affect ophthalmology in 2 days
the ability to palpate an aortic aneurysm. (C) Systemic antibiotics, admission to hospital
(D) Systemic antivirals, admission to hospital
~ Which ofthe following is the most common etiology (E) Topical and systemic antibiotics, admission to
of death from child abuse? hospital
(A) Retroperitoneal hemorrhage
(B) Hemothorax [!g) A 28-year-old man presents to the ED stating that
(C) Intracranial hemorrhage he drank a whole bottle of antifreeze 4 hours before
(D) Burns presentation. He had drunk a fifth of liquor just
(E) Drowning before drinking the antifreeze. Except for moderate
intoxication, he is asymptomatic and his vital signs
~ The typical sequence of color changes in the fingers and physical examination are normal. Which of
of patients experiencing Raynaud's phenomenon is the following is the most appropriate next step in
management?
(A) Blue to white to red.
(B) Red to white to blue. (A) Discharge him without further testing.
(C) White to red to blue. (B) Check an oxalic acid level and discharge him if
(D) White to blue to red. <50 mg per dL.
(E) Blue to red to white. (C) Check urine for crystals and discharge him if
negative.
l!Z] A previously healthy 5-year-old boy presents with (D) Check urine for fluorescence and discharge him
painless rectal bleeding. The bleeding seems to have if negative.
resolved but his mother states that he had four or (E) Check an ethanol level and administer
five large, brick-colored stools earlier in the day. His fomepizole if negative.
stool guaiac test is positive. Which of the following is
the most likely cause of his symptoms? [!!) A 44-year-old man presents with hypotension after
a motor vehicle collision. His chest x-ray is normal.
(A) Duodenal ulcer
Pelvis x-ray is shown in Fig. 6-7. Which of the
(B) Meckel's diverticulum
following is the most important next step in
(C) Esophagitis
management?
(D) Anal fissure
(E) Inflammatory bowel disease
TII!St6 167

(C) N. meningitides
(D) N. gonorrhoeae
(E) P. aeruginosa

~ Which ofthe following arteries supplies the atrioven-


tricular (AV) node in most peoplcl
(A) Left coronary artery
(B) Right coronary artery
(C) Left anterior descending artery
(D) Left circumflex artery
(E) Right poo.erior descending

[MJ Which of the following is tint-line therapy for seizure


prophyl.uis in the patimt with head injlll')1
(A) Pentobarbital
Figure 6-7. (B) Phenobubital
(C) Phenytoin
(D) Valproate
(A) Poley catheterization
(E) Carbamazepine
(B) CT scan of the abdomen and pdvis
(C) Tightening a bedsheet around the pelvis
(D) ED thoracotomy ~ Which of the following is true in alcoholic ketoaci-
dosis (AKA)?
(E) Inlet and outlet radiographs of the pelvis
(A) Inaulin is useful in management.
~ A 34-year-old woman presents with fever, headache, (B) The al.coholl.evd is usually > 100 m.g per dL.
and blurry vision. Physial. examination reveals com- (C) The oamolal gap is usually elevated.
plete oculomotor paralysis bilateraD.y. A cr scan of (D) fJ -hydroxybutyrate is the primary .ketone
the brain with N contrast is shown in Fig. 6-8. Which responsible for the acidosis.
of the following is the most likdy etiologic aga1t1 (B) All of the above.

[!!) Which of the following findings is present in most


patients with endocarditis?
(A) Signa of embolic stroke
(B) Painful subcutaneow nodules on finger pads
(C) Heart murmur
(D) Splenomegaly
(E) Peteclllae

1!1) A 31-year-old G,.P, presents to the ED in active labor


at 38 weeks' gestation. On examination, the patient
is maximally dilated and the umbilical cord is noted
at the cervical opening. Which of the following may
be helpful?
(A) Place the mother on a stretcher in
Trmdelenburg position.
(B) Place a Foley catheter and instill the bladder
with 750 mL ofsaline.
(C) Place the mother's legs in a knee to chest
position.
(D) FJevate the presenting fetal part to relieve
figure 6-a. compression on the umbilical cord.
(E) All of the above.

(A) S. aumLS
(B) H. inftuenz~~e
168 1000 ~ tDHdp YouPo~W the~ MetlidMBoGrds

(!iJ A patient presenting to the ED with which of the (jj) A 65-year-old man with end-stage renal disease
followins is least likdy to have audible wheezes on (ESRD), who is on hemodialysis, presents with
physical examination? progressively worsening dyspnea over the last 3 days.
(A) Congestive heart failure (CHF) Which of the following is the most likely cause for
(B) COPD his symptoms?
(C) Aspirated. foreign body (A) Ml
(D) Asthma (B) Volume overload
(E) Sarcoidosis (C) Pneumonia
(D) Pulmonary embolism (PE)
(!!) What is chalcosis? (E) Diabetic ketoacidosis
(A) A teardrop pupil associated with incomplete
closure of fetal ocular cleft. {jiJ A 25-year-old woman presents with progressively
(B) An inflammatory reaction to worsening right eye vision loss and pain with ex-
copper-containing intraocular foreign body. traocular movements over the last 3 days. An af-
(C) Streaming fluorescent dye seen on slit lamp ferent papillary defect is present on the right, and
examination. direct fundoscopic examination reveals a swollen
(D) Shimmering ofiris associated with posterior optic disc. You make the diagnosis of optic neu-
lens dialocation. ritis. The patient is very concerned that her vision
(E) Double pupil seen with blunt trauma. loss may be an early sign of MS. You tdl her that
her risk of devdoping MS within 5 years is approx-
~A 57-year-old woman with hypertension who has imately:
not seen a doctor in 30 years presents with weakn.e&s (A) 5%.
and fatipe. The EKG is shown in fis. 6-9. Which (B) 25%.
of the following is the most appropriate next step in (C) SO%.
management? (D) 75%.
(A) Defibrillation at 200 J (E) 100%.
(B) Cardioversion at 50 J
(C) Lidocaine 150 mg IV {ij] Women are affected by gout:
(D) Calcium glu.conate 100 mglks IV (A) Veryrarely.
(E) No acute management, follow-up with primary (B) Most commonly in childhood.
care (C) Most commonly during their adult years.

Figure 6-9. (See color insert.)


TII!St6 169

(D) Most commonly in their postmenopausal years. [@!) An 18-year-old man presents with fever and left
(E) Mostly in the hip joints. periorbital pain and swellins as shown in Pig. 6-10.
Which of the following additional tests is indicated
~ Neglect or "hmri-inattention" usually indicates a at this time f
stroke in which of the following distributions?
(A) Frontal lobe
(B) Occipital lobe
(C) Left hemisphere
(D) Right hemisphere
(E) Brainstem

~ Comatose patients with bilateral miosis thought to


be due to narcotic overdose may sometimes be
confi.ued with patients with which of the follow-
OW.
(A) Lateral medullary syndrome
(B) Locked-in syndrome
(C) Pontine hemorrhage
(D) Paeudobulbarpalay
(E) Retinal detachment Flgun~ 6-10.

(@!] A 10-year-old. boy presents with chest pain. Which


of the following is the most likely cause? (A) Brain magnetic resonance imaging (MRI)
(A) Cardiac (B) Orbital Cf scan
(B) Gutrointatinal (GI) (C) Erythrocyte eedimentation rate (ESR)
(C) Psychogenic (D) Cerebrospinal ftuid ( CSP) analysis
(D) Musculoskeletal (B) Slit lamp examination
(E) Endoaine
~ A 17-year-old man presents with pain and swelling
~ Which of the following is true regarding anticholin- in his right knee. He denies any trauma. He had
ergic crises? pain in his left ankle the week before, and pain in
his right wrist before that. He had a sore throat last
(A) Seizures are seen more commonly in adults
month. which spontaneously resolved after 1 week.
than in c.:bildren.
Physical examination demonstrates a blowing api-
(B) Hyperthermia is a significant cause of death
cal systolic murmur and a tender effusion in his
in patients who receive adequate supportive
right knee. He complains of shortness of breath
care.
and chest pain, so an EKG is ordered. Whic:h of
(C) Reduced gastric motility is the earliest sign.
the following is the most likely EKG abnormality
(D) Sympathomimetic and anticholinergic crises
in this patient?
are clinically indistinguishable.
(E) Nicotinic antagoniam exceeds mUBCarinic (A) Prolonged PR interval
antagonism. (B) Wide QRS complex
(C) ST depression
(@!1 Which of the following is true regarding cancer of (D) Prolonged QT interval
the biliary tree? (E) Sl-Q3-T3 morphology
(A) Cholangiocarcinoma is the most common
~ Which ofthe following is the optimal vascular access
primary biliary tract .tJJalipncy.
for adult trauma patients?
(B) Metastatic disease to the biliary tract is more
common than any primary malignancy. (A) Single 1-t~g peripheral IV
(C) Porcelain gallbladder is a risk factor for (B) Double 16-g peripheral IVs
gallbladder carcinoma. (C) Double 18-g peripheral IVs
(D) The m oat common presenting symptom of (D ) Single triple lumen catheter
gallbladder carcinoma is pruritus. (E) lntraosseus catheter
(E) All of the abow.
170 1000 Questions to Help You Pass the Emergency Medicine Boards

~ Which of the following is the biggest risk factor for (A) Hypercalcemia is the most common electrolyte
delirium in the elderly? abnormality.
(B) Detection of serum myoglobin is the most
(A) Hearing impairment
sensitive means of diagnosing rhabdomyolysis.
(B) Immobility
(C) Use of 3-hydroxy-3methylglutaryl coenzyme A
(C) Underlying depression
(HMG-CoA) reductase inhibitors ("statins")
(D) History of multiple falls
may cause myalgias but not rhabdomyolysis.
(E) Underlying chronic dementia
(D) Rhabdomyolysis occurs in 90% of patients who
survive a high-voltage electrical injury.
~ A 22-year-old male basketball player presents to the
(E) Influenza and Legionella are the most common
ED after developing sudden shortness of breath and
infectious causes of rhabdomyolysis.
a painful sensation on the left side of his chest, which
worsens with breathing. Initial chest x-ray reveals a
~ Which of the following mushroom toxins is known
15% pneumothorax at the left apex. The patient's
to cause hepatotoxicity?
vitals are 128/72, P 95, RR 22 and shallow, and Pox
of 97%. The best course of action is (A) Coprine
(B) Ibotenic acid
(A) Administer supplemental 02, and admit the
(C) Amatoxin
patient to the hospital for 24-hour observation
(D) Orellanine
with repeat chest films q 6 hours.
(E) Psilocybin
(B) Administer supplemental 0 2 , observe the
patient in the ED, and repeat a chest film in 3 to
~ Which of the following toxins is suggested by a fruity
6 hours.
odor to the breath?
(C) Discharge the patient home with routine
follow-up the next day with his Pneumocystis (A) Cyanide
carinii pneumonia (PCP). (B) Isopropanol
(D) Insert a chest tube and connect it to water-seal. (C) Salicylate
(E) Consult cardiothoracic surgery (CT)-surgery. (D) Acetaminophen
(E) Arsenic
~ A 32-year-old man presents to the ED after an
accident while cleaning a paint sprayer. He had been ~ A 34-year-old man with a history of leukemia on
using an industrial caliber paint sprayer to work on active chemotherapy presents with fever of 102.5°F.
the exterior of his house and was trying to dislodge He denies any symptoms except for fever and chills.
an apparent clog in the nozzle when he inadvertently Physical examination is normal except for the fever.
triggered the sprayer with his index finger over the Basic laboratory work is normal except for a total
nozzle. He now comes in with mild pain and a WBC count of 400 cells per mm3 • Which of the
nearly punctate wound at the tip of his left index following is the most appropriate next step in
finger. Which of the following is the next best step in management?
management? (A) Discharge home with oncology follow-up
(A) Tetanus prophylaxis, oral antibiotics, splint in (B) Discharge home with oral amoxicillin and
the "safe" position and discharge with oncology follow-up
orthopedic follow-up in 2 days. (C) Admission to hospital with broad-spectrum
(B) Tetanus prophylaxis, irrigate the wound with antibiotics
tap water, oral antibiotics, and discharge with (D) Admission to hospital with surgical
orthopedic follow-up in 2 days. consultation
(C) Hand surgeon consultation for immediate (E) Admission to medical ICU
operative debridement.
(D) Tetanus prophylaxis, oral antibiotics, incision, [!!) Which of the following is a risk factor for central
and drainage of the volar tip of the finger. retinal artery occlusion (CRAO)?
(E) Tetanus prophylaxis, oral antibiotics, and (A) Hypertension
digital block with thorough wound exploration (B) Systemic lupus erythematosus (SLE)
to determine the extent of injury. (C) Atrial fibrillation
(D) Diabetes mellitus
~ Which of the following is true regarding rhabdomy- (E) All of the above are risk factors.
olysis?
Test6 171

[100] A 34-year-old man presents after a high-speed (B) Place the patient in the left lateral decubitus
motor vehicle collision with shortness of breath. position.
A large flail segment is noted on his right lateral (C) Place a heavy weight on the flail segment.
chest. Which ofthe following is the most appropriate (D) Give the patient 100% oxygen by nonrebreather
therapy at this time? mask.
(E) Perform rapid sequence intubation.
(A) Place the patient in the right lateral decubitus
position.
Answers and Explanations
[!] Answer D. All of the drugs listed are useful for W Answer E. Organophosphates bind to and inhibit
migraine prophylaxis, but only amitriptyline has been acetylcholinesterase, causing a cholinergic syndrome
found useful in the prophylaxis of chronic tension- of systemic hypersecretion: bronchorrhea, diarrhea,
type headache. Propranolol is probably the first-line lacrimation, salivation, emesis, and incontinence.
agent for migraine prophylaxis, whereas verapamil is Effects on heart rate (HR) are variable. Mortality
the agent of choice for cluster headache prophylaxis. from organophosphate overdose is usually due to
hypoxia from excessive bronchorrhea. Treatment in-
00 Answer E. Because glucose exerts an osmotic valves high-dose atropine-the endpoint of atropine
pressure on cell membranes, water is extruded from therapy is the reduction of bronchial secretions.
cells into the intravascular space, thereby diluting Tachycardia and hypertension are not indications
plasma sodium. The sodium level must therefore to stop atropine therapy. Over time, the binding of
be corrected for the hyperglycemia by using the organophosphates to acetylcholinesterase becomes
following formula: irreversible. Pralidoxime acts to break up this com-
plex before this process (known as aging) occurs.
CorrectedNa+ = [Na+] + (1.6 X ([Glucose]-100)/100)

While this patient's plasma concentration is at [1) Answer C. The patient has CHF from acute aortic
the low end of the normal range, patients with regurgitation. The history ofa new diastolic murmur
hyperosmolar hyperglycemic syndrome are typically with signs of pulmonary edema and normal EKG
severely sodium deplete. This occurs because sodium and cardiac markers indicates a valvular etiology.
is lost in the urine as a result of extensive glycosuria. Surgical aortic valve replacement is the only definitive
treatment for these patients. Choices A and E are
[I) Answer B. There are no validated criteria to de- appropriate for ST-elevation myocardial infarction
termine which patients with respiratory symptoms (STEMI) and are not indicated here. Choice B is
should receive a chest x-ray. Furthermore, while the not indicated, as the EKG does not demonstrate
presence of a lobar infiltrate on a chest x-ray is a clas- dysrhythmia as a cause for the heart failure. Choice
sic finding in "typical" bacterial pneumonia, it may D may be indicated in cases of cardiagenic shock, but
also be present in cases of "atypical" pneumonia. is not a definitive treatment.
Conversely, in the setting of a patient with clinical
symptoms of pneumonia, a normal chest x-ray does [!) Answer A. The incidence of B. pertussis infection
not preclude a diagnosis of"typical" bacterial pneu- is rising, with the number of reported cases in the
monia. Therefore, chest x-rays are not useful to dif- United States increasing sixfold since 1980. This
ferentiate between typical and atypical pneumonia. has occurred despite immunization rates of 80%
However, abnormal findings on a chestx-rayremain among young children. Most of the increase is due
central to making a diagnosis of pneumonia in pa- to an increased number of adolescents and adults
tients with symptoms consistent with pneumonia. diagnosed with the disease. Furthermore, it is widely
Although the World Health Organization (WHO) thought that the true scope of the problem is grossly
has established wholly clinical criteria for the diag- underestimated because ofthe failure ofphysicians to
nosis ofpneumonia owing to the scarcity of resources recognize the illness, as well as their failure to report
in the developing world, most guidelines (e.g., IDSA, the illness when it is diagnosed. Almost all cases of
BTS) in developed nations include the presence of pertussis in adolescents and adults occur in patients
an infiltrate on a chest x-ray as central to the diag- who have been previously vaccinated or in patients
nosis. Lobar consolidation, however, is present in a who have been previously infected with B. pertussis.
minority ofchest x -rays in patients with community- Contrary to popular belief, neither B. pertussis
acquired pneumonia (CAP). Its presence, however, infection, nor vaccination with either the cellular
may point to S. pneumoniae or K. pneumoniae as the or acellular vaccine confers lifelong immunity. In
causative microbial agents. Chest x-rays that are per- fact, natural infection with B. pertussis results in
formed very early in the disease course or in patients approximately 15 years of immunity to reinfection.
who are very dehydrated may be falsely negative, but This is much greater than the 4 to 8 years' worth
most x-rays have at least some abnormality. Chest of immunity offered by the vaccines (the acellular
x-rays do not have a role in determining empiric vaccine, which is currently used in the United States
antibiotic therapy in CAP. offers a shorter duration of immunity than the
172
Test6 173

cellular vaccine, roughly from 4 to 6 years). The pneumonia in preschool-aged children (6 months
recommended vaccination schedule in the United to 5 years). Overall, viruses are the most common
States advises that infants are vaccinated at 2, 4, and pathogens causing pneumonia in this age-group,
6 months of age, with boosters at 18 months and with respiratory syncytial virus (RSV) being the
then again between 4 and 6 years of age. Therefore, most common, followed by parainfluenza and in-
most children born in the United States should fluenza viruses, as well as adenovirus and rhinovirus.
carry immunity through the ages of 10 to 12 years However, this patient did not present with a viral
of age. Because the most severe illness occurs in prodrome, and is mildly toxic upon examination.
children less than 1 year of age, adolescents and It is critical, therefore, to treat this patient with
adults have not been offered booster shots beyond antibiotics that targetS. Pneumoniae. Because of in-
childhood. However, because of the rising incidence creasing resistance amongst S. Pneumoniae isolates,
of recognized disease in adolescents and adults, as high-dose amoxicillin is the drug of choice, although
well as the likely enormous asymptomatic disease patients who are hospitalized may require ampicillin,
burden in this population, recommendations to offer cefuroxime, or cefotaxime delivered intravenously.
booster vaccinations to adults may be forthcoming. K. pneumoniae is an uncommon cause ofpneumonia
B. pertussis infection in adults ranges from subclinical in children though it can cause severe infections in
infection to a prolonged illness mostly characterized irnmunocompromised hosts. Community-acquired
by a nagging, paroxysmal cough, which may interfere Klebsiella is primarily a disease of debilitated older
with sleep. The mean duration of illness in adults is men with a history of alcoholism. M. pneumoniae
36 to 48 days. Erythromycin is the drug of choice is the most common pathogen causing pneumo-
for the treatment of B. pertussis infection, although nia in children aged 5 to 15 years. C. trachomatis
azithromycin and clarithromycin have been shown may cause pneumonia in infants aged 3 weeks to
to be equally efficacious with fewer side effects. 3 months, typically causing an afebrile, subacute
Trimethoprim-sulfamethoxazole is an additional interstitial pneumonia. RSV bronchiolitis and pneu-
alternative. Erythromycin is also recommended for monia are the primary causes for hospitalization
prophylaxis of individuals who have come into during the first year oflife.
contact with patients who are infected. It is thought
to be effective in preventing disease as long as it [!] Answer B. Emergency thoracotomy is indicated
is given before the onset of symptoms. Owing to in patients who have traumatic arrest in the ED
the decreased infectivity of B. pertussis as the disease or shortly before arrival from penetrating thoracic
progresses, prophylaxis is generally unnecessary in trauma. The initial incision of the chest wall be-
individuals who come into contact with a patient gins in the sternal area and sweeps along the su-
who has been symptomatic for >3 weeks. perior border of the rib all the way laterally to
the edge of the bed. The rib spreaders are then
[I) Answer D. Blunt cardiac injury results from blunt placed and the left lung is moved out of the
trauma directed at the sternum, usually from patients way to expose the pericardium. An incision to
striking the steering wheel in a motor vehicle col- the pericardium should be made anterior to the
lision. Patients with blunt cardiac injury (formerly prominent phrenic nerve, which is visible in the
known as cardiac contusion) may develop myocardial figure. Damage to the phrenic nerve may cause di-
stunning, CHF, dysrhythmia, and in rare instances, aphragmatic weakness and seriously impair respira-
when a coronary vessel is damaged, MI. The diagno- tory function. (Figure courtesy of Mark Silverberg,
sis should be suspected in any case of blunt thoracic MD. Reprinted with permission from Silverberg M.
trauma, but physical examination is often not re- Greenberg text-atlas of emergency medicine. Lippin-
vealing. The best screening tool for the diagnosis is cott Williams & Wilkins; 2004:22.)
EKG. Patients with any significant abnormal finding
on EKG should be admitted for observation, teleme- ~ Answer E. Y. enterocolitica most commonly causes
try monitoring, and confirmatory echocardiogram. an invasive disease of the terminal ileum with
Cardiac markers have been extensively studied to symptoms that are similar to other gastroenteritis
evaluate for screening or confirming the diagnosis syndromes (watery diarrhea, anorexia, crampy ab-
but are not particularly useful in either regard. Stress dominal pain, malaise, vomiting). However, yersin-
testing is not indicated in patients with suspected iosis causes an invasive disease of the terminal
blunt cardiac injury, as the tachycardic response may ileum and cecum (or ileocecitis) in many adolescent
actually exacerbate the traumatic insult. and adult patients. The ileocecitis is characterized
by a relative lack of diarrhea and closely mim-
[!) Answer E. S. Pneumoniae (also known as Pneu- ics acute appendicitis. Also unique to yersiniosis
mococcus) is the most common bacterial cause of is the duration of its symptoms, which often last
174 1000 Questions to Help You Pass the Emergency Medicine Boards

for 10 to 14 days or longer. Therefore, yersiniosis not clear why it occurs, patients with recurrent exac-
should be considered in any patient with symptoms erbations ofMS become less responsive to high-dose
of prolonged gastroenteritis. Though the disease pulsed steroids over time. Baclofen is a useful agent
is most common in childhood, adults experience for the spasticity that occurs in patients with MS,
postinfectious polyarthritis or erythema nodosum particularly in those who are wheelchair bound and
2% to 5% of the time. cannot walk.

[!!] Answer E. The most commonly cited abnormality [H) Answer B. This HN patient has bilateral fluffy
is QTc shortening, but any of the listed changes may infiltrates consistent with Pneumocystis carinii pneu-
occur. monia (PCP). Over three fourths of all patients with
acquired immunodeficiency syndrome (AIDS) will
~ Answer B. Patients with retropharyngeal abscesses develop PCP at some point in their lifetimes. It is
(RPAs) generally prefer to lie supine to prevent the also the most common identifiable cause of death
abscess and posterior wall edema from infringing in patients with AIDS. Pneumocystis is classified
upon their airway. Such patients should never be as a protozoan, but has many characteristics of a
forced to sit upright. Although aggressive treatment fungus. Symptoms of PCP, like all pneumonias, in-
of pediatric pharyngitis with early antibiotics has clude fever, cough, and shortness of breath, but a
reduced the incidence of subsequent RPAs, children subacute or mild course is characteristic. Chest ra-
remain the most commonly affected group. This diography classically demonstrates diffuse, bilateral
is due to the presence of large retropharyngeal interstitial infiltrates, but can be completely nor-
lymph nodes in children younger than 4 years, which mal up to 20% of the time. First-line therapy is
may become infected and subsequently develop into with trimethoprim-sulfamethoxazole (TMP-SMX).
RPAs. Adults frequently present with a history of Adjunctive corticosteroid therapy is indicated in pa-
antecedent trauma, such as ingestion of a fish bone or tients who have significant hypoxia (Paco2 <70 mm
caustic agents as well as vertebral fractures. Because Hg). Albuterol may be used in patients with pneu-
the retropharyngeal lymph nodes rapidly involute monia or bronchitis who have a large bronchospastic
after the ages of 4 to 6, adults usually require component to their symptoms. Aspirin is not in-
some insult to the intact retropharyngeal mucosa dicated in most infectious processes. Vasopressin
in order to develop a subsequent infection. An RPA may be used in patients with septic shock who are
should be suspected if the prevertebral soft tissue adequately volume resuscitated. Hyperbaric oxygen
from the anteroinferior aspect of C2 to the border does not currently have a role in management ofPCP.
of the tracheal air column is > 7 mm in children (Figure courtesy of Mark Silverberg, MD. Reprinted
and adults or the same space at the level of C6 is with permission from Silverberg M. Greenberg text-
> 14 mm in children and 22 mm in adults. Although atlas of emergency medicine. Lippincott Williams &
M. tuberculosis may cause an RPA, the most common Wilkins; 2004:1006.)
cause is Staphylococcus. Finally, the most common
fatal complication is airway obstruction. Atlantoaxial ~ Answer E. Nearly all patients with venous throm-
separation may occur due to damage ofthe transverse boembolism have elevated o-dimer levels. Therefore,
ligament of the atlas by the abscess. Such patients assays measuring o-dimer levels have proliferated as
present with neurologic symptoms and an enlarged they are a highly sensitive, but poorly specific screen-
predental space. All patients diagnosed with an RPA ing test for pulmonary embolism (PE). Owing to its
require immediate ENT consultation, admission to elevation in numerous other scenarios, an elevated
the ICU for airway monitoring, and broad spectrum o-dimer level can never be used as proof of the pres-
antibiotic coverage. ence of a pulmonary embolus. However, because of
the high sensitivity of most o-dimer assays (sensi-
~ Answer E. Patients presenting to the ED with ex- tivity varies depending on the assay used), normal
acerbations ofMS should be admitted to the hospital o-dimer levels can safely rule out the presence of a
for a course of high dose intravenous corticosteroids pulmonary embolus in very low-risk patients.
(typically methylprednisolone). However, the mech-
anism of action, the appropriate duration, and the ~ Answer B. Only ionizing radiation has been shown
potential benefits of this therapy are still not clear. to increase the risk of tumors, as it may increase the
An analysis of the Optic Neuritis Treatment Trial incidence of meningiomas by a factor of 10 and the
revealed that treatment of optic neuritis with pred- incidence of glial tumors by a factor of 3 to 7. None
nisone alone may increase the risk of future episodes of the other factors has been proved to increase the
ofMS and therefore, treatment with oral prednisone risk of intracranial neoplasms.
alone is not recommended. Furthermore, while it is
Test6 175

[!1] Answer C. Nausea and vomiting are common in this case due to the absence of signs and symptoms
in pregnancy, with symptoms usually developing of meningitis.
between 4 and 7 weeks' gestation and resolving by
16 weeks' gestation. All of the agents listed have been ~ Answer B. Meperidine may cause serotonin syn-
used in pregnant women suffering from nausea or drome in patients who are chronically taking selective
vomiting in early pregnancy. However, only vitamin serotonin reuptake inhibitors (SSRis) or monoamine
B6 has been demonstrated to be beneficial among oxidase inhibitors (MAOis). Dextromethorphan also
those agents listed. Although vitamin B6 has been exhibits this effect.
proved to be of use in reducing symptoms of nausea,
its ability to reduce vomiting related to pregnancy ~ Answer D. Intraoral lacerations are best repaired
is less clear. More recently, ginger supplementation with absorbable sutures such as vicryl. Vicryl causes
has been shown to reduce both nausea and vomiting less tissue reactivity than silk and is preferred over
related to pregnancy and may be superior to vitamin nylon because it avoids the problem of a repeat visit
B6. The mutagenic effects of ginger are not known for removal. Large intraoral lacerations should be
but it is presumed to be safe. repaired primarily to prevent food particles from
becoming entrapped and causing abscess formation
~ Answer E. The incidence of perforation at the and cellulitis. Small intraoral lacerations should
time of appendectomy has an inverse correlation be left to heal by secondary intention. Antibiotics
with the age of the patient. Therefore, >90% of (penicillin or dindamycin) may be given to patients
patients younger than 3 years have evidence of per- who have through-and-through lacerations (through
foration in the operating room. In contrast, only external skin and intraoral mucosa).
15% of adolescents have perforation at the time of
appendectomy. This difference relates to the diffi- ~ Answer D. The patient has the characteristic
culty and subsequent delay in making a diagnosis ''wine-and-cheese" reaction due to ingestion of
in infants and toddlers. Most patients younger than a tyramine-containing food with pharmacologic
2 years have diffuse tenderness rather than focal monoamine oxidase inhibitors (MAOI) activity.
tenderness over the right lower quadrant. Appen- Tyramine is normally converted to endogenous stim-
dicoliths are considered pathognomonic for appen- ulatory amines and monoamine oxidase (MAO)
dicitis but are only present in roughly 15% of cases. functions to break these down. Use of MAOis in-
On the basis of the few studies performed to date, hibits this degradation function, and excess dietary
most authors recommend CT as superior to ultra- tyramine in this setting causes a disorder similar to
sonography for the diagnosis of appendicitis. How- serotonin syndrome or a sympathomimetic crisis.
ever, more data needs to be collected before cr is Tyramine is present in high quantities in cheese,
routinely recommended as the standard of care. alcohol, dried meats and fruits, and soy.

[!ID Answer C. The patient presents with diffuse ~ Answer C. The patient likely has bronchiolitis,
arthralgias and fever in the setting ofa recent pharyn- most commonly due to respiratory syncytial virus
geal infection. Rheumatic fever is the most important (RSV). It usually occurs during the winter months
diagnosis to rule out in this setting. A migratory pol- and is more severe in preterm infants. Low-grade
yarthritis is common and often involves large joints. fevers, cough, upper respiratory symptoms, and
Major Jones criteria include carditis, polyarthritis, wheezing are seen commonly. Most patients do
chorea, erythema marginatum, and subcutaneous not appear toxic, but up to 10% of patients
nodules. Minor criteria include fever, arthralgias, with bronchiolitis require hospitalization because
and various study abnormalities. EKG is indicated of hypoxemia and severe respiratory distress. The
to assess for the presence of conduction abnormal- only treatment that improves clinical status is
ities, and further evaluation with echocardiogram oxygen. Bronchodilator therapy with f3 -agonists is
may be necessary to evaluate for valvular abnormal- controversial and not clearly proved to be effective.
ities. Treatment is with anti-inflammatory agents, Corticosteroids and antibiotics are not indicated.
antibiotics, and supportive care. Discharging the pa- Ribavirin is used in preterm infants or those with
tient home puts the patient at risk for valvular and a history of congenital heart/lung disease. Repeated
conductive complications of rheumatic fever and episodes of bronchiolitis as an infant may increase
is contraindicated. Corticosteroids are controversial. the risk of developing asthma later in life, but the
Urinalysis may be conducted in the course of evalu- association between the two conditions is still unclear
ation to assess for the presence of poststreptococcal and most patients with bronchiolitis do not go on to
glomerulonephritis but does not aid the diagnosis of develop asthma.
rheumatic fever. Lumbar puncture is not indicated
17 6 1000 Questions to Help You Pass the Emergency Medicine Boards

~ Answer A. Anal fissures are actually the most for endotracheal intubation, mechanical ventilation,
commonly encountered anorectal problem in all of and length ofiCU stay. In addition, NIPPV has also
pediatrics. However, they are especially common in been shown to decrease mortality in patients with
infants. acute COPD exacerbations.

~ Answer E. Nonsteroidal anti-inflammatory drugs ~ Answer A. The HELLP syndrome is a severe


(NSAIDs), including aspirin, or any other mem- manifestation of preeclampsia characterized by
bers of the NSAID family may exacerbate asthma microangiopathic hemolytic anemia (hemolysis),
through inhibition of cyclooxygenase-1 (COX-1). elevated liver enzymes, and thrombocytopenia
In patients with aspirin-induced asthma, exposure <100,000/ rnm3 (low platelets). The prothrombin
to NSAIDs may be the only common trigger of time (PT), partial prothrombin time (PTT) and
asthma symptoms. Nonselective P-blockers cause fibrinogen levels are all normal. The disease is charac-
bronchoconstriction and increase bronchial hyper- terized by hepatic endothelial dysfunction resulting
reactivity to typical asthma triggers due primarily in platelet aggregation and consumption. However,
to the disinhibition of parasympathetic pathways this pathophysiology differs from disseminated in-
innervating bronchial smooth muscle. Angiotensin travascular coagulation in which the PT, PTI, and
converting enzyme (ACE) inlubitors are generally fibrinogen levels are abnormal. Although several dif-
thought to be safe in asthmatics, but there are ferent criteria for the diagnosis of HELLP have been
reports of patients suffering from bronchoconstric- proposed, each focuses on a low platelet count, as
tion and bronchial hyper-reactivity when challenged well as elevated levels of aspartate aminotransferase
with ACE inhibitors. The mechanisms for this are not (AST) and lactate dehydrogenase (LDH).
completely understood and it is not clear whether
there is a relation with ACE inhibitor-induced cough, ~ Answer B. The presence of WBC casts indicates
which is thought to be mediated by bradykinin and infection from a renal source. Other indices on
substance P. urinalysis cannot distinguish between upper urinary
tract infection (UTI) (pyelonephritis) and lower
~ Answer D. The "crossed" straight leg raise (SLR) UTI (cystitis or urethritis). Clinically, pyelonephritis
test is performed by raising the unaffected leg of usually involves back pain and systemic symptoms
a patient complaining of radicular low back pain of fever, nausea, vomiting, and signs of sepsis.
while keeping the knee straight The occurrence of Symptoms of uncomplicated cystitis are generally
pain radiating below the knee in the affected leg limited to dysuria, increased urinary frequency, and
is nearly pathognomonic for a herniated disk with urgency.
nerve root compression. Although the normal SLR
test (performed on the affected leg) is more sensitive ~ Answer D. Ingestion of even small quantities of
than the crossed SLR test, it has a low specificity. methanol is extremely lethal without appropriate
Therefore, many patients without true disc disease therapy. One tablespoon of40% methanol can kill an
will have a positive SLR test. Pain which worsens adult, and less than one teaspoon is enough to cause
when the ankle is dorsiflexed (Lasegue's sign) may blindness. Methanol is metabolized to formaldehyde
be a helpful adjunct to the initial examination. Of by alcohol dehydrogenase, and formaldehyde is con-
note, the SLR and crossed SLR tests are considered verted to formic acid by aldehyde dehydrogenase.
positive only if the patient complains of radicular Ethylene glycol is metabolized to glycoaldehyde by al-
pain radiating down the leg past the knee. The mere cohol dehydrogenase, and glycoaldehyde is converted
presence of back pain is considered a negative test. to glycolic acid by aldehyde dehydrogenase. Alcohol
dehydrogenase has greatest affinity for ethanol, then
~ Answer E. There are no absolute contraindications methanol, then ethylene glycol. GI absorption is very
to the use of epinephrine in anaphylaxis. Anaphylaxis rapid for both ethylene glycol and methanol, usually
can rapidly lead to cardiovascular collapse, respira- occurring within 1 hour.
tory failure, and death; so prompt recognition and
treatment is necessary. Epinephrine is the only estab- ~ Answer E. Chest tubes should generally be placed
lished first-line agent. Risks and benefits of treatment in the third or fourth intercostal space during
should be weighed in any patient, but there are no un- pregnancy and should never be placed lower than
conditional contraindications to using epinephrine the fourth intercostal space. Because the diaphragm
in the setting of anaphylaxis. elevates approximately 4 em during pregnancy, there
is an increased risk of abdominal placement of chest
~ Answer D. Noninvasive positive pressure ventila- tubes if they are placed in the typical fifth intercostal
tion (NIPPV) has been shown to decrease the need space. As in nonpregnant women, the midaxillary
Test6 177

line is the least muscular area ofthe chest wall, which given in 5 to 10 mg doses every 15 to 20 minutes.
makes it an ideal location for chest tube insertion. Lorazepam and phenytoin are second-line agents
Fetal evaluation should always be delayed until for seizure control in patients with eclampsia. An
maternal evaluation, treatment and stabilization, obstetrician should be involved in the care of
even in the setting of obvious fetal distress. (Figure all patients with eclampsia and can help direct
reprinted with permission from Harwood-Nuss A. therapy if the initial magnesium bolus is ineffective.
The clinicalpractice ofemergency medicine. Lippincott Finally, although recommendations vary, treatment
Williams & Wilkins; 2005.) of seizures is the first priority. Once seizures are
terminated, BP may be controlled only ifthe diastolic
~ Answer B. Y. pestis, the etiologic agent of bubonic BP remains elevated above 105 to 110 mm Hg. In the
plague, is a gram-negative coccobacillus which can absence ofseizures, the same BP guidelines apply, and
cause a number of different clinical syndromes. magnesium is given for prophylaxis against seizures.
In this country, it is endemic in the southwestern
United States but it has gained notoriety along with ~ Answer D. Erysipelas is a superficial infection of
anthrax and tularemia because of its potential use the dermis with extensive lymphatic involvement. It
as a possible biologic weapon. Pneumonic plague is characterized clinically by painful and intensely
is caused by the inhalation of infective droplets erythematous skin, which is indurated, raised, and
from animals or persons. Rodents are the natural sharply demarcated from the surrounding normal
hosts but pets can ''bring the disease home." After skin. It is almost always caused by group A
an incubation period of 1 to 6 days, pneumonic streptococci. Though facial involvement has been
plague is an aggressive disease and many patients classically described, 70% to 80% of infections
progress rapidly to septic shock and death without involve the legs. Erysipelas frequently occurs in areas
early treatment. Initially, patients may complain of with poor lymphatic drainage and it frequently recurs
typical symptoms of pneumonia, and their chest in these areas.
x-rays frequently show alveolar infiltrates. Chest
x-rays may also demonstrate an ARDS-like picture ~ Answer E. The patient has acute chest syndrome,
with diffuse patchy bilateral infiltrates and cavitation. as indicated by the presence offever, cough, dyspnea,
None of the other agents demonstrate person-to- and new infiltrate on chestx-rayin the setting ofsickle
person transmission. cell disease. It is a very common cause of death in pa-
tients with sickle cell disease. Etiologies of acute chest
~ Answer B. The patient has a Stanford class A syndrome include bone marrow pulmonary em-
aortic dissection-a tear involving the ascending bolism {PE), pneumonia, pulmonary thromboem-
aorta. Management involves emergent surgical repair bolism, and intraparenchymal pulmonary infarction.
along with early, aggressive BP controL Patients with Treatment involves fluids, analgesics, oxygen, and
aortic dissection often complain of severe chest pain antibiotics. Heparin is used to manage pulmonary
radiating to the back or both arms. Aspirin should thromboembolism, but is not indicated in patients
not be given to any patients suspected of having with undifferentiated acute chest syndrome. It may
an aortic dissection, as this may increase the degree be indicated in patients with acute chest syndrome
of bleeding into the false lumen. Heart rate (HR) due to pulmonary thromboembolism, but this is eval-
should be kept well below 100 to minimize the shear uated on a case-by-case basis in close consultation
stress on the wall of the aorta that is related to the with a hematologist. Tissue plasminogen activator
number of beats per minute. Transesophageal, not may be indicated for acute stroke, Ml, or severe pul-
transthoracic, echocardiography may provide useful monary thromboembolism. Angioplasty is indicated
structural information about the descending aorta, for acute STEMI, but has no role in the management
heart, and pericardium, but CT aortogram and MRI of acute chest syndrome. Albuterol and prednisone
are far more specific. are used to treat bronchospastic lung disease, which
is not part of the pathophysiologic process of acute
~ Answer D. Magnesium remains the drug ofchoice chest syndrome.
for the treatment of seizures in eclampsia as well
as for the prophylaxis of seizures in patients with ~ Answer C. Keraunoparalysis typically occurs in
preeclampsia. The recommended dose is 6 g given the lower extremities andis characterized by transient
intravenously over 15 to 20 minutes followed by a paralysis associated with mottled, cool, blue, and
continuous infusion at 2 g per hour. Hydralazine, pulseless extremities. It results from sympathetic
labetalol, and nimodipine are all agents that have nervous system instability and vascular spasm. These
been used for BP control in patients with eclampsia. changes typically resolve without treatment within
Hydralazine is most commonly used, and is typically minutes to a few hours and are therefore infrequently
178 1000 Questions to Help You Pass the Emergency Medicine Boards

seen by emergency physicians in the ED. Asystole is ~ Answer C. Palpable purpura is the most sensitive
the most common cause ofdeath in patients struck by finding in patients with leukocytoclastic vasculitis
lightning. Although the normal automaticity of the (also known as hypersensitivity or allergic vasculi-
heart may resume spontaneous activity, respiratory tis). The pathophysiology involves immune complex
arrest may persist, resulting in prolonged hypoxia deposition in dermal postcapillary venules followed
and the occasional development ofsecondary cardiac by complement activation, vessel destruction, and
dysrhythmias such as ventricular fibrillation. Bums extravasation of red blood cells resulting in palpa-
and myoglobinuric renal failure are uncommon in ble purpura. Henoch-SchOnlein purpura (HSP) is
the setting of lightning strikes. "Flashover" describes perhaps the most classic example ofleukocytoclastic
the rapid movement of electrical current over the vasculitis, but other causes include mixed cryoglob-
surface of the skin rather than through the patient's ulinemia, vasculitis associated with connective tissue
body. This usually results in no cardiac or pulmonary diseases, viral hepatitis, and hairy cell leukemia. The
effects or cutaneous burns, although a fern-like or vasculitis may also affect internal organs, as in pa-
"arborescent" skin pattern may result. tients with HSP and renal involvement. The other
diseases listed may all produce purpura, but none of
~ Answer D. This patient has endometritis, which is them produce palpable purpura (and Rocky Moun-
the most common puerperal infection. The primary tain spotted fever (RMSF) typically presents more of
risk factor for endometritis is cesarean section, a petechial rash).
although young age, low socioeconomic status,
prolonged stage 2 of labor, prolonged ruptured ~ Answer A. Haloperidol is a potent dopamine
membranes, and multiple vaginal examinations are antagonist, which does not have anticholinergic
also risk factors. Patients typically present 2 to 3 days or hypotensive effects. Phenothiazine&, such as
after delivery with fever, abdominal pain, and foul prochlorperazine and chlorpromazine, cause ortho-
smelling lochia. Infections are polymicrobial and static hypotension, lower the seizure threshold, and
most commonly caused by gram-negative enteric have strong anticholinergic properties which can ex-
pathogens as well as Bacteroides and Prevotella species. acerbate delirium. Diphenhydramine, while sedating,
Chlamydia is rarely responsible and may cause late- also shares these anticholinergic properties. Opioids
onset puerpal infection. may induce dysphoria and can exacerbate brain dys-
function. Diazepam has a long half-life due to its
~ Answer C. Lung abscesses are usually a conse- metabolites, and may result in hypotension and res-
quence of aspiration of contaminated oropharyngeal piratory depression. Promethazine is primarily an
flora. Therefore, the same population of patients antihistamine that also has strong anticholinergic
who are at risk for aspiration are also at risk for properties.
the development of a lung abscess. Risk factors in-
clude any syndrome that results in depressed levels of ~ Answer A. P. aeruginosa is a common nosocomial
consciousness and consequently increases the risk of pathogen, especially in ICUs. It rarely causes in-
aspiration such as alcoholism, massive stroke, head fection in healthy hosts but it has an increasingly
trauma, seizures, and anesthesia. Patients with poor appreciated role in community-acquired infections.
oral hygiene are particularly susceptible because of Most patients with P. aeruginosa infections have
the increased numbers oforganisms among their oral known risk factors, including patients who are me-
flora. Still, lung abscesses develop in a scant minor- chanically ventilated, immunocompromised, HIV+,
ity of patients who aspirate contaminated oral flora. as well as patients with underlying malignancies.
Although immunocompromised patients are also at Among these, patients with neutropenia and those
risk, the HIV+ patient with a CD4+ T-cell count under mechanical ventilation are at highest risk.
> 200 is unlikely to develop a lung abscess. Although This is why empiric coverage for neutropenic fever
drug abuse may lead to depressed levels of con- has, in the past, included two antibiotics with ac-
sciousness, IV drug use puts patients at greater risk tivity against Pseudomonas. P. aeruginosa is also the
for infectious complications related to poor skin ster- most prominent pathogen in patients with CF. Some
ilization techniques, such as endocarditis, phlebitis, studies have demonstrated that as many as 97%
or other local skin infections. It also increases the of children with CF were colonized with P. aerug-
risk of pneumonia due to S. Aureus, which rarely inosa by the age of 3. It is known that P. aerug-
causes necrotizing pneumonia. Patients with Parkin- inosa has a prominent role in the progression of
son's disease may have significant dementia, but it CF, resulting in significant morbidity and mortal-
is primarily a motor disease that should not initially ity. However, the exact mechanisms by which it
put patients at high risk for aspiration. achieves this are not entirely elucidated. Ceftriaxone
Test6 179

and azithromycin do not provide any cover- chest pain. Neither symptom is sensitive or specific
age against pseudomonas infections. Antibiotics for the diagnosis, and a substantial proportion
which have antipseudomonal activity include some of patients with PE only have vague complaints
cephalosporins, such as ceftazidime and cefepime, such as lightheadedness, dizziness, or palpitations.
.f:l-lactam/.f:l-la.ctamase inhibitor combinations (e.g., Additionally, PE is commonly found at autopsy
piperacillin/tazobactam), monobactams (e.g., aztre- in patients who were suspected to have died of
onam), carbapenems (imipenem, meropenem), other causes. As such, it remains an underdiagnosed
aminoglycosides, and fluoroquinolones. Resistance condition due to variability in clinical presentation.
patterns will vary depending on the local community. Choices C, D, and E are all seen in PE, but each
P. aeruginosa is an important cause ofmalignant otitis occurs <50% of the time.
externa, not media, in patients with diabetes.
~ Answer B. Antiplatelet therapy with aspirin is still
~ Answer C. The single most common group of the most effective and cheapest medical therapy for
psychiatric disorders seen by primary care physi- treatment of acute MI. No other single pharmaco-
cians is anxiety disorders. This includes simple pho- logic agent can boast ofas large a mortality reduction,
bia, generalized anxiety disorder, panic disorder, including metoprolol and abci:ximab. Nitroglycerin
and obsessive-compulsive disorder. Treatment usu- and morphine do not reduce mortality in acute
ally entails long-term selective serotonin reuptake Ml, but they are effective at managing symptoms of
inhibitors (SSRis) combined with benzodiazepines, angina.
as needed for acute anxiety attacks. Mood disorders
are also extremely commonly seen and treated by ~ Answer B. Hydralazine, isoniazid, and pro-
primary care physicians. Thought disorders such cainamide may all precipitate a lupus-like syndrome.
as schizophrenia are usually managed primarily SLE typically affects the hands, wrists, and knees
by psychiatrists, as are somatoform disorders and and is most common in young women of childbear-
factitious disorders. Most patients who use recre- ingage.
ational drugs do not inform their primary care
physicians about their drug use and those that do ~ Answer C. After 6 hours ofwarm ischemia, 10% of
tend to be referred to addiction psychiatry special- patients will begin to develop irreversible damage to
ists and outpatient rehabilitation centers. Appropri- muscles, and nerves. After 12 hours, 90% of patients
ate referral from the ED for anxiety disorders and will have irreversible damage.
mood disorders can involve primary care follow-up
exclusively. However, patients with thought, so- ~ Answer C. Levels of alpha fetoprotein (AFP)
matoform, or factitious disorders should receive >500 ng per mL are present in 80% to 90% of
dedicated psychiatric follow-up if they do not al- patients with hepatocellular carcinoma (in high-
ready have a preexisting therapeutic relation with incidence populations). This cutoff is used because
a psychiatrist. elevated levels below 500 ng per mL may be present in
patients with acute and chronic hepatitis or cirrhosis.
~ Answer B. Prinzmetal's (or variant) angina is Note that although it has been estimated that hepati-
characterized by chest pain caused by coronary tis B is responsible for 75% to 90% of hepatocellular
artery vasospasm, which can result in STEMI, carcinoma cases worldwide, metastatic disease is the
arrhythmia, and sudden death. Prinzmetal's can most common cause of hepatic cancer in the United
occur in concert with atherosclerotic heart disease, States.
or may be completely unrelated. A relative reduction
in nitric oxide is hypothesized to be the cause. ~ Answer D. The treatment of choice for QRS pro-
It is often clinically and electrocardiographically longation and wide complex tachycardias in patients
indistinguishable from atherosclerotic CAD. Patients with tricyclic antidepressant overdose is sodium bi-
with Prinzmetal's angina can have a decrease, carbonate. Tricyclics inhibit fast sodium channel
increase, or no change in their pain or EKG-for this conductance and the sodium bicarbonate counter-
reason, history of exertional angina or exercise stress acts this effect. Stable dysrhythmias do not require
testing is of limited value in diagnosing Prinzrnetal's. immediate cardioversion. Lidocaine has no proven
Variant angina may be relieved by nitroglycerin. efficacy in patients with wide complex tachycardia
.f:I-Blockers, like in atherosclerotic CAD, form part of due to tricyclic overdose. Procainamide {lA antidys-
the cornerstone of management. rhythmic) and propafenone (IC antidysrhythmic)
further exacerbate the inhibition of sodium channel
~Answer A. Dyspneaisthemostcommonsymptom conductance and are contraindicated.
of pulmonary embolism (PE), followed by pleuritic
180 1000 Questions to Help You Pass the Emergency Medicine Boards

~ Answer A. The image demonstrates a subarach- world. The readyavailabilityoffirearmsin the United
noid hemorrhage (SAH). Seizures may occur in up States combined with complex socioeconomic in-
to one third of patients and may result in rebleeding, equalities put urban African American youths at the
a common source of morbidity and mortality in these highest risk for firearm-related death. The leading
patients. Although the efficacy of prophylactic anti- cause of death in this subset of the population is due
convulsant therapy has not been rigorously tested in to homicide from handguns.
these patients, most authors recommend prophylac-
tic anticonvulsant therapy in all patients with SAH. ~ Answer C. In the classic surgical text, "Cope's
As the cr scan demonstrates blood in the subarach- Early Diagnosis of the Acute Abdomen," Cope states
noid space, there is no need for lumbar puncture. that "the diagnosis of gastritis or gastroenteritis is
Nimodipine 60 mg, should be given orally as soon as usually made in the emergency ward by a young
the diagnosis ofSAH is made and every 4 hours there- physician who is "not impressed" by a patient's
after. In obtunded patients, it should be crushed and abdominal pain or physical findings." He goes on
administered through an orogastric tube. Nimodip- to write that, "the diagnosis of gastroenteritis in the
ine is used to prevent vasospasm, which may result emergency ward is so often incorrect as to raise a
in secondary {or "delayed") cerebral ischemia. No serious question whenever the emergency physician
other calcium antagonist has proved to be as effective comes to this conclusion." Although this quote
and even the effects of nimodipine are not irrefutably employs a bit ofhyperbole, gastroenteritis is the most
positive. However, because of its safety and ease of common misdiagnosis applied to cases of missed
use, it is currently recommended in all patients with appendicitis. In contrast, many patients initially
aneurysmal SAH. Hypertension should be controlled diagnosed with acute appendicitis have a normal
in the ED with intravenous labetalol or nicardipine. appendix upon appendectomy. In fact, the rate
Sodium nitroprusside and nitroglycerin should be of negative appendectomy has essentially remained
avoided due to their potential to cause an increase unchanged. Among these patients, the most common
in intracranial pressure. (Figure reprinted with per- diagnoses are unexplained abdominal pain (35.1 %),
mission from Haines DE. Neuroanatomy: An atlas of mesenteric adenitis (22.8%), lymphoid hyperplasia
structures, sections, and systems. Lippincott Williams (10.6%) or other diseases of the appendix (9.9%),
& Wilkins; 2003.) gastroenteritis (4.4%), and ovarian cyst (3.3%).

~ Answer D. Tooth fractures are classified by the ~ Answer D. Trimethoprim-sulfamethoxazoleisthe


Ellis system-type I is through the enamel and the initial drug of choice for treatment of bacterial
tooth appears white; type II is through the dentin cystitis in children. It has excellent efficacy against
and the tooth appears yellow; and type III is through the common organisms implicated (gram-negative
the pulp and the tooth has a spot of blood which enteric bacilli) and is well tolerated. Patients with
reappears when wiped away. Tooth fractures should sulfa allergies should be given a penicillin derivative
all be followed up byadentist-thetime offollow-up such as amoxicillin. Amo:ricillin-clavulinic acid is
varies by type. Type I requires only routine follow- a broad-spectrum agent which may be used in
up within 1 week, and types II and III require either UTis that are resistant to the common first-line
immediate dental consultation or next day follow- drugs. It has the important side effect of diarrhea,
up. Calcium hydroxide paste may be placed on type which may limit compliance. Ciprofl.oxacin is a
II and III fractures to cover the exposed dentin first-line agent for treatment of adult UTis but
and pulp. concerns about musculoskeletal effects in children
have prevented its use in the pediatric population.
~ Answer E. The scaphoid is by far the most Doxycycline is contraindicated in children because
commonly broken carpal bone, usually from a fall it chelates calcium and permanently stains the
on an outstretched hand. All other carpal bones are enamel of teeth. Gentamicin is an intravenous
rarely fractured ( <5% each). Both the scaphoid and aminoglycoside antibiotic appropriate for cases of
the lunate are highly susceptible to avascular necrosis severe pyelonephritis but is not used for outpatient
and all suspected cases with negative radiographs treatment of either lower or upper UTI.
should be managed with immobilization and follow-
up with an orthopedist. Most carpal bone fractures ~ Answer D. The patient has an abdominal aortic
diagnosed in the ED are managed with short arm aneurysm (AAA) seen on ultrasonograph. In as-
casts and take 4 to 6 weeks to heal. sociation with hypotension and abdominal pain this
must be interpreted as an acutely rupturing AAA, and
~ Answer D. The United States has the highest rate the only management that will save him is surgery.
of homicide due to firearms in the industrialized Further confirmatory imaging in this scenario will
Test6 181

only prolong definitive management with little yield cephalosporin plus a macrolide or a fluoroquinolone.
and high risk for a poor outcome. Administration Anaerobic coverage may be added for patients who
of intravenous fluids, though important in resusci- are at particular risk for aspiration pneumonia.
tation efforts pending operating room and surgeon
availability, will not be able to normalize BP or pre- ~ Answer B. Mallet finger is a disruption of the
vent certain death in patients who are hypotensive extensor tendon at the level of distal interphalangeal
with a rupturing AAA. (Figure reprinted with per- (DIP) joint with or without an associated avulsion
mission from Silverberg M. Greenberg text-atlas of fracture of the dorsal base of the distal phalanx. It
emergency medicine. Lippincott Williams & Wilkins; is caused by a flexion force on the volar tip with an
2004:194.) extended DIP joint and may commonly occur during
ball sports in which the participant may describe
~ Answer B. Breech presentation occurs in approx- as a jammed finger. Patients should be splinted in
imately 4% of births, or roughly 1/25 live births. extension with either a padded aluminum splint
Shoulder dystocia and abnormal fetal lie each occurs applied to the dorsal aspect of the distal phalanx
in roughly 1/300 live births. Face presentation occurs or an unpadded aluminum splint applied to its
when the fetus is in a longitudinal lie and there is full volar aspect. There is some disagreement regarding
extension of the fetal head, with the occiput against the optimal management of mallet finger when it
the upper back. It is associated with an increased risk involves a fracture. Some advocate operative repair
of perinatal mortality (2% to 3%) and fetal anoma- if subluxation is present whereas other authors opt
lies (e.g., anencephaly) and it occurs in roughly 1/550 for splinting in extension for 6 to 8 weeks. (Figure
live births. Brow presentation occurs when the fe- reprinted with permission from Harris JH. The
tal head is inadequately flexed in a longitudinal lie, radiology of emergency medicine, 4th ed. Lippincott
taking an intermediate position between flexion and Williams & Wilkins; 1999:430.)
extension. Further extension results in face presen-
tation. Brow presentation is also associated with an ~ Answer D. Although calcium may potentiate the
increased perinatal mortality rate of 1% to 8%, and effects of digoxin on the cell membrane, it is the
occurs in approximately 1 per 1,400 live births. first-line agent for the treatment of hyperkalemia
resulting in a disturbance of cardiac conduction. In
~ Answer A. The large majority of kidney stones patients taking digoxin, 10 rnL calcium gluconate
<5 mm will pass spontaneously without the need for should be diluted in 100 mL of 5% dextrose in
lithotripsy or surgical extraction. The large majority water and infused over 20 to 30 minutes. The
of stones >5 mm will not pass spontaneously. effects of calcium last from 30 to 60 minutes.
Urinalysis in patients with kidney stones most often Bicarbonate therapy is one of the least effective
shows microscopic hematuria, although 10% to 20% means of treating hyperkalemia, and is less effective
of cases will have completely normal urinalyses. The than either albuterol or insulin. Furthermore, some
narrowest portion of the ureter, and one of the most authors recommend completely discontinuing its
common sites of obstruction, is the ureterovesicular use for the treatment of hyperkalemia. Sodium
junction at the distal-most point of the ureter. polystyrene sulfate may exacerbate volume overload
Medical management of kidney stones involves due to systemic absorption of sodium in exchange
aggressive pain and nausea treatment, copious fluid for removed potassium. However, the most serious
intake, and stone analysis to assess for risk factors complication of sodium polystyrene sulfate use is
to prevent recurrence. The most common stone ischemic colitis and colonic necrosis, which are more
types are, in decreasing order of frequency: calcium, common with the enema form of therapy.
struvite, uric acid, and cystine.
~ Answer C. Aneurysms > 5 em in size are usually
~ Answer B. Although the incidence of K. pneumo- palpable on physical examination and approximately
nia is higher in alcoholics than nonalcoholics, the halfthe number ofaneurysms between 4 and 5 em are
overall most common cause of community-acquired palpable. Audible abdominal bruits due to abdominal
pneumonia (CAP) in the alcoholic patient is still aortic aneurysm (AAA) are rare. Rupture of an
pneumococcus. Alcoholic patients tend to have a aneurysm due to even vigorous palpation almost
higher incidence ofaspiration pneumonia and tuber- never occurs. Femoral pulses are usually intact in
culous disease compared with the general population. patients with AAA. Truncal obesity makes detection
Alcohol itself is immunosuppressive and predisposes ofAAA on physical examination much more difficult.
patients to a higher incidence of bacterial infections.
Treatment of CAP in the alcoholic patient is similar [!ID Answer C. More than 2,000 children per year die
to that of the general population-third generation of child abuse. The most common mechanism is
182 1000 Questions to Help You Pass tM Emergency Medicine Boards

head injury, followed by intra-abdominal bleeding. presence of nausea, vomiting, or fevers generally
Evaluation and treatment of injuries from child warrants in-hospital management.
abuse are often delayed due to the abuser's status as
primary or secondary caretaker. Intracranial injuries [!QJ Answer E. Toxic alcohol ingestions often present
commonly include subdural hematoma, SAH, and with delayed morbidity and mortality, especially
cerebral contusions. A vigorous shaking mechanism when ethanol is co-ingested. Ethylene glycol is the
alone in an infant is enough to cause a fatal brain main toxic alcohol present in antifreeze, and its half-
hemorrhage. The other answer choices listed are less life without co-ingestants is up to 9 hours. In the
common causes of death from child abuse. The most presence of ethanol, the half-life roughly doubles.
common overall manifestations of child abuse are Therefore, patients who have ingested both ethanol
soft tissue injuries, followed by long-bone fractures. and ethylene glycol may be asymptomatic on presen-
tation (other than inebriation). Diagnosis involves
~ Answer D. Raynaud phenomenon occurs in three cardiac monitoring, basic chemistry labs, ethanol
phases. Initially, digital pallor (white) results from level, blood gas, EKG, urinalysis, and creatine phos-
total closure of the palmar and digital arteries caus- phokinase ( CPK). Fomepizole, a pharmacologic alco-
ing a cessation in digital blood flow. When mild hol dehydrogenase inhibitor, is administered if there
relaxation occurs, a trickle of blood is able to perfuse is suspicion of ethylene glycol overdose, especially if
the ischemic digit but the hemoglobin is rapidly de- an ethanol level is negative, which indicates that alco-
saturated resulting in cyanosis (blue). Finally, arterial hol dehydrogenase is free to convert ethylene glycol
spasm resolves and restores blood flow to baseline, to its toxic metabolites. Fomepizole does not detox-
resulting in a reactive hyperemia (red). ify the parent compound-it simply buys time for
the definitive removal of the toxic alcohol by dialy-
1!1] Answer B. Meckel's diverticulum is the most com- sis. Choices A and B are inappropriate because they
mon cause ofsubstantial GI bleeding in children. The fail to consider the delayed toxicity of co-ingested
diverticulum is a remnant ofthe omphalomesenteric toxic alcohol and ethanol. Regarding choices C and
(or vitelline duct), which is frequently lined with gas- D, only approximately half the number of patients
tric mucosa or other heterotopic tissues. It follows with ethylene glycol poisoning develop urine crystals
the "rule of2 s." It is present in 2% ofthe population, or urinary fluorescence on presentation.
and only 2% of patients will ever develop symptoms
or complications from the duct. It is located within [!!) Answer C. The patient has an open-book pelvis
2 feet proximal to the ileocecal valve, is 2 em long and fracture in association with hypotension, which
2 em wide. Half of all patients develop symptoms by may be rapidly fatal if not treated promptly.
the age of two. Bleeding is usually painless and often Temporizing management revolves around reducing
resolves spontaneously due to splanchnic vasocon- the effective volume into which hemorrhage can
striction. A Meckel's scan which is performed with occur bytightlysecuringthe pelvis with a commercial
technetium Tc 99 m pertechnetate is the diagnostic device or simple bedsheet. Definitive management
test of choice. involves angiography with embolization to control
hemorrhage and surgical fixation to repair the pelvis
[!!] Answer A. Permethrin has become the treatment fracture. Foley catheterization may be performed
of choice for scabies because it is equally efficacious in patients with pelvic fractures if there are no
to lindane, yet it is not appreciably absorbed through hard signs of urethral trauma (e.g., blood at the
the skin making systemic side effects less likely. urethral meatus), but priority should be given
Malathion shampoo can be used for pediculosis to hemorrhage control rather than evaluation of
capitis (head lice), although permethrin is still urethral trauma. CT should never be performed
preferred because of its more pleasant odor and on the hemodynamically unstable trauma patient.
more rapid administration (malathion requires 8 to Thoracotomy is not indicated in patients with blunt
10 hours of administration in cases of head lice while traumatic mechanisms as survival rates are dismally
permethrin requires only 10 minutes). low. Additional radiographs of the pelvis should
be performed after hemodynamic compromise has
~ Answer E. The patient likely has gonococcal con- been addressed. (Figure courtesy of Mark Silverberg,
junctivitis. The patient has signs and symptoms MD. Reprinted with permission from Silverberg M.
of pelvic inflammatory disease and urethritis and Greenbergtext-atlm ofemergency medicine. Lippincott
systemic antibiotic therapy is warranted. Treat- Williams & Wilkins; 2004:659.)
ment should include ceftriaxone, doxycycline or
azithromycin, topical antibiotics, and saline irri- ~ Answer A. The patient has clinical evidence ofcav-
gation. Admission is not absolute--however, the ernous sinus thrombosis, with fever, headache, and
Test6 183

bilateral ocular paralysis. Contrast head CT demon- zero as AKA is primarily a starvation ketosis. Glucose
strates thrombosis in the area ofthe cavernous sinus. levels are not usually elevated and insulin is unneces-
Cavernous sinus thrombosis is most commonly sary. Treatment is with saline and glucose solutions
caused by staphylococci and streptococci. Treatment as well as electrolyte replacement.
involves broad-spectrum antibiotics and neurosurgic
consultation for possible surgical drainage. Heparin ~ Answer C. Audible heart murmur or fever is
may also be indicated in patients who have extensive present in almost 90% of patients with endocarditis.
thrombosis. Without treatment, mortality is dose to Murmurs are less common and fever is more
100%, and even with treatment it is close to 30%. common in N drug users. Choices A, B, D, and
(Figure from Cannon ML, Antonio BL, McCloskey E occur in less than half the number of patients with
JJ, et al. Pediatr cavernous sinus thrombosis com- endocarditis. Some type of vasculitic skin lesion will
plicating sinusitis. Grit Care Med. 2004;5(1):86--88, occur in most patients, but a specific type, such as
with permission.) Osler nodes, Janeway lesions, petechiae, or splinter
hemorrhages each occur in less than one fourth of
[ZID Answer B. The right coronary artery supplies the patients. Splenomegaly is present in roughly one
AVnodein >80% ofpatients, and the left circumflex third.
artery supplies the AV node in the rest.
[ll) Answer E. Umbilical cord prolapse occurs in
~ Answer C. Many patients with head injury are association with fetal malpresentation approximately
at risk for posttraumatic seizures, which can worsen 50% ofthe time (e.g., footling breech). However, the
cerebral injury due to hypoxia. Indications for seizure remaining 50% of cases occur in normal cephalic
prophylaxis are not absolute, but often include presentations. The overall incidence is roughly 1 in
paralyzed or intubated patient, seizure at time of 160 to 1 in 600 births. Steps to arrange immediate
injury or in ED, penetrating or depressed skull injury, cesarean delivery should be taken without delay. In
Glasgow Coma Scale (GCS) <8, subdural or epidural the interim, the goal ofthe ED physician is to preserve
hematoma, and past history of seizures. Phenytoin umbilical circulation by relieving pressure from the
is the first-line medication of choice to prevent umbilical cord. All of the procedures listed may be
posttraumatic seizures, partly because it has few helpful in the setting of umbilical cord prolapse.
negative hemodynamic effects in the trauma patient. If cesarean delivery is not available, the umbilical
Barbiturates are second-line agents for treatment cord should be placed back into the uterus (funic
of acute seizures after benzodiazepines because reduction) and the fetus should be delivered vaginally
they can cause hypotension, which is detrimental as soon as possible. Umbilical cord entanglement is
in the patient with a head injury. Valproate and a common complication of funic reduction.
carbamazepine are indicated for prophylaxis of
nontraumatic seizures in patients with chronic [!j} Answer E. Wheezes are continuous, high-pitched,
seizure disorders. musical sounds that can be heard on inspiration or
expiration. They are caused by high-velocity air flow
~ Answer D. Alcoholic ketoacidosis (AKA) is an el- through a narrowed airway (in much the same way
evated anion gap metabolic acidosis that usually that a murmur is caused by high-velocity blood flow
occurs in chronic alcoholics after a recent binge of through a narrowed vessel or valve). Any pulmonary
alcohol results in vomiting, starvation, dehydration, disease characterized by obstruction can result in
and acidosis. Owing to complex pathophysiology, the audible wheezes on physical examination. A good,
acidosis is dominated by ,8-hydro:xybutyrate, which short mnemonic for obstructive airway disease is
is not detected by standard urinary ketone tests. Fur- LACE:
thermore, as the acidosis resolves, ,8-hydro:xybutyrate Local airway obstruction
is converted to acetoacetate and acetone resulting in Asthma
a paradoxical (or false) "worsening" of the acidosis Chronic Bronchitis
evidenced by increased detection of acetoacetate in Emphysema
the urine. Traditionally, the osmolal gap is normal,
Unilateral, or localized wheezes should prompt
although there are case reports of patients with AKA
a consideration of local airway obstruction, caused
and an elevated osmolal gap. It may be difficult to
by an aspirated foreign body, endobronchial can-
differentiate patients with AKA from chronic alcohol
cer, lymphadenopathy, or infection. Diffuse bilateral
abusers with toxic alcohol ingestion. Toxic alcohol
wheezes are generally caused by asthma or COPD.
ingestion should always be in the differential of AKA
Patients with CHF exacerbations may also have dif-
and should be the top consideration whenever the
fuse wheezes caused by relative airway obstruction as
osmolal gap is elevated. The alcohol level is typically
184 1000 Questions to Help You Pass the Emergency Medicine Boards

a result of airway congestion from the transudation ~ Answer B. The lifetime risk of developing MS in a
of fluid and cellular debris from the interstitium and patient with acute optic neuritis is 25% to 30%.
epithelium. However, patients with CHF usually have Although administration of intravenous steroids
other historical and physical findings that help to dif- reduces the duration of acute ocular symptoms, it is
ferentiate them from pulmonary diseases. Pulmonary unclear whether it reduces the risk of development
sarcoidosis is a restrictive disease that results primar- of MS. Up to half the number of patients with
ily from interstitial fibrosis. Physical examination MS will develop optic neuritis over the course
frequently reveals dry rales, or fine crackles. of their illness. The classic historic triad of optic
neuritis is eye pain, loss of vision, and defects in
~ Answer B. Chalcosis appears as golden deposits in color vision. White patients of northern European
the eye due to inflammation from copper-containing descent are at the highest risk demographically.
foreign bodies, leading to endophthalmitis and rapid Bilateral optic neuritis and recurrent optic neuritis
visual loss. Choice A appears as a defect in the ring each occur in approximately one third of patients.
of the iris and is referred to as a coloboma. Choice Recurrent disease is more common in patients who
C is known as Seidel test, and it indicates anterior are ultimately diagnosed with MS.
chamber leakage, usually from trauma. Choice D is
called iridodonesis. Choice E is called iridodialysis. ~ Answer D. Because estrogens increase the renal
excretion ofuric acid, women have a decreased risk of
~ Answer D. The EKG demonstrates diffuse tall T developing gout until menopause. Postmenopausal
waves consistent with hyperkalemia. The history of women affected by gout are more often taking
absence of medical care, untreated hypertension, diuretic therapy, and often have underlying renal
weakness, and fatigue suggests the possibility ofrenal insufficiency. Gout is a primarily a disease of the
failure as the underlying cause of hyperkalemia. lower extremities, although the hip joints are very
Potassium levels should obviously be confirmed rarely affected.
on laboratory tests, but cardioprotective agents
such as calcium chloride or calcium gluconate ~ Answer D. Neglect or hemi-inattention results
should be ordered early. Defibrillation should never from an infarction of the parietal lobe in the
be performed in awake patients. Cardioversion nondominant hemisphere. In most people, the left
should only be performed with a clear history hemisphere is the dominant hemisphere whether
of hemodynamic instability. Lidocaine is used for they are right-handed or left-handed. In left-sided
wide complex tachycardias, but has generally been lesions, a milder neglect typically results on the
supplanted by amiodarone for acute use. patient's right side.

li!J Answer B. The most common life-threatening ~ Answer C. Although some patients with pontine
emergencies in patients with chronic renal failure hemorrhage may have only very small areas of
are volume overload and hyperkalemia. Dyspnea in involvement and only moderate symptomatology,
the end-stage renal disease (ESRD) patient is usually pontine hemorrhagic stroke typically involves the
secondary to fluid overload and pulmonary edema. entire anterior aspect of the pons, resulting in
Treatment involves standard therapy for pulmonary coma, miosis (pinpoint pupils), horizontal gaze
edema (nitrates, oxygen, diuretics, and morphine) palsy, and erratic breathing requiring intubation
as well as consideration for urgent hemodialysis and mechanical ventilation. Miosis results from
depending on the severity of the fluid overload. MI disruption of ascending sympathetic fibers that
is common in ESRD patients, but occurs less often innervate the pupillary dilator apparatus. Horizontal
than simple volume overload. Pneumonia may also gaze palsy results from involvement of the abducens
occur, but usually causes cough, fever, or malaise nucleus (CN VI), which is located in the central
in addition to dyspnea. Pulmonary embolism (PE) pons. lateral medullary syndrome is due to occlusion
does not occur at an appreciably higher rate in of the posterior inferior cerebellar artery (PICA)
patients who are hemodialysis dependent compared whereas locked-in syndrome is due to occlusion of
with patients with normal renal function-this is the basilar artery. Neither syndrome involves miosis.
thought to be due to constant small infusions of Pseudobulbar palsy is due to the bilateral lesions of
heparin used to maintain graft patency. Diabetic the corticobulbar tracts to lower cranial nerves and
ketoacidosis may occur in patients with ESRD, but may result in dysphagia, dysarthria, and dysphonia. It
usually causes the sign of tachypnea rather than a spares the cranial nerves controlling the extraocular
subjective sense of dyspnea. Additionally, diabetic muscles (CN III, IV, and VI).
ketoacidosis would be far less likely without a history
of preexisting diabetes in a 65-year-old patient.
Test6 185

~ Answer D. Musculoskdetal and pulmonary eti- blindness, death, and intracranial extension. Brain
ologies combine for roughly half of all cases of MRI may detect complications of orbital celluli-
pediatric chest pain. GI, cardiac, and psychogenic tis but provides little advantage in the emergent
conditions each account for approximatdy 10%. Id- evaluation. ESR is an extremely nonspecific labora-
iopathic cases comprise a large minority. Endocrine tory test, which is not useful in diagnosis of most
causes are rare. As in the adult, most pediatric pa- emergent conditions, except temporal arteritis, sep-
tients with acute chest pain should have a screening tic arthritis, and osteomyelitis. Lumbar puncture and
chest x-ray and EKG to evaluate for pulmonary and cerebrospinal fluid (CSF) analysis are not indicated
cardiac causes, respectivdy. Although cardiac causes in this patient in the absence of headache or stiff
ofpediatric chest pain tend to not be immediatdy life neck. Slit lamp examination of the affected side is
threatening, they often radically alter management likely to be impossible given the degree ofperiorbital
and are not as rare as once believed. edema and not useful as a corneal or anterior cham-
ber process is not the primary source of pathology.
[jl) Answer B. Anticholinergic poisoning causes a (Figure from Tasman W, Jaeger EA, eds. The Wills
characteristic toxidrome of mydriasis, dry mucous Eye Hospital atlas of clinical ophthalmology, 2nd ed.
membranes, hyperthermia, tachycardia, impaired Philaddphia: Lippincott Williams & Wilkins; 2001,
gastric motility, urinary retention, and altered mental with permission.)
status. Patients who receive timdy supportive care are
still at risk of dying from hyperthermia, which can be ~ Answer A. The patient has acute rheumatic fever,
difficult to control. Seizures during anticholinergic which occurs several weeks after untreated strep-
poisoning are seen more commonly in children than tococcal pharyngitis. The diagnosis is made by the
adults. Anticholinergics affect the skin and mucosal Jones criteria-either two major (polyarthritis, ery-
surfaces first, then pupils, heart, and bladder, and thema marginatum, chorea, carditis, subcutaneous
finally the central nervous system (CNS) and the GI nodules) or one major and two minor (arthralgias,
systems. Sympathomimetic crises may be differenti- fever, increased ESR or C-reactive protein [CRP],
ated from anticholinergic crises with the presence of prolonged PR interval). The patient has evidence of
diaphoresis. The effects of anticholinergics are more carditis, with a mitral regurgitation murmur and pro-
pronounced on muscarinic acetylcholine receptors longed PR interval, which is the most common EKG
than nicotinic receptors. abnormality seen. The S1-Q3-T3 morphology can
occur in patients with pulmonary embolism (PE),
~ Answer C. As a group, biliary tract malignancies but this finding is neither sensitive nor specific.
are uncommon. Ofthis group, gallbladder carcinoma
is the most common and is the fifth most common [!!) Answer B. Trauma patients require rapid access
GI malignancy as it accounts for 3% to 4% of all GI with large-bore peripheral IVs to optimize fluid
tumors. Metastatic disease to the biliary tract is not administration. Two 14 g or 16 g IVs are ideal. Rate
common. Gallbladder carcinoma primarily occurs of fluid flow is inversely proportional to the length of
in the elderly, and it is more common in women the vessel and directly proportional to the radius
than in men. Almost all patients have a history of of the vessel to the fourth power. Therefore, short,
gallstones and 20% of patients have a history of a wide-bore catheters are preferred over long, narrow-
porcelain gallbladder (calcification of the gallbladder bore catheters. A single IV is not adequate, given the
wall). Therefore, all patients in the ED who are risk of expulsion from patient movement during a
found to have incidental gallbladder wall calcification trauma resuscitation. Eighteen-gauge catheters do
should be referred for elective cholecystectomy. The not provide fust enough flow to keep up with
most common presenting symptom is right upper severe hemorrhage. Triple lumen catheters, though
quadrant pain with jaundice developing later in the generally large-bore lines, are limited in their fluid
disease course. passage rates by their length, which can be up to 10
times that of a peripheral line. Intraosseus catheters
~ Answer B. Fever and periorbital edema should provide ready vascular access in pediatric patients,
prompt differentiation between periorbital and or- but are not preferred in the adult trauma patient
bital cellulitis. Patients with proptosis, oculomotor due to limited flow rates, difficulty of placement, and
dysfunction, and pain on oculomotor exercises are potential complications.
more likely to have orbital cellulitis. Staphylococci
and streptococci are the most common causes. Diag- ~ Answer E. All of these conditions are risk factors
nosis is confirmed by orbital CT scan. Treatment of for delirium, but underlying dementia is the most
orbital cellulitis is with intravenous antibiotics and common.
possibly surgical drainage. Complications include
186 1000 Questions to Help You Pass the Emergency Medicine Boards

~ Answer B. Guidelines regarding the management wide incision and debridement to decompress the
of primary spontaneous pneumothorax continue hand and eliminate inflammatory debris (e.g., paint
to evolve. In the past, small pneumothoraces were or grease). Digital blocks are contraindicated by
defined as those pneumothoraces occupying <20% ED physicians due to the subsequent increase in
of the hemithorax. However, multiple different compartment pressures.
systems are used to estimate pneumothorax volume
and analysis of plain posteroanterior (PA) chest ~ Answer E. Hypocalcemia is the most common
films tends to underestimate pneumothorax volume. electrolyte abnormality as calcium floods the intracel-
Recently, the British Thoracic Society published new lular space when myocyte membranes fail. Because
guidelines dividing pneumothoraces into "small" hyperphosphatemia may also occur, treatment of
and "large" categories to avoid estimations of hypocalcemia is not recommended unless symptoms
percentages. Patients with a rim ofair around the lung are severe or unless severe hyperkalemia develops.
~2 em are considered to have a small pneumothorax Otherwise, calcium phosphate may precipitate and
while those with ~2 em of air around the lung form deposits in tissues. Myoglobin has a plasma half-
are considered to have a large pneumothorax. life of only 1 to 3 hours, making detection sometimes
Most pneumothoraces estimated as ~15% have no difficult. Although myoglobinuria is pathognomonic
persistent air leak and recurrence in those managed for rhabdomyolysis, measurement of creatine ki-
with observation alone is less than in patients treated nase is the most sensitive method to detect muscle
with tube thoracostomy. Such patients do not require cell injury. The classic finding of a urine dipstick
hospital admission, but most ED physicians observe that is positive for blood while microscopic urinal-
the patient while applying supplemental 0 2 (which ysis reveals no red blood cells is present only 50%
increases the rate of resorption of the pneumothorax of the time. HMG-CoA reductase inhibitors are a
by a factor of 4) over a period of 3 to 6 hours after well-known cause of rhabdomyolysis. In such pa-
which a film is repeated to ensure there is no increase tients, rhabdomyolysis appears to be more common
in the size of the pneumothorax. It should be stressed in patients with preexisting renal insufficiency and
before discharge that these patients should return in patients taking fibrates (e.g., gemfibrozil). Rhab-
immediately in the event of developing worsening domyolysis only occurs in 10% of patients surviving
dyspnea. Any patient with hypoxia or a patient with a high-voltage electrical injury. Influenza (A and B)
more than minimal symptoms of dyspnea requires is the most common viral cause of rhabdomyol-
treatment with either aspiration and admission for ysis while Legionella is the most common bacte-
observation, or tube thoracostomy and admission rial cause. HN and coxsackievirus may also cause
for observation. Patients with small pneumothoraces rhabdomyolysis.
who fail simple observation often have secondary
pneumothoraces. ~ Answer C. Mushroompoisoningsaredividedinto
the seven major toxins responsible for the pathologic
!!!] Answer C. High-pressure injection injuries to the effects:
hand are associated with a very high rate of
amputation depending on the injected substance
Mushroom Toxin Pathologic Effed
and the rapidity of treatment. Amputation rates
as high as 80% have been reported. The index Amatoxin Hepatotoxicity
finger of the nondominant hand is the most Coprine Disulfiram-like
common digit involved. Entrance wounds from Gyromitrin Seizures
high-pressure injection injuries may look deceptively Ibotenic acid Anticholinergic
benign. The jet of liquid is under such high Muscarine Cholinergic
pressure that it easily penetrates the skin and Orellanine Nephrotoxicity
gains access to the tendon sheaths causing rapid
Psilocybin Hallucinations (drug of abuse)
distension and an inflammatory response. Over time,
compartment pressures increase, the inflammatory
cascade mushrooms, and the tissues become ischemic ~ Answer B. Patient or toxin odor may provide im-
and necrotic. The flexor sheaths of the thumb and portant clues to the toxic agent. Isopropanol is me-
index finger (most commonly involved) extend to tabolized to acetone, which causes a fruity odor to the
the thenar space while the long, ring, and little finger breath. Ethanol and certain hydrocarbons can also
sheaths extend to the midpalmar space. In addition to produce this finding. Cyanide smells like almonds,
tetanus prophylaxis and antibiotics, a hand surgeon methyl salicylate like wintergreen, and arsenic like
should be consulted immediately for emergent garlic. Acetaminophen has no particular odor, but
Test6 187

fulminant hepatic failure from acetaminophen toxi- dioxide content which leads to vasodilation of the
city may cause fetor hepaticus. retinal arteries. Reduction of intraocular pressure
with acetazolamide and timolol may be performed,
~ Answer C. The patient has fever in the presence and emergent ophthalmologic consultation should
of severe leukopenia, which implies the presence of besought.
neutropenia. Patients with neutropenic fever are at
considerable risk for serious bacterial infection and 11001 Answer D. Flail chest occurs during blunt tho-
require broad-spectrum antibiotics. These patients racic trauma when three or more ribs are each in two
are usually admitted to the hospital for observation places, causing a discrete chest wall segment that is
and bacterial culture monitoring, though carefully unattached to the rest of the chest wall. Paradoxi-
selected, low-risk patients may be candidates for out- cal motion of the flail segment is characteristic: The
patient management. Broad-spectrum antibiotics are flail segment moves inward during inspiration and
always administered, however. Amoxicillin is usually outward during expiration. Severity of the flail chest
not adequate therapy to cover pathogens commonly injury is due to the underlying pulmonary contu-
isolated in patients with neutropenic fever. Surgical sion that results from the blunt trauma. Diagnosis
consultation is not necessary in patients without a is made by physical examination and confirmed by
clear surgical cause for their fever. Intensive care is either chest x-ray or CT. Management is directed
utilized for patients who demonstrate evidence of at treating the underlying pulmonary contusion and
severe sepsis or septic shock and is not necessarily should first involve administration of 100% oxygen
indicated for neutropenic fever alone. to assess for the presence of severe pulmonary shunt-
ing. If the patient does not respond to noninvasive
[!!) Answer E. Central retinal artery occlusion oxygen and is persistently hypoxemic, then endotra-
(CRAO) is essentially a retinal infarction. All the cheal intubation should be performed. Hemothorax
chronic medical conditions listed in the preceding or pneumothorax may also be present and requires
text as well as any condition that increases intraoc- tube thoracostomy. Decubitus positioning is unlikely
ular pressure predispose to CRAO. Sudden, painless to be helpful in treating the flail chest and may ex-
loss of vision is characteristic, and a pale retina (due acerbate atelectasis in the contused lung region. A
to decreased blood flow) with a cherry-red fovea heavy weight placed on the flail segment is also likely
is the classic physical examination finding. Treat- to exacerbate the pulmonary contusion with little
ment involves globe massage to increase local carbon benefit
Test 7
Questions
(I) Which of the following is the earliest electrocar- (D) Cholecystitis.
diogram (EKG) finding in acute myocardial infarc- (E) Intrahepatic cholestasis of pregnancy (ICP).
tion (MI)?
(A) Hyperacute T waves
00 A 23-year-old man presents with &b.ou1der pain
after falling. A shoulder radiograph is shown in
(B) ST elevation
Fig. 7-1. Which of the following is the most likely
(C) ST depression
additional injury'!
(D) T-wave inversion
(E) Qwaves

(!) A 3-year-old girl swallows a button battery. Plain


radiographs demonstrate that the battery is lodged
in the esophagus. Which of the following is the most
appropriate next step in management?
(A) Expectant management
(B) Endoscopic removal
(C) Ipecac for therapeutic emesis
(D) Activated charcoal
(E) Whole bowel irrigation

[!) Which of the foUowing routes of administration


causes the fastest onset of action of cocainel
(A) Intranasal
(B) Sublingual
(C) Oral
(D) Inhalation Figure 7-1.
(B) Transdermal

(!)A 9-month-old infant is broupt to the emergency (A) Humeral head fracture
department (ED) with a bruise on his thigh suffered (B) Glenoid rim disruption
from falling out ofb.is high chair. Radiographs reveal (C) Brachial artery injury
a mi<hhaft femur fracture. Which of the foUowing is (D) Acromioclavicular (AC) separation
the moat likely contributing factor? (E) Clavicular fracture
(A) Child abuse
(B) Ost:eogene&is imperfecta (!] A 46-year-old woman presents to the ED with
(C) Bonetumor cough, fever, and dyspnea. Her chest x-ray is shown
(D) Bone cyst in Fig. 7-2. You initiate empiric treatment for
(E) Hypocalcemia community-acquired pnewnonia, and perform a
diagnostic thoracentesis because of the .size of the
[!) A 27-year-oldwoman at 33 weeks' gestation present- effusion and her declining respiratory status. Which
ing with liver tenderness and evidence of coagulopa- of the foUowing results on pleural fluid compels you
thy is most suggestive of. to perform urgent tube thoracostomy?
(A) Acute fatty liver of pregnancy (AFLP). (A) pH <7.0
(B) Preeclampsia.. (B) Malisnmt cells
(C) Hepatitis.

188
Tat7 189

[!) Primary chronic adrenal insufficiency is usually


due to:
(A) Sarcoidosis.
(B) Hemorrhage.
(C) Pituitary insufficiency.
(D) Idiopathic.
(E) Iron deposition.

[!g) A 75-year-old woman presents to the ED for


evaluation of meningitis. She has had 12 hours
of acute oll5et of headache, stiff neclc. and fever.
Physical aamination demonatrates a febrile patient
in moderate distress with nuchal rigidity and severe
photophobia. A lumbar puncture is pcrfonned with
the following results:
Cerebrospinal fluid 700 cells per mL, 80%
white blood cell (CSF neutrophlls, 20%
WBC): lymphocytes.
Figure 7-2. CSF glucose: Decreased.
CSF protein: Elevated.
CSF Gram stain: No organisms seen.
On the basis of these re.mlta, the presumptive
(C) Protein consistent with emdative effusion diagnosis of bacterial meningitis is made. Which of
(D) Amylase elevated the following is the most appropriate therapy?
(E) Gram stain negative
(A) De:wnetlwone alone
(B) Ceftriaxone alone
(!] The znoat common dysrhythmia in hypothyroid
(C) Vancomycin alone
cardiovascular disease is (D) Ceftriaxone and dexamethasone
(A) Atrial fibrillation. (E) Ampicillin, ceftriamne, and daamethaaone
(B) Long QT syndrome.
(C) Junctional escape rhythm. !H) A 65-ymr-old man presents with the following
(D) Sinua rhythm with left or right bundle branch asymptomatic EKG, as shown in Pig. 7-3. Which
block. of the following is the most likely associated con-
(E) Sinua bradycardia. dition?

II

Figure 7-3.
190 1000 Qwstimu to Hdp You Ptw the Emerpnt;y Maicitw Bo.rmfs

(A) Hypertrlglyceric:lemia ~ A 34-year-old woman presents with shoulder pain


(B) Rheumatic m.er following a :6ill. Physical examination is normal
(C) Chronic obstructive pulmonary except for tenderness in the middh: of the clavicle.
disease (COPD) Radiographs demonstrate a nondisplaced clavicle
(D) Pulmonary embol.i5m fracture. Which of the following is the most
(E) Hyperblemia appropriate course of actionr
(A) Operative repair
(!j) A 19-year-old woman presents with a red, painful
(B) Figure-of-eight brace
eye for 2 da)'S- She is a contact len& wearer. The eye
(C) Magnetic resonance imaging (MRI) to evaluate
ia shown in white light, as seen in Fig. 7-4. Which of
rotator cu1f injury
the following is the most likely diagnosis?
(D) Shoulder sling
(E) Shoulder arthrocentes.is

(!j] Security is called to help restrain an agitated patient


in the ED. In helping to restrain the patient, one of
the officers is inadvertently stuck by a contaminated
needle that the nurse was using to obtain an IV. The
patient is known to have chronic active hepatitis B
(HepB) and the officer says he was immunized once
against HepB but is a "nonresponder." Which of the
following ia true?
(A) Passive inununization with hepatitis 8 immune
globulin (HBIG) but not active immunization
with the HepB vaccine should be given.
(B) The HepB vaccine is incompatible with typical
figure 7-4. prophylactic drug therapy for human
immunodeficiency virus (HIV).
(C) Approximately 30% to 50% of initial
(A) Corneal abruion nonresponders wiD respond to a second HepB
(B) Corneal ulcer vaccine seri.el.
{C) Herpes zoster ophtbalmicus (D) The patient's wound should be wuh.ed with a
{D) Herpes aimplexkeratitis dilute bleach solution.
(E) Hyphema (E) HepB is tranml.itted much less effectively than
hepatitis C through needlestick injuries.
~ Which of the following ia a risk factor for cholelithi-
.
am. ~
(!11 A 7-year-old girl presenta with syncope without
(A) Pregnancy prodrome. EK.G shows QT prolongation. Family
(B) Obesity history is most likely to reveal which of the following?
(C) Female gender (A) Cystic fibroais
(D) Oral cont:ra.ceptive use (B) Hirscluprung's disease
(E) All of the above (C) Juvenile rheumatoid arthritis
(D) Deafness
(HJ A 22-year-old woman at 36 weeks' gestation is be- (E) Short stature
ing treated with magnesium sulfate for preeclampsia
while awaiting~r to a nearby hospital for defini- I!!J Which of the following is true regarding aspirin-
tive management Upon reevaluatiq the patient, induced asthma (AlA)?
you find her to be quite somnolent with markedly
(A) It accounts fur half of all asthmatics.
decreased deep tendon rdlaes and a decreased res- (B) Children are more commonly affected than
piratory rate. After managing the airway, the next
adults.
best step in manasement is intravenoua: (C) Males are more commonly affected than
{A) Denmethuone. fenWes_
{B) Lidocaine. (D) It is a oom.mon precipitant of severe,
{C) Labetalol life-threatening asthma.
(D) Calcium gluconate. (E) It is associated with Wegener's granulomatosis.
(E) Atropine.
Tm7 191

(!!) A 24-year-old woman at 35 weeks' gestation presents the following is the most important nat step in
to your community ED after experiencing a small
...... .
..:~: , ~
---ea•OSlS·
gush of fluid nmning down her leg approximately (A) X-rays
2 hours before presentation. She has no abdominal (B) Computed tomography (Cf)
pain, vaginal bleeding, or other complaints. Which (C) MRI
of the following is true? (D) Tympanocentesis
(A) A speculum and digital vaginal examination (E) Lumbar puncture
should be performed to assess the extent of
cervical clilation. ~ Which of the following is true regarding anaphylaxis?
(B) Periling is present in only 50% of patients with (A) Bee stings are the moat common cause.
premature rupture of membranes (PROM). (B) Exercise may trigger anaphylaxis.
(C) After 'ftrification of fetal heart tones, she can be (C) Anaphyluis usually occurs upon first aposure
discharged with a prescription for antibiotics to an allergen.
and prednisone with close obstetrician (OB) (D) Anaphylaxis is a type IV hypersensitivity
follow-up. reaction.
(D) Trichonumas vaginitis can result in a (E) The risk of anaphylaxis is greatest in the very
false-positive nitrazine paper test. young and elderly.
(E) All of the above.
~ Which ofthe following is the main therapeutic action
(igJ Which of the following is the main pathologic of mannitol in patients with head injlll"f.
mechanism ofaction ofCorynebacterium diphtheritle! (A) Promotes coagulation
(A) Direct cellular death (B) Osmotic diuresis
(B) Toxin-mediated cellular death (C) Anti-inflammatory action
(C) Fungal superiDfection (D) Seizure prevention
(D) Protozoal superinfed:ion (E) Renal protection
(E) Immune suppression
~ A 4-year-old girl is brought in by her mother, who
(ij) Which of the following is the most common cause of is worried that "she swallowed something and it's
encephalitis? stuck." The patient was in her usual health until this
(A) Herpes virus morning's breakfast. which she could not swallow
(B) Mumps virus andrapidlyvomited.AnAPchestX-raydemonstrates
(C) Enteroviruses a coin positioned en {as% in the upper chest. The
(D) Neurocysticercosis patient looks well and is otherwise asymptomatic.
(E) Adenovirus What is the next appropriate step?
(A) Glucagon 1 mg IV
(UJ A 42-year-old man presents with left ear pain for (B) Endoscopy
3 days. The ear is shown in Figure 7-5. Which of (C) 12 ounces ofsoda PO
(D) Oralpapain
(E) Heimlich maneuver

(ij) Inacute Ml, which of the following advantages


does hirudin possess over unfractionated and low
molecular weight heparins?
(A) Improved mortality
(B) Lower incidence of major bleeding
(C) Lower incidence of thrombocytopenia
(D) Increased anti-X.. activity
(E) Increased anti-XTII activity

~ Which of the following, when associated with the


presence ofan infection, comprises the most accurate
definition of septic shock?
(A) Fever
(B) Hypothermia
Figure 7-5.
19 2 1000 Questions to Help You Pass tM Emergency Medicine Boards

(C) Hypertension (C) Nebulized racemic epinephrine, IV


(D) Adrenal insufficiency dexamethasone, IV ampicillin
(E) Inadequate tissue perfusion (D) Humidified oxygen, IV levofloxacin
(E) IV dexamethasone, IM penicillin G benzathine
~ Which of the following vasculitis syndromes is most
likely to present as pulmonary hemorrhage requiring ~ A 22-year-old gang member is brought to an urban
emergent airway management? ED with a gunshot wound to his arm. Primary survey
(A) Churg-Strauss syndrome of the patient is intact, but a handgun is found in the
(B) Polyarteritis nodosa patient's clothes. Which of the following is the most
(C) Wegener's granulomatosis appropriate action by the physician at this time?
(D) Goodpasture's syndrome (A) Check to see if the gun is loaded.
(E) Mixed cryoglobulinemia (B) Fire the gun into the floor to discharge all the
ammunition.
~ Shortly after intubating a 70-kg patient for severe, (C) Place the gun away from care providers and
refractory asthma, the ventilator alarm is set off due to notify local law enforcement.
slightly elevated peak airway pressures. The plateau (D) Hold the gun personally so no one can take it.
pressure is within normal limits ( <30 mm Hg). (E) Use the gun to attack rival gang members in
Your ventilator settings are assist-control, rate 12 per retribution.
minute, tidal volume 500 mL, peep 0 em H20, and
inspiratory flow rate 60 L per minute. The Pox is ~ A 54-year-old man presents with weakness, fatigue,
98% to 99% with an Fi02 of 100%. The BP is 135/80, and decreased urinary output over the last 2 days.
and the pulse is 115 with sinus tachycardia on the His laboratory results are shown in the subsequent
monitor. The next best step is to: text:
Na 145
(A) Decrease the inspiratory flow rate in order to
K 4.4
decrease the peak airway pressure.
Cl 100
(B) Increase the tidal volume in order to decrease
HC03 22
the peak airway pressure.
Blood urea nitrogen (BUN) 54
(C) Increase the positive end-expiratory pressure
Cr 2.2
(PEEP) in order to decrease the peak airway
UrineNa 10
pressure.
Urine Cr 95
(D) Do nothing, the elevated peak airway pressure
No Urine WBC, red blood cell (RBC), or casts
is unlikely to harm the patient. Which of the following is the most likely cause?
(E) Decrease the respiratory rate and increase the
(A) Gastroenteritis
tidal volume to decrease the amount of
(B) Transitional cell carcinoma
auto-PEEP and thereby decrease the peak
(C) Acute tubular necrosis
airway pressure.
(D) Diuretic
@QJ A 26-year-old woman presents with intermittent (E) Benign prostatic hypertrophy
nausea and vomiting, orthostatic lightheadedness,
and mild diffuse abdominal pain. She also notes ~ A rash that starts on the wrists and ankles is typical
that her skin has darkened over the last month in cases of.
although she has been indoors. Which of the fol- (A) Meningococcemia.
lowing laboratory abnormalities are you most likely (B) Rubella (German measles).
to find? (C) Rubeola (measles).
(D) Henoch-SchOnlein purpura (HSP).
(A) Hyponatremia
(E) Pityriasis rosea.
(B) Hyperglycemia
(C) Low thyroid-stimulating hormone (TSH) levels
(D) Hypokalemia ~ High-altitude pulmonary edema (HAPE):
(E) Elevated urinary metanephrines (A) Is the most common cause of high
altitude-related death.
~ Appropriate initial therapy in a patient with adult (B) Causes cyanosis of the nail beds and lips.
epiglottitis includes which of the following? (C) Is typically worse at night.
(A) Nebulized racemic epinephrine, IV levofloxacin (D) May be treated in part with nifedipine.
(B) Humidified oxygen, IV ceftriaxone (E) Is all of the above.
Test7 193

~ A 4-year-old girl presents with vomiting and diar- (C) 2 mm ofST depression
rhea. The diagnosis of acute gastroenteritis is made. (D) Tall T waves
Presence of which of the following findings still al- (E) Inverted T waves
lows the patient to be a suitable candidate for oral
rehydration therapy (ORT)? [!!) Which of the following is the most common
complication of Epstein-Barr virus pharyngitis?
(A) Continued diarrhea
(B) Lethargy (A) Asymptomatic elevated transaminases
(C) Shock (B) Airway obstruction
(D) Abdominal rebound (C) Splenic rupture
(E) Bowel obstruction (D) Hemolytic anemia
(E) Meningoencephalitis
[llJ A 22-year-old woman presents with drug over-
dose in a suicide attempt. She states she took ~ A 22-year-old man presents with acute onset of
60 of her fluoxetine tablets 2 hours before pre- right scrotal pain for 2 hours. He has severe, colicky
sentation. She is asymptomatic, awake, and alert, pain with nausea and vomiting, but no fevers, chills,
and vital signs and physical examination are nor- or dysuria. Vital signs are normal, but the patient
mal. Which of the following is true regarding this is in extreme discomfort. Abdominal examination is
patient? normal. Testicular examination reveals a tender right
testis with an absent ipsilateral cremasteric reflex.
(A) Death from dysrhythmia is the most likely
Which of the following is the most appropriate next
outcome.
step in evaluation?
(B) Ipecac is indicated to prevent gastric
absorption. (A) CT scan of the abdomen and pelvis
(C) Cyproheptadine has not been proved to (B) MRI of the abdomen and pelvis
improve outcomes. (C) Color Doppler ultrasonography of the scrotum
(D) Sodium bicarbonate should be used for (D) Retrograde urethrogram
ventricular dysrhythmias. (E) Elicitation of the bulbocavernosus reflex
(E) Hemodialysis may be helpful in massive
overdose. ~ Which of the following is the most common
intracranial complication of otitis media?
~ The most common cause of traveler's diarrhea is (A) Meningitis
(A) Shigella spp. (B) Brain abscess
(B) Giardia Iamblia. (C) Subdural empyema
(C) Salmonella spp. (D) Lateral venous sinus thrombosis
(D) Rotavirus. (E) Focal otic encephalitis
(E) Enterotoxigenic Escherichia coli (ETEC).
~ A 22-year-old woman presents with sudden onset
~ Which ofthe following is the most common cause of of palpitations. She is 10 weeks pregnant with a
septic arthritis in children? confirmed intrauterine fetus. Her vital signs are 98.6,
190,20,124/67,98% RA. The EKG shows a regular,
(A) Neisseria gonorrhoeae
narrow-complex tachycardia with absent P waves.
(B) Streptococcus pneumoniae
Which of the following is the most appropriate next
(C) Staphylococcus aureus
step in management?
(D) Haemophilus influenzae
(E) Streptococcus pyogenes (A) Digoxin 0.125 mg N
(B) Adenosine 6 mg N
~ A 67-year-old woman with coronary artery disease (C) Lidocaine 100 mg N
(CAD) presents with dyspnea, which is her last (D) Amiodarone 150 mg N
anginal equivalent. She is known to have a preexisting (E) Procainamide 1 g N
left bundle branch block. Which of the following
EKG findings, when present in two contiguous leads, ~ A 55-year-old woman presents with progressively
indicates the highest risk for acute Ml? worsening painful, swollen, and discolored left leg
shown in Figure 7-6. Which of the following is the
(A) 4 mm of discordant ST elevation
most likely pathophysiologic mechanism?
(B) 4 mm of concordant ST elevation
194 1000 Quatioru to Htlp You Ptw tile~ MetliciM BoGrr1s

(A) Lowers the seizure threshold


(B) Dopamine 02-receptor anmgonilt
(C) al receptor agonist
(D) Histamine receptor agonist
(E) Asranulocytosis

~ A 62-year-old woman with a history of chronic


atrial fibriUation presents with 2 hours of acute-
onset, continuous. excruciating abdominal pain. A
few minutes after the onset of pain, she had an
urge to defecate and had a large, forceful bowel
movement. She denies bloody or melenic stools
or a history of postprandial pain. On eJ31llination,
her abdomen is soft, flat, and only mildly tender.
Labs reveal a leukocytosis of 16,000 per mm~ with
Figure 7-t. a normal chemistry. An initial flat and upright
abdominal film is nonspecific and a cr scan is
read by the radiologist as having evidence of small
(A) Thrombosis bowel thickening. What is the next most important
(B) Embolus
step in rnaJ188Clllent?
(C) Infection
(D) Autoimmune (A) lnterventional radiology consult for emergent
(E) Allergy angiography
(B) Surgical consult for emergent laparotomy
~ A 4-year-old boy presents with progressive perior- (C) Gastroenterology consult fur e:mcrgent
bital edema, weight gain, anorexia, and nausea for endoscopy
several weeks. Hypertension is noted on physical (D) Intravenous antibiotics and admission for
cumination. Urinalysis demonstrates 4+ protein. presumed infectious colitis
Which of the following is most likely to be present in (B) Admission for serial abdominal examinations
this patient~ without specific therapy
(A) Hyperalbuminemia
~ Which of the following conatitutea definitive treat-
(B) Thrombophilia
ment for ethylene glycol poiso~
(C) Hypotriglyceridemia
(D) Urinary bacteria (A) Ethanol drip
(E) Gross hematuria (B) Fomepizole
(C) Pyridoxine
(D) Thiamine
~ A 29-year-old man presents to the ED with a 3-week
history of diarrhea, aampy intermittent abdominal (B) Dialysis
pain, and a 10-lb weight loss. He denies any bloody
stools or vomiting even though his stool guaiac test ~ A 23-year-old woman presents with 2 days of
is positive. He returned from a 1-month trip to India fever and severe right ankle pain and swelling.
approrimately 6 weeks ago and had no problems She denies history of acute or chronic trauma.
while he was there. Which of the following is the Paat medical history is unremarkable. Physical
most likdy cause of his symptoms? examination reveals significant edema, effusion,
tenderness, and pain on range of motion in the
(A) Shi~l/4 .spp.
(B) Eoterotoxigenk E. coli
risht ankle. Which ofthe following is the most likely
etiology?
(C) Bnmobius wrmicularis
(D) Bntmnoeba histolytica (A) S. AUmlS
(E) Campylolxlaer spp. (B) S. pneumoniae
(C) S. pyogenes
(D) Salmonella
~Which of the following is a known dJec;t of
haloperidol? (E) N. gonomweae
Test7 195

~ Which of the following corresponds to ischemia of (C) He has latent TB infection and he may infect
the anterior circulation? other persons.
(D) He has latent TB infection, and requires
(A) Hemiparesis and hemisensory loss of the left leg
prolonged treatment with INH and B6.
(B) Transient monocular blindness
(E) He must be admitted for acid-fast bacilli (AFB)
(C) Aphasia and hemiparesis of the right arm, leg,
sputum cultures to determine if he has been
andfuce
infected with TB.
(D) Ataxia, vertigo, nausea, and vomiting
(E) A, B, and C
~ A 56-year-old man presents with tremor, anxiety,
tachycardia, and hypertension. He has a past medical
~ Which of the following is indicated for treatment
history of chronic renal insufficiency. He reports
of a stable, wide complex regular tachycardia at a
heavy alcohol use, but has not had a drink in the last
rate of200 in a patient with Wolff-Parkinson-White
2 days. Which ofthe following is the most appropriate
(WPW) syndrome?
next step in management?
(A) Adenosine
(A) Haloperidol
(B) Diltiazem
(B) Lorazepam
(C) Esmolol
(C) Chlordiazepoxide
(D) Digoxin
(D) Diphenhydramine
(E) Arniodarone
(E) Droperidol
~ Which of the following is a true about extrapul-
~ A 35-year-old man presents with a grade II ankle
monary tuberculosis (TB) infection?
sprain. Which of the following is true regarding this
(A) Pericarditis is a potential manifestation. patient?
(B) Painful lymphadenopathy is common.
(A) The injury is minor and should heal in 3 to 5
(C) The central nervous system (CNS) is typically
days.
spared.
(B) Knee immobilization is the mainstay of
(D) Skeletal TB most commonly involves the pelvis.
management.
(E) Adrenal TB is typically unilateral.
(C) Cold therapy is likely to be more beneficial than
heat therapy.
~ Which of the following is the most common
(D) Ibuprofen is superior to acetaminophen for
complication of cirrhosis?
pain control.
(A) Encephalopathy (E) Emergent MRI is indicated to rule out grade III
(B) Gastrointestinal (GI) bleeding sprain.
(C) Spontaneous bacterial peritonitis
(D) Ascites ~ Which ofthe following is true regarding radiographic
(E) Hepatocellular carcinoma studies and pregnancy?
(A) The maximum amount of safe fetal radiation
~ A 52-year-old previously healthy American-born
exposure is 10 rnrad.
man presents 2 months after having a purified protein
(B) Focused assessment of sonography in trauma
derivative (PPD) placed on his left forearm. He had
(FAST) ultrasonography evaluation is
been in Central America approximately 6 months
approximately 90% sensitive in all trimesters of
ago with the Peace Corps. He states that the injection
pregnancy.
site became quite large, red, and firm, but he never
(C) The cumulative background radiation exposure
followed up with his doctor. He now wants to know
to a fetus throughout a 9-month gestation is
what this result means. He denies cough, weight loss,
greater than the exposure due to a single
drenching night sweats, or fever. His chest x-ray is
maternal chest x-ray.
unrevealing. Which of the following is most likely
(D) Lead shields minimally reduce fetal radiation
true?
exposure in the third trimester due to scatter
(A) He has active TB and requires isolation and a caused by the enlarged uterus.
multidrug treatment regimen. (E) The highest risk period of radiation exposure is
(B) He is not infected with TB as he has no in the first 2 weeks of pregnancy.
symptoms ofTB.
~ A 60-year-old man presents with painless hematuria. Which of the following is the most appropriate next
Which of the following is the most likely cause? step in management?

(A) Renal carcinoma (A) Topical antivirala


(B) Bladder carcinoma (B) Topical steroids
(C) Urinary tract infedion (UTI) (C) Topical anuoiotics
(D) Glomerulonephritis (D) Intravenous acetazolamide
(B) Nephrotic syndrome (E) Emergent ophthalmologic consultation

~ A 62-year-old man with a history of controlled ~ Which of the following is true regarding sarcoidosis?
hypertension presents to the ED with a fever, (A) The incidence is higher in blacb than whites.
headache, and vomitiJls. He is mildly soiD.D.olent oo. (B) Skin lesions are the most common presenting
aamio.ation and has eviden.ce of mild neck stiffness. IIlllllifestation.
Suspecting meningitis as the cause of his symptoms, (C) Systemic symptoms such as fatigue, fever, and
which of the following empiric regimens should you anorexia are uncommon.
start? (D) Arthralgias and artluitis are usually asymmetric
(A) Ceftriaxone, vancomycin and involve the fingers.
(B) Ceftriaxone, ampicillin (E) The 2-year mortality rate is 65%.
(C) Ceftriaxone, ampicillin, dexamethasone
(D) Ceftriaxone, vancomycin, ampiciD.in, ~ A 20-year-old man presents with a painful, ulcerated
dexametlwone lesion on his penis. He noticed it 3 days before and
(B) Ceftriaxone, amphotericin, vancomycin, the pain became progressively worse. Examination
dexamethasone shows a tender, 1-cm ulcerated lesion at the base of
his penis with a single, large, tender inguinal lymph
~ Which of the following is the most common serious node. Gram stain of the ulcer shows gram-negative
complication of pulmonary contusion? bacilli. Which of the following is the most lik.ely
(A) Acute respiratory distress syndrome (ARDS) cause?
(B) Pneumonia (A) Herpes simplex virus
(C) Pulmonary embolism (B) Chlamydia tradtom~atis
(D) Myocardial contusion (C) Staphylococcus epidermidis
(B) Pericardial tamponade (D) Haemophilus ducreyi
(E) TTq~Dnmlll paUidum
(!!) A 37-year-old man presents with left ~ pain and
redness after rubbing his eye the day before. Slit lamp ~ Which of the following has the highest sensitivity for
evaluation with fluorescein stain is shown in Fig. 7-7. ruling out testicular torsion?
(A) Normal cremuteric :reflex
(B) Presence of Prehn's sign
(C) Normal urinalysis
(D) Absence of fever
(E) Absence of vomiting

(!Zl A 4-day-old neonate is brought in by her mother


fur irritability, excessive shaking and tremulousnea.s,
and an abnormal c.ry. Tbe infant's glucose level is 20.
Which ofthe following is the best initial treatment of
this infant?
(A) 0.03 mg per .q ofglucagon delivered N
(B) 2to4mLpcrkgofDIO
(C) l to 2 mLperkgof D25
(D) 1 amp ofD50 given IV
(E) Octreotide infusion

flgurw 7-7.
Test7 197

~ A 64-year-old woman presents with paresthesias in (A) Acetoacetate


her hands and feet for the last few weeks. She is (B) Acetone
concerned that she has diabetes. She denies any (C) ,8-hydroxybutyrate
past medical history. Physical examination reveals (D) Oxalate
decreased vibratory sensation and proprioception in (E) None of the above
her hands and feet. Laboratory data demonstrates a
normal glucose and macrocytic anemia. Which of ~ Which of the following is true about the diagnosis of
the following is most likely to be present on further acute appendicitis?
patient history? (A) CT scanning may be less sensitive in thin
(A) Smoking history patients and children.
(B) Chronic aspirin use (B) The most sensitive CT study involves thin-cut
(C) Strict vegetarian diet helical CT with rectal contrast.
(D) Chronic melena (C) CT scanning in patients with early symptoms of
(E) Family history of thalassemia appendicitis has a lower sensitivity.
(D) Negative ultrasonography examinations should
~ Which of the following is true regarding epidural be followed by aCT scan or in-patient
hematomas? observation.
(E) All of the above.
(A) Even with early diagnosis and aggressive
therapy, mortality is approximately 100%.
~ A 30-year-old woman presents with painless hema-
(B) The "lucid interval" is pathognomonic.
turia. Which of the following is the most likely
(C) The temporal area is the most common region
cause?
affected.
(D) Concave shape on CT scan is classic. (A) Renal carcinoma
(E) Patients are asymptomatic in 50% of cases. (B) Bladder carcinoma
(C) UTI
~ Which of the following traumatic mechanisms is the (D) Glomerulonephritis
best candidate for ED thoracotomy? (E) Nephrotic syndrome
(A) Blunt trauma to the chest
(B) Blunt trauma to the abdomen
[!ID Which of the following is true regarding scapular
fracture?
(C) Blunt trauma to the pelvis
(D) Penetrating trauma to the chest (A) Associated thoracic injury is usually more
(E) Penetrating trauma to the abdomen serious than the fracture itself.
(B) Operative repair is usually required.
[!!] Which of the following is the most serious compli- (C) Penetrating trauma to the back is the most
cation seen with four-part fractures of the humeral common mechanism.
head? (D) The arm is usually held in full abduction.
(E) Associated contralateral scapular fracture is
(A) Associated olecranon fracture
common.
(B) Adhesive capsulitis
(C) Pneumothorax
(D) Associated scapular fracture
[ll) A 55-year-old man without any past medical his-
tory presents with chest pain. Physical examina-
(E) Avascular necrosis
tion is normal. The EKG is shown in Figure 7-8.
Which of the following represents the likely site of
~ Which of the following is the most sensitive indicator
pathology?
of high-altitude cerebral edema (HACE)?
(A) Pericardium
(A) Cerebellar ataxia
(B) Pulmonary artery
(B) Vomiting
(C) Left circumflex artery
(C) Abducens nerve palsy
(D) Left anterior descending (LAD) artery
(D) Seizures
(E) Right coronary artery
(E) Slurred speech

[ZID Typical nitroprusside urine dipstick analysis primar-


ily detects which of the following ketones?
198 1000 Quatioru tv Hdp Y11u Ptw the Emetpnq Medicine Bcumh

figure 7-8. (See color insert.)

~ A 6-month-old inCant falls out of her high chair (B) Mortality is close to 50% even in isolated cases
onto the kitchen floor and is brought by her parents of sternal fracture.
for evaluation. 'The parents note that there was no (C) Lateral chest radiography is c:iiagnostic.
definite lou of con.tciouaness (LOC), vomiting, or (D) Spinal fracture is the most common associated
seizure activity. She has not been abnormally sleepy, bony injury.
but her parents note that she should be hungry at (E) Unreatrained pasaengers are at higher ri&k than
this time and hu been refusing feeds. Her physical restrained passengers.
aamination. indudins neurologic examination, is
unremarkable. Which of the following is the most f!!) Parents of a 3-year-old boy 'brin8 him to the ED with
appropriate next step in management? a yellowish, crusting facial rash, as shown in Fig. 7-9.
(A) Contact Department of ChiLdren and Family Which of the following is trud
Services (DCFS)
(B) Skull x-rays
(C) Cfbrain
(D) MRI brain
(E) Discharge home

(!!} Risk factors fur decompression sickness (DCS)


include which of the following?
(A) Patent foramen ovale
(B) Obesity
(C) Cold ambient temperature after diving
(D) Dehydration
(E) AU of the above

~ Which of the following ia true regarding patients with


sternal fractures?
(A) Coexisting aortic injury occurs in half of cases.

Ffgure 7--9. (See color insert.)


Tm7 199

(A) The lesions are not easily transmitted to other


patients.
(B) Most cases are caused by group A Streptococcus.
(C) Antibiotic therapy reduces the incidence of
poststreptococcal glomerulonephritis.
(D) Regional lymphadenopathy is a common
finding.
(E) The lesions are typically painful.

lii) Which of the following is the most common organ


affected by Francisella tularensis?
(A) Skin
(B) Liver
(C) Kidney
(D) Heart
(E) Lungs

~ A28-year-oldmanwithahistoryofastbmais brought
to the .resuscitation room in your ED with a chief
complaint of wheezing. dyspnea, and cough. Your
ED has an asthma ..protocol" in which all severe
asthmaticsreceiveanimmediateportablechestx-ray.
Which of the following is true of chest x-rays in
asthma?
(A) .All asthmatics who present to the ED with
wheezing should received a chest x-ray.
(B) Chest x-rays reveal an infiltrate in 50% Figure 7-10.
of patients experiencing an asthma
exacerbation.
(C) Approximately 50% of patients not responding (iiJ Which ofthe following leads to the most severe acute
to optimal therapy have unsuspected ocular injury?
pulmonary complications evident on chest
(A) Acid
x-ray.
(B) Alkali
(D) Chest x-rays are most useful to detect the
(C) Ultraviolet light
pulmonary complications ofasthma
(D) Handsoap
exacerbations.
(E) Cigarette ashes
(E) Chest :x:-rays should only be used if an asthma
patient is febrile, with symptoms suggestive of
pneumonia.
I!!) A 42-year-old previously healthy man develops
headache, dimness, dec:reased responsiveness, right-
sided hemiplegia, and aphasia within a minute of
~ A26-year-oldman is brought to the ED after a motor
emerging from the water after diving to a shipwreck
vehicle accident. He was the restrained passenger in
with friends. The most likely cause ofhis symptoms is
a jeep traveling 45 mph when the driver lost control
and struck a tree. The patient's left knee struck the (A) Arterial gas embolism (AGE).
dashboard and he is now compJ.ainins of pain in his (B) Nitrogen narcosis.
left .knee and hip. Which of the following is the most (C) Decompression sickness (DCS).
likdy diagnosis (see Fig. 7-10)? (D) Contaminated air exposure.
(E) Altemobaric vertigo.
(A) Femoral neck fracture
(B) Open book pelvis fracture
(C) Ischial tuberosity fracture
!!!) A 24-year-old woman presents with bilateral eye
redness and pain on waking. She states that her
(D) Posterior hip dislocation
eyes are extremely watery and irritated, but denies
(E) Greater trochanter fracture
200 1000 Questions to Help You Pass the Emergency Medicine Boards

purulent discharge. She states her roommate had (E) Commonly presents with neck pain and
the same symptoms a few days ago. She does torticollis.
not wear contact lenses. Her visual acuities are
normal Physical examination demonstrates bilateral ~ A patient in your ED refuses to take penicillin
conjunctival injection without discharge. Slit lamp because she is "allergic." The historical feature most
examination is unremarkable. Which ofthe following suggestive of a true allergy in this patient is
is the most likely etiology? (A) Paresthesias.
(A) Herpes simplex virus type 1 (HSV-1) (B) Vomiting.
(B) Varicella-zoster virus (VZV) (C) Fatigue.
(C) Adenovirus (D) Urticaria.
(D) S. pneumoniae (E) Palpitations.
(E) Pseudomonas aeruginosa
~ Which of the following is true regarding brown
~ Cold allodynia, the sensation of pain or dysesthesia recluse spiders?
when coming into contact with cool or cold objects (A) Most bites require no treatment and resolve
(often called cold reversal) is virtually pathognomonic without complications.
for which of the following causes of acute food (B) Hemorrhagic lesions require urgent
poisoning? dermatologi.c consultation for excision.
(A) Scombroid (C) Dapsone is used to prevent secondary bacterial
(B) Shigellosis infection of brown recluse bites.
(C) Clostridium perfringens (D) Topical steroids decrease the severity of the
(D) Ciguatera tissue reaction.
(E) Bacillus anthracis (E) Aspirin should be used to decrease platelet
aggregation and thrombosis.
~ A 57-year-old man with hypertension, hypercholes-
terolemia, peripheral vascular disease, and smoking ~ A 24-year-old woman presents to the ED with left
history presents with inability to walk due to pain. facial pain. She had been diagnosed with right-sided
He has had progressive worsening of pain in both trigeminal neuralgia 1 year ago and was started on
his legs on walking for several weeks, and the left carbamazepine. Although her symptoms resolved
calf has been extremely painful for 2 days. Vital over several weeks, she is now complaining ofsimilar
signs are: T 98.2, HR 90 regular, BP 175/90, RR 20, symptoms on the right side of her face. In addition,
Sp02 98% RA. Examination reveals a regular heart she notes a history of mild right arm weakness and
rate, 1+ DP/PT pulses on the right, absent DP/PT numbness several months ago that seems to have
pulses on left, ABI 0.55 on the right and 0.40 on resolved without any intervention. Which of the
the left, no bruits, and no signs of infection. Which following should be considered in this patient?
of the following is the most likely pathophysiologic (A) Lyme disease
mechanism? (B) Multiple sclerosis (MS)
(A) In situ thrombosis (C) Guillain-Barre syndrome
(B) Arterial embolism (D) Myasthenia gravis (MG)
(C) Inflammation (E) Cerebellopontine angle tumor
(D) Vasospasm
(E) Arteriovenous (AV) fistula ~ A 25-year-old man presents after a motor vehicle
collision. Primary survey is intact and vital signs
~ Oropharyngeal dysphagia: are normal. Genitourinary examination is shown in
(A) Results in more difficulty swallowing solids Figure 7-11. Which of the following is the most
than liquids. appropriate next step in management?
(B) Is a common result of stroke. (A) Foley catheter placement
(C) Is almost never associated with myasthenia (B) Retrograde urethrogram
gravis. (C) Retrograde cystogram
(D) Is characterized by progressive, unremitting (D) Urinalysis
dysphagia. (E) Aspiration of the corpora cavemosa
Tm7 201

Figure 7-12.
Figure 7-11.
(A) Epidural abscess.
(B) Myelitis.
~ Which of the following statements about foreign
(C) Central disk herniation.
body aspiration in the elderly is correct?
(D) Metastatic disease.
(A) Asphyxia due to foreign body aspiration is eaaily (E) Epidural hematoma.
recognized by emergency medicine persotmel
(B) Asphyxia is not posSl'ble from aspiration of ~ A 22-year-old woman presents with a gunshot
semisolid foods such as mashed fruit or puree. wound to the left chest. Chest X-ray is shown in
(C) Most symptomatic aspirations in dderly Figure 7-13. Which of the followins is the most
patients are located in the supraglottic region. likely diagnosisf
(D) The best initial treatment ofsuspected
aspiration in an elderly person is the
application of multiple back blows.
(E) Poor dentition is not a factor in foreign body
aspiration.

l!iJ In a patient with a systolic blood pressure of 90 mm


Hg after blunt abdominal trauma, the pictured
finding on PAST examination, as shown in Pis- 7-12:
(A) Is an indication for exploratory laparotomy.
(B) Indicates that at least 2.5 L ofblood is present in
the peritoneum.
(C) Indicates a liver laceration is present.
(D) Should be followed by aCT scan to elucidate
the injury.
(E) Shows all of the above are true.

I!!) The most common cause of epidural spinal cord


Figure 7-13.
compression is
202 1000 Questions to Help You Pass the Emergency Medicine Boards

(A) Pneumothorax (E) Posteriorly placed leads are more accurate than
(B) Pericardia! tamponade anterior leads.
(C) Hemothorax
(D) Pneumoperitoneum [100] Which of the following is the most common
(E) Diaphragmatic rupture relationship of the caregiver to the victim of
Munchausen's syndrome by proxy?
I!!] Which of the following is true regarding EKGs in (A) Biological father
posterior wall myocardial infarction (MI)? (B) Step-father
(A) Abnormalities are most commonly seen in V3. (C) Biological mother
(B) ST elevations in precordial leads are diagnostic. (D) Step-mother
(C) Tall R waves are the earliest findings. (E) Nonparent guardian
(D) T -wave inversions are common in Vl-V3.
Answers and Explanations
[I) Answer A. The temporal sequence of EKG mor- [!) Answer B. The HELLP syndrome is a severe mani-
phologies in acute MI is generallyhyperacute Twaves, festation ofpreeclampsia characterized by hemolysis,
ST elevation, T-wave inversion, and Q waves. Hy- elevated liver enzymes, and low platelets. However,
peracute T waves may also be seen in hyperkalemia, liver function tests are abnormal in 20% to 30%
and (along with ST elevation) in benign early re- of patients overall, and is not limited to patients
polarization, acute pericarditis, and left ventricular with HELLP syndrome. Acute fatty liver of preg-
hypertrophy (LVH). ST elevation may also be seen in nancy (AFLP) is an exceedingly rare disorder (at
bundle branch block and ventricular aneurysms. The least ten times less common than HELLP syndrome)
evolution in the preceding sequence in the appro- that occurs during the third trimester. It is her-
priate clinical setting usually points to ST -elevation alded by the presence of nausea and vomiting in
myocardial infarction (STEMI). This is a strong argu- the third trimester with associated epigastric pain
ment for performing repeated EKGs in the evaluation and liver dysfunction. In contrast to the HELLP
of acute coronary syndrome in the ED. syndrome, coagulation abnormalities, including an
elevated prothrombin time (PT) are present early in
[!) Answer B. Button batteries lodged in the esopha- the disease course. Hepatitis is the most common
gus or trachea can cause obstruction, necrosis, and cause of liver disease in pregnancy. As in nonpreg-
perforation. Esophageal button batteries should be nant women, however, most patients experience a
removed urgently with upper endoscopy. Any but- subclinical illness and do not report any symp-
ton battery distal to the esophagus may be managed toms. Symptomatic patients present with jaundice
expectantly and will likely pass without any specific or scleral icterus, nausea and vomiting, and right
treatment. Ipecac is rarely indicated for any inges- upper quadrant tenderness with aminotransferase
tion anymore. Activated charcoal is not indicated for levels in the thousands. The course in pregnant and
foreign body ingestions. Whole bowel irrigation may nonpregnant women is typically benign and indis-
be useful to help speed passage of a postesophageal tinguishable from one another. Cholecystitis is the
button battery that has been slow to progress with second most common surgical emergency during
expectant management alone. pregnancy and patients typically present with fever,
right upper quadrant pain, nausea and vomiting, and
[I) Answer D. Cocaine may be taken in a variety of leukocytosis (although this can be confused with the
routes, most commonly intranasal and inhalational leukocytosis of pregnancy). Intrahepatic cholestasis
(crack). Inhalational and intravenous use causes the of pregnancy (ICP) is, like AFLP, a rare disorder that
quickest onset of action, followed by intranasal, and typically complicates the third trimester. Patients
then oral. The duration ofaction is longest in oral, fol- present with moderate to severe pruritus that typi-
lowed by intranasal, then intravenous/inhalational. cally begins on the palms and soles and progresses
The transdermal route is not used for cocaine abuse. in an ascending manner. Approximately 20% of pa-
tients will also be jaundiced on presentation. ICP is
[!] Answer A. Fractures are found in most abused associated with increased preterm delivery, increased
children. Although no fracture is 100% specific perinatal mortality, and meconium staining. Fetal
for child abuse, several types are extremely high mortality approaches 20% in untreated patients. The
risk-any fractures in in£mts (especially of the fe- optimal treatment for preeclampsia, AFLP, and ICP
mur) and spiral, multiple, rib, metaphyseal, humerus, is delivery.
and scapula fractures. Complete skeletal surveys are
indicated for patients younger than 5 years who are 00 Answer A. The radiograph demonstrates an an-
suspected of being abused. Osteogenesis imperfecta terior shoulder dislocation, with the humeral head
is a rare disease, which causes problems in bone syn- displaced anteriorly relative to the glenoid. Com-
thesis due to collagen defects. Frequent fractures are mon complications include (in order of decreasing
common and physical examination may demonstrate frequency) axillary nerve injury (causing deltoid
blue sclerae, deafness, and ligamentous laxity. Sub- anesthesia), humeral head fracture (Hillman-Sachs
clinical cases may be more common than previously deformity), and glenoid rim disruption (Bankart le-
recognized. Bone tumors and cysts and hypocalcemia sion). Vascular compromise in shoulder dislocations
may predispose to fracture but are not as likely to is rare. Acromioclavicular (AC) separation and clav-
cause femoral fractures in the infant as child abuse. icular fractures are rarely associated with shoulder
203
204 1000 Questions to Help You Pass the Emergency Medicine Boards

dislocations because of different mechanisms. The [!] Answer D. Increasingly, acute adrenal insuffi-
mechanism of an anterior dislocation is trauma to ciency is thought to be a rare condition, and
the abducted and externally rotated upper extremity. most cases of acute adrenal insufficiency probably
Clavicle fractures and AC separations tend to occur represent an exacerbation of chronic disease. When
with trauma to the adducted arm. (Figure courtesy of acute adrenal insufficiency occurs, it is most com-
Shoma Desai, MD. Reprinted with permission from monly due to exogenous glucocorticoid adminis-
Desai S. Greenberg's text-atlas of emergency medicine. tration. Chronic adrenal insufficiency is idiopathic
Lippincott Williams & Wilkins; 2004:497.) (thought to be autoimmune-mediated destruction of
the adrenal gland) in 66% to 75% of cases.
(I] Answer A. Complicated parapneumonic effusion
and empyema are rare, but severe complications ~ Answer E. The treatment of bacterial meningitis
of pneumonia are associated with a mortality rate varies by age of the patient. All patients should
of 15% to 20%. Although diagnostic thoracentesis be treated with a third-generation cephalosporin to
is a procedure that is rarely performed in the ED, cover meningococcus, pneumococcus, and gram-
it is necessary to diagnose life-threatening con- negative bacilli. At the extremes of age, Listeria
ditions such as emphysema as soon as possible. monocytogenes is a more common pathogen and
Approximately 60% of patients with pneumonia should be specifically treated with either ampicillin or
develop some amount of pleural fluid, but most trimethoprim-sulfamethoxazole. Vancomycin may
of these patients have a simple parapneumonic ef- be used in addition to these therapies to cover
fusion. The characteristics of pleural fluid in the methicillin-resistantS. aureus in endemic areas. Dex-
setting of pneumonia determine whether the ef- amethasone should be given before or simultaneously
fusion is simple, complicated, or if frank pus is with the first dose of antibiotics.
present, an empyema. These three distinct cate-
gories of disease represent the typical progression ~ Answer C. The EKG demonstrates multifocal
of disease in patients with parapneumonic ef- atrial tachycardia (MAT), which is associated most
fusions. Pleural fluid pH, lactate dehydrogenase commonly with COPD, followed by congestive
(LDH), glucose, and Gram stain determine into heart failure (CHF), hypokalemia, and hypomag-
which category the effusion is classified. Simple nesemia. There are P waves of at least three
parapneumonic effusions are clear, with a pH different morphologies associated with an irregu-
>7.2, LDH <1,000, glucose >40 mg per mL, lar tachycardia. Management involves treatment of
and a negative Gram stain. Conversely, compli- the underlying condition. Patients with MAT rarely
cated parapneumonic effusions may be turbid, have hemodynamic compromise as a result of their
have a pH <7.2, LDH >1,000 (reflecting more dysrhythmia. (Figure from Fowler NO. Clinical elec-
severe inflammation), glucose <40 mg per mL trocardiographic diagnosis: A problem-based approach.
and may have a positive Gram stain. A pH <7.0 Philadelphia: lippincott Williams & Wilkins; 2000,
is typically found only in the setting of frank pus with permission.)
in the pleural space, called empyema. Indications
for tube thoracostomy include a pH <7.2 {i.e., ~ Answer B. The patient has a lesion at 7 o'clock in-
a complicated parapneumonic effusion), a posi- dicative of a corneal ulcer. In a contact lens wearer,
tive Gram stain, or the presence of frank pus Pseudomonas is a potentially devastating cause and
in the pleural space. Infected parapneumonic ef- should be treated aggressively with topical and/or sys-
fusions must be drained as soon as possible to temic antibiotics and emergent ophthalmologic con-
prevent the development of subsequent adhesive sultation. Further contact lens use is contraindicated
disease resulting in significant loculations and scar until approved by the ophthalmologist. A corneal
tissue. A chest surgeon should be consulted any- abrasion appears as a greenish patch on fluorescein-
time a chest tube is inserted for empyema or staining under cobalt blue light. Herpes zoster oph-
complex parapneumonic effusion, because surgery thalrnicus is suggested by the presence of facial skin
may ultimately be necessary to ensure adequate lesions with pseudodendrites on slit lamp examina-
drainage. (Figure reprinted with permission from tion. Herpes simplex keratitis is diagnosed when den-
Harris JH. The radiology of emergency medicine, drites are seen on slit lamp examination. Hyphema
4th ed. Lippincott Williams & Wilkins; 1999:444.) (blood layering at the bottom of the anterior cham-
ber) is not evident here. (Figure from Tasman W,
(!] Answer E. The most common dysrhythmia in Jaeger EA, eds. The Wills Eye Hospital atlas of clini-
hypothyroid cardiac disease is sinus bradycardia. cal ophthalmology, 2nd ed. Philadelphia: Lippincott
Cardiomegaly and depressed cardiac contractility are Williams & Wilkins; 2001, with permission.)
other manifestations.
Test7 205

[!jJ Answer E. The risk for cholelithiasis is two- to the wound further reduces the risk of bloodbome
threefold higher in women than in men through the pathogen transmission.
fifth decade, after which time, the risk is roughly
equal This is thought to be related to estrogen [!1) Answer D. Prolongation of the QT interval may
secretion. Other risk factors include estrogen use be due to congenital or acquired causes. Congenital
(either in oral contraceptives or as isolated estrogen long QT syndromes may be associated with deafness
therapy), age (because gallstones rarely dissolve (Jervell-Lange-Nielsen syndrome), or without deaf-
spontaneously, their prevalence increases with age), ness (Romano-Ward syndrome). Acquired causes
pregnancy (probably due to estrogen), obesity, and can be from electrolyte abnormalities, environmental
rapid weight loss. conditions, or medication effects. Excessive pro-
longation can result in potentially fatal ventricular
[HJ Answer D. Calcium gluconate antagonizes the dysrhythmias, such as ventricular fibrillation and tor-
effect ofmagnesium and should be given immediately sade de pointes. Treatment is usually with magnesium
in all patients with any sign ofrespiratory depression. and standard advanced cardiac life support (ACLS)
The magnesium infusion is discontinued, and 1 g of protocols. Prevention of ventricular dysrhythmias
calcium gluconate is infused over 2 to 3 minutes. is with chronic ,8-blocker therapy and sometimes
pacemakers/defibrillators.
~ Answer D. Clavicular fracture is extremely com-
mon with shoulder trauma. The vast majority of [!jJ Answer D. Although there is variability in the re-
clavicular fractures are treated conservatively with ported incidence of aspirin-induced asthma (AlA),
shoulder sling and orthopedic follow-up. Operative recent studies estimate that its incidence is 3% to
repair is almost never indicated in nondisplaced, 5% in asthmatics. Overall, it accounts for 10% to
closed fractures, because conservative management 15% of asthmatics. The actual prevalence is likely a
results in excellent functional outcomes. There is bit higher because many asthmatics fail to recognize
no proven advantage of the figure-of-eight brace mild aspirin-related symptoms or purposely avoid
over simple sling, and it may actually cause skin aspirin-containing products after becoming aware of
necrosis if applied too tightly. Emergent MRI is potentially adverse complications without conveying
almost never indicated in acute injuries, and clav- this to their physicians. Diagnosis is made through
icle fracture with concomitant rotator cuff injury aspirin-provocation tests after patients present with
is rare. Shoulder arthrocentesis is indicated only in asthmatic symptoms associated with aspirin use. It is
cases of suspected septic arthritis and has little role far more common in adult asthmatics than in chil-
in acute trauma. dren with asthma, and is more common in women
than men. After ingesting acetylsalicylic acid (ASA)
~ Answer C. HepB is very effectively transmitted or nonsteroidal anti-inflammatory drugs (NSAIDs),
through percutaneous needlestick exposure, even asthmatics may experience an acute exacerbation of
though the rate of transmission depends on the symptoms within 3 hours that is frequently accom-
presence of the "e" antigen (hepatitis B e antigen panied by profuse rhinorrhea, conjunctival injection,
[HBeAg]) indicating higher infectivity. Health care periorbital edema, and occasionally facial flushing.
providers exposed to HBeAg-positive needlesticks Unfortunately, bronchoconstriction may be severe
develop clinical evidence of hepatitis in approxi- and life threatening, requiring mechanical ventila-
mately 33% of cases and serologic evidence in up tion. Compared with all patients presenting with
to 62%. In contrast, exposure to HBeAg-negative asthma exacerbations, aspirin-induced asthmatics
patients results in clinical hepatitis in only 1% to more frequently require intubation and mechanical
6% of cases and serologic evidence in up to 37%. In ventilation. Wegener's granulomatosis is an antineu-
contrast, exposure to a hepatitis C-positive source trophil cytoplasmic antibody (ANCA)-associated
results in an infection rate of roughly 1.8% (0%-7%). small vessel vasculitis that most commonly affects
All patients who have not received the HepB vaccine the lungs and kidneys, but is not associated with AlA.
as well as patients who were vaccinated with a sin-
gle series but who failed to respond should receive [!!) Answer D. This patient experienced preterm pre-
both the HBIG and the HepB vaccine series. The mature rupture of membranes (PPROM), defined as
HepB vaccine should always be given in the deltoid premature rupture of membranes that occurs before
muscle with a needle 1 to 1.5 in. long (apparently 37 weeks' gestation. PROM is defined as rupture of
better response rates have resulted from deltoid in- fetal membranes before the onset of labor and it
jection). There is no evidence that using antiseptics occurs in approximately 8% ofpregnancies. The first
for wound care or expressing fluid by squeezing step in the evaluation of any pregnant woman with
206 1000 Questions to Help You Pass the Emergency Medicine Boards

a history of a "gush of fluid" or a "sensation of wet- ~ Answer C. Enteroviruses, such as coxsackievirus,


ness" is verification that the fetal membranes have account for more than half of all the cases of en-
ruptured. A sterile speculum examination should be cephalitis. Herpes virus is the most common cause
performed on all women to collect fluid for further of severe encephalitis in the United States. Typi-
testing. In contrast, a digital examination should not cally, in frank infections of the brain parenchyma
be performed until a diagnosis of PROM is excluded (encephalitis), there is some degree of concomi-
or until labor has begun and delivery is expected tant infection and inflammation of the meninges.
within 24 hours. Digital examinations increase the Therefore, meningoencepluditis is a more accurate
risk of chorioamnionitis as well as neonatal sepsis term describing most of these infections. When the
and mortality. Although pooled fluid in the poste- infection is limited to the meninges, it is termed
rior fornix is suggestive of PROM, the presence of meningitis, reflecting meningitis in the absence of
ferning (arborization pattern of dried amniotic fluid a pyogenic organism. Mumps is another important
when allowed to air dry on a microscope slide) and cause of aseptic meningitis, especially in the setting
a positive nitrazine test (blue or blue-green color of outbreaks of the virus such as in Iowa in early
of nitrazine paper upon exposure to the fluid) are 2006. Meningoencephalitis is the most common
diagnostic. Both feming and the nitrazine test have complication of childhood mumps and frequently
sensitivities >90%. However, the presence of blood, occurs at the same time as parotitis. Neurocysticer-
semen, excessive cervical mucus and TrichomotulS cosis is a rare cause typically found in immigrants
vaginitis can cause false-positive nitrazine tests be- from areas where undercooked pork harboring the
cause each of these entities raises the vaginal pH. parasite Taenia solium may be eaten. Adenovirus
The normal vaginal pH during pregnancy is 3.5 to is an uncommon cause.
6.0, although the pH of amniotic fluid is 7.1 to 7.3,
which causes nitrazine paper to turn a blue color. In ~ Answer B. Thepatienthasacutemastoiditis. There
addition, cervical mucus also causes a ferning pat- is erythema behind the auricle in the setting of otic
tern, but it tends to be more floral in appearance pain. Mastoiditis is usually seen as a complication
and lacks the branches of amniotic fluid ferning. of otitis media and occurs when the mastoid air
Once a diagnosis of PROM is confirmed, all such cells are blocked by infectious debris. Streptococci
patients should be transferred to a tertiary care facil- are the most common cause. The infection can
ity for further management. Although management spread to cause a generalized skull osteomyelitis
techniques vary, there is broad agreement that all with associated cranial neuropathies and meningitis.
patients with PROM should receive intravenous an- Diagnosis is primarily made clinically, but CT scan of
tibiotics to prevent chorioamnionitis and neonatal the mastoid area can provide important information
sepsis. Most authors recommend prompt delivery for the consulting ENT doctor. Plain radiographs
rather than conservative attempts to prolong ges- are too insensitive for the diagnosis. MRI is not
tation once the fetus has reached 34 to 36 weeks necessary in the acute setting. Tympanocentesis is
of age. unlikely to change management. Lumbar puncture is
indicated only in the setting of signs and symptoms
~ Answer B. C. diphtheriae is a gram-positive bacil- of associated meningitis, but a Cf scan should be
lus, which exerts its pathologic effects through an performed first to assess for possibility of mass
exotoxin. The bacterium initially affects the phar- effect from localized infection. (Figure courtesy of
ynx, where the exotoxin kills cells directly and forms Mark Silverberg, MD. Reprinted with permission
a membrane. The exotoxin then spreads to other from Silverberg M. Greenberg's text-atlas ofemergency
tissues and causes neurologic and cardiac dysfunc- medicine. Lippincott Williams & Wilkins; 2004: 135.)
tion, including paralysis of the pharyngeal muscles
and myocarditis. Although most Americans are im- ~ Answer B. Foods are the single most common
munized as children, a substantial portion of the cause of anaphylaxis although up to one third
population is not. Patients usually present with of causes are unknown. Exercise accounts for 7%
fever and sore throat, and most exhibit the gray of anaphylaxis cases. Anaphylaxis occurs as an
pseudomembrane. Positive cultures for Group A immediate type I hypersensitivity reaction mediated
{J- hemolytic streptococcus does not rule out diphthe- by immunoglobulin E (IgE) antibodies. It requires
rial pharyngitis in the presence of suggestive history prior sensitization to the allergen in order to
and physical examination findings. Airway obstruc- develop allergen-specific lgE antibodies. Subsequent
tion is the most serious sequela. Treatment involves exposure to the allergen allows mast cell and basophil
macrolide antibiotics, horse-derived antitoxin, and degranulation leading to the subsequent anaphylactic
active immunization.
Test7 207

response. Anaphylactoid reactions are immediate life-saving technique for laypersons to help dislodge
type I hypersensitivity reactions that do not require a foreign body which is obstructing a patient's airway.
prior allergen sensitization and which clinically
mimic anaphylaxis. Anaphylactoid reactions (e.g., ~ Answer C. Hirudin, a protein originally extracted
response to iodinated radiographic contrast material) from leeches, directly inhibits thrombin without
typically require a larger dose of the offending requiring antithrombin III activity. Although it also
agent but the clinical management is identical to inhibits platelet aggregation, hirudin does not cause
anaphylaxis. Patients at the extremes of age are less thrombocytopenia, which can be associated with
likely to have anaphylaxis, probably because of less both unfractionated heparin and low molecular
mature immune responses. weight heparins. It may cause a slightly higher
incidence of bleeding than heparins when used
~ Answer B. Mannitol is used in patients with severe clinically, and is therefore limited to use for patients
head injury and signs of increased intracranial pres- who cannot use heparins due to the risk for heparin-
sure. It functions primarily as a diuretic to limit cere- induced thrombocytopenia.
bral edema, but has important secondary functions
as a volume expander and antioxidant. Mannitol ~ Answer E. The definition of septic shock is sepsis
improves cerebral perfusion pressure by increasing plus inadequate tissue perfusion. This inadequate
mean arterial pressure and decreasing intracranial perfusion may be manifested by the presence of
pressure. It begins to work within 10 minutes, peaks hypotension, but septic shock can occur in the
at approximately an hour, and lasts up to 6 hours. absence ofhypotension (and vice versa).The definition
Mannitol has no coagulant properties and does not of sepsis is suspicion/presence of infection plus two
limit the extent of bleeding. It does not inhibit the in- out ofthe four criteria for the systemic inflammatory
flammatory cascade. Mannitol has no direct effect on response syndrome (SIRS)
seizure prevention except by reduction ofintracranial • Temp >38°C or <36°C
pressure and hypoxia. Renal failure is an important • WBC > 12 K or <4 K or > 10% bands
adverse effect that may occur with repeated doses. • RR >20 or Paco2 <32 mmHg
• HR>90
~ Answer B. Flexible endoscopy is the procedure of
choice for removal of esophageal foreign bodies. A ~ Answer D. Goodpasture's syndrome is character-
recent trial demonstrated that between 25% and 33% ized by glomerulonephritis and diffuse alveolar hem-
of esophageal coins will spontaneously pass without orrhage (DAH), associated with the presence of a
complications within 8 to 16 hours of ingestion. glomerular antibasement membrane antibody. In the
Spontaneous passage is more likely in older, male past, the 6-month mortality was 80% with half the
children with coins lodged in the distal third of number of patients succumbing to DAH. A recent
their esophagus. However, this patient has already study estimated that mortality has improved some-
presented after a tincture of time has passed, and what, with a 2-year survival rate of 50% in all treated
endoscopy remains standard of care. Papain, a pro- patients. DAH is the most common cause of death.
teolytic enzyme that is a common active ingredient
in commercially available meat tenderizer, has been ~ Answer D. Barotrauma and ventilator-associated
used in the past to aid in the passage of impacted lung injury (VALI) are important considerations
meat boluses. However, its use is associated with in the mechanically ventilated patient with asthma.
an unacceptably high rate of complications includ- Injury to the lung is primarily dependent on the
ing esophageal perforation, aspiration pneumonitis, transpulmonary pressure (the pressure gradient be-
and hemorrhagic pulmonary edema. Therefore, its tween the alveoli and the pleura). Alveolar pressure
use should be avoided in the ED. Glucagon and is best estimated by determining the plateau pres-
effervescent agents such as carbonated beverages sure, which is measured by the ventilator at the
may both prove useful in alleviating an impacted end of inspiration. Because no air is flowing at the
food bolus. Glucagon is thought to work by re- end ofinspiration, plateau pressure reflects the static
laxing the smooth muscle of the distal esophagus pressure in the ventilator circuit that is exerted on
and most markedly, the lower esophageal sphincter. the alveolar wall. The incidence of barotrauma and
It does not have any appreciable effect on upper VALI increases exponentially as the plateau pressure
esophageal motility. Effervescent agents should be rises above 35 mm Hg. Peak airway pressure re-
avoided in cases of complete obstruction or obstruc- flects the maximal pressure achieved in the ventilator
tion persisting for longer than 24 hours due to the circuit throughout a ventilatory cycle. The pressure
theoretically increased risk of perforation due to is- is typically maximal during inspiration when air
chemia. The Heimlich maneuver is indicated as a
208 1000 Questions to Help You Pass the Emergency Medicine Boards

is being pushed into the lungs by the ventilator. ~ Answer C. During primary and secondary survey
Most of this force is absorbed by the "resistance of all victims of violent crime, care must be taken
airways" (primarily medium-sized bronchi) rather to identify objects which are potentially lethal to
than the alveoli. Therefore, slight elevations in peak caregivers. Handguns are particularly dangerous as
airway pressure are better tolerated than elevations they may discharge with only minimal movement
of plateau pressure. Furthermore, patients with re- during exposure of the patient. Any firearm should
fractory asthma usually have significant airway ob- preferably not be handled at all-ifit is in a potentially
struction and require high inspiratory flow rates and dangerous position, it should be placed carefully in a
correspondingly short inspiratory times. This max- safe location and secured by law enforcement. Care
imizes the time for expiration, thereby limiting air providers should never check to see if the firearm
trapping and a subsequent increase in plateau pres- is loaded, fire it, or carry it on their person in case
sure. However, peak pressure rises as a result of this spontaneous discharge occurs. Using the firearm for
strategy and should be tolerated in order to preserve any reason is obviously contraindicated.
normal plateau pressures. PEEP is often not used
when mechanically ventilating patients with asthma ~ Answer A. The patient has evidence of acute renal
due to intrinsic air trapping, which generates auto- failure, with abnormal BUN and creatinine levels.
PEEP. Lower tidal volumes are often necessary in Etiologies include prerenal azotemia, intrinsic renal
asthmatic patients to ensure adequate exhalation, disease, and postobstructive uropathy. No specific
avoiding air trapping. history is given to indicate any particular cause so
the diagnosis must be based on laboratory data. The
~ Answer A. This patient has primary adrenal in- fractional excretion of sodium (FENa) is used as
sufficiency. Owing to mineralocorticoid deficiency, surrogate to measure the reabsorption capacity of
the most common electrolyte abnormalities are hy- the kidneys-a low urinary sodium indicates that
ponatremia and hyperkalemia. Hypoglycemia due to the kidneys are adequately reabsorbing sodium and
decreased gluconeogenesis and increased peripheral working appropriately. The FENa is calculated from
glucose utilization is also common. The patient's the serum and urine sodium and creatinine values
hyperpigmentation is due to increased adrenocorti- according to the following equation
cotropic hormone (ACTH) secretion by the pituitary (Urine Na)(Plasma Cr)
gland and subsequent stimulation of melanocytes. FENa = (Urine Cr)(Plasma Na).
More than 50% ofpatients with adrenal insufficiency A FENa < 1% implies adequate sodium reab-
have intermittent nausea and vomiting, and often sorption and a prerenal process--the kidneys are
present with volume depletion. working appropriately, responding to reduced blood
volume by conserving sodium and water. A FENa
~ Answer B. Despite their widespread use, neither > 1% implies either intrinsic renal disease or pos-
intravenous corticosteroids nor racemic epinephrine tobstructive uropathy. In this patient, the FENa is
has been shown to be beneficial in the management of < 1%, indicating a prerenal cause such as diarrhea
adult epiglottitis. The foundation of effective man-
from gastroenteritis. Transitional cell carcinoma and
agement is the early administration of appropriate prostatic hypertrophy would both cause chronic ob-
antibiotics and airway management, which may in-
struction and a FENa > 1%. Acute tubular necrosis
clude early intubation or observation in a monitored and pharmacologic diuresis also cause a FENa > 1%.
setting. While H. influenzae type Bremains an impor-
tant cause of this disease in adults (the Haemophilus ~ Answer A. Roughly 70% of patients with mening-
influenzae type b [Hib] vaccine has drastically re- ococcemia have a rash which usually begins as
duced this entity in children), it is found in as few petechiae of the wrists and ankles. However, the
as 17% of cases. Other important bacterial causes in- rash may also be composed oflight pink macules that
elude Streptococcus, and numerous gram-negative or- become petechial later in the disease course. Petechial
ganisms. While ampicillin was a drug of choice in the lesions may have a "smudged" appearance although
past, H. influenzae and other pathogens are increas- purpura may have a "gun-metalgray" center. Rubella
ingly resistant to this therapy due to the presence of and rubeola are maculopapular eruptions that tend
IJ-lactamase. Therefore, second and third-generation to begin on the face and spread inferiorly. Henoch-
cephalosporins are the drugs of choice (e.g., ceftri- Sch5nlein purpura (HSP) is a vasculitis that is
axone, cefotaxime, ceftizoxime). Humidified oxygen characterized by palpable purpura on the buttocks
is another component of epiglottitis therapy that has
and lower extremities. Pityriasis rosea is a benign,
no proven benefit. However, because there is little self-limited rash characterized bypatches and plaques
potential harm in humidifying delivered oxygen, it
that typically occur on the trunk.
remains a part of recommended treatment.
Test7 209

~ Answer E. High-altitude pulmonary edema SSRI overdose, as the drug is highly bound to plasma
(HAPE) is the most common cause of high alti- proteins.
tude-related death. The mechanism is not completely
understood but it is thought to result from overper- ~ Answer E. Enterotoxigenic Escherichia coli (ETEC)
fusion of a constricted pulmonary vascular system is responsible for 45% to 50% of traveler's diarrhea.
and subsequent vascular injury and leak. Clinical Shigella is the second most common cause of trav-
symptoms include a dry cough, dyspnea on exertion, eler's diarrhea, whereas viral causes and protozoa
and fatigue with minimal effort. Symptoms typically make up the remainder.
worsen at night, and patients decompensate result-
ing in dyspnea at rest, cyanosis of the nail beds and ~ Answer C. Septic arthritis in children is most com-
lips, and development of a productive cough and monly caused by S. aureus. Gonococcal infection is
severe weakness. Orthopnea is an uncommon find- the most common cause in sexually active adolescents
ing. Dyspnea at rest is the hallmark of severe disease. and young adults. Pneumococcus and Group A strep-
The crucial therapeutic action is immediate descent tococcus are important (but less common) causes of
and the application of high-flow oxygen. In most septic arthritis in all age-groups. Haemophilus infec-
cases, these steps alone will resolve symptoms. How- tion has decreased in incidence due to Hib vaccine.
ever, nifedipine, furosemide and morphine may all Septic arthritis is the most important diagnosis to
be useful as adjuncts. Nifedipine causes pulmonary rule out in children presenting with unilateral hip or
vasodilation and is thought to counteract the high knee pain or limp. Joint aspiration is the only com-
pulmonary vascular resistance in HAPE. Nifedipine pletely reliable method of ruling out the diagnosis,
may also be used for prophylaxis. although presence of fever, joint tenderness, and ab-
normalities in WBC and erythrocyte sedimentation
~ Answer A. Standard treatment for pediatric pa- rate (ESR) levels may aid identification of those pa-
tients with acute gastroenteritis is oral rehydration tients who should undergo arthrocentesis. Treatment
therapy (ORT). Standard solutions are available and of septic arthritis in children should always include
consist of hypotonic fluid and essential electrolytes. admission, an antistaphylococcal antibiotic, and sur-
Administration of ORT may occur in the ED, hos- gical irrigation. Vancomycin should be considered in
pital floor, or at home. Patients may continue to patients at high risk for methicillin resistance.
have vomiting and/or diarrhea and still be candi-
dates for ORT. Contraindications to ORT include ~ Answer B. The Sgarbossa criteria were devised to
shock, lethargy, severe abdominal tenderness or re- assess the likelihood of infarction in patients with a
bound, bowel obstruction, and severe underlying left bundle branch block. Three different electrocar-
medical illness. Failure of ORT should necessitate diographic criteria are given specific scores: A total
intravenous fluid hydration. Treatment of acute gas- score of 3 or greater indicates that the patient likely
troenteritis with ORT observed in the ED is cost- has acute MI. Concordant (in the same direction as
effective and clinically efficacious. Hospital admis- the QRS complex) ST elevation > 1 mm is given 5
sion may be prevented by this ED observation of points, ST depression > 1 mm in Vl-V3 is given
parent-administered ORT. 3 points, and discordant ST elevation >5 mm (in
the opposite direction as the QRS complex) is given
~ Answer C. Fluoxetine is a type of selective sero- 2 points.
tonin reuptake inhibitor (SSRI). As a class, SSRis are
generally benign in overdose, causing mild GI and [!!) Answer A. In practice, probably the most com-
CNS symptoms and rarely leading to dysrhythmias. mon complication is the ampicillin or amoxicillin-
Cyproheptadine is a serotonin antagonist that has associated rash that occurs when patients with
unproven clinical efficacy in most SSRI overdoses. Epstein-Barr virus (EBV) pharyngitis are mistakenly
Its use is mainly limited to patients with serotonin treated for a bacterial pharyngitis. Such a rash may
syndrome, a constellation of neurologic, Gl, and occur in up to 95% of patients with EBV pharyngitis
cardiac findings. Dysrhythmia is rare and should who are treated with one of these two medications.
be treated according to current advanced cardiac The rash occurs less commonly when patients are
life support (ACLS) guidelines. Unlike with tricyclic treated with other P-lactam antibiotics. Elevations
antidepressant overdose, there is no indication for of the hepatic enzymes AST and ALT commonly
routine use of sodium bicarbonate to treat dysrhyth- occur (up to 80%) to approximately four times the
mias. As in virtually all overdoses, ipecac is not upper limit of normal, and peak during the sec-
recommended as a method of gastric decontamina- ond to fourth week of illness. However, levels well
tion. Hemodialysis is not indicated in patients with over 1,000 may occur. Jaundice may also be seen
210 1000 Questions to Help You Pass tM Emergency Medicine Boards

(5%) but coagulopathy is not typical. Spontaneous ~ Answer B. Aregular,narrow-complextachycardia


recovery is the rule, and supportive care is the only at a rate of close to 200 without P waves is likely
required treatment. Airway obstruction is probably due to paroxysmal arteriovenous nodal reentrant
the most important complication for the emergency tachycardia (AVNRT). The initial treatment for a
medicine physician. It is due to tonsillar hypertro- hemodynamically stable patient with AVNRT is
phy and occurs more commonly in young children adenosine, which is safe to use in pregnancy. Digoxin,
because of their prominent lymphoid tissue. Over- amiodarone, and procainamide are not indicated
all, however, this complication occurs in <5% of as first-line therapy for AVNRT. Lidocaine is not
patients and can usually be managed conservatively indicated for any narrow-complex tachycardias.
by elevating the head of the bed, giving intravenous
fluids and corticosteroids and using humidified air. ~ Answer A. The patient has phlegmasia cerulea
Splenic rupture receives a lot ofattention, but occurs dolens, which is diagnosed by the clinical triad of
in <0.5% of adults with EBV infection. The rate cyanosis, pain, and edema due to massive lower
in children is thought to be much lower. It most extremity venous thrombosis. The cyanosis is due
commonly occurs in the second and third week of to a combination of increased venous backflow
illness, and is rarely fatal when it does occur. Pa- and compromised arterial flow. Phlegmasia alba
tients present with left upper quadrant abdominal dolens is similar but lacks cyanosis and instead
pain, with or without radiation to the left shoulder appears as a blanched extremity. Treatment involves
(Kehr's sign), and splenic rupture should be con- admission, intravenous anticoagulation, and possibly
sidered especially in the setting of shock. Numerous surgical thrombectomy and fasciotomy. (Figure
hematologic complications are associated with EBV courtesy of William S. Schroder, MD. Reprinted
infection, including autoimmune hemolytic anemia with permission from Schroder WS. Greenberg's text-
(3%), mild thrombocytopenia (25% to 50%), and atlas of emergency medicine. Lippincott Williams &
mild transient neutropenia (50% to 80%). Headache Wilkins; 2004:203.)
is the most common neurologic manifestation of
EBV infection (50%), although meningoencephalitis ~ Answer B. The patient has evidence of nephrotic
is the most common severe neurologic complication syndrome with proteinuria accompanied by gener-
(1% to5%). alized fluid overload and nonspecific constitutional
symptoms. Periorbital edema is often the first lo-
~ Answer C. The patient has evidence of acute cation where fluid overload is noted. Frank re-
testicular torsion, with scrotal pain, nausea, vomiting, nal failure does not usually occur in patients with
and testicular tenderness. Pathophysiology involves nephrotic syndrome. In children, minimal change
twisting ofthe testis on the spermatic cord due to an disease is the most common cause. Hypoalbumine-
anatomic abnormality or trauma. The most sensitive mia, thrombophilia, and hyperlipidemia are charac-
physical examination finding is the cremasteric teristic. UTI is not usually concomitant or causal
reflex-presence of this reflex virtually rules out the Although microscopic hematuria may be present,
diagnosis. Diagnosis can be made clinically in some gross hematuria is rare and suggests glomeru-
cases, but confirmation is made with color Doppler lonephritis, infection, stone, tumor, or extrarenal
ultrasonography, which has excellent sensitivity and cause. Acute management generally involves careful
specificity when performed by experienced operators. fluid resuscitation, systemic corticosteroid therapy,
Prompt diagnosis is essential, as testicular survival and admission for observation and possible renal
is directly dependent on duration of symptoms-if biopsy.
surgical management is instituted within 6 hours of
pain, approximately 100% of cases are salvageable. ~ Answer D. Bloody diarrhea in a traveler or im-
Advanced imaging techniques such as CT and MRI migrant from an endemic area should always raise
scans do not add to the diagnostic accuracy of the possibility of amebic colitis. While patients with
testicular torsion and waste valuable time. Retrograde bacterial dysentery can also have bloody stools and
urethrogram is used to diagnose traumatic injuries crampy abdominal pain, their symptoms do not typ-
to the urethra and is not indicated here. The ically last for longer than 1 week. Enterotoxigenic
bulbocavernosus reflex is used to evaluate spinal E. coli is the most common cause of traveler's diar-
cord injuries in trauma patients and has no role in rhea, but is noninvasive, causes a nonbloody, water
the evaluation of the acute scrotum. diarrhea, and typically resolves within a few days to
a week. Entamoeba is found worldwide but it is par-
~ Answer A. All of the above are potential intracra- ticularly endemic to the Indian subcontinent, central
nial complications of otitis media, but direct spread and South America and Africa. Extraintestinal man-
causing meningitis is the most common.
Test7 211

ifestations, the most common of which is a liver therapeutic intra-arterial infusion of papaverine, a
abscess, only rarely occur. potent vasodilator, or thrombolytic drugs.

~ Answer B. Haloperidol is a high-potency an- ~ Answer E. Only dialysis can definitively remove
tipsychotic agent whose primary therapeutic ac- ethylene glycol from the body. Ethanol and fomepi-
tion is to block dopamine-2 receptors in the zole are temporizing measures to inhibit alcohol
basal ganglia to cause rapid sedation and control dehydrogenase from catalyzing the conversion of
of psychotic behavior. Haloperidol is commonly toxic alcohols into their toxic metabolites. Pyridox-
given with benzodiazepines for this use. Although ine and thiamine are cofactors in the conversion of
haloperidol is often mistakenly considered to be glyoxylic acid, a toxic metabolite ofethylene glycol, to
part of the phenothiazine class of drugs (which nontoxic compounds. They are useful as adjunctive
lower the seizure threshold), it is actually part of therapies for ethylene glycol poisoning, but do not
the butyrophenone category, which does not af- constitute definitive therapy.
fect the seizure threshold. Antipsychotics are a-1
antagonists (causing orthostatic hypotension) and ~ Answer E. The patient has evidence of acute
antihistaminergic (causing sedation). Agranulocy- monoarticular arthritis. In a young, sexually active
tosis is a side effect peculiar to clozapine, a newer patient without prior history of arthritis, the most
generation antipsychotic. likely bacterial cause is gonococcus. The overall
most common cause of septic arthritis is S. aureus.
~ Answer A. This patient has mesenteric ischemia, Choices B, C, and D are less common causes. Septic
which is a disease that still has a mortality rate of arthritis is a joint-threatening infection diagnosed by
70%. Therefore, any patient older than 50 years synovial fluid analysis. It must be aggressively treated
who has risk factors for acute mesenteric ischemia with intravenous antibiotics and possible surgical
and who presents with acute-onset abdominal pain irrigation, even though gonococcal arthritis rarely
should be presumed to have mesenteric ischemia requires surgical management.
until proved otherwise. The key to diagnosis is
recognizing patients at risk, which includes any pa- ~ Answer E. The anterior circulation refers to there-
tient older than 50 years who presents with acute gion of brain tissue served by the internal carotid
abdominal pain and who has known vascular dis- arteries. Approximately 80% of cerebral blood flow
ease, cardiac arrhythmias, recent MI, hypovolemia, is derived from the carotid arteries. This includes the
hypotension, or sepsis. The most commonly cited anterior cerebral artery, the middle cerebral artery
clinical finding is patient pain that is out of pro- and the ophthalmic artery (branch of the internal
portion to tenderness elicited on physical exam- carotid artery just before joining the circle ofWillis).
ination. This is a nonspecific finding that needs Hemiparesis and hemisensory loss of the leg is a re-
to be considered carefully in light of the clini- sult of anterior cerebral artery ischemia. Transient
cal scenario. Unfortunately, there are no laboratory monocular blindness (amaurosis fugax) is a result
markers or radiologic studies apart from angiogra- of ischemia in the ophthalmic artery. Aphasia and
phy that have sufficient sensitivity and specificity hemiparesis are due to ischemia in the middle cere-
to exclude acute MI early in its course. Lactate lev- bral artery. Ataxia and vertigo along with nausea and
els are elevated in approximately 100% of patients vomiting are caused by posterior circulation ischemia
with bowel infarction, but this is a late finding and (to the cerebellum). The posterior circulation origi-
mortality rates are high by the time infarction has nates in the vertebral arteries (which originate from
occurred. Plain films are most commonly nonspe- the subclavian arteries), which join to form the basilar
cific, although even minimal findings such as ileus artery and then bifurcate to form the two posterior
correspond to more severe disease and a higher cerebral arteries. This system supplies the remain-
mortality rate. The sensitivity of cr has been cited ing 20% of cerebral blood flow, delivering blood to
to be as high as 82%, but the most common early the cerebellum and brainstem as well as to the visual
finding is bowel wall thickening, present in 26% occipital lobe and medial aspect ofthe temporal lobe.
to 96% of cases. Unfortunately, this is also the
least specific finding and is often not present in ~ Answer E. Wolff-Parkinson-White (WPW) syn-
mesenteric ischemia due to arterial embolism or drome is the most frequently occurring accessory
thrombosis, which are the most common causes pathway syndrome. Patients have an accessory con-
of acute MI. Pneumatosis intestinalis or gas in the ductive pathway from the atria to the ventricles which
portal venous system is a specific finding but is preexcites the ventricular myocytes before the AV
only present after bowel infarction has occurred. node releases the normal sinoatrial depolarization.
Angiography is the key to diagnosis and allows for As a result, patients with WPW have a shortened
212 1000 Questions to Help You Pass tM Emergency Medicine Boards

PR interval and a delayed QRS upstroke, called the common, and occur in 25% to 40% of patients with
6 wave. Patients with WPW syndrome can have reen- cirrhosis. Of those patients, 30% develop bleeding
trant dysrhythmias, where the accessory pathway can within 2 years. Hepatic encephalopathy is also a
either conduct retrograde (where the AV node con- common complication, but its incidence depends on
ducts in the normal direction, producing a narrow the criteria used to diagnose encephalopathy. It most
QRS complex and an "orthodromic" pattern) or an- commonly presents as a sleep disturbance, although
terograde (where the AV node conducts backwards, patients may have trouble with mood, disorientation,
producing a wide QRS complex and an "antidromic" or speech.
pattern). A patient with WPW syndrome, tachycar-
dia, and wide QRS complexes suggests the presence of ~ Answer D. The Mantoux test is the most common
an antidromic conduction pattern (where the acces- test used to screen for TB exposure and infection.
sory pathway conducts anterograde and the AV node It consists of an intradermal injection of 10 units
conducts retrograde). AV nodal active agents are con- (0.1 mL) of a standardized purified protein deriva-
traindicated in this circumstance, as inhibition ofthe tive (PPD) from M. tuberculosis. Positivity is deter-
AV node will cause faster conduction through the an- mined by the amount of induration, not erythema,
terograde accessory pathway, which is already at high in response to the injection and is typically mea-
risk for degeneration into an unstable rhythm. The sured between 48 and 72 hours. Induration <5 mm
treatment of choice in stable antidromic or irregular in diameter is negative and induration > 15 mm is
tachycardias in WPW patients is arniodarone or pro- positive. Induration between 5 and 15 mm may be
cainarnide. Unstable patients require cardioversion. positive, depending on other factors, such as prior
immunization with Bacille Calmette-Guerin (BCG)
~ Answer A. Mycobacterium tuberculosis can affect or the presence ofimmunosuppression as in patients
nearly any organ system in the body. The most with HIV. As the BCG vaccine is not used in this
common extrapulmonary manifestation is painless country, and because this patient is American-born,
lymphadenopathy, usually in the cervical region. Al- BCG is an unlikely cause of his positive test result.
though classically described as a disease ofchildhood, In this country, patients with HIV are considered
the peak incidence is between 20 and 40 years of age. to have a positive PPD if the amount of induration
Although the nodes are initially discrete, they may exceeds 5 mm, as are patients who have had close
form a firm, matted, nontender mass over time. Peri- contact with active TB, and patients who have a
carditis may result from direct extension TB from fibrotic chest x-ray. Patients in any other high-risk
mediastinal lymph nodes, or the spine, lungs, or group are considered positive when the amount of
sternum. The onset may be insidious or acute, and induration exceeds 10 mm in diameter. Regardless
may result in a restrictive pericarditis. The CNS is of the group to which a patient belongs, a positive
not spared in extrapulmonary TB, and tuberculous PPD means that a patient has been exposed to and
meningitis is the most common presentation ofneu- infected with M. tuberculosis and that the organism
rologic TB disease. In such cases, the CSF usually remains in their body. Because this patient does not
reveals very high protein levels, ranging from 100 to have symptoms of active disease (e.g., fever, cough,
500 mg per dL, although levels as high as 2 to 6 g hemoptysis, night sweats, anorexia, weight loss) and
per dL have been reported. In contrast, CSF glucose does not have any findings on his chest x-ray, he
concentration usually is <45 mg per dL. Skeletal TB has latent, not active disease. Patients with latent TB
most commonly involves the spine (Pott's disease), infection are not contagious, but they require treat-
with most lesions in the thoracic pine. Vertebral de- ment with INH to dramatically reduce their lifetime
struction usually begins at the anteroinferior portion risk of developing active TB.
of the vertebral body, eventually resulting in an an-
terior wedge defect and a palpable bony prominence ~ Answer B. Alcohol withdrawal occurs as early as
posteriorly (termed Gibbus). M. tuberculosis typically 6 hours after cessation of alcohol consumption and
spreads to the bilateral adrenal glands, causing bilat- generally peaks at 2 to 3 days of abstinence. Signs and
eral adrenal enlargement and subsequent destruction symptoms are similar to a sympathomimetic tox-
which ultimately causes adrenal insufficiency. idrome--hyperthermia, tachycardia, hypertension,
tremor, anxiety, hallucinations, and seizures. Partic-
~ Answer D. Ascites is the most common compli- ularly severe cases are often referred to as delirium
cation of cirrhosis, occurring in roughly 60% of tremens, which may be life threatening. Management
patients with compensated cirrhosis for 10 years. of alcohol withdrawal involves aggressive supportive
Spontaneous bacterial peritonitis is a complication care with fluids, vitamin and electrolyte supple-
of ascites, and occurs in 8% to 25% of patients mentation, and benzodiazepine therapy. Although
with cirrhosis and ascites. Esophageal varices are also no single benzodiazepine is more efficacious than
Test7 213

another, lorazepam is preferred due to its multiple used. The period of highest risk is between 2 and 7
routes of administration and predictable pharma- weeks (organogenesis). Finally, FAST examination is
cokinetics. Haloperidol and droperidol are two drugs poorly sensitive in the second and third trimesters.
ofthe butyrophenone class which are used as adjunc- Therefore, CT scanning should be performed in all
tive therapy to manage agitation, but have no effect in patients in whom abdominal injury is suspected.
preventing seizures. Chlordiazepoxide is a benzodi-
azepine that may be used to treat alcohol withdrawal ~ Answer B. The causes of painless hematuria vary
but has prolonged half-life in patients with renal in- by age and gender. The most common cause in
sufficiency. Diphenhydramine, an antihistamine and children is glomerulonephritis, in young adults and
anticholinergic agent, has no role in the management older women, UTI, and in older men, bladder cancer.
of alcohol withdrawal. Renal carcinoma is a less common cause of painless
hematuria in all age-groups. Nephrotic syndrome
~ Answer C. Sprains refer to ligamentous injuries causes proteinuria without frank hematuria. The
and may be graded according to the severity of in- combination of urinalysis and appropriate imaging
jury-grade I is a mild injury to ligamentous fibers studies (such as helical CT scan) yields the diagnosis
with minimal tenderness and edema, grade II is a in most cases.
moderate injury with partial tearing of the ligament
and moderate tenderness and edema, and grade III ~ Answer D. Patients older than 50 years are at risk
injuries refer to complete disruption of the ligament for S. pneumococcus and N. meningitidis, as well as
causing instability of the joint. Ligamentous (espe- L. monocytogenes. Ceftriaxone covers pneumococ-
cially grade II and III) injuries take several weeks to cus, whereas vancomycin is necessary for resistant
months to heal. Immobilization of the affected joint pneumococcus. However, in older patients at risk
is indicated and motion of other joints is indicated for Listeria infection, ampicillin is also necessary.
to prevent stiffness. Although less comfortable for Patients younger than 50 who are immunocompro-
the patient than heat, cold therapy minimizes the mised or who are alcoholic are also at risk for Listeria
hemorrhage and edema accompanying ligamentous infection and should be covered with ampicillin. All
injuries and probably improves healing and recov- patients with suspected meningitis should be given
ery times. Thermotherapy appears to have no similar 10 mg of dexamethasone IV just before or with
pathophysiologic effect. Acetaminophen and ibupro- the first dose of antibiotics. Adjunctive dexametha-
fen are roughly equivalent in regard to pain control sone reduces mortality and neurologic sequelae. The
in most traumatic conditions. Emergent MRI is al- reduction is most marked in patients with interme-
most never indicated in acute injury, except in cases diate disease severity, defined as those patients with a
of suspected acute cauda equina syndrome (CES). Glasgow Coma Scale rating of8 to 11, as well as in pa-
tients with pneumococcal meningitis. Amphotericin
~ Answer C. Although there is no consensus, a is used for confirmed fungal infections.
cumulative dose of 5 rads has been proposed as
an acceptable threshold for safe fetal exposure. ~ Answer B. Pulmonary contusion refers to direct
Intrauterine exposure of 10 rads is associated parenchymal injury from blunt thoracic trauma. It
with a small increase in the number of childhood is often associated with other intrathoracic and chest
cancers, but does not result in an increase in fetal wall injuries, most commonly multiple rib fractures.
malformations, spontaneous abortion, or growth Impaired oxygenation and ventilation may cause se-
retardation. Maternal plain films of the head, cervical vere deficits in respiratory function requiring en-
spine, thoracic spine, chest, and extremities each dotracheal intubation and mechanical ventilation.
expose the fetus to <5 mrad (i.e., 1,000 times less The diagnosis is made by initial chest radiography,
than the safe threshold). Plain films of the lumbar although CT scan is more sensitive and able to de-
spine, hip, and pelvis expose the fetus to higher doses, tect other thoracic injuries as well. Atelectasis and
but none of these studies comes close to approaching pneumonia are the most common complications of
5 rads. CT scanning of the abdomen, however, pulmonary contusion, but may take several days af-
results in approximately 2.6 rads offetal exposure. In ter the injury to develop. Prophylactic antibiotics
comparison to these films, the cumulative radiation before the onset of clinical signs and symptoms of
exposure to fetus over a 9-month gestation is between postcontusion pneumonia are not recommended.
50 and 100 mrad (fur more than a single chest ARDS may develop in patients with particularly se-
x-ray). Lead shielding of the maternal abdomen can vere pulmonary contusions but is less common than
reduce fetal exposure by 50% to 75%. The risk to concomitant pneumonia. Pulmonary embolism may
the fetus during the first 2 weeks of pregnancy is also occur in patients who are bedridden after severe
so low that normal radiographic procedure can be traumatic injuries but do not occur at significantly
214 1000 Questions to Help You Pass tM Emergency Medicine Boards

higher rates in patients with pulmonary contusion. ~ Answer D. The patient has evidence of chancroid,
Myocardial contusion and pericardial tamponade caused by H. ducreyi, a gram-negative bacillus. A
may occur from the same blunt traumatic forces that painful chancre-like lesion combined with a solitary
caused the pulmonary contusion but do not gener- tender unilateral lymph node which may also ulcerate
ally occur as a complication of isolated pulmonary is classic. Chancroid, unlike syphilis (caused by
contusion. T. pallidum), is painful and tender. Treatment
of chancroid is with azithromycin or ceftriaxone.
~ Answer C. The patient has a large central corneal Herpes simplex virus can cause ulcerated or vesicular
abrasion as demonstrated by the irregular patch of lesions, but these are usually grouped and Gram
fluorescein uptake from 3 o'clock to 6 o'clock. Treat- stain of the lesions will be negative. C. trachomatis, a
ment of corneal abrasions involves pain control with spirochete, may cause lymphogranuloma venereum,
topical or oral analgesics, short-acting cycloplegics, which is manifested by a painless ulcer combined
and topical antibiotics to prevent secondary infec- with significant lymphadenopathy with a negative
tion. Topical antivirals would be indicated in patients Gram stain. S. epidermidis may cause skin lesions
with herpes simplex or zoster keratitis, which are in the genital region but Gram stain would show
signaled by the presence of dendrites or pseudo- gram -positive cocci.
dendrites, respectively. Topical steroids should never
be given by the emergency physician (EP) without [!!) Answer A. Patients with testicular torsion will
ophthalmologic consultation beforehand, as conse- almost never exhibit a normal cremasteric reflex
quences may be devastating in patients with herpetic on the affected side. The sensitivity of this sign
keratitis. Acetazolamide is used to increase aqueous is extremely high (>95%). Prehn's sign refers to
humor excretion as part of noninvasive temporiz- relief of scrotal pain on elevation of the scrotum.
ing therapies for acute glaucoma attacks. Emergent Prehn's sign was previously thought to distinguish
ophthalmologic consultation is not indicated in pa- epididymitis from testicular torsion, but has been
tients with corneal abrasions, even in patients with found to be inaccurate in this regard. Although
abrasions associated with contact lenses. If there is urinalysis is usually normal, up to one third of
suspicion of corneal ulcer (which would appear as patients with torsion may have detectable urinary
a yellowish spot on the cornea), then consultation leukocytes. Fever is present in 20% and vomiting in
should be obtained emergently. (Figure courtesy of 30% of cases, so absence of either should not be used
Anthony Morroco, MD. Reprinted with permission to rule out the diagnosis.
from Morroco A. Greenberg's text-atlas of emergency
medicine. Lippincott Williams & Wilkins; 2004:135.) ~ Answer B. As with adults, neonatal hypoglycemia
is treated with dextrose. Traditionally, D25 and D50
~ Answer A. Sarcoidosis is a multisystem disease of have been discouraged because of rebound hypo-
unknown cause characterized by the presence of glycemia in hyperinsulinemic infants. In addition,
noncaseating granulomas in multiple tissues. The D25 and D50 are thought to put the infant at risk
incidence peaks in the 20s to 30s, with women for a potentially dangerous rise in plasma osmolarity.
slightly more affected than men, whereas it is three Glucagon is useful as a second-line agent if the infant
times more common in blacks than in whites in the is refractory to dextrose. Octreotide and diazoxide
United States. Systemic symptoms such as fatigue, are useful agents in infants with hyperinsulinemia
fever, and anorexia are the most common presenting but are not usually necessary in the ED.
symptoms. The lungs are the most common organ
system affected and patients may present with ~ Answer C. Neurologic symptoms of paresthesias
symptoms of cough or dyspnea. Patients may also combined with dorsal column findings in the face
present with vague chest discomfort that is atypical of macrocytic anemia are indicative of vitamin B12
in nature. Skin lesions such as erythema nodosum deficiency. Ataxia, depression, and paranoia may also
or lupus pernio (a violaceous, purpuric rash of the accompany these symptoms. Pain and temperature
nose, cheeks, and ears that is more common in are usually spared, as these sensations are not carried
blacks) affect 25% of patients. Sarcoidosis produces by the dorsal spinal columns. Causes of vitamin
symmetric arthralgias or arthritis, and usually affects B12 deficiency include chronic malabsorption, strict
the larger joints such as the wrists, elbows, knees, vegetarian diet, chronic alcohol use, ileal disease,
and ankles. Two thirds of patients have spontaneous and pernicious anemia. Chronic smoking can cause
resolution of symptoms within 2 years, whereas 10% COPD, which may lead to polycythemia rather
to 30% ofpatients have persistent symptoms beyond than anemia. Chronic aspirin use and melena can
this point. Mortality from the disease is low. predispose patients to iron-deficiency anemia, which
Test7 215

is either microcytic or normocytic. Family history of this is due to the commonly associated serious
thalassemia may be a risk factor for younger patients thoracic injury.
to develop a well-compensated microcytic anemia,
but has little relevance in an older adult. ~ Answer A. High-altitude cerebellar ataxia (HACE)
is the most severe form of high-altitude illness and
~ Answer C. Epidural hematoma is usually due to typically follows acute mountain sickness (AMS).
a traumatic injury to the temporal area, causing Therefore, symptoms of AMS (headache, fatigue,
damage to the middle meningeal artery. This leads nausea, dizziness, anorexia, and difficulty sleeping)
to blood accumulating between the skull and the precede the development of HACE. However, AMS
dura, causing a biconvex, lens-shaped hematoma may progress to HACE in as a little as 12 hours
readily seen on noncontrast CT scan of the brain. although 1 to 3 days is more typical. Most cases
Signs and symptoms from an arterial source of of severe HACE occur above 12,000 ft. In addition
the epidural hematoma occur immediately, but a to symptoms of AMS, the rough and dyspnea of
venous cause may produce a delayed presentation. high-altitude pulmonary edema (RAPE) are also
Mortality from epidural hematoma is rare with early typically present. Specific signs for HACE include
diagnosis and surgical treatment, and even most generalized seizures, slurred speech, rare neurologic
patients who present in coma do not die of their deficits, delirium, and ataxia.
head injury. The '1ucid interval," described as a
period where patients awaken from their altered ~ Answer A. Nitroprusside reacts with acetoacetate
state and have normal mental status before suddenly at concentrations of 5 to 10 mg per dL to produce a
becoming comatose again, is neither sensitive nor purple color. It does not react appreciably with other
specific for the diagnosis of epidural hematoma. ketones.
Almost all patients are symptomatic, with headache,
vomiting and altered mental status as the most ~ Answer E. The most sensitive CT technique ap-
common manifestations. pears to be thin-cut helical CT with rectal contrast.
As many patients will refuse such an examination,
[!g) Answer D. Emergencythorarotomyismostuseful an alternative study is CT of the abdomen and pelvis
and successful in patients with penetrating chest after oral contrast administration. However, a delay
wounds who develop traumatic cardiac arrest during of 60 to 90 minutes is required to allow the contrast
or just before the ED resuscitation begins. Patients to reach the distal small bowel and cecum. cr scan-
with pericardia) tamponade due to stab wounds have ning is less sensitive in thin patients and in children
the highest survival rates after ED thoracotomy. In because periappendiceal fat stranding is harder to de-
the past, thoracotomies were performed routinely tect due to the dearth of fat. CT scanning in patients
in the ED for traumatic arrest patients. However, with only a few hours of symptoms is less sensitive
concerns about risk to hospital personnel, cost, and than patients who have more prolonged symptoms.
low success rates of the procedure have restricted In patients with early signs and symptoms of appen-
indications for thoracotomy to penetrating chest dicitis, changes occurring in the appendix may not
wounds. Any patient with blunt trauma (even to the yet be visible by CT. Therefore, normal scans in such
chest) is not a candidate for ED thoracotomy. patients may provide a false sense of comfort, and
such patients should be carefully instructed that they
[!!) Answer E. The femoral and humeral heads are at may still get worse. Finally, ultrasonography is a use-
significant risk for avascular necrosis after fracture ful study for the diagnosis of appendicitis, especially
due to interrupted blood supply. Intra-articular and in children and pregnant women. However, a nega-
four-part fractures ofthe humerus pose an especially tive study does not conclusively rule out appendicitis,
high risk. Elbow injury may occur in association with especially ifa normal appendix is not visualized. Fur-
humeral head fracture, but does not significantly thermore, ultrasonography is less useful in cases of
increase morbidity. Adhesive capsulitis is managed appendiceal perforation. Therefore, further diagnos-
conservatively with range-of-motion exercises and tic study with cr should be performed.
generally responds well. Pneumothorax is rare in
patients with humeral head fractures except when [!ID Answer C. The causes of painless hematuria vary
they are associated with rib or scapular fractures. by age and gender. The most common cause in
Although scapular fracture may be associated with children is glomerulonephritis, in young adults and
humeral head fractures, the vast majority ofhumeral older women, UTI, and in older men, bladder cancer.
head fractures do not involve scapular fractures. Renal carcinoma is a less common cause of painless
Furthermore, scapular fracture by itself does not hematuria in all age-groups. Nephrotic syndrome
contribute significantly to morbidity or mortality; causes proteinuria without frank hematuria. The
216 1000 Questions to Help You Pass tM Emergency Medicine Boards

combination of urinalysis and appropriate imaging and is much more technically difficult in the infant.
studies (such as helical cr scan) yields the diagnosis Discharging the patient home without Cf scan or
in most cases. serial neurologic examinations is not recommended.

~ Answer A. Scapular fractures occur in patients ~ Answer E. The presence ofa patent foramen ovale
with high-force blunt trauma such as motor vehicle is probably the most significant risk factor for
crashes or falls from heights. Because of its well- decompression sickness (DCS). In general, the risk
protected position, isolated fracture of the scapula ofDCS increases with the length and depth of a dive
is rare, and associated injuries to the chest and (e.g., violating the no-decompression limits). Other
upper extremity are present in the large majority risk factors include increasing age, obesity (nitrogen
of cases. Operative repair of the scapular fracture is lipid soluble), dehydration, fatigue, diving at high
is not usually necessary as most patients heal altitude, flying soon after diving, performing heavy
with conservative management and range-of-motion work while diving, exercise after diving, cold water
exercises. Patients with scapular fractures hold their temperature, and rough seas.
ipsilateral arm in full adduction and any movement
elicits extreme pain. Bilateral scapular fracture is ~ Answer C. Sternal fractures are usually caused by
uncommon and usually signifies life-threatening blunt thoracic trauma and passengers wearing seat
associated injuries. belts are at much higher risk than those who are
unrestrained. The belt's position across the chest
[!1) Answer D. The EKG demonstrates deeply inverted is thought to put extreme force on the sternum
T waves in the anterior leads, which is specific during rapid deceleration. Coexisting mediastinal
for the LAD distribution. When associated with hematoma or myocardial contusion may occur in
unstable angina, these T waves indicate stenosis of < 10% of cases, but aortic injury does not occur at
the proximal LAD, called the Wellen's syndrome. appreciably higher rates. Mortality of isolated sternal
Electrocardiographic findings of pericarditis include fractures is< I%. Routine anteroposterior (AP) views
concave ST elevation, sinus tachycardia, and PR of the chest often miss sternal fractures, but tender-
depression. Sinus tachycardia and right axis deviation ness of the sternum should prompt either a lateral
are common EKG findings in patients with a radiograph or cr scan, both of which demonstrate
pulmonary artery embolism. Lateral (I, L, V5, V6) sternal fracture. Concomitant rib fractures are the
and posterior (VI, V2) abnormalities are often seen most common bony injuries associated with sternal
in left circumflex lesions. Right-sided coronary artery fractures, but vertebral fractures do occur at higher
disease (CAD) may cause an inferior MI (II, III, aVF) rates as well. Treatment involves evaluation for other
or posterior MI (VI, V2). thoracic injuries and aggressive pain control.

(!jJ Answer C. Head trauma accounts for the large [j!) Answer D. This patient has impetigo, which is ini-
majority of all pediatric traumatic deaths. Children's tially a superficial vesicular eruption that later de-
heads are proportionally larger and heavier relative velops into multiple honey-crusted lesions. In the
to the rest of their bodies than adults and are more United States, S. aureus is the most common cause,
likely to be seriously injured. Pediatric head trauma whereas group A Streptococcus is responsible for the
patients younger than I year with abnormal neu- bulk of the remainder. In cases caused by Streptococ-
rologic examination, loss of consciousness (LOC), cus, antibiotic therapy does not reduce the incidence
protracted vomiting, abnormal feeding, or abnormal of poststreptococcal glomerulonephritis. Further-
irritability or sleepiness should all be indications for more, topical therapy with mupirocin is as effective
CT scan to evaluate for intracranial injury. Con- as systemic therapy, although systemic therapy is rec-
troversy exists as to whether brief LOC should be ommended when a large area is involved or when
considered as a sole indication for CT scan in the the involvement is near the mouth (allowing topi-
older child, but in the infant it is an accepted indica- cal antibiotics to be licked away). The lesions are not
tion given the difficulty of neurologic examination. painful even though they may be pruritic. However,
The EP should always consider contacting DCFS in regional lymphadenopathy is a common associated
cases of suspected child abuse, but a dear, consistent finding. The lesions are highly contagious and eas-
history with a normal physical examination is less ily transmitted to other children. (Figure reprinted
likely to have high potential for abuse. Skull x-rays with permission from Pilliterri A. Maternal and child
are almost never indicated in head trauma as they health nursing. Lippincott Williams & Wilkins; 2006.)
do not often change management and require fur-
ther testing with CT scan anyway if abnormal. MRI ~ Answer A. Tularemia is a tick-borne illness caused
offers little benefit over CT scan in the acute setting by the spirochete, F. tularensis. Rabbits, rodents,
Test7 217

and cats are the common vectors. Human-to-human nontoxic and cause only local conjunctival irritation.
transmission rarely occurs. The most common man- Cigarette ashes may cause superficial corneal burns,
ifestation of disease is cutaneous, with formation of but the duration ofexposure is rarely long enough to
skin ulcers and painful lymphadenopathy. Systemic, produce serious injury.
pharyngeal, pulmonary, and ocular disease may also
occur. Diagnosis is made by clinical suspicion and ~ Answer A. Arterial gas embolism (AGE) is the sec-
confirmed with antibody titers. Treatment is with ond most common cause of diving-related death, and
intramuscular streptomycin, although tetracyclines, the most severe form of pulmonary barotraumas. As
aminoglycosides, and chloramphenicol may be used a diver ascends, alveoli expand due to the decreasing
with lower efficacy. Pulmonary tularemia may be atmospheric pressure. If the diver does not contin-
caused by aerosolization from a bioterrorist attack, uously expire during ascent, the alveoli will expand
due to the organism's extremely infectious properties. and may rupture. Air may then cross the ruptured
alveolar-capillary membrane, enter the pulmonary
~ Answer D. Chest x-rays are commonly ordered venous circulation and subsequently embolize to any
but rarely useful in the evaluation of patients organ system. Embolization to the coronary or cere-
with acute asthma. They are indicated, however, bral (usually anterior or middle cerebral) arteries is
when there is clinical evidence of pneumonia, a the most catastrophic. The most common presenta-
complication of asthma, such as pneumothorax tion of AGE is neurologic, consisting of decreased
or pneumomediastinum, or a pulmonary disorder consciousness, dizziness, confusion, headache, cra-
that mimics asthma (e.g., a first-time adult wheezer nial nerve symptoms, hemiplegia, and hemisensory
who fails to respond to typical asthma therapy). loss. Any diver who surfaces unconscious or who loses
Approximately 15% ofpatients who fail to respond to consciousness within 10 minutes of surfacing should
optimal therapy will have an unsuspected pulmonary be presumed to have AGE until proved otherwise.
complication evident on x-ray. Fewer than 10% The loss of consciousness (LOC) is a sharp contrast
of patients with asthma exacerbations will have from decompression sickness (DCS), in which LOC
some kind of infiltrate on chest x-ray. Most of 1s rare.
these abnormalities are slight increases in interstitial
markings of unknown significance. [ill Answer C. The patient likely has uncomplicated
viral conjunctivitis, which is most commonly caused
~ Answer D. This patient has a posterior hip dislo- by adenovirus, given the lack of dendrites (HSV),
cation. Posterior hip dislocations account for 90% pseudodendrites or facial rash (varicella-zoster virus
of all hip dislocations. The most common mecha- [VZV] ), or purulence on physical examination (bac-
nism of injury is a "dashboard injury," in which a terial). Viral conjunctivitis is extremely contagious
seated patient strikes the dashboard with a flexed and a sick contact is usually identified. Bacterial con-
knee, driving the femur posteriorly. Due to the force junctivitis may be difficult to distinguish early in the
required to dislocate the well-protected hip joint, course from viral, but is much less common. Contact
posterior hip dislocations are often associated with lens use is the major risk factor for pseudomonal
multisystem trauma. Patients will present with a conjunctivitis.
shortened leg, with the hip internally rotated, ad-
ducted, and slightly flexed. Posterior hip dislocations [!!) Answer D. Ciguatera fish poisoning is the most
must be reduced emergently due to the high risk common cause of fish-related food poisoning in the
of avascular necrosis of the femoral head. Radio- United States. The classic syndrome includes the
graphs should be obtained before reduction, unless development of both GI and neurologic manifes-
a pulse deficit is present. (Figure courtesy of Robert tations. The relative amount of neurologic or GI
Hendrickson, MD. Reprinted with permission from symptoms varies by region. GI symptoms occur first
Hendrickson R. Greenberg's text-atlas of emergency and include nausea, vomiting, watery diarrhea, and
medicine. Lippincott Williams & Wilkins; 2004:515.) crampy abdominal pain. Neurologic manifestations
are more variable but >90% of patients have distal
~ Answer B. Alkali causes liquefaction necrosis, and perioral paresthesias, cold allodynia, and numb-
which allows the alkali to spread to deeper tissues ness. Cold allodynia is often incorrectly referred to
and cause further necrosis. Acid causes coagulation as temperature reversal due to the burning sensation
necrosis, which hardens the tissue and limits spread patients experience when coming into contact with
of the acid to deeper tissues. Chronic exposure to cool or cold objects (although it actually represents
ultraviolet light causes long-term ocular injury and a painful, burning dysesthesia). Symptoms last an
eventual blindness but acute symptoms are usually average of 1 to 2 weeks. There is no effective antidote;
mild. Most commercially available soaps are generally therefore treatment is supportive. Rare cases may be
218 1000 Questions to Help You Pass tM Emergency Medicine Boards

associated with bradycardia and hypotension due to for brown recluse spider bites, including early surgical
anticholinesterase activity. Such cases may require excision, dapsone, electric shock, steroids, hyperbaric
atropine and dopamine. oxygen, colchicine, antihistamines, anticoagulants,
or prophylactic antibiotics. Despite this, many of
~ Answer A. The patient has chronic peripheral these therapies are still used. Dapsone, in particular,
vascular disease leading to acute in situ arterial has been advocated as a means of limiting the
thrombosis in the left leg. Ankle-brachial index is toxic effects of the venom. It is not used as
>0.9 in patients without peripheral vascular disease, an antibiotic. Most bites result in burning pain,
and bilateral reduction in the ABI indicates chronic in mild erythema, pruritus, and minimal swelling.
situ disease. Arterial embolism is also a very common Occasionally, hemorrhagic vesicles develop along
cause of acute arterial occlusion, but the history of with central necrosis in the days following the bite.
progressivelyworsening claudication before the acute Even in the case ofthese more severe bites, supportive
event is more often seen with in situ thrombosis. therapy is all that is required.
Isolated inflammation, vasospasm, and fistulas may
also cause arterial occlusion but are far less common [!ID Answer B. Two percent to 4% of patients with
than thrombosis and embolism. trigeminal neuralgia have MS. This patient's history
of recurrent attacks of neurologic dysfunction in
~ Answer B. Oropharyngeal dysphagia is due to different regions suggests a diagnosis of MS. In
difficulty in the initiation of swallowing. Approx- addition, primary trigeminal neuralgia peaks in the
imately 80% of cases are due to neuromuscular sixth to seventh decade with an average onset of
diseases, strokes. (CVAs) being chief among them. age 50 and >90% of patients who present are older
Polymyositis and dermatomyositis are the second than 40. Therefore, physicians need to have a higher
most common causes. Two thirds of patients with suspicion for secondary causes in young patients with
myasthenia gravis (MG) have dysphagia, but because trigeminal neuralgia. The peak age at onset of MS
MG is a rare disease, they do not account for a large is 25 to 30 years although women have a slightly
number of patients with oropharyngeal dysphagia. younger age at onset than men. In addition, the
Liquids usually result in more dysphagia than solids, incidence in women exceeds that in men by almost
particularly when the liquids are extremely hot or 2: 1. Roughly 5% of cases are diagnosed before the
cold. Symptoms tend to be intermittent rather than age of 20(early onset) and 10% diagnosed after the
progressive. age of 50 (late onset). Other than MS, posterior fossa
tumors, and vascular or aneurysmal compression of
~ Answer D. Adverse drug reactions (ADRs) are the trigeminal nerve should be considered.
serious events that may have fatal consequences.
The incidence of ADRs among hospitalized patients ~ Answer B. The patient has frank blood at the
is estimated at 15.1 %, with half of these events urethral meatus, which is a sign of likely urethral
characterized as "serious,'' whereas the incidence injury. This precludes placement of a Foley catheter
of fatal ADRs is estimated at 0.32% resulting in by anyone but a urologist. Initially, a retrograde
> 100,000 deaths per year. However, true allergic urethrogram is performed to evaluate for urethral
drug reactions (i.e., through type I IgE-mediated injury. Ifthis is normal, a Foley catheter may be placed
hypersensitivity) represent only 6% to 10% of and retrograde cystogram performed to evaluate for
ADRs (some reports quote as high as 25%). bladder rupture. Urinalysis in a trauma patient with
Although 10% of patients claim to be penicillin- grossly bloody urine is not helpful. Aspiration of the
allergic, 90% of those patients are subsequently corpora cavemosa is indicated only in patients with
found through skin testing not to have an allergy. priapism. (Figure copyright James R. Roberts, MD.)
Among those mentioned earlier, urticaria is the most
likely manifestation of a true allergic response. The ~ Answer C. Autopsy studies reveal that emergency
remaining symptoms are most likely expected side personnel, including emergency medical services
effects of drug therapy. However, it is very difficulty (EMS) personnel and physicians, frequently mis-
to rely on the patient's history as nearly a third of diagnose foreign body aspiration. Although most
patients with proven penicillin allergy by skin testing episodes of foreign body aspiration in elderly pa-
have vague histories. tients occur in the presence of bystanders, sudden
coughing attacks followed by severe respiratory dis-
~ Answer A. As with other spider or insect bites, a tress is managed with cardiopulmonary resuscitation
diagnosis of a brown recluse spider bite cannot be (CPR) instead of efforts to remove foreign bodies.
reliably made without the spider. Furthermore, there Cardiopulmonary arrest due to asphyxia from for-
remains no evidence for any of a variety of therapies eign body aspiration was originally called the cafo
Test7 219

coronary syndrome. This title reinforces the notion and the infection usually involves multiple spinal
that most bystanders interpret spastic coughing as a segments. Epidural hematoma is another extremely
brief precursor to cardiac arrest rather than a sign of rare entity that may complicate lumbar puncture in
foreign body aspiration. Paradoxically, many studies patients with a coagulopathy. Myelitis, also known
reveal that the rate of foreign body aspiration in- as transverse myelitis, is a rare inflammatory disorder
creases in the setting of a semisolid diet. Although that is intrinsic to the spinal cord although it may be
these data may not be definitive, it is clear that for- mimicked by epidural compression. Its exact cause
eign body aspiration and asphyxia occur despite such is not known although it is thought to follow a viral
"dysphagia" diets. Most foreign body aspirations in syndrome in roughly 30% of patients.
elderly patients occur at the supraglottic level. Such
aspirations may therefore be theoretically visible and ~ Answer C. The patient has nearly complete opaci-
retrievable with fingers or Magill forceps. As with fication of the left lung. In trauma patients who
other adults, the best initial approach in the setting are usually supine, this indicates a large hemoth-
of aspiration is to perform the Heimlich maneuver. orax, as blood will layer through gravity throughout
Back blows in adults actually directs the food more the entire lung field. Treatment involves tube thora-
deeply into the lungs rather than toward the mouth. costomy drainage and possible thoracotomy if the
Poor dentition is a common risk factor for foreign exsanguination is rapid or severe. Pneumothorax
body aspiration. Other risk factors include decreased may be concomitant but is not evident on this chest
mental status, neurologic disease, older age, and the x-ray. Pericardia! tamponade is best diagnosed by
use of sedative medication. FAST scan. Pneumoperitoneum is not evident-air
under the left diaphragm is much more likely to be
[!!) Answer A. The image demonstrates hemoperi- a physiologic gastric bubble. Diaphragmatic rupture
toneum with blood in Morison's pouch (right up- would be indicated by abdominal organ herniation
per quadrant). In an unstable patient, this is an into the thorax which is not evident. (Figure cour-
indication for immediate laparotomy. In unmis- tesy of Mark Silverberg, MD. Reprinted with per-
takably stable patients, a cr scan of the abdomen mission from Silverberg M. Greenberg's text-atlas of
should be performed to better delineate the injury emergency medicine. Lippincott Williams & Wilkins;
and the potential need for surgery. The sensitiv- 2004:637.)
ity of FAST for the detection of 100 to 500 mL
of blood is as high as 95%. Therefore, while at ~ Answer E. PosteriorwallMisoccurwithinfarction
least 500 mL of blood is present, it is not pos- of the right coronary artery or left circumflex
sible to state unequivocally that 2.5 L is present. artery, depending on the anatomic dominance of
Furthermore, 2.5 L of blood loss would place the the patient. Standard EKG electrodes are anteriorly
average adult male in class IV hemorrhagic shock placed and may not exhibit infarction patterns of ST
which typically presents with a systolic blood pres- elevation-posterior electrodes are more accurate at
sure <70 mm Hg and a heart rate > 14Q. After diagnosing posterior wall Mls. Infarction patterns in
confirming the presence of hemoperitoneum by posterior wall Mls are opposite of those in anterior
ultrasonography, CT scanning should only be per- wall Mls-posterior Mls will show ST depression
formed in undoubtedly stable patients if a clear rea- instead of ST elevation, R waves instead of Q waves,
son for laparotomy does not already exist. (Figure and upright T waves instead of T -wave inversions.
reprinted with permission from Harris }H. The ra- Leads Vl and V2 are the most specific anterior
diology of emergency medicine, 4th ed. Lippincott leads for posterior wall infarction. Tall R waves, like
Williams & Wilkins; 1999:694.) Q waves, are the latest findings in posterior Mls.

~ Answer D. Roughly 85% to 90% of cases are [tool Answer C. Munchausen's syndrome by proxy
due to metastatic disease. Breast, prostate, and refers to the intentional faking or production of
lung cancer each account for 15% to 20% of illness of a child by a caregiver. Virtually any type
cases. The most common site of metastases is the of illness is simulated by the caregiver in order
thoracic spine (60%), followed by the lumbosacral to attain medical attention evaluation. The vast
spine (30%). Although central disk herniation may majority of caregivers who perpetrate Munchausen's
result in cauda equina syndrome (CES), only 2% syndrome by proxy are biologic mothers, usually with
of patients with central lumbar disk herniation a history of mood disorder or prior abuse. Fathers
typically develop CES. Epidural abscess is a rare and nonparent guardians are much less likely to be
entity and most commonly occurs in intravenous involved. The victims are usually young children
drug users. S. aureus is the typical causal organism without gender predominance.
Test 8
Questions
(!) Which of the following is true regarding temporo- (!] Which of the following is the drug of choice for the
mandibular joint (TMJ) syndrome? prevention of future cluster headache attacks?
(A) It is an extremely rare cause offac:ial pain. (A) Lithium
(B) Young women are at highest risk. (B) Verapamil
(C) Pain is normally bilateral. (C) Valproate
(D) Muscle relaxants are not helpful in (D) Topiramate
management. (E) Sumatriptan
(E) Avoidance ofhard foods is rarely necessary.
(!] What is the most common cause ofdeath in patients
(!) A patient presents with the electrocardiogram (EKG) with hemophilia A?
shown in Figure 8-1. Which of the following is the (A) Septic shock
most likely pathophysiologic mechanism? (B) Myocardial infarction (MI)
(A) Reentry (C) Gastrointestinal bleeding
(B) Increased automaticity (D) Intracranial hemorrhage
(C) Atrioventricular (AV) blockade (E) Congestive heart failure
(D) Preexcitation
(E) Infarction

~~~
II aVL

~~
Ill aVF V3

Figure 8-1.

220
Test8 221

[!] Which of the following results from a lesion within [!] A 34-year-old man with schizophrenia is brought to
the pons? the ED by his family because he ''keeps ignoring"
(A) Ipsilateral oculomotor nerve palsy with them. They report that for the last few hours, when
contralateral hemiparesis. they argue with him about taking his risperidone,
(B) Upward gaze paralysis with torsional he stares off into space and does not acknowledge
nystagmus. them in any way. This lasts for a few minutes
(C) Ipsilateral facial droop with contralateral and gradually improves until the next conversation.
hemiparesis. They want to speak with the psychiatrist about
(D) Ipsilateral lateral gaze palsy with ipsilateral putting him on a new antipsychotic medication.
hemiparesis. His vital signs are normal and physical examination
(E) Deviation of the tongue away from the lesion is unremarkable except that when you question him
and contralateral hemiparesis. about his medication, he becomes visibly angry and
his eyes look up to the ceiling. After he calms down
00 Patients with botulism most classically present with in a few minutes, he resumes normal eye contact
which of the following? and conversation. Which of the following is the most
appropriate action at this time?
(A) Ascending symmetric anesthesia.
(B) Nausea, vomiting, and lower extremity (A) Discharge home with outpatient psychiatry
weakness within 1 to 2 hours of toxin exposure. follow-up
(C) Descending symmetric paralysis. (B) Consult psychiatry for alprazolam prescription
(D) Anticholinergic symptoms of constipation, dry (C) Haloperidol5 mg IM
skin, dry eyes, and urinary retention. (D) Lorazepam 2 rng IM
(E) Altered mental status. (E) Benztropine 1 mg IM

W Which ofthe following is an indication for antibiotic [!g) Which ofthe following laboratory abnormalities may
prophylaxis in patients with prosthetic heart valves? be seen in patients with hyperemesis gravidarum?
(A) Endoscopic retrograde (A) Hyperkalemia
cholangiopancreatography (ERCP) (B) Elevated liver enzymes
(B) Endotracheal intubation (C) Thrombocytopenia
(C) Vaginal delivery (D) Hyperglycemia
(D) Dilation and curettage (E) Elevated erythrocyte sedimentation rate
(E) Local anesthesia
~ A 67-year-old woman with a history of hyperlipi-
[!] The family of a 49-year-old woman with a history demia, coronary artery disease, and hypertension is
of hypertension brings her to the emergency de- brought in by emergency medical services (EMS) af-
partment (ED) with a chief complaint of mumbled, ter developing acute vertigo, ataxia with a tendency
incomprehensible speech that started approximately to veer to the left, nausea, and vomiting. She has had
5 hours ago along with weakness of her right arm, no tinnitus or decreased hearing. On examination,
leg, and face. Computed tomography ( CT) of her you find her to be mildly dysarthric, and unable to
head reveals an area of infarction in her left hemi- stand or walk due to imbalance. Her right arm is
sphere but no evidence of bleeding. Upon returning very clumsy but her left arm appears to be normal.
from CT, the nurse tells you her blood pressure is Her family tells you that she does not normally have
200/105. Which of the following summarizes the best problems with balance but that she had an acute fall
approach to managing this patient's blood pressure? without loss of consciousness approximately 2 weeks
(A) Start a sodium nitroprusside drip and titrate to ago. What do you expect to see on her head CT?
a systolic blood pressure (SBP) of 160 mm Hg. (A) Right parietal lobe infarction
(B) Give the patient her oral antihypertensive (B) Right cerebellar infarction
medications at her usual doses. (C) Lacunar infarcts of the basal ganglia
(C) Administer 5 to 10 mg oflabetalol N every 10 (D) Large left hemispheric infarct
to 20 minutes until the patient's SBP is between (E) Right basal ganglia infarction
140 and 160 mmHg.
(D) Give the patient 60 mg of nimodipine PO ~ Which of the following is true regarding pulsus
because of its dual effects in lowering blood paradoxus (PP)?
pressure and in preventing vasospasm. (A) Tension pneumothorax is the most common
(E) Continue to monitor the patient's blood extracardiac cause of PP.
pressure without treatment.
222 1000 ~ tDHdp YouPo~W the~ MetlidMBoGrds

(B) ThepresenceofaPPofupto 15 mmHgis (D) Due to hypothyroidism.


considered nonnal. (E) Idiopathic.
(C) The •-paradox., refers to the fact that heart
sounds are heard when at the same time no (!!) A 62-year-old woman with a history of known
peripheral pulse is felt. peptic ulcer disease presents with a chief complaint
(D) Patients with severe asthma exacerbations of hematem.esis and black tarry stools. Her vital
rarely have a widened PP. signs include a pulse of 105 and an systolic blood
(E) None of these are true. pressure (SBP) of 115 mm Hg. She has no history of
hypertension. Her initial hemoglobin is 9.6 g per dL.
(!j) Which of the following is true regarding lipase and Which of the following is most likely to be useful in
amylase in the setting of acute pancreatitis? this patient?
(A) Lipase is more specific than amylase. (A) Continuous fiunotidine infusion
(B) Amylase is more sensitive than lipase. (B) Sengstalwl-Blakemore tube placement
(C) Amylase peaks earlier and remains elevated for (C) Ewald tube placement
a longer period than lipase. (D) Continuous odreotide infusion
(D) The degree of elevation of either amylase or (E) Continuous pantoprazole infusion
lipase correlates with disease severity.
(E) The amylase to lipase ratio may be useful in (!!] A 4-week-old male infant is brought in by his parents
determining the etiology of pancreatitis. with progressive, projectile, and nonbilious emesis.
Labs are drawn and an IV is placed and the patient is
(H) Which ofthe following is true about gastric volvulus? sent for an ultrasonograph. Ultrasonography reveals
(A) It is usually associated with a large umbilical a hypertrophic pylorus and the surgeon is consulted
hernia. for pyloric stenosis. What are the labs likely to reveal?
(B) It most commonly results from twisting about (A) A hypochloremic, eukalemic, and metabolic
its short axis from the greater to the lesser acidosis
curvature of the stomach. (B) A compematory respiratory alkalosis
(C) It most commonly occurs in infants younger (C) A hypochloremic, hypoblemic, metabolic
than 1 year. alkalosis
(D) In most cases, a nasogastric tube cannot be (D) A hyperchloremic, hyperkalemic, metabolic
passed into the stomach. acidosis
(E) Gastric infarction and death occur in 80% of (E) A high anion gap metabolic addoais
patients if not rapidly diagnosed and treated.
(iii A 56-year-old man with long-standing hypertension
(!!) Most cases of pseudogout are and mild renal insufficiency presents to the ED with
(A) Due to hyperparathyroidism. vague complaints offatigue and generalized malaise.
(B) Due to hemochromatosis. His EKG is shown in Figure 8-2. Which of the
(C) Due to hypomagnesemia.

Figure 8-2.
Test8 223

following is the next best step in management of this ~ In a normal singleton pregnancy, which of the
patient? following is true?
(A) Nebulized albuterol (A) In cephalic presentations, the smallest possible
(B) Insulin presenting diameter occurs when the fetal head
(C) Calcium chloride is maximally extended.
(D) Sodium polystyrene sulfate (Kayexalate) (B) Only fetuses with a longitudinal lie can be
(E) Furosemide safely delivered vaginally.
(C) Position refers to the presenting fetal part that
[!!) A 35-year-old patient is brought to the ED by his overlies the pelvis.
parents for evaluation ofpsychiatric illness. Which of (D) Station refers to the distance of the fetal
the following is an indication for hospital admission presenting part from the vaginal introitus.
on psychiatric grounds? (E) All of the above.
(A) Past medical history of schizophrenia
(B) Past medical history of major depression ~ Among adults, which of the following is the most
(C) Acute ethanol intoxication common cause of acute diarrheal illness in the United
(D) Homicidal ideation States?
(E) Major depressive episode without suicidal (A) Viruses
ideation (B) Escherichia coli
(C) Campylobacter spp.
~ Which of the following correctly matches the (D) Giardia Iamblia
vasculitic syndrome to its primary clinical manifes- (E) Staphylococcus aureus
tations?
(A) Polyarteritis nodosa (PAN) and peripheral ~ Which of the following is the most effective method
neuropathy and bowel ischemia of eliminating symptoms in acquired immune defi-
(B) Takayasu's arteritis and oral and genital ciency syndrome (AIDS) patients with Cryptosporid-
ulcerations ium diarrhea?
(C) Wegener's granulomatosis and cardiac (A) Loperamide
ischemia (B) Highly active antiretroviral therapy (HAART)
(D) Beh~t's disease and sinusitis, otitis and nasal (C) Azithromycin
congestion (D) Metronidazole
(E) Churg-Strauss syndrome and (E) Octreotide
glomerulonephritis
~ Which of the following is true regarding liver
~ In a human immunodeficiency virus (HIV) pos- abscesses?
itive patient with Pneumocystis carinii pneumonia (A) Pyogenic liver abscesses are more common
(PCP) who has a sulfa allergy precluding the than amebic liver abscesses.
use oftrimethoprim-sulfamethoxazole (TMP-SMX), (B) Most cases of pyogenic liver abscesses occur as
which ofthe following is the best outpatient regimen? a consequence of untreated appendicitis.
(A) Primaquine plus clindamycin (C) Most cases of pyogenic liver abscesses are
(B) Levofloxacin caused by a single organism.
(C) Albendazole plus trimethoprim (D) Biliary disease is uncommon in patients with
(D) Pentamidine pyogenic liver abscesses.
(E) Doxycycline (E) None of the above.

~ Which of the following is true regarding traumatic [g1) A 45-year-old woman with a history of untreated
aortic injury (TAI)? hyperthyroidism presents with acute onset of left
(A) Dyspnea is the most common symptom. foot pain. Physical examination reveals normal vital
(B) Normal chest x-rayrules out the diagnosis. signs, an irregular heart rhythm, clear lung sounds,
(C) CT aortography has better sensitivity than and loss of pulses in the left foot with decreased
transthoracic echocardiogram. capillary refill, and cyanotic, paralyzed toes. Which
(D) Blood pressure control is not useful in the of the following is the most appropriate next step in
preoperative setting. management?
(E) The ascending aorta is the site at highest risk (A) Anticoagulation and emergent embolectomy
for rupture. (B) Anticoagulation and emergent bypass surgery
224 1000 Questions to Help You Pass the Emergency Medicine Boards

(C) Anticoagulation alone (D) Steroids play no role in management.


(D) Lumbar sympathectomy (E) Resolution generally occurs 1 month after the
(E) Hyperbaric oxygen therapy onset of symptoms.

~ Which of the following toxins is most commonly ~ The most common cause of immediate postpartum
associated with seizures? hemorrhage is
(A) Cocaine (A) Retained products of conception.
(B) Alcohol (B) Vaginal or vulvar tears.
(C) Opiates (C) Uterine atony.
(D) Ecstasy (D) Placenta accreta.
(E) Ephedra (E) Uterine rupture.

~ Which of the following is true regarding the physical ~ A 35-year-old man presents with severe head injury
examination of a patient with asthma? after being struck in the head with a baseball bat.
(A) Cyanosis is a common finding. His field vital signs are: HR 135, BP 82/45, RR 20.
(B) Respiratory rate correlates poorly to the Which of the following is the most likely cause of his
severity of an asthma exacerbation. hypotension?
(C) Pulsus paradoxus (PP) is present in 90% of (A) Epidural hematoma
patients with severe asthma. (B) Subdural hematoma
(D) Patients with refractory severe asthma who (C) Subarachnoid hemorrhage (SAH)
progress to status asthmaticus have (D) Cerebral contusion
increasingly audible wheezing. (E) Ertracranial cause
(E) Accessory muscle use is typically present in
mild asthma exacerbations. ~ Which of the following is true regarding urine
analysis?
~ A 47-year-old woman without any past medical (A) Urine dipstick is 99% sensitive for microscopic
history presents with several days of progressively hematuria.
worsening left eye pain, blurry vision, and redness. (B) Red blood cell (RBC) casts indicate interstitial
Visual acuity in the left eye is slightly reduced. cystitis.
The patient notes that exposure of the right eye (C) White blood cell casts indicate renal
to light causes increased pain in her left eye. Her parenchymal inflammation.
left pupil is constricted and minimally reactive to (D) Transitional cells indicate bladder cancer.
light with perilimbic conjunctival injection. There (E) Normal urinary pH is 9-11.
is no discharge. Which of the following is the most
appropriate treatment? ~ Which of the following is true regarding rheumatoid
(A) Topical antibiotics arthritis (RA)?
(B) Hypertonic eye drops (A) The distal interphalangeal (DIP) joints are
(C) N mannitol commonly affected.
(D) Ocular massage (B) Arthritis is commonly polyarticular and
(E) Topical cycloplegic mydriatics asymmetric.
(C) Fifteen percent of patients will have a negative
~ Which ofthe following is the most common cause of rheumatoid factor (RF).
death in patients with sickle cell disease? (D) The disease is equally common in women and
(A) Myocardial Infarction (MI) in men.
(B) Stroke (E) Arthritic involvement of the spine is
(C) Sepsis uncommon in RA.
(D) Aplastic crisis
(E) Splenic sequestration @1) A 24-year-old man came to the ED with dyspnea
and pleuritic chest pain for 10 days, complaining
~ Which of the following is true regarding traumatic that his breathing is getting worse. You are surprised
iridocyclitis? to find a 40% right sided pneumothorax and you
(A) It is generally painless. immediately place a chest tube, connecting it to
(B) Findings include a fixed and dilated pupil. wall suction. Although the patient initially improved,
(C) Treatment involves long-acting cycloplegics.
TmB 225

1 hour later he began coughing vigorously, and he (D) Amiodarone 150 mg IV


appears tachypneic and dyspneic. Assuming he has (E) Adenosine 6 mg IV
not experienced a recurrence or worsening of his
pneumothorax. what is the most likely cause of his ~ An 85-year-old woman presents with a painless mass
problem? in the right side of her neck. She first noticed the
(A) Hemothorax caused by intercostal artery mass while brushing her teeth 3 days ago, but
laceration waited to see if it would go away before seeking
(B) Development of an empyema medical attention. She has had pain in her right
(C) Asthma exacerbation triggered by the ear for the last week. She denies fevers, weight Ioas,
procedure foreign travel, night sweats, or history of smoking.
(D) Reexpansion pulmonary edema (REPE) She also denies dysphagia, odynophagia, stridor, or
(E) Pubn.onary embolism (PE) globus. On examination, the patient has a 4 x 2 em
firm, inunobile, nontender mass just lateral to her
~ Which of the following is the average duration of a right sternocleidomastoid muscle at the level of her
typical generalized tonic-clonic seizure? thyroid cartilage. Her right tympanic membrane is
retracted with a serous effusion. What is the most
(A) 15 seconds
likely diagnosis?
(B) 30 seconds
(C) 1 minute (A) Virallymphadenitis
(D) 2 minutes (B) Bacteriallymphadenitis
(E) 3 minutes (C) Acute otitis media with reactive lymphadenitis
(D) Benign neoplasia
~ A 55-year-old patient presents with palpitations for (E) Malignancy
1 week. The E.KG done in the primary care doctor's
officeisshowninFigure8-3. Thepatientissenttothe ~ Which of the following is associated with a normal
emergency room (ER) for further evaluation. Which anion gap in overdose!
of the following is the most appropriate nat step in (A) Salicylates
management? (B) Methanol
(A) Diltiazem 20 mg IV (C) Isopropanol
(B) Esmolol50 #{-glkg/minute IV (D) Ethylene glycol
(C) Enoxaparin 1 mg per kg SC (E) Isoniazid

n, ,,
n:I=II

Figure 8-3. (See color insert.)


22 6 1000 Questions to Help You Pass the Emergency Medicine Boards

~ Which ofthe following is true regarding the diagnosis (D) Macular degeneration
of aortic dissection? (E) Retinal detachment
(A) Magnetic resonance imaging (MRI) is more
specific than CT or transesophageal
l!l) A 27-year-old nonpregnant woman presents with
pelvic pain. Pelvic ultrasonography reveals a 4-cm
echocardiogram.
right adnexal cystic mass. Which of the following is
(B) Chest x-ray is normal in most cases.
the most likely etiology?
(C) Transthoracic echocardiogram is useful to
confirm the diagnosis. (A) Corpus luteum cyst
(D) Aortography is the best screening test. (B) Dermoid cyst
(E) EKG has excellent specificity. (C) Theca lutein cyst
(D) Follicular cyst
~ Which of the following is true regarding ibuprofen (E) Ovarian fibroma
overdoses?
~ Which of the following statements regarding foreign
(A) Rapid progression to coma and death often
body aspiration is correct?
occurs within 24 hours.
(B) Significant morbidity and mortality is (A) Most aspirated foreign bodies are
prevented by timely administration of a specific radio-opaque.
antidote. (B) Children younger than 12 months more
(C) Hemodialysis is required in approximately half commonly have a delayed presentation.
the cases. (C) Most foreign bodies are aspirated into the
(D) Urinary alkalinization is effective at reducing trachea instead of the more distal airways.
toxicity. (D) Obstructive emphysema is one of the most
(E) Even without treatment, a benign course is common radiologic findings.
characteristic. (E) Persistent cough is the most predictive finding
for foreign body aspiration.
~ Which of the following statements about joint
pathology is true? ~ The most common cause of pruritus ani is
(A) Viscosity of synovial fluid increases in (A) Inadequate anal hygiene.
inflammatory or infectious arthritis. (B) Anal fissure.
(B) Osteoarthritis (OA) classically affects the (C) Hemorrhoids.
metacarpophalangeal (MCP) and proximal (D) Lichen sclerosus.
interphalangeal (PIP) joints. (E) Diabetes mellitus.
(C) A cardiac rub in the setting of arthralgias
suggests systemic lupus erythematosus ~ Which of the following is a possible complication of
(SLE). a peritonsillar abscess?
(D) Sausage-shaped swelling of the fingers or toes (A) Spontaneous rupture and aspiration
suggests pseudogout. (B) Airwayobstruction
(E) An abducted, externally rotated hip in a (C) Mediastinitis
neonate suggests an occult hip fracture. (D) Ludwig's angina
(E) All of the above
~ Which of the following is the most appropriate
medication to induce rapid, measured reduction of ~ A 35-year-old woman without any past medical
blood pressure in hypertensive crises? history presents with a red, painful region on her
(A) Nifedipine right arm where she had a bug bite 3 days before. She
(B) Isoproterenol denies fever. She is allergic to penicillin. Vital signs
(C) Phenoxybenzamine are normal. Physical examination is remarkable for
(D) Nitroprusside a 10 x 6 cm2 area on her right arm that is red,
(E) Hydrochlorothiazide warm, tender, and sharply demarcated. There is no
lymphangitic streaking or axillary lymphadenopathy.
~ Which of the following is the leading cause of legal You diagnose her with cellulitis. Which of the
blindness in the United States? following is the most appropriate choice ofantibiotic?
(A) Cataracts (A) Gentamicin
(B) Glaucoma (B) Doxycycline
(C) Diabetic retinopathy
TmB 227

(C) Didoudmn
(D) Linezolid
(E) Metronidazole

~ A 38-year-old woman who has a history of "infer-


tilitY, presents to the ED with low abdominal pain.
She recently became pregnant after using in vitro fer-
tilization. Her physical examination reveals mild low
abdominal tenderness, no adnexal mass, and a closed
cervical os with a normal bimanual examination.
An ultrasonography demonstrates a live intrauter-
ine gestation appropriately sized fOr elates, but also
reveals a slightly enlarged left ovary. Which of the
fOllowing is the approximate risk of a heterotopic
pregnancy in this patient?
(A) 0.25%
(B) 1%
(C) 10% (D) Westermark's sign (oligemia)
(D) 25% (E) None of the above
(E) 50%
~ A 25-year-old woman presents with the following
~ A 44-year-old alcoholic man presents with nausea injury while playing with darts. Which of the
and vomiting. He reports drinking alcohol for 2 clays following is the most appropriate next step in
straight. Physical aamination is normal except fOr a management (see Pig. 8-4)?
mildly intoxicated patient with atremelydrymucous (A) Measurement ofintraocular pressure
membranes. Urine dip is positive for ketones. The (B) Ri:moval of foreign body
chemistry panel is as given: (C) Slit lamp examination
Na+ 134 mEq per L (D) Emergent MRI
J<+ 4.0 mEq per L (E) Intravenous antibiotics
cl- 92 mEq per L
HC03- 18 mEq per L ~ Which of the following personality disorders is seen
BUN 32 mg per dL in most patients with somatization disorder?
Cr 1.6 mg per dL
(A) Histrionic
Glu 114 mg per dL
(B) Antisocial
Which of the following is the most appropriate
(C) Schizoid
next step in management?
(D) Schizotypal
(A) Bicarbonate (E) Narcissistic
(B) Insulin
(C) Glucose (ii} A 38-year-old woman presents to the ED with a
(D) Potassium complaint of extreme hand and finger pain, which
(E) Magnesium she says is exacerbated any time her fingers are
exposed to the cold. She first noticed the problem
~ The most commonly injured ligament during ankle when.reachingintothefreezertograbafrozendinner.
sprains is She notes that her fingers become ""ghost white" or
(A) Anterior talofibular. blue at the tips and have a painful ache. Which of the
(B) Calcaneofibular. following is the most common underlying disorder
(C) Posterior talofibular. that produces these symptomsf
(D) Anterior inferior tibiofibular. (A) Systemic lupus erythematosus (SLE)
(E) Deltoid. (B) rheumatoid arthritis (RA)
(C) Scleroderma
~ Which of the following x-ray findings is diagnostic (D) Polyarteritis nodosa (PAN)
for pulmonary emboliBm (PE)? (E) ID1lammatory bowel disease
(A) Atelec:taais
(B) Hampton's hump (pleural wedge density) ~ A 55-year-old woman presents with uncontrollable
(C) Elevated hemidiaphragm twist:ing movements ofher face and tongue. She has
228 1000 Questions to Help You Pass tM Emergency Medicine Boards

been on an antipsychotic for 25 years for treatment ~ Which of the following is the most common
of schizophrenia. Which of the following is true mechanism of trauma in the elderly?
regarding this condition?
(A) Falls
(A) Patients with concomitant depression are at (B) Motor vehicle collisions (MVCs)
higher risk. (C) Gunshot wounds
(B) Symptoms usually start within the first week of (D) Stab wounds
treatment. (E) Elder abuse
(C) It is 100% reversible if the medication is
stopped. ~ Which of the following is associated with cystitis in
(D) Young men are the highest risk group. adults?
(E) Treatment with anticholinergics is usually
(A) Hematuria
successful.
(B) Flankpain
(C) Fever
~ A 9-month-old boy is brought by his p~ents ~r
(D) Chills
evaluation ofabdominal mass that they noticed while
(E) Vomiting
changing him. Physical examination demonstrates a
nontoxic, active infant with a palpable, nontender
~ Which of the following is indicated as suppl~me~tal
mass measuring 4 x 6 em several centimeters to the
treatment for patients with ethylene glycol poasomng?
right of and cephalad to the umbilicus. Which of
the following studies is most likely to reveal the (A) Folate and niacin
diagnosis? (B) Thiamine and pyridoxine
(C) Vitamin D and vitamin K
(A) Gall bladder ultrasonography
(D) Cobalamin and vitamin A
(B) Scrotal ultrasonography
(E) Potassium and selenium
(C) Renal ultrasonography
(D) Urinalysis
~ Which of the following is true regarding topical
(E) Meckel's scan
ophthalmic antibiotics?

~ The approximate mortality from subarachnoid hem- (A) Ointments have a shorter duration of action
orrhage (SAH) is which of the following? than drops.
(B) Ointments are absorbed more rapidly than
(A) 5%
drops.
(B) 15%
(C) Systemic side effects may occur.
(C) 30%
(D) Drops are preferred in pediatric patients.
(D) 50%
(E) Antibiotics should not be prescribed for
(E) 75%
conjunctivitis without positive cultures.
~ Which of the following is true regarding herpes
~ Which of the following is a common cause of death
simplex encephalitis?
in patients with acute renal failure?
(A) Bilateral frontal lobe involvement is
(A) Hypercalcemia
characteristic.
(B) Hyperkalemia
(B) HSV-2 is the most common cause in adults.
(C) Hypermagnesemia
(C) Acyclovir reduces mortality.
(D) Hypernatremia
(D) Most patients recover without neurologic
(E) Hyperphosphatemia
sequelae.
(E) The sensitivity of cerebrospinal fluid (CSF)
~ Which of the following associations is true?
culture is 90%.
(A) Mallet finger: Disruption of the flexor
~ Which of the following is used in addition to digitorum profundus (FDP) tendon.
acetaminophen concentration to determine severity (B) Bennett fracture: Extra-articular fracture of the
in overdose? base of the thumb metacarpal.
(C) Trigger finger: Volar plate entrapment.
(A) Lipase
(D) Jersey finger: Primary involvement of the ring
(B) Amylase
finger.
(C) Gamma Glutamyl Transpeptidase (GGT)
(E) Gamekeeper's thumb: Radial collateral
(D) Aspartate aminotransferase (AST)
ligament injury.
(E) Alkaline phosphatase
Test8 229

~ A 19-year-old woman is brought to the ED after examination demonstrates 3/5 strength in both of
an accidental submersion. She was drinking with his lower extremities and tenderness to palpation
friends at a dock when she accidentally fell into of his lower back. An emergent MRI demonstrates
the water. She was submerged in the water for epidural lumbar spinal cord compression secondary
approximately 1 minute before her friends pulled to metastasis. Which of the following is the most
her out. She was gagging and coughing when they appropriate initial consultation for the emergency
pulled her from the water but she did not require physician (EP) to initiate?
cardiopulmonary resuscitation (CPR). Her blood (A) Radiation oncology
alcohol level is 120 mg per dL and her pulse oximetry (B) General surgery
reveals an oxygen saturation of94% on room air. An (C) Urology
initial chest x-ray is normal. Assuming the remainder (D) Neurology
of her examination is normal and she has no signs of (E) Physical medicine and rehabilitation
trauma, the best course of management is
(A) Admission to the hospital for overnight ~ A 22-year-old woman presents with cyanosis. She
observation. complains of shortness of breath, headache, and
(B) Observation for 4 hours, then discharge if she is slurred speech. Her friends report that she was using
asymptomatic with Sao2 >94% on room air. "some medicine," to get high, but cannot recall the
(C) Repeat chest x-ray in 6 hours, then discharge if exact one. Pulse oximetry is 85% despite aggressive
she is asymptomatic with Sao2 >94% on room oxygen therapy. Blood drawn from the patient
air. appears extremely dark. Which of the following is
(D) Administer oral steroids and prescribe a short the most likely drug ingested?
pulse of steroids to limit aspiration (A) Lorazepam
pneumonitis and discharge home without (B) De:xtromethorphan
observation. (C) Amyl nitrate
(E) Administer oral antibiotics and oral steroids, (D) Diphenhydramine
prescribe a course of both medicines, and (E) Ketamine
discharge home without observation.
~ Which of the following is the most common
[!!) Which of the following physiologic changes is complication of mitral stenosis (MS)?
expected in hypothermic patients? (A) Atrial fibrillation
(A) Hemoconcentration (B) Endocarditis
(B) Hypoglycemia (C) Congestive heart failure
(C) Metabolic alkalosis (D) Myocardial infarction (MI)
(D) Oliguria (E) Pneumonia
(E) Seizures
~ Which of the following is true regarding lung
~ Which of the following is true regarding the abscesses?
management and prognosis of trigeminal neuralgia? (A) The most common pathogen is Streptococcus
(A) Remission, with or without treatment, rarely pneumococcus.
occurs. (B) Patients with anaerobic lung abscesses typically
(B) Antiherpes virus medications and present with acute-onset chest pain, cough, and
corticosteroids have been shown to reduce the fever.
duration of pain and prevent recurrence. (C) Lung abscess occurs most commonly in
(C) In addition to medical treatment, all patients patients with poor oral hygiene.
should be referred to a neurologist for further (D) Thoracotomy with abscess excision is the
evaluation by MRI. cornerstone of therapy.
(D) Fifty percent of patients will eventually require (E) Lung abscesses most commonly develop as a
neurosurgical ablation of the trigeminal nerve. complication of pediatric pneumonia.
(E) All patients should be loaded with phenytoin
and prescribed an outpatient regimen. [!1) Hypematremia . ..
(A) Almost never occurs in alert patients with an
(nJ A 65-year-old man presents with acute onset of intact thirst mechanism.
back pain and bilateral leg weakness. He was (B) May occur after administration of single doses
just diagnosed with prostate cancer on a recent of charcoal.
biopsy and has not seen his oncologist yet. Physical
230 1000 Questions to Help You Pass the Emergency Medicine Boards

(C) Is most commonly due to increased body stores is normal. Which of the following is the most
of sodium (or sodium gain). appropriate medication at this time?
(D) Is best treated with normal saline. (A) Penicillin G
(E) May result in central pontine myelinolysis if (B) Doxycycline
not treated within 72 hours. (C) Acyclovir
(D) Clindamycin
ll!J Which ofthe following is the most common cause of (E) Trimethoprim-sulfamethoxazole (TMP-SMX)
hypercalcemia?
(A) Malignancy ~ A 44-year-old cancer patient, with known
(B) Paget's disease chemotherapy-induced neutropenia is sent by her
(C) Hyperparathyroidism oncologist to the ED with afeverof101.5°F. Which of
(D) Adrenal insufficiency the following is the most appropriate initial empiric
(E) Thiazide therapy antibiotic regimen?
(A) Doxycycline
[Z!J Which of the following patients is at increased risk (B) Cefepime
for developing cellulitis? (C) Clindamycin
(A) A 27-year-old intravenous drug abuser who is (D) Metronidazole+ gentamicin
engaging in "skin popping." (E) Metronidazole + aztreonam
(B) A 46-year-old diabetic man.
(C) A 60-year-old woman with a history of ~ You suspect that a lumbar puncture you performed
left-sided mastectomy. to evaluate for subarachnoid hemorrhage (SAH)
(D) A 53-year-old man who is status postcoronary is a "traumatic tap." Which of the following
artery bypass grafting with a saphenous vein cerebrospinal fluid (CSF) findings is most reliable
graft. for differentiating a SAH from a traumatic lumbar
(E) All of the above. puncture?
(A) A relative CSF leukocytosis will be present in
~ Which of the following is true regarding rust rings? cases of SAH.
(A) Removal should be undertaken immediately by (B) The presence of xanthochromia indicates
theEP. aSAH.
(B) They are usually caused by copper-containing (C) A twofold or greater decrease in the number of
foreign bodies. RBCs from tube 1 to tube 4 is always due to a
(C) Topical steroids should be used for treatment. traumatic tap.
(D) MRI is indicated to evaluate for intraocular (D) A positive CSF clotting assay is consistent with
foreign bodies. a traumatic tap.
(E) Ophthalmology follow-up should occur within (E) All of the above are equally useful.
48hours.
~ Which of the following laboratory tests is most likely
!!!] A 75-year-old man with history of atrial fibrillation to be normal in patients with acute disseminated
presents with fatigue, nausea, and halos in his vision. intravascular coagulation (DIC)?
He states that he has been depressed lately and took (A) Prothrombin time (PT)
some pills in an effort to commit suicide. A serum (B) Partial thromboplastin time(PTT)
digoxin level is elevated. Which of the following is (C) Platelet count
an indication to administer digitalis antibody (Fab) (D) Hemoglobin
fragments? (E) Fibrinogen
(A) Atrial fibrillation with ventricular rate of 120
(B) Potassium level of 6 mEq per L [ij) Which of the following joints is most commonly
(C) Digoxin level of 3 ng per mL affected in septic arthritis?
(D) Total digoxin ingestion of3 rng (A) Hip
(E) Magnesium level of 3 mEq per L (B) Knee
(C) Ankle
~ A 19-year-old man presents with fevers, chills, (D) Wrist
malaise, and rash for 5 days. He recently returned (E) Shoulder
from a camping trip. The rash is macular and
located on the wrists, ankles, palms, and soles. ~ A 35 year-old man presents with progressively
Routine laboratory work including lumbar puncture worsening severe right leg pain and swelling after
TmB 231

being kicked while playing soccer 3 days ago.


Physical examination of his leg demonstrates firm,
tender compartment of his right anterior leg with
diminished sensation in the area. Pulses in the foot
are 2+. You suspect comparbnent syndrome, so
you call the orthopedist for consultation. He says
that if pulses are present, it cannot be comparbnent
syndrome, and that you should send the patient
home to follow-up in clinic in 1 week. Which of the
following is the most appropriate course ofaction?
(A) Have the patient follow-up in 1 week.
(B) Have the patient follow-up in 2 days.
(C) Get an MRI of the leg. Figure 8-5. (See color insert.)
(D) Check ankle/brachial index (ABI) and
discharge patient ifABI >0.9.
(E) Insist that the orthopedist come to see the ~ A mother brings in her 7-year-old daughter with a
patient. ehi.ef complaint of a rash, colicky abdominal pain,
and achy ankles. The rash is palpable upon physical
(ii) A 23-year-old man presents with difficulty breathing, examination (see Fig. 8-5). Which of the following is
altered mental status, and a petechial rash. He true?
suffered a leg fracture the previous day from being (A) When present in adults, this tends to be a much
kicked in the leg in a fisht. Which of the following is more severe disease.
the most likely diagnosis? (B) Her prognosis is primarily dependent on the
(A) Meningococcemia presence ofgastrointestinal bleeding.
(B) Fat embolism (C) lntussusceptions complicating this condition
(C) Pulmonarythromboembolism are most commonly ileocecal in location.
(D) Pneumothorax (D) This patient should receive a short course of
(E) Pneumonia
high dose intravenous corticosteroids.
(E) None of the above.
lii} Which of the following populations baa the lowest (!j) Which ofthe following is true regarding renal injury
rate ofotitis media?
in trauma?
(A) Eskimos
(A) In penetrating trauma, absence of hematuria
(B) Caucasians
rules out renal injury.
(C) Natift Americans
(B) In blunt trauma, microscopic hematuria alone
(D) AfricanAmericans
is rarely associated with renal injury.
(E) Natift Alaskans
(C) Most renal injuries require operative repair.
(D) Renal injuries are extremely uncommon in
{!gJ Bites from which ofthe following snakes would most children with blunt trauma.
likely result in respiratory failure and death? (E) Plain radiography is the imaging test ofchoice
(A) Rattlesnake for diagnosis.
(B) Cottonmouth (water moccasin)
(C) Coral snake ~ Clinically significant hypermagnesemia almost al-
(D) Copperhead snake ways occurs in the setting of:
(E) All of the above (A) ReDal insufficiency.
(B) Pancreatitis.
(!!) Which ofthe following is a risk &ctor for developing (C) Trauma.
otitis media? (D) I.amtive abuse.
(A) Breast feeding (E) Alcoholism.
(B) Parental smoking
(C) Environmental allergies ~ A 43-year-old alcoholic woman presents in coma.
(D) Female gender A relative states that the patient overdosed on her
(E) Age older than 5 years alprazolam, which she has been taking for many
232 1000 ~ toHdp Youl'lw tile~ MetliciM.Bomds

years as an am:iolytic. After initial airway manage-


ment and IV hydration, which of the following i.s
the most appropriate nm step in pharmacologic
management?
(A) Fomepizole
(B) Flumazenil
(C) Physostigmine
(D) Glucagon
(B) Thiamine

~ Possible complications of acute asthma exacerbations


include all of the following except:
(A) Pneumothorax.
(B) Subconjunctival hemorrhqe.
(C) Subcutaneous emphysema.
(D) Puhnonary embolus.
(E) Myocardial .infarction (MI).

~ A 35-year-old woman presents after a high-speed


motor vehicle collision (MVC). She was unrestrained
and there waa considerable damase to the vehicle.
She complaina of chest pain and right leg pain.
Paramedics report that the riFt ankle is visibly
deformed. The patient i.s brought in on backboard
and C-collar. Primary survey in the ED i.s intact, in Fig 8-6. Which of the following is the correct type
including present pulses and sensation in the of injury?
deformed right ankle. Vital signs are: T 99.0 F,
p 90, RR 22, BP 144/92, POx 95% RA. Which of (A) Salter-Harris I
the following is the moat appropriate next step in (B) salter-Harris n
(C) Salter-Harris III
management?
(D) Salter-Harris IV
(A) Chestx-ray (E) Salter-Harris V
(B) Lateral cervical spine x-ray
(C) Right ankle x-ray
(100] Which of the following is true~ diabetic~­
(D) CI' scan of the abdomen/pdvia toacidosi.s (DKA) and hype:rosmolar hyperglyceunc
(B) CT scan of the head syndrome or state (lfliS)l
~ Which of the following findings may be present in (A) Fluid depletion is larger in DKA.
hypothyroidism? (B) Seizures are the most common fatal
manifestation in HHS.
(A) Nonpitting periorbital edema (C) All patient& with HHS have potassium
(B) Delayed relantion phase of deep tendon
deficiency.
reflexes (D) Thromboembolic events are more common in
(C) Median nerve neuropathy
DKA.
(D) Hypothermia (E) AU of the above.
(B) All of the above

~ An 8-year-old boy presents with wrist pain~ a fall


on hi.s outstretched right hand. The x-ray lS shown
Answers and Explanations
[I) Answer B. Temporomandibular joint (TMJ) syn- such as prednisone and dexamethasone may be
drome refers to a vague set of disorders involving used for short-term prophylaxis but verapamil is
the TMJ, such as pain, joint locking, and dislo- the drug of choice for long-term prevention of dus-
cation. It is considered the most common cause ter headaches. Steroids are not used because of their
of facial pain after dentalgia. Young women com- poor side effect profile. In contrast, verapamil is
prise the highest risk category, and many patients well tolerated and has a benign side effect profile.
have concomitant psychiatric conditions. The pain Sumatriptan has been shown to be of no benefit
is normally unilateral Evaluation involves imaging in cluster headache prophylaxis, whereas topiramate
to assess for structural abnormalities and laboratory has shown mixed results. Lithium and valproate
tests to check for associated systemic diseases such have demonstrated some benefit in reducing future
as rheumatoid arthritis (RA), degenerative joint dis- attacks, but because of their side effect profiles, they
ease, and ankylosis. Treatment is with nonsteroidal have limited use.
anti-inflammatory drugs (NSAIDs), muscle relax-
ants, and a soft food diet during acute episodes to [!) Answer D. Hemophilia A occurs with a genetic
prevent further exercise ofthe muscles ofmastication. deficiency in factor VIII, predisposing the patient
to hemorrhagic complications. The most common
[!] Answer D. The EKG demonstrates a preexcitation cause of death is from head trauma causing mas-
pattern. Leads V1 through V6 exhibit a gradual sive intracranial hemorrhage. Treatment of bleed-
sloping of the QRS complex (delta wave) combined ing episodes in patients with hemophilia A is with
with a shortened PR interval. The most common type factor VIII. Every patient with hemophilia A should
of preexcitation is Wolff-Parkinson-White (WPW) receive factor VIII after head trauma (whether or
syndrome. Patients have an accessory conductive not there is evidence of intracranial bleeding), as
pathway from the atria to the ventricles which the risk of delayed bleeding is high. Sepsis and
preexcites the ventricular myocytes before the AV MI occur with similar incidence to the general
node releases the normal sinoatrial depolarization. population. Gastrointestinal bleeding does occur in
As a result, patients with WPW have a shortened hemophiliacs but is not as common a cause of mor-
PR interval and a delayed QRS upstroke, called bidity or mortality as intracranial bleeding. Conges-
the delta wave. Patients with WPW syndrome tive heart failure may be secondary to high-output
can have reentrant dysrhythmias, in which the failure caused by anemia, but is usually a more
accessory pathway can either conduct retrograde chronic process and is not primarily responsible
(in which the AV node conducts in the normal for death.
direction, producing a narrow QRS complex and an
"orthodromic" pattern) or anterograde (in which the [!) Answer C. The hallmark of lesions within the
AV node conducts backwards, producing a wide QRS brainstem is the presence of "crossed signs." This
complex and an "antidromic" pattern). A patient refers to the occurrence of ipsilateral brainstem le-
with WPW syndrome, tachycardia, and wide QRS sions with contralateral hemiparesis and hemisensory
complexes suggests the presence of an antidromic loss. The oculomotor nucleus is located in the mid-
conduction pattern (where the accessory pathway brain, whereas the hypoglossal nucleus is located in
conducts anterograde and the AV node conducts the medulla. Therefore, neither of these nerves is
retrograde). Selective AV nodal active agents are affected by pontine lesions. Although the abducens
contraindicated in this circumstance, as inhibition nucleus is located within the pons, a pontine lesion
of the AV node will cause faster conduction through resulting in ipsilateral lateral gaze palsy should be
the anterograde accessory pathway, which is already accompanied by contralateral hemiparesis. Of note,
at high risk for degeneration into an unstable because brainstem lesions affect cranial nerve nuclei
rhythm. The treatment of choice in stable antidromic (or lower motor neurons of cranial nerves), such
or irregular tachycardias in patients with WPW lesions result in a complete deficit in the distribu-
is amiodarone or procainamide. Unstable patients tion of the nerve. With respect to the facial nerve,
require cardioversion. this results in complete facial droop rather than the
forehead sparing seen with cortical lesions (due to
W Answer B. Verapamil is the drug ofchoice for pro- bilateral innervation of the fudal nucleus).
phylaxis of future cluster headaches. Corticosteroids
233
234 1000 Questions to Help You Pass the Emergency Medicine Boards

[!) Answer C. Botulinum toxin works by irreversibly patients. Whether it exerts its protective effects by
binding to the presynaptic membrane of periph- reducing vasospasm is still an object of study.
eral and cranial nerves where it inhibits the release
of acetylcholine. The patient improves as new re- [!] Answer E. The patient appears to have evidence of
ceptors are manufactured. Because the disorder is a specific dystonic reaction known as the oculogyric
localized to the neuromuscular junction, there are crisis, where both the patient's eyes stare upward and
no sensory findings. The classic presentation is de- do not come back to the neutral position. Symptoms
scending, symmetric paralysis which typically starts can fluctuate based on emotional state. Dystonic re-
in the bulbar muscles. Such patients present with actions are usually due to an excess of cholinergic
dysarthria, diplopia and dysphagia which progresses activity due to overblockade of dopaminergic re-
to generalized weakness. The toxin does produce ceptors by antipsychotic medications. The normally
anticholinergic symptoms due to its inhibition of inhibitory effect ofdopamine on the cholinergic neu-
cholinergic output. Therefore, the patient's pupils rons is reduced with the use of antipsychotic agents.
may be dilated and unreactive, which can be a useful Treatment for acute dystonic reactions is with an
feature for differentiating such patients from those anticholinergic agent, either benztropine or diphen-
with myasthenia gravis. Gastroenteritis may or may hydramine. Discharging the patient home neither
not occur, but the onset of symptoms is not until6 to treats the patient nor adequately manages the social
48 hours after ingestion. Mental status remains intact. situation. Psychiatric consultation for a new psy-
chotropic medication prescription is not appropriate
IIJ Answer A. Endocarditis prophylaxis with antibi- until the primary cause of the complaint has been
otics for patients with prosthetic or damaged valves evaluated and addressed. Haloperidol would further
is recommended for the following procedures: Dental worsen the oculogyric crisis. Lorazepam might se-
cleaning, rigid bronchoscopy, ERCP, and cystoscopy. date the patient but would probably not improve the
dystonia.
[!] Answer E. Because cerebral perfusion pressure
( CPP) = mean arterial pressure (MAP)-intracranial [!g) Answer B. There are no clear diagnostic criteria
pressure (ICP), acute reductions in the MAP may for hyperemesis gravidarum. However, it is generally
have drastic and potentially grave effects on a considered to be nausea and vomiting related to preg-
patient's CPP. One of the most common errors nancy that is severe enough to result in weight loss,
in the treatment of patients with ischemic strokes starvation ketoacidosis, electrolyte imbalance, alka-
is the overly aggressive treatment of hypertension. losis from vomiting, and dehydration. Liver enzymes
Because this patient is not a candidate for tPA may be elevated in up to 25% of hospitalized women,
(because she presented >3 hours after the onset although they are not normally increased beyond
of her symptoms), the general consensus is that her four times the upper normal limit. This more fre-
hypertension does not warrant treatment unless her quently occurs when hyperthyroidism accompanies
blood pressure is >220/120 mmHg or her MAP hyperemesis. Hyperthyroidism complicating hyper-
is >130 mmHg. If this is the case, then either emesis is transient and usually resolves by 14 to 16
nitroprusside, labetalol or IV enalapril may be used to weeks' gestation, along with the symptoms of hy-
rapidly gain control of the patient's blood pressure. peremesis. The elevated levels of thyroid hormone
Additionally, in patients who are tPA candidates, are thought to result from high levels of f:l-hCG,
these medicines can be used to bring a patient's which is known to stimulate the thyroid-stimulating
blood pressure below the 185/105 mm Hg threshold hormone (TSH) receptor.
required for the administration of tPA. Exceptions
to this approach may also apply in patients with ~ Answer B. Ischemic or hemorrhagic cerebellar in-
an ischemic stroke and concomitant Ml, aortic farction is frequently associated with vertigo due to
dissection or acute renal failure due to malignant disruption of various vestibulocerebellar pathways.
hypertension. Such patients may require emergent Typical symptoms associated with cerebellar infarc-
interventions to decrease blood pressure due to tion include headache, nausea and vomiting, ataxia
these concomitant conditions. In all other patients with inability to walk (i.e., truncal ataxia), vertigo,
with acute ischemic stroke, moderate hypertension and dysarthria. The fall that this patient had 2 weeks
as defined in the preceding text is thought to ago was likely a drop attack due to ischemia within
be neuroprotective by maintaining adequate CPP. the vertebrobasilar arterial system. Lesions within
Nimodipine is a calcium-antagonist that is used in the cerebellum result in ipsilateral clinical deficits
patients with subarachnoid hemorrhage {SAH), and due to "double crossing" although many pathways
it has been shown to improve outcome in such also have bilateral projections. Such patients may
Test8 235

deteriorate rapidly and require vigilant observation in infants younger than 1 year due to congenital
in an intensive care setting. diaphragmatic defects. In older people, it is frequently
associated with large paraesophageal hiatal hernias.
~ Answer C. PP refers to the "paradox'' that heart The classic triad is known as Borchardt's triad, and
sounds are heard, although no peripheral pulse consists of severe epigastric pain and abdominal
is felt. Furthermore, the clinical assessment of distension, vomiting, and the inability to pass a
this "pulse" is performed through measurement nasogastric tube. If the diagnosis is suspected, the ED
of the peak systolic blood pressure (SBP). It is physicians should attempt to pass a nasogastric tube
caused by a physiologic decrease in the amplitude because this occasionally reduces the volvulus. Owing
of the pulse up to 10 mmHg during inspiration to its redundant blood supply, gastric infarction is
and an increased amplitude during expiration. A uncommon, even in delayed cases, occurring in as
difference of> 10 mm Hg is pathologic. The negative many as 25% of cases.
intrathoracic pressure generated during inspiration
causes increased venous return and right ventricular [!ID Answer E. Calcium pyrophosphate dihydrate-
distension. The interventricular septum bulges to CPPD) crystal deposition disease or pseudogout,
the left, reducing the size and the subsequent is most commonly idiopathic. However, it may also
stroke volume of the left ventricle. The decreased be secondary to any of the underlying conditions
stroke volume causes cardiac output to fall, which listed. Attacks are typically not as severe as in gout,
decreases blood pressure. Although PP is present in although they share the same management.
the setting of a tension pneumothorax, the most
common extracardiac cause is asthma. This is in ~ Answer E. Patients with severe upper gastroin-
part due to chest hyperinflation and in part due to testinal bleeding (UGIB) require emergent blood
an exaggerated difference in intrathoracic pressures transfusion which may be life saving. In patients
that occur throughout the respiratory cycle. Patients with more moderate bleeding, continuous infusions
with severe asthma exacerbations frequently have of proton pump inhibitors (PPis) have been shown
a widened PP. One important thing to consider, to improve outcome by reducing the need for blood
however, is that the measurement ofPP depends on products and reducing the need for reintervention.
patient effort. Therefore, in a deteriorating asthmatic However, intermittent bolus administration is much
who has decreasing thoracic excursion, the PP less effective than continuous infusion. This is be-
will fall, and this should not be interpreted as an cause continuous PPI infusions maintain gastric pH
improvement in the patient's condition. >4 (the threshold for pepsin inactivation). Bolus
administration allows gastric pH to fluctuate and
[YJ Answer A. Lipase and amylase have roughly the episodes of increased gastric acidity may disrupt dot
same sensitivity for diagnosing acute pancreatitis, formation. H2 inhibitors do not alter the natural his-
although their sensitivity depends on the threshold tory ofUGIB. Sengstaken-Blakemore tubes are rarely
value above normal used to establish the diagnosis used adjuncts to stop hemorrhage from esophageal
(most authors suggest a cutoff of three times the varices. Though they are effective, they have a high
upper limit of normal). Lipase is almost certainly complication rate and should only be used when en-
more specific than amylase, because almost all lipase doscopyis not immediately available. Ewald tubes are
originates from the pancreas. However, there is a large bore nasogastric tubes used for gastric lavage.
small amount of gastric lipase, and lipase levels may They may be useful to irrigate the stomach before
be elevated in the setting of a gastric or duodenal endoscopy or in cases of acute toxic overdose as a
ulcer, severe renal insufficiency or in some cases of means of decontamination. Octreotide is primarily
bowel obstruction. Although both enzymes tend to used for acute variceal hemorrhage although it may
rise at approximately the same rate, lipase remains be useful as an adjunct in cases of nonvariceal UGIB.
elevated for a longer period of time (lipase remains This question is still being studied.
elevated for 8 to 14 days, although amylase returns
to normal after 5 to 7 days). The degree of elevation [!1) Answer C. Infants with pyloric stenosis typically
of amylase or lipase does not correlate with disease present between 2 and 6 weeks of age with
severity. The ratio ofamylase to lipase has not proved progressive, projectile, nonbilious emesis. Persistent
to be useful. emesis results in a loss of hydrogen and chloride ions
from the gastric juices (hydrochloric acid) resulting
~ Answer D. Gastric volvulus is a rare disorder in a hypochloremic alkalosis. With time, cellular
that chiefly occurs in older people and results exchange mechanisms pump hydrogen ions into the
from twisting of the stomach about its long axis blood in exchange for potassium ions resulting in
(organoa.xial volvulus). Twenty percent ofcases occur hypokalemia.
23 6 1000 Questions to Help You Pass the Emergency Medicine Boards

~ Answer C. This EKG reveals changes consistent glomerulonephritis at a later stage. Beh~et's disease
with hyperkalemia. Cardiac arrhythmias are the most is characterized by recurrent oral and genital ulcer-
serious consequence of hyperkalemia and the pres- ations and recurrent hypopyon (its rarely seen, but
ence of EKG changes mandates emergent therapy. pathognomonic finding) . Churg-Strauss syndrome
This patient's EKG demonstrates peaked T waves, involves the lungs and most patients have symptoms
which are among the early EKG changes in the setting of asthma in the 2 years preceding a diagnosis.
of hyperkalemia, typically occurring at levels above
6.5 mEq per L. In general, hyperkalemia decreases ~ Answer A. Both primaquine/clindamycin and
cardiac excitability resulting in flattened P waves, a pentamidine are alternative regimens for PCP
prolonged PR interval, and a widened QRS interval. treatment. However, pentamidine can only be ad-
Although all the agents listed are beneficial in patients ministered intravenously or by inhalation. Therefore,
with hyperkalemia, calcium is the agent of choice, as primaquine and clindamycin comprise the best alter-
it has a rapid onset of action (1 to 3 minutes), and sta- native outpatient regimen. Intravenous pentamidine
bilizes myocardial membranes. Calcium gluconate or is the treatment of choice for patients unable to tol-
calcium chloride may be given, but calcium chloride erate TMP-SMX. In addition to allergic responses,
provides three times the amount of elemental cal- TMP-SMX has many side effects including a high in-
cium per unit dose. However, calcium chloride may cidence of skin rash and bone marrow suppression.
cause tissue necrosis upon extravasation from intra- In the outpatient regimen of primaquine and clin-
venous lines and is irritating to local veins. Therefore, damycin, the clindamycin is included because pri-
most authors recommend that calcium chloride is de- maquine lacks activity against community-acquired
livered through a large-bore central venous catheter. pathogens. Because only patients with mild to mod-
(Figure reprinted with permission from Wagner G. erate disease will be discharged from the hospital,
Marriott's practical electromyography, lOth ed. Lip- it is imperative to include coverage for routine
pincott Williams & Wilkins; 2001 :225.) community-acquired pneumonia (CAP) in addition
to giving antibiotics which target PCP. In mild cases,
(!!] Answer D. Psychiatric emergencies necessitating or when the patient's CD4+ T -cell count hovers close
admission to the hospital include suicidal ideation, to 200, it may be difficult to distinguish between PCP
homicidal ideation, acute mania, acute psychosis, and CAP.
or inability to cooperate with treatment or care
for self. A past history of psychiatric disease, ~ Answer C. Traumatic aortic injury (TAl) occurs
including mood or thought disorders, is not sufficient most commonly from high-speed motor vehicle
to warrant admission, especially if symptoms are collision (MVCs) causing blunt thoracic trauma.
well controlled with medication. Acute ethanol Most traumatic aortic ruptures are immediately fatal,
intoxication should not be a criterion for admission but patients who survive to ED evaluation are usually
to the hospital-patients should be questioned successfully treated. The descending aorta just distal
regarding specific psychiatric symptoms after the to the subclavian artery is the most commonly
ethanol has been metabolized and the patient has injured site. Chest and back pain are the most
the capacity to give a coherent history. A patient common symptoms. The initial screening test is
with a major depressive episode that lacks significant plain chest x-ray-however, the sensitivity of plain
risk for suicide need not be admitted to the hospital films is only up to 85%. In cases where suspicion
if adequate outpatient follow-up is arranged, if the for TAl is high, confirmatory CT aortography should
patient can contract for safety, and if the patient has be performed, as it has close to 100% sensitivity.
a sufficient support system. Transesophageal echocardiography may be used in
select cases where cr scan is not possible, but
~ Answer A. The vasculitic syndromes have multiple transthoracic echocardiography is much less accurate
areas of overlap in their clinical manifestations and and should be used only to evaluate for pericardia!
it is sometimes difficult for rheumatologists to apply effusion or tamponade, not TAI. Management of
a specific diagnosis. However, classically, polyarteri- TAl involves operative repair, but blood pressure
tis nodosa (PAN) causes mononeuritis multiplex and heart rate control with ,8-blockers is essential to
and mesenteric ischemia. Cutaneous findings are prevent further damage to the aorta from shear forces.
also common. Takayasu's arteritis is very com-
mon in Japan and results in coronary ischemia. ~ Answer B. Lie refers to relation of the longitudi-
Wegener's granulomatosis initially presents with nal axis of the fetal spine to the longitudinal axis
symptoms of upper airway problems such as si- of the uterus. Only fetuses with a longitudinal lie
nusitis, otitis, and nasal congestion while developing may be safely delivered vaginally. Presentation refers
Test8 237

to the fetal part that overlies the maternal pelvis. arterial occlusion, but is usually not adequate to
Approximately 95% of pregnancies are cephalic in treat limb-threatening ischemia due to an embolus.
presentation. Any presentation that is not cephalic Lumbar sympathectomy and hyperbaric oxygen
is referred to as a malpresentation. In cephalic pre- therapy provide no benefit in these circumstances.
sentations, the smallest possible presenting diameter
occurs when the fetal head is muimally flexed (at- ~ Answer B. Alcohol is the tmcin most commonly as-
titude refers to the relationship of the fetal head to sociated with seizures. Most alcohol-related seizures
its spine, i.e., flexion or extension). Position refers to are due to alcohol withdrawal and typically occur
the relationship of the presenting part of the fetus between 6 and 48 hours after discontinuation of
to the maternal pelvis. In cephalic presentations, the drinking. However, alcohol withdrawal seizures have
occiput is used as a reference point, whereas in breech been known to occur as long as 7 days after dis-
presentations, the sacrum is the fetal reference point. continuation of drinking, particularly in cases of
Station refers to the distance of the fetal presenting polysubstance abuse with benzodiazepines and bar-
part from the maternal ischial spines. A station of 0 biturates. Interestingly, acute alcohol intoxication
implies that the leading bony edge of the fetus is at can also provoke seizures and there is some elec-
the level of the ischial spines. troencephalographic evidence to suggest a lowered
seizure threshold in this setting.
~ Answer A. Noroviruses, which include Norwalk
virus, are responsible for 50% to 80% of all cases ~ Answer B. Cyanosis, a bluish discoloration of the
of acute infectious diarrhea. However, most patients skin and mucous membranes caused by the pres-
with acute infectious diarrhea do not seek medical ence of deoxyhemoglobin (unsaturated hemoglobin)
treatment. Patients who seek medical attention are is actually an uncommon finding in asthmatic pa-
more likely to have a bacterial cause, most commonly tients. It is only visible when the absolute quantity
Campylobacter spp. of unsaturated hemoglobin exceeds 4 g per dL. Pa-
tients experiencing an asthma exacerbation have a
~ Answer B. Cryptosporidium parvum is a parasite, respiratory alkalosis (Paco2 ..J..) resulting from their
which often causes subacute and chronic diarrhea pronounced hyperventilation. This alkalosis shifts
in patients with AIDS. Highly active antiretrovi- the oxyhemoglobin dissociation curve to the left,
ral therapy (HAART) is the best treatment for which means that at any given partial pressure of
Cryptosporidium diarrhea. Symptoms are virtually oxygen there is more saturated hemoglobin. There-
eliminated if CD4 counts are maintained > 100 cells fore, only profoundly hypoxic patients or patients
per JLL. Antidiarrheal agents and antibiotics work who have ventilatory failure (Paco2 t) will exhibit
with only varying degrees of success and the symp- cyanosis. In most asthmatics (>50%) with an exacer-
toms are often recurrent after these drugs are stopped. bation, respiratory rates range between 20 breaths per
Octreotide has no role in the management of HIV- minute and 30 breaths per minute, although <20%
associated infectious diarrhea. of patients have rates ~ 30 breaths per minute. Thus,
while patients with a RR <40 may be experiencing a
~ Answer E. Pyogenic liver abscesses are uncommon severe asthma exacerbation, patients with a RR >40
infections that most commonly occur as a compli- almost always have severe asthma (as defined by an
cation of biliary tract infections (e.g., cholangitis, FEVl <l.OL). Pulsus paradoxus (PP) may be absent
cholecystitis). A sizable number of cases are also in 50% of patients with severe asthma. In severe
cryptogenic. In the past, untreated appendicitis com- obstruction, PP is >10 mmHg (the normal value)
plicated by pylephlebitis was a very common cause, but only extremely high PP ( > 25 mm Hg) correlates
particularly in young patients. Most infections are with severe asthma. Accessory muscle use is a sign of
polymicrobial and a broad range of organisms is severe airflow obstruction and is not present during
typically involved. The treatment for pyogenic liver mild exacerbations. In patients with refractory severe
abscesses is surgical drainage and antibiotic therapy. airflow obstruction, especially those who are becom-
ing fatigued, the volume ofwheezing may decrease as
~ Answer A. The patient has acute arterial occlusion their condition worsens because the total volume of
from arterial embolism, likely due to atrial fibrillation air movement is small. Despite the fact that the ob-
caused by hyperthyroidism. Treatment involves struction is severe, the patient may not be ventilating
anticoagulation and emergent embolectomy due enough air to generate wheezes.
to the limb-threatening nature of the occlusion.
Bypass surgery is usually used in patients who have ~ Answer E. The patient has iritis, which is treated
in situthrombosis. Anticoagulation alone is used primarily with topical steroids and mydriatics.
as adjunctive ED therapy for patients with acute Ophthalmologic consultation is generally pursued
238 1000 Questions to Help You Pass the Emergency Medicine Boards

before the initiation of steroids. The history of methylergonovine maleate or ergonovine maleate,
consensual photophobia and physical examination or carboprost tromethamine. Tears of the maternal
demonstrating perilimbic conjunctival injection birth canal may also result in significant hemor-
(ciliary flush) is characteristic. Topical antibiotics rhage and are the second most common cause (as
are used to prevent bacterial superinfection in a group). Retained placental tissue accounts for
corneal abrasions or viral conjunctivitis. Hyper- roughly 10% of immediate postpartum hemorrhage.
tonic eye drops are used for corneal hydrops If uterine massage and oxytocic agents fail to control
(extreme corneal edema). Mannitol therapy for bleeding thought to be due to uterine atony, a metic-
lowering intraocular pressure is indicated for pa- ulous search should be conducted for maternal birth
tients with glaucoma. Ocular massage is indicated trauma or retained placental tissue.
for patients with central retinal artery occlusion to
try to dislodge embolus or thrombus and cause it ~ Answer E. Adult trauma patients with head injury
to migrate to a more distal site in the circulation. are rarely hypotensive because of the intracranial
process itself, except in the end stages of herniation
[llJ Answer C. Sickle cell disease is a hemoglobinopa- or severe scalp injuries. The fixed bony skull limits the
thy causing sickling of RBCs with any systemic degree of hemorrhage in adult patients. In infants,
stress, which results in diffuse microinfarctions. the flexibility and larger proportional size of the skull
Sickle cell trait is present in approximately 10% may allow enough bleeding to cause hypotension.
of all African Americans, and sickle cell disease is In adult trauma patients, an extracranial cause of
primarily a disease of this population. Symptoms hypotension should aggressively be sought, such as
involve multiple organ systems and result in specific bleeding in the chest, abdomen, retroperitoneum,
acute crises-vaso-occlusive, acute chest syndrome, pelvis, or femurs. Treatment of hypotension in the
splenic sequestration, and aplastic. The overall most head-injured patient should be undertaken quickly
common cause of death in patients with sickle cell because cerebral blood flow is dependent on mean
disease is from infection, usually pneumonia. Owing arterial pressure (MAP) and limited by intracranial
to autoinfarction of the spleen, patients are at risk pressure (ICP).
for overwhelming sepsis from encapsulated organ-
isms, such as Streptococcw pneumoniae, E. coli, and ~ Answer C. Urinalysis is a crucial diagnostic tool
Haemophilus influenzae. Stroke is another common for the evaluation of all urinary system conditions.
cause. Aplasia and splenic sequestration occur less The presence of casts in the urine indicates a renal
often. MI is rare in sickle cell patients, as coronary source--RBC casts are associated with glomeru-
artery disease, although probably accelerated in these lonephritis and white blood cell casts with parenchy-
patients, does not usually progress far enough to mal inflammation, such as pyelonephritis. Urine
significantly increase the risk of infarction. dipstick is a rapid screening tool to detect the presence
of glucose, leukocytes, protein, and blood. Unfortu-
~ Answer C. Inflammation of the iris caused by nately, sensitivity for most ofthese parameters is only
trauma causes constant pain and photophobia, on the order of 75% to 85% and negative dipstick
especially consensual photophobia (light exposure to should not be used to rule out their presence. Transi-
the unaffected eye causes pain in the affected eye due tional cells are from a bladder source, but are usually
to consensual constriction). Long-acting cycloplegics a normal finding and do not necessarily indicate a
and steroids are the mainstay of treatment. The malignant process. Normal urinary pH is from 5 to 8
pupil is reactive and constricted, and ciliary flush and usually mirrors serum pH except in certain dis-
(conjunctival injection in a circular rim around ease states, such as renal tubular acidosis or urinary
the limbus) is prominent. Resolution should occur tract infection.
within 1 week.
~ Answer C. The distal interphalangeal (DIP) joints
~ Answer C. Uterine atony is the most common are never affected in rheumatoid arthritis (RA),
cause of immediate postpartum hemorrhage (de- which provides a useful means of differentiating
fined as blood loss that occurs within the first the disease from osteoarthritis (OA). The arthritis
24 hours of delivery), as it is responsible for ap- of RA is typically polyarticular and symmetric,
proximately 50% of cases. The risk factors for particularly affecting the hands (MCP and PIP
uterine atony are multiparity, prolonged labor, ex- joints), wrists, and elbows. The disease is twice
cessive uterine manipulation, and general anesthesia as common in women and peaks in the fourth
with halogenated anesthetic agents. Management in- to sixth decade. Two thirds of patients with RA
volves abdominal or bimanual uterine massage as develop cervical spine disease, although thoracic
well as the use of oxytocic agents such as oxytocin, and lumbar disease is uncommon. The disease
Test8 239

most commonly involves the occipitoatlantoaxial seizures without an interictal return to baseline
junction and anterior atlantoaxial subluxation may mental status.
occur. Rheumatoid factors (RFs) are autoantibodies
directed at the crystallizable fragment (Fe) of human ~ Answer C. The EKG demonstrates atrial flutter at
immunoglobulin molecules. The exact incidence of a ventricular rate of 50. The risk of atrial thrombus
RF depends on the assay used and the threshold titer increases with the amount of time the patient
used to separate positive from negative results. In is in atrial fibrillation or atrial flutter. Emergent
general, roughly 15% of patients with RA will be management of atrial fibrillation or flutter involves
seronegative (RF within the normal range} and those reduction of rate to below 100 and anticoagulation
patients tend to have milder disease. if the duration of the dysrhythmia is longer than
48 hours, unless echocardiogram indicates no cardiac
~ Answer D. Tube thoracostomy is a procedure that thrombus. This patient is not tachycardic and
is fraught with potential complications. In a recent requires no rate controlling agents such as diltiazem
series of 47 trauma patients, the complication rate or esmolol. Amiodarone is not indicated as this
was 30%. Complications that will be evident in the ER may actually terminate the atrial flutter and put
are usually related to tube insertion, such as kinked the patient back into sinus rhythm and at risk
or clotted tubes, intercostal artery lacerations, lung for thromboembolus. Adenosine is indicated for
lacerations, diaphragmatic perforation, or insertion paroxysmal supraventricular tachycardia-it may be
of the tube subcutaneously. Infectious complica- used in unclear cases ofnarrow-complex tachycardia,
tions such as empyema occur well after insertion, but has no role in obvious atrial flutter.
requiring at least a few days to develop. Reexpansion
pulmonary edema (REPE) is a rare, but potentially ~ Answer E. This case demonstrates the "80% rule"
fatal consequence of tube thoracostomy. Its inci- of neck masses. Eighty percent of neck masses
dence is uncertain, however, because early studies in children are benign, 80% of nonthyroid neck
did not report this complication whereas more re- masses in adults are neoplastic and 80% of those are
cent studies have reported an incidence as high as malignant. Therefore, most nonthyroid neck masses
14%. Patients with pneumothoraces >30% are at in adults are malignant. Referred ear pain and signs
greatest risk for developing REPE. Some studies have of otitis media with effusion increase the likelihood
also shown that the presence of a pneumothorax for of cancer. Any degree of stridor, dysphagia, or severe
a prolonged period (>3 days) before reexpansion is hoarseness mandates immediate ENT consultation,
also a risk factor. No controlled studies have demon- as airway obstruction may be imminent.
strated the best treatment for individuals with such
risk factors. However, the consensus of the American [!!) Answer C. Isopropanol classically does not cause
College of Chest Physicians is that in patients with a an elevated anion gap when ingested. The osmolar
2:30% pneumothorax, a small bore chest tube ( 16-22 gap, however, is elevated and should be calculated and
French) should be used and placed to water-seal only measured when there is suspicion of toxic alcohol
or to a Heimlich valve device. As some studies have overdose. Elevation of the anion gap due to lactic
suggested that the rate ofreexpansion may also play a acidosis can occur in cases of severe isopropanol
role, vacuum suction should not be used. All such pa- poisoning ifthere is associated coma, gastrointestinal
tients should be admitted. Because negative pressure hemorrhage, or hypotension. Choices A, B, D, and E
is not being applied, lung reexpansion may not occur, all cause an elevation in the anion gap at some point
and suction may be required especially if the patient during their metabolism.
is clinically unstable. If REPE develops, treatment is
supportive as with other causes of noncardiogenic ~ Answer A. MRI is the most specific test for the
pulmonary edema. diagnosis of aortic dissection. Logistic difficulties
prevent routine use of MRI in this setting-for
~ Answer C. Electroencephalogram (EEG} changes this reason, CT aortogram is the most commonly
last for an average of59.9 seconds (standard deviation used test and has excellent sensitivity and specificity.
of 12 seconds}, whereas behavioral changes last 52.9 Chest x-ray is abnormal in most cases, but the
to 62.2 seconds (with a standard deviation of 14 sensitivity is not high enough to rule out the
seconds). Therefore, a seizure that has lasted for 5 diagnosis in high-risk patients. Transesophageal, not
minutes is more than 17 standard deviations longer transthoracic, echocardiography may provide useful
than the "typical" seizure. This is partly why status structural information about the descending aorta,
epilepticus is now "operationally" defined as any heart, and pericardium, but Cf aortogram and MRI
seizure lasting 5 minutes. The traditional definition are far more specific. Aortography is used only in
has been any seizure lasting 30 minutes, or recurrent confirmatory settings. Electrocardiography is useful
240 1000 Questions to Help You Pass the Emergency Medicine Boards

only in ruling out other causes of the patient's ~ Answer D. Follicular cysts are the most frequent
symptoms and have no utility in confirming the adnexal cystic structures in women with normal
diagnosis of aortic dissection. ovaries. Follicular cysts represent remnants of previ-
ously normal follicles that grew in response to follicle
~ Answer E. Nonaspirin NSAIDs, including ibupro- stimulating hormone (FSH) and then failed to in-
fen, produce generally benign and self-limited con- volute. They are typically clinically silent but they
ditions in overdose. Symptoms will occur within may cause pelvic pain or heaviness, as well as urinary
4 hours of ingestion, are usually mild, and resolve frequency and constipation if they are large enough.
within 24 hours. Patients rarely have life-threatening They are self-limited and involute over a period of
overdoses and almost never require antidotes, decon- weeks to months. Corpus luteum cysts are less com-
tamination, augmented renal excretion, or invasive mon but more clinically relevant. They also represent
therapies such as hemodialysis. Serum levels of remnants of formerly normal physiologic structures;
nonaspirin NSAIDs are not clinically useful. Of in this case, the corpus luteum. Unlike follicular cysts,
overdoses with nonaspirin NSAIDs, phenylbuta- they have a propensity to be complicated by intra-
zone and mefenamic acid are more serious, poten- cavitary hemorrhage. If the hemorrhage is brisk, the
tially causing multiorgan dysfunction and seizures, intracystic pressure may rise very quickly resulting
respectively. in rupture. Such an event may result in acute onset,
severe pelvic pain, and may be associated with signif-
~ Answer C. Viscosity decreases with any inflam- icant hemorrhage depending on the size of the cyst.
matory process of the joint because of decreased Theca lutein cysts are uncommon, typically bilateral,
hyaluronic acid, which is the main contributor to and associated with prolonged or excessive ovarian
synovial fluid viscosity. Rheumatoid arthritis (RA) stimulation. Dermoid cysts are benign ovarian ter-
classically affects the metaphalangeal (MP) and PIP atomas that contain tissue from all three germ cell
joints of the hand, whereas oteoarthritis ( OA) affects layers. They do not pose an immediate danger but
the first carpometacarpal joint as well as the prox- patients should be referred for further management
imal (PIP) and distal interphalangeal (DIP) joints. because they may undergo malignant transforma-
Systemic lupus erythematosus (SLE) may cause in- tion, particularly in women older than 40. Finally,
flammation of serosal surfaces such as the pleura ovarian fibromas are the most common, benign,
or pericardium. Pericarditis in a patient with SLE solid neoplasms of the ovary. They are extremely
may result in an audible cardiac friction rub. Re- slow growing but may grow to very large sizes.
iter's syndrome may cause sausage-shaped swelling
of the digits. An abducted, externally rotated hip in a ~ Answer D. Most aspirated foreign bodies are of
neonate suggests infection, even in patients who are vegetable origin and are therefore radiolucent. There-
afebrile. fore, the diagnosis of foreign body aspiration is most
commonly made by virtue of symptoms of respi-
~ Answer D. Only intravenous medications are ap- ratory distress such as wheezing, persistent cough,
propriate for rapid, measured control of blood and choking. In addition, clinicians may detect de-
pressure. Nitroprusside is very easily titrated and creased breath sounds on the side ofthe obstruction.
extremely effective, making it the drug of choice Although most aspirated foreign bodies are radi-
for hypertensive crises. Nifedipine has been asso- olucent, most chest x-rays are abnormal. Unilateral
ciated with severe side effects due to its unpre- obstructive emphysema is the most common in-
dictable response. Isoproterenol is a ,8-agonist and direct finding indicating airway obstruction. The
will not decrease blood pressure. Phenoxybenza- aspirated foreign body creates a one-way valve effect,
rnine is a-blocking agent used mostly in the pre- allowing inspired airflow but preventing complete
vention of catecholamine surge during therapy for exhalation. This creates hyperexpansion and relative
pheochromocytoma. Hydrochlorothiazide is an oral hyperlucency with decreased lung markings on the
medication appropriate for outpatient therapy for affected side. However, atelectasis and pneumonia
chronic hypertension. are also common findings. Delayed presentations
of foreign body aspiration are common, due in
~ Answer D. Age-related macular degeneration is part because many children aspirate food particles
the most common cause of blindness in the while unattended. They subsequently experience a
industrialized world. It occurs primarily because of brief symptomatic phase and then may be relatively
retinal damage from unknown causes. Almost one asymptomatic, or have "low-level" symptoms that
fourth of all Americans older than 90 are affected by may be mistaken for a viral respiratory illness. Ow-
macular degeneration. ing to the relatively smaller caliber of their airways,
Test8 241

however, infants younger than 12 months typically increase the risk of a heterotopic pregnancy. The
present more acutely than older children. In gen- risk of a heterotopic pregnancy in the setting of in
eral, children between the ages of 1 and 3 years are vitTofertilization is approximately 1%, whereas the
at highest risk for foreign body aspiration. Classic risk in the setting of some ART therapies may be
teaching is that most aspirated foreign bodies lodge as high as 4.5%. The key aspect to recognize is that
in the right m.ainstern bronchus due to its more acute ART is the primary risk factor for heterotopic preg-
angle with the trachea. Some studies however, do not nancy and the resulting risk ofheterotopic pregnancy
reveal a statistically significant difference between is approximately the same as the risk for ectopic
foreign body aspiration to the left or right rnainstem pregnancy in the general population.
bronchus. They do, however, demonstrate that for-
eign bodies are more commonly located in either the ~ Answer C. The patient has nausea, vomiting, an
right or left bronchus than in the trachea. The most elevated anion gap, ketosis, and a normal glucose in
predictive finding in foreign body aspiration is an the setting of excessive alcohol use with starvation.
episode of witnessed aspiration followed by choking. Alcoholic ketoacidosis (AKA) is the most likely cause.
Treatment ofAKA is with fluid resuscitation, glucose,
~ Answer A. All of the listed items are potential and thiamine. Bicarbonate is not indicated in most
causes of pruritus ani, but the presence of fecal patients with high anion gap metabolic acidosis
matter on the perianal skin is the most common. except in severe, life-threatening cases. Insulin is
indicated in patients with diabetic ketoacidosis
~ Answer E. The most dangerous acute complica- (DKA), which rarely presents with a normal glucose
tion is upper airway obstruction. Like any other level or in an adult patient with no prior history
patient with supraglottic airway obstruction, such of diabetes. Potassium repletion may be indicated
patients usually appear toxic and present with res- if hypokalemia is present or expected during the
piratory distress, inspiratory stridor, and accessory course of therapy. Magnesium supplementation is
muscle use. Such patients should be allowed to find often indicated in chronic alcoholic patients, but
their own most comfortable position and should glucose therapy is of more importance as an energy
never be forced to lie flat. Furthermore, anesthesia substrate in patients with alcohol ketoacidosis.
consultation may be required in order to perform
fiberoptic intubation. The ED physician should also ~ Answer A. Thelateralorfibularcollateralligarnent
be prepared to perform emergency cricothyroido- complex comprises of three ligaments that tend to
tomy. In addition to the other complications listed, rupture in an anterior to posterior sequence during
jugular vein thrombosis, carotid artery erosion, ankle sprains. The anterior talofibular ligament is the
pneumonia, empyema, pulmonary abscess, cervi- weakest, and rupture results in a positive anterior
cothoracic necrotizing fasciitis, and intracranial or ankle drawer test. The calcaneofibular is next and the
parapharyngeal extension of the infection may all posterior talofibular is the most posterior portion of
complicate peritonsillar abscesses. the fibular collateral ligament. The deltoid ligament
is 20% to 50% stronger than its lateral counterpart
~ Answer B. Cellulitis in the healthy patient is most and is infrequently injured in isolation. The anterior
often caused by streptococci and staphylococci. Ini- inferior tibiofibular ligament is the weakest ligament
tial antibiotic therapy is usually with a penicillinase- of the four syndesmotic ligaments that attach the
resistant penicillin or first generation cephalosporin. distal tibia and fibula. The syndesmosis prevents
Doxycycline and clindamycin are appropriate al- displacement of the tibia and fibula relative to one
ternatives. Gentamicin covers mostly gram-negative another and disruption can contribute to significant
bacteria which might be indicated as part oftreatment instability.
for cellulitis in an immune-compromised patient.
This patient has an allergy to penicillin and di- ~ Answer E. Although > 75% of patients with pul-
cloxacillin is therefore contraindicated. Linezolid is monary embolism (PE) will have an abnormal chest
generally restricted for the treatment ofvancomycin- x-ray, no single finding is diagnostic for PE. Wester-
resistant organisms, which are usually nosocomial. mark's sign (decreased peripheral vascular markings
Metronidazole covers anaerobes only and would not in the lung field affected by the embolus) and Hamp-
be appropriate as monotherapy for cellulitis. ton's hump (a wedge-shaped homogeneous density
with its base along the pleural surface and its apex
~ Answer B. The baseline risk of a heterotopic preg- pointed toward the hilum representing pulmonary
nancy is reported to be 1 in 4,000 pregnancies or infarction) are the classic findings but both find-
0.25%. However, in vitTofertilization and other as- ings have very low sensitivity and specificity. In
sisted reproductive technologies (ART) dramatically addition to the other findings listed, parenchymal
242 1000 Questions to Help You Pass the Emergency Medicine Boards

infiltrates, hilar or mediastinal enlargement, car- The exact mechanism of Raynaud's phenomenon
diomegaly, pleural effusions, pulmonary edema, and in scleroderma (or in other autoimmune diseases)
a prominent central pulmonary artery (Fleischner's is not known. Raynaud's phenomenon may also be
sign) may also be seen. None are sensitive or specific a primary problem (in which case it is sometimes
findings. referred to as Raynaud's disease), rather than sec-
ondary to an underlying disease. To be considered
~ Answer E. Penetrating injury to the globe with a primary process, patients need to suffer no is-
a foreign body still in place mandates operative chemic damage in the affected digits (e.g., gangrene,
removal. No attempts should be made by the EP necrosis), have negative serology (e.g., particularly
to manipulate the foreign body. Broad-spectrum for antinuclear antibodies), have a normal erythro-
antibiotics should be given for suspicion of globe cyte sedimentation rate, have symmetric attacks, and
rupture and emergent ophthalmologic consultation lack physical examination findings which suggest a
should be obtained. Measurement of intraocular secondary cause.
pressure is absolutely contraindicated in cases of
possible globe rupture. Slit lamp examination is ~ Answer A. The patient has evidence of tardive
neither indicated (because of planned operative dyskinesia, a syndrome of uncontrollable contrac-
evaluation anyway) nor possible (because of the tions of the facial muscles because of long-term
mechanical blockade by the foreign body). MRI therapy with neuroleptic medications. The risk for
is contraindicated in patients with a possibility development ofthis disorder is increased with longer
of an intraocular metallic foreign body. (Figure duration of treatment, total cumulative dosage of
from Tasman W, Jaeger EA, eds. The Wills Eye medication, concomitant mood disorder, and pa-
Hospital atlas of clinical ophthalmology, 2nd ed. tient age. Symptoms do not usually start until several
Philadelphia: Lippincott Williams & Wilkins; 2001, years into the neuroleptic therapy. Tardive dyskinesia
with perrrrission.) is only rarely reversible if the causative medication is
stopped. Elderly women are the highest risk group.
~ Answer A. Somatization disorder refers to a con- Specific treatment does not exist for the condition,
stellation of physical symptoms that cannot be although benzodiazepines and the newer atypical an-
explained by a known medical condition. Pain, tipsychotic medications have reduced the incidence.
gastrointestinal, sexual, and neurologic symptoms
predominate. An integral part ofthe diagnosis is that ~ Answer C. Abdominal masses in infants are usu-
the patient is not faking the symptoms-he or she ally renal in origin, most commonly benign tumors
truly is experiencing them and will argue against or cysts. Both neuroblastoma, most often arising
any evidence that indicates somatization. Somatiz- from the adrenal glands, and Wilms tumor, the most
ing patients have an uncontrollable need to assume common renal malignancy, are frequent causes of
the "sick role," which allows them to be cared for. abdominal masses. Renal ultrasonography or CT of
The most common personality disorder associated the abdomen and pelvis should be performed to
with somatizing patients is histrionic-well over half better evaluate the mass. Gall bladder tumors and
the number of patients meet the diagnostic criteria. stones are rare in infants. Scrotal ultrasonography
Management of these patients in the ED involves will help to evaluate groin and testicular pathology,
empathetic recognition and acknowledgement ofthe but is not useful for abdominal evaluation. Urinaly-
patient's symptoms, evaluating for true medical ill- sis is commonly normal in patients with renal cysts
ness as a cause of the symptoms, and referral to or tumors. A Meckel's scan is useful to evaluate a
psychiatry or primary care for outpatient evaluation. Meckel's diverticulum, which usually presents with
It is crucial for the EP to review old records of pa- painless rectal bleeding rather than mass.
tients suspected of somatization disorder, as these
patients often undergo repeated unnecessary testing ~ Answer D. The average case-fatality rate is 51%
in the ED. Discussions with the patient's primary with most deaths occurring within 2 weeks of the
care physician is mandatory as this will help to tai- ictus and an estimated 10% of deaths occurring
lor emergent workup. Pharmacotherapy is of little before a patient receives any medical attention.
benefit in the acute setting.
~ Answer C. Herpes simplex encephalitis is clinically
~ Answer C. Raynaud's phenomenon is nearly uni- indistinguishable from other types of meningoen-
versal in patients with scleroderma (also known as cephalitis, causing headache, stiff neck, fever, and
systemic sclerosis) and is the earliest sign of the dis- altered mental status. Temporal lobe involvement is
ease. Raynaud's phenomenon is also common in typical and may be visible on neuroimaging. HSV-1
patients with lupus and rheumatoid arthritis (RA). is the usual cause in adults; neonates have a higher
Test8 243

incidence ofHSV-2 due to maternal infection. As in almost never exhibit systemic symptoms. Systemic
other cases of suspected meningoencephalitis, lum- symptoms may occur in pediatric patients with cys-
bar puncture and cerebrospinal fluid (CSF) studies titis alone.
are indicated, though herpes simplex virus (HSV)
culture is negative in most cases. Acyclovir reduces ~ Answer B. During the metabolism of ethylene
mortality and the frequency ofresidual neurologic se- glycol, glyoxylic acid is produced. Glyoxylic acid
quelae, which occur in the large majority ofuntreated may be metabolized in three ways-two pathways
patients. form nontoxic compounds and the third forms
the toxic oxalic acid, which predisposes to calcium
~ Answer D. Acetaminophen is metabolized by a oxalate crystals. Pyridoxine and thiamine are each
variety of pathways, the most important of which cofactors in the two pathways that form nontoxic
is through the cytochrome P-450 system, which compounds and are recommended as supplemental
produces N-acetyl-p-benzoquinoneimine (NAPQI) therapy in addition to the standard treatment of
which is the toxic metabolite causing hepatocyte ethylene glycol poisoning (bicarbonate, competitive
necrosis. The drug N -acetylcysteine (NAC) reduces alcohol dehydrogenase inhibitors such as alcohol or
the amount of acetaminophen metabolized by this 4-methylpyrazole (Fomepizole), dialysis)
route by replenishing glutathione, the reducing
agent which induces sulfation of acetaminophen to ~ Answer C. Ocular antibiotics may be given by
a nontoxic compound. Severity of acetaminophen ointment or drops. Either form can cause systemic
overdose is measured by a 4-hour acetaminophen absorption and side effects. Drops usually require
concentration as well as markers of liver damage, more frequent dosing due to a shorter duration of
the most important of which is AST. Amylase and action. Ointments are easier to apply in pediatric
lipase are important indicators ofpancreatic damage. patients for this reason. Antibiotics are indicated in
Although GGT and alkaline phosphatase are present most cases of conjunctivitis, as early presentations
in the biliary ductal epithelium, they are less specific of bacterial cases may be clinically indistinguishable
for hepatocellular damage than ASTor ALT. from viral ones.

~ Answer A. Falls are the most common mechanism ~ Answer B. The most common causes of death in
of traumatic injury in the elderly. Impaired balance patients with acute renal failure are volume over-
and vision, medications, and orthostatic hypotension load and hyperkalemia. Hyperkalemia can result
are some reasons for frequent falls in the elderly. Head in fatal dysrhythmias. Treatment involves correc-
trauma is the most important injury for the EP to tion of the renal insufficiency, potassium-binding
evaluate in falls from standing height, and CT scan resin, intravenous calcium for cardioprotection, bi-
should be used liberally in these patients, even in carbonate, and insulin and glucose. Hypocalcemia,
the absence of hard neurologic findings. MVCs are not hypercalcemia, occurs with acute renal failure
the next most common cause of injury, and elderly due to decreased levels of activated vitamin D. Hy-
patients in motor vehicle collision (MVCs) have permagnesemia and hypernatremia may occur in
a much higher risk of death than younger adults. renal failure but are usually clinically inconsequen-
Penetrating trauma is much less likely to occur in tial. Hyperphosphatemia also occurs, but is usually
elderly patients compared with younger adults, but adequately managed with correction of the renal
self-inflicted gunshot wounds in suicide attempts failure and calcium antacids to bind excess gastroin-
by the elderly carry almost 100% mortality. Elder testinal phosphate.
abuse is underappreciated as a cause of injury in the
geriatric population, but is unlikely to be as common ~ Answer D. Mallet finger is a disruption of the
as falls or MVCs. extensor tendon at the level of distal interphalangeal
(DIP) joint with or without an associated avulsion
~ Answer A. Urinary tract infection is generally di- fracture of the dorsal base of the distal phalanx. It
vided into lower (cystitis) and upper (pyelonephri- is caused by a flexion force on the volar tip with an
tis) types. Symptoms of cystitis are almost always extended DIP joint. Conversely, jersey finger results
local-dysuria, increased frequency, urgency, hema- from avulsion of the flexor digitorum profundus
turia, and suprapubic pain. Pyelonephritis is gen- (FDP) tendon at the level of the DIP joint. It most
erally characterized by systemic symptoms, such as commonly occurs when an extension force is applied
fevers, chills, and vomiting, as well as local symptoms to a flexed DIP such as occurs during tackling another
of flank pain in the involved side. Clinically silent player in football by grabbing his jersey. The ring
renal involvement may occur in patients without finger is involved in 75% of cases. In contrast to
systemic symptoms, but adult patients with cystitis the mallet finger, surgical repair is the treatment
244 1000 Questions to Help You Pass the Emergency Medicine Boards

of choice in nearly all cases. A Bennett fracture is production from shivering and decreased tissue
an intra-articular fracture of the base of the thumb perfusion and impaired hepatic function. These
metacarpal (at the carpometacarpal joint) with lateral effects are blunted somewhat because of the &ct
displacement and retraction ofthe distal segment due that as blood cools, it becomes more alkalotic.
to the abductor pollicis longus. Such fractures require A progressive diuresis, not oliguria, occurs as the
thumb spica splinting and frequently need operative temperature cools. The most common neurologic
fixation. Trigger finger refers to a stenosing flexor finding in hypothermic patients is a decreased level
tenosynovitis typically due to overuse. It results in of consciousness. EEGs in hypothermic patients
the formation of a nodule in the flexor tendon sheath, demonstrate generalized slowing and decreased
which prevents extension of the digit at the level of amplitude. In addition, pupillary responses and deep
the MCP joint. It most commonly occurs in the ring tendon reflexes are decreased and patients tend to
and long fingers and local corticosteroid injection have increased muscle tone.
usually results in significant improvement. Patients
should then be splinted in extension and referred to a ~ Answer C. Spontaneous remission is the rule in
hand specialist for further evaluation. Gamekeeper's trigeminal neuralgia as >50% of patients will
thumb is an avulsion injury of the ulnar collateral experience a remission for 6 months. Antiherpes
ligament (UCL) at the thumb-MCP joint. It most virus medications and corticosteroids should be used
commonly occurs during a skiing accident in which for patients with postherpetic neuralgia. This is a
a patient's thumb is trapped in the loop of the pole, separate entityfrom trigeminal neuralgia and patients
resulting in forced abduction and extension of the should not be placed on antiherpes virus medicines
thumb. An avulsion fracture may also occur at the unless they have a history of herpes zoster (shingles)
site ofthe UCL insertion. Patients should be placed in involving the &ce. All patients with trigeminal
a thumb spica splint and referred to a hand surgeon neuralgia should be referred to a neurologist for
for further evaluation. Volar plate entrapment may further evaluation. Up to 2% to 4% of patients
occur with dorsal PIP dislocations preventing ED with trigeminal neuralgia also have multiple sclerosis
reduction. and up to 10% ofpatients have an intracranial lesion.
Therefore, all such patients should receive an MRI on
(!g) Answer B. Neither antibiotics nor corticosteroids an outpatient basis. Unfortunately, roughly 30% of
have any role in prophylactic therapy of potential patients will &it medical therapy and require surgical
aspiration. Corticosteroids may be of use in patients ablation. Phenytoin is not indicated for trigeminal
with a history of reactive airway disease, who neuralgia. Carbamazepine is the standard front-line
have active symptoms upon presentation to the agent, and is started at 100 mg b.i.d.
ED. Otherwise, asymptomatic patients should be
observed for a minimum of 4 hours in the ED. ~ Answer A. Acute spinal cord compression due to
Repeat x-rays are not required. Ifthe patient remains vertebral column metastasis occurs with many can-
asymptomatic and maintains an oxygen saturation cers, including lung, breast, and prostate. Patients
>94% without supplemental oxygen, she may be present with typical findings of epidural compres-
safely discharged. sion, including pain, weakness, or bowel/bladder dys-
function. Any patient suspected of having metastatic
I!!] Answer A. Due to decreased plasma volume, the epidural compression should have an emergent MRI
hematocrit increases approximately 2% for every 1° C of the spine to evaluate the symptoms. Rapid di-
decrease in the core temperature. This is thought agnosis and management is essential to prevent
to be due to increased vascular permeability and irreversible neurologic sequelae. Corticosteroids may
third spacing of fluids as well as cold-induced be started in the ED to reduce edema of the spinal
diuresis and free water loss. In acute hypothermia in cord. Radiation and spinal surgery are the primary
otherwise healthy individuals, hyperglycemia occurs treatments ofmalignancy-related epidural cord com-
because of circulating catecholamines and cold- pression. Other specialties need not be emergently
related inhibition of insulin secretion. There is a consulted in these cases.
tendency toward metabolic acidosis in hypothermic
patients although limited experimental data suggest [!!] Answer C. The patient has evidence of methe-
that patients may present either acidotic or alkalotic. moglobinemia, with cyanosis, unresponsive hypox-
Patients experience respiratory depression with emia on pulse oximetry, and dark-colored blood.
decreasing temperature, in part due to the decrease Oxidation of iron from the ferrous to the ferric state
in metabolism that occurs as the body cools. This prevents hemoglobin from carrying oxygen. Among
raises the Pco2 and decreases the pH. Other factors the answer choices, amyl nitrate is most likely to
that contribute to an acidotic state include lactate cause methemoglobinemia. Co-oximetry must be
Test8 245

performed to accurately calculate the oxygen satu- disorders, decreased mental status, impaired cough,
ration. Treatment is with methylene blue to reduce or swallowing dysfunction).
methemoglobin back to hemoglobin. Lorazepam,
like other benzodiazepines, may cause sedation and [ll) Answer A. Hypernatremia is almost always caused
hypotension. Dextromethorphan may cause an opi- by the loss of free water and rarely by sodium
oid toxidrome with sedation, constricted pupils, and gain (which is usually iatrogenic). Regardless of the
respiratory depression. Diphenhydramine causes an underlying cause, it almost never occurs in alert
anticholinergic state, with sedation, tachycardia, dry patients with an intact thirst mechanism. It is a
mucous membranes, and mydriasis. Ketamine causes known complication of multiple doses of activated
a dissociated state of altered mental status with pre- charcoal, occurring in 6% of patients receiving such
served respiratory reflexes. therapy. Treatment with saline should only occur
in patients who have hemodynamic compromise.
~ Answer A. The most common complication of Almost all patients, however, require treatment with
mitral stenosis (MS) is atrial fibrillation, which puts half-normal saline or more dilute solutions with
the patient at high risk for thrombus formation and lower tonicity. The main complication of therapy
embolism. Atrial fibrillation occurs due to the severe is cerebral edema, so the rate of correction should
atrial hypertrophy that results from the stenosed be 0.5 to 1.0 mEq/L/hour on average (though it
mitral valve preventing flow into the left ventricle. may be more rapid for the initial few hours if the
patient is suffering life-threatening complications of
~ Answer C. The development of a lung abscess is hypernatremia). Central pontine myelinolysis is a
most commonly a consequence of aspiration in pa- complication of therapy for hyponatremia.
tients with poor oral hygiene. Less commonly, it
may occur as a result of necrotizing pneumonia. [!!) Answer C. More than 90% of cases of hypercal-
Classically it has been thought that anaerobic bac- cemia are caused by primary hyperparathyroidism
teria are responsible for the great majority of lung or malignancy. Most cases of primary hyperparathy-
abscesses. However, recent studies have revealed that roidism are caused by benign parathyroid adenomas.
the microbiology may differ between immunocom- Malignant causes of hypercalcemia occur either
petent and immunocompromised patients. Although through osteolytic metastatic lesions as in breast can-
anaerobes are predominant in immunocompetent cer or melanoma or through secretion of parathyroid
patients, patients with depressed immune systems hormone-related peptide (PTHrP) as in squamous
are more frequently infected with aerobic bacteria cell carcinoma or renal cell carcinoma.
such as S. aureus, Pseudomonas aeruginosa, Klebsiellll
pneumoniae, and H. influenzae. Frequently, however, [!!) Answer E. Lymphedema or lymphatic disruption
infections are polymicrobial. Nonbacterial organ- as occurs after mastectomy or saphenous vein harvest
isms such as fungi and parasites may also cause puts patients at increased risk for infection. Patients
lung abscesses. S. pneumococcus is not a common with diabetes have an increased susceptibility to cel-
cause. Patients with anaerobic or fungal lung ab- lulitis due to their generalized immunocompromised
scesses typically experience an indolent course of state. Drug abusers engaging in skin popping may de-
fever, productive cough, night sweats, anorexia, and velop cellulitis due to a variety of unusual pathogens
weight loss. The sputum is classically malodorous, in addition to S. aureus.
which should be a clue to the diagnosis. Immuno-
compromised patients presenting with a lung abscess [ig) Answer E. Rust rings are the result of iron-
as a result of aerobic necrotizing pneumonia present containing foreign bodies leaving a residue on the
with acute symptoms ofpneumonia and may be quite cornea. Patients should see the ophthalmologist
ill. Chest radiography typically reveals a cavitary le- within 48 hours, as the ring will migrate to more
sion with an air-fluid level. Antibiotics remain the superficial corneal layers over time, allowing for
cornerstone of therapy although surgical manage- easier removal. Topical steroids have no role in the
ment may ultimately be necessary. Typical surgical management of rust rings or ocular foreign bodies.
indications are medical treatment failure, suspected Metallic foreign body evaluation of the eye with MRI
cancer, or congenital lung malformation. Lung ab- is absolutely contraindicated.
scesses very rarely develop in the setting of pediatric
community-acquired pneumonia (CAP) and only [j!) Answer B. Digitalistoxicitycauseslethaldysrhyth-
occur typically in patients who are immunocom- mias, and treatment involves management of elec-
promised because of acquired immune deficiencies, trolytes, digitalis antibody (Fab) fragment therapy,
malignancy or chemotherapy, and in patients who are and dialysis. The most important indications for
predisposed to aspiration (children with neurologic Fab fragment therapy in acute digitalis toxicity
246 1000 Questions to Help You Pass the Emergency Medicine Boards

are hyperkalemia, ventricular dysrhythmias, and thought to appear within 2 hours of subarachnoid
co-ingestions of other cardiotoxic drugs. Elevated hemorrhage (SAH) achieving a peak concentration
digoxin level and massive ingestion are other relative between 24 and 46 hours. Bilirubin does not appear
indications, but usually mandate additional rhyth- until approximately 10 hours after SAH. Although
mic disturbance to warrant Fab fragment therapy. the presence of xanthochromia remains the most
Supraventricular dysrhythmias are only an indication reliable method for differentiating between a trau-
for Fab fragment therapy if the patient is hemo- matic lumbar puncture and an SAH, its presence is
dynamically unstable. Magnesium level aberrations not pathognomonic for SAH. Recent studies have
may exacerbate hypokalemia or hyperkalemia, but in demonstrated that xanthochromia begins to develop
the absence of potassium abnormalities, they do not immediately after mixing of blood and CSF as seen
constitute an indication for Fab fragment therapy. in traumatic lumbar puncture. The degree of xan-
thochromia correlates with the amount of bleeding
~ Answer B. With the history of a recent camping induced by trauma. Clinically, recent studies have
trip, tick-borne illness should be suspected. In this established that in the presence of a traumatic lum-
case, the patient likely has Rocky Mountain spotted bar puncture and an RBC concentration of 10,000
fever, caused by the intracellular bacterium, Rick- RBC per 1!-L, xanthochromia cannot be reliably used
ettsia rickettsii. The organism causes endothelial cell to confirm SAH. Conversely, xanthochromia in the
damage, resulting in diffuse vasculitis and multiple setting ofRBC concentrations <5,000 RBC per t!-L is
organ microinfarctions and failure. Acute respira- a reliable indicator of SAH. The method of compar-
tory distress syndrome, disseminated intravascular ing RBC counts in the first and fourth tubes has been
coagulation (DIC), and shock all may occur in se- shown to be an unreliable means of differentiating
rious cases. Rash on the wrists and ankles following these two entities. None of the other methods listed
a nonspecific viral-like syndrome is characteristic. has been shown to be reliable.
Laboratorywork is usually normal, though thrombo-
cytopenia may occur. Treatment is with doxycycline ~ Answer D. Disseminated intravascular coagula-
or chloramphenicol. Choices A, C, D, and E will not tion (DIC) is an acquired consumptive coagulopathy
adequately treat the rickettsial infection. which may occur in patients who are critically ill.
Disorder of the clotting cascade causes platelets
~ Answer B. Neutropenic fever patients are at con- and clotting factors to be consumed, leading to
siderable risk for serious bacterial infection and both hemorrhagic and thrombotic complications.
require broad-spectrum antibiotics. Either gram- Almost all have decreased levels of clotting factors,
positive (Streptococcus or Staphylococcus species) leading to prolongation in the prothrombin and
or gram-negative (Pseudomonas, E. coli, and oth- partial thromboplastin times. Fibrinolysis occurs
ers) organisms can be responsible for infection. commonly, leading to decreased fibrinogen levels
Cefepime monotherapy is an appropriate initial an- and an increase in fibrin split products. Acute ane-
tibiotic choice, covering all organisms except for mia may not always occur, as some patients have
some strains of methicillin-resistant Staphylococcus relatively low hemorrhagic burden and hemoglobin
aureus (MRSA). Vancomycin may be added to any levels may take time to equilibrate and not show
antibiotic regimen to adequately cover MRSA in subtle, acute decreases.
hospitals with a high incidence of this pathogen.
Doxycycline covers some gram-positive organisms [!!] Answer B. The most commonly affected joints in
and atypicals, but does not adequately cover gram septic arthritis are the knee (40% to 50%), hip (13%
negatives. Clindarnycin does not cover gram-negative to 20%), shoulder (10% to 15%), ankle (6% to 8%),
organisms. The combination of metronidazole and wrist (5% to 8%), and elbow (3% to 7%).
either gentamicin or aztreonam leaves out adequate
gram-positive coverage. ~ Answer E. Compartment syndrome refers to is-
chemia that occurs in the extremities when pressure
~ Answer B. Xanthochromia refers to the presence in the soft tissues exceeds that ofthe microcirculation.
of a yellowish color to the supernatant of centrifuged Such high pressures occur from either increased con-
cerebrospinal fluid (CSF) samples. It results from the tents in the compartment or external compression.
breakdown of hemoglobin first to oxyhemoglobin Normal compartment pressure is zero, and pressures
and then to bilirubin. Methemoglobin may also be >30 mm Hg are usually enough to predispose to
produced, but, like bilirubin, it occurs after oxyhe- compartment syndrome. Long bone fractures are the
moglobin is generated. These latter molecules have a usual reason for compartment syndrome, causing
yellowish tint and characteristic spectrophotometric extravasated blood and soft tissue edema to accu-
absorption curves. Traditionally, oxyhemoglobin was mulate. The most common symptoms are pain and
Test8 247

paresthesias. Diminished pulses generally occur only ~ Answer A. This patient has Henoch-Sch6nlein
in extremely advanced cases, as pressure in com- purpura (HSP), a small-vessel vasculitis that pri-
partment syndrome is usually well below arterial marily affects children who present with palpable
pressure. For this reason, ankle-brachial indices are purpura, arthralgias, abdominal pain, and glomeru-
usually normal in compartment syndrome unless lonephritis. Purpura is present in 100% of patients,
there is simultaneous arterial insufficiency. Outpa- although 75% of patients have arthralgias, typically
tient follow-up in cases of suspected compartment of the ankles, 65% have abdominal pain, and 40%
syndrome is contraindicated. Imaging may help de- have renal involvement. Prognosis depends on the
lineate the cause of the compartment syndrome, but presence and severity of renal involvement. Colicky
diagnosis is still clinical. Diagnosis is made by directly abdominal pain is the most common gastrointesti-
measuring compartment syndromes with a Stryker nal manifestation, though vomiting, bleeding, and
needle device. Treatment is with urgent fasciotomy. more rarely, intussusception, may occur. In contrast
to the typical ileocolic intussusception that occurs
~ Answer B. Fat embolism can occur after fracture in the general population, patients with HSP ex-
of any long bone, usually in the lower extremities. Fat perience ileoileal intussusception 70% of the time.
droplets from the bone marrow reach the systemic In the absence of renal disease, HSP is self-limited
circulation and can embolize to the lungs, brain, and only supportive care is required. In the setting
eyes, and extremity microvasculature. Respiratory of hematuria or proteinuria, corticosteroids may be
distress, altered mental status, fever, and petechiae beneficial but renal consultation should be sought.
commonly occur. Diagnosis is aimed at ruling out When present in adults, HSP is a much more severe
other causes ofsymptoms, and treatment is primarily disease due to the increased frequency and severity
supportive. Meningococcemia can occur acutely and of nephritis. (Figure reprinted with permission from
cause altered mental status and petechiae, but the Fleisher GR. Atlas of pediatric emergency medicine.
history of the leg fracture and the respiratory distress Lippincott Williams & Wilkins; 2003.)
point away from this. Pulmonary embolism (PE)
due to thrombus can occur in the posttrauma [!ID Answer B. Significant renal injury in blunt trauma
patient but usually occurs later in the course of rarely occurs in the absence of other organ injury.
the recovery. Pneumothorax and pneumonia in Gross hematuria is the standard indication for further
a previously healthy patient are unlikely to cause evaluation of the urologic tract, and CT scan with
altered mental status or petechiae. N contrast is the best imaging tool to evaluate
renal injuries. In stable patients with microscopic
[!!) Answer D. hematuria due to blunt trauma, significant renal
injury almost never occurs and should not be
~ Answer C. Coral snakes are part of the family, pursued. Penetrating trauma to the back or abdomen
Elapidae, whereas the remaining snakes listed are may cause renal injury in the absence of either gross
part of the family, Crotalidae. Elapidae venom is or microscopic hematuria and should aggressively
neurotoxic, as several of the venom components be evaluated, either with CT scan or laparoscopy.
block acetylcholine transmission. In contrast to The vast majority of renal injuries do not require
victims of pit viper envenomation, victims of coral operative management; exceptions include large
snake envenomation usually experience minimal renal lacerations and major vascular injuries. The
pain and swelling at the bite site. However, signs kidneys are among the most commonly injured
of neurotoxicity may develop rapidly or be delayed abdominal organs in blunt abdominal trauma in
for up to 12 hours. Ptosis is frequently the initial sign children.
of neurotoxicity and may be followed by delirium,
tremors, drowsiness, hypersalivation, and multiple ~ Answer A. The kidneys are almost wholly respon-
cranial nerve abnormalities (dysarthria, diplopia, sible for magnesium excretion and are able to
and dysphagia). In severe envenomations, respiratory enhance excretion in the setting of a magnesium
muscle paralysis occurs leading to respiratory failure load. Therefore, in the absence of renal insuffi-
and death. ciency, hypermagnesemia rarely occurs. Abuse of
magnesium-containing laxatives may cause a tran-
~ Answer B. Parental smoking, male gender, bottle sient increase in magnesium levels but will not persist
propping, Down's syndrome and facial anatomic in the setting of normal renal function. Trauma could
abnormalities are the commonly implicated risk feasibly lead to hypermagnesemia if associated with
factors for developing otitis media. rhabdomyolysis.
248 1000 Questions to Help You Pass the Emergency Medicine Boards

~ Answer E. The patient presents after benzodi- which is targeted to identify immediate life threats
azepine overdose and may or may not have co- first and other injuries later. Cervical spine radio-
ingested alcohol. The history of chronic alcohol use graphs may be left until after the secondary survey,
and altered mental status dictates the use ofthiamine assuming appropriate spine precautions are used
therapy, along with folate, multivitamin, magne- when moving the patient. CT scans should not be
sium, and dextrose. Fomepizole is indicated only in initiated until the primary and secondary surveys
cases of toxic alcohol poisoning. Flumazenil, a spe- are complete, except in special circumstances of
cific benzodiazepine antagonist, is contraindicated isolated, severe head injury.
here, as it can precipitate withdrawal seizures in pa-
tients who are chronically using benzodiazepines. ~ Answer E. Hypothermia is one of the most com-
Physostigmine is an acetylcholinesterase inhibitor mon manifestations of severe hypothyroidism (e.g.,
used in selected patients with anticholinergic toxic- myxedema) although body temperature is rarely
ity. Glucagon is used in patients with ,8-block.er and <95°F. In the setting of myxedema, a "normal"
calcium channel blocker toxicity. temperature should trigger a search for a focus of in-
fection. Nonpitting edema is due to hyaluronic acid
~ Answer D. Pneumothorax, pneumomediastinum, deposition and is initially found in the periorbital
subcutaneous emphysema, and subconjunctival region. Pseudomyotonic or "hung up" reflexes are
hemorrhage are all complications related to the acute another common finding (delayed relaxation phase
elevations in intrathoracic pressure during an asthma of deep tendon reflexes). Paresthesias are present
exacerbation. Pressures may be further exaggerated in >80% of patients and median nerve neuropa-
by fits of coughing which may accompany an asthma thy (carpal tunnel syndrome) is the most common
exacerbation. Mis may occur in patients who have manifestation.
underlying cardiac disease, as severe asthma exac-
erbations place an extensive demand on the heart. [!!) Answer C. The x-ray demonstrates a Salter-
Furthermore, patients with severe asthma attacks Harris III fracture of the distal radius. The Salter-
may be hypoxic for a period of time resulting in Harris classification is used to describe pediatric long
cardiac ischemia as oxygen supply is outstripped by bone fractures near the growth plate. Type I frac-
demand. Pulmonary emboli are not related to asthma tures go through the physis only, type II from the
exacerbations. metaphysis into the physis, type III from the epi-
physis into the physis, type N is a combination of
~ Answer A. After the primary survey, radiographs types II and III, and type V is a crush injury to
of the chest and pelvis are indicated to rule out the physis. The most common is type II. Types I
important causes of immediate death, including and V may be invisible on initial plain films. Type V
pneumothorax, hemothorax, and pelvic fracture. carries the poorest prognosis. (Figure courtesy of
Wtrasonography in the form of a focused assess- Mark Silverberg, MD. Reprinted with permission
ment of sonography in trauma (FAST) scan may from Silverberg M. Greenberg's text-atlas ofemergency
also be performed to evaluate for significant in- medicine. Lippincott Williams & Wilkins; 2004.)
traperitoneal hemorrhage. After these initial studies
are performed (or in conjunction with them) the [100] Answer C. Patients with HHS have a larger fluid
secondary survey is conducted to identify injuries deficit and more significant potassium deficiency
that may cause significant morbidity without mor- than patients with diabetic ketoacidosis (DKA).
tality. Obvious external injuries may distract the Although seizures may occur in HHS, the most
trauma leader from identifying the immediate life common immediate life threat is hypovolemic shock.
threat. In this patient's case, the broken right an- Hypokalemia is the next most serious immediate risk
kle, though impressive, is unlikely to be the cause of to patients with HHS. Thromboembolic events may
death. If the initial chest x-ray is omitted, however, occur in either DKA or HHS, but more commonly
the potential pneumothorax missed on physical ex- complicate HHS. Thromboembolic events occur
amination may be lethal. This is the main reason as a result of severe dehydration and resulting
for the stepwise, algorithmic approach to trauma hyperviscosity.
Test 9
Questions
mWhich of the following is true regarding adult
epiglottitiJ?
(B) The patient should immediately receive
crotalidae polyvalmt immune fab orine
(CroFab) antivenin.
(A) Airway obstruction is usually caused by
(C) The wound should be irrigated with sterile
inflammation of the infraglottic tissues.
saline and a sterile suction catheter to help
(B) ~rooling and stridor are infrequent presenting
remove venom.
5J811S.
(D) The patient should immediately receive North
(C) The disease is more common in winter.
American coral snake antivenin.
(D) Nebulized racemic epinephrine has been
(E) The patient should receive an intramuscular
shown to dec:reaae the need for intubation.
(E) Normal lateral neck x-rays can safely exclude mixture of antivenins as close to the bite site as
epiglottitis. possible.

(I] A 46-year-old male golfer suffered a snakebite on his [!) A 74-year-old man with a history of hyperlipidemia
is brought in by emergency medica.l services (EMS)
hand while he was looking for his golf ba11 in the
with an acute ischemic ri8bt hemispheric stroke.
brush. His friend snapped a picture of the make and
Soon after returning from CI', he has a generalized
brinp it in for you to examine. He has ahnost no
seizure which terminates without treatment after
pain or swelling at the bite site but he presented fur
1 minute. Which of the following is true about this
evaluation as he had never been bitten by a snake
patient?
before. Which of the following is the nat best step
in management (Fig. 9- l)l (A) The patient should have been treated with
prophylactic phenytoin as soon as the diagnosis
ofischemic stroke was made.
(B) The patient should be given a loading <be of
phenytoin after ru.. seizure.
(C) Statw epilepticus occurs more commonly in
the setting of ischemic strokes than in other
settings.
(D) Phenytoin is contraindicated in patients with
ischemic stroke due to its potential for causing
ataxia.
(E) Although isolated seizures are common in
patients with ischemk strokes, treatment with
antiepileptic drugs is unnece&Sary because
recurrence is uncommon.

ffi What is the half-life of carboxyhemoglobin with a


100% oxygen nonrebreather mask?
(A) 6hours
(A) The patient should be observed fur a minimum (B) 3hours
of6 hours and then disc:hargl:d if he remains (C) 90minutes
asymptomatic.

249
250 1000 ~ 10 Hdp You Plw tile~ MetlicW .Bomds

(D) 60 minutes
(B) 30 minutes

[!) Rmobulbar hemorrhage can result in what devastat-


ing complication?
(A) Corneal abrasion
(B) Hypopyon
(C) Central retinal artery occlusion (CRAO)
(D) Hyphema
(B) Corneal ulcer

(]] The pictured deformity in Fig. 9-2 occun in:

Figure t-3.

figure t-2.
(!] A 27-year-old man presents to the ED with pruritus
ani., tenesmus, and yellowish mucoid discharge
from his rectum. Upon further questioning, he
(A) Rheumatoid arthritis. acknowledges recent unprotected anal intercourse.
(B) Osteoarthritis. The recommended regimen for treating this patient
(C) Systemic lupua erythematosus. is
(D) Reiter's syndrome.
(A) Topical podophyllin b.i.d x 7 days.
(B) Psoriatic arthritis.
(B) Ceftriaxone 125 mg 1M plus doxycycline
rn Which of the following is an ingredient in sitz 100 mg PO b.Ld. x 7 days.
(C) Valacyclovir 1 gPO daily x 5 days.
bathsf
(D) Benzathine penicillin G 2.4 million units 1M
(A) Baking soda xl dose.
(B) Salicylate (E) Lopinavir.
(C) Corticosteroids
(D) Insoluble fiber (!g) Patients with inflamm.atory bowel disease (lBO)
(B) None of the above may develop which of the following extraintestinal
complications?
[!) A 12-year-old boy presents with progressive tes-
ticular swelling. Testicular examination is shown (A) Arthralgiaa
(B) Episcleritis
in Figure 9-3. Which of the following is the most
(C) Cholelithiasis
common complication of this conditionf
(D) Pyodermagangrenosum
(A) Testicular torsion (E) All of the above
(B) Epididymitis
(C) Infertility (!!]A 62-year-old man ~th a history of ~c
(D) Malignancy alcoholism presents With acute-onset epigastric
(B) Deep venoua thrombosis
Test9 251

abdominal pain. Initial labs reveal a white blood cell [H) Which of the following deficits below the level of
(WBC) count of 19,000 per mm3, a normal chem- injury is consistent with an anterior cord syndromd
istry with a glucose value of 168 mg per dL, a lactate
(A) Loss of total sensation and motor function.
dehydrogenase (LDH) of 400 IU per L, liver enzymes
(B) Bladder and bowel incontinence and loss of
demonstrating an aspartate aminotransferase (AST)
motor function.
of 137 SF units and an alanine aminotransferase
(C) Loss of motor function only.
(ALT) of 94 SF units, and a lipase level that is five
(D) Loss of vibration and position sensation and
times the lab's upper limit ofnormal. You admit him
motor function.
with a diagnosis of acute pancreatiti.s. Which of the
(E) Loss of pain and temperature sensation and
following is true?
motor function.
(A) He has 0 Ranson's criteria upon admission.
(B) He has 1 Ranson'a criteria upon admission. ~ Which of the following represents the approximate
(C) He has 2 Ranson's criteria upon admission. proportion of acute myocardial infarctions (Mls)
(D) He has 3 Ranson's criteria upon admission. that occur without a history ofchest painf
(E) His elevated WBC count suggests an infectious
(A) 1%
etiology
(B) 5%
(C) 10%
~ The most common complication of peptic ulcer
(D) 25%
disease (PUD) is
(E) 50%
(A) Perforation.
(B) Gamic adenocarcinoma ~ A 22-year-old man presents with an episode of left-
(C) Gaatrointestinal bleeding. sided chest pain that lasted 4 hours, but resol:wd an
(D) Obstruction. hour ago. The pain was dull and radiated to his left
(E) Penetration into an adjacent organ. shoulder. The patient states that he used cocaine at
a party 12 hours ago. Physical aamination is unre-
~ Which of the following is true regarding treatment m.arbble and vital signs are normal. The electrocar-
for acute aspirin tmici¥ diogram (EKG) is shown in Figure 9-4. Which of the
(A) A urinary pH goal of 7S to 8 is desirable. following ia most appropriate at this time?
(B) Forced diuresis is an effective adjunctive (A) Nitroglycerin
therapy. (B) Metoprolol
(C) Activated charcoal is ineffective. (C) Morphine
(D) Whole bowel irrigation is contraindicated. (D) Aspirin
(E) Hemodialysis plays no role in management. (E) Tissue pluminogen activator (tPA)

Plgure t -4. (See color insert.)


25 2 1000 Questions to Help You Pass the Emergency Medicine Boards

(g) A 3-year-old boy had a ventriculoperitoneal shunt (D) 50%.


(VPS) placed at 22 months due to hydrocephalus. It (E) >99%.
has not been revised since then and he has had no
problems with it. He is now brought to the ED with a ~ Which of the following is true regarding post-MI
history of fever of 38.2° C, headache, and "fussiness." pericarditis?
Which of the following is true about this patient? (A) It occurs in a majority of patients with MI.
(A) The risk ofVPS infection rises 1 year after (B) Concave ST-elevations are usually seen on
insertion. EKG.
(B) Urgent lumbar puncture is indicated. (C) Treatment generally involves nonsteroidal
(C) The mortality of a VPS infection is roughly anti-inflammatory drug (NSAID) therapy.
75%. (D) A pericardia! friction rub is almost never
(D) Hydrocephalus on Cf scan rules out the audible.
presence ofVPS infection. (E) Etiology is likely infectious.
(E) Most patients with a VPS infection have
peripheral leukocytosis. ~ After physiologic jaundice of the newborn (icterus
neonatorum), which of the following is the most
~ An 85-year-old woman presents with right shoulder common cause of neonatal jaundice?
stiffness. She was diagnosed with a shoulder sprain 3 (A) Breast milk jaundice
weeks ago after a fall, and has been wearing a sling (B) Cephalohematoma
since then. Physical examination demonstrates an (C) Sickle cell anemia
afebrile patient with restricted range ofmotion ofthe (D) Gilbert's syndrome
shoulder in all directions with mild pain. Which of (E) Biliary atresia
the following is the most likely diagnosis?
(A) Rotator cuff tear ~ Erosion into the carotid artery is most commonly a
(B) Adhesive capsulitis complication ofwhich of the following?
(C) Septic arthritis (A) Ludwig's angina
(D) Associated scapular fracture (B) Parapharyngeal abscesses
(E) Rheumatoid arthritis (C) Peritonsillar abscesses
(D) Retropharyngeal abscesses
~ Which of the following is the most common serious (E) Epiglottitis
complication of the edrophonium (Tensilon) test?
(A) Bradycardia ~ A 55-year-old man presents with right arm and leg
(B) Atrial fibrillation weakness and left-sided facial droop. Which of the
(C) Oculogyric crisis following arteries is most likely involved?
(D) Cough (A) Anterior cerebral artery
(E) Seizure (B) Anterior communicating artery
(C) Middle cerebral artery
~ Which of the following is associated with carpal (D) Posterior cerebral artery
tunnel syndrome? (E) Basilar artery
(A) Hypertension
(B) Diabetes ~ Which of the following is the most common arrhyth-
(C) Congestive heart failure mia in patients with pulmonary embolism (PE)?
(D) Coronary artery disease (A) Multifocal atrial tachycardia
(E) Osteogenesis imperfecta (B) Sinus tachycardia
(C) Atrial fibrillation
~ A 50-year-old man develops acute onset of severe (D) Sinus rhythm with atrial premature
right flank pain. A CT scan demonstrates a passed contractions
kidney stone in the bladder. The patient has never (E) Ventricular fibrillation
had a kidney stone before. He asks you what his risk
of getting another stone is. You tell him that the ~ Which of the following is true regarding the use of
lifetime risk of recurrence is approximately: corticosteroids in asthma management?
(A) <1%. (A) IV corticosteroids have been proved to be of
(B) 10%. greater efficacy than oral corticosteroids.
(C) 25%.
Tm9 253

(B) Discharged patients who have received (C) Glaucoma


systemic corticosteroids in the ED require (D) Lens dislocation
tapered oral corticosteroids therapy as an (E) Comeal ulcer
outpatient for up to 10 clays.
(C) Long-term systemic corticosteroid use may be I!ID Which ofthe following represents the correct medial
complicated by weight gain, aseptic necrosis of to lateral configuration of permanent teethr
the femur, and peptic ulcer disease (PUD). (A) Canine, central incisor, lateral incisor,
(D) Inhaled corticosteroids are not useful for premolar, molar
long-term asthma control. (B) Central incisor, lateral incisor, premolar,
(E) The onaet ofaction ofIV corticosteroids is molar, canine
within 1 hour. (C) Central incisor,lateral incisor, canine,
premolar, molar
(i!) Which of the following patients with febrile seizure (D) Premolar, molar, canine, lateral incisor, central
should receive a lumbar puncture to look for incisor
infectious source? (E) Premolar, molar, central incisor,lateral
(A) A 10-month-old incisor, canine
(B) An 18-month-old
(C) A 2-year-old ~ Which ofthe following is true regardingperimortem
(D) A 4-year-old cesarean section?
(E) A 5-year-old (A) The fetus should be delivered within 5 minutes
of maternal cardiac arrest.
(i!) A 5-year-old boy presents with confirmed rotaYirus (B) Family consent should be obtained before the
diarrhea. He is tachycardic and lethargic with sunken procedure.
eyes, poor skin turgor, and dry mucous membranes. (C) It should only be performed ifthe fetal age is
Which of the following is the most appropriate next determined to be > 20 weeks' gestation.
step in managementf (D) A low horizontal abdominal incision affords
(A) 0.45 NS 100 mL per hour drip the best opportunity for fetal recovery.
(B) 0.45 NS 10 mL per kg bolus (E) A lateral approach is best in cases ofa suspected
(C) 0.9 NS 100 mL per hour drip anterior placenta.
(D) 0.9 NS 10 mL per kg bolus
(E) 0.9 NS 20 mL per kg bolus ~ A 6-year-old boy presents with left hip pain and
limp. There is no history of trauma. The pain is
~ The most common cause of death in recipients of a relieved by rest. Plain radiographs are shown in
solid-organ transplant is Pig. 9-5. Which of the following is true regarding
(A) Recurrent organ failure. this condition?
(B) Infection.
(C) Drugtcmcity.
(D) Organ rejection.
(E) Effects of the primary disease process.

~ A ~year-old man with AIDS presents with chronic


diarrhea. flatulence, and generalized malaise for one
month. Which of the following is the most likely
cause ofhis symptoms?
(A) Cryptosporidium spp.
(B) Campylobaaer spp.
(C) Giardia IRmblia
(D) Escheri&hia coli
(E) Entmnnorw hominis

~ Which of the following is the major complication of


ischemic central retinal vein occlusion (CR.VO)?
(A) Conjunctivitis
(B) Iritis
FigllNI-5.
254 1000 Qwstimu to Hdp You Ptw the Emerpnt;y Maicitw Bo.rmfs

(A) It is much more common in boys than in girls. ~ A 22-year-old man presents with forearm pain after
(B) It is usually bilateral. being assaulted. Radiograph& demonstrate a proximal
(C) Etiology is viral in half of all cues. ulnar fracture with dislocation of the radial head.
(D) Almost all patients require sUIBical fi.J:ation. Which of the following is the most likely nerve
(E) Joint aspiration confirms the diagnosis. injuryf
(A) Median
Iii] A 22-year-old woman presents with a severe sore
(B) Radial
throat and difficulty swallowing. Her physical ewn·
(C) Ulnar
ination is consiatent with pharyngitia. Which of the
(D) Axillary
following criteria make group A streptococcus (GAS)
(E) Sciatic
more likely as a cause of her illness?
(A) Tender anterior cervical lymphadenopathy ~ Which of the following is true regarding osteomyeli-
(B) Concomitant otitis media tis?
(C) Nonexudative tonsillitis
(A) Patients usually appear toxic.
(D) The presena: ofa cough
(B) The sensitivity oferythrocyte sedimentation
(E) Increased atypical lymphocytes on her
rate (ESR) is 50%.
peripheral blood smear (C) The sensitivity of radiographs is higher early in
the illness than later on.
li!) A 62-year-old man presents with right eyelid swelling
(D) cr is superior to MRI for diagnosis.
and crusting. He reports no pain or redness in the
(E) Stllphylocoa:us aureus is the most common
eye itself. Physical eumination of the eyelid is shown
cause.
in Figure 9-6. Which of the following is the most
appropriate therapy?
~ Which of the following is true regarding intussus-
a:ption?
(A) Thirty percent of all cases of intussusc:eption
occur in adults.
(B) Children with intussusception are more likely
to have an anatomic abnormality of the
intestine than adults.
(C) The classic triad of abdominal pain, vomiting,
and bloody stools occurs in the majority of
patients.
(D) Ultrasonography is the most useful noninvasive
means ofdiagnosing intussusception.
(E) All of the above.

[4iJ Of the following. which is the strongest risk fActor


for an ectopic pregnancy?
(A) Prior ectopic pregnancy
figure 9-6.
(B) HistoryofpeMcinHammatorydisease (PID)
(C) Current use of an intrauterine device (IUD)
(D) Prior C-sect.ion
(A) Topical erythromycin (E) Oral contraceptives
(B) Topical prednisolone
(C) Topical proparacaine ~ A 23-year-old woman with sickle cell disease presents
(D) Intravmoua ceftriamne with pain in her right shin and fners for 2 weeks. She
(E) Intravenous acetazolamide never has leg pain with her sickle cell pain crises. An
x-ray demonstrates evidence of osteomyelitis. Which
~ The main problem posed by breech presentations is of the following is the most likely etiologic agent?
(A) Inadequate cervical dilation. (A) S. atmUS
(B) Entrapment of the fetal head. (B) SalmcmeJUa
(C) Umbilical cord prolapse. (C) Aspergillw
(D) Fetal spinal cord injuriea. (D) Neisseria gonorrhoeae
(E) All of the above. (E) Pseudomoruu
Test9 255

~ Which of the following is true regarding myasthenia ~ Which of the following is true regarding gastroe-
gravis (MG) and Lambert-Eaton myasthenic syn- sophageal reflux disease (GERD) in infants?
drome (LEMS)? (A) Most infants fail to respond to conservative
(A) Ocular muscle weakness is the most common measures such as smaller, thickened feedings
initial presentation in both MG and LEMS. and frequent burpings.
(B) Autonomic dysfunction is a common finding (B) Vomiting is typically nonbilious and
in MG but not in LEMS. progressive, resulting in projectile emesis.
(C) Colon cancer is the most common neoplastic (C) Ranitidine and metoclopramide are the
disease associated with LEMS. mainstays of medical therapy.
(D) The distinguishing feature ofLEMS is proximal (D) Most infants with GERD ultimately suffer from
muscle weakness that is most prominent in the failure to thrive.
lower extremities. (E) Infant GERD typically persists into adulthood.
(E) Deep tendon reflexes (DTRs) in both MG and
LEMS tend to be preserved. ~ Which cardiac chamber is most commonly injured
in penetrating thoracic injury?
~ Which of the following is more characteristic of (A) Right atrium
subdural hematoma than epidural hematoma? (B) Left atrium
(A) Lucid interval (C) Right ventricle
(B) Coma (D) Left ventricle
(C) Focal neurologic deficits (E) All chambers are injured equally as often.
(D) Increased intracranial pressure
(E) Delayed presentation ~ Which of the following is a criterion for the systemic
inflammatory response syndrome (SIRS)?
~ Which of the following is true regarding traumatic (A) Systolic BP <90
hemothorax? (B) Diastolic BP <60
(A) Continuous chest tube output of 300 mL per (C) HR<90
hour for 4 hours is an indication for (D) Temperature <36°C
thoracotomy. (E) RR <20
(B) Costophrenic angle blunting occurs on upright
chestx-raywith as little as 50 mL of ~ A mother brings her 3-year-old daughter to the ED
intrapleural blood. for evaluation of a persistent, foul-smelling, bloody
(C) The subclavian artery is the most common vaginal discharge. The patient has been complaining
source ofbleeding. of "itching down there" and her mother noted that
(D) A 7-French pigtail catheter is adequate for she has been frequently placing her hands in her
drainage of most hemothoraces. perineal region. Which of the following is the most
(E) Pneumothorax almost never occurs likely cause of her symptoms?
concomitantly with hemothorax. (A) Trichomonas vaginitis
(B) Poor perineal hygiene
~ Which of the following occurs in most patients with (C) Dysfunctional uterine bleeding (DUB)
myocarditis? (D) Candidalvaginitis
(A) Chest pain (E) Vaginal foreign body
(B) Fever
(C) Antecedent viral syndrome ~ A 32-year-old woman presents with recurrent
(D) S4 heart sound episodes of headaches, palpitations, and profuse di-
(E) Leukocytosis aphoresis. Her primary care doctor diagnosed her
with an anxiety disorder but various selective sero-
~ Which of the following effects does digitalis exhibit tonin reuptake inhibitors have been ineffective. In the
attherapeuticleveh? ED, her vital signs include a temperature of 99.0°F,
(A) Decreases intracellular calcium a pulse of 90, and a blood pressure of 175/100.
(B) Decreases intracellular sodium Which of the following is the best agent to treat her
(C) Increases intracellular potassium hypertension?
(D) Increases heart rate (A) Metoprolol
(E) T -wave inversion (B) Hydrochlorothiazide
25 6 1000 Questions to Help You Pass the Emergency Medicine Boards

(C) Phenoxybenzamine (D) Central cord syndrome


(D) Nifedipine (E) All of the above
(E) Enalapril
~ A 22-year-old primigravida presents to your commu-
~ A concerned mother brings her 15-year-old daughter nity ED at 34 weeks' gestation with a chief complaint
to the ED with a chief complaint of irregular vaginal of headache and mild crampy abdominal pain. Her
bleeding. The patient experienced menarche at age blood pressure is 160/100. Suspecting preeclampsia,
13 and has never had regular periods. Recently, the you start a magnesium drip. While the patient is
bleeding has been heavier and more irregular than awaiting transfer 2 hours later, the nurse alerts you
normal. She reports no history of easy bruising and that she believes the patient is magnesium toxic.
has no petechiae on examination. Her hemoglobin is Which of the following is a sign of magnesium
11 g per dL. Which ofthe following is the most likely toxicity?
cause of her symptoms? (A) Atrial fibrillation
(A) Hyperthyroidism (B) Increased deep tendon reflexes
(B) Anovulation (C) Somnolence
(C) Endometriosis (D) Hyperventilation
(D) Asymptomatic Chlamydia infection (E) Diarrhea
(E) Prolonged luteal phase (Halban's disease)
~ Which of the following findings is seen in most
~ Which of the following can be prevented by patients with meningococcemia?
treating group A beta-hemolytic streptococcal throat (A) Bilateral adrenal infarction
infection (GAS pharyngitis) with antibiotics? (B) Skin lesions
(A) Erythema marginatum (C) Hypothermia
(B) Endocarditis (D) Seizure
(C) Migratory arthritis (E) Arthritis
(D) Glomerulonephritis
(E) A, B, and C only ~ Which of the following is most useful in differenti-
ating a patient with acute cholangitis from a patient
~ Which of the following antibiotics is most likely to with acute cholecystitis?
cause aplastic anemia? (A) Jaundice
(A) Vancomycin (B) Fever
(B) Piperacillin-tazobactam (C) Abdominal tenderness
(C) Doxycycline (D) Leukocytosis
(D) Chloramphenicol (E) Murphy's sign
(E) Azithromycin
~ A 45-year-old man presents after a high-speed motor
~ A 14-year-old boy comes to the ED for a diffuse rash. vehicle crash. He was the unrestrained driver going
He was in his primary care doctor's office four days at 70 mph and rear-ended a car stopped in front of
prior for a sore throat and fatigue and was told he had him. He remembers that he struck his chest on the
a viral throat infection. The patient's mother insisted steering wheel and notes that his car lacks an airbag.
that the patient be given an antibiotic, which he has He complains of severe chest pain. His primary
been taking regularly since then. What antibiotic has survey is completely intact and his vital signs are
this patient most likely been taking? normal. A chest x-ray is performed and is normal.
(A) Penicillin V Pelvic x-ray and focused assessment ofsonography in
(B) Doxycycline trauma (FAST) scans are negative. Secondary survey
(C) Ciproflo.xacin reveals no additional injuries and the cervical spine is
(D) Amoxicillin cleared clinically. Which of the following is the most
(E) Clindamycin important next step in management?
(A) Diagnostic peritoneal lavage
@jJ Which of the following traumatic injuries is more (B) Pericardiocentesis
common in elderly patients? (C) CT aortogram
(A) Subdural hematoma (D) Admit for observation
(B) Odontoid fracture (E) Discharge home
(C) Flail chest
Tm9 257

~ A 55-year-old woman presents with wrist pain after (D) Cooling decreases symptoms.
fa1lins on her outstretched right hand. A lateral wrist (E) Muscle weakness tends to worsen after long
radiograph is shown in Figure 9-7. Which of the periods of rest.
following is the most likd.y diagnosisf
I!!) Which ofthe following is the most common area to
be af£eded in compartment syndrome?
(A) Arm
(B) Foreann
(C) Hand
(D) Thigh
(E) Leg

liZ) A 15-year-olcl boy presents with a severe sore throat


and a scar1atiniform rash. A throat culture is taken
but the patient is treated presumptively for group A
P-hemolytic streptococcus (GAS) pharyngitis with
oral penjcjlljn V. Three days later, he returns without
improvement, and you are surprised to find that
his throat culture is negative. He has an exudative
tonsillitis and no posterior lymphadenopathy or
splenomegalyon examination with a di1fuse, pruritic,
scarlatiniform rash.. What is the most likely organism
responsible for his illness?
(A) Resistant group A P-hemolytic Streptococcus
pyogenes
(B) Epstein-Barr virus (EBV)
(C) Arcanokcterium hRemolytkum
(D) Mycoplamul pneumoniae
(E) Chlamydia pneumoni4e

(ii} A 23-year-old woman presents for right ear pain and


Figure t-7. drainage after being struck on the side of the head
with a basketball. Her tympanic canal and membrane
are shown in Fig. 9-8. Which of the following is the
(A) Scaphoid fracture most appropriate nat step in management?
(B) Lunate dislocation
(C) Perilunate dislocation
(D) Distal radius fracture
(E) Thumb metacarpal fracture

~ Direct synthesis of which of the following clotting


factors is inhibited by coumadin?
(A) Factor XII
(B) FactorX
(C) FactorVlll
(D) FactorY
(E) Factor III

(HJ Which of the following is true about myasthenia


gravis (MG)f FigllNI-8.
(A) Incidence peaks in the eighth decade of life.
(B) Sensory deficits are most severe in the lower
extremities. (A) Prednisone 40 mg PO q.d. x 4 days
(C) The most frequent initial symptom is (B) Doxycycline 100 mg PO b.i.d. x 10 days
dysarthria. (C) Gentamicin 100 mg IV t.i.d. x 1 days
258 1000 Qwstimu to Hdp You Ptw the Emerpnt;y Maicitw Bo.rmfs

(D) Copious ear irrigation with saline and peroxide (MJ Which of the following is true regarding hemoptysis?
(E) No specific therapy
(A) Massive hmzoptysis is defined as expectorated
blood > 1 L per 24-hour period
~ A 45-year-old man pracnts in a coma after being
(B) Chest x-rays hdp to localize the site of bleeding
exposed to smoke from a building fire. The patient is
in 90% of patients with hemoptysis.
immediately intubated. An arterial blood gas (ABG)
(C) The most common source of bleeding in
demonstrates metabolic aci<l.oW and an extremely massive hemoptysia are the bronchial arteries.
elevated 1act.ate level. Which of the following is the
(D) Ifthe site of bleeding is lbnited to one lung. the
most important medication to administer?
patient ahould be placed in a lateral decubitus
(A) Amyl nitrite position with the a1fected side up to promote
(B) Sodium nitrite drainage.
(C) Sodium thiosulfate (E) The most effective nonsurgical approach to
(D) Methylene blue massive hemoptysis is laser photocoagulati.on.
(B) Dexamethasone
(!iJ An institutionalized patient with psychiatric disease
~ Which of the following is true regarding drug- presents with abdominal pain, distension, and nausea
induced gingival diseasef without vomiting. The image shown in Fig. 9-9
(A) Gingivitis can complicate drug-induced suggests which of the following diagnose& ?
gingival hyperplaaia.
(B) Almost all patients on phenytoin eventually
develop gingival hyperplasia.
(C) Phenytoin-associated gingival hyperplasia is
dose dependent.
(D) Good oral hygiene is unlikely to improve the
condition.
(E) Phenytoin is the only drug that causes gingival
hyperplasia.

(!j) Which of the following is true for patients with


swpected globe ru~
(A) Succinylcholine is the panlytic ofchoice for
rapid-sequence intubation.
(B) Tonometry is indicated to assess for
concomitant glaucoma.
(C) Bye shielding should be avoided due to
infectious complications.
(D) Intravenous antibiotics should be given.
(E) Tetanus immunization is contraindicated.

(!i) Which of the following is the most common


opportunistic infectiow agent in AIDS patients?
Figure 1-t.
(A) Tuberculosis
(B) Pneumocystis carinii
(C) Cryptomccw nwfomunu
(D) Toxopllmrul gondii (A) Sigmoid volvulus
(E) Cytomegalovirus (CMV)
(B) Small bowel obstruction
(C) Intuuuac.eption
~ Which of the following is indicated Cor treabnent of (D) Diabetic gastroparesis
acute angle closure glaucoma? (E) Hirschsprung'a disease

(A) Topical cydoplegic.s


(!j] A 3-week-old term neonate is brought by his parents
(B) Topical antivi.tab
with fever ofl02.0°P. He appears active and nontoxic,
(C) Aspirin
his fisbt tympanic membrane (TM) is slightly red,
(D) AcetBzolamide
but the rest ofthe physical aamination is completely
(E) Lateral canthotomy
Test9 259

unremarkable. Which of the following is the most demonstrates a 30% pneumothorax on the right.
appropriate next step in management? Which of the following is the most appropriate
(A) Discharge home with routine follow-up. management at this time?
(B) Discharge home with next-day follow-up. (A) Needle thoracostomy at the second intercostal
(C) Discharge home with next-day follow-up and space, midclavicular line
amoxicillin. (B) Needle thoracostomy at the fifth intercostal
(D) Admit for observation with prophylactic space, midclavicular line
antibiotics. (C) Tube thoracostomy at the second intercostal
(E) Admit for observation with prophylactic space, midaxillary line
antibiotics and blood, urine, and cerebrospinal (D) Tube thoracostomy at the fifth intercostal
fluid (CSF) cultures. space, midaxillary line
(E) Observation alone
[!!] Intussusception in adults:
(A) Most often presents with symptoms of partial [j!) A 12-month-old girl presents with pain on urination.
intestinal obstruction. Which ofthe following is the most appropriate urine
(B) Most commonly occurs in the large intestine. collection method for this patient?
(C) Is most commonly idiopathic, without an (A) Diaper collection
identifiable lead point. (B) Bag collection
(D) Is the second most common cause oflarge (C) Urethral catheterization
bowel obstruction. (D) Midstream clean catch
(E) Is best diagnosed with barium or water contrast (E) Suprapubic catheterization
enema.
~ A 26-year-old woman presents with dyspnea and
[!!) A 42-year-old man with hypertension presents with pleuritic chest pain and is subsequently diagnosed
significant swelling of his lips and tongue. He has with a pulmonary embolism (PE). She is not
been taking lisinopril for the past 8 months. Which pregnant, takes no oral contraceptive therapy, and is
of the following is true? a nonsmoker, but she notes that her mother has had
two PEs. Which of the following is the most likely
(A) This adverse drug event is most common in the
cause of this patient's PE?
first week after therapy.
(B) This patient's symptoms may be resistant to (A) Plasminogen deficiency
treatment with epinephrine. (B) Nephrotic syndrome
(C) These patients typically do not have urticaria. (C) Cervical cancer
(D) Angiotensin-receptor blockers are unsafe to (D) Factor V Leiden
use in this patient. (E) Protein S deficiency
(E) All of the above.
~ In patients with tibial shaft fractures, which of the
~ A 64-year-old woman with past history of sick sinus following is the most common associated finding?
syndrome and recent pacemaker placement presents (A) Fibular fracture
with neck pain after a low-speed motor vehicle (B) Common peroneal nerve injury
collision. She has midline cervical spine tenderness at (C) Posterior tibial nerve injury
C6-7. A cervical spine plain film series is inadequate (D) Dorsalis pedis artery injury
for visualizing these segments, so a CT scan is (E) Posterior tibial artery injury
performed. The CT scan is normal, but the patient
still has pain in her neck. Which of the following is ~ Fracture ofwhich ofthe following structures is readily
the most appropriate next step in management? identified by oblique views of the cervical spine?
(A) Discharge home with soft collar (A) Spinous process
(B) Discharge home with hard collar (B) Lamina
(C) Flexion-extension cervical spine x-rays (C) Pedicle
(D) Oblique cervical spine x-rays (D) Vertebral body
(E) MRI of the cervical spine (E) Dens

~ A 34-year-old man presents with chest pain and liiD A 34-year-old women presents to the ED with
shortness of breath after being struck in the right increasing low abdominal pain. She was seen by
side of the chest with a baseball bat. Chest x-ray a colleague a week ago and diagnosed with an ectopic
260 1000 Questions to Help You Pass tM Emergency Medicine Boards

pregnancy in her left fallopian tube. Her OB was ~ Which of the following is the most appropriate
consulted at that time and the patient was started on outpatient management for mechanical corneal
methotrexate therapy. What is the most likely cause abrasions?
of her abdominal pain?
(A) Eye patching
(A) Treatment failure and increasing size of the (B) Topical anesthetic
ectopic pregnancy (C) Topical steroids
(B) Tubal rupture (D) Topical antibiotics
(C) Appendicitis (E) Topical saline solution
(D) Normal "separation pain" from methotrexate
use ~ In healthy adults, which of the following is a normal
(E) PID CSF opening pressure?
(A) 0 em water
~ A 50-year-old woman presents with sudden onset
(B) 5 em water
of crushing chest pain at rest. She has no prior
(C) 15 em water
history of chest pain. Her EKG demonstrates 5 mm
(D) 30 em water
of ST-segment elevation in all anterior leads. An
(E) 60 em water
immediate angiogram is performed, which reveals
coronary arteries completely dear of occlusion. The
[!!) A 28-year-old woman at 29 weeks' gestation is
diagnosis of coronary vasospasm is made. Which of
brought to the ED by ambulance after a minor motor
the following represents the proportion of acute MI
vehicle accident. She was the restrained driver of a
due to coronary vasospasm alone?
car traveling approximately 20 mph when she lost
(A) 1% control on "black ice" and collided with a road sign.
(B) 10% There was minimal damage to the car according to
(C) 25% emergency medical services (EMS) but they placed
(D) 33% her in a cervical spine collar and on a backboard
(E) 50% for transport. She has no complaints in the ED
except for discomfort related to the board and collar.
~ A 22-year-old man states that he ingested 20 to After finding that her primary and secondary survey
30 condoms containing a total of 1 kg of cocaine is intact, she is asking to go home. Which of the
12 hours ago. He subsequently became concerned following is the next best step in management?
about rupture of the condoms and now wants to get
(A) Discharge the patient with close obstetric
them out ofhis body. He is completely asymptomatic.
follow-up.
His vital signs are normal and his physical examina-
(B) Document fetal heart tones before discharge.
tion is unremarkable. Which of the following is the
(C) Perform a transabdominal ultrasonography to
most appropriate next step in management?
ensure fetal viability and absence of placental
(A) Polyethylene glycol abruption.
(B) Activated charcoal (D) Perform 4 hours of cardiotocographic
(C) Hemodialysis monitoring before discharge.
(D) Endoscopic removal (E) Admit her for 23 hours of cardiotocographic
(E) Operative removal monitoring in labor and delivery.

[jjJ A 65-year-old woman with hypertension, atrial ~ Which ofthe following is the most common cause of
fibrillation, and type II diabetes presents with acute upper gastrointestinal bleeding (UGlli)?
vision loss in her right eye with no eye pain. On
(A) Esophageal varices
examination, visual acuity is markedly decreased and
(B) Mallory-Weiss tear
the patient has a striking afferent pupillary defect.
(C) peptic ulcer disease (PUD)
She also has a right-sided carotid bruit. Which of
(D) Esophagitis
the following is the most appropriate next step in
(E) Gastritis
management?
(A) Lateral canthotomy ~ A 44-year-old man with a history of kidney stones
(B) Globe massage presents with progressively worsening left flank pain
(C) Aspirin for several days, dysuria, nausea, vomiting, and fever
(D) Heparin to 101°F. Urinalysis demonstrates 50 WBC per hpf,
(E) TPA positive leukocyte esterase, and positive bacteria. A
Tm9 261

cr scan of the abdomen and pems demonstrates a (B) •"You have a ligament tear."
7-mm stone at the left ureteropeMc junction. Which (C) •oyou have a knee dislocation."
of the followins is the most appropriate nat step in (D) •oyour knee will heal just fine."
management? (E) "I can't Illllh a definite diagnosis at this time.''
(A) Disclwge home with oral hydrooodone
(B) Discharge home with oral hydrocodone and I!!) A 50-year-old man presents with right lower &cial
promethazine swelling for 3 days. He has had right lower
(C) Discharge home with oral hydrocodone, molar pain Cor several weeks but has not seen
promethazine, and ciproftoxacin a dentist. The patient hu a history of alcohol
(D) MRI of the abdomen and pelvis abuae. On examination, the patient has a low-
(E) Emergent urologic consultation grade fever, restricted neck movement, trismus, and
firm swelling in the bilateral submandibular and
~ A 17-year-old girl presents with bilateral lower submental regions. Which of the following is true
quadrant abdominal pain and vqinal discharge for regarding this patient?
3 days. She reports having been sexually active (A) The most commomy affected teeth with this
with multiple partners in the last month. Her last condition are the lateral incisors.
menstrual period just finished 3 days ago. She denies (B) PseudomotW species are the most common
fever, vomiting, dysuria, and diarrhea. Her urine cause.
pregnancy test is negative. Which of the following is (C) The moat common cause of death is septic
most likely to yield the correct diagnosis? shock.
(A) Physical examination (D) Fiberoptic nasotracheal intubation is the
(B) CBC preferred method ofairway control.
(C) Urinalysis (E) Corticosteroids are clearly associated with
(D) Liver function tests better outcomes.
(E) Cf sc:an of the abdomen/peiYis
(!!) A 10-year-old boy presents with fever, diarrhea,
(!!] In most patients diagnosed with community- pallor, and weakness. Renal function is abnormal
acquired pneumonia, which of the following con- and the patient is anemic. A peripheral blood smear
tributes most to a patient's Pneumonia SeverityIndex is shown in Figure 9-10. Which of the following is
(PSI) score? the most likely diagnosis?
(A) Underlying neoplastic disease
(B) Cirrhosis
(C) Age
(D) Sex
(E) Pulse > 125

(!!] A 65-year-old man needs an emergent cr sc:an with


0
- -• 0

IV contrast. Which of the following is a risk &dar


for development of contrast-induced acute tubu.lar
necrosis?
(A) Hypertension
I
(B) Pheochromocytoma
(C) Urinarytractinfection
(D) Diabetes
(E) Immune-compromised state Figllnt 1-10.
• 4

(!11 A23-year-old woman presents aftertwistingherright
knee while playing tennis. She developed swelling
(A) Henoch-Scholll.cln purpura
in her knee immediately after the injury but was
(B) Hemolytic uremic syndrome (HUS)
able to ambulate. Physical euminationdemonstrates
(C) Disseminated intravascular coagulation (DIC)
a moderate effusion without definite ligamentous
(D) Idiopathic thrombocytopenic purpura
instability in any direction. Which of the following
(E) Nephrotic syndrome
should you tell this patient?
(A) "You will require surgery."
262 1000 ~ tDHdp YouPo~W the~ MetlidMBoGrds

[too) A 4-year-old boy presents with penile pain. His


physical examination is shown in Figure 9-11. Which
of the following is the corred: diagnosis?
(A) Phimosis
(B) Paraphimosis
(C) Balanitis
(D) Testicular torsion
(E) Scrotal hernia

Figure t-11.
Answers and Explanations
(I) Answer B. Other than the epiglottis, epiglottitis delayed for up to 12 hours. In addition, patients
may involve several supraglottic structures, including may experience nausea, vomiting, headache and
the vallecula, aryepiglottic folds, arytenoids, lingual sweating. Ptosis is frequently the initial sign of
tonsils, and base of the tongue. Inflammation does neurotoxicity and may be followed by delirium,
not extend to the infraglottic tissues because of the tremors, drowsiness, hypersalivation, and multiple
robust attachments between the infraglottic mucosa cranial nerve abnormalities (dysarthria, diplopia,
and submucosa. Owing to the variable involve- dysphagia). In severe envenomations, respiratory
ment of several supraglottic structures, epiglottitis muscle paralysis occurs leading to respiratory failure
is sometimes referred to as supraglottitis. Drooling and death. If a pit viper bite was suspected, then
and stridor are unusual presenting signs in patients observation would be the best course of action. If
with epiglottitis. Historically, however, it has been no signs of envenomation develops 4 to 6 hours
thought that patients presenting with these symp- after pit viper envenomation, the patient may safely
toms, especially if they have developed over a short be discharged with routine follow-up instructions.
time period, are at higher risk for subsequent airway (Figure© Dr. Julian White.)
loss. No large, prospective trials have been conducted
to sort this out. Most typically, patients with epiglot- (I) Answer B. The incidence ofseizures after ischemic
titis present with a severe sore throat and painful stroke is uncertain but is cited to be as high as 13%.
dysphagia. Adult epiglottitis does not demonstrate Typically, patients with seizures after stroke are di-
any seasonal variation, but appears more common vided into patients who have seizures within 7 days
in males and smokers. Neither epinephrine nor cor- (early seizures) and those who develop seizures after
ticosteroids has been shown to be beneficial, despite 7 days (late seizures). Prophylactic use of anticon-
their widespread use. Caution is advised regarding vulsants in patients with ischemic stroke has not
the use of epinephrine as a temporizing measure been shown to reduce either early or late seizures
in patients with epiglottis due to possible rebound although there is limited data on this topic. There-
upper-airway constriction after the treatment is com- fore, the prophylactic use of anticonvulsants in such
pleted. Ninety percent ofpatients with epiglottitis will patients is not recommended. When seizures do
have abnormal lateral neck films. The classic find- occur, their management is the same as "conven-
ing is the "thumb" or "thumbprint" sign, indicating tional" seizure management, and status epilepticus
the presence of a swollen, inflamed epiglottis. How- is rare. Current guidelines recommend that patients
ever, a normal film cannot exclude the disease. Direct with an ischemic stroke in the ED who develop
nasopharyngoscopy has been the gold standard ofdi- seizures warrant standard treatment (if necessary)
agnosis, as it allows direct visualization of the tissue to terminate the seizure followed by treatment with
in question. Recently, however, the "vallecula" sign an anticonvulsant to prevent recurrence. There are
has been suggested as another method of screening no special contraindications to antiepileptic use in
for the presence of epiglottitis on lateral neck films. stroke patients.
This method relies on the physician's ability to locate
the base of the tongue and trace it inferiorly toward [!) Answer C. The half-life of carboxyhemoglobin is
the hyoid bone to locate the vallecula. If the vallecula 6 hours on room air, 90 minutes on 100% nome-
is not deep and roughly parallel to the pharyngo- breather, and 30 minutes on hyperbaric oxygen.
tracheal air column, then epiglottitis is present. In a
small trial, this sign was shown to be 98% sensitive [!) Answer C. Retrobulbar hemorrhage is a result of
and 100% specific for epiglottitis. ocular trauma that causes pressure on the posterior
portion of the eye. The globe is pushed outward,
[!] Answer D. The patient was bitten by a coral and proptosis may be seen on physical examina-
snake, which is identifiable in North America by tion. Increased pressure in the orbit can compress
the presence of red bands that are bounded by the central retinal artery or vein and cause loss of
yellow bands ("red on yellow kill a fellow''). All vision. Choices A, B, D, and E are all pathologic pro-
patients suffering coral snake envenomation should cesses involving the anterior portion of the eye and
be treated with antivenin even in the absence of are not usually caused by a retrobulbar hematoma.
clinical symptoms. Coral snake venom is neurotoxic Secondary glaucoma may occur as a result of in-
and signs of neurotoxicity may develop rapidly or be creased overall pressure in the globe, including in the

263
264 1000 Questions to Help You Pass tM Emergency Medicine Boards

anterior chamber. Treatment of a retrobulbar hem- [!] Answer B. Any patient with a history of recent
orrhage involves emergent lateral canthotomy and unprotected anal intercourse who presents with
drainage ofthe hematoma out of the temporal border symptoms of proctitis should be treated empirically
of the globe. Failure to perform lateral canthotomy for N. gonorrhoeae proctitis. Because concurrent
for acute retrobulbar hemorrhage may result in irre- infection with Chlamydia trachomatis is common
versible vision loss in as little as 90 minutes. in patients infected with gonorrhea, empiric therapy
should cover this organism as well. Podophyllin is
[!] Answer A. This is the swan neck deformity. a treatment for human papillomavirus (condyloma
It is caused by hyperextension at the proximal acuminatum). Valacydovir is a treatment for herpes
interphalangeal (PIP) joint and flexion at the proctitis, penicillin is a treatment for syphilis, and
distal interphalangeal (DIP) joint. The DIP joint lopinavir is a protease inhibitor used to treat
flexion occurs due to elongation or rupture of the human immunodeficiency virus (HIV). Owing to
extensor tendon attachment to the distal phalanx this patient's high-risk lifestyle, he should undergo
(i.e., similar to a mallet injury). Left untreated, PIP testing for HIV and for syphilis but empiric treatment
hyperextension occurs as a consequence of the distal is not necessary.
mallet deformity. However, the deformity can begin
in the PIP joint as well due to synovitis of the [!Ql Answer E. Twenty-five percent of patients with
volar capsule resulting in PIP hyperextension. In the Crohn's disease will have extraintestinal manifesta-
latter case, DIP flexion occurs as a secondary effect. tions. Approximately 20% will have arthralgias of
Both the swan neck and boutonniere deformities are various types that usually occur in concert with
common in rheumatoid arthritis. (Figure reprinted disease activity. Episcleritis and anterior uveitis (or
with permission from Oatis CA. Kinesiology: The iritis) are the most common ocular complications.
mechanics and pathomechanics of human movement. Episcleritis does not affect visual acuity but anterior
Lippincott Williams & Wilkins; 2004.) uveitis can ultimatelylead to a decrease in vision ifthe
posterior segment becomes involved. Cholelithiasis
(I] Answer E. Sitz is a word that comes from the and nephrolithiasis are common complications of
German word, sitzen, meaning "to sit." A sitz bath Crohn' s disease due to disease of the terminal ileum,
refers to any device that allows a patient to immerse which interrupts bile acid resorption and results in
only the perineum and buttocks in water while oxalate malabsorption. Pyoderma gangrenosum and
draping the rest of their body outside the tub. While erythema nodosum are the most common cutaneous
patients may add various medications to the water, manifestations of Crohn's disease although rashes
it is not recommended and there is no evidence that commonly occur due to hypersensitivity reactions to
such additives help. Sitz baths are thought to help a frequently used drugs as well.
variety of perianal complaints, such as hemorrhoids,
because anal canal pressure decreases significantly in ~ Answer D. Ranson's criteria are best used to ex-
warm water (40°C) and blood flow improves. clude severe disease as its sensitivity for predicting
severity ranges from 57% to 85%. However, the
(!] Answer C. The patient has a large left-sided presence of ~3 Ranson's criteria is a marker of
varicocele. Varicoceles are caused by abnormal more severe disease as patients with 3, 4, or 5 crite-
dilation of the testicular vein and pampiniform ria have a 10% mortality rate (vs. <5% for patients
plexus of the scrotum due to venous pooling from with <3 criteria, and >60% for patients with ~6
impaired drainage of the left internal spermatic criteria). The use of Ranson's criteria is problem-
vein into the left renal vein. Large varicoceles atic, however, because 5 of the criteria are assigned
substantially increase the risk of infertility due upon admission, but the remaining 6 criteria are
to impaired blood flow and temperature of the not assigned until 48 hours after admission. The
ipsilateral testis. Testicular torsion may occur in five Ranson's criteria are glucose >200 mg per dL,
patients with varicoceles but is not as common age older than 55, LDH >350 IU per L, AST
as infertility. Epididymitis and malignancy do not >250 SF units, and WBC count >16,000 per mm3
occur at appreciably higher rates in patients with (mnemonic: "Georgia law'' or GA LAW).
varicoceles. Deep venous thrombosis due to inferior
vena cava thromboses may cause varicoceles, but this [YJ Answer C. Peptic ulcer disease (PUD) is the most
is rare. (Figure from Fleisher GR, Ludwig S, Henretig common cause of UGIB, accounting for half of all
FM, et al. eds. Textbook of pediatric emergency cases. Bleeding is the most common complication of
medicine, 4th ed. Philadelphia: Lippincott Williams PUD, and perforation is the next most common.
& Wilkins; 2000, with permission.)
Test9 265

Gastric outlet obstruction occurs in only 2% of peripheral vascular resistance. Tissue plasminogen
patients with known PUD. activator (tPA) is not emergently indicated in
a patient with chest pain without ST -elevation
[!j) Answer A. The treatment for aspirin overdose in- myocardial infarction (STEMI).
volves gastrointestinal decontamination, hydration,
and enhanced excretion of drug. Urinary alkalin- [!1) Answer E. There is a dearth ofdata regarding pedi-
ization with intravenous bicarbonate increases the atric patients with ventriculoperitoneal shunt (VPS)
amount of ionized salicylate, which is excreted more infections. However, several studies have suggested
easily by the kidneys than unionized salicylic acid. that as many as one third of patients may present
A urine pH goal of 7.5 to 8 optimizes this ap- with nonspecific clinical findings. The risk of infec-
proach. Forced diuresis has been found to increase tion is highest in the first 6 months after insertion
the risk of cerebral and pulmonary edema and does or instrumentation (in the case of revision). In ad-
not result in increased excretion of salicylic acid. dition, patients younger than 4 years old and older
Activated charcoal and whole bowel irrigation are than 61 years old have been shown to be at in-
recommended in cases of acute intoxications and in- creased risk of infection. Lumbar puncture should
toxications with enteric-coated aspirin, respectively. never be performed without first obtaining a CT
Hemodialysis may be life saving for patients who have scan and reviewing the findings with a neurosur-
severe salicylate toxicity, organ failure, or failure of geon. The mortality of VPS infection is roughly
standard, noninvasive management. 30% to 40%. As many as one third of patients with
VPS infection may present with signs and symp-
~ Answer E. In trauma patients, anterior cord syn- toms of obstruction or VPS failure with or without
drome most commonly occurs in hyperflexion in- a fever. These include hydrocephalus, papilledema,
juries in which herniated vertebral discs or vertebral hypertension with concomitant bradycardia, and ir-
body fragments compress the anterior aspect of the regular respirations (Cushing response), personality
spinal cord or the anterior spinal artery. The anterior changes, ataxia, and cranial nerve palsies. Over 80%
spinal artery provides blood supply to the anterior of patients with a VPS infection have peripheral
two thirds of the spinal cord. The primary structures leukocytosis.
affected in the spinal cord are the spinothalamic
tract, which is responsible for the transmission of ~ Answer B. Limited range of motion in the shoul-
pain and temperature sensory input, and the corti- der in patients with preceding trauma is most likely to
cospinal tract, which carries descending voluntary be due to adhesive capsulitis (also called frozen shoul-
motor signals. The dorsal or posterior columns, der). Adhesive capsulitis is characterized by stiffness
which are responsible for proprioception and vi- with or without pain in all directions of shoulder
bration sensation, are unaffected so those functions movement. Prevention and treatment of adhesive
are preserved. capsulitis after shoulder injury are accomplished by
range of motion exercises. Rotator cufftear is usually
~ Answer D. Elderly patients, diabetic patients, and heralded by significant pain when the arm is moved
women with acute Mls may have atypical symptoms in a particular direction. Septic arthritis usually in-
without the classic history of left-sided chest pain. volves a febrile patient with extreme pain on any
Dyspnea, nausea, and diaphoresis may be anginal movement of the joint. Scapular fracture is rare in
equivalents in many patients. Additionally, patients patients without high-force mechanisms such as mo-
may deny "pain"-rather, they may describe their tor vehicle crashes or falls from height. Rheumatoid
symptoms as "pressure" or "discomfort." arthritis would be unlikely in an 85-year-old without
prior history.
~ Answer D. The EKG demonstrates no specific
findings consistent with acute myocardial ischemia [!?) Answer A. Edrophonium is a cholinesterase in-
or infarction. Cocaine-induced myocardial ischemia hibitor and may therefore produce symptoms of
is due to both vasospasm and hyper-aggregatory cholinergic toxicity, including bradycardia, excessive
platelets, which may cause acute thrombosis. Aspirin airway and oral secretions, tearing, and dyspep-
is indicated in patients with cocaine-induced chest sia associated with nausea or vomiting. Of these,
pain untilitisknownforcertain thatMiorischemiais bradycardia is the most common serious side ef-
not present. In a patient without current symptoms fect, even though it occurs very infrequently, having
of chest pain, nitroglycerin and morphine are not been reported in only 0.16% of patients. However,
indicated. Metoprolol is relatively contraindicated in as excessive airway secretions may also occur, cau-
cocaine-induced chest pain as there is a theoretic risk tion is advised in patients with lung diseases such
of reducing cardiac output in the face of increased as asthma or chronic obstructive pulmonary disease
26 6 1000 Questions to Help You Pass the Emergency Medicine Boards

(COPD). Atropine should be at the bedside in case develop pericarditis. EKG changes are usually absent,
either of these symptoms occurs. Oculogyric crisis masked by the MI findings. Pericardia! friction rub
describes a dystonic reaction in which patients expe- is characteristic and often audible. The proposed
rience severe torticollis and extreme upward gaze that etiology is autoimmune. Dressler's syndrome is the
may persist for hours. It is most commonly due to term given to pericarditis which occurs 2 to 3 weeks
treatment with neuroleptics (antipsychotics) and is after MI. It is clinically and electrocardiographically
terminated with the antimuscarinic drug benztropine indistinguishable from ordinary pericarditis and
(Cogentin) in concert with benzodiazepines. Cough, should be treated in the same way.
atrial fibrillation, and seizures are not associated with
edrophonium use. ~ Answer A. Breast milk jaundice develops in 2% of
breast-fed infants after the seventh day of life. Levels
~ Answer B. Carpal tunnel syndrome refers to me- peak during the second to third week of life and
dian neuropathy due to compression at the wrist. may be as high as 10 to 30 mg per dL. Treatment is
Common conditions associated with carpal tunnel straightforward and involves stopping breast-feeding
syndrome include diabetes, hypothyroidism, preg- for 1 to 2 days and substitution ofbottle feeding with
nancy, and rheumatoid arthritis. Findings include formula. This results in a rapid decline in serum
pain and paresthesias in the thumb, index, and long bilirubin after which nursing can be resumed with-
fingers. Phalen's test (pain from holding the wrists out a recurrence of the prior hyperbilirubinemia.
flexed for 60 seconds) is more sensitive than Tinel's Breast milk jaundice is an unconjugated hyper-
test (pain from tapping the volar wrist), but neither bilirubinemia. Hyperbilirubinemia is described as
is reliable enough to exclude or prove the diagnosis "conjugated" when direct bilirubin exceeds 2 mg
and nerve conduction studies are the gold standard. per dL or represents >20% of the total bilirubin
Treatment involves wrist splinting, with surgery re- level. Although breast milk jaundice is generally a
served for refractory cases. benign process, phototherapy may be required if
total bilirubin levels exceed 18 to 20 mg per dL.
[g!j Answer D. Kidney stones most commonly occur Finally, breast milk jaundice should be differentiated
in middle-aged patients, usually men. Recurrence from breast-feeding jaundice, which is an early-onset
occurs in approximately half the number of pa- unconjugated hyperbilirubinemia that occurs in the
tients. Risk factors include age, male gender, family first week in breast-fed infants. The mechanism, as
history, and conditions which increase serum and with breast milk jaundice, is unclear, but is thought to
urinary calcium levels. Kidney stones are divided be related to decreased milk intake with dehydration
into four main categories-calcium, magnesium- or reduced caloric intake.
ammonium-phosphate, uric acid. and cystine. Cal-
cium stones represent approximately two third of ~ Answer B. Posterior parapharyngeal space infec-
all stones and occur more often in patients with tions are more dangerous than anterior infections
common precipitants of hypercalcemia, including as they may encroach on the cervical sympathetic
hyperparathyroidism, milk-alkali syndrome, laxative chain as well as the carotid artery and jugular vein.
abuse, and sarcoidosis. Inflammatory bowel dis- Patients with such infections may develop an ipsi-
ease (IBD) also causes the formation of calcium lateral Homer's syndrome or cranial neuropathies
oxalate stones, due to hyperoxaluria. Magnesium- of cranial nerves IX, X, XI, and XII. Jugular vein
ammonium-phosphate (struvite) stones account for thrombosis may also occur along with erosion of the
one fifth of all calculi and occur in patients with carotid artery, resulting in life threatening hemor-
urinary tract infections due to Proteus, Klebsiella, and rhage or aneurysm formation. Involvement of the
Pseudomonas. Uric acid stones occur in patients with jugular vein may result in septic thrombophlebitis
hyperuricemia, often due to gout. They are usually and a subsequent Lemierre' s syndrome, which is also
radiolucent and missed on plain radiographs. Cystine known as postanginal septicemia. Patients with this
stones are the least common and are due to hyper- problem present with symptoms of severe sepsis after
cystinuria, an inborn error of metabolism usually their symptoms ofpharyngitis have resolved. All deep
diagnosed at birth. space neck infections have the capability of causing
such severe complications by virtue of their ability
~ Answer C. Post-MI pericarditis generally occurs to extend to adjacent spaces resulting in a poste-
within four days of MI and is characterized by a rior parapharyngeal space infection. However, as the
change in the quality of chest pain. It is treated posterior pharyngeal space lies alongside the carotid
with NSAIDs, in the same manner as idiopathic sheath, infections in this area most commonly erode
pericarditis. Less than a quarter of all patients with MI into the adjacent vasculature.
Test9 267

~ Answer E. "Crossed signs," in which a patient effective long-term asthma management. Although
has unilateral cranial nerve deficits but contralateral corticosteroids are vital in the long-term man-
hemiparesis and hemisensory loss are diagnostic of agement of asthma, chronic exposure to systemic
brainstem infarction. The vertebral arteries, which corticosteroids is fraught with potential complica-
have their origin from the subclavian arteries, merge tions. Even short-term use may result in transient
to form the basilar artery at the pontomedullary insulin-resistance, peptic ulcer disease (PUD), asep-
junction. At the junction of the pons and the tic necrosis of the femur, edema, and weight gain
midbrain, the basilar artery again separates into the as well as mood disturbances. Long-term use may
two posterior cerebral arteries. The brainstem, from result in muscle weakness or wasting, immunosup-
the medulla to the midbrain, is therefore supplied pression, Cushing's syndrome, cataracts, diabetes,
by branches of the vertebrobasilar arterial system. hypertension, growth retardation in children, and
The facial nucleus, which originates within the pons, adrenal axis suppression.
may be infarcted when branches of the basilar artery
are occluded. As the infarction involves the facial ~ Answer A. Patients younger than 12 months of
nucleus, the entire face, including the forehead, is age with febrile seizure are at higher risk for having
affected. As the descending corticospinal tract has meningitis as the cause of their febrile seizure. The
not yet reached the medullary decussation, occlusion AmericanAcademyofPediatrics (AAP) recommends
on one side ofthe pons affects the descending motor that patients younger than 12 months of age should
fibers reaching the contralateral body and results all receive a lumbar puncture and that patients
in contralateral hemiparesis. When this syndrome younger than 18 months of age should be strongly
includes an ipsilateral rectus palsy, it is known as considered for lumbar puncture. Other higher risk
Millard-Gubler syndrome due to infarction of both groups include patients with focal or prolonged
the facial and abducens nuclei which are in very close seizures, abnormal physical examinations, or toxic
proximity in the pons. Infarction of the basilar artery appearance. Most ordinary febrile seizures are treated
proper is typically a catastrophic event, resulting in symptomatically with antipyretics. Approximately
quadriplegia and often respiratory failure and death. one third of patients with febrile seizures will have
at least one more febrile seizure episode. Risk factors
~ Answer B. The chief use of EKG in the evaluation for this include young age at first seizure, lower
of patients with a suspected pulmonary embolism temperature with the seizure episode, a first-degree
(PE) is to rule out the presence of other causes relative with febrile seizure, and short duration
of chest pain such as myocardial ischemia or acute between fever onset and seizure event. Patients
MI and pericarditis. Numerous EKG findings have with febrile seizures are twice as likely to develop
been reported in the setting of acute PE, although generalized epilepsy than the average population.
"St 03 T3" is perhaps the most famous. Several papers
have reliably refuted the utility ofthis finding. S1 Q3T3 ~ Answer E. The initial intravenous resuscitation
was first described in a 1935 study of seven patients fluid of choice in pediatric patients is 0.9 normal
who all likely had massive pulmonary embolus. saline. It is given in a 20 mL per kg bolus and may
Similar studies have all been plagued by a selection need to be repeated twice in patients with severe
bias for patients with large or massive PE. Several dehydration. When sufficient bolus hydration has
EKG findings are more specific in patients with been given, maintenance fluids according to the
massive PE, but other clinical signs and symptoms "4/2/1 rule" are instituted: 4 rnL/kg!hour for the first
(e.g., hypoxia, dyspnea, tachypnea and chest pain) 10 kg of body weight, 2 rniJkg!hour for the next
are typically more useful than EKG. 10 kg, and 1 rnL/kg!hour for every 10 kg after that.
The maintenance fluid composition varies by age and
[gl) Answer C. There is no evidence that intravenous may be looked up in a reference.
corticosteroids provide additional benefits over oral
corticosteroids. However, because patients present- ~ Answer B. Due to the need for powerful im-
ing to the ED with acute asthma exacerbations may munosuppressant agents, all solid-organ transplant
often be in extremis, many physicians prefer to de- recipients are at increased risk for infection. In-
liver steroids intravenously. Regardless, it takes from fections are divided into three time periods-those
6 to 24 hours for corticosteroids to exert an effect occurring within the first month of transplanta-
on pulmonary mechanics. Unless patients are tak- tion, those occurring between 1 and 6 months after
ing chronic oral corticosteroids, there is no need to transplantation and those occurring >6 months after
taper outpatient therapy for acute asthma exacerba- transplantation. Nosocomial agents are prominent in
tions. Inhaled corticosteroids are the cornerstone of the first month, although CMV is the most prevalent
268 1000 Questions to Help You Pass tM Emergency Medicine Boards

infection between 1 and 6 months (particularly CMV this information is not a contraindication and no
pneumonitis). time should be wasted attempting to document fe-
tal viability.
~ Answer A. Cryptosporidium is the most common
cause of chronic diarrhea in patients with AIDS. ~ Answer A. The patient has avascular necrosis of
However, it is much less common in the era of the femoral head, or Legg-Calve-Perthes (LCP)
successful antiretroviral therapy. Cryptosporidiosis disease. It is much more common in boys than
is usually self-limited in immunocompetent patients girls and is usually unilateral. Pain may be referred
as well as in patients with AIDS when the CD4 to the groin or the knee. Young children are affected
count is > 180 per JLL. In contrast. patients with more commonly than young adolescents. Etiology
CD4 counts <100 may develop a chronic course of is unknown. Management may be surgical, but
diarrhea and weight loss. Patients with CD4 counts this is considered on a case-by-case basis. Joint
<50 may experience fulminant diarrhea. aspiration is useful to rule out septic arthritis
as a cause for the symptoms, but radiography
~ Answer C. Central retinal vein occlusion (CRVO) and MRI are the cornerstones of diagnosis of
causes backup of blood flow into the eye and car- LCP disease. Emergency management consists of
ries the potential for increased intraocular pressure, prompt orthopedic referral or consultation for
eventually leading to glaucoma. The classic history consideration of surgical management. Leg-length
is acute or subacute painless loss of vision, although discrepancy, deformity, and limitation of movement
pain may occur in some cases. Risk factors include are important long-term sequelae. (Figure courtesy
hypertension, diabetes, thrombophilia, Funduscopic ofJames T. Guille, MD. Reprinted with permission
examination reveals disc edema with tortuous veins from Guille JT. Greenberis text-atlas of emergency
and retinal hemorrhages. ED management is sup- medicine. Lippincott Williams & Wilkins; 2004:601.)
portive in conjunction with ophthalmologist consul-
tation. Choices A, B, D, and E may be caused by ~ Answer A. The Centor criteria, which have been
trauma or anterior eye disorders. validated in several trials, are most useful to rule
out GAS as the cause of pharyngitis. In several
~ Answer C. Teeth are numbered starting with the trials, patients with none of the four Centor criteria
upper right posterior-most molar, extending to the were found to have had only a 2.5% chance of
left side and then back down the lower left posterior- having a positive throat culture for GAS. In contrast,
most molar, then to the right. There are three molars, patients with all four criteria were found to have
two premolars, one canine, one lateral incisor, and a 56% chance of having a positive throat culture.
one central incisor on each quarter jaw. Furthermore, as the complications of GAS infection
are less common in adults than in children, it
~ Answer A. Perimortem cesarean section should seems reasonable that the goal of treating low-
be performed in all pregnant patients suffering risk patients should be symptomatic relief rather
traumatic cardiac arrest with a fetus > 24 weeks' than the prevention of adverse sequelae. The four
gestational age. Maternal resuscitation always takes Centor criteria are the presence of fever (before
precedence over the fetus, but once cardiac arrest the use of antipyretics), the absence of cough, tender
has occurred, an immediate decision to undergo anterior cervical lymphadenopathy, and the presence
perimortem cesarean section must be made. Sev- of exudative tonsillitis. Mcisaac et al. modified the
enty percent of fetuses will survive if delivered score such that patients younger than 15 years old
within 5 minutes of maternal arrest. None will receive an extra point, although a point is subtracted
survive after 25 minutes. Consent is unnecessary from patients who are older than 45. When these
and should not be sought before the procedure. data are applied, patients with a score of 0 to 1 have
If the fetal gestational age is unknown, cesarean a 1% chance of having a positive throat culture. In
delivery should be performed if the fundal height this case (a 22-year-old woman), no modification
exceeds the umbilicus. To perform the procedure, of her Centor score occurs. Properly implemented
a large midline vertical incision ("classic" midline throat cultures have a sensitivity >90%. Posterior
incision) is made from the subxiphoid process to cervical lymphadenopathy, nonexudative tonsillitis,
the symphysis pubis. If an anterior placenta is en- and atypical lymphocytes are all features of Epstein-
countered upon entering the uterus, it should be Barr virus (EBV) infection, also known as infectious
incised to reach the fetus, as bleeding can be ad- mononucleosis. The presence of a cough, along with
dressed after the procedure. Ideally, verification of other ''viral" symptoms such as rhinorrhea and
fetal viability through fetal heart tones would be conjunctivitis, also make GAS less likely.
documented before the procedure, but absence of
Test9 269

~ Answer A. The patient has blepharitis, as seen by fall on outstretched hand. Significant displacement
the crusting and edema of the upper eyelid. Staphy- ofthe radial head can put the patient at risk for radial
lococcal infection is the most common etiology. nerve injury, exhibited by wrist drop. Treatment is
Treatment is with gentle soap cleansing and topi- surgical in most cases.
cal erythromycin ointment applied to the eyelids.
Topical steroids should never be prescribed by the ~ Answer E. Osteomyelitis is a bacterial infection
emergency physician (EP) without ophthalmologic of the bone that generally follows a subacute
approval Topical anesthetics (such as proparacaine) course. The most common cause is S. aureus,
should never be prescribed to patients, as they will but streptococci and gram-negative bacilli are also
retard corneal healing. Intravenous ceftriaxone is implicated. Patients usually complain of pain in the
used in patients with hyperacute bacterial con- affected bone, but do not appear toxic and often
junctivitis due to gonococcus. This patient has no lack vital sign abnormalities. The sensitivity ofESR is
evidence of conjunctival involvement on physical approximately 90%, and the sensitivity of C-reactive
examination. Acetazolamide is used in patients with protein may be even higher. Radiographs, although
acute angle closure glaucoma, but the absence of commonly used to evaluate osteomyelitis, have
eye pain, headache, and corneal or conjunctival ab- notoriously poor sensitivity in the first week after
normality effectively rules this out. (Figure reprinted the onset of symptoms. Bone scintigraphy and MRI
with permission from Silverberg M. Greenberg's text- are the tests of choice in diagnosing osteomyelitis.
atlas of emergency medicine. Lippincott Williams & MRI is superior to CT scan in characterizing the
Wilkins; 2004:89.) infection.

~ Answer E. The perinatal mortality associated with [!!) Answer D. Only 5% ofintussusception cases occur
breech presentation has been described to be as in adults, and adults are much more likely to have an
high as 25%, although this is largely due to fatal anatomic abnormality of the intestine. Ninety-five
fetal anomalies and premature deliveries. Excluding percent of adults have an identifiable abnormality
these anomalies and premature deliveries, breech (75% are neoplasms) versus only 2% to 8% in
presentation poses little added risk in the hands pediatric cases. The classic triad of abdominal pain,
of an expert. However, delivery in the ED will vomiting, and bloody stools is only present in one
always be fraught with additional risks and poorer third of pediatric cases. Abdominal radiographs are
outcomes due to physician and staffinexperience and noninvasive but are usually nonspecific and therefore
the hurried nature of emergent delivery. In breech not useful in making a diagnosis. Ultrasonography
presentation, the infant is in a longitudinal lie but the is very sensitive, fast, noninvasive, and easy to use.
head is in the uterine fundus whereas the buttocks Classically, barium enemas have been the study of
are the fetal presenting part. The buttocks are less choice because they are diagnostic and therapeutic.
effective than the fetal head in serving as a wedge However, air enemas are equally efficacious and
to dilate the cervix. Therefore, inadequate cervical have a decreased rate of perforation, which is the
dilation is a common problem. As the fetus descends primary complication ofeither procedure. Therefore,
through the cervix, inadequate dilation may result in air enemas are preferred by many authors.
entrapment ofthe fetal head. In addition, in complete
and footling breech presentations (as opposed to ~ Answer A. History of prior ectopic pregnancy
frank breech presentations), the fetus does not is the strongest risk factor for current ectopic
completely occlude the cervix, increasing the risk pregnancy. The most common risk factor is pelvic
of umbilical cord prolapse (the risk is greatest with inflammatory disease (PID). The sequelae of PID
footling breech). Cord prolapse is more common account for approximately half of all cases of
in low birth weight or premature infants, which ectopic pregnancy. Although intrauterine device
account for almost one third ofbreech presentations. (IUD) use increases the relative risk of ectopic versus
Finally, there is a relatively small increased risk of intrauterine pregnancy, it decreases the overall risk
hyperextension of the fetal head, which carries a 21% of all pregnancy significantly. C-section increases the
risk of fetal spinal cord injuries, when it occurs. risk of placenta previa, but not of ectopic pregnancy.
Oral contraceptive use may reduce the symptoms of
~ Answer B. The patient has evidence of a PID (thus predisposing to more untreated PID), but
Monteggia's fracture. Proximal ulnar fracture along overall pregnancy rate (including ectopic) is reduced.
with dislocation of the radial head from the capitel-
lum (which may be subtle) is the specific finding. ~ Answer A. The patient has a lytic lesion with fever
The typical mechanism is a blow to the forearm or in a subacute course consistent with osteomyelitis.
270 1000 Questions to Help You Pass the Emergency Medicine Boards

The most common cause in all patients is S. aureus. ~ Answer A. Traumatic hemothorax may occur af-
Salmonella is more commonly seen in sickle cell ter both blunt and penetrating thoracic trauma.
patients than healthy patients, but is still not as Hemorrhage results most often from parenchymal
common as S. aureus. AspergiUus is a rare cause of vessel damage, then intercostal and internal mam-
septic arthritis. Gonococcus is the most common mary artery lacerations, and uncommonly from great
cause of septic arthritis in the young, sexually active vessel injury. At least 200 mL of blood must be
adult. Pseudomonas causes osteomyelitis in patients present for costophrenic angle blunting to appear
with puncture wounds to the feet and IV drug users. on upright chest x-ray. Tube thoracostomy with a
large chest tube (36-French or greater) at the fifth
~ Answer D. Lambert-Eaton myasthenic syndrome intercostal space and midax:illary line is the treat-
(LEMS) is an autoimmune disorder which targets ment of choice. Smaller tubes will cause blood to
presynaptic voltage-gated calcium channel receptors. clot and prevent adequate drainage. Pneumothorax
In contrast, myasthenia gravis (MG) is an autoim- occurs concomitantly in almost a third of hemotho-
mune disorder which targets postsynaptic acetyl- races and requires suction drainage. Initial chest tube
choline receptors. In both cases, muscle weakness is drainage of 1,500 mL, 250 mL per hour of drainage
the most predominant clinical feature. However, in over 4 hours, worsening hemothorax, hemodynamic
MG, ptosis or diplopia is the most common initial instability, and cardiac arrest are all indications for
symptom. In LEMS, ocular symptoms are uncom- thoracotomy.
mon and proximal muscle weakness of the lower
extremities more than the upper extremities is the ~ Answer C. Myocarditis is most commonlypostvi-
most common presentation. This results in difficulty ral, including Coxsackie B, adenovirus, and influenza.
rising from a seated position and climbing stairs. Au- Bacterial and protozoal pathogens are less common
tonomic dysfunction is a common finding in patients etiologies. More than half the number of patients
with LEMS, but not MG. LEMS may occur sporad- report an antecedent nonspecific viral syndrome.
ically, but it occurs as a paraneoplastic syndrome in Acute symptoms include myalgias, fever, fatigue,
50% to 70% of patients. In these patients, it is most chest pain, and shortness ofbreath, but none of these
commonly associated with small cell lung carcinoma. is present in most cases. Myocarditis may lead to
Finally, DTRs in patients with MG tend to be pre- dilated cardiomyopathy, causing overt heart failure.
served, but they are typically drastically reduced or Leukocytosis is uncommon, EKG findings are non-
absent in patients with LEMS. Briefly exercising the specific, and echocardiogram may show evidence of
muscles involved in the reflex before reflex testing, global hypokinesia. The gold standard for diagnosis is
however, may result in restored or normal reflexes. endomyocardial biopsy, but there are many false neg-
atives due to the patchy nature of the inflammation.
~ Answer E. Subdural hematomas occur due to
disruption ofthe cranial bridging veins from trauma, ~ Answer E. Digitalis inhibits the membrane Na-K
causing blood to collect deep to the dura. The ATPase which normally functions to pump sodium
elderly are at much higher risk than the average out of the cell and potassium into it. Digitalis there-
population due to brain atrophy which causes these fore increases intracellular sodium and decreases
bridging veins to stretch and be susceptible to even intracellular potassium. The increased intracellular
minor trauma. Due to this low pressure venous sodium causes an increase in intracellular calcium,
bleeding, patients with subdural hematoma may which produces a positive inotropic effect. In thera-
have a subacute or chronic presentation with mild peutic doses, digitalis reduces the heart rate and can
initial clinical manifestations of injury. Patients with cause slight ST depression and T -wave inversions.
epidural hematoma, on the other hand, usually have
an arterial source and exhibit signs and symptoms ~ Answer C. Gastroesophageal reflux disease
almost immediately after the trauma. Some patients (GERD) is a very common cause ofvomiting during
with subdural hematoma, especially the elderly and infancy. Emesis is typically nonbilious and begins in
alcoholics, may not even recall their antecedent head early infancy, remaining fairly constant over time.
trauma. Many patients with subdural hematoma It does not demonstrate the progressive pattern to
have a lucid interval (where the mental status is projectile emesis that characterizes pyloric stenosis.
completely normal before becoming altered again), Most infants respond to conservative measures.
which is more classically thought to be associated with However, if those measures fail, infants frequently
epidural hematoma. Coma, focal neurologic deficits, respond to ranitidine (an H2 histamine receptor
and increased intracranial pressure are associated antagonist) or metoclopramide (which increases
with both subdural and epidural hematomas.
Test9 271

lower esophageal tone and gastric emptying). Most ~ Answer C. This patient has a pheochromocytoma.
infants continue to gain weight normally and only Pheochromocytomas are catecholamine producing
rarely do infants demonstrate failure to thrive. For tumors that account for < 1% of cases of hyper-
the most part, infant GERD peaks at 4 months and tension. They are most commonly found in the
resolves by 12 months with nearly all cases resolved adrenal medulla although 10% are extra-adrenal (the
by 24 months of age (as the lower esophageal rule of"10s" in pheochromocytoma: 10% bilateral,
sphincter becomes more competent with age). 10% malignant, 10% extra-adrenal, 10% associ-
ated with familial disorders, e.g., multiple endocrine
~ Answer C. Due to its anterior position and size, neoplasia). The diagnosis is based on detection of
the right ventricle is the most commonly affected urinary metanephrines and vanillylmandelic acid
cardiac chamber from penetrating thoracic injury. (VMA) and has a diagnostic sensitivity of 98%
The left ventricle is affected next, followed by for detecting pheochromocytoma. Assays for plasma
both atria equally. Multiple chambers are injured metanephrines may also be performed. Finally, CT
in almost one third of cases. Death occurs from or MRI is used for tumor localization. Hypertension
either exsanguination or pericardia! tamponade, the has traditionally been treated with phenoxybenza-
latter of which is treated by ED thoracotomy and mine, an a-blocker, whereas ,8-blockers should be
pericardia! incision. avoided (,8 -blockers lead to unopposed a stimulation
which may result in a worsening of hypertension).
~ Answer D. The SIRS was devised to be able to Due to associated peripheral edema and orthostatic
formally describe basic vital sign and laboratory hypotension, other a-blockers have also been used
abnormalities associated with severe inflammation, such as terazosin or doxazosin.
such as infection, trauma, burns, and severe medical
illness. SIRS is said to be present when two out of the ~ Answer B. This patient has Dysfunctional uterine
following four criteria are present: bleeding (DUB), which is defined as excessive,
• Temp >38°C or <36°C. prolonged, or erratic uterine bleeding that is not
• WBC >12 Kor <4Kor >10% bands. related to an underlying anatomic uterine defect or
• RR >20 or Paco2 <32. systemic disease. Therefore, DUB is a diagnosis of
• HR>90. exclusion. However, anovulation is by far the most
Sepsis is the presence of SIRS with a suspected common cause of irregular vaginal bleeding in an
or proven infectious cause. Septic shock is defined as adolescent. Although anovulatory menstrual cycles
sepsis plus inadequate tissue perfusion, which may are most common in the first 2 to 3 years after
be manifested as hypotension or end-organ dys- menarche, it may take up to 6 years before most
function. Note that hypotension is not a criterion cycles become ovulatory. Oral contraceptive pills are
for SIRS or sepsis, but is present in most cases of very effective for the management ofDUB. Nearly any
septic shock. regimen can be used, and the most common involves
combination oral contraceptives (containing both
~ Answer E. A history of a persistent, foul-smelling, estrogen and progesterone) with at least 30 to 35 mg
bloody vaginal discharge is highly suggestive of a of ethinyl estradiol. The pills are initially used four
vaginal foreign body. Although the history of a times daily in women with more extensive bleeding
bloody discharge is helpful, few patients remember (and concomitantly low hemoglobin levels) and are
placing a foreign body in their vagina and few gradually tapered by one pill every 3 days until only
parents witness the act. Therefore, ED physicians one pill is being used on a daily basis. Antiemetics
should have a high degree of suspicion whenever may be needed due to the nausea that is a frequent
a patient presents with a foul-smelling bloody side effect ofhigh -dose estrogen therapy. In addition,
discharge. Although many foreign bodies have been iron supplementation should be used to boost red
documented, small wads of toilet paper are most blood cell production.
common. Simple removal followed occasionally
by local irrigation, if necessary, is all that is ~ Answer E. Although acute pharyngitis is a com-
required. Antibiotics are not usually necessary unless mon reason for presentation to the emergency room,
a concomitant infectious vulvovaginitis is present GAS is the cause of a minority of infections, ranging
Poor perineal hygiene is the leading cause of from 5% to 30% (estimated to be 5% to 10% in
vulvovaginitis in the pediatric population. Bacterial adults, as it is a less common cause of adult pharyn-
pathogens responsible for the infection are typically gitis than pediatric pharyngitis). GAS is, however,
nonspecific. Ifthe predominant symptom is pruritus, the most common bacterial cause of the disease and,
pinworms (Enterobius) should be suspected. regardless of the etiology, it is the only common
27 2 1000 Questions to Help You Pass the Emergency Medicine Boards

cause that requires treatment. As the pharyngitis of the dural bridging veins, increasing the risk
is self-limited with or without treatment, the rea- of subdural hematoma formation after falls and
son for treating GAS pharyngitis is to prevent the relatively minor head trauma. Degenerative joint
sequelae of the disease, primarily acute rheumatic disease and osteoporosis result in an increased
fever. Acute rheumatic fever is a nonsuppurative tendency to fracture bones after falls and blunt
complication of GAS pharyngitis. The other rare trauma. Due to these degenerative changes, the
nonsuppurative complications include poststrepto- cervical spine is less mobile in elderly patients
coccal glomerulonephritis (PSGN) and streptococcal and is more commonly fractured. Type 2 odontoid
toxic shock syndrome. Suppurative complications fractures are the most common cervical spine
include peritonsillar cellulitis or abscess, otitis me- fractures in the elderly. In addition, hyperextension
dia, sinusitis, necrotizing fasciitis, and meningitis. injuries lead to central cord syndrome in which the
The ability of antibiotic therapy to prevent the de- ligamentum flavum is thought to buckle into the
velopment of suppurative complications is not well spinal cord, resulting in a contusion to the cord's
defined, but the effect is thought to be small. How- central elements. This results in flaccid paralysis
ever, antibiotics are thought to be more effective in of the upper extremities but relatively unaffected
reducing the rate of acute rheumatic fever, and they lower extremities (although they may suffer from
have been shown to decrease the length and sever- spastic paralysis in large cord lesions). The chest
ity of acute GAS pharyngitis. There is no definitive wall becomes more rigid and the lungs become less
evidence that antibiotic therapy reduces the rate of compliant in elderly patients. This places them at
PSGN. Acute rheumatic fever typically occurs 2 to risk for flail chest, even from relatively minor injuries
4 weeks after acute pharyngitis develops, and most such as a simple fall. Elderly patients with even single
commonly occurs in children 4 to 9 years old. The rib fractures should be admitted in the setting of
diagnosis of rheumatic fever is made in the presence concomitant lung disease such as COPD.
of one or more of the five major Jones criteria in
association with an antecedent GAS pharyngitis. The ~ Answer C. Magnesium depresses the CNS and
major Jones criteria include: pancarditis, migratory slows nerve conduction. It is used in preeclampsia
arthritis, CNS involvement (classically Sydenham's to prevent progression to eclampsia, which is
chorea), erythema marginatum (diffuse rash sparing characterized by the presence ofseizures. Magnesium
the face), and subcutaneous nodules. slows neuromuscular conduction and decreases CNS
irritability. Remembering this provides an easy means
~ Answer D. Aplastic anemia causes normocytic of remembering the actions of magnesium. It will
anemia and pancytopenia due to immune-mediated decrease the respiratory rate, decrease deep tendon
bone marrow destruction. It is caused by drug or reflexes, and decrease the degree of consciousness.
toxin exposure in half of all cases, and viral and The loss of deep tendon reflexes is generally the first
primary immune etiologies make up the remainder. sign of magnesium toxicity.
Chloramphenicol is the drug most often associated
with aplastic anemia. Other medications implicated ~ Answer B. Meningococcemia refers to systemic
include antiepileptics and sulfonamides. Most other infection with Neisseria meningitidis, a gram-negative
antibiotics do not predispose to aplastic crises. diplococcus. Mortality is as high as 50%, due to
multiorgan failure from septic shock which can
~ Answer D. This patient has Epstein-Barr virus occur within hours. Fever and rash occur in most
(EBV) pharyngitis, also known as mononucleosis. patients. Fifty percent of patients present with
As it is a viral infection, antibiotics have no true petechiae, and another 20% to 30% exhibit a
role in treatment. Furthermore, the specific use maculopapular rash which later turns into petechiae
of amoxicillin may result in the development of or purpura. Bilateral adrenal infarction, part of a
a morbilliform (i.e., measles-like) rash. Although constellation of signs known as the Waterhouse-
this does not represent a true allergy, nor is Friderichsen syndrome, occurs in approximately 10%
it dangerous to the patient, it is an irritating of cases. Hypothermia, seizure, and arthritis each
and unnecessary outcome. The tonsillitis in EBV occurs < 10% of the time. Laboratory studies may
infections is nonexudative and may be considerably demonstrate a significant leukocytosis {although
large, rarely resulting in airway obstruction. leukopenia, when present, is a poor prognostic
indicator), thrombocytopenia, and disseminated
~ Answer E. There are several normal physiologic intravascular coagulation (DIC). Treatment is with
changes that occur with aging and that place a third-generation cephalosporin and aggressive
elderly patients at greater risk of serious injury management of shock (fluids, vasoactive agents,
from trauma. Cerebral atrophy results in stretching intensive care unit [ICU] monitoring).
Test9 273

~ Answer A. There is considerable overlap in the best seen on an anteroposterior (AP) wrist or dedi-
clinical presentation of patients with acute cholecys- cated scaphoid view. Perilunate dislocation involves
titis and acute cholangitis. However, patients with the capitate dislocating relative to the lunate and
acute cholecystitis rarely exhibit jaundice and tend radius. Distal radius fractures and metacarpal frac-
to be less toxic-appearing. Although the cystic duct tures are best seen on AP views. (Figure courtesy
is usually blocked in acute cholecystitis, the hepatic of Silverberg M. Greenberg's Text-Atlas ofEmergency
and common bile ducts are patent and free of infec- Medicine, Lippincott Williams & Wilkins; 2005.)
tion and inflammation. Charcot's triad (fever, right
upper quadrant pain, jaundice) is the hallmark of ~ Answer B. Coumadin directly inhibits vitamin
acute cholangitis. Fever is nearly universal, present K-mediated synthesis ofthe following factors: II, VII,
in 95% of patients, right upper quadrant tenderness IX, and X. Indirect inhibition of factor I (fibrin) also
in 90% and jaundice in 80%. Hypotension and al- occurs due to the downstream effects ofreduced levels
tered mental status are present in 15% of patients of factor II (prothrombin). The prothrombin time is
and suggests gram-negative sepsis. When present prolonged in patients who are taking coumadin and
in concert with Charcot's triad, these findings are is used to track therapeutic anticoagulation.
known as Reynold's pentad. Although mildly elevated
bilirubin levels may be present in patients with acute ~ Answer D. The fact that cooling improves the
cholecystitis, these levels rarely rise above 4 mg per symptoms of myasthenia gravis (MG) is the basis
dL. for the "ice test." When ptosis is present, an ice
pack is placed over the affected eye for 2 minutes.
[g) Answer C. The patient is at risk for traumatic It is thought that the local cooling results in a
aortic injury (TAl), which is a common cause of slowing of the kinetics of the acetylcholine receptor,
immediate death in motor vehicle crashes. TAI may allowing for a prolonged effect ofacetylcholine and an
result in sudden hemodynamic instability and death improvement in symptoms. In clinical studies, 80%
in patients who initially appear to be stable after of patients with ptosis due to MG experienced some
blunt trauma. Lateral and front-impact motor vehicle improvement with a locally applied ice pack. MG has
crashes at high speed, steering wheel impact to chest, a bimodal peak of incidence with the first peak in
and sudden deceleration injuries each constitutes a the 20s and the second peak in the 50s. Interestingly,
high-risk mechanism for TAl. Although a normal women are more commonly affected during the first
chest x-ray has good sensitivity ("-'90%), patients peak, but men are more commonly affected in the
who have a high-risk mechanism and symptoms second peak. Sensory loss is not a feature of MG.
consistent with the diagnosis ofTAI should undergo The most frequent initial symptoms ofMG are ptosis
CT angiography (negative predictive value of close and diplopia due to ocular muscular weakness or
to 100%). Diagnostic peritoneal lavage is usually weakness of the levator palpebrae superioris.
indicated in patients who are hemodynamically
unstable to evaluate for presence of intraperitoneal ~ Answer E. Compartment syndrome refers to is-
injury. Pericardiocentesis is only indicated in patients chemia that occurs in extremities when pressure in
with pericardia! tamponade in whom a pericardia! the soft tissues exceeds that of the microcirculation.
window may not be performed at the bedside. Such high pressures occur from either increased con-
Admission to the hospital could eventually be tents in the compartment or external compression.
indicated, but a delay in the diagnosis of TAl Normal compartment pressure is zero, and pressures
could cause immediate death, and observation alone > 30 mm Hg are usually enough to predispose to
would be unhelpful in prevention of this occurrence. compartment syndrome. Long bone fractures are the
Discharging the patient home without any further usual reason for compartment syndrome, causing
workup for either TAl or blunt cardiac injury is extravasated blood and soft tissue edema to accumu-
contraindicated in a patient with such a high-risk late. Tibial fracture is the single most common cause
mechanism and suggestive symptoms. of compartment syndrome, but all the other regions
listed can also be involved. The most common symp-
~ Answer B. The lateral wrist radiograph demon- toms are pain and paresthesias. Diminished pulses
strates volar displacement and angulation of the generally occur only in extremely advanced cases, as
lunate relative to the radius, indicating a lunate dis- pressure in compartment syndrome is usually well
location. The "spilled teacup" sign is present with below arterial pressure. Diagnosis is made by directly
the lunate appearing as a teacup that is tilted for- measuring compartment syndromes with a Stryker
ward. Treatment involves orthopedic consultation needle device. Treatment is with urgent fasciotomy.
and urgent surgical repair. Scaphoid fractures are
274 1000 Questions to Help You Pass the Emergency Medicine Boards

~ Answer C. A luremolyticum usually affects the 10- cyanide molecule off the electron transport chain,
to 30-year-old age-group and may cause an infec- and detoxification of cyanide with thiosulfate. When
tion which is impossible to differentiate from GAS there is poisoning with both cyanide and carbon
pharyngitis with rash (also known as scarletfever). In a monoxide, induction of methemoglobinemia by ni-
patient in the appropriate age-group with all the signs trites should be avoided, as it would further prevent
and symptoms of GAS pharyngitis or scarlet fever, hemoglobin from carrying oxygen. The treatment
but with a negative throat culture and a poor response of choice in this case would be sodium thiosul-
to therapy, A haemolyticum should be considered. fate. Methylene blue is the treatment for severe
The drug of choice is erythromycin, as response methemoglobinemia. Dexamethasone has no role
to penicillin is generally poor, but highly variable. in the management of carbon monoxide or cyanide
On occasion, dermatologic findings may be the only poisonings.
manifestation of A. haemolyticum infection, includ-
ing scarlatiniform and urticarial rashes as well as ~ Answer A. Infection and loc:al irritation often
erythema multiforme. In addition, A luremolyticum exacerbate gingival hyperplasia caused by drugs such
may cause a membranous pharyngitis that closely as phenytoin. Poor oral hygiene is cited as the most
resembles diphtheria. Despite years of use to treat important contributing factor. Less than half of all
GAS pharyngitis, penicillin resistance has not devel- patients on phenytoin exhibit gingival hyperplasia
oped among GAS isolates. Epstein-Barr virus (EBV) and the reaction is not dose related. Calcium channel
is associated with a nonexudative tonsillitis, posterior blockers such as nifedipine have also been implicated.
lymphadenopathy, splenomegaly, and an increased
number of atypical lymphocytes on the peripheral [!!) Answer D. Globe rupture is a true ophthalmologic
blood smear. Both M. pneumoniae and C. pneumo- emergency, usually requiring operative care. Antibi-
niae may cause pharyngitis that typically occurs in otics and tetanus boosters should be given to all
epidemics due to crowded conditions. patients with suspected globe rupture to prevent
infectious complications. Succinylcholine without
[!!] Answer E. The figure demonstrates a tympanic pretreatment with nondepolarizing paralytics can
membrane (TM) perforation in the setting of head increase intraocular pressure. Tonometry is con-
trauma. Treatment of traumatic TM perforations traindicated as this will also increase intraocular
in a dry environment is purely supportive with pressure. Eye shielding is mandatory to prevent fur-
close ENT follow-up. Perforations in a wet envi- ther damage to the injured eye and to restrict eye
ronment require prophylaxis with antipseudomonal movement.
antibiotics. Perforations associated with preceding
symptoms of otitis media also require standard an- ~ Answer B. More than three fourths of all patients
tibiotic therapy with an aminopenicillin. All patients with AIDS will develop Pneumocystis carinii pneumo-
should be instructed to keep the ear canal dry. Heal- nia (PCP) at some point in their lifetimes. It is also the
ing of TM perforations occurs over several weeks most common identifiable cause of death in AIDS
to months. (Figure from Benjamin B, Bingham B, patients. Pneumocystis is classified as a protozoan,
Hawke M, et al. A color atlas of otorhinolaryngol- but has many characteristics of a fungus. Symptoms
ogy. Philadelphia: JB Lippincott Co; 1995. Artwork of PCP, like all pneumonias, include fever, cough,
© Bruce Benjamin, Brian Bingham, Michael Hawke, and shortness of breath, but a subacute or mild
and Heniz Stammberger, with permission.) course is characteristic. Chest radiography classically
demonstrates diffuse, bilateral interstitial infiltrates,
[!!] Answer C. Patients with smoke inhalation from but can be completely normal up to 20% of the time.
building fires are exposed to two major toxins, carbon First-line therapy is with TMP-SMX. Corticosteroids
monoxide and hydrogen cyanide. A metabolic acido- are indicated in patients who have significant hy-
sis with elevated lactate level in these patients is highly poxia. Pentamidine, dapsone, or clindamycin plus
suggestive of dual toxicity. Carbon monoxide binds primaquine may be used as alternatives. The other
to hemoglobin with more than 200 times greater answer choices are all common opportunistic infec-
affinity than oxygen, and prevents hemoglobin from tions in AIDS patients, but occur less often than
carrying oxygen. Cyanide poisons complex N of the Pneumocystis.
mitochondrial electron transport chain. Treatment
ofcarbon monoxide alone is 100% oxygen and hyper- ~ Answer D. Acute angle-closure glaucoma is caused
baric oxygen is severe cases. Treatment of hydrogen by increased pressure in the anterior chamber due to
cyanide alone involves two major steps: Induction decreased outflow ofaqueous humor. Acetazolamide
of methemoglobinemia with nitrites, which pulls the can increase the excretion of aqueous humor and is
Test9 275

indicated as one of the medical therapies, along with on colonic motility predisposing to volvulus. It is
pilocarpine, timolol, and antiemetics. Cycloplegics also frequently associated with chronic constipation.
inhibit ciliary muscle contraction (which limits mio- Patients with small bowel obstruction usually have
sis), and are contraindicated in glaucoma, as they a history of vomiting, and plain films would
further inhibit aqueous humor outflow from the an- reveal circumferential plica circularis instead of
terior chamber. Aspirin and antivirals play no role in non-circumferential haustrae of the large bowel.
the management of glaucoma. Lateral canthotomy is Intussusception is rare in adults andx-rayfindings are
used to treat retrobulbar hematoma. Definitive ther- neither sensitive nor specific. Diabetic gastroparesis
apy for acute angle closure glaucoma is surgical and would demonstrate a large gastric bubble or dilation
emergent ophthalmologic consultation is necessary. rather than colonic dilation. Almost all patients with
Hirshsprung's disease are diagnosed before the age of
~ Answer C. There is some variability in the litera- two. (Figure reprinted with permission from Harris
ture regarding the definition of massive hemoptysis. JH. The radiology of emergency medicine, 4th ed.
Most authors define massive hemoptysis as expec- Lippincott Williams & Wilkins; 1999.)
torated blood >600 mL per 24 hours. Since it is
impossible to accurately quantify the amount of ~ Answer E. Patients younger than 8 weeks of age
bleeding before presentation, a supplemental defi- with fever should be admitted to the hospital for
nition is hemoptysis occurring in the ED > 100 mL observation, culture analysis, and prophylactic an-
with signs of respiratory distress. Chest x-rays are tibiotics. In this age-group, serious bacterial infection
normal or nonlocalizing in up to 40% of patients is common and often completely undetectable by
with hemoptysis. In such patients, a combination of physical examination or routine blood tests. Pa-
high-resolution CT, bronchoscopy, or ventilation/ tients between 8 and 12 weeks may be assessed
pefusion (VJQ) scanning is typically able to di- for toxic appearance before further evaluation, but
agnose the cause of hemoptysis. Bleeding may aggressive management with cultures and antibi-
come from alveolar capillaries, bronchial capillaries, otics in this population is preferable. The potential
bronchial arteries, or pulmonary arteries. Bleeding presence of otitis media should never result in outpa-
from bronchial and alveolar capillaries is usually the tient management of the febrile neonate. Antibiotic
result of infection and inflammation. Bleeding from administration without appropriate culture and lab-
pulmonary arteries is rare, and may occur when oratory analysis results in the inability to diagnose
tumors erode into the pulmonary vasculature. The serious bacterial infection.
higher systemic pressure bronchial arteries are the
source ofbleeding in 90% ofcases ofmassive hemop- [ll) Answer A. Only 5% ofintussusception cases occur
tysis. Most authors recommend keeping the affected in adults and almost all of them occur in the
lung in the dependent position in cases of hemop- small intestine. In contrast to pediatric patients,
tysis limited to one lung. This prevents flooding of adults with intussusception almost always have an
the unaffected lung with blood, thereby maintaining identifiable lead point, which is a malignancy 75% of
adequate ventilation and preventing complications the time (other lesions include inflammatory lesions
related to aspiration. Bronchial artery embolization and Meckel's diverticulum). In adults, the diagnosis
is the most effective nonsurgical treatment of mas- is best made with a CT scan. Although barium or
sive hemoptysis. Angiography is used to localize the water contrast enemas may diagnose and reduce
bleeding followed by the injection of embolizing rna- intussusception, it is not as useful in adults because
terials such as Gelfoam or steel coils to stop bleeding. most lesions are in the small intestine and it is thought
The 24-hour success rate approaches 98%. Although that the contrast material may help spread malignant
16% of patients may experience rebleeding over the cells. Most patients present with signs of incomplete
next 30 days, bronchial artery embolization will at obstruction (only 20% of patients have complete
least serve as a bridge to more definitive surgical obstruction) with a chief complaint of abdominal
management in such patients. Despite this, a tho- pain. Large bowel obstruction in adults is most
racic surgical consult is recommended in all cases of commonly due to malignancy, with volvulus and
massive hemoptysis. diverticulitis being the next most common causes.

~ Answer A. The image shows sigmoid volvulus. [!jJ Answer E. Patients receiving angiotensin convert-
Sigmoid volvulus is primarily a disease of the elderly ing enzyme (ACE) inhibitors are at risk for devel-
as well as patients in long-term care facilities and oping angioedema, which is a nonpitting, symmetric
patients with neurologic or psychiatric disease. It is edema that typically involves the face, tongue, and
thought that medications used to treat psychiatric supraglottic tissues. Most patients present with lip
and neurologic disease may have detrimental effects and tongue swelling and do not have either urticaria
27 6 1000 Questions to Help You Pass the Emergency Medicine Boards

or pruritus. The response is most common within placed in the fourth or fifth intercostal space at the
the first week of therapy but can occur months mid- or anterior axillary lines to minimize cosmetic
or years after starting the drug. It is not safe for defects. A large tube (36-French or larger) should be
patients developing angioedema due to ACEI ther- placed in adults with traumatic pneumothoraces in
apy to take angiotensin-receptor blockers. Finally, case a hemothorax is also present.
patients with angioedema due to ACEI therapy may
be resistant to all first-line agents for anaphylaxis [!!) Answer C. Patients with suspected urinary tract
including epinephrine, corticosteroids, and antihis- infections require either urethral catheterization or
tamines. Symptoms typically resolve within 24 to a midstream clean catch specimen for adequate
48 hours of discontinuation of the drug, but elective sampling and culture. In a 12-month-old child,
intubation should be performed early in the course midstream clean catch would be extremely difficult.
if there is any sign of respiratory compromise. Typ- Diaper and bag collection methods are notoriously
ical "rescue" airways may fail in angioedema due nonspecific and should never be used. Suprapubic
to edema of the glottic structures. Therefore, ENT catheterization would be necessary only in cases
and anesthesia should be involved in the case, and of urethral anatomic abnormalities where direct
intubations without the use of paralytics should be urethral catheterization would be contraindicated.
considered.
~ Answer D. There are numerous risk factors for
~ Answer C. The patient has been adequately evalu- pulmonary embolism (PE), which can be divided into
ated for bony injury of her cervical spine, including inherited and acquired disorders. Inherited hyper-
cervical spine x-rays and CT scan. However, lig- coagulable disorders should be suspected in patients
amentous injury has not yet been ruled out. A with a documented PE who are younger than 40 years
flexion-extension series or MRI may be used to old, or in similar patients who have a positive family
evaluate for ligamentous injury. The patient has a history or who have recurrent PEs. Of the inherited
pacemaker, which is a contraindication for MRI, so disorders, factor V Leiden is the most common, and
a flexion-extension series is indicated. Subluxation may be present in as many as 20% of patients with
seen on flexion-extension films is indicative of a sig- venous thromboembolic disease. In normal individ-
nificant ligamentous injury and must be evaluated by uals, factor V is normally inactivated (along with
a spine surgeon before discharge planning. Oblique factor VIII) by protein C, thereby disrupting the nor-
films add nothing to the evaluation of ligamentous mal clotting cascade. Patients with factor V leiden
injury and are useful only to evaluate for laminar have a factor V protein that is resistant to inactivation
fracture or unilateral facet dislocation. Addition- by protein C, which allows the coagulation cascade
ally, reasons for the motor vehicle collision should to continue in an unregulated manner resulting in
be aggressively sought-with a recent pacemaker a thrombophilic state. Plasminogen deficiency and
placement for sick sinus syndrome, pacemaker mal- protein S deficiency are also inherited disorders that
function and dysrhythmia causing syncope may be increase the risk of thromboembolic disease, but
the causative process. Discharge, therefore, should they are far less common. Nephrotic syndrome is
not take place before both medical and traumatic also associated with an increased risk of PE due to a
issues are completely evaluated. Patients may be dis- relative deficiency of coagulation cascade regulatory
charged with a soft collar for 1 to 2 days to help proteins, which are lost in the urine. Cancer in any
with muscle spasm, but should never be discharged form is a well-described risk factor for PE.
with a hard collar in the absence of ligamentous or
bony injury, as this will quickly lead to neck muscle ~ Answer A. Patients with tibial shaft fractures usu-
atrophy. ally have concomitant fibular fractures as well, due
to the close proximity of the two bones. Mechanism
~ Answer D. Patients with moderate traumatic of injury usually involves direct trauma; only rarely
pneumothoraces (>20%) generally require tube do pathologic fractures occur in the tibia. Due to the
thoracostomy to remove intrapleural air and pre- sparse soft tissue surrounding the anterior surface
vent conversion to a tension pneumothorax. Small of the tibia, fractures are often open and require
pneumothoraces (<15%) may be managed conser- emergent operative repair. The most common site
vatively, with supplemental oxygen and observation, for compartment syndrome is in the leg, due to
but should be monitored carefully for signs and tibial fracture. The common peroneal nerve is the
symptoms of deterioration. Needle thoracostomy is most common nerve to be injured in tibial fractures,
only indicated in patients with tension pneumotho- but this does not occur in most patients. Vascular
rax who are hemodynamically unstable and require compromise with tibial fractures is rare, but when
immediate decompression. Chest tubes are ideally present, mandates aggressive management.
Test9 277

~ Answer B. Oblique views of the cervical spine of drug. Management of asymptomatic patients
do not generally add significant sensitivity to the involves diagnosis with oral contrast radiographs,
standard three-view cervical spine series (lateral, AP, followed by bowel irrigation with polyethylene glycol
odontoid). Their primary utility is in identifying and admission to a monitored bed. Charcoal may be
fractures of the lamina and better characterizing ofbenefit in symptomatic patients shortly after packet
unilateral facet dislocations. Both these injuries may ingestion. Hemodialysis is used only if packets have
also be identified with CT scan, rendering oblique ruptured and drug has been absorbed, but cannot be
views less useful in current trauma evaluations ofthe set up quickly enough in these patients, as death will
cervical spine. Spinous process and pedicle fractures be sudden. Endoscopic removal is rarely indicated
are best seen on the lateral view. Vertebral body as rupture may occur during packet withdrawal.
fractures are seen well on the AP and lateral views. Operative removal is emergently indicated in patients
Dens fractures are the primary reason for obtaining with any symptoms suggestive of coaline packet
the open-mouth odontoid view of the cervical spine. rupture, as uncontrolled sympathomimetic crisis
will cause death despite even the most aggressive
~ Answer D. A large percentage of women, esti- non-surgical measures.
mated to be from 30% to 60%, experience abdominal
pain approximately 1 week after starting methotrex- ~ Answer B. The patient has central retinal artery
ate for ectopic pregnancy. This is known as separation occlusion (CRAO), probably caused by a throm-
pain as it is thought to result from tubal distension boembolus from atrial fibrillation or carotid
as a result of tubal abortion or hematoma formation. atherosclerosis. The conditions listed in her past
However, all women with a history of methotrex- medical history are all risk factors for CRAO. Inter-
ate treatment for ectopic pregnancy who present mittent globe massage is indicated to increase ocular
with abdominal pain merit further investigation by carbon dioxide content, which leads to vasodilation
ultrasonography to explore the possibility of tubal of the retinal arteries. Lateral canthotomy is used
rupture. Interestingly, the size of the ectopic mass to treat retrobulbar hematoma and has no role in
may actually increase before involution, but this management here. Aspirin, heparin, and TPA have
finding has not been shown to be associated with not been shown to improve outcomes in CRAO.
treatment failure. However, if patients have an in-
crease in the amount ofpelvic free fluid or a decrease ~ Answer D. Corneal abrasions should be treated
in their hemoglobin, a presumptive diagnosis of with prophylactic topical antibiotics. Short-term
tubal rupture should be made and an OB should be cycloplegics may also be used to reduce the ciliary
urgently consulted. Although it is a risk factor for spasm associated with many abrasions. Eye patching
ectopic pregnancy, active PID at the same time as has been shown to increase infection rates. Topical
pregnancy (ectopic or intrauterine) is extremely rare. anesthetics impede corneal healing if used beyond the
acute setting. Topical steroids are indicated only in
[!!) Answer B. Coronary vasospasm is an important cases ofiritis and are absolutely contraindicated when
cause of nonatherosclerotic MI. It may be idiopathic the possibility of herpetic infection exists. Topical
or caused by certain drugs, such as cocaine or saline solution may be used in corneal abrasions
vasopressin. It is clinically indistinguishable from as a remoisturizing medium, but does not reduce
coronary thrombosis and can only be diagnosed with superinfection rates or improve healing.
cardiac catheterization. Treatment in the ED should
be exactly the same as a STEMI due to coronary ~ Answer C. Lumbar puncture in healthy adults is
occlusion. normally 8 to 18 em ofwater. Children have a normal
range of 3--6 em of water. Opening pressure can
lilJ Answer A. People ingest packets of illicit drugs only accurately be measured in the lateral decubitus
to evade law enforcement officials by two main position.
methods-packing and stuffing. Body packers ingest
large amounts of drug in well-sealed packets in a ~ Answer D. All pregnant patients with a viable
methodical manner; body stuffers, on the other fetus (defined as 24 weeks' gestation and beyond)
hand, are under time pressure to ingest a small require continuous cardiotocographic monitoring
number of packets quickly to avoid being captured for 4 hours before discharge. Even in the setting
with the drugs in their possession. Body packers of "minor" trauma, approximately 4% of pregnant
are less likely than body stuffers to have packets patients will develop placental abruption. In the
rupture, but more likely to die from ruptured setting of major trauma or in the presence of
packets, as they usually contain a large amount vaginal bleeding or any uterine contractions, patients
278 1000 Questions to Help You Pass the Emergency Medicine Boards

should be admitted for 24 hours of cardiotocographic bilateral adnexal tenderness with significant cervical
monitoring. Cardiotocographic monitoring is the motion tenderness and cervical discharge. PID is a
most sensitive indicator of trauma-related fetal clinical diagnosis with laboratory and imaging stud-
distress. Furthermore, as the fetus is more sensitive ies useful only to rule out other causes of symptoms.
than the mother to decreases in maternal blood Treatment involves antibiotics to cover Chlamydia
pressure and blood flow, fetal distress can be an early and gonococcus-the most common regimen is IM
indicator of occult maternal shock. Most fetal losses ceftriax:one plus doxycycline/azithromycin. Sequelae
in trauma occur due to placental abruption and of untreated PID are extremely serious, including
cardiotocographic monitoring is the most sensitive tubal scarring causing infertility and ectopic preg-
marker of fetal distress due to abruption. In contrast, nancy, chronic pelvic pain, and tuba-ovarian abscess.
ultrasonography is notoriously insensitive, detecting
only 50% ofplacental abruptions. Finally, in patients ~ Answer C. The Pneumonia Severity Index (PSI)
with a nonviable fetus, intermittent documentation is a scoring system which was first developed in a
of fetal heart tones, as well as ultrasonography to retrospective, observational study by Fine et al. in
assess fetal viability are probably adequate. 1997. The aim of the study was to develop a rule to
identify patients with low 30-day mortality from
~ Answer C. UGIB is defined as bleeding that community-acquired pneumonia. Calculating the
originates proximal to the ligament of Treitz. It PSI is a two-step process. In step one, the physician
usually manifests as hematemesis or melena, but brisk determines if a patient has any "high-risk" criteria,
UGIBs can cause hematochezia as well. Peptic ulcer the presence of which is independently associated
disease (PUD) accounts for SO% to 80% of UGIB. with mortality. These criteria include patients older
Esophageal varices are the most common cause of than SO, any of five comorbid diseases (neoplastic
"severe and persistent" bleeding, accounting for 33% disease, congestive heart failure, cerebrovascular
of such cases. After PUD, gastric erosions and varices disease, renal disease, and liver disease), and five
are the next most common causes of UGIB. The physical examination findings (altered mental status,
etiology remains uncertain in some cases as well. pulse 2:::125 per minute, respiratory rate 2:::30 per
minute, systolic blood pressure <90 mm Hg, and
~ Answer E. The patient has an infected kidney temperature, <35°C or 2::40°C). Patients without
stone, which, at 7 mm, is very unlikely to pass any of these criteria are classified as risk class I. All
spontaneously. This represents a true emergency other patients are assigned to risk classes II, III, IV, or
and will likely require specific urologic management. V based on the number of points calculated in step 2
The patient should be admitted to the hospital and of the PSI. In step 2, patients are assigned points for
given intravenous fluids, analgesics, antiemetics, and each risk factor that they have. Of all factors, age is
antibiotics after urinary culture has been sent. Other the most heavily weighted in calculating the PSI.
indications for emergent urologic consultation in
patients with kidney stones are the presence of ~ Answer D. Contrast-induced nephrotoxicity oc-
acute renal failure, high-grade obstruction due to curs more often in patients who have preexisting
the calculus in a patient with only one kidney, renal insufficiency, diabetes, hypovolemia, and mul-
and a stone >5 mm with intractable symptoms tiple myeloma. Age is also an independent risk factor
of pain and/or nausea. Discharging the patient with for development of contrast-induced acute tubu-
an infected kidney stone without urologic approval lar necrosis. Prevention is with adequate hydration,
is contraindicated. MRI will not add significantly to limiting the dose of the contrast material, and pre-
this patient's diagnosis or management and is an treatment with N -acetylcysteine, which appears to
unnecessary waste of time. attenuate the toxic effects of the contrast (although
the ED indications for this are controversial and not
~ Answer A. Pelvic inflammatory disease (PID) is universally accepted). It is important for the EP to
an infection of the fallopian tubes. Risk factors in- recognize patients who are at risk for developing
dude young age (15 to 25 is the highest risk group), renal injury and limit IV contrast administration for
multiple sexual partners, smoking, and bacterial vagi- patients with equivocal indications for its use.
nosis. It is caused by Chlamydia, gonococcus, and
organisms which cause bacterial vaginosis. The peak ~ Answer E. Patients with twisting injuries of the
time of onset is within 1 week of menses, as men- knee often present acutely to the ED. Physical
strual flow is thought to provide an optimal culture examination findings to assess for ligamentous
medium for bacterial ascension. Symptoms include instability (anterior drawer, varus/valgus stress,
diffuse pelvic pain, fever, nausea, vomiting, vaginal Lachman) are often unreliable in the acute setting
discharge, and dyspareunia. Patients usually exhibit due to limited range of motion from acute effusion.
Test9 279

No specific predictions should be made by the be on the same spectrum of disease. The toxin-
EP regarding prognosis for knee injuries. Follow- forming bacterium E. coli 0157:H7 is responsible for
up examination by an orthopedist with possible most epidemic cases of HUS. Treatment is primar-
MRI should be the standard evaluation for most ily supportive, aimed at preventing complications
nontraumatic knee injuries. Distal femur or proximal of severe anemia and thrombocytopenia. Plasma-
tibial fractures in young, healthy patients without pheresis is used for cases of idiopathic HUS or TTP.
trauma are extremely rare. Knee dislocation and Henoch-Schonlein purpura is a vasculitis heralded
spontaneous relocation would demonstrate evidence by renal dysfunction in the setting oflower extremity
of gross knee instability in all directions. palpable purpura, abdominal pain, and arthralgias.
Disseminated intravascular coagulation (DIC) is due
~ Answer D. The patient has evidence of Ludwig's to distortion ofthe clotting cascade from severe asso-
angina, a cellulitis of the connective tissues of the ciated illness. Idiopathic thrombocytopenic purpura
mouth and neck. The most commonly affected teeth causes thrombocytopenia without schistocyte forma-
are the molars--usually lower and posterior. Al- tion. Nephrotic syndrome causes renal dysfunction
though odontogenic infection is the most common without hematologic abnormalities. (Figure from
cause, trauma or oral malignancy may also pre- Anderson SC, Poulsen KB. Anderson's atkls of hema-
dispose to the condition. Ludwig's angina is usu- tology. Philadelphia: Lippincott Williams & Wilkins;
ally polymicrobial, and the most common species 2003, with permission.)
are Streptococci, Staphywcocci, and Bacteroides. Air-
way obstruction is the number one cause of mor- [tOOl Answer B. The patient has symmetric swelling
tality, which may be as high as 10% even in the of his foreskin behind his glans without active
presence of adequate therapy. Standard oral endo- retraction indicative of a paraphimosis. This is a
tracheal intubation and cricothyroidotomy may be urologic emergency that can occur in uncircumcised
difficult given the edema, secretions, and friabil- patients which requires prompt reduction to prevent
ity of tissues. Treatment involves high-dose peni- necrosis of the glans. Reduction is performed
cillin, clindamycin, or broader-spectrum agents such by squeezing the glans for several minutes to
as piperacillin-tazobactam or ampicillin-sulbactam. clear the capillaries of blood and pulling the
The role of steroids is not clearly defined at this foreskin over the glans. A dorsal slit procedure
time. Intensive care admission should be initiated may need to be performed if noninvasive reduction
and ENT should be consulted to address possible is unsuccessful. A phimosis refers to inability to
tracheostomy placement. retract the foreskin over the glans--it does not
require emergent reduction. Balllnitis refers to
[!!) Answer B. The peripheral blood smear demon- bacterial or fungal infection of the glans without
strates schistocytes, which, in the setting of renal concomitant foreskin infection (which would be
dysfunction, anemia, and diarrhea indicates the pres- called balllnoposthitis). Testicular torsion and scrotal
ence of hemolytic uremic syndrome (HUS). The hernia are not often evident on simple inspection and
triad of anemia, thrombocytopenia, and renal insuf- require palpation and ultrasonography to definitively
ficiency should prompt evaluation for either HUS diagnose. (Figure from Fleisher GR, Ludwig S, Baskin
or thrombotic thrombocytopenic purpura (TTP). MN, eds. Atkls ofpediatric emergency medicine, 4th ed.
Fever and neurologic signs and symptoms are more Philadelphia: Lippincott Williams & Wilkins; 2004,
common in the latter, but the two are thought to with permission.)
Test 10
Questions
(I) Which of the following is true regarding isopropanol Figure 10-1. Which of the following is the most likely
poisoning? diagnosis~

(A) Blood urea nitrogen (BUN) may be falsely


elevated.
(B) Ketosis without acidosis is the characteristic lab
abnormality.
(C) Elevated anion gap is seen in most patients.
(D) OcuJar accumulation causing blindness is the
typical pathophysiologic finding.
(E) Fomepizole should be administered empirically
in patients with high suapidon of iaopropanol
poisoning.

00 Which of the following studies has the highest


sensitivity for traumatic pericardial tamponade?
(A) Anteroposterior (AP) chest x-ray
(B) Lateral chest x-ray
(C) Electrocardiogram (EKG)
(D) Focused assessment of sonography in trauma
Figun 1~1.
(FAST) scan
(E) Diagnostic peritoneal lavage (DPL)

(!] Which of the following is true regarding genital (A) Pneumothorax


herpes simplex infection? (B) Small bowel rupture
(C) Duodenal hematoma
(A) The primary attack is uaually more severe than
(D) Diaphragmatic rupture
recurrent episodes. (E) Hemothorax
(B) Lesions are almost always painless.
(C) The Tzanck smear has >90% sensitivity and (!] A 42-year-old man presents with left foot pain.
specificity. He states his foot folded underneath him after he
(D) Vesicles should be unroofed to allow drainage jumped from a short ledge during a game with his
of fluid. son. His body weight first landed on his plantar-
(E) Antibiotics are indicated during outbreaks to
flexed foot and then the foot folded beneath him. He
prevent bacterial superinfection. now has difficulty ambulating. An x-ray is shown in
Figure 10-2. Which of the following is true?
[!) The moat common cause of lower gastrointestinal
bleeding (LGIB) in children is (A) The fracture fragment indicates that the
ligaments are intact.
(A) Anal fissure. (B) This is a stable injury.
(B) Hemorrhoids. (C) Degenerative arthritis is the most common
(C) Henoch-Schonlem purpura. complication.
(D) Food allergy. (D) Subtle plantar displacement is sometimes the
(E) Meckel's diverticulum. only sign of this injury.
(E) The posterior tJ.oial artery may be damaged
(!) A 34-year-old man presents after a high-speed motor during this injury.
vehicle cruh. Chest x-ray is performed and shown in

280
TcstlO 281

discharge in the vaginal vault with a tender uterus.


The most likely cause of her symptoms is
(A) Abruptio placentae.
(B) Pelvic inflammatory disease.
(C) Trichomonas.
(D) Pyelonephritis.
(E) Chorioamnionitis.

(jg) Which of the following has the greatest immediate


effect on preload in the management of acute
congestive heart failure (CHF)?
(A) Morphine
(B) Enalaprilat
(C) Digoxin
(D) Furosemide
(E) Nitroglycerin

(ll) Which ofthe following is true regarding candidiasis?


(A) Cutaneous candidiasis is the most common
manifestation ofinfection.
Figure 10.2. (B) Candida is part of the normal oral flora in most
humans.
(C) Thrush in otherwise healthy newborns is
self-limited and does not require treatment.

mWhich of the following most places women at risk


for abruptio placentae?
(D) Candida is the most common cause of jock itch
(tinea cruris).
(E) Maceration and lichenification with thick scale
(A) Preeclampsia is the hallmark ofcutaneous candidiasis.
(B) Ogarette smoking
(C) Premature rupture of membranes (PROM) (!i) An 85-year-old woman presents from a nursing
(D) Cocaine use home with fever and hypotension. According to
(E) Trauma the paramedic history, she began to have lethargy
and refused to get out of bed the day before.
(!] Which of the following is true regarding imaging of There is no reported history of cough, shortness
patients with suspected sinusitis~ of breath, diarrhea, headache, or rash. Her vital
(A) Plain 61ms are more accurate in diagnosing signs are: 102.4°P, 122, 22, 72/44, 95% RA. The
frontal and ethmoid than ma.,11ary sinusitis. patient's physical examination is nonfocal, and
(B) Water's view plain film is the most sensitive test routine laboratory work is sent. Which of the
for maxillary sinusitis. following is the most appropriate empiric antibiotic
(C) Computed tomography (CT) scans are able to therapy at this time?
differentiate between acute bacterial and viral (A) Ampicillin
sinusitis. (B) Ampicillin and gentamicin
(D) CT scans are both highly sensitive and highly (C) Vancomycin and piperadllin-tazobactam
specific in the diagnosis of sinusitis. (D) Vancomycin, piperacillin-tazobactam,
(E) Most patients with viral upper respiratory metronidazole
infections (URis) have abnormal CT scan (E) Vancomycin, piperacillin-tazobactam,
findings. clindamycin

(!] An 18-year-old primigravida at 35 weeks' gestation [!j) Which ofthe following is true regarding the Ottawa
presents with abdominal pain, vaginal discharge and Ankle Rules (OAR)r
a low-grade fever. She states that she was well until (A) Patients who do not meet the OAR criteria
two days ago when, after coughing, she experienced never have an ankle fracture.
a small stream of 1luid running down her leg which (B) The OAR should not be applied to intoxicated
she thought was urine. Her temperature is 102°F and patients.
her pulse is 108. Her aamination revuls a yellowish
282 1000 ~ tDHdp YouPo~W the~ MetlidMBoGrds

(C) The OAR criteria include a positive eyes as wd. as numerous ••spots" on her trunk. but
"squeeze" test. now the rash has spread throughout her body. She
(D) The OAR can be applied to pediatric patients is uncomfortable and her skin is warm and tender.
older than 8 yean. Pressure applied to skin adjacent to the lesions seems
(E) The specificity of the OAR is roughly 90%. to extend the lesion into the normal appearing skin.
Inexploring her recent history, which ofthe following
~ A 47-year-old man with a history of alcoholism is most likely?
presents with a chief complaint of jaundice, right
upper quadrant pain, and a low-grade fever. He
had his last drink earlier in the day. Which of the
following is true?
(A) His AST and ALT levels will be more than ten
times normal
(B) He should be transferred for immediate liver
transplantation.
(C) The patient's fever suggests an infectious cause
of his symptoms is most likely.
(D) Reactive leukopenia is the most common
cellular laboratory abnormality.
(E) Hepatomegaly is the most common physical
examination finding.

(!!} Which of the following can be used to treat


hypertrophic cardiomyopathy?
(A) Digitalis
(B) Isoproterenol
(C) Furosemide
(D) Phenylephrine
(E) Metoprolol Figure to-3.

(!j) Which ofthe following Salter-Harris fractures carries


the poorest prognosis! (A) She recently had a viral upper respiratory
(A) Typel infection (URI).
(B) Typell (B) She was being treated for a urinary tract
(C) TypeDI infection.
(D) TypeiV (C) Shehashumanimmunodefi.ciencyvirus (HIV).
(E) TypeV (D) She was recently diagnosed with leukemia.
(E) She has gout.
Ill) A 60-year-old man presents with acute urinary
retention. He has been unable to urinate for (!!) A 23-year-old man presents with bilateral lower
the last 10 hours and has extreme discomfort in extremity flaccld paralysis 1 clay after being assaulted.
his lower abdomen. His physical examination is An anterior lumbar spinal cord injury is identified.
normal except for a distended suprapubic region Which of the following is the most appropriate
and nontender prostatic hypertrophy. He appears therapy at this time?
very uncomfortable. Which of the following is the (A) Normal saline hydration
most appropriate nat step in managementt (B) Naloxone
(A) Magnetic resonance imaging (MRI) of the (C) Methylprednisolone
abdomen (D) Vancomycin
(B) CT scan abdomen with and without IV contrast (E) Vasopressin
(C) Renal ultrasonography
(D) Complete blood count, chemistry panel (igJ Which of the following characteristics are true
(E) Foley catheter placement regarding Kernig's and Bn1dzjnski's signs in the
diagnosis of meningitis?
(!j) A 24-year-old woman presents with a diffuse rash (A) High sensitiYity but low specifi.city
(see Pig. 10-3). She had ••sores" of her mouth and (B) Low sensitiYity but high specifi.city
TcstlO 283

(C) High sensitivity and specificity cultures are most likely to show which of the
(D) Low sensitivity and specificity fol1.owins etiologic organisms!
(E) Sensitivity and specificity depend on the (A) Staphy1ococcus aureus
prevalence ofthe disease in the population (B) Streptocom45 pneumonine
being studied (C) Streptocom45 viridans
(D) Pseudomorw aeruginosa
(ij} Which of the following spinal injuries is most likely (E) Candida albicans
to be stable?
(A) Flexion teardrop fracture (ij) Which of the following is true regarding creatine
(B) Bilateral facet dislocation kinase-myocardial bound (CK-MB) in acute my-
(C) Transverse process fracture ocardial infarction (MI)?
(D) Hangman's fracture (A) Starts elevating 1 hour after MI
(E) Jefferson's fracture (B) Peaks at 36 hours after MI
(C) Returns to baseline within 72 hours
(iii Which of the following is more characteristic of (D) Sensitivity at 3 hours after MI is 95%
endocarditis in IV drug users than in nondrug users? (E) Specificity at 3 hours after MI is 95%
(A) Higher mortality from Staphylocom45 aureus
(B) Audible heart murmur (i1) A 42-year-old woman who is a self-described
(C) Splinter hemorrhages "seafood fanatic" presents with a chief complaint
(D) Septic pulmonary emboli of an "allergic reaction.•• Thirty minutes after eating
(E) Roth spots tuna at a local restaurant she developed a severe
headache, palpitations, nausea, abdominal cramping
~ A 4-year-old boy is brought by his parents because and remarkable facial ftushing. She has eaten fish
he swallowed some of his grandmother's antihy- for her entire life without incident. Which of the
pertensive medicine. He is sleepy, bradycardic, and following is true?
hypotensive. Which of the following is the most (A) She should be given subcutaneous epinephrine
appropriate treatment? and parenteral corticosteroids.
(A) Atropine (B) Perioral paresthesias are typically a cl.assic
(B) Glucagon feature of this illness.
(C) Atropine and glucagon (C) Upon discharge, the patient should be advised
(D) Atropine, glucagon, and calcium to avoid all seafood products in the future.
(E) Atropine, glucagon, calcium, and high-dose (D) The patient should expect symptoms to resolve
insulin with glucose slowly over the course of 1 week.
(E) The symptoms are due to excessive histamine
~ A 72-year-old man develops acute onset of shaking levels in the fish.
chills and shortness of breath a few days after an
upper respiratory infection (URI). His past med- ~ A22-year-old man presentswithaswollenareaonhis
ical history is significant for diabetes and stable scrotum shown in Figure 10-4. The area is 6nn and
coronary artery disease. Vital signs are 101°F, ll5,
24, 144/95, 90% RA. Physical examination reveals
a patient with rigors and right lower lobe crack-
les. Which of the following is the most appropriate
next step in management!
(A) Doxycycline PO and discharge
(B) Azithromycin PO and discb.arJe
(C) Ceftriaxone and Azithromycin IV and admit
(D) CJjndamycin IV and admit
(E) Piperacillin-tazobactam IV and admit

(ii} A 22-year-old woman presents with intermittent


fever and chills for 2 weeks. She baa no past medical
history, but reports using intravenous heroin several
times a week. Physical examination reveals a febrile,
ill-appearing woman with a heart murmur. Blood
284 1000 ~ tDHdp YouPo~W the~ MetlidMBoGrds

nontender to palpation and does not transilluminate. (C) Miosis


Which of the following is the most appropriate nat (D) Nasalcongestion
step in management? (E) All of the above
(A) Emergent surgery
(B) Azithromycin PO ~ A34-year-oldmanpresentswithacuteonsetofpenile
(C) Outpatient urology referral pain and swelling which CKCWTed during sexual
(D) Scrotal elevation intercourse. His penis is shown in Figure I 0-5. Which
(E) Corticosteroids of the following is the most appropriate definitive
management?
(i!} A 64-year-old woman is brought in by emergency
services (EMS) with a chief complaint of "anaphy-
laxis." The patient has a known peanut allergy and
inadvertently ingested some ground nuts in a dish
prepared by a friend. She has a .known history of
coronary artery disease and is on metoprolol. Her
symptoms do not respond to epinephrine, corti-
costeroids, or antihistamines. She is intubated but
remains hypotensive and un&table. Which of the
following may be ofbenefiU
(A) Calcium chloride
(B) Atropine
(C) Glucagon
(D) Nebulized albutcrol
Figure 10-5.
(E) Terbutaline

~ Which of the following best supports a diagnosis of


(A) Observation
pertussis in a child with cough?
(B) Foley catheterization
(A) Pneumonia on chest x-ray (C) Surgical repair
(B) Eosinophilia (D) Penilesplinting
(C) Prolonged course (E) Penile pressure dressing
(D) Age older than 7 years
(E) Fever> 102°F ~ A 45-year-old woman with a history of hyperten-
sion presents with tongue swelling. as shown in
~ Which of the following is the recommended treat- Figure 10-6. She was recently started on a new
ment in a patient with group A P-hemolytic strepto-
coccal (GAS) pharyngitis and a penicillin allergy?
(A) Tetracycline
(B) Trimethoprim-sulfamethoxazole
(C) Levofloxacin
(D) Erythromycin
(E) Chloramphenicol

~ The most common cause of death in patients with


toxic epidermal neaolysis (TEN) is
(A) Electrolyte abnonnalities.
(B) Respiratory fiillure.
(C) Sepsis.
(D) Dehydration.
(E) Ventricular dysrhythmias.

~ Which of the following signs and symptoms may


accompany a cluster headache?
(A) Ptosis
(B) Lacrimation
Flgure1D-I.
TcstlO 285

blood pressure medication. Despite therapy with (B) Thiazide diuretics are more helpful than loop
steroids, antihistamines, and epinephrine, she is still diuretics in promotins urine calcium excretion.
worseJlins. In addition to measures for airway con- (C) Calcitonin can be used as monotherapy in the
trol. which ofthe following is most likely to be helpful treatment of hypercalcemia.
in this patientr (D) Sodium bicarbonate infusion will increase the
(A) Verapamil amount ofionized calcium.
(B) Metoprolol (E) Glucocorticoids may be helpful ifthe patient
(C) Fresh frozen plasma has an underlying hematologic malignancy or
(D) Danazol granulomatous disease.
(E) Aldoaterone
~ The development oflethargy, headache, and vomit-
~ Which of the following is true about Salmonella in- ing in a pediatric patient being treated for diabetic
fections? ketoacidosis (DKA) suggests the presence of:
(A) Seventy-five percent of household pets, such as (A) Meningitis.
dogs and cats, excrete Salmonella. (B) Cerebral edema.
(B) Salmonella requires only a very small inoculum (C) Worsening acidosis.
to result in symptomatic infection. (D) Central pontine myelinolysis.
(C) Nearly half the number of patients hospitalized (E) Hyponatremia.
with salmonellosis are immunocompromised.
(D) Treatment with cipro:Ooxacin is recommended ~ A 5-year-old boy presents with right hip pain. There
in all cases ofdocumented salmonellosis. is no history of trauma. Which of the following is the
(E) All of the above. most likely cause?
(A) Femoral neck fracture
fll) A 22-~ar-old man presents with rash, lighthead- (B) Osteogenesis imperfecta
edness, and generalized malaise. He denies fever or (C) Transient syno'litis
pruritus. A few hours before presentation, he was (D) SJippedcapitalfemoralepiphysis (SCFE)
seen in another emergency department (ED) and (E) Legg-Calve-Perthes disease
received treatment for syphilis. He denies any medi-
cation allergies. His vital signs are 99.2, 94, 16, 134/65, ~ Which of the following is the most common sign
99% RA. His physical examination demonstrates a or symptom in patients with normal pressure
normal uvula, no pulmonary wheezes, and a faint hydrocephalus (NPH)?
macular rash on his trunk and abdomen. which he (A) Ataxia
states was there before he receiftd the treatment to- (B) Dementia
day. Which of the following is the most appropriate (C) Papilledema
next step in management? (D) Urinaryincontinence
(A) Immediate endotracheal intubation (E) Headache
(B) 1M epinephrine
(C) Diphenhydramine and famotidine ~ A 24-year-old married woman presents to the ED
(D) Prednisone with the rash shown in Fig.I0-7. She notes that while
(E) Acetaminophen and observation

~ Which of the following is the most common


admisaion diagnosis in patients older than 65 years?
(A) Pericarditis
(B) Myocarditis
(C) Hypertensive emergency
(D) CHF
(E) ST-segment elevation myocardial infarction

~ A 40-year-old man presents with delirium, polyuria,


and a serum calcium level of 15 mg per dL. Which of
the following is true?
(A) Nifedipine is the agent of choice for initial
treatment.
FigllN10..7.
28& 1000 Qwstimu to Hdp You Ptw the Emerpnt;y Maicitw Bo.rmfs

she has been well, she had a recent "cold sore" that ~ Which of the following is the most common came of
resolved on it. own. She now complains of general Ludwijs angina?
malaise, mild arthralgia&. and a rash. The rash is
(A) Preaisting dental disease
symmetric and is spread over her a.tremities u weD
(B) Diabetes mellitus
as her palms and soles. Which of the following is true (C) Mandible fractures
about her illness? (D) Tongue piercing
(A) Muco.sal involvement u common. (E) Oral malignancy
(B) A positive N'tkolsky's sign will be present.
(C) Intravenous immunoglobulin is the only ~ A 42-year-old man presents with a swollen, erythe-
effective treatment. matous, and tender left knee. He is in a long-term
(D) The leaions are typically not pruritic. monogamous relationahip. A concomitant history of
(E) All of the above. recurrent renal stones in this man suggests a diagnosis
of.
~ Which of the following is usually the earliest symp-
(A) Reiter's syndrome.
tom to occur in patient. with aortic stenosis (AS)? (B) Systemic lupus erythematosus.
(A) Dyspnea on ewtion (C) Rheumatoid arthritb.
(B) Angina (D) Gout.
(C) Syncope (E) Nongonococcal septic arthritis.
(D) Orthopnea
(E) Paroxysmal nocturnal dyspnea ~ A 23-year-old man preaents after a stab wound to his
right lumbar paraspinal area. Which of the following
~ A 35-year-old woman presents with severe weakness, is the most likely neurologic deficitl
lightheadedness, and chest pain. Her blood pressure
(A) Left leg weakness and left-sided loss of pain
is 70/.W and her pubes are weak. The EKG is shown
sensation
in Figure 10-8. Which of the following is the most (B) Left leg weakness and right-sided loss of pain
important nat step in management?
sensation
(A) Adenosine 6 mg IV (C) Right leg weakness and left-sided loss of pain
(B) Diltiazem 20 mg IV sensation
(C) Amiodarone 300 mg IV (D) Right leg weakness and right-sided lou ofpain
(D) Synchronized Cardioversion at 50 J sensation
(E) Defibrillation at 200 J

llUlLU
illuLU_,-..jaV'--"'-' -""-
" aVI.

Agurw 10-1.
TcstlO 287

(E) Right arm and leg weakness and right-sided ~A 34-year-old man presents wi~ alten:d ~en~
loss of pain sensation status after being assaulted. Otic eDDWlatl.On u
shown in Figure 10-9. Which of the following is
~ Which of the following is true about vertigo? the most likdy diagnosis?
(A) Purely vertical nystagmus is almost always
consistent with peripheral vertigo.
(B) Nystagmus in central vertigo may change
direction.
(C) Patients with central and peripheral causes of
vertigo generally have equal dDficulty with gait
(ataxia).
(D) Nausea and vomiting are classically more
prominent in central vertigo.
(E) Nystagmus in central vertigo is inhibited with
fixation.

~ Which of the following is true regarding retinal


detachment?
(A) It is usually painfuL
(B) Presbyopia is a risk factor. Rgure 1~9.
(C) Visual acuity may be normal.
(D) The devated retina often appears dark red.
(E) Ophthalmologic follow-up is indicated in
(A) Otitis media
1 week.
(B) Anterior fossa skull fracture
(C) Middle fossa skull fracture
~ Which ofthe following is true regarding urinary tract
(D) Posterior fossa skull fracture
infectiOJlS in pregnan.cy!
(E) Nasal fracture
(A) Pydonephritis in pregnancy is almost always
left sided. ~ The hallmark of rubella is
(B) Asymptomatic bacteriuria is less common in
(A) Generalized lymphadenopathy.
pregnant women than in nonpregnant women.
(B) Tonsillar exudates.
(C) All pregnant women with asymptomatic
(C) Koplik's spots.
bacteriuria should receive antimicrobial
(D) Febrile seizures.
treatment.
(E) Pastia"s lines.
(D) Candida spp. is a common cause of urinary
tract infections in pregnant women.
(E) All of the above.
I!!) Which of the following muscles is the most com-
monly injured in rotator c:uff tears?
~ The Judetviewismosthdpful to image which. portion (A) Supraspinatus
of the pdvis? (B) Infraspinatus
(C) Subscapularis
(A) Inlet
(B) Oudet (D) Teres minor
(E) Deltoid
(C) Acetabulum
(D) Diacwing
(E) Pubic symphysis (@A 41-year-old man with ~betes and hyp~~i?D
presents with acute left &.cial weakness, the mability
~ Which of the following is a biochemical effect of to dose his left eye. and a loudness sensation in
tricyclic antidepressants? his left ear. His symptoms started yesterday and he
is concerned that he has had a stroke. He has no
(A) a-1 Agonist weakness or numbness in his left arm or leg. Which
(B) Dopamine reuptake inhibitor of the following ia true?
(C) Muscarinic agonist
(D) y-Aminobutyric acid (GADA) agonist (A) ACT scan ofhis brain should be performed to
(E) Antihistamine rule out the possibility of stroke.
288 1000 Qwstimu to Hdp You Ptw the Emerpnt;y Maicitw Bo.rmfs

(B) A vesicular raah over the ear indicates a better


prognosis.
(C) The patient should be given a prescription for
prednisone and acyclovir.
(D) The patient's diabetes and hypertension do not
affect the outcome ofhb illness.
(E) After recovery, he has a 50% chance of having
another epiaode in his lifetime.

(ii) Which of the following is the most common


symptom of rheumatic fever?
(A) Chorea
(B) Carditis
(C) Erythema marginatum
(D) Migratory polyarthritis Figure 10-10.
(E) Fever

Iii) Which ofthe following test results is most concerning (C) Neither A nor Bare a reflection ofBoehler's
in a patient with an acute asthma exacerbation? angle.
(D) Letter A is Boehler's angle and is used to
(A) Leukocytosis of 14,000
determine the integrity of the calcaneus.
(B) I(+ 3.4
(E) Boehler's angle is calculated by subtracting
(C) Hyperlactatemia
angle A from angle B and should be
(D) Eosinophilia
appromnatdy 40 degrees.
(E) Arterial blood gas (ABG) with pH 7.35, Po2 60,
Pca;z 45
(jjJ A 27-year-old man presents with a rash (see
Fig. 10-11). The patient states that hb wife :lint
~ Which of the followin8 heart valves does infective
noticed a single spot on his back approximately
end.oc:arditis most commonly involve in the patient
10 days ago. He denies fever or pain and com-
with rheumatic heart disease? plains only of mild pruritus. ~r the last few
(A) Tricuspid days, many new "spots" have cropped up on his
(B) Pulmonic back and tnmk. On examination, you note that
(C) Mitral
(D) Aortic
(E) Affects all vams equally
~ Which of the following is true regarding major
depressionf
(A) VISual hallucinations have high specifu:ity for
the diagnosis.
(B) Paranoid delusions have high sensitivity for the
diagnosl&.
(C) Either depressed mood or anhedonia is
required for the diagnosis.
(D) The prevalence of major depression is the same
as bipolar diaorder.
(E) In the elderly, depression is eu:lly distinguished
from dementia on clinical grounds.

(ijJ Regarding the imase (see Fig. 10-10):


(A) Letter B refers to Boehler's angle and the
normal measurement is between 20 and 40
degrea.
(B) Letter A refers to Boehler's angle and the
normal measurement is <20 degrees.
Figure 10-11. (See color insert.)
TcstlO 289

the lesions have a fine scale around the border shock. Which ofthe followins is the most appropriate
and seem to be arranged aloq the skin lines blood product to administer immediately!
of the back. Which of the followiq is the most (A) Type 0 negative crossm.atched blood
1!1--1 ..:~: ......, • ~
UNaY -ea•OSUI· (B) Type 0 negative uncrossmatched blood
(A) Secondary syphilis (C) Type 0 positive crossmatched blood
(B) Pityriasis rosea (D) Type 0 positive unaossmatched blood
(C) Molluscum contagiosum (E) Fresh frozen plasma
(D) Tinea corporis
(E) Atopic dermatitis (!g) An 18-month-old previouslyhealthychild is brought
in by his mother with bilateral oral commissure burns
~ Which of the following is the most common after biting an extension cord. He cried immediately
symptom in patients diagnosed with an abdominal after the injwy, did not lose consciousness, and
aortic aneurysm (AAA)? he has been acting normally since the injury. On
(A) Nausea examination, you note a 5-mm burn to both of the
(B) Abdominal distention patient's oral commissures. Which of the following
(C) Constipation is true?
(D) Urinary retention (A) He should be transferred to a bum center for
(E) Aaymptomatic immediate debridement and reconstructive
repair.
~ Which of the following parasites is aasociated with (B) Delayed labial artery bleeding can usually be
seizures? managed with direct pressure.
(A) Trypanosoma cruzi (C) All patients with bilateral oral commissure
(B) Taenia solium burns should have a cr scan of the face to
(C) Taeniasaginnta search for deep tissue injury.
(D) Trichuris trichiura (D) He should be admitted for 24 hours ofcardiac
(E) Leishmania brazilimsis monitoring.
(E) Serum creatine kinase and troponin levels
(ij) Most radioisotopes decay by: should be measured.
(A) a Particles.
(B) fJ Particles.
(fi) A first-time mother brings in her 4-day-old boy to
the ED with a chief complaint ofabdominal pain and
(C) y Rays.
sudden yellowish-green vomiting. She notes that he
(D) Electron capture.
had been "fussY• all day but became ill only a few
(E) Microwaves.
hours earlier. On examination, he appears ill and has
a mildly distended abdomen. An abdominal film is
(i!) Which ofthe followiq is the most common findiq shown in Figure 10-12. Which ofthefollowinsis the
in propanolol overdoses in adults?
next best step in management?
(A) Ventricular tachycardia
(B) Hypoglycemia
(C) Hyperkalemia
(D) Seizure
(E) Renal failure

(iii The earliest indicator of acute radiation syndrome is


(A) Thrombocytopenia.
(B) Eosinophilia.
(C) Deaease in absolute lymphocyte count.
(D) Aplastic anemia.
(E) Increase in the number ofatypical
lymphocytes.

(!!} A 35-year-old man presents after a motor vehicle


colliaion with hypotension, tachycardia and altered
mental status. He is determined to be in hemorrhagic Figure 1~12.
290 1000 ~ tDHdp YouPo~W the~ MetlidMBoGrds

(A) Barium enema (A) Partial recovery OCCW'S after induction of


(B) Emergent surgical consultation hyponatremia through infusion of 5%
(C) Broad spectrum antibiotics for presumed Dextrose in water.
necrotizing enterocolitis (B) It is more common after correction ofacute
(D) Obtain an ultrasound to rule out pyloric rather than chronic hyponatremia.
stenosis (C) It is more common in diabetic patients with
(E) Recommend smaller, thickened feedings, pseudohyponatremia due to hyperglycemia.
frequent burping and discharge home (D) Most common initial symptom is burning
paresthesias in the hands and feet
(!i} Which ofthe following is true regarding thromboem- (E) Patients do not typically present until a few
bolic disease in pregnancf. days after treatment of hyponatremia.
(A) Ventilation-perfusion (V/Q) scans apose the
fetus to less radiation than helical CT chest ~ A 56-year-old man presents after a high-speed
scans in ruling out pulmonary embolism (PE). motor vehicle collision. He complains of severe
(B) War&rin is omy contraindicated during fetal chest pain and chest x-ray demonstrates a widened
organogenesis. mediastinum. ChestCTscanshowsatraumatic aortic
(C) Thromboembolic disease is the leading cause of injury (TAl). The patient•s heart rate is 95 and blood
death in pregnancy. pressure is 175/77. Operative repair is scheduled in
(D) The risk of deep venous thrombosis is highest 30 minutes. Which of the following is the most
in the third trimester. appropriate therapy at this time?
(E) All of the above. (A) No therapy, observe for deterioration
(B) Oonidine
~ A 44-year-old man presents with chest pain after a (C) Hyclralazine
motor vehicle collision. He receives an upright chest (D) Labetalol
x-ray which is shown in Figure 10-13. Which of the (E) Enalaprilat
following is the most likdy diagnosis?
(!!] A 34-year-old man with a history of HIV presents
with headache. Contrast CT scan of the bram is
shown in Figure 10-14. Whichofthefollowingisthe
most appropriate therapyf

Figure 10-IJ.

(A) Pneumothorax
(B) Hemothoru
(C) Pulmonary contusion
(D) Traumatic aortic rupture
(E) Cardiac contusion

(H) Which of the following is true regarding osmotic


demyelination syndrome (or ODS, also known as
central pontine myelinolysis)? FlguretD-14.
TestlO 291

(A) Surgical excision demonstrates decreased internal rotation of both


(B) Mebendazole hips. Radiographs show slippage of the femoral
(C) Sulfadiazine and pyrimethamine epiphysis. Which of the following is the most
(D) Methylprednisolone common associated condition?
(E) Clindamycin (A) Hyperthyroidism
(B) Diabetes mellitus
[ll) The preferred method of hand splinting is (C) Obesity
(A) The position of greatest comfort for the patient. (D) Renal failure
(B) Wrist extended to 30 degrees, (E) Juvenile rheumatoid arthritis
metacarpophalangeals (MCPs) in full
extension, intraphalangeal (IP) joints flexed to ~ Which of the following is the most common
60 degrees. metabolic abnormality causing adynamic ileus?
(C) Wrist neutral, MCPs flexed to 60 degrees, IP (A) Lactic acidosis
joints free. (B) Hypokalemia
(D) Wrist extended to 30 degrees, MCPs flexed to (C) Hypomagnesemia
90 degrees, IP joints extended. (D) Hypercalcemia
(E) Wrist extended to 30 degrees, MCPs flexed to (E) Uremia
30 degrees, IP joints flexed to 30 degrees.
~ Patients with which of the following are most likely
[!j] Which of the following is adequately absorbed by to develop superimposed septic arthritis:
activated charcoal?
(A) Osteoarthritis
(A) Lithium (B) Rheumatoid arthritis
(B) Ethanol (C) Reiter's syndrome
(C) Iron (D) Systemic lupus erythematosus
(D) Toluene (E) Gout
(E) Acetaminophen
~ Which of the following is true regarding troponin I
~ A 24-year-old man ingests liquid drain cleaner and inacuteMI?
immediately presents to the ED. Which of the
(A) Starts elevating 12 hours after MI
following is the most appropriate treatment?
(B) Peaks at 36 hours after MI
(A) Ingestion of a small cup of water (C) Returns to baseline within 72 hours
(B) Calcium gluconate (D) Sensitivity at 6 hours after MI is 95%
(C) Dexamethasone (E) Specificity at 6 hours after MI is 95%
(D) Ampicillin
(E) Neutralization therapy with hydrochloric acid ~ Examination of which of the following is part of the
primary survey in trauma evaluation?
~ Which ofthe following is the most common cause of
(A) Heart
acute food poisoning in the U.S.?
(B) Lungs
(A) Clostridium perfringens (C) Abdomen
(B) S. aureus (D) Ears
(C) Escherichia coli (E) Nose
(D) Bacillus cereus
(E) Vibrio parahaemolyticw [j!J A 53-year-old woman presents 3 days after extraction
of a premolar tooth with exquisite pain and
~ The synovial white blood cell (WBC) count in cases tenderness in the area unrelieved by ibuprofen.
of septic arthritis usually exceeds at least: Halitosis is present. Which of the following is true
(A) 250 per rnm3 • regarding this patient?
(B) 1,000permm3• (A) The patient is likely exhibiting drug-seeking
(C) 25,000 per mm3 • behavior.
(D) 50,000 per mm3 • (B) Irrigation and packing are generally of no
(E) 100,000 per mm3• benefit.
(C) Pathophysiology involves premature loss of
~ A 14-year-old boypresents with intermittent bilateral clot from the socket.
hip pain for several weeks. Physical examination
292 1000 Qwstimu to Hdp You Ptw the Emerpnt;y Maicitw Bo.rmfs

(D) Treatment involves inducing mild bleeding to ~ Which of the following is the strongest risk factor for
forma clot. devdopment of right -aided endocarditis~
(E) Antibiotics and nonsteroidal
(A) Rheumatic heart disease
anti-inflammatory drugs (NSAIDs) play no
(B) Intravenous drug use
role in management.
(C) Papillary muscle rupture
(D) Coronary artery disease
[iiJ Which of the following is an indication for antibiotic
(E) Systemic lupus erythematosis
prophylui.s in patients with prosthetic heart valves?
(A) Endoscopy ~ A healthy 23-year-old woman preaents with acute bi-
(B) Intrauterine device (TIJD) placement lateral facial weakness. She complains that she "can't
(C) Cystoscopy taste anything" and that sounds are abnormally loud.
(D) Urethral catheterization Shedenies anyunu&Ual travel and notes only her usual
(E) Tympanostomy tube insertion summer camping trip. Which of the following is the
most likely cause ofher symptOlllll?
~ Anal fissures are almost always located:
(A) Infectious mononucleosis
(A) Anterior midline. (B) Miller-Fisher variant Guillain-Barr~
(B) Posterior midline. syndrome (GBS)
(C) Right lateral (C) Neurosyphilis
(D) Left lateral (D) Lymphoma
(E) Eccentrically. (E) Lyme disease

~ Which of the following is the In06t common lead ~ The most common ultrasonographic finding in
point in cases of pediatric intussusception? women with ovarian torsion is
(A) Meckd's diverticulum (A) lntraovarian hemorrhage.
(B) Lymphoma (B) Ovarian enlargement.
(C) Intestinal polyp (C) Lack of blood flow through color Doppler
(D) Mucosal hemorrhage in Henoch-Schonlein imaging.
purpura (D) Pelvic-free fluid.
(E) Peyer's patches (E) AU of the above are common findings.

~ A father brings in hia 8-year-old daughter with a chief ~ Which of the following is true regarding exacer-
complaint of a facial raah and rash on her arms (see bations and complications of inflammatory bowel
Pig. 10-15). The raah seems to spare the nasolabial disease (mD)?
fold and perioral area. Which of the following is the
(A) Patients with mild to moderate exacerbations
cause of this patient's illnessf
can be sent home with a resumption of therapy
or with outpatient corticosteroids.
(B) Bowcl rest has not been shown to be useful in
patients with flares ofiBD.
(C) Fistulae are initially treated medically.
(D) Initial treatment for toxic megacolon includes
fluids, antibiotics and corticosteroids.
(E) AU of the above.

~ A 44-year-ol.d woman with a history of end-stage


renal disease on peritoneal dialysis presents with
abdominal pain. You suspect peritonitis as a cause of
her pain. Which of the following is the most likely
figure 10-15.
etiologic agent?
(A) St4phylorocrus species
(A) RubelJa (B) Strtpt«occus species
(B) Human herpesvirus 6 (C) E. coli
(C) Parvovirus Bl9 (D) Klebsiella
(E) Pseudomoruu
(D) Measles virus
(E) Group A streptococci
TestlO 293

~ Posterior hip dislocations: ~ A 67-year-old woman with a history of hypertension


(A) Are less common than anterior hip and diabetes presents to the ED with a complaint
dislocations. of double vision. On examination you find that she
(B) Most commonly result in compression of the has mild left-sided ptosis and the inability to move
femoral nerve. her left eye superiorly and medially. The remainder
(C) Are more commonly associated with fractures of her eye movements and her pupils are normal.
of the femoral head than anterior dislocations. A noncontrast CT scan of the brain you ordered is
(D) Are associated with an inability to see the lesser normal. Which of the following is the next step?
trochanter on an anterior-posterior view. (A) Cerebral angiogram
(E) Cause the patient to have an externally rotated, (B) Lumbar puncture (LP)
abducted and shortened leg. (C) MRI/MRA of the brain with contrast
(D) Administer oral aspirin and admit with a
~ A 65-year-old woman presents with right eye pain, diagnosis of "stroke"
irritation, foreign body sensation, and tearing. Skin (E) Discharge with referral to an ophthalmologist
lesions are seen on the right side of the forehead and
the conjunctivae are injected. Slit lamp examination [tOOl Which of the following is the most common
reveals pseudodendrites. Which of the following is organism isolated in spontaneous bacterial peritoni-
true regarding this condition? tis (SBP)?
(A) Corneal hypoesthesia, although rare, is usually (A) E. coli
permanent. (B) S. aureus
(B) True dendrites are more commonly seen than (C) S. pneumoniae
pseudodendrites. (D) Klebsiella pneumonilre
(C) Cranial nerve VII is most commonly involved. (E) Anaerobic species
(D) Anterior uveal involvement is dependent upon
severity of corneal disease.
(E) Systemic antivirals are more effective than
topical antivirals.
Answers and Explanations
[!] Answer B. Most ingested isopropanol is converted population, particularly in the first 2 years of life.
to acetone by hepatic alcohol dehydrogenase. Ace- Meckel's diverticulum is the most common cause
tone is a ketone body, but is not acidic or charged, and of substantial gastrointestinal (GI) bleeding in this
does not contribute to the anion gap. Ketosis without population.
acidosis is the characteristic finding in patients with
isopropanol poisoning. Serum creatinine, not blood 00 Answer D. The chest x-ray demonstrates presence
urea nitrogen (BUN), may be falsely elevated be- of the gastric bubble inside the thoracic cavity,
cause of laboratory interference by acetone. Choice indicating a large diaphragmatic rupture from blunt
D is seen with methanol poisoning. Fomepizole is trauma. The left diaphragm is much more likely than
not indicated, as the metabolites of isopropanol, like the right to rupture due to protection on the right
the parent compound, cause generalized central ner- from the liver. Management involves nasogastric
vous system (CNS) depression without other major decompression and surgical repair. Patients with
organ system effects. Dialysis may be indicated to penetrating thoracoabdominal trauma may have
treat extremely severe poisoning. delayed abdominal herniation into the thorax as
far out as decades after the initial injury. Right-
[!) Answer D. Pericardia! tamponade usually results sided diaphragmatic injuries are less likely to have
from a penetrating thoracic mechanism causing abdominal herniation into the thorax, also because
cardiac or mediastinal injury and accumulation of of the presence of the liver. Diaphragmatic injuries
blood into the pericardium. Pericardia) effusion and may be very subtle and diagnosed only on direct
tamponade are readily seen on the subxiphoid and visualization with laparoscopy or thoracoscopy. DPL,
parasternal views of the FAST scan. Chest x-ray may FAST, and CT scan all lack sufficient sensitivity
show a large, water-bottle heart ifthe effusion is large for ruling out the diagnosis. (Figure courtesy of
enough, but tamponade physiology can occur with Mark Silverberg, MD. Reprinted with permission
small, rapidly accumulating effusions which may be from Silverberg M. Greenberg's text-atlas ofemergency
invisible on plain radiography. The EKG finding medicine. Lippincott Williams & Wilkins; 2004:654.)
of electrical altemans due to swinging of the heart
throughout the cardiac cycle is highly specific for [!] Answer C. The image reveals a Lisfranc fracture-
pericardia! effusion, but occurs in less than one third dislocation. Lisfranc injuries are uncommon injuries
of cases. DPL is highly sensitive for intraperitoneal most often resulting from high-energytrauma such as
injury but has no utility in screening for pericardia! motor vehicle accidents. They may also occur because
effusion. of axial loading on a foot in extreme plantarflexion.
Any displacement indicates inherent instability and
[!) Answer A. Herpes simplex virus (HSV) type 2 patients develop a high rate of degenerative arthritis,
causes most genital herpes infections. Primary even in the setting of operative repair. Owing to
HSV infection is almost always more severe than the low incidence of Lisfranc injuries as well as
recurrences. Outbreaks can occur with any systemic the significant trauma that is often required to
or local stress. The lesions are usually extremely produce such an injury, Lisfranc injuries are often
painful and irritating and should never be drained overlooked. Furthermore, radiographic evaluation
or unroofed as this will inoculate other sites. The can produce subtle findings. Dorsal displacement of
sensitivity and specificity of the Tzanck smear are the metatarsals on a lateral foot radiograph is one
both <80% and viral culture is the gold standard such finding and indicates ligamentous disruption.
for diagnosis. Antibiotics should not be used in the Plantar displacement does not occur because of
absence of clear signs of bacterial superinfection. the strength of the plantar fascia. Most commonly
Treatment of HSV outbreaks is with acyclovir, however, radiographs will reveal a misalignment of
famciclovir, or valacyclovir. Continuous treatment the second metatarsal and the medial border of the
may be necessary to prevent outbreaks in susceptible middle cuneiform. In addition, the medial border
individuals. of the fourth metatarsal should be aligned with the
medial border ofthe cuboid on an oblique view. Any
(!] Answer A. Anal fissures are the most commonly displacement indicates a Lisfranc injury. Trauma to
encountered anorectal problem in children and the dorsalis pedis artery may occur with Lisfranc
they are the most common cause of LGIB in this injuries as the dorsalis pedis descends between the

294
TestlO 295

first and second metatarsals to form the plantar reduction by nitrates is much less prominent. Mor-
arch. (Figure reprinted with permission from Harris phine and loop diuretics such as furosemide are
]H. The radiology of emergency medicine, 4th ed. also used for preload reduction but have a smaller
Lippincott Williams & Wilkins; 1999:885.) effect than nitrates. While morphine has been used
for years as part of the standard treatment of CHF
[I) Answer A. Increasing maternal age, cigarette for preload reduction, there have been few rigor-
smoking, cocaine use, twin or multiple gesta- ously conducted randomized trials confirming its
tion, preexisting or pregnancy induced hypertension benefit. Morphine most likely exerts its preload re-
(preeclampsia), trauma, chorioarnnionitis, oligohy- ducing effect by reducing anxiety and subsequent
dramnios as well as thrombophilias all place women catecholamine production and release. Loop di-
at risk for abruptio placentae. Ofthese, preeclampsia uretics cause preload reduction by inhibiting renal
is the most significant risk factor. sodium reabsorption thereby increasing urine vol-
ume and decreasing plasma volume. However, the
[!] Answer E. Sinus imaging has very little role in elevated afterload in patients with CHF reduces re-
the ED except in the evaluation of patients with nal perfusion and thereby limits the effectiveness
suspected complications of sinusitis (e.g., orbital cel- of loop diuretics. Afterload reduction is most com-
lulitis, cavernous sinus thrombosis). However, in the monly achieved through the use of ACE inhibitors
modern era it is clear that plain films have such a such as enalaprilat, although nitroprusside and hy-
low sensitivity for sinus disease that they are virtually dralazine may also be used to achieve this effect.
never requested. When plain films are used, they Digoxin causes an increase in cardiac contractility
are best used for the diagnosis of maxillary sinusitis without major changes on preload or afterload, and
as their sensitivity drops when used to image other has no role in the management of acute CHF. In
sinuses. CT scans are currently the most useful imag- fact, prior digoxin use has been shown to correlate
ing modality in the evaluation of sinusitis. Although to increased in-hospital mortality amongst patients
they are highly sensitive tests, they have a poor speci- admitted with CHF.
ficity (high false positive rate). Several studies have
demonstrated that asymptomatic patients with un- [!!) Answer B. Oral candidiasis is the most common
complicated viral upper respiratory infection (URis) form of candida! infection and Candida species
have abnormalities of one or both maxillary sinuses colonize the oropharynx of 80% of healthy infants
on CT scan. The same is true for asymptomatic pa- by 3 to 4 weeks of age. Oral candidiasis is also an
tients with seasonal allergies. Therefore, CT scans AIDS-defining illness. Because Candidn spp. are part
should never stand alone in the diagnosis of sinusitis. of the normal flora of some many people, cultures
A diagnosis of sinusitis should be supported with are rarely useful. Instead, diagnosis is based on
clinical findings of facial or dental pain, headache, clinical examination and the finding of white, curd-
and purulent nasal discharge, often preceded by a like exudates on the buccal and gingival mucosa
viral URI. and less frequently on the tongue and soft palate.
The exudates can be scraped away to reveal an
[!] Answer E. This patient experienced preterm pre- erythematous, mildly eroded, and painful mucosa
mature rupture of membranes (PPROM) 2 days underneath. Although the infection is frequently
before presentation, which dramatically increased self-limited, infants with thrush should be treated
her risk for developing chorioamnionitis. Chorioam- with oral nystatin suspension to hasten healing and
nionitis is an intra-amniotic infection that is most primarily to prevent problems with feeding (due
commonly due to vaginal flora that have gained en- to pain). Tinea cruris is most commonly caused
try to the amniotic cavity. Risk factors include young by Trichophyton spp., as with other dermatophyte
age, low socioeconomic status, nulliparity, extended infections. The hallmark of cutaneous candidiasis is
duration of labor and ruptured membranes, multi- the presence of satellite papules and pustules beyond
ple vaginal examinations, and preexisting infections the margins of a patch of macerated, sometimes
of the lower genital tract. The typical causative or- weeping skin with scalloped borders. It typically
ganisms are group B streptococci and E. coli and occurs in intertriginous areas such as the groin,
the most widely used antibiotic regimen is a com- axilla, or underneath pendulous skin folds.
bination of ampicillin and gentamicin. Ampicillin
may be replaced with vancomycin, erythromycin, or [!j) Answer C. The patient is presenting in septic
clindamycin in penicillin-allergic patients. shock from an unclear source. She presents from
a nursing home, where rates of methicillin-resistant
~ Answer E. Nitroglycerin directly dilates the great Staphylococcus aureus (MRSA) and P. aeruginosa are
veins and reduces left ventricular preload. Afterload extremely high. Treatment involves broad-spectrum
29 6 1000 Questions to Help You Pass the Emergency Medicine Boards

antibiotic coverage to cover both these pathogens. (PT) have been classically used to stratify patients
Metronidazole and clindamycin afford excellent into low or high-risk categories by calculating the
anaerobic coverage, but piperacillin-tazobactam also Maddrey discriminant function.
kills anaerobes and double coverage of anaerobes Discriminant function =
is not routinely indicated. Choice B would be a
4.6 x (PT in seconds - control in seconds) + bilirubin
reasonable regimen in a patient with septic shock
(mgperdL).
who is not at risk for nosocomial pathogens.
Levels over 32 correspond with severe disease and a
~ Answer B. The Ottawa Ankle Rules (OAR) are a 1-month mortality >50%.
set of criteria that were devised to limit unnecessary
ankle radiography in patients presenting with ankle ~ Answer E. Hypertrophic cardiomyopathy is usu-
pain. The criteria require ankle radiography in any ally a familial condition, which causes increased
patient with ankle pain who has the following: left ventricular and septal wall size and resultant
diastolic dysfunction. Left ventricular hypertrophy
1. Unable to ambulate four steps both at the time of
can eventually lead to outflow obstruction. Clini-
injury and in the ED, or
cal clues include exertional syncope or chest pain
2. Bony tenderness to palpation of the tip of either
in young patients with a systolic murmur at the
malleolus or tenderness of the distal 6 em
left lower sternal border. Any agents which increase
posterior to either malleolus.
systemic afterload (phenylephrine) or cardiac con-
The original paper, published in 1992, described tractility (digitalis, isoproterenol) or decrease preload
a sensitivity of 100% and a specificity of 40%. Since (furosemide) are contraindicated in patients with
then, however, several other studies have published hypertrophic cardiomyopathy.
a variety of other values, suggesting a wide variability
for specificity and sensitivity somewhere >96%. ~ Answer E. The Salter-Harris classification is used
However, even in the original paper (sensitivity to describe pediatric long bone fractures near the
of 100%), occasional patients had "insignificant" growth plate. Type I fractures go through the physis
fractures such as chip fractures that would be only, type II from the metaphysis into the physis,
managed in the same way as a sprain. The squeeze test type III from the epiphysis into the physis, type IV
involves squeezing the tibia and fibula approximately is a combination of types II and III, and type V is
5 em proximal to the malleoli in an effort to provoke a crush injury to the physis. The most common is
pain in the ankle. Increased ankle pain indicates type II. Types I and V may be invisible on initial plain
disruption ofthe distal tibiofibular syndesmosis. The films. Type V carries the poorest prognosis.
OAR have not been validated for pediatric use. Like
other clinical decision rules, the OAR should not [!!) Answer E. For any patient with acute urinary
be used in unreliable patients such as patients with retention, the goal is to find the cause of urinary
decreased mental status or intoxicated patients. obstruction and correct it as rapidly as possible.
In this case, as in the majority of older men, the
IHJ Answer E. This patient has acute alcoholic hep- most likely cause is benign prostatic hypertrophy
atitis. Although most cases of alcoholic hepatitis (BPH). Pain and distension in the suprapubic area
are subclinical or asymptomatic, it may be life indicates bladder distention from a more distal
threatening. Patients presenting for treatment are obstruction. The correct management is to urgently
characterized by fever, right upper quadrant pain, place a Foley catheter to temporarily stent open the
jaundice, anorexia, and occasionally nausea and vom- prostatic urethra and decompress the bladder. Any
iting. Physical examination most commonly reveals imaging and blood studies may follow this initial
hepatomegaly (as opposed to the shriveled, firm management, but should not delay relief of the
liver in patients with cirrhosis), jaundice, ascites, obstruction. Further management usually involves
splenomegaly, and signs of alcohol withdrawal. AST a basic chemistry panel to assess kidney function,
and ALT levels are only modestly elevated, typi- urinalysis to look for concomitant infection, and
cally remaining below 300 U per L, and the ratio of outpatient urology follow-up.
AST:ALT is usually greater than 2:1. Bilirubin lev-
els are elevated but are variable depending on the ~ Answer B. The patient has toxic epidermal necrol-
severity of the disease. Leukocytosis is common, with ysis (TEN), which is a vesiculobullous disease that is
a mean WBC count of 12,400 per mm3 , whereas characterized by diffuse epidermal detachment. It is
counts up to 20,000 are not uncommon. Fever is a part of a continuum with Stevens-Johnson syndrome
common finding but concomitant infection is un- (SJS) and is diagnosed when the degree of epider-
common. Bilirubin levels and the prothrombin time mal detachment exceeds 30% of the body surface
TestlO 297

area (whereas SJS is diagnosed when epidermal are considered potentially or definitely unstable. A
detachment is < 10%). Drug exposure, particularly to flexion teardrop fracture occurs when the anterior
sulfonamides, and to a lesser degree, anticonvulsants, portion ofthe vertebral body shears off from a flexion
is the most common cause of SJS and TEN. A recent force, causing ligamentous disruption. Bilateral facet
urinary tract infection implies that this patient had dislocation is an extremely unstable injury resulting
been taking trimethoprim/sulfamethoxazole when from significant flexion, causing the superior facets
she developed TEN. (Figure © David Effron, MD, of the inferior vertebra to lose their articulation
2004. Used with permission.) with the inferior facets of the vertebra immediately
superior to it. Spinal cord injury is common due
[!ID Answer A. Acute spinal cord injuries may benefit to the significant displacement of the vertebrae.
from high -dose corticosteroid therapy if given within Solitary transverse process fracture is usually of no
8 hours of injury. A longer delay than this may clinical significance as it is far removed from the
actually result in worsened outcomes. Normal saline articular surfaces of the vertebrae and spinal cord.
or lactated Ringer's hydration should be given to A Hangman's fracture occurs from extreme extensor
all trauma patients. Naloxone may be given to forces, causing bilateral fractures ofthe pedicles ofC2
patients with altered mental status as empiric therapy and dislocation ofC2 from Cl. Unlike the past where
for opioid intoxication but is not generally used hanging gave this fracture its name, the common
in patients with spinal cord injury. Vancomycin mechanism now is motor vehicle collision. The
and other antibiotics are not routinely indicated in Jefferson fracture results from vertical compression
trauma patients, except in clinically evident cases of forces causing the anterior and posterior portions of
posttraumatic infection. Vasopressin may be used in the ring of C 1 to break and putting the spinal cord at
patients with medical causes of shock who do not extreme risk for severe injury.
respond to crystalloid or colloid administration but
is unlikely to be useful in patients with traumatic ~ Answer D. Endocarditis in intravenous drug users
shock. (IVDUs) is different from that in nondrug users in
several ways: Patients are generally younger, almost
~ Answer B. Kernig's and Brodzinski's signs are universally have fever, and have more pulmonary
performed to evaluate for meningeal irritation in symptoms due to right-heart vegetations causing
the patient who is suspected of having bacterial septic emboli to the lungs. Although S. aureus
meningitis. Kernig's sign is elicited by having the is a far more common cause of endocarditis in
patient lie supine with the hip flexed at 90 degrees. IVDUs, it has less mortality among this patient
Extension of the knee in this position causes pain population. Heart murmurs are less commonly heard
down the posterior thigh or lower back in patients in endocarditis in IVDUs because the tricuspid valve,
with inflamed meninges. Brodzinski's sign is positive the most common valve affected, is difficult to hear
when flexion of the neck of a supine patient causes on physical examination. Splinter hemorrhages and
involuntary flexion of the hips. These named signs Roth spots are, if anything, less common in right-
have poor sensitivity (in the range of 10% to heart endocarditis as the lungs will filter most septic
40%) but good specificity (in the range of 80% emboli.
to 100%). Positive and negative predictive values
are affected by the prevalence of a given disease in ~ Answer E. Overdose with an antihypertensive
the population being studied. In contrast, sensitivity causing bradycardia, hypotension, and depressed
and specificity are not affected by prevalence but are mental status is most likely from a calcium chan-
unique characteristics of a given test. nel blocker (CCB) or ,B-blocker. These overdoses can
be lethal, especially in children. Traditional treatment
~ Answer C. Spinal injuries are classified by the has focused on atropine, glucagon, and calcium to
mechanism of injury and mechanical stability in counteract the effects of both classes of drug. At-
reference to their potential to cause neurologic ropine reverses the effect of vagal stimulation and
damage. Unstable injuries are considered likely to prevents further bradycardia. Glucagon functions in-
cause spinal cord damage and usually require surgical dependently of the ,B-receptor to improve inotropy
stabilization. Stable spinal injuries are more common and chronotropy. Calcium competes for the calcium
than unstable ones and are easier to remember channel in CCB overdose. Recently, high-dose in-
because there are only a few types--wedge fracture, sulin and glucose has been found to be beneficial in
spinous and transverse process fractures, unilateral the management of severe overdoses of ,8-blockers,
facet dislocation, and vertebral burst fracture(with and probably helps with CCB overdoses as well. Of
the exception of a Jefferson fracture, which is a note, these symptoms could also be due to clonidine,
burst fracture of Cl). All other spinal injuries which can cause severe hypotension, bradycadria,
298 1000 Questions to Help You Pass the Emergency Medicine Boards

respiratory depression, miosis and somnolence oral- sense of diffuse warmth, a burning sensation in the
tered mental status. Such symptoms may be mistaken mouth and throat, palpitations, nausea, anorexia,
for acute opiod intoxication or overdose. Naloxone vomiting, abdominal cramps, conjunctival injection,
has been advocated by some authors although it may and pruritus. Symptoms are rapidly relieved after
cause severe hypertension and its use is controver- the injection of parenteral antihistamines such as
sial. Generally, the treatment of clonidine overdose diphenhydramine or cimetidine and most symptoms
is aggressive supportive care (decontamination with will have completely abated within 6 hours. The
charcoal, intravenous crystalloids, pressors). patient should not be told that he or she has an
allergic reaction and they should not be prevented
~ Answer C. The patient has evidence of moder- from eating fish in the future.
ately severe community-acquired pneumonia, with
tachycardia, tachypnea, and hypoxia. Accordingly, ~ Answer C. The patient likely has a testicular
the patient should be admitted to the hospital tumor, usually of germ-cell type in this age-group.
for intravenous antibiotics and fluids and oxygen. Evaluation may include screening for metastases
Community-acquired pneumonia in this age-group with a chest x-ray and a Cf scan of the abdomen
is most commonly due to pneumococcus, atypi- and pelvis, although this may occur on an urgent
cal organisms, and gram-negative bacilli. Among outpatient (rather than emergent inpatient) basis.
the choices in the preceding text, C is the most Emergent surgery is not indicated in patients with
appropriate. Other acceptable regimens could in- testicular tumors except in certain patients who have
clude a fluoroquinolone or ampicillin-sulbactam. evidence of testicular torsion. Azithromycin may be
Oral medications and discharge would not be used to treat chlamydia! epididymitis but has no
appropriate because of the vital sign abnormali- role in the management of testicular tumors. Scrotal
ties and comorbidities. Clindamycin possesses little elevation is often used as a diagnostic maneuver
gram-negative coverage and would not be adequate to distinguish between epididymitis and testicular
therapy. Piperacillin-tazobactam is a potent, broad- torsion (relief is called Prehn's sign), but it is an
spectrum, antipseudomonal antibiotic used only for unreliable finding for this purpose. Corticosteroids
nosocomial pathogens causing severe illness. may be used in patients with testicular tumors as
part of certain chemotherapeutic regimens but has
~ Answer A. IVDUs have bacteria that are trans- no role in the acute management. (Figure courtesy
mitted through the skin to superficial veins, into of Mark Silverberg, MD. Reprinted with permission
the greater venous circulation, and eventually to the from Silverberg M. Grunberg's text-atlas ofemergency
heart These pathogens (S. aureus in more than three medicine. Lippincott Williams & Wilkins; 2004:330.)
fourths ofcases) lodge most commonly in the tricus-
pid valve to cause right-heart endocarditis and may ~ Answer C. Patients on P-blockers may be difficult
lead to septic pulmonary emboli. to treat in the setting ofanaphylaxis. On the one hand,
If-blockade may blunt or prevent some of the ben-
~ Answer C. The MB fraction of creatine kinase is eficial effects of epinephrine. However. epinephrine
present mostly in cardiac muscle, but also in small also has the potential to worsen anaphylaxis in the
quantities in skeletal muscle. Levels ofCK-MB begin setting of beta blockade due to subsequent unop-
to rise between 3 and 8 hours after infarction, peak at posed alpha receptor stimulation. This may result in
12 to 24 hours, and return to normal within 3 days. an increased release of the vasoactive mediators in
Neither the sensitivity nor the specificity of CK-MB anaphylaxis. Clinically, this may be manifest in wors-
at 3 hours after acute MI is >50%. ened bronchoconstriction, bradycardia, and coro-
nary vasoconstriction. Like epinephrine, glucagon
~ Answer E. This patient experienced scombroid exerts its influence through the formation of intra-
fish poisoning. Although not an allergic reaction, cellular cyclic AMP. However, glucagon bypasses the
the symptoms are due to excessive histamine levels If-adrenergic receptor by binding to a discrete G
in the fish and results in symptoms that are very protein receptor. Therefore, glucagon may be effec-
similar to an allergic response. This patient's prior tive even in the setting of If-blockade. This is also
history of avid seafood intake also points against the basis for glucagon's use in P-blocker overdose.
allergy. Histamine levels build up in inadequately Other agents that may be beneficial include va-
refrigerated, or inadequately preserved dark-muscled sopressors (dopamine, norepinephrine), nebulized
fish due to the action of histidine decarboxylase albuterol (specifically for relief of bronchospasm),
by enteric bacteria in the fish. Symptoms usually atropine (for bradycardia), and isoproterenol (as a
occur within minutes of eating the fish and include last resort).
severe throbbing headache, facial flushing and a
TestlO 299

~ Answer C. Uncomplicated pertussis in an unvac- 5 years. Additionally, adolescents and adults more
cinated child is a clinical diagnosis. In children, the often present with atypical signs and symptoms such
illness classically follows a three-stage pattern-the as an isolated spasmodic cough without an associ-
catarrhal phase, paroxysmal phase, and convalescent ated inspiratory whoop. Previously vaccinated adults
phase. The catarrhal phase usually lasts between 5 and rarely demonstrate the lymphocytosis that is charac-
10 days but may last up to 3 weeks. During this phase, teristic of pediatric infections.
it is impossible to clinically differentiate these patients
from patients with common cold. Unfortunately, [!j) Answer D. Penicillin remains the drug of choice
patients are most contagious during this phase. Pa- for GAS pharyngitis. The standard course of therapy
tients are typically afebrile or have only a minimal is Penicillin V 500 mg every 6 hours for 10 days.
fever during this phase and throughout the illness. There is some evidence that twice-daily dosing may
The presence of significant fever should prompt a be equally efficacious but the guidelines remain at
search for a secondary infection. Additional typical 6-hour dosing intervals. Alternatively, 1.2 million
symptoms include rhinorrhea, mild cough, sneezing units of penicillin G benzathine may be given as an
and occasionally conjunctivitis. These signs may be intramuscular injection in the ED (in adult patients).
absent in very young infants. The paroxysmal phase Pediatric patients may also receive penicillin G
is characterized by paroxysms of cough, often occur- but the dosing is weight based. In patients with
ringduringthenightandassociatedwithpost-tussive a penicillin allergy, erythromycin is the drug of
emesis, especially in young infants. The inspiratory choice, given 400 mg four times per day for a 10-day
whoop, which is responsible for the common name, course. None of the other drugs listed are effective
Whooping Cough, given to Bordetella pertussis in- against GAS infections. Alternatives to erythromycin
fections, occurs at the end of a paroxysm, as air is include azithromycin, cefadroxil, cephalexin, and
inspired against a partially closed glottis. Owing to clindamycin.
the immaturity of their respiratory system, young
infants are susceptible to developing apneic episodes ~ Answer C. Fluid loss in patients with toxic epider-
during this phase, as well as choking or gasping for mal necrolysis (TEN) is sizable but not as severe as
air. Apnea and choking may be sufficiently prolonged in burn patients. However, the widely denuded skin
to cause hypoxia and cyanosis. Therefore, infants and is an easy access point for a variety of bacteria and
toddlers presenting during the paroxysmal phase fre- sepsis is the most common cause of death.
quently require admission to an intensive care unit
(ICU) setting where appropriate monitoring can be [!ID Answer E. Cluster headaches may be associated
provided. Feeding may also be a problem during with ipsilateral autonomic instability reflecting both
this phase, as it may provoke paroxysms of coughing sympathetic dysfunction such as ptosis, miosis, and
and subsequent vomiting. Therefore, several infants forehead as wen as facial sweating and parasympa-
and toddlers with this disease present with severe thetic activation, such as rhinorrhea, lacrimation,
dehydration. The paroxysmal phase typically lasts and nasal congestion. Owing to the combination
from 1 to 6 weeks. During the convalescent phase, of these findings in concert with the distnbution
the paroxysms typically become less frequent and of pain, it is thought that the area responsible for
less distressing, although the cough may actually be- cluster headaches is the cavernous sinus. In the
come louder. Although classical pertussis is a clinical cavernous sinus, the trigeminal nerve, sympathetic
diagnosis, lymphocytosis supports the diagnosis. Ab- and parasympathetic fibers converge.
solute lymphocyte counts > 20,000 per mm3 may be
seen with a total WBC count >100,000 per mm3. ~ Answer C. The patient has ecchymosis and de-
The chest x-ray is most often normal, although it formity of his penile shaft indicative of penile
may demonstrate a "shaggy" right-heart border. fracture. Although important ancillary studies such
However, the presence of an infiltrate may indi- as ultrasonography, retrograde urethrography, and
cate a secondary infection as bacterial pneumonia cavemosography may be necessary for further evalu-
may complicate pertussis in up to 20% of patients. ation, definitive management almost always requires
Although the incidence of B. pertussis is rising in the urgent urologic repair. Conservative management
population as a whole, this rise is almost solely due to with penile splinting and pressure dressings has an
an increase in the number of adolescents and adults unacceptably high risk of complications such as de-
with the disease. However, children and infants still formity and impotence. Foley catheterization may
represent most cases ofB. pertussis. Thirty-eight per- be performed in patients without urethral injuries
cent of cases occur in infants younger than 6 months to help guide the surgical repair. (Figure courtesy of
and 71% of cases occur in children younger than Donald Sallee, MD. Reprinted with permission from
300 1000 Questions to Help You Pass the Emergency Medicine Boards

Sallee D. Greenberg's text-atlas ofemergency medicine. demonstrated on physical examination and the
Lippincott Williams & Wilkins; 2004:318.) rash that was present was already there before
the antibiotic treatment. The patient should not be
~ Answer C. The patient has severe tongue swelling, intubated or given epinephrine because of the lack of
likely from angiotensin-converting enzyme (ACE) upper airway or pulmonary symptoms. Prednisone,
inhibitor-induced angioedema given the history of diphenhydramine, and famotidine would be useful
new antihypertensive medication. Pathophysiology in an allergic reaction but have little role in the
involves buildup of bradykinin due to inhibition management of the Jarisch-Herxheimer reaction.
of angiotensin converting enzyme. Acute treatment
with epinephrine, steroids, and antihistamines is ~ Answer D. CHF is increasing in prevalence in the
generally ineffective. Fresh frozen plasma contains United States and the mortality for severe cases
kininase II, which can cleave bradykinin and re- is extremely high. Mortality has been reduced by
duce the angioedema. Verapamil, metoprolol, and ,8-blockers and ACE inhibitors as well as automated
aldosterone have no role in the management of an- internal cardiac defibrillators. Exacerbations of CHF
gioedema. Danazol is used for angioedema due to are commonly due to medication noncompliance,
hereditary Cl esterase inhibitor deficiency, but its dietary indiscretion, dysrhythmias, MI, pulmonary
role is generally limited to prophylaxis rather than embolism (PE), and infections.
acute treatment. (Figure reprinted with permission
from Hendrickson R. Greenberg's text-atlas of emer- ~ Answer E. This patient has hypercalcemic crisis
gency medicine. Lippincott Williams & Wilkins; 2004. (generally defined as any patient with a serum cal-
Courtesy of Robert Hendrickson, MD.) cium> 14 mgperdL), which is typically characterized
by altered mental status, polyuria, and dehydration.
~ Answer C. Salmonella is a common cause of acute The initial goals are to restore intravascular volume
infectious diarrhea in the United States. Most cases with intravenous saline and to rapidly lower the
are caused by non-typhi strains and present with serum calcium level by enhancing urinary excretion
crampy abdominal pain, fever, and watery diarrhea and reducing bone resorption (primarily through
with occasional bloody stools. Nausea and vomit- osteoclast inhibitors such as pamidronate or other
ing also commonly occur but are typically mild. bisphosphonates). Calcium channel blocker (CCBs)
Ten percent of household pets excrete Salmonella, such as nifedipine play no role in the treatment of
although poultry products are the most common hypercalcemia. Loop diuretics are the most effec-
source, including eggs. Immunocompromised pa- tive drugs in enhancing urinary calcium elimination.
tients, such as patients with underlying leukemia, In contrast, thiazides increase calcium reabsorption
lymphoma, sickle cell anemia or acquired immune in the distal tubule resulting in a further increase in
deficiency syndrome (AIDS), are more susceptible serum calcium levels. Calcitonin is the fastest acting,
to bacteremia and severe infection. Most patients but weakest agent in reducing serum calcium and
with salmonellosis are effectively managed with sup- cannot be used for monotherapy. Bicarbonate is not
portive care alone and will be better by the time useful in hypercalcemia. However, in the setting of
stool cultures have revealed the infecting organism. a true alkalosis, more free calcium will be bound to
Furthermore, treatment with antibiotics prolongs albumin, thereby decreasing the amount of free ion-
shedding of non-typhi strains. Therefore, antibiotics ized calcium. Glucocorticoids are useful in patients
should only be used for patients who represent a with hypercalcemia caused by an underlying hema-
public health risk (e.g., food or health care workers) tologic malignancy or granulomatous disease due to
or for patients who are ill with persistent infection. their effects on vitamin D metabolism and cytokine
Ciprofloxacin, norfloxacin, and azithromycin are all release. Glucocorticoids do not exert their effect for
effective for outpatient management. Ceftriaxone is 1 to 2 days after initiation of treatment.
the drug of choice for inpatient management.
~ Answer B. Cerebraledemaisthemostfearedcom-
Ill] Answer E. The patient is exhibiting signs of the plication of diabetic ketoacidosis (DKA) treatment
Jarisch-Herxheirner reaction after treatment for in children. Although reports vary, cerebral edema
secondary syphilis with penicillin G. The reaction complicates roughly 1% ofpediatric DKA cases, with
occurs because massive death of spirochetes on a mortality rate of 20% to 90%. Survivors have
exposure to the penicillin causes systemic symptoms persistent neurologic sequelae 20% to 40% of the
in a serum sickness-like reaction. Treatment is time. The development of cerebral edema is almost
symptomatic with acetaminophen or ibuprofen. exclusively a complication of pediatric DKA. There
An allergic reaction to the antibiotic that he was is still no consensus regarding the etiology of cere-
given is possible, but there were no urticaria bral edema in pediatric DKA. However, early clinical
TestlO 301

signs include headache, vomiting, decreased arousal, Headache is not a typical feature of patients with
relative bradycardia, and relative hypertension. NPH.

!!!] Answer C. Transient synovitis is the most com- ~ Answer D. This patient has erythema multiforme
mon cause of atraumatic hip pain in children. It (EM) minor, which is a relatively benign disease
usually occurs in children <6 years old, and the that results from host hypersensitivity to an anti-
cause is unknown. Referred pain to the knee may be genic challenge. EM minor is at the benign end
the only complaint, so careful inspection of the hip of a clinical spectrum that includes EM major,
is mandatory in all patients with isolated knee pain. Stevens-Johnson syndrome (SJS), and toxic epider-
Diagnosis is made by excluding other more serious mal necrolysis (TEN). The classic rash is typically
causes of hip pain, namely septic arthritis, frac- symmetric and involves the extremities, palms, and
ture, slipped capital femoral epiphysis (SCFE), and soles. The archetypal lesion is a "target" lesion mea-
Legg-Calve-Perthes disease. Patients with transient suring <3 em and consisting of two concentric rings
synovitis are less likely than those with septic arthri- around a dusky, central disk. Mucosal involvement
tis to have fever, elevated erythrocyte sedimentation almost never occurs in EM minor, whereas it is very
rate (ESR), and tenderness of the hip. Management common in EM major and is almost exclusively lim-
of transient synovitis is purely supportive with rest ited to the oral cavity. Nikolsky's sign is not present
and NSA!Ds. Hip fracture in the pediatric patient in either EM minor or major. Corticosteroids are
is suggested by high-force trauma and severe ten- sometimes used for treatment but their benefit has
derness is common. Osteogenesis imperfecta is a not been proved. Immunoglobulin plays no role in
rare disease that causes problems in bone synthesis treatment. The lesions resolve without intervention
due to collagen defects. Frequent fractures are com- within 1 to 3 weeks. Although almost SO% of cases of
mon and physical examination may demonstrate EM minor are idiopathic, HSV is the most common
blue sclerae, deafness, and ligamentous laxity. Sub- infectious agent triggering the disease. EM minor
clinical cases may be more common than previously and major are typically caused by infectious agents,
recognized. SCFE occurs when the femoral epiph- whereas SJS and TEN are typically due to drug ex-
ysis slips off the metaphysic, usually in adolescents. posure. (Figure courtesy of Colleen Campbell, MD.
Legg-Calve-Perthes disease is avascular necrosis of Reprinted with permission from Campbell C. Green-
the femoral head due to unknown reasons. It occurs berg's text-atlas of emergency medicine. Lippincott
from childhood to adolescence but is less common Williams & Wilkins; 2004:395.)
than transient synovitis.
~ Answer B. AS usually occurs due to congenital bi-
~ Answer A. NPH was initially described in 1965 cuspid structure, calcific degeneration, or rheumatic
as a triad of difficulties of gait (ataxia), altered heart disease. The classic murmur is a systolic
mentation (dementia), and urinary sphincter dys- crescendo-decrescendo murmur at the right upper
function (incontinence) in concert with enlarged sternal border or cardiac base. The natural history
ventricles (hydrocephalus) but apparently normal of AS usually follows a sequence of findings, starting
cerebrospinal fluid (CSF) pressure upon lumbar with angina, followed by syncope, and then heart
puncture (LP). Recent research has shown that while failure. As each subsequent finding occurs, the mor-
the CSF pressure may be normal during a single LP, tality increases significantly. Valve replacement is the
patients with NPH experience transient increases in only definitive treatment once symptoms appear.
CSF pressure which can now be appreciated in centers
capable of performing CSF monitoring. Therefore, ~ Answer D. The EKG shows a regular, narrow
many authors have advocated changing the name complex tachycardia at a rate of 175, most likely
of this illness to "chronic hydrocephalus." Though paroxysmal atrioventricular (AV) nodal reentrant
there is no "classic" gait, walking difficulties are the tachycardia. The patient is hemodynamically un-
most consistent and prominent feature ofNPH. Fur- stable and synchronized cardioversion at 50 J is
thermore, ataxia is most likely to improve after CSF indicated after appropriate sedation. Adenosine is
shunting. Alterations in cognition and mentation the next best option, but hypotension and severe
may be so mild that they are not noticed by either the symptoms warrant more emergent conversion to si-
patient or the patient's family and the term dementia nus rhythm. Diltiazem and amiodarone can be used
is overly broad generalization. Urinary incontinence to convert the rhythm, but will cause further hy-
is present only in the later stages although a sensation potension. Defibrillation is only indicated in patients
of urinary urgency is almost always present. Pa- who lack pulses (ventricular fibrillation, pulseless
pilledema is a sign of increased intracranial pressure. ventricular tachycardia).
302 1000 Questions to Help You Pass the Emergency Medicine Boards

~ Answer A. Dental infections are by far the most with vertigo to walk during bedside examination.
common cause of Ludwig's angina (98% to 99% Finally, nystagmus is an important physical exami-
are odontogenic), and Ludwig's angina may follow nation finding in patients with vertigo. Spontaneous
dental extraction. Other predisposing conditions in- nystagmus of peripheral origin does not change di-
elude diabetes mellitus, malnutrition, alcoholism, rection with gaze to either side. Nystagmus in such
immunocompromised states such as AIDS or or- patients, however, does increase in amplitude when
gan transplantation, mandible fractures, tongue patients look in the direction ofthe fast phase (known
piercing, peritonsillar or parapharyngeal abscesses, as the Alexander's law). In patients with central le-
submandibular sialoadenitis and trauma. Ludwig's sions, nystagmus generally changes direction when
angina describes a rapidly progressive gangrenous the patient looks in the direction of the fast phase.
cellulitis of the soft tissues of the neck and floor Nystagmus that is purely vertical (upbeat or down-
of the mouth that originates in the submandibular beat nystagmus) is almost always caused by a central
space. Patients present with dysphagia, neck swelling, lesion, typically of brainstem vestibular pathways.
neck pain, and elevation of the tongue. Airway com- Finally, nystagmus due to peripheral lesions typically
promise may occur rapidly and without warning, fatigues or ceases when the patient fixates his or her
so attention to the airway is the primary task of vision on a target object. Fixation does not generally
emergency physicians. affect nystagmus due to central lesions.

[!Z) Answer D. Ten percent to 25% of patients with ~ Answer C. Floaters with or without painless visual
gout have renal stones, and the rate correlates with loss is the usual clinical presentation of retinal
the degree of hyperuricemia. For example, >50% of detachment Direct funduscopy and visual acuities
patients with a serum uric acid level > 13 mg per dL may be completely normal if the detachment is
have stones. peripheral or small. The elevated retina will appear
out of focus and as a hazy gray membrane.
~ Answer C. Patients with penetrating injuries just Ophthalmologic consultation should be emergent
lateral to the spine often have incomplete cord since reattachment is often successful if performed
injury, usually causing hemisection of the cord, early in the disease course.
known as Brown-Sequard syndrome. Neurologic
findings include loss of ipsilateral motor function ~ Answer C. Asymptomatic bacteriuria is a common
and vibratory/position sensation and contralateral finding amongst healthy pregnant and nonpreg-
pain/temperature sensation distal to the injury. The nant women. There is a slightly higher incidence of
fibers carrying pain/temperature sensation decussate asymptomatic bacteriuria amongst pregnant women,
(cross) a few levels after they enter the cord ranging from 2% to 10% (vs. 1% to 5% in nonpreg-
from the periphery, so contralateral findings are nant women). However, most of these women have
observed. The fibers carrying motor axons and preexisting bacteriuria, which is detected during rou-
vibratory/position sensation do not decussate until tine early pregnancy screening. It is critical to treat
high in the spinal cord, so ipsilateral findings all pregnant women with asymptomatic bacteriuria
predominate. The functional outcome for patients because pregnant women with asymptomatic bac-
with cord hemisection is favorable overall with teriuria have a 20- to 30-fold increased rate of
few patients losing bowel, bladder, or ambulatory pyelonephritis. The development of pyelonephritis
function. in pregnancy increases the risk of preterrn delivery
and low birth weight infants. The increased risk of
~ Answer B. It may be difficult to clinically differ- pyelonephritis in pregnancy is a result of ureteral sta-
entiate between patients with central and peripheral sis, which is due in part to the effects of progesterone
vertigo. However, peripheral vertigo is classically on ureteral peristalsis and in part to the effects of
associated with marked nausea and vomiting. direct uterine compression of the ureters. The uterus
Furthermore, peripheral vertigo more commonly oc- primarily compresses the right ureter such that 75%
curs along with auditory symptoms such as hearing to 80% of pyelonephritis cases in pregnancy are
loss, tinnitus, or a feeling ofpressure or fullness in the right sided. As in nonpregnant women, the principle
ear. Although all patients with vertigo may have some pathogens in pregnant women with urinary tract
difficulty walking, many patients with central lesions infection are E. coli, K pneumoniae, and Proteus spp.
cannot even stand or take a single step without falling.
In contrast, patients with peripheral vertigo can usu- ~ Answer C. Although CT scan is the most sensitive
ally walk, even in the acute phase of their illness. imaging technique for viewing pelvic fractures, plain
This is why it is important to encourage patients films may add important anatomic information for
TestlO 303

the orthopedist. The Judet view is an oblique view ~ Answer A. The rotator cuff is a group of mus-
mosthelpfulforviewingtheacetabulum. TheJudetx- cles composed of the supraspinatus, infraspinatus,
ray should never be performed in unstable patients, teres minor, and subscapularis (SITS). Its primary
as isolated acetabular fracture rarely causes severe function is to stabilize the shoulder joint. It is most
hemodynamic compromise. The AP view of the commonly injured chronically with repetitive mo-
pelvis is the most important initial x-ray of pelvis tions or acutely with a fall on an outstretched hand.
to obtain in the general trauma patient. The supraspinatus is the most common ofthe rotator
cuff muscles to be injured. Physical examination of
~ Answer E. Tricyclic antidepressants cause blockade patients with supraspinatus tears indicates inability
of the following: a-1, muscarinic, histamine, GABA, to maintain active abduction at 90 degrees without
cardiac potassium effiux, fast sodium channel, limitation in passive range of motion. Diagnosis may
serotonin reuptake, and norepinephrine reuptake. be confirmed with MRI and acute tears may require
Because of these effects and the potential for lethality early surgery. The deltoid muscle is not technically
in overdose, they are no longer indicated as first-line part of the rotator cuff.
therapy for the management of major depression.
~ Answer C. This patient has Bell's palsy. Bell's
~ Answer C. The picture demonstrates hemotym- palsy is defined as paresis or paralysis of the fu-
panum, indicative of a basilar skull fracture in the cial nerve (seventh cranial nerve), which is usually
middle fossa. Other signs of middle fossa fracture unilateral. The first step in the diagnosis is to de-
include Battle's sign (posterior auricular ecchy- termine that the paresis or paralysis is due to a pe-
mosis), otorrhea, facial nerve palsy, and hearing ripheral instead of a central nervous system (CNS)
loss. Signs of anterior fossa fractures include cere- lesion. In patients with CNS lesions, furrowing of
brospinal fluid (CSF) rhinorrhea and raccoon eyes the eyebrows and closure of the eye is unaffected.
(bilateral periorbital ecchymoses). Signs of posterior This is due to the fact that the neurons of the facial
fossa fractures involve impairment in swallowing and nucleus that innervate the upper face receive input
maintenance of airway due to cranial nerve IX and X from both cerebral hemispheres (whereas the neu-
injuries. Otitis media is unlikely to be involved in pa- rons of the facial nucleus that innervate the lower
tients with head trauma. (Figure from Benjamin B, face receive input primarily from the contralateral
Bingham B, Hawke M, et al. A rolor atlas of otorhi- cerebral hemisphere). A CT scan would only be
nolaryngology. Philadelphia: JB Lippincott Co; 1995. necessary in patients with evidence of a CNS le-
Artwork© Bruce Benjamin, Brian Bingham, Michael sion upon physical examination. The presence of a
Hawke, and Heniz Stammberger, with permission.) vesicular rash over the ear indicates Ramsay-Hunt
syndrome. Ramsay-Hunt syndrome, also known as
~ Answer A. In children, rubella (German measles) herpes zoster oticus, results from reactivation of
is characterized by a minimal prodromal illness (as Varicella Zoster virus in the geniculate ganglion
opposed to adults), a "3-day rash," and generalized of the facial nerve. It is much less common and
lymphadenopathy. Forchheimer spots, pinpoint or carries a more severe prognosis for recovery than
larger erythematous spots on the soft palate, may Bell's palsy of unknown cause or Bell's palsy due
also be found. However, similar lesions may also be to reactivation of HSV. More than 75% of patients
seen in measles and scarlet fever, so the presence will recover without treatment. Therefore, medical
of Forschheimer spots is not pathognomonic for treatment is aimed at the remaining population.
rubella. The rash is a pink-red maculopapular Because it is not possible to identify which patients
eruption that first appears on the face and rapidly will recover fully without treatment, all patients are
spreads downward to the neck, arms, trunk, and legs. generally treated upon making the diagnosis. The
On the second day, it begins to disappear from the treatment for Bell's palsy includes corticosteroids,
face and the entire rash typically disappears by the antiviral agents directed against herpes viruses (acy-
end ofthe third day. Lymphadenopathy is generalized clovir, valacyclovir, famciclovir), and artificial tears
but most commonly involves the suboccipital, and eye ointment to prevent corneal drying. There
postauricular, and cervical nodes. Although lymph is disagreement over the best regimen and the tim-
node tenderness typically subsides after 1 to 2 days, ing of presentation. However, it is generally felt that
lymphadenopathy may persist for several weeks. earlier treatment yields better results and that corti-
Tonsillar exudates are present in several causes of costeroids are of limited utility if a patient presents
pharyngitis. Koplik's spots are pathognomonic for after 7 days of symptoms. Patients with diabetes
measles. Febrile seizures classically occur in patients and hypertension have a more severe course and
with roseola infantum (exanthem subitum). Pastia's less complete recovery. Recurrence of Bell's palsy
lines occur in scarlet fever.
304 1000 Questions to Help You Pass the Emergency Medicine Boards

rarely occurs. In cases of recurrence, consideration anhedonia (loss of interest or pleasure in pleasurable
should be given to alternative diagnoses. activities) for the diagnosis. According to the Diag-
nostic and Statistical Manual of Mental Disorders,
~ Answer D. Rheumatic fever occurs several weeks Fourth Edition (DSM-IV) criteria, symptoms need to
after untreated streptococcal pharyngitis. The di- be present for 2 or more weeks for formal diagnosis.
agnosis is made by the Jones criteria-either two Symptoms include disturbances in appetite, sleep,
major (polyarthritis, erythema m.arginatum, chorea, concentration, activity, and energy and thoughts of
carditis, subcutaneous nodules) or one major and guilt or suicide. The symptoms should not be caused
two minor (arthralgias, fever, increased ESR or by a thought disorder, medical condition, or sub-
C-reactive protein (CRP), prolonged PR interval). stance abuse. Delusions and visual hallucinations are
Migratory arthritis of major joints is the most com- more characteristic of thought disorders than major
mon symptom, followed by carditis. Chorea and depression. Major depression is almost ten times as
erythema marginatum are uncommon, but fairly common as bipolar disorder. It is extremely difficult
specific given a history of antecedent pharyngitis. to distinguish dementia from depression in the el-
Fever is neither sensitive nor specific. derly on clinical grounds alone. Sometimes a trial of
antidepressant therapy may be necessary to make the
~ Answer E. The most common acid-base distur- diagnosis-major depression is extremely responsive
bance in the setting of acute asthma exacerbations to pharmacotherapy whereas dementia is not.
is a respiratory alkalosis. However, concomitant
metabolic acidosis due to lactic acidosis occurs in ~ Answer D. Boehler's angle is used to help evaluate
up to 28% of patients. The etiology of the lactatemia the calcaneus in the setting of calcaneal fractures.
is not known but it has been hypothesized to occur Boehler's angle is formed by the intersection of a
because of fatiguing respiratory muscles. However, line drawn from the apex of the anterior tuberosity
recent case reports have demonstrated that lactatemia to the apex of the posterior facet and a line from
can occur even in intubated patients who are par- the apex of the posterior tuberosity to the apex of
alyzed and therefore have no respiratory muscle the posterior facet. It normally measures between 20
action. Regardless, the clinical relevance of an ele- and 40 degrees. Angles < 20 degrees are suggestive of
vated lactate level accompanying an acute asthma compression of the posterior facet. The other angle
exacerbation is not known, and lactate levels do not pictured is the angle of Gissane (also known as the
predict respiratory failure in critically ill patients. crucial angle) which can also be used to evaluate the
The presence, however, of a respiratory acidosis as posterior facet. (Figure reprinted with permission
indicated by the ABG above is an ominous sign. A from Bucholz RW. Rockwood and green's fractures in
normal or elevated Pco2 in a tachypneic asthmatic adults. Lippincott Williams & Wilkins; 2005.)
typically suggests severe obstruction and impend-
ing ventilatory failure. Exceptions to this rule occur ~ Answer B. This patient has pityriasis rosea, which
most commonly in patients with underlying chronic is a common, self-limited rash that is thought to be
obstructive pulmonary disease who may retain C02 of viral etiology. It most commonly occurs between
at baseline. However, such patients should have a the ages of 10 and 35 with a mean age of 23.
normal pH and an elevated Hco3 - owing to chronic Approximately 20% to 50% of patients present with
renal compensation. Leukocytosis, mild hypokalemia a "herald patch," a single, 2- to 10-cm salmon-
and eosinophilia are not useful in the management pink plaque with a fine silvery scale that rings the
of acute asthma. border of the lesion. Many patients believe they
have ringworm, and the lesion may commonly be
~ Answer C. Rheumatic heart disease most com- mistaken for ringworm before the development of
monly predisposes the mitral valve to structural additional lesions (KOH testing can rule this out).
damage as well as susceptibility to infective endo- Approximately 1 to 2 weeks after the herald patch,
carditis. The tricuspid valve is affected most often multiple additional 1- to 2- em round and oval
in patients who use IV drugs. The pulmonic valve lesions develop on the trunk. The longitudinal axis
is not usually affected by rheumatic heart disease of the oval lesions runs along the skin lines of
or endocarditis. Irregularities of the aortic valve are the back and the overall pattern may resemble the
usually due to congenital bicuspid structure, calcific branches of a pine tree. For this reason, pityriasis is
degeneration, or less commonly, rheumatic heart often said to have a "Christmas tree" distribution.
disease. No treatment is necessary and the lesions resolve
within 2 to 3 months, although ultraviolet B
[!!) Answer C. Major depression is a type of mood (UVB) phototherapy will hasten resolution and
disorder which requires either depressed mood or decrease pruritus. (Figure reprinted with permission
TestlO 305

from Barankin B. Stedman's illustrated dictionary should be part of the cocktail of atropine, glucagon,
of dermatology eponyms. Lippincott Williams & and insulin therapy for patients with {3-blocker
Wilkins; 2004.) overdose. Hyperkalemia and renal failure may also
occur, but much less often than seizure.
~ Answer E. Aneurysms are most often asymp-
tomatic and diagnosed incidentally on routine ex- ~ Answer C. The absolute lymphocyte count is not
amination or imaging studies. Most symptomatic only the earliest indicator of the acute radiation
patients complain of vague abdominal pain or ab- syndrome, but it also provides prognostic informa-
dominal distension, although urinary retention and tion. Patients with an absolute lymphocyte count
constipation may occur. Nausea occurs only in the <500 have a very poor prognosis with very high
setting of other symptoms pointing to the diagnosis. lethality when the absolute lymphocyte count < 100.
Acute abdominal pain or distension in patients at In contrast, patients with an absolute lymphocyte
risk for abdominal aortic aneurysm (AAA) indicates count > 1,000 have significant, but usually nonlethal
rapid expansion or rupture. injuries.

~ Answer B. The pork tapeworm, T. solium, causes [!!) Answer D. The patient presents in stage N hem-
neurocysticercosis, an extremely common cause of orrhagic shock, with tachycardia, hypotension, and
seizures worldwide. Infection occurs after humans alteration of mental status. These patients will re-
eat contaminated pork. The parasite multiplies in quire both crystalloid and colloid fluid resuscitation
the small intestine and eventually enters the blood- during the primary survey. The initial blood product
stream and brain. The tapeworm produces cysts in to administer to men in hemorrhagic shock should
the brain, which appear as ring-enhancing lesions be type 0 positive uncrossmatched blood. Uncross-
on contrast Cf scan. Seizures are the most com- matched blood is easily obtainable in the ED and
mon serious manifestation of disease. Praziquantel may be given immediately, without laboratory anal-
and corticosteroids are the medical treatment. Neu- ysis. The Rh fuctor may be positive in the donor unit
rosurgical consultation is required in all cases to when transfusing men and women beyond child-
evaluate and manage increased intracranial pressure. bearing years, as there is no danger of formation of
T. cruzi causes Chagas disease, a cardiomyopathy due antibodies which might occur in future pregnancies
to parasitic infection occurring primarily in Latin in Rh negative women of child-bearing age. Fresh
America. T. saginata, the beef tapeworm, causes a frozen plasma is indicated to keep up with clotting
self-limited gastroenteritis. T. trichiura, the whip- factor losses when patients have received four to five
worm, also causes gastroenteritis, which may lead units of packed red blood cell (RBC) transfusions
to iron-deficiency anemia due to malabsorption. during the course of a resuscitation.
L. braziliensis causes chronic cutaneous ulcerations.
~ Answer B. Generally, pediatric patients with oral
~ Answer B. Most radioisotopes decay by beta decay burns can be safely discharged home with minimal
which results in the emission of high-velocity elec- or no ancillary testing. Delayed labial artery bleeding
trons or positrons called beta (in symbol) particles. occurs 7 to 14 days after the initial injury, when
f3 Particles penetrate the skin to approximately 8 mm the eschar falls off the wound. It typically responds
and may result in skin burns, although customary to local, direct pressure, although patients will
clothing protects covered areas. Therefore, only ex- occasionally require a figure-S suture to control
posed skin is susceptible to exposure. y Rays are often bleeding. Otherwise, the most serious complication
emitted after f3 -particle emission and penetrate more of burns to the oral commissure is a cosmetic facial
deeply than f3 particles. y Rays are responsible for deformity. Therefore, patients should be urgently
the acute radiation syndrome. a Particles are helium referred to a facial or oral surgeon for further
nuclei and have very superficial penetration which is evaluation and possible oral splinting. There is no
limited to the epidermis. They generally present no correlation between an isolated oral burn and cardiac
health hazard unless they are inhaled. injury or myoglobinuria. However, the patient
should receive a thorough examination to ensure
~ Answer D. Propanolol is the most dangerous that there are no other contact wounds indicating a
{3-blocker in overdose. It is highly lipophilic and more extensive electrical path.
readily enters the CNS, causing seizures and coma.
{3-Blocker overdoses commonly cause bradycardia, ~ Answer B. The abdominal film demonstrates gas-
AV block, and hypotension, and less often lead to tric and duodenal dilation. Malrotation with midgut
tachydysrhythmias. Hypoglycemia is much more volvulus must be a leading consideration in any
common in children than in adults, and glucose toxic-appearing infant with a history ofsudden onset
306 1000 Questions to Help You Pass the Emergency Medicine Boards

bilious emesis. Malrotation occurs in approximately ~ Answer E. ODS is a complication ofhyponatremia


1 in 500 live births, or about half as often as pyloric treatment that most commonly affects alcoholics and
stenosis. Of all infants with malrotation, 75% will de- patients with chronic malnutrition. It carries a grave
velop volvulus and 75% of these infants will present prognosis with no known treatment. It does not typ-
in the first month (most in the first week). Most ically occur after the treatment ofpatients with acute
infants present with acute-onset bilious emesis and hyponatremia ( <48 hours). Patients with pseudohy-
obstruction. Plain films are usually nonspecific and ponatremia (e.g., diabetic patients with severe hyper-
may be normal, but they may also reveal the "double glycemia) are not at risk for ODS and have normal
bubble" sign which reflects the dilated stomach and sodium body stores. Sensory abnormalities are not
proximal duodenum (the intervening pylorus sepa- common in ODS. The classic presentation is altered
rates the two "bubbles," not shown in Figure 10-12). mental status (lethargy to coma) with pseudobulbar
This may also be seen with duodenal atresia but that is palsies and spastic quadriplegia after treatment for
an illness that presents in the newborn nursery. Mal- hyponatremia. Patients most commonly have anini-
rotation with midgut volvulus is a life-threatening tial improvement in their sensorium after treatment
illness that requires emergent surgical intervention ofhyponatremia with hypertonic saline, but patients
to reduce the volvulus and relieve the ischemia caused developing ODS then rapidly decline in the ensuing
by constriction of the bowel's mesenteric blood sup- 48 to 72 hours. Most patients present 1 to 6 days after
ply. Necrosis of the bowel may occur in as little initial treatment of hyponatremia.
as 3 hours. (Figure reprinted with permission from
Fleisher GR. Atlas of pediatric emergency medicine. ~ Answer D. Traumatic aortic injury (TAl) occurs
Lippincott Williams & Wilkins; 2003.) most commonly from high-speed motor vehicle
collisions causing blunt thoracic trauma. Most
~ Answer C. Thromboembolic disease is the leading traumatic aortic ruptures are immediately fatal,
cause of death in pregnancy. The risk of deep venous but patients who survive to ED evaluation are
thrombosis is highest in the postpartum period usually successfully treated. The descending aorta
(puerpium), although the risk is elevated throughout just distal to the subclavian artery is the most
pregnancy. Warfarin is contraindicated throughout commonly injured site. Chest and back pain are
pregnancy due to its strong association with fetal the most common symptoms. Diagnosis is made by
malformations even when given after the period of a combination of chest x-ray and CT aortography.
organogenesis (days 21 through 56 offetallife). V/Q Management of TAl involves operative repair, but
scans actually expose the fetus to more radiation blood pressure and heart rate control with fJ- blockers
than helical CT scans when looking for pulmonary is essential to prevent further damage to the aorta
embolism (PE). from shear forces. Labetalol is an ideal single agent
for reduction of blood pressure and heart rate.
~ Answer C. The patient has air-space consolidation Clonidine, hydralazine, and enalaprilat all reduce
in the right mid-lung field in the setting of trauma, blood pressure, but often cause reflex tachycardia
which is consistent with a pulmonary contusion. and require a fJ-blocker in addition to reduce the
Management is directed at adequate oxygenation number of heart beats and shear forces to the aorta.
and ventilation and prevention of secondary compli- Observation alone will result in worsening aortic
cations such as acute respiratory distress syndrome injury and risks immediate death ifthe aorta ruptures
(ARDS) and pneumonia. There is no obvious pneu- completely.
mothorax present, although an occult pneumothorax
may be picked up if CT chest is performed. On an [I!) Answer C. The patient has a ring-enhancing le-
upright chest x-ray, hemothorax would appear as sion seen in the left parietal area on contrast CT
a pleural effusion around the lower lung segments. scan of the brain. In a patient with HIV, toxo-
Chest radiography is not specific for the diagnosis of plasmosis, due to the parasite Toxoplasma gondii,
traumatic aortic rupture, which requires CT angiog- is the most likely cause. Treatment is with sulfa-
raphy for definitive diagnosis. Cardiac contusion is diazine, pyrimethamine, and adjunctive leucovorin.
an older term to describe blunt cardiac injury, which Corticosteroids may be used as adjunctive therapy
is diagnosed by a combination of EKG, echocar- in severe cases. Surgical excision is not indicated in
diography, and sometimes cardiac markers. (Figure toxoplasmosis. Mebendazole is an agent used to treat
courtesy of Robert Hendrickson, MD. Reprinted other parasitic infections. Clindamycin may be used
with permission from Hendrickson R. Greenberg's to treat brain abscesses along with surgical drainage.
text-atlas ofemergency medicine. Lippincott Williams (Figure reprinted with permission from Silverberg M.
& Wilkins; 2004:634.)
TestlO 307

Greenberg's text-atkis of emergency medicine. Lippin- • C. perfringens: Toxin-mediated, predominantly


cott Williams & Wilkins; 2004:999.) diarrheal illness that occurs 6 to 12 hours after
ingestion and is common in meats, poultry,
[ll] Answer D. The MCP joint and IP joints are and gravies. Requires ingestion of live organisms
constructed differently. In the MCP joint, the meta- because the toxin is produced in vivo. The illness
carpal head acts as a cam, such that the distance resolves in 24 hours with supportive care.
between the metacarpal head and the base of • S. aureus: Toxin-mediated emetogenic illness that
the proximal phalanx is greater in flexion than occurs 1 to 6 hours after ingestion. Common in
in extension. Therefore, when the MCP joint is high-protein foods such as ham, eggs, poultry,
extended, the collateral ligaments are relatively custard-based pastries as well as potato or egg
flaccid, while they are stretched during flexion. salads. Symptoms typically resolve within 8 hours
If patients are splinted with the MCP joints in without specific treatment.
extension, contraction of the collateral ligaments • B. cereus: Causes two distinct illnesses. First
occurs and patients may not be able to fully flex their is toxin-mediated illness that results in acute
digits once the splint is removed. In contrast, the food poisoning, predominantly characterized by
IP joint is a hinge joint with two condyles on the vomiting. Very similar to the syndrome caused
proximal phalangeal head. Although some flexion by S. aureus except it almost always occurs after
can be used, extension of the IP joints ensures that ingestion of fried rice. Second illness is a diarrheal
its collateral ligaments will also be taut. illness almost indistinguishable from the illness
caused by C. perfringens. As with C. perfringens, it
~ Answer E. Activated charcoal prevents absorption is a toxin-mediated event but requires ingestion of
of many drugs from all sites in the GI tract. No- live organisms as the toxin is produced in vivo.
table exceptions to the drugs absorbed by activated • V. parahaemolyticus is increasingly a cause of
charcoal are ions, heavy metals, ethanol, and hydro- foodbome illness in raw or undercooked fish or
carbons. Choices A to D all fall into one of these shellfish. It causes disease through direct intestinal
categories, and only acetaminophen is absorbed by invasion as well as enterotoxin production and is
activated charcoal. almost always characterized by explosive watery
diarrhea and crampy abdominal pain. Vomiting
~ Answer A. Liquid drain cleaner usually contains and bloody stools may occur but are less common.
a strong base, such as sodium or potassium hy-
droxide. The treatment of caustic ingestions is gen- [!!) Answer D. In septic arthritis, the synovial WBC
erally supportive and involves diagnosis of severe usually exceeds 50,000 WBC per mm3 and often ex-
esophageal burns with endoscopy. Small amounts ceeds lOO,OOOper mm3, with >75% polymorphonu-
of water or milk may be taken immediately after the clear cells. However, patients with gout, pseudogout,
ingestion to wash away the excess caustic material. and rheumatoid arthritis may also have WBC counts
Large amounts of fluids should never be taken, as in this range with a similar differential. Therefore,
this may precipitate vomiting, which will dramati- when the WBC is elevated approximately 50,000
cally worsen esophageal injury. Calcium gluconate per mm3 , septic arthritis must be presumed until
is indicated in patients with hydrofluoric acid ex- ruled out.
posure to replete the calcium which is bound by
the extremely electronegative fluoride ion. Steroids ~ Answer C. The patient has evidence of slipped
have not been proved to be beneficial in patients capital femoral epiphysis (SCFE). The most common
with most caustic injuries, although it may offer presenting age is early adolescence, it is more
some benefit in patients with moderate esophageal common in boys, and African Americans are the
injuries. Antibiotics should not be given unless that highest risk population. Obesity is the most common
patient has received steroids or unless there are associated condition, but hypothyroidism may also
clear signs of perforation. Neutralization therapy be involved. Patients typically present with unilateral
with acid or base should never be pursued, as this or bilateral hip or knee pain, which is intermittent
will lead to further injury. and worse on activity. Physical examination may
demonstrate progressive loss of internal rotation,
~ Answer A. Although C. perfringens is probably the muscle atrophy, and leg length discrepancy. Lateral,
most common cause of acute food poisoning in AP, and frog-leg radiographs show the femoral
the United States, all of the above bacteria may epiphysis slipping inferiorly and posteriorly off
cause food-borne illness. Their classic associations the femoral neck. Emergency management includes
are given below. non-weight-bearing and orthopedic consultation.
308 1000 Questions to Help You Pass the Emergency Medicine Boards

~ Answer B. Postoperative ileus is a common, ex- is distinguished from normal postex:traction pain
pected outcome after laparotomy. However, mul- by the severity, delayed onset. and lack of relief
tiple electrolyte abnormalities may also cause or with NSAIDs. Halitosis is almost always present in
exacerbate the condition. Hypokalemia is the patients with dry socket. Direct anesthesia, irrigation,
most common electrolyte abnormality responsible and gentle packing are indicated. Treatment may also
for ileus, though hypomagnesemia, hyponatremia, include NSAIDs and narcotics for pain and penicillin.
hypocalcemia, and uremia may also contribute. Recreating a clot is associated with a higher incidence
of osteomyelitis.
~ Answer B. Any patient with underlying joint dis-
ease is more likely to develop septic arthritis. How- [iiJ Answer C. Endocarditis prophylaxis with antibi-
ever, patients with underlying rheumatoid arthritis otics for patients with prosthetic or damaged valves
are most likely to develop concurrent septic arthri- is recommended for the following procedures: Dental
tis. The diagnosis is usually delayed because patients cleaning, rigid bronchoscopy, endoscopic retrograde
are often afebrile, the presentation is frequently in- cholangiopancreatography (ERCP), and cystoscopy.
dolent and the symptoms are initially attributed to
an exacerbation of the patient's chronic condition. ~ Answer B. Ninety percent of anal fissures are
The case-fatality rate is 30% to 50%. Therefore, any located in the posterior midline. The remaining
patient with an underlying connective tissue or joint 10% are located in the anterior midline. Fissures
disease who develops sudden inflammation in one or located elsewhere should prompt consideration of
two joints should be presumed to have septic arthritis an underlying disease, such as Crohn's disease,
until proved otherwise. leukemia, HIV infection, tuberculosis, or syphilis.

~ Answer E. Troponin I is highly specific for cardiac ~ Answer E. Recognizable lead points other than
muscle. It starts rising between 3 and 6 hours after Peyer's patches are found in only 2% to 8% of
infarction, peaks at 12 to 24 hours, and returns to patients. All of the listed choices may serve as lead
normal within 7 days. The specificity of troponin I is points in intussusception. However, most cases are
excellent at 6 hours postinfarction, but the sensitivity thought to occur when an enlarged Peyer's patch
does not approach 100% until12 hours. telescopes into adjoining bowel The most common
location is ileocolic.
~ Answer B. The primary survey of trauma is in-
tended to diagnose immediate life threats. The simple [!!) Answer C. This patient has erythema infectiosum
mnemonic "ABCDE" indicates the appropriate eval- (or fifth disease) caused by parvovirus B19. The
uation: Airway, breathing, circulation, disability, and illness is common and frequently asymptomatic.
exposure. The airway is considered intact when there In symptomatic children, papules develop on the
is clear, coherent speech and lack of obstruction. cheeks which promptly coalesce to form a bright,
The lungs are next examined to assess for evidence erythematous plaque which spares the perioral area
of hemothorax, pneumothorax, or flail segments. (circumoral pallor) and nasolabial fold. The rash
Pulses in various extremities give a rough estimate typically lasts for approximately 4 days. Parvovirus
of the patient's hemodynamic status. A quick neu- B19 is most important in causing aplastic crises
rologic examination to check for pupillary reactivity, in patients with underlying hemolytic anemias.
extremity movement, and rectal tone completes the In addition, infection of nonimmune pregnant
disability portion of the primary survey. Lastly, com- women rarely results in fetal hydrops and death.
pletely disrobing and turning the patient to expose all No treatment of immunocompetent individuals is
body parts will reveal any hidden external injuries. required, though practitioners should determine
The cardiac and abdominal examinations, though the immune status of exposed pregnant women.
important. are not part ofthe primary survey because (Figure courtesy of John Loiselle, MD. Reprinted
they will rarely reveal life threats that are immediately with permission from Chung EK. Visual diagnosis in
lethal and treatable. The otic and nasal examinations pediatrics. Lippincott Williams & Wilkins; 2006:433.)
should only take place in the secondary survey.
~ Answer B. IVDUs have skin flora that are trans-
r!Z) Answer C. The patient has dry socket, or acute mitted through superficial veins into the greater
alveolar osteitis. It occurs in 5% to 20% of all venous circulation to the right heart. These pathogens
tooth extractions, and the posterior mandibular (usually S. aureus) lodge most commonly in the tri-
teeth are at highest risk. Age, smoking, and poor cuspid valve to cause right-heart endocarditis and
dental hygiene are strong risk factors. Dry socket may also lead to septic pulmonary emboli.
TestlO 309

~ Answer E. This patient has evidence of bilateral Such patients may be managed as outpatients by
fucial nerve paresis. Bilateral facial nerve paresis or resuming their preexisting therapy or by adding cor-
paralysis is rare in the setting of Bell's palsy and ticosteroids ifthey have been compliant with therapy.
its presence should prompt a search for alternative Bowel rest has not been shown to be useful and is only
diagnoses. In general, patients with bilateral facial required in patients requiring surgery. The initial
nerve paresis should be considered to have Lyme treatment of fistulae is medical therapy. Toxic mega-
disease until proved otherwise. This patient's recent colon has been classically associated with ulcerative
camping trip suggests the possibility of a remote colitis, although Crohn's disease is increasingly rec-
tick bite, which led to infection with the spirochete ognized as a cause. Initial therapyis generally medical,
Borrelia burgdorferi. Ten percent of patients with and includes aggressive intravenous fluids, broad-
erythema migrans, which is not treated will develop spectrum antibiotics, parenteral corticosteroids, and
neurologic manifestations, the most common of decompression (nasogastric tubes, rectal tubes). Sur-
which is facial nerve palsy. Interestingly, patients with gical treatment is reserved for refractory patients or
facial nerve paralysis due to Lyme disease typically in cases of perforation.
have other constitutional symptoms or neurologic
findings that point toward the diagnosis. Other causes ~ Answer A. Unlike cases of spontaneous bacte-
of bilateral facial nerve palsy include myasthenia rial peritonitis (SBP), where gram-negative enteric
gravis, lymphoma, sarcoidosis, brainstem tumors, organisms predominate, patients with peritoneal
and Guillain-Barre syndrome (GBS). dialysis catheters tend to develop peritonitis from
gram-positive organisms, most commonly Staphy-
~ Answer B. Ovarian torsion is relatively uncom- lococcus species. Clinical signs and symptoms of
mon amongst patients in the ED but it is an important infection can be extremely mild and asymptomatic
cause of acute pelvic pain. The classic history is the infection is common. Diagnosis is through analysis
development of severe, acute pelvic pain associated of the peritoneal fluid-100 or more WBC per mm3
with nausea and vomiting. The patient may note that with a predominance of neutrophils or a positive
a sudden change in position precipitated the pain. Gram stain makes the diagnosis. Most cases can
Interestingly, recent reviews have demonstrated that be treated on an outpatient basis with intraperi-
the onset is often much more subtle, with pain lasting toneally administered antibiotics. The antibiotics of
from several hours to several weeks and only rarely choice are vancomycin plus any antibiotic with good
associated with nausea or vomiting. Torsion is more gram-negative coverage, such as a fluoroquinolone,
common in young women with an average age at third generation cephalosporin, aminoglycoside, or
onset in the mid 20s. In addition, it is much more aztreonam.
prevalent in patients with adnexal masses as ovar-
ian tumors are ultimately revealed in approximately ~ Answer D. Posterior hip dislocations account for
60% of women. Finally, there is an increased rate of 90% of all hip dislocations. The most common
torsion in pregnant women, as roughly 20% of cases mechanism of injury is a "dashboard injury," in
occur during pregnancy. Owing to obstructed venous which a seated patient strikes the dashboard with a
drainage, the ovary enlarges and ovarian enlargement flexed knee, driving the femur posteriorly. Owing to
is the most common finding. Doppler ovarian imag- the force required to dislocate the well-protected hip
ing is difficult because of the dual ovarian blood joint, posterior hip dislocations are often associated
supply, which may lead to the false perception of with multisystem trauma. Patients will present with
maintained arterial flow. Furthermore, many pa- a shortened leg, with the hip internally rotated,
tients may experience spontaneous and recurrent adducted, and slightly flexed. Approximately 10% of
torsion and detorsion. Ultrasonographies performed posterior hip dislocations are associated with sciatic
with a normal ovarian lie will not reveal vascular nerve injury, which is usually a neurapraxia. After
abnormalities. Finally, the presence of a large ovar- reduction, this is manifest by hamstring weakness
ian mass or hemorrhage within the ovary may make as well as weakness of all lower leg muscles, loss of
detection of vascular flow exceedingly difficult. Ab- ankle reflex, and hypoesthesia of the posterior thigh
normal color Doppler imaging is highly predictive of and complete lower leg. Posterior hip dislocations
torsion, but 50% of patients with surgically proved must be reduced emergently because of the high
torsion have normal Doppler imaging. risk of avascular necrosis of the femoral head. Films
should be obtained before reduction to determine
~ Answer E. Most patients with inflammatory bowel the presence of associated femur or pelvis fractures,
disease (IBD) (Crohn's disease or ulcerative colitis) however, unless a pulse deficit is present. Femoral
who present to the ED have only mild to moderate ex- head fractures are much more common in anterior
acerbations caused by a stoppage of their medicines. femoral dislocations, occurring in as many as 77% of
310 1000 Questions to Help You Pass the Emergency Medicine Boards

patients versus only 10% of patients with posterior of the posterior communicating artery ( [PCOM]
dislocations. Because the femur is internally rotated aneurysm). The oculomotor nerve exits the brain-
and adducted, the shadow of the lesser trochanter stem between the PCOM and the superior cerebellar
is not seen on an AP projection. This is one critical artery, so a PCOM aneurysm is well positioned to
tool to help differentiate posterior dislocations from impinge on the nerve as it exits the brain. However,
anterior dislocations. Another is to examine the compression of the nerve usually affects the pupils as
size of the femoral head. Because the posteriorly well, resulting in anisocoria. In addition to a PCOM
dislocated femoral head is closer to the x-ray aneurysm, it is important to evaluate the brainstem
cassette, it often appears smaller than the unaffected for signs of ischemia (vertebrobasilar insufficiency)
femoral head. as well as ischemic or hemorrhagic infarction. These
findings may not be picked up on a routine CT scan
~ Answer E. The patient has herpes zoster oph- ofthe brain. Although a cerebral angiogram would be
thalmicus. Treatment involves systemic antivirals useful for diagnosing an aneurysm, it is more invasive
with topical antivirals and antibiotics as optional and provides less information about the brainstem
adjunctive therapy. Urgent ophthalmologic consul- thananMRI.
tation is generally pursued. Corneal hypoesthesia is
common and over three fourths of patients recover [100] Answer A. E. coli is isolated in 47% to 55% of
completely. Dendrites (with bulbs) are seen in recur- the cases of spontaneous bacterial peritonitis (SBP)
rent herpes simplex conjunctivitis rather than herpes and gram-negative organisms are the most common
zoster conjunctivitis. The ophthalmic division ofcra- etiologic agents as a group. K. pneumoniae is the
nial nerve Vis involved. Anterior uveitis occurs often second most commonly isolated organism. This is
in herpes zoster ophthalmicus and the frequency is followed by S. pneumoniae, and other Streptococcus
independent of severity of corneal involvement. and Staphylococcus species. Although there have
been isolated reports of anaerobic and polymicrobial
I!!] Answer C. This patient has a mononeuropathy of infections in SBP, they are generally not considered to
cranial nerve III. In addition, the parasympathetic be causes ofSBP. Fever or abdominal pain in a patient
fibers of the oculomotor nerve seem to be spared. with ascites should raise the suspicion of infection
The most likely diagnosis is a diabetic mononeu- and prompt a paracentesis. The presentation of SBP
ropathy, which results from microvascular ischemia may be subtle, however, and include only mental
of a nutrient artery feeding the core of the oculomo- status changes without abdominal pain or tenderness
tor nerve. The peripheral aspect of the nerve, which upon examination. All patients with an ascitic fluid
contains the parasympathetic fibers to the pupil, is neutrophil count ~250 per mm3 and a clinical
less affected because of collateral blood supply. How- picture consistent with infection should be treated
ever, a more ominous possibility is an aneurysm with antibiotic therapy.
Index
Note: Page numbers followed byf indicate figures; those followed by t indicate tables.

A. haemolyticum, 257, 274 AIDS patients. See human anaphylactoid syndrome of pregnancy, 41,
abdominal aortic aneurysm (AAA), 2, 2, 15, immunodeficiency virus (HIV)/AIDS 58
71,87,137, 154,165,166,18o-181, patients anaphylaxis, 161, 176, 191,206-207
289,305 airflow obstruction, 35, 51 anemias
abdominal fullness, 102, 102, 118-119 air-space consolidation, 290, 306 aplastic, 256, 272
abdominal masses in infants, 228, 242 albendazole, for pinworm, 25 hemolytic, 161, 176
abdominal trauma, 21, 40, 57 albuterol, 70, 86 iron-deficienc;y,34,49-50,197,214-215
penetrating, 137, 155 alcohol, ingested, 33, 47 normocytic, 256, 272
abnormal fetal lie, 165, 181 alcohol abuse, 15 aneurysms
abruptio placentae, 32-33, 47, 281, 295 acute alcoholic; hepatitis and, 282, 296 abdominal aortic (AAA), 71,87
absolute lymphocyte count, 289, 305 acute epigastric abdominal pain and, 96, popliteal artery, 71, 87
absorbable sutures, 160, 175 96, 111 rupture and, risks of; 64, 80
acakxUouscholecystins,72-73,89 alcoholic: ketoacidosis (AKA) and, 167, angina, Prinzmetal's (or variant), 163, 179
acces-sory pathWliY syndromes, 195, 211-212 183,227,241 angioedema, 259, 275-276
acclimation or acclimatization, 98, 114--115 benzodiazepine overdose and, 231-232, angi.oplasty, 69, 85
acetaminophen 248 with stenting, for ischemic heart disease,
metabolism of, 9, 23 Boerhaave syndrome and, 136, 153 17
toxicity hematemesis and, 70, 86 angiotensin-converting enzyme (ACE), 16,
N -acetylcysteine (NAC) and, 36, 51 ''holiday heart syndrome" and, 61, 61, 76 161, 176, 259, 275-276, 284, 284--285,
patient or toxin odor and, 170, K. pneumoni4 and, 165, 181 300
186-187 magnesium deficiency and, 67,84 anion gap (AG), 131,146-147
Rumack-Matthew nomogram for, 48 seizures and, 224, 23 7 Anopheles mosquito, 96, 111
severity of, determining, 228, 243 vomiting blood and, 43, 60 anorectal problems, 161, 176
treatment for, 5, 18 Werniclce-K.orsakoffsyndrome and, 63, 79 antepartum asphyxia, 8, 21
acetazolamide Wernicll:e's enc;ephalopathy and, 63, 78-79 anterior c:erebral artery, 68, 84
withdrawal and, 195,212-213 anterior circulation, ischemia of, 195,211
acetazolamide, for acute mountain sicknes-s
alcoholic; hepatins, ac:ute, 282, 296 anterior communicating artery, 68, 84
(AMS), 17-18
alcoholic: ketoacidosis (AKA), 167,183,227, anterior cord syndromes, 8, 12, 22, 27,251,
a~cholinesterase,37,52,158,172
241 265
achalasia, 8, 22
alcohol ingestions, toxic;, 166, 182 anterior c:ruciate ligament (ACL), 128, 142
acromioclavicular (AC) separation, 42, 42,
methanol and, 161, 176 anterior glenohumeral (shoulder)
58-59,188,203-204
alkali, 199,217 dUdocations,72,88
acromioclavicular ligament, torn, 42, 42,
allergic: interstitial nephritis, 98, 114 anteriorshoulderdidocations, 134,151
58-59
allergi.c:reactions,40,56,131,147 anterior shoulder dislocations, reduction of,
activated charcoal, 291, 307
allergi.c;vasc;ulitis, 163,178 131, 146
acute angle-closure glaucoma, 64, 79, 97,
allodynia, 2o-21 antJ."biotics, for submersion injuries, 17
113, 258, 274--275
alpha fetoprotein (AFP), 164, 179 anticholinergic crises, 7, 20, 135, 135, 152
acute chest syndrome, 162, 177
alternating c:urrent (AC), 45 vs. sympathomimetic crises, 5, 18
acute fatty liver of pregnancy (AFLP), 188,
alveolar osteitis, acute, 291-292,308 anticholinergic; poisoning, 169, 185
203 amaurosis fugax, 102, 119 antidysrhythmic;s, 64, 79-80
acute mesenteric ischemia (AMI), 126,
amebiasis, 138, 138, 155-156 anti-Helicobacter therapy, 11, 27
14o-141 amebic colitis, 194, 21o-211 antihypertensive overdose, 283, 297-298
acute mountain sickne&S (AMS), 5, 6, 17-18, amebic liver abscess, 138, 138, 155-156 antiplatelrt thempy, 163, 179
19, 197,215 Amerkan Academy of Family Physicians anxiety disorders, 163, 179
acute necrotizing ulcerative gingivitis (AAFP),46 aortic; dissection, 9, 11, 22, 25, 67, 84, 226,
(ANUG), 71, 87 Americ:an Academy of Pediatric:s (AAP), 46, 239-240
acute renal failure (ARF), 14, 29, 192,208, 61,76 Stanford class A, 162, 177
228,243 Amerkan Cancer Society, 127, 141-142 aortic regurgitation, acute, 158, 172
acute respiratory distres-s syndrome (ARDS), Amerkan dog tick (D. variabilis), 72, 88 aortic stenosis (AS), 286, 301
40, 56, 127, 141 Amerkan dog tick (Dermacenwr variabilis), aortic valve replac:ement, 158, 172
adenoid, 39, 55-56 63,78 aplastic; anemia, 256, 272
adenosine aminopeni.cill:i:n, 42, 42, 59 appendectomy, perfomtion at the time of;
serum half-lire of, 71, 87 amiodarone, for tachydysrhythmias, 17 160,175
for supraventricular tachycardias, 17 amitriptyline, for headaches, 158, 172 appendicitis
adenosine triphosphate (ATP), 62, 76 amniotic; fluid embolism (APE), 41, 58 acute, diagnosis of, 197, 215
adenovirus, 42, 59, 73, 89-90 amoxicillin, 42, 59 acute, missed, 164, 180
adenovirus pharyngitis, 42, 59 ampicillin in pregnant vs. nonpregnant women, 35,
adhesivecapsulins, 197,215,252,265 drug rash and, 42, 59 50, 128, 143, 180
adnexal cystic structures, 226, 240 for tubo-ovarian amc:es.s (TOA), 15 are8exia,16
adrenal insufficiency, 189, 204 amputations arrhythmia, 137, 154
adrenocorticotropic hormone (Acn:I), 54, high-pressure injection injuries to the arsenic, 170, 186-187
192,208 handand,170, 186 poisoning, ac:ute, 62, 76
adverse drug reactions (ADRs), 200, 218 replantation indicated for, 13, 28 arterial gas embolism (AGE), 199, 217
age-related macular degeneration, 226, 240 anaerobes, 36, 52 arterial occlusion, ac:ute, 223, 237
agitation, acute, 129, 145 anal fissures, 161, 176, 280, 292, 294, 308 arthralgi.as, 7, 20

311
312 Index

arthritis backpacker's diarrhea, 63, 78 brainstem lessions, 220, 233


Reiter's syndrome and, 72, 88 bou:t<ri.J cystitis, 164, 180 brain tumors, primary, 127, 141-142
rheumatoid, 43, 59 bacterial dysentery, 194,210--211 breast milk jaundice, 252, 266
&epti£,36,43,51,59, 193,209,230,246, bacterial meningitis, 5, 8, 18, 22, 189, 204 breech presentation, 165, 181, 254, 269
291, 308 bacterid pharyngitis, 193, 209-210 British antilewisite (BAL), 62, 76
ascites, 195,212 bacterial prostatitis, acute, 38, 53 bronchiolitis, 73, 89--90, 161, 175
Aspergjllus species, 33, 47 bacterial vaginosis (BV), 18, 36, 52, 104, 104, brown recluse spider bite, 200, 218
asphyxia, 8, 21 121 Brown-Sequard syndrome, 8, 22,286--287,
aspirated foreign bodies, 94,94--95, 109-110 bacteriuria, 126--127, 141 302
aspiration, potential, 229, 244 balanitis circinata, 10, 25 brow presentation, 165,181
aspiration pneumonia, 108, 125 barotrauma, 192, 207-208 Brodzinski's sign, 282, 297
aspiration pneumonitis, 9-10, 24 Bartholin's gland abscess, 3, 3, 16 bubo, 82
aspirin (acetyl-salicylic acid) Barton's fracture, 62, 78 bubonicplague,162,177
acid-base disorder caused by, 31, 45 basal cell carcinoma (BCC), 8, 21 bupropion, 12, 27
and antiplatelet therapy, for myocardial ,8-Block.ers, 16,284,298 bursitis, 2, 2, 16
infarction, 163, 179 Bell's palsy, 287-288,303-304
iron-de:fici.ency anemia and, 197, 214--215 benign early repolarization. 129, 145 C. botulinum, 7, 21
for myocardial ischemia, cocaine-induced, benzodiazepines, 19 c. difficile, l 00, 117
15 for anticholinergic aises, 7, 20 c. d;phtheriae, 191,206
overdose, 251,265 overdose, 6, 19, 231-232,248 C. jejuni, 29
transient ischemic attack (TIA) and, 136, for sedation of intubated patients, 10, 24 C. perfringens, 17,291, 307
154 for status epilepticus, 8-9, 22 C. pneumoniae, 24, 61, 76
aspirin-induced asthma (AlA), 190, 205 beta (in symbol) partick• 289, 305 c. tetrmi, 39, 55
asthma ,B-hexilev~,6, 13-14,19,29,132,148 c. tnu:homatis, 15, 30, 67, 83, 128, 136, 143,
acid-base disturbance in exacerbations, bicarbonate, for tricyclic antidepressant 153, 250, 264
288,304 overdoses, 16 cafe coronary syndrome, 201, 218-219
aspirin-induced (AlA), 190, 205 bilateral decerebrate posturing. 28 calcaneal fracture, 21
corticosteroids used in, 252-253, 267 bilateral facial nerve paresis, 292, 309 cal.d.um gluconate, 190, 205
critical ventilatory strategy for, 66, 82 bilevel positive airway pressure (BiPAP) calcium pyrophosphate diliydrate (CPPD)
cyclooxygenase-1 (COX-!) inhibition ventilation, 62, 77 crystal deposition disease, 222, 235
and, 161, 176 biliary tract malignancies, 169, 185 Campylobacter, 17, 103, 120
death in patients with, risk factors for, 100, binocular diplopia, 2, 15
cancer patients
117 bites, human, 68, 68, 85
acute spinal cord compression and, 229,
effective management of exacerbations black widow spider bites, 67, 83--84
244
and, 97,113 bladder rupture, 97, 112-113
hypercalcemia in, 96, 112
exercise-induced (EIA), 6, 20 blepharitis, 71, 72, 87-88, 254,254, 269
pancreatic, 131-132, 147
intrathoracic pressure during exacerbation blindness, 226, 240
Candida albicans, 137, 155
and, 232, 248 blockade, 287, 303
Candida species, 137, 155
ipratropium for, 70, 86 blood pressure
carbonic anhydrase inhibitor, 17-18
mechanical ventilation and, 192,207-208 intravenous medications for rapid control
carboxyhemoglobin, half-life of, 249-250,
postintubation hypotension and, 32, 46 of, 72, 88, 226, 240
labetalol and, 39, 55 263
refractory, epinephrine dosing and, 22
tissue plasminogen activator (tPA) and, cardiac arrhythmias, hyperkalemia and, 222,
X-rays for acute, 199,217
36,51 222-223, 236
asymptomatic bacteriuria, 287, 302
blood urea nitrogen (BUN) levels, 192, 208 cardiac contusion, 159, 173
atopic dermatitis, 94, 109
blunt trauma cardiac muscle, 291, 308
atraumatic hip pain in children, 285, 301
abdominal, spleen and, 104, 121 cardiac rhythm disturbances, in pediatric
atraumatic sudden death, 36, 51
cardiac injuries resulting from, 159, 173 patients, 131, 146
atrial flutter, 225, 239
of chest, in elderly patients, 96, 111-112 cardiogenic pulmonary edema, 32, 46
atrial thrombus, 225, 239
atrioventricular (AV) block in children, 6, 19 cardiomyopathy
first degree, 1, 15 diaphragmatic injuries and, 66, 83 dilated, 2, 15-16
Mobitz type I (Wenckebach flail chest, thoracic trauma and, 171, 187 hypertrophic, 16
phenomenon),3,4, 17 handlebar injuries and, 107, 125 cardiotocographic monitoring of pregnant
atrioventricular (AV) node, 167, 183 hypotension and, 43, 59--60 patients, 260, 277-278
atrioventricular nodal reentry (AVNRT), 40, mult:isystem, falls from buildings and, 7, carpal bone, broken, 164, 180
57 21 carpal tunnel syndrome (CTS), 10--11, 25,
atropine, for bradydysrhythmias, 17 parenchymal injury, thoracic trauma and, 252,266
"atypical" pneumonia, 61, 76 196, 213--214 caxpometacarp.J (CMC) joint, 19
auricular hematoma, 138, 138, 156 radiographic evaluation and, 36, 51-52 cauda equina syndrome, 8, 22
automated internal cardiac defibrillators renal injury in, 231, 247 cavernous sinus thrombosis, 39, 54-55,
(AICDs),45 scapular fractures, high -force blunt 166--167,167,182-183
"autonomous zone" of innervation, 131, 147 trauma and, 197, 216 cecal volvulus, 133, 149
avascular necrosis, 197,215 sternal fractures, thoracic injuries and, cellulitis, 226--227, 241
of the femoral head, 253, 253--254, 268 198,216 Centers for Disease Control (CDC), 99, 115
avulsed teeth, 66, 82-83 vascular injuries to the neck and, 62, 77 Centor criteria, 254, 268
avulsion fracture, 130,130, 145 body packers, 260, 277 central cord syndrome, 8, 21-22, 34,48--49
azithromycin. for pelvic inflamatory disease Boehler's angle, 288, 288, 304 central corneal abrasion, 196, 196,214
(PID), 18 Boerhaave's syndrome, 48, 136, 153 centralcyanosis,101, 101, 118
Borrelia burgdorferi, 12, 27 central retinal artery occlusion (CROA), 43,
B. burgdorfori, 5, 18-19 botulinum toxin, 7, 21, 221,234 59,170,187,260,277
B. cereus, 307 botulism, 7, 21 central retinal vein occlusion (CRVO), 43,
B. fragiU, 15 bracbi.J artery disruption, 73, 90 59,253,268
B. pertussis, 68, 85, 132, 135, 148, 152, 158, bradydysrhythmias, in Wenckebach central transtentorial herniation, 13, 28
172-173 phenomenon, 17 cephalexin, for community-acquired
babinski reliexe• 28 brain ischemia, 8, 21 pneumonia, 9, 24
Index 313

cephalosporin, for community-acquired community-acquired pneumonia (CAP), 9, dactylitis, 10, 25


pneumonia, 9, 24 24, 61, 75, 76, 92, 94, 109, 165, 181, "dashboard injury'', 199,199, 217
cerebellar hematomas, 13, 28 283,298 Datum stramonium. 135, 135, 152
cerebellar hemorrhage, acute, 34--35, 50 comparbnent syndrome, 73, 90, 230--231, daunorubicin, 15
cerebellotonsillar herniation, 13, 28 246--247,257,273 D-DIMER LEVELS, ELEVATED, 160, 174
cerebral contusions, 13, 28 complete airway obstruction, 63, 79 decompression sickness (DCS), 130, 146,
cerebral edema, 136, 137, 154, 155, 285, complete cord injury, 8, 22 198,216
300--301 complete knee instability, 137, 155 deep space neck infections, 130. 145
cerebral herniation syndromes, 13, 28 complex regional pain syndrome (CRPS), 7, delirium, 67, 83, 170, 185
cerebral perfusion pressure (CPP), 221, 234 20-21 acute, 163, 178
Cerebral venous thrombosis (CVT), 135, 151 concussions, 103, 120 tremens, 212
cerebrovascular accidents (CVA), 8-9, 22 condyloma acuminatum, 72, 89 delta wave, 57
cervical motion tenderness (CMT), 35, 50 congenital long QT syndromes, 205 deltoid ligament, rupture of, 13 7, 137, 154
cervical spine, oblique views of, 259--260, 277 congestive heart failure (CHF) dementia, 170, 185
cervical spine fractures, in elderly, 41,58 acute aortic regurgitation and, 158, 172 dental infections, 286, 302
Chagas disease, 15 cardiogenic shock and, 69, 69, 85 de<>xyhemoglobin, 224,237
chalcosis, 168, 184 exacerbation of, 127, 141 dexamethasone
hemodynamic decline and, 31,45 for acute mountain sickness (AMS), 17, 18
chancroid, 66, 81-82, 196, 214
mortality rates for, 285, 300 for meningitis, 18
chelation, 62, 76, 99, 116
multifucal atrial tachycardia (MAT) and. dextromethorphan, 175
chemical pneumonitis, 9--10,24
189,204 diabetic ketoacidosis (DKA), 11, 25, 232,
chemotherapeutic agents, 15 pleural effusions and, 39, 54
chemotherapy, tumor lysis syndrome and, 248, 285, 300--301
conjunctivitis, 42, 59 diabetic retinopathy, 71, 86--87
10,24 Reiter's syndrome and, 10, 25
chest pain, pediatric, 169, 185 diagnostic peritoneal lavage (DPL), 54, 62, 76
connective tissue disease, 15--16 dialysis, 194, 211
chest tubes, pregnancy and, 161-162, 162, consensual photophobia, 224, 238
176-177 for lithium toxicity, 16
continuous ambulatory peritoneal dialysis diaphoresis, 18, 75, 92
chest wounds (CAPO), 100, 117
gunshot,201, 201-202,219 diaphragmatic excursion during pregnancy,
continuous positive airway pressure (CPAP), 73,90
penetratiog, 197,215 62,77
chicken pox, 44, 44, 60, 134, 151 diaphragmatic injuries, 66, 83
contrast-induced nephrotoxicity, 261, 278 diarrhea, 103, 106, 120, 123,253,268,285,
child abuse "Cope's Early Diagnosis of the Acute
fractures found in, 188, 203 300
Abdomen", 180
pediatric deaths from, 166, 181-182 diffuse alveolar hemorrhage (DAH), 192, 207
copperheads,43,60
diffuse arthralgias, 160, 175
Chlamydia, 18, 39, 41, 55, 58, 61, 76, 128, coral snake envenomation, 249,249, 263
diffuse axonal injury (DAI), 126, 140
132,142,148-149 cord hemisection, 8, 22
diffuse esophageal spasm, 8, 22
chloramphenicol (CAM), 63, 78 core temperature afterdrop, 12, 27
diffuse maculopapular rash, 29
chocking episod<o, 94-95, 109-110 corneal abrasions, 260, 277
diffuse tall T waves, 168, 168, 184
cholangitis, acute, 256. 273 corneal ulcer, 190, 190, 204
Wgitalis, 255, 270
cholecystitis, acute, 256. 273 coronary artery disease, 11, 25
toxicity, 6. 20, 230, 245--246
cholelithiasis, 105, 122, 190, 205 coronary syndrome, acute, 75, 92
coronary vasospasm, 260,277
dihydroergotamine (DHE), 9, 23
cholinergic toxicity, 252, 265-266
corticosteroids dilated cardiomyopathy, 2, 15--16
chromosomal abnormalities, miscarriage dimercaprol, for chelation, 62, 76
and, 129, 144 for multiple sclerosis (MS), 159, 174
for poison ivy, 34, 49 diplopia, 2, 15, 53
chronic hemolysis, 99--100, 116 direct current (DC), 45
chronic obstructive pulmonary disease for submersion injuries, 17
coumadin, 257,273 direct modification, 25
(COPD), 5, 18 direct platelet inhibitors, 13, 28-29
acute exacerbation and, 62, 77 Coxsackie B, 11, 25
coxsacldevirus, 12, 27, 191,206 discriminatory zone, 132,148
multifocal atrial tachycardia (MAT) and, disseminated intravascular coagulation
189, 189, 204 cranial nerve palsy, 2, 15
creatine kinase-myocardial bound (CK-MB), (DIC), 32-33,47,230,246
noninvasive positive pressure ventilation distal fibula, oblique fracture of, 137, 137,
(NIPPY) ond, 32, 46 283,298
creatinine levels, 192, 208 154
polycythemia ond, 197,214-215 distal interphalangeal (DIP) joints, 13, 28,
Crohn•s disease, 131, 133, 147, 150,250, 264,
chronic peripheral vascular disease, 200, 218 165, 181,224,228, 238-239,
292,309
chronic radiation enteritis, 34, 49 243-244
"crossed signs", 221, 233, 252, 267
chronic renal failure, 168, 184 distal symmetric polyneuropathy, 71, 86-87
"crossed" straight kg nrlse (SLR) test, 161,
chronic tension-type headaches, 158, 172 diverticula, 74--75, 92
176
ciguatera fish poisoning. 200, 217-218 CRPS-IT,21 diverticulitis, 75, 92-93, 99, 115
cirrhosis, 195,212 cryptococcal meningitis, 28 diverticulosis, 41--42,58
clavicular fractures, 188, 190, 203--204, 205 Cryptococcus neofonnans, 34, 49 diving-related death, 199, 217
clindamycin, 100, 117 Cryptosporidium, 253, 268 domestic abuse, 11, 25--26
for community-acquired pneumonia, 9, Cryptosporidium parvum, 233, 237 domestic violence, 11, 25--26
24 cyanide, 170, 186-187 Done nomogram, 33, 48
for tubo-ovarian abscess (TOA), 15 cyanosis, 224, 229,237,244-245 doxorubicin, 15
cluster headaches, 32, 47, 62, 74, 76-77, cyanotic heart diseases in children, 13 7, 154 doxyqcline
91-92,172,220,233,284,299 cyclooxygenase-1 (COX-1) inhibition, 161, for community-acquired pneumonia,
cocaine 176 9,24
administration methods for, 188, 203 cyclosporine, 128, 143 for pelvic inftamatory disease (PID), 18
cardiac effects of, 15, 16 cyproheptadine, 193, 209 for pneumonia in children, 14, 30
coelenterate nematocyst toxin, 38, 53 cystic fibrosis (CF), 97, 113 drug rash, 42, 42, 59
Colles's fracture, 62, 77-78 cystitis, 161, 176,228,243 dry socket, 291-292,308
coloboma, 168, 184 cytoalbumin dissociation, 131, 147 duodenal dilation, 289, 289--290, 305--306
colon ischemia, 133, 149--150 cytochrome P-450 system, 18 duodenal ulcers, 11,26-27, 34, 49, 65, 81
common bile duct (CBD), 105, 122 cytotoxic edema, 136, 154 & wave, 195,212
314 Index

dysfunctional uterine bleeding (DUB), 256, erythromycin, for pneumonia in children, food poisoning, 106, 123, 291,307
271 14,30 forced hyperextension injuries, 34, 48-49
dl"Phagia, 8, 22 esophageal button batteries, 188, 203 foreign body aspiration, 63, 79, 226, 240--241
dyspnea, 168, 184 esophageal foreign bodies, removal of, 191, misdiagnosis of, 201, 218--219
dysrhythmias, 97, 113, 189, 193,204,209, 207 Fournier's gangrene, 12, 27
230,245--246 esophageal injuries, penetrating, 74, 92 frostbite, 40--41, 57, 74, 90--91
dysuria, 128, 142 esophageal irritation, oral medications and, frozen shoulder, 252, 265
106, 123 fungal balanoposthitis, 138, 156
E. coli, 15, 37, 39, 52, 55, 293, 310 esophageal motility disorder, 8, 22 Fusobacterium sp., 42-43, 59
E. coli 0157:H7, 106, 123 esophageal perforations, 136, 153
E. vermicularis (pinworm), 10,25 ethanol alcohol, 33, 47, 170, 186-187 gallbladder carcinoma, 169, 185
"e" antigen, 190, 205 ethylene giyco~ 194, 211 Gamekeeper's thumb, 34, 49
EBV infectious mononucleosis, 104, 120 ingestion of, 166, 182 Gardnerella, 36, 52
ecchymosis, 284, 284, 299 metabolism of, 100, 117-118,228,243 GAS phaxyogitis, 14, 29,284,299
eclampsia, 162, 177 overdose, 95, 110 gastric bubbles, 280, 294
ecstasy, 11, 21,26 exanthem subitum (sudden rash), 128--129, gastric dilation, 289, 289-290, 305--306
ectopic pregnancy, 6, 13-14, 18, 19, 29, 254, 144 gastric ul=s, 11, 26-27,34,49
260, 269, 277 exercise-induced asthma (EIA), 6, 20 gastric volvulus, 222, 235
edrophonium, 252, 265--266 exogenous glucocorticoid administration, gastritis, 67, 84
•eight-ball", 37, 37-38, 53 189,204 gastroen~tis,4,17, 103,120

..so% rule", 22s, 239 expiratory positive airway pressure (EPAP), in pediatric patients, 193, 209
EKG, right-sided, 7, 21 62,77 gastroesophageal •eflux diseaoe (GERD), 41,
elapidaevenom, 231,247 extraintestinal manifestations, 250, 264 57,255,270-271
elbow injury, 197, 215 gentamicin, for tubo-ovarian abscess (TOA),
F. tularensis, 199,216-217 15
intraarticular, 14, 29
Fab, 20 giant cell arteritis, 63, 78
elbow trauma, 74, 92
elder abuse, 130, 145
face presentation, 165, 181 Giardia lamblia, 63, 78
electrical flow, resistance to, 11, 26
facial nerve paralysis, 74, 91 GI bleeding mclrildren, 166, 182
factitious gastrointestinal bleeding, 73, 90 gingival hyperplasia, 258, 274
electrical injuries, 31,45
factor II (inactivate thrombin) inlu'bitor, 13, glaucoma, 43, 59
elevated anion gap metabolic acidosis, 167,
28 acute angle closure, 64, 79, 97, 113
183
factor XIII inhibitor, 13, 28, 29 acute angle-closure, 258,274-275
elevated intracranial pressure (ICP), 136, 153
facto,. X. inhilrito•, 28 globe injuries, 65, 81, 101, 118
Ellis system, 164, 180 fall on outstretched hand (FOOSH) injuries, penetrating, 227, 227, 242
emboli, 31,45
126, 140 rupture, 258,274
emergent laparotomy, 38--39, 54
falls glomerulonephritis, 192, 196, 207,213,
empyema. 188--189,204 from bWldings, multisystem blunt trauma 215-216
encephaliti$,36,51, 191,206 and, 7,21 glucagon, for P-blocker overdoses, 16
encephalopathy, 32, 46 elderly and, 228, 243 glucocorticoids, 134, 150--151
hypertensive, 98, 115
endocarditis, 167, 183, 283,297 familial'""""· 15-16
fat embolism, 231, 247
glutathione, 18
giyoxylic acid, 228, 243
prophy!Ws, 292, 308 feeding, resuming after illness, 94, 109 gonococcal conjunctivitis, 166, 182
endometritis, 162-163, 178 femoral neck fractures, 127-128,142 gonococcus,12,18,27,39,55, 128,130,142,
end-stage renal disease (ESRD), 168, 184 fetal demise, evidence of, 107, 124 146
entactogens, 11, 26 fetal malpresentation, 167, 183 Goodpasture•s syndrome, 192,207
entamoeba, 194, 210--211 fetor hepaticus, 187 gout, 286, 302
enteric parasite infection, 63, 78 FEV1>35,51 in women, 168-169, 184
enterohemorrhagicE. coli (ETEC), 106,123 fever, in infants, 258--259, 275 gram-negative en~c rods, 128, 142
enterohemorrhagic Escherichia coli (EHEC), fibrinogen levels, 161, 176 gram-positive cocci, 37,52
106, 123 fifth disease, 292, 292, 308 gnmuk>ma inguinak, 66, 82
Enterotoxigenic E. coli (ETEC), 193, 194, fueanns gross hematuria, 97, 112-113
209,210-211 handeling, 192, 208 group A beta-hemolytic streptococcal throat
enteroviruses, 191,206 homicide due to, 164, 180 infectioo (GAS phaxyogitis), 256,
envenomations, 13, 29, 43, 60, 67, 83-84, first-trimester spontaneous miscarriage, 129, 271-272
200,218,231,247 144 group A P-hemolytic streptococci, 36, 51
eosinophilia, 25 first-trimester vaginal bleeding in group A streptococcal pharyngitis, 31, 45
epididymitis, acute, 39, 55, 103, 120 Rh-negative pregnant patients, 35, 50 group B streptococcus, 8, 22
epidural hematomas, 13, 28, 62, 62, 76, 197, Fitz-Hugh-Curtis syndrome (perihepatitis), Guillain-Barr'e syndrome (GBS), 3, 13, 16,
215 15 29, 131, 147
epiglottitis, 95, 95, 110--111, 192, 208, 249, flail chest, 171,187 gunshot wound of the chest, 201, 201-202,
263 flex1'ble endoscopy, 191,207 219
epinephrine flexor tenosynovitis, 67, 84
administration of, 38, 54 floaters, 287, 302 H. capsulatum, 135, 152
in anaphylaxis, 161, 176 fluid administration to trauma patients, H. ducreyj. 196,214
dosing for, 8, 22 initial, 136, 153, 169, 185 H. infiuenzae, 8, 22, 37, 51,52
epistaxis, 98, 115 flumazenil, 6, 19 H. pylori, 11, 26-27,67, 84,95-96, 111
Epstein-Barr virus (EBV) phaxyogitis,193, fluoroquinolone, for community-acquired halitosis, 292, 308
209-210, 256, 272 pneumonia, 9, 24 haloperidol
erythema, 40--41, 41, 57 fluoxetine, 193, 209 for acute delirium, 163, 178
erythema in:fectiosum, 292, 292, 308 focused assessment of sonography in trauma for rapid sedation and control of psychotic
erythema marginatum, 29 (FAST) scan, 66, 83 behavior, 194, 211
erythema migrans, 19 Foley catheter placement, urethral injury Hamman's sign or crunch, 48, 134, 134, 151
erythema multiforme (EM) minor, 285, and,200,201,218 hand injuries, high- pressure injection, 170,
285--286, 301 follicular cysts, 226, 240 186
erythema nodosum (EN), 64, 64, 80 food allergies, 40, 56, 131, 147 handlebar injuries, 107, 125
Index 315

"hard" signs of vascular injury, 96, 112 hip dislocations, posterior, 293, 309-310 hyponatremia, 38, 39, 54, 55, 107, 124, 192,
headaches. See individual listings hip fractures, 127-128, 142 208
head trauma Hippocratic method, 146 hypophosphatemia, causes of, 103, 120
as cause of pediatric deaths, 99, 115--116, Hirschspnmg'' disease, 64, 79 hypore.tlexi.a, 16
198, 216 hiro&n, 191, 207 hypotension
cerebral contusion, in elderly, 13, 28 mY-encephalopathy, 28 hemorrhagic shock in pediatric patients
cerebral edema and, 136, 154 ''holiday heart syndrome.., 61, 61, 76 and, 43, 59--60
hypotensive adults and, 224, 238 homicide due to firearms, 164, 180 nitroglycerin for preload and, 13, 28
intracranial hypertension after, 7, 20 hordeolum, 71, 72, 87-88 orthostatic, 39, 55
mannitol for, 191, 207 Homer's syndrome, 43, 59 pelvic fracture and, 107, 125
seizures and, 167, 183 ''huffing'', 7-8, 21 postintubation, 46
hearing loss, 61, 76 human bites, 68, 68, 85 septic shock and, 191-192,207
heart transplant patients, 127, 142 human herpesvirus 6, 128-129, 144 hypothermia, 41, 58, 229, 232, 244, 248
heat exhaustion, 100, 118 human immunodeficiency virus hypothyroid cardiac disease, 189, 204
heat stress, 98, 114-115 (HIV)/AIDS patients hypothyroidism, 232, 248
heat stroke, 97, 100, 113, 118, 133, 149 Cryptococcus neoformans, 34, 49 hypovolemia, 131-132, 147-148
HELLP syndrome (hemolysis, elevated liver Crypt:usporidium diarrhea and, 233, 237, hypoxemia, 5, 18
enzymes, low platelets), 161, 176, 253,268
188, 203 HIV -encephalopathy and, 28 ibuprofen overdose, 226, 240
hematemesis, 70, 86 pneumocysbS carinii pneumonia (PCP) idiopathic intracranial hypertension, 74, 91
hematogenous, 15, 47 and, 159-160, 160, 174 idiopathic thrombocytopenic purpura (ITP),
hematuria, painless, 196,213, 215--216 66, 68, 68, 82, 85
prophylactic TMP-SMX then>py and, 99,
hemi-inattention (neglect), 169, 184 ileocecal valve perforation, 37, 52
115
hemodynamically unstable patients, 38-39, illicit drugs, ingestion of, 260, 277
toxoplasmosis and, 13, 28
54 IM ceftriaxone plus
human papillom.avirus (HPV), 72, 89
hemodynamic decline, 31, 45 doxycycline/azithromycin
human rabies immune globulin (HRIG), 41,
hemolysi, 161, 176 for pelvic inflamatory disease (PID), 18
43, 57,60
hemolytic anemia, 161, 176 hnpetigo,198, 198-199,216
humer.ol head fracture, 197,215
hemolytic disorders, 100, 117 inactivate thrombin (factor II) inln"bitor, 13,
humer.ol shafr fracture~ 133, 133-134, ISO
hemolytic-uremic syndrome (HUS), 37, 28
humerus fractures, 197,215
52-53,123 incarcerated hernia, 38, 54
hydatidiform moles, 137, 155
hemopericardium, 9, 22 incomplete cord injury, 286-287, 302
hydrocarbons, 170, 186-187
hemophilia, 220, 233 increased intracranial pressure (ICP), 74, 91
tcuicity, 7-8,21
hemoptysis, 63--64, 79 infectious esophagitis, 137, 155
hymenoptera, 56
in children, 9, 23 inferolateral knee pain and tenderness,
hyperacute bacterial conjunctivitis, 136,
hemorrhagic cerebellar infarction, 221, 31,45
152-153
234--235 infertility, 18
hypercalcemia, 96, 112, 159, 174,230,245
hemorrhagic shock inllammatory bowel disease (IBD), 292, 309
hypercalcemic crisis, 285, 300
in pediatric patients, 43, 59--60 induenza, 73, 89-90
hyperchloremic metabolic acidosis (HCMA),
stage IV, 289, 305 initial intravenous resuscitation fluid of
126,140
hemorrhoids, acutely thrombosed, 129, 144 choice in pediatric patients, 253, 267
hyperemesis gravidarum, 221,234 inner cranial pressure (ICP), cerebral
hemotympanum, 287, 287, 303
hyp~.40-41,57 herniation syndromes and, 13, 28
Henoch-SchOnlein purpura (HSP), 68, 68,
hyperextension injuries, forced, 34, 48--49 in situ arterial thrombosis, acute, 200, 218
85,126,132,132-133,140,149,192,
208,231,231,247 hyperfle:xion injuries, 251,265 intermediate pretest probability, 136,
heparins, low molecular weight (LMWHs), hyperglycemic hyperosmolar nonketotic 153--154
13,28-29 coma (HHNC or HHS), 72, 88--89 internuclear ophthalmoplegia. 53
hepatic encephalopathy (HE), 46 hyperkalemia, 10, 24, 33, 47, 48, 97, 113, 166, intertrochanteric fractures, 127-128,142
hepatib3,133,135, 136,150,152,153,190, 168, 168, 181, 184, 192,208,222, intestinal ischemic malady, 133, 149--150
205 222-223, 236 intimate partner abuse (IPA), 11, 25--26
hepatibS Be antigen (HBeAg), 190,205 hypermagnesemia, 33, 48, 231,247 intra-abdominal hemorrhage, 38-39, 54
hepatocellular carcinoma, 164, 179 hypernatrem:ia, 229-230, 245 intraarticular elbow injury, 14, 29
hepatocyte necrosis, 18 hyperosmolar hyperglycemic syndrome or intracranial hemorrhage, 21
hereditary spherocytosis, 100, 117 stste (HHS), 232, 248 intracranial hypertension after head trauma,
herniated disk, 161, 176 hyperparathyroidism, 230, 245 7,20
herp~129,144 hyperphosphatemia, 10, 24, 33, 48, 170, 186 intracranial pressure (ICP), 221, 234
herpes simplex encephalibS (HSE), 36, 51, hyperpigmentation, 38, 54 intraepidermal acantholysis, 25
228,242-243 hypersensitivity, 163, 178 intraoral lacerations, 160,175
herpes simplex keratitis, 41, 57 hypertension intrapartum asphyxia. 8, 21
herpes simplex virus type 2 (HSV-2), 65--66, aortic dissection and, 67, 84 intraphalangeal (IP) joints, 291, 307
66,66,8!-82,!9!,206,280,294 elevated intracranial pressure (ICP) and, intrauterine pregnancy, 13-14,29
herpes zoster, 44, 44, 60, 101, 101-102, 118 136, 153 intravenous drug users (IVDUs), 283, 292,
herpes zoster ophthalmicus, 293, 310 tFA administration and, 6-7,20 297' 298, 308
heterotopic pregnancy, 227,241 hypertensive encephalopathy, 98, 115 intravenous saline rehydration, for lithium
HiB vaccine, 8, 22 hypertonic saline, for hyponatremia, 39, 55 toxicity, 16
high-altitude =ebellar ataxia (HACE), 197, hypertrophic cardiomyopathy, 16, 36, 51, intubated patients, inadequate sedation of,
215 282,296 10,24
high-altitude pulmonacy edema (HAPE), 18, hyperuricemia, 10, 24 intussusception, 254, 259, 269, 275
192, 209 hyperviscosity syndrome, 64, 80 pediatric, 292, 308
high altitude-related death, 192, 209 hyphema, 3, 16 ipratropium, 70, 86
highly active antiretrovrral then>py hypocalcemia, 10, 24, 95, 110, 170, 186 ipsilateral otitis media, 74, 91
(HAART), 233, 237 hypoglycemia, 38, 54, 192, 208 ipsilateral ptosis, 28
high-pressure injection hand injuries, 170, hypokalemia, 33, 38, 47, 48, 54 Dido~ysis. 168,184
186 lithium toxicity and, 16 bidodonesis,l68,184
high-probability V/Q scan, 136, 153-154 hypomagnesemia, 67, 84 Uib3,224,237-238
316 Index

iron-deficiency anemia, 197,214--215 ligamentous injuries, 195, 213,259,276 management of, 106, 123
iron toxicity, 74,91 liquefaction necrosis, 199, 217 otitis media and, 193, 210
irritable bowel syndrome (IBS), 105, 122, liquid drain cleaner, 291, 307 preseptal cellulitis and, 55
133, 150 Lisfranc fracture dislocation, 280, 281, Meningococcemia, 192, 208, 256, 272
ischemia of anterior circulation. 195, 211 294--295 meningococcus, 12, 27
ischemic colitis, 133, 149-150 Listeria, 8, 22, 196, 213 meningoencephalitis,191,206
ischemic heart disease lithium toxicity, 3, 16 meniscal injuries, 6, 19
antihypertensive agents to use in, 3, 16 liver disease, in pregnancy, 188, 203 meperidine, serotonin syndrome and, 160,
cocaine-induced chest pain and, 15 liver failure, acute, 136, 153 175
mortality due to, 4, 17 liver transplantation. criteria for, 107, 124 mesenteric ischemia, 31, 45, 194, 211
ischemic stroke, 36, 51, 137, 155,249,263 liver trauma in children, blunt, 6, 19 metabolic acidosis, 33, 48
isoniazid (INH), 163, 179 "LOAF" muscles, 10--11, 25 metacarpal neck fractures, 6, 19-20
isopropanol. 33, 47, 170, 186-187,225, 239, locking or clicking of the knee, 6, 19 metacarpophalangeal (MCP) joint
280,294 lower gastrointestinal bleeding (LGIB), amputation of, 13, 28, 290, 307
41-42,58 methanol poisoning, 66, 82, 100, 117, 161,
jammed finger, 165,165, 181 lower urinuy tract infuction (UTI) (cymtis 176
Jarisch-Herxheimer reaction, 285, 300 or urethritis), 161, 176, 228,243 methemoglobinemia, 229, 244--245
jellyfish stings, 38, 53 low molecular weight heporins (LMWHs), methicillin-resistant Staphyloooccus aureus
Jervell-Lange-Nielsen syndrome, 205 13, 2S-29 (MRSA), 281,295-296
jimsonweed, 135, 135, 152 LSD (lysergic acid diethylamine), 11, 21, 26, methylenediox:ymethamphetamine
joint pathology, 226, 240 96, 112 (MDMA), 11, 21, 26
Jones criteria, 17, 29, 160, 175, 185 Ludwig's angina, 100, 116-117,261,279, methylprednisolone, for multiple sclerosis
Judetview, 287, 303 286,302 (MS), 159, 174
lumbar puncture, 260, 277 methyl salicylate, 170, 186-187
K. pneumonia, 165, 181 for meningitis, 18 metoprolol, 15
keratoderma blenorrhagica, 10, 25 lung abscesses, 163, 178,229, 245 metronidazole, for pinworm, 25
keraunoparal}'5is, 162, 177-178 Iyme di....., 5, 18-19 microcytic anemia, 34, 49-50
Kernig's sign, 282, 297 lymphatic disruption. 230, 245 microcytosis, 34, 49-50
kidney stones, 95, 110, 131, 132, 146, 148, lymphedema, 230, 245 middle ear barotrauma, 132, 148
165, 181,252, 260--261, 266,278 lymphogranuloma venereum (LGV), 66, 82 "middle ear squeeze", 132, 148
Kiesselbach plexus, 129, 144 lysergic acid diethylamine (LSD), 11, 21, 26, midgut volvulus, 289, 289-290, 305-306
King's College criteria, 107, 124 96, 112 midstream clean catch specimen. 259, 276
Kleihauer-Betke test (KBT), 73, 89 lytic lesion, 254, 269-270 migraine headaches, 32, 47
knees migraine prophylaxis, 158, 172
complete instability of, 137, 155 M. catarrhalis, 37, 52 migratory arthritis, 29
dislocation of, 137, 155 M. pneumrmiae, 30, 61, 76, 94, 109 mineralocorticoid deficiency, 192, 208
locking or clicking of, 6, 19 M. tuberculosis, 195,212 mitral stenosis (MS), 74, 91,229, 245
pain and tenderness in, 31, 45 macrolide therapy, 41,58 mitral valve prolapse (MVP), 43, 60
twisting injuries of, 261, 278--279 for community-acquired pneumonia Mobitz type II second degree atrioventricular
Kocher method, 146 (CAP), 9, 24 (AV) block, 70, 70, 86
Koplik's spots, 9, 24 macular degeneration. 43, 59 moderate traumatic pneumothoraces, 259,
maculopapular rash, 63, 78 276
L. monocytogenes, 196,213 magnesium, 190, 205,256, 272 monoamine oxidase (MAO) inhibitor, 11,
L. pneumophila, 75, 92 deficiency, 67, 84 26, 160, 175
labetalol, 39, 55 for eclampsia seizures, 162, 177 monoarticular arthritis, 194,211
lacate dehydrogenase (LDH) levds, 10,24-25 excretion,231,247 monocular diplopia, 15
LACE (mnemonic for obstructive airway Maisonneuve fracture, 134--135, 151 mononeuropathy of cranial nerve III, 293,
disease), 168, 183-184 major depression, 137, 154--155, 288, 304 310
LAD distribution, 197, 197, 216 malaria, 96, 111 mononucleosis, 42, 42, 59
Lambert-Eaton myasthenic syndrome Malasseziafurfur,134, 150 EBV infectious, 104, 120
(LEMS), 255, 270 malignant neoplasms, 128, 142 Monteggia's fracture, 254, 269
laparotomy, 291, 308 mallet finger, 165, 165, 181,228, 243-244 mood disorders, 179
large bowel obstruction, 128, 142 malrotation. 289, 289-290, 305--306 morbilliform rash, 42, 42, 59
..late decelerations" of the fetal heart mte, mannitol, for severe head injury, 191, 207 Morison's pouch, 201, 219
102-103, 103, 119--120 Mantoux test, 195,212 motor vehicle collisions (MVCs),
lateral or fibular collateral ligament complex, Marfan's syndrome, 11, 25, 67, 84, 99, 116 management steps for, 232, 248
227,241 marginal modification, 25 mucormycosis, 9, 23
lead toxicity, 99, 116 massive hemoptysis, 258, 275 multifocal atrial tachy=dia (MAT), 40, 57,
leeches, 191, 207 mastoiditis, 107, 124, 191, 191,206 189, 189, 204
left bundle branch block, 193,209 matemalfetal hemorrhage (MPH), 73, 89 multiple myeloma, 64, 80
"left shift", 36, 52 MDMA (methylenedioxymetham- multiple sclerosis (MS), 38, 53, 159, 168, 174,
left ventricular hypertrophy, 129, 145 phetamine), 11, 21,26 184,200,218
Legg-Calve-Perthes (LCP) disease, 253, mean arterial pressure (MAP), 221, 234 Munchausen's syndrome by proxy, 202, 219
253-254, 268 mebendazole, for pinworm, 25 murmurs, 167, 183
~n.U.,41,58,61,76 Meckel's diverticulum, 166, 182 mushroom poisonings, 170, 186
lenticular dislocation. 99, 116 medial malleolar fracture, 134--135, 151 myasthenia gravis (MG), 257, 273
"let-go current", 45 median nerve injuries, 10--11,25, 62, 73, Mycobacterium tuberculosis, 139, 156--157,
leukocytoclastic vasculitis, 163, 178 77-78,90 195,212
leukocytosis, 36, 52 meibomian gland, 71, 72, 87--88 Mycop"""'a, 36, 38, 41, 52, 53, 58, 61,76
leukopenia, 170, 187 m~omas,160,174 Mycoplasma pneumoniae, 24
levofl.oxacin, for pneumonia in children, 14, meningitis mydriasis, 28
30 bacterial, 5, 8, 18, 22, 189, 204, 282, 297 myocanlial infiorction (MI)
Lhermitte's sign, 53 cryptococcal, 28 antiplatelet therapy with aspirin and, 163,
Hdocame, 64. 79-80 CT scan for, 18 179
lie, 233, 236-237 fungal, 34, 49 left bundie b=ch block and, 193, 209
Index 317

myoglobin for, 38, 53-54 normal anion gap metabolic acidosis, 106, paroxysmal arteriovenous nodal reentrant
posterior wall, 202, 219 123 tachycardia (AVNRT), 193, 210, 286,
sequence ofEKG morphologies in, 188, normal pressure hydrocephalus, 285, 301 301
203 normocytic anemia, 256, 272 paroxysmal supraventricular tachycardia
ST segment elevation (STEMI), 69, 69, 85 noroviruses, 233, 237 (PSVT), 131, 146
symptoms o~ in elderly patients, 35, SO, 51 Norwalk. virus, 233,237 partial intestinal obstruction, 64, 79
without chest pain, 251,265 nosebleeds partial prothrombin time (PT), 161,176
myocardial ischemia (MI), 251,251, 252, anterior, 129, 144 parvovirus, 44, 60
265,266 posterior, 132, 149 Pastia's lines, 29
myocarditis, 11, 25, 255, 270 nosocomial pathogens, 163, 178-179 patellar fractures, 35, 50
tube~ous,15-16 nursemaid's elbow, 71, 87 patent foramen ovale, 198, 216
vrrai, 15 nutcracker esophagus, 8, 22 patient odor, 34, 49,170,186-187
myoglobin, for myocardial infarction (MI), PCO:l, range for hyperventilation and, 20
38,53-54 oblique fracture of the distal fibula, 137, 137, PCP (phencyclidine) intoxication, 7, 11, 21,
myringitis, 38, 53 !54 24-25,26,223,236
octreotide, 43, 60 peak expiratory flow rate (PEFR), 35, 51
N. gonorrhoeae, 15, 36, 51,69-70, 86, 136, ocular foreign body, 130, 146 pediatric intussusception, 292, 308
153 ocular HSV, 41, 57 pediatric long bone fractures, 282, 296
N. gonorrhoeae proctitis, 250, 264 ocular trachoma, 67, 83 pediatrics
N. meningitidis, 196,213 oculogyric crisis, 221,234 airway, 134, 150
N -acetylcysteine (NAC) bacteremia, 66, 82
oculomotor dysfunction. 28
for acetaminophen toxicity, 18, 23 chest pain, 169, 185
oculomotor nerve injuries, 13, 28
acetaminophen toxicity and, 36, 51 deaths
odontoid fractures, 41,58
N-acetyl-p-benzoquinone imine (NAPQI), common causes of, 94, 99, 109,
odynophagia, 31,45
18 115-116
olecranon bursitis, 2, 2, 16
Na-K ATPase, 255,270 fron child abuse, 166, 181-182
opacification of the lung, 201, 20 1-202, 219
naloxone, 33, 47 head injuries as cause of, 99,
ophthalmoplegia, 33, 47
NAPQI,9,23 115-117,198,216
opiate overdose, 33, 47
nasogastric tubes (NGTs), 94, 95, 109, 110 pelvic fractures, 287, 302-303
opiods, for QRS prolongation, 1, 15
nasopharyngoscopy (NP) is, 95, 110--111 hypotension and, 107, 125
optic neuritis, 38,53,168,184
nausea and vomiting in early pregnancy, 160, open, 11, 26
Optic Neuritis Treatment Trial, 174
175 open-book, 166-167, 167, 182
oral burns, 289, 305
neck pelvic inflammatory disease (PID), 12,27,
oral candidiasis, 281, 295
fractures 166,182,254,269
oral medications, esophageal irritation and,
femoral, 127-128, 142 risk factors for, 5, 18
106, 123
metacarpal, 6, 19-20 tuboovarian abscess (TOA) and, 1-2, 15
oral rehydxation therapy (ORT), 193, 209
infections pelvic pain, chronic, 18
orbital cellulitis, 33, 39, 47, 54, 169,169, 185
deep space, 130, 145 pemphigus vulgaris, 25, 69, 85--86
orbital floor fracture, 63, 63, 78
injuries, 62, 77 penetrating abdominal trauma, 137, 155
organophosphates. 158, 172
zone II, 101, 118 penicillin, 284, 299
oropharyngeal dysphagia, 200, 218
necrotizing enterocolitis (NEC), 106, 123 peptic ulcer disease (PUD), 11,26-27,34,49,
necrotizing fasciitis, 4, 17 oropharyngeal flora, aspiration of, 163, 178
95-96,111,251,264-265
of the perineal area, 12,27 orthostatic hypotension, 39, 55
perforation, 37, 37, 52
necrotizing (malignant) otitis externa, 128, osmotic demyelination syndrome (ODS),
appendectomy and, 160,175
290,306
143 peptic ul=, 71, 87
neglect (hemi-inattention), 169, 184 osteomyelitis, 254, 269-270
spontaneous, 136, 153
Neisseria meningitidi.s, 256, 272 otitis externa, 67, 83 pericardia! effusions, 100, 116
Neisseria species, 136, 152-153 otitis media pericardia! tamponade, 280, 294
neonatal conjunctivitis, 69-70, 86 acute, common causes of, 133, 150 pericarditis, 129,144-145,197,197,216
neonatal hypoglycemia, 196, 214 acute mastoiditis and, 107, 124 perihepatitis (Fitz-Hugh-Curtis syndrome),
neoplasms, malignant, 128, 142 causes of, 63, 79 15
nephrotic syndrome, 194, 196, 210,213, complications of, 61, 76 perilunate dislocation. 129, 144
215-216 intracranial complications of, 193, 210 perimortem cesarean section, 253, 268
neuroblastoma, 228, 242 populations with lowest rate of, 231,247 periorbital cellulitis, 33, 47
neurocysticercosis, 289, 305 risk factors for developing, 231, 247 periorbital edema, 169,169, 185, 194, 210
neurogenic intermittent claudication, 27 treatments for, 31-32, 45--46 periorbital (preseptal) cellulitis, 37, 52
neuroleptic malignant syndrome, 71, 87 Ottawa Ankle Rules (OAR), 281-282, 296 peripartum cardiomyopathy, 11, 26
neurologic dysfunction. 100, 118 ovarian torsion, 292, 309 peripheral nerve testing, 131, 147
neuromuscular disease, 32, 46 peritonitis, 15
neutropenic fever, 170, 187, 230, 246 P. aerugi.nosa, 163, 178-179 peritonsillar abscesses (PTAs), 42--43, 56, 59,
new-onset psychosis, 32, 46 painless lymphadenopathy, 195,212 106-107,123--124,226,241
nicardipine, 72, 88 palatine tonsils, 39, 55-56 permethrin, for scabies, 166, 182
nifedipine, 18 palpable porpura, 163, 178 pertussis, uncomplicated, 284, 299
Nikolsky's sign, 10, 25, 69, 85-86 pancarditis, 29 peteebUUlerions,192,208
nitroglycerin, 281,295 pancreatic cancer, 131-132, 147 Peyer's patches, 292, 308
for preload, 13, 28 pancreatitis, 96, 96, 104, 111, 120. 222, 235 pharyngeal infection, 160, 175
nitroprusside, 68, 84--85, 197,215 parainfLuenza, 73, 89-90, 173 pharyngealtonsil,39,55-56
nocturnal symptoms, 133, 150 paralytics, for sedation of intubated patients, pharyngitis, 42, 42, 59, 254, 268
nongonococcal urethritis, 132, 148-149 10,24 streptococcal, 31,45
nonintentional blunt thoracic injury, 4, 17 parapharyngeal abscess, 130, 145 pharyngoconjunctival fever, 42, 59
noninvasive positive pressure ventilation paraphimosis, 262, 262, 279 phencyclidine (PCP) intoxication, 7, 11, 21,
(NlPPV), 32, 46, 161, 176 parapneumonic effusion, 188-189, 189, 204 24-25,26,223,236
nonsteroidal anti-inflammatory drugs parenchymal injury, 196, 213 phenelzine, for depression, 160, 175
(NSA!Ds), 161, 176 paresthesias, 197, 214-215 phenytoin, 64, 79-80
norepinephrine reuptake inhibitor, 9, 23 parietal lobe, infarction of, 169, 184 pheochromocytoma, 255-256, 271
318 Index

phlegmasia cerulea dolens, 193--194,194,210 preseptal cellulitis, 39, 54-55 renal stones, 286, 302
physostigmine, 3 7. 52 preterm premature mpture of membranes replantation, indications for, 13, 28
pill esophagitis, 107, 124-125 (PPROM), 191, 205-206,281, 295 respiratory syncytial virus (RSV), 63, 73, 79,
pilon fractures, 98, 114 prevertebral space, ssessing on X-rays, 95, 89-90,159,161,173,175
pilonidal sinuses, obstruction of, 33, 48 llO retinal detachments, 71,86--87, 87, 287, 302
pinworm (E. vermicularis), 10, 25 primary adrenal insufficiency, 38, 54, 192, retinal hemorrhage, 129,144
piperacillin-tazobact:.am, for 208 retrobulbar hemorrhage, 250, 263--264
community-acquired pneumonia, 9, primary spontaneous pneumothorax, 33--34, retroperitoneal hematoma, 2, 15
24 48,63-64,79,170,186 retroperitoneal injuries, 21
pityriasis rosea, 288, 288, 304 Prinzmetal's (or variant) angina, 163, 179 retropharyngeal abscesses (RPAs), 159, 174
Pityrosporum obiculare, 134, 150 propanolol, 289, 305 rhahdomyolysis. 74, 91, 170, 186
Pityrosporum ovale, 134, 150 prophylactic TMP-SMX therapy, 99, 115 rheumatic fever, 4, 17, 160, 169, 175, 185,
placental abruption, 32-33, 47 propoxyphene, QRS prolongation and, 15, 288,304
placenta previa, 32-33,47, 73,90 40,56 rheumatic heart disease, 74, 91, 288, 304
Plasmodium parasite, 96, 111 proptosi• 33, 47 rheumatoid arthritis, 20, 43, 59
platy>ma, 41,58 prosthetic heart valves rhinocerebral mucormycosis (RCM), 9, 23
pleural effusions, 39, 54 chronic hemolysis and, 99-100, 116 rhinovirus, 73, 89-90
pleuritic chest pain, 163, 179 endocarditis of, 104, 120 Rh-negative pregnant patients,
Pneumococcus, 24, 66, 82, 159, 173 endocarditis prophylaxis with antibiotics first-trimester vaginal bleeding in, 35,
pneumocystis, 174 for patients with, 221, 234 50
Pneumocystis carinii, 10, 24-25 proteinuria, 194, 210 RhoGAM, 35, 50
Pneu.mocystis carinii pneumonia (PCP), 99, prothromhin time (PT), 161, 176 rib fractures, 39, 55
ll5, 159-160,160, 174,258, 274 proximal fibular fracture, 134-135, 151 in children, multiple, 4, 17
pneumomediastinum, 48 proximal interphalaogeal (PIP) joint fl=d, "Richmond" study, 22
pneumonia 19-20 Rickettsia rickettsii, 63, 78, 230, 246
aspiration, 108, 125 pruritus ani, 226, 241 right-slded EKG, 7, 21
"atypical", 61, 76 pseudocyst, 96, 96, 111 right-to-left shunting, 137, 154
bacterial cause of, 159, 173 pseu.dogout, 222, 235 right ventricular infarction, 7, 21
as cause of death in long-term care Pseudomonas, 189, 204 ring-enhancing lesion, 290, 306-307
&cilities, 98, 114 pseu.dosubluxation, 40, 56 ringworm, "black dot". 48
in children, 14, 30, 94, 109 pseu.dotumorcerebri, 74,91 Ritter's disease, 65, 81
community-acquired (CAP), 9, 24, 61, 75, psychedelics, 96, ll2 R<>cky Mountain spotted fever (RMSF), 12,
76, 92, 94, 109, 158, 165, 172, 181, psychiatric disorders, 163, 179 27,63,72,78,88,230,246
283,298 psychiatric emergencies, 223, 236 Rocky Mountain wood tick (D. andersoni),
complicated parapneumonic effusion and, psychogenic seizures, 98, 113-114 63, 72, 78, 88
188-189, 204 puerperal infection, 162-163,178 Romano-Ward syndrome, 205
Pneumocystis carinii (PCP), 99, 115, 258, pulmonary barotraumas, 199,217 roseola infantum, 128-129, 144
274 pulmonary contusion, 196, 213, 290, 306 rotator cuff, 287,303
S. pneumoniae, 30 pulmonary embolism (PE), 137,154,174, rotavirus, 65, 81--82, 103, 120
"walking", 61, 76 227,241-242,252,259,267,276 mbella, 192, 208, 287, 303
World Health Orgaoizatioo's (WHO) dyspnea and, 163, 179 mbeola, 192, 208
clinical criteria for the diagnosis of, pulmonaryfungalinfi:ctioo, 135,152 Rumack-Matthew nomogram, 48
172 pulsus paradoxus (PP), 221-222,235 rust~,230,245
Pneumonia Severity Index (PSI), 261, 278 purified protein derivative (PPD), 195,212
pneumothorax, 197,215 pyelonephritis, 106, 122, 161, 176,228,243 S. aureus, 36, 37, 51, 52, 193,209, 307
poisoned patients, diagnosing. 75, 93 pyloric stenosis, 222, 235 S. pneumococcus, 196, 213
poison ivy, 34, 49 pyogenic liver abscesses, 223, 237 S. pneumoniae, 159, 173
polycythemia, 197,214-215 pyrantel pamoat,e for pinworm, 25 s. pyogenes. 37, 52
polyethylene glycol, for lithium toxicity, 16 salicylate toxicity, 33, 48
polymicrobial infection, 37, 52 QRS complexes, 195, 212 Salmonella, 9, 17,23-24, 37, 52,285,300
pontine hemorrhage, 169, 184 QRS prolongation, 1, 15, 40, 56, 96, 112, 164, Salter-Harris classification, 282, 296
popliteal artery aneurysms, 71, 87 179 III fracture, 232, 232, 248
pork tapeworm, 289, 305 QTc shortening, 159, 174 sarcoidosis, 196, 214
positive end-expiratory pressure (PEEP), 62, QT interval prolongation, 97, 113 scabies, 166, 182
77 QT prolongation, 190, 205 scaphoid, 164, 180
posterior communicating artery (PCOM) quinsy, 56 scapular fractures, 197,215, 216
aneurysm, 293,310 scarlet fever, 14,29
posterior elbow dislocation, 73, 73, 90 R. rickettsii, 12, 27, 72, 88 Schatzki's ring, 8, 22
"posterior fat pad", 14,29 rabies, 41, 43, 57,60 schistocytes, 261,261,279
posterior hip dislocations, 199,199, 217,293, radial head subluxation, 71, 87 schizophrenia, 2, 15, 71, 87
309-310 radiation enteritis, acute, 34, 49 scleroderma, 227,242
posterior parapharyngeal space infections, radiation syndrome, acute, 289, 305 scombroid fish poisoning, 283, 298
252,266 radiographic materials (RCMs), 131, 147 scorpion envenomation, 67, 83
postintubation hypotension in asthmatics, radiographic studies during pregnancy, 195, segment elevation myocardial infarction
32,46 213 (STEMI),l5
postoperative ileus, 291, 308 radioisotopes, 289, 305 Seidel test, 168, 184
postpartum hemorrhage, 224, 238 Ranson's aiteria, 250--251,264 scizures
postvoid residual volume, 9, 22-23 Raynaud's phenomenon, 166, 182, 227,242 alcohol abuse and, 224, 237
potassium chloride, for lithium toxicity with reciprocal ST depressions, 72, 88 in anticholinergic crisis, 18
severe hypokalemia, 16 rectal prolapse in children, 97, 113 average duration of, 225, 239
preeclampsia, 32-33,47, 131-132, 147-148, reentrant dysrhythmias, 40, 57 bupropion overdose and, 12, 27
161, 176, 190, 205,256,272 reflex sympathetic dystrophy, 7, 20--21 diarrhea and, associated with, 17
preexcitation pattern, 220, 233 refractory asthma, 22 eclampsia and, 162, 177
pregoaocy, expected findings during. 73, 90 Reiter's syndrome, 10, 25, 72, 88 febrile, 253, 267
preload, 13, 28 renal carcinoma, 196,213,215--216 head trauma and, 167, 183
Index 319

in human immunodeficiency virus classification of, 282-283, 297 "teardrop" sign, 63, 63, 78
(HIV)/AIDS patients, 13, 28 spinal stenosis, 11-12, 27 teeth
hyponatremia in well newborn and, 107, spinous injuries, 21 avulsed, 66, 82--1!3
124 spirochete, 5, 18-19 fractures, 164, 180
ischemic stroke and, 249, 263 spleen. blunt trauma in children and, 6, 19 numbering of, 253, 268
psychogenic vs. true seizure disorder, 98, splitting the fourth digit, 10-11, 25 temporal arteritis, 63, 78, 127, 141
113-114 spontaneous bacterial peritonitis (SBP), 292, temporomandibular joint (TMJ)
Shigella and, 24 293, 309,310 dWdocations,62,77
shoulder dislocations and, 31, 45 spontaneous miscarriage, first trimester, 129, syndrome, 220, 233
subarachnoid hemorrhage (SAH) and, 144 testicular torsion, 135,151-152,193,196,
152, 152, 180 spontaneous perforation, 136, 153 210,214
in sympathomimetic crisis, 18 spontaneous subconjunctival hemorrhage, testicular tumor, 283, 283--284, 298
in term neonate, 8, 21 101, 101, 118 tetanus, 39, 55
selective serotonin reuptake inhibitor (SSRI), spousal abuse, 11, 25--26 tetralogyofFallot, 101,101,118,137, 154
11, 26,137, 154-155, 160,175 sprains, 195, 213 thalassemia, 34, 49-50
for anxiety disorders, 163, 179 squamous cell carcinoma (SCC), 8, 21 third trimester bleeding, 32-33, 47
overdose of, 193, 209 stage IV hemorrhagic shock, 289, 305 thoracic outlet syndrome, 4, 17, 99, 116
separation pain, 260, 277 Stanford class A aortic dissection, 162, 177 thoracic trauma, 21, 105, 122
sepsis, 127, 141 staphylococcal scalded skin syndrome nonintentional blunt, 4, 17
definition of, 207 (SSSS), 25, 65, 65, 81 penetrating, 255, 271
septic arthritis, 36, 43, 51, 59, 193, 209,230, staphylococcal TSS, 65, 81 thoracotomy, emergency, 159, 159, 173, 197,
246, 291, 308 Staphylococcus, 106, 123 215
septic bursitis, 16 Staphylococcus aureus, 16 thorn apple, 135, 135, 152
septic shock, 138, 156, 191-192, 207,281, status epilepti.cus, 8--9, 22 thrombocytopenia, 29
295--296 stenting. in angi.oplasty for ischemic heart thromboembolic disease, 290, 306
seronegative spondyloarthropathies, 88 disease, 17 thrombotic thrombocytopenic purpura
serotonin5-HT1B/ID receptor agonists, 14, sterile bursitis, 16 (TI'P), 37,52-53, 102, 119, 123
29-30 sternal fractures, 198,216 thyroid storm, 64--65, 80--81, 134, 150--151
serotonin syndrome, 72, 89, 160, 175 sternoclavicular dislocations, 5, 18, 42, 42, tibial fractures
sexually transmitted disease, 128, 143 58-59 plafond 98, 114
Sgarbossa criteria, 193,209 "stone heart", 20 plateau, 31,45
shaken baby syndrome, 129, 144 streptococcal pharyngitis, 17, 31, 45, 169, shaft, 259, 276
Shigella., 9, 17,23--24,37, 52, 193,209 185, 288, 304 stress, 32, 46
Shigella dysenteriae, 9, 23-24 Streptococcus pneumoniae, 37, 52 tick-borne illness, 230, 246
ShigeUa sonnei, 9, 23--24 stress fractures, tibia, 32, 46 tick parn!ysis, 104, 121
Shigellosis, 9, 23--24 ST segment elevation myocardial infarction tinea capitis, 33, 47--48
shingles, 44, 44, 60, 101, 101-102, 118 (STEMI), 69, 69, 85 tinea versicolor, 134, 150
shock in pregnant patients, signs and subarachnoid hemorrhage (SAH), 13, 28, 42, Tinel's sign, 10-11, 25
symptoms of, 98, 114 58,129-130,130,145,152,152,180 tissue plasminogen activator ( tPA)
shoulder dislocations, 31, 45 average case-fatality rate of, 228, 242 ~sttation,6-7,20,36,51
acromioclavicular (AC) separation and, subconjunctival hemorrhage, spontaneous, tongue swelling. 284, 284-285, 300
188, 203--204 101, 101, 118 tonsilliu enhu-gement, 31,45
anterior, 188, 188, 203--204 subdural hematomas, 13, 28, 255, 270 tonsils, 39, 5>-56
anterior, recurrent dislocation risks and, submersion injuries, 5, 17 topical ophthalmic anitibiotics, 228, 243
134,151 subtherapeutic drug levels, 8--9, 22 torsade de pointes, 40, 57
anterior, reduction of, 131, 146 sudden infant death syndrome (SIDS), 105, total anomalous pulmonary venous return,
anterior glenohumeral, 72, 88 121 137,154
clavicular fracture and, 190,205 suicide, completed total hyphema, 37, 37-38, 53
clavicular fractures and, 188, 203-204 common mode of, 13, 28 toxic epidermal neaolysis (TEN), 25, 282,
shoulderdystocia,128,143-144,165,181 family history as risk factor for, 71, 87 282,284,296--297,299
sickle cell anemia, 100, 117 statistical chances of, 11, 26 toxic shock syndrome (TSS), 65,81
sickle cell disease, 131, 147,224,238 superior mesenteric artery (SMA), 31, 45 toxin odor, 34, 49, 170, 186-187
sick sinus syndrome, 67, 84 superior vena cava (SVC) syndrome, 97, 112 Toxoplasma gondii, 290, 306--307
sigmoid volvulus, 133, 149, 258,275 swan neck deformity, 250, 250, 264 toxoplasmosis, 13,28
silver nitrate sticks, 98, 115 Sydenham's chorea, 29 transient ischemic attack (TIA), 37, 53, 102,
sinus bradycardia, 189, 204 sympathomimetic crises vs. anticholinergic 119, 136, 154
sinus imaging, 281,295 crises, 5, 18 Transient Ischemic Attack Working Group,
sinusitis, acute, 39, 54 symptomatic ch.olelithiasis, 100, 117 53
sinus node dysfunction, 67, 84 ')'llOOpe, 9, 22 transient synovitis, 285, 30 1
sinus tachycardia, 137, 154, 197,216 syndrome of inappropriate secretion of transplant recipients, solid-organ, 253,
sitz bath, 250, 264 antidiuretic hormone (SIADH), 39, 267-268
sixth <lisease, 128-129, 144 55 transposition of the great vessels, 137, 154
slipped capital remota! epiphysis (SCFE), syphilio, 66, 81 trauma patients
291,307 systemic inflammatory response syndrome elderly, 256, 272
slit lamp examination, 41, 41, 57 (SIRS), 255, 271 initial fluid administration to, 136, 153,
small bowclobrnuction (SBO), 69, 69, 85 systemic lupus erythematosus (SLB), 7, 20, 169,185
Smith's fracture, 62, 77-78 138-139,156,163,179 primary survey of, 291, 308
smoke inhalation, 258,274 systemic sclerosis, 227,242 traumatic aortic injury (TAl), 223, 236, 256,
sodium channel blockers, 15 273, 290, 306
sodium levels, 158, 172 T. solium, 289, 305 traumatic arrest, 159, 173
somatization disorder, 227, 242 tachydysrhythmias, in W enckebach traumatic hemothorax, 255, 270
"spilled teacup" sign, 257,257, 273 phenomenon, 17 traveler's diarrhea, 99, 115, 193, 209
spinal cord compression, 104, 120-121,229, tamponade, 9, 22 trench mouth, 71, 87
244 tarantula bites, 13, 29 Trichomonas vaginitis, 36, 52
spinal cord injuries, 282, 297 tardive dyskinesia, 227-228, 242 Trichophyt<m t<Jnsllrom, 33, 47-48
320 Index

tricuspid atresia, 137, 154 urinalysis, for evaluation of all urinary vision, painful loss of, 64, 79
tricyclic antidepressants, 287,303 system conditions, 224, 238 visual acuity, decrease in, 33, 47
poisoning, 96,112,164, 179 urinary obstruction, 282, 296 vital sign abnormalities in neonates, 128,
trigemfualneuralgia, 107, 125,200,218,229, urinary retention, 16, 282,296 143
244 urinary tract infections (UTis), 99,116, vitamin~. 160, 175
trimethoprim-sulfamethoxazole, for 126-127,128,141,142,228,243,259, vitreous hemorrhage, 43, 59, 71, 86--87
bacterial cystitis, 164, 180 276 volar displacement 257,273
triptaru, 14, 29-30 complicated, 25 Volkmann's ischemic contracture, 73, 90
troponin I, 291,308 pinworms and, 25 volume overload, 168, 184
true seizure disorder, 98, 113--114 WBC casts and, 161, 176 vomitting blood, alcohol abuse and, 43, 60
truncal, morbilliform rash, 42, 42, 59 urine containing crystals, 100, 117-118 von Willebrand factor, 13, 29
truncus arteriosus, 137, 154 uterine atony, 224, 238 vulvovaginitis, pinworms and, 25
tubal scarring, 18 uterine contractions, maternal trauma and,
tuberculosis (TB), 139, 156--157, 195,212 102, 119 "walking'' pneumonia, 61, 76
tuberculous myocarditis, 15-16 uterine prolapse, 39-40, 40. 56 "warm ischemia," 163,179
tube thoracostomy, 105, 122,224-225, 239 uterine rupture, maternal, 105--106, 122 water intoxication, 107, 124
tuba-ovarian abscess (TOA), 1-2, 15, 18 WBC count, 36, 52, 94, 109
tularemia, 199,216--217 V. parahaemolyticus, 307 Weber's glands, 56
tumor lysis syndrome, 10, 24 vaginal foreign body, 255, 271 Weii-Felixtest,63, 78
tumon vaginal lesions, 65-66, 66, 81-82 Wellen's syndrome, 197, 197, 216
"80% rule" and, 225, 239 variant angina, 163,179 Wenckebach phenomenon (Mobitz type I
gliol, 174 variceal hemorrhage, 60 AVblock), 3, 4, 17
ionizing radiation and, 160, 174 Varicella-Zoster virus (VZV), 44, 60, 134, Wernicke-Korsako:lf syndrome, 63, 79
meningiomas, 174 151 Wernicke's encephalopathy, 63, 78-79
tympanic membrane (TM) perforation, 257, varicoceles, 250, 250, 264 Westermark's sign, 227, 241-242
257,274 vascular claudication, 27 West Nile virus, 36, 51
vascular injury, "hard" signs of, 96, 112 wheezes, 168, 183
ulcerative colitis, 131, 147, 292,309 vasculitic syndromes, 223, 236 whole bowel irrigation, for lithium toxicity,
ulcerative vulvar, 65-66, 66, 81--82 vasogenic edema, 136, 154 16
ulnar collateral ligament (UCL), 34, 49 venous thromboembolism, 160, 174 whooping cough, 135, 152
ulnar nerve injuries, 73, 90 ventilator-associated lung injury (VALI), wide complex tachycardias, 164, 179
ultrasonography, 47, 132, 148 192, 207-208 ''wine-and-cheese" reaction, 160, 175
umbilical cord prolapse, 167, 183 ventricular infarction, 7, 21 Wolff-Parlcinson-White {WPW) syndrome,
uncal herniation, 13, 28 v.ntriculartachy<ar<lia (VT), 104,105,121 40,56-57,195,211-212,220,233
unilateral cranial nerve deficits, 252, 267 ventriculoperitoneal shunt (VPS), 252, 265 World Health Organization (WHO), 172
upper airway obstruction, 226, 241 verapamil, for cluster headaches, 220, 233
upper gastromtestinal bleedmg (UG1B), 34, vertigo, 105, 121-122,221,234-235,287, xantho~nria,230,246
49,222,235,260,278 302
upper G1 bleeding, 70, 86 Vibrio, 17 Y. enterocolitica,l59, 173-174
upper urinary tract infection (UTI) vicryl absorbable sutures, 160, 175 Y. pestis, 162, 177
(pyelonephritis), 161, 176,228, 243 violent aime, victims of, 192, 208 Yersinia, 17
urethral catheterization, 259, 276 viral conjunctivitis, uncomplicated, 199-200,
urethral mjury, 97, 113, 200,201, 218 217 zone II neck. injuries, 101,118
urethritis. 136, 153, 161, 166, 176, 182 viral streptucoccal pharyngitis, 31, 45 zygomycosis, 9, 23
n ~aVY

Ffgunt 1-1.

Ffgunt 1-5.
Ffgure 3-1.

Ffgutw 3-11. Figure from Tasman W. Jaeger EA. eels. The WiUs Eye
Hospital atiDs of dinicol ophthalmology, 2nd ed. Philadelphia: Lip-
pincott WiWams & Wil.lrins; 2001, wilf1 permission.
Ffgure 4·6. Figure reprinted with pennission
from Weber J. Heolth assessment in nursing,
2nd ed. Lippincott WiUiams & Wilkins: 2002.

Ffgurw 4-10.
Ffgure 6-9.

Figure 7-8.
Ftgure 7-t. Figure reprinted with permission
fTom PiUiteni A. Mamnal and child health
nursing. Lippincott Williams &Wilkins; 2006.

Ffpre 8-3.
Ffgunt 8-5. Figure reprinted with pennission from Fleisher GR. Atlos of
pediatric emergency medicine. Lippincott WiUiams &Wilkins: 2003.

Ffgure 9-4.

Ffgunt 10-11. Figure reprinted with pennission from Ba~ankin B. S~man's


illustrated dictionaty of demrattJ!ogy eponyms. Lippincott WiUiams & Willcins;
2004.

Das könnte Ihnen auch gefallen